{"key": 0, "questions": "Which of the following is the most appropriate underlying explanation for this patient's presentation?", "options": [{"label": "A", "disease": "Gastroenteritis"}, {"label": "B", "disease": "Immunodeficiency"}, {"label": "C", "disease": "Intentional contamination"}, {"label": "D", "disease": "Meningitis"}, {"label": "E", "disease": "Urinary tract infection"}], "answer_idx": "C", "symptoms": ["A 9-year-old girl presents to the emergency department with a fever and a change in her behavior", "She presented with similar symptoms 6 weeks ago and was treated for an Escherchia coli infection", "She also was treated for a urinary tract infection 10 weeks ago", "Her mother says that last night her daughter felt ill, and her condition has been worsening", "Her daughter experienced a severe headache and had a stiff neck", "This morning she was minimally responsive, vomited several times, and produced a small amount of dark cloudy urine", "The patient was born at 39 weeks and met all her developmental milestones", "She is currently up to date on her vaccinations and did not have infections during early childhood", "Her parents are divorced and her father has noted she does not seem to get sick when he takes care of her", "Her temperature is 99.5°F (37.5°C), blood pressure is 60/35 mmHg, pulse is 190/min, respirations are 33/min, and oxygen saturation is 98% on room air", "The patient is started on intravenous fluids, vasopressors, and broad-spectrum antibiotics"], "s1": [0, 1, 2, 3, 4, 5, 9, 10], "s2": [6, 7, 8]} {"key": 1, "questions": "Which of the following is the most likely diagnosis?", "options": [{"label": "A", "disease": "Gout"}, {"label": "B", "disease": "Lyme arthritis"}, {"label": "C", "disease": "Pseudogout"}, {"label": "D", "disease": "Rheumatoid arthritis"}, {"label": "E", "disease": "Septic arthritis"}], "answer_idx": "A", "symptoms": ["A 55-year-old man presents to the emergency department with right knee pain", "He woke last night with sudden-onset, sharp, 10/10 non-radiating pain in his right knee", "He denies recent trauma and he has no known medical problems", "He denies tobacco use and will often drink 6-8 beers or spirits on weekend nights", "His temperature is 98.6°F (37.0°C), pulse is 90/min, blood pressure is 140/90 mmHg, respirations are 16/min, and oxygen saturation is 97% on room air", "Physical exam reveals an erythematous, edematous right knee", "Passive extension of the knee is limited by pain", "Synovial fluid aspiration is performed and sent for analysis, which reveals 20,000 leukocytes/mm^3, and no organisms visualized", "Microscopic examination of synovial fluid is shown in Figure A"], "s1": [0, 2, 3, 4], "s2": [1, 5, 6, 7, 8]} {"key": 2, "questions": "Which of the following is the most likely etiology of this patient’s symptoms?", "options": [{"label": "A", "disease": "Alpha cell tumor"}, {"label": "B", "disease": "Beta cell destruction"}, {"label": "C", "disease": "Beta cell tumor"}, {"label": "D", "disease": "Insulin overdose"}, {"label": "E", "disease": "Sulfonylurea overdose"}], "answer_idx": "C", "symptoms": ["A 44-year-old nurse presents to the emergency department with confusion", "This has happened several times this past month with increasing frequency", "Each time her symptoms improved with eating", "She is otherwise healthy", "She lives with her mother who has diabetes whose medications include metformin, insulin, lisinopril, amlodipine, and glyburide", "Her temperature is 98.0°F (36.7°C), blood pressure is 132/81 mmHg, pulse is 85/min, respirations are 16/min, and oxygen saturation is 98% on room air", "Physical exam reveals a confused woman", "She is moving all her extremities but follows commands poorly", "Laboratory studies are ordered as seen below", "Hemoglobin: 14 g/dL", "Hematocrit: 41%", "Leukocyte count: 7,500/mm^3 with normal differential", "Platelet count: 199,000/mm^3", "Serum:", "Na+: 140 mEq/L", "Cl-: 102 mEq/L", "K+: 4.0 mEq/L", "HCO3-: 23 mEq/L", "BUN: 30 mg/dL", "Glucose: 29 mg/dL", "Creatinine: 1.4 mg/dL", "Ca2+: 10.2 mg/dL", "Sulfonylurea level: undetectable", "C-peptide level: 55 ng/mL (normal < 5 ng/mL)"], "s1": [0, 1, 2, 3, 4, 5, 6, 7, 8], "s2": [9, 10, 11, 12, 13, 14, 15, 16, 17, 18, 19, 20, 21, 22, 23]} {"key": 3, "questions": "Which of the following is the most likely diagnosis based on this patient’s ECG?", "options": [{"label": "A", "disease": "Acute myocardial infarction"}, {"label": "B", "disease": "Hypokalemia"}, {"label": "C", "disease": "Intermittent torsades des pointes"}, {"label": "D", "disease": "Previous myocardial ischemia"}, {"label": "E", "disease": "Pulmonary embolism"}], "answer_idx": "D", "symptoms": ["A 67-year-old man presents to the emergency department after fainting", "He was outside tending to his lawn for several hours in the heat, when he stood up suddenly from pulling weeds and fainted", "He denies any preceding symptoms and returned to baseline within 1 minute", "The patient is not aware of any medical problems and just started seeing a primary care physician last week", "He recently used a friend's prescription for ondansetron for nausea", "His temperature is 99.3°F (37.4°C), blood pressure is 142/88 mmHg, pulse is 107/min, respirations are 14/min, and oxygen saturation is 99% on room air", "Physical exam reveals intact cranial nerves, normal strength and sensation, and a stable gait", "His abdomen is soft and nontender", "An ECG is performed as seen in Figure A"], "s1": [0, 1, 2, 5], "s2": [3, 4, 6, 7, 8]} {"key": 4, "questions": "Which of the following is most strongly associated with this patient's condition?", "options": [{"label": "A", "disease": "Cardiac rhabdomyoma"}, {"label": "B", "disease": "Glaucoma"}, {"label": "C", "disease": "Optic glioma"}, {"label": "D", "disease": "Polyostotic fibrous dysplasia"}, {"label": "E", "disease": "Renal cell carcinoma"}], "answer_idx": "A", "symptoms": ["A 1-year-old girl is brought to a neurologist due to increasing seizure frequency over the past 2 months", "She recently underwent a neurology evaluation which revealed hypsarrhythmia on electroencephalography (EEG) with a mix of slow waves, multifocal spikes, and asynchrony", "Her parents have noticed the patient occasionally stiffens and spreads her arms at home", "She was born at 38-weeks gestational age without complications", "She has no other medical problems", "Her medications consist of lamotrigine and valproic acid", "Her temperature is 98.3°F (36.8°C), blood pressure is 90/75 mmHg, pulse is 94/min, and respirations are 22/min", "Physical exam reveals innumerable hypopigmented macules on the skin and an irregularly shaped, thickened, and elevated plaque on the lower back"], "s1": [0, 1, 2, 5], "s2": [3, 4, 6, 7]} {"key": 5, "questions": "Which of the following is the most likely diagnosis?", "options": [{"label": "A", "disease": "Addison disease"}, {"label": "B", "disease": "Cerebral salt wasting"}, {"label": "C", "disease": "Diuretic overuse"}, {"label": "D", "disease": "Primary polydipsia"}, {"label": "E", "disease": "Syndrome of inappropriate anti-diuretic hormone"}], "answer_idx": "B", "symptoms": ["A 55-year-old woman is brought to the emergency department by her husband with a 1 hour history of an unremitting headache", "The headache started suddenly while she was eating dinner and she says it feels like the “worst headache of my life", "” An emergent CT scan of the head without contrast confirms the diagnosis, and a CT angiogram identifies the source of bleeding", "The patient undergoes surgical management of her condition", "On hospital day 3, she is found to be disoriented to person, place, and time", "She also develops nausea and vomiting", "Her medical problems consist of heart failure for which she takes furosemide, spironolactone, and metoprolol, which were continued at admission", "Her temperature is 99.6°F (37.6°C), blood pressure is 100/60 mmHg, pulse is 112/min, and respirations are 16/min", "Physical examination shows poor skin turgor", "Capillary refill time is 4 seconds", "Serum laboratory results are shown below: Na+: 120 mEq/L Cl-: 92 mEq/L K+: 3.9 mEq/L HCO3-: 26 mEq/L BUN: 32 mg/dL Creatinine: 1.0 mg/dL Serum osmolality is 265 mEq/L and urine osmolality is 340 mEq/L", "Urine sodium is 44 mEq/L"], "s1": [0, 1, 2, 3, 4], "s2": [5, 6, 7, 8, 9, 10, 11]} {"key": 6, "questions": "Which of the following is the most likely underlying pathophysiology?", "options": [{"label": "A", "disease": "Anti-platelet antibodies"}, {"label": "B", "disease": "Factor 8 deficiency"}, {"label": "C", "disease": "Factor 9 deficiency"}, {"label": "D", "disease": "GP1b deficiency"}, {"label": "E", "disease": "Von Willebrand factor deficiency"}], "answer_idx": "B", "symptoms": ["A 6-year-old boy presents to the emergency department after falling from his scooter", "He has dull, aching pain along his left side where he hit the ground", "He fell sideways off the scooter as he rounded a curve in the road", "He has never had any serious injuries but that he always seems to bruise easily, especially after he started playing soccer this fall", "His parents deny that he has an abnormal number of nosebleeds or bleeding from the gums", "They have never seen blood in his stool or urine", "His mother notes that her brother has had similar problems", "His temperature is 98.6°F (37°C), blood pressure is 112/74 mmHg, pulse is 82/min, and respirations are 11/min", "On physical exam, the patient has extensive bruising of the lateral left thigh and tenderness to palpation", "Laboratory tests are performed and reveal the following:", "Hemoglobin: 14 g/dL", "Hematocrit: 41%", "Mean corpuscular volume: 89 µm3", "Reticulocyte count: 0.8%", "Leukocyte count: 4,700/mm3", "Prothrombin time (PT): 13 seconds", "Partial thromboplastin time (PTT): 56 seconds", "Bleeding time (BT): 4 minutes"], "s1": [0, 1, 2, 5, 7, 8], "s2": [3, 4, 6, 9, 10, 11, 12, 13, 14, 15, 16, 17]} {"key": 7, "questions": "Which of the following is the most likely diagnosis?", "options": [{"label": "A", "disease": "Decreased metabolism of bilirubin"}, {"label": "B", "disease": "Increased enterohepatic circulation of bilirubin"}, {"label": "C", "disease": "Increased production of bilirubin"}, {"label": "D", "disease": "Obstruction of the extrahepatic biliary tree"}, {"label": "E", "disease": "Obstruction of the intrahepatic biliary tree"}], "answer_idx": "D", "symptoms": ["A 1-month-old girl presents to her pediatrician with her mother", "The patient was born at 38 weeks gestation via Caesarean section for cervical incompetence", "The patient’s mother has no complaints", "The child had a runny nose and cough for a few days last week", "The patient’s mother endorses decreased appetite during the aforementioned illness which has now returned to baseline", "The patient’s family history is significant for an older brother with glucose-6-phosphate dehydrogenase (G6PD) deficiency and a maternal uncle with cirrhosis secondary to chronic hepatitis B", "On physical exam, the patient has scleral icterus and dark urine staining her diaper", "Laboratory testing reveals the following: Serum: Na+: 137 mEq/L Cl-: 102 mEq/L K+: 4.2 mEq/L HCO3-: 24 mEq/L Urea nitrogen: 12 mg/dL Glucose: 96 mg/dL Creatinine: 0.36 mg/dL Alkaline phosphatase: 146 U/L Aspartate aminotransferase (AST): 86 U/L Alanine aminotransferase (ALT): 76 U/L Total bilirubin: 4.6 mg/dL Direct bilirubin: 3.8 mg/dL"], "s1": [0, 1, 2, 3, 4, 5], "s2": [6, 7]} {"key": 8, "questions": "Which of the following is the most likely diagnosis?", "options": [{"label": "A", "disease": "Apical lung tumor"}, {"label": "B", "disease": "Brachial plexopathy"}, {"label": "C", "disease": "Cerebral infarction"}, {"label": "D", "disease": "Scalenus anticus syndrome"}, {"label": "E", "disease": "Subclavian steal syndrome"}], "answer_idx": "A", "symptoms": ["A 55-year-old male bodybuilder presents to the emergency department with weakness of his right arm", "The patient states he has experienced these symptoms for a few weeks", "however, today his hand felt so weak he dropped his cup of tea", "The patient has a medical history of diabetes", "He drinks 2-7 alcoholic drinks per day and has smoked 2 packs of cigarettes per day since he was 25. The patient uses anabolic steroids", "He has lost 17 pounds since he last came to the emergency department 1 month ago", "His temperature is 99.5°F (37.5°C), blood pressure is 177/108 mmHg, pulse is 90/min, respirations are 17/min, and oxygen saturation is 98% on room air", "Physical exam reveals decreased sensation in the right arm and 2/5 strength in the right arm and 5/5 strength in in the left arm", "The patient states that he is experiencing a dull aching and burning pain in his right arm during the exam", "Symptoms are not changed with changes in head and neck position"], "s1": [0, 1, 2, 7, 8, 9], "s2": [3, 4, 5, 6]} {"key": 9, "questions": "Which of the following is the most likely diagnosis?", "options": [{"label": "A", "disease": "Apical lung tumor"}, {"label": "B", "disease": "Brachial plexopathy"}, {"label": "C", "disease": "Cerebral infarction"}, {"label": "D", "disease": "Scalenus anticus syndrome"}, {"label": "E", "disease": "Subclavian steal syndrome"}], "answer_idx": "A", "symptoms": ["A 55-year-old male bodybuilder presents to the emergency department with weakness of his right arm", "The patient states he has experienced these symptoms for a few weeks", "however, today his hand felt so weak he dropped his cup of tea", "The patient has a medical history of diabetes", "He drinks 2-7 alcoholic drinks per day and has smoked 2 packs of cigarettes per day since he was 25. The patient uses anabolic steroids", "He has lost 17 pounds since he last came to the emergency department 1 month ago", "His temperature is 99.5°F (37.5°C), blood pressure is 177/108 mmHg, pulse is 90/min, respirations are 17/min, and oxygen saturation is 98% on room air", "Physical exam reveals decreased sensation in the right arm and 2/5 strength in the right arm and 5/5 strength in in the left arm", "The patient states that he is experiencing a dull aching and burning pain in his right arm during the exam", "Symptoms are not changed with changes in head and neck position"], "s1": [0, 7, 8, 9], "s2": [1, 2, 3, 4, 5, 6]} {"key": 10, "questions": "Which of the following is the most likely diagnosis?", "options": [{"label": "A", "disease": "Antithrombin III deficiency"}, {"label": "B", "disease": "Factor V Leiden"}, {"label": "C", "disease": "Liver failure"}, {"label": "D", "disease": "Nephrotic syndrome"}, {"label": "E", "disease": "Prothrombin gene mutation"}], "answer_idx": "D", "symptoms": ["A 57-year-old immigrant from Nigeria presents to the emergency department for sudden, severe pain and swelling in her lower extremity", "She was at a rehabilitation hospital when her symptoms started", "The patient has a medical history of obesity, diabetes, bipolar disorder, and tonic-clonic seizures", "Her current medications include metformin, insulin, lisinopril, and valproic acid", "The patient has IV drug and alcohol use disorder and has presented to the ED many times for intoxication", "Her temperature is 98.6°F (37°C), blood pressure is 115/71 mmHg, pulse is 72/min, and respirations are 12/min", "Physical exam reveals anasarca and asymmetric lower extremity swelling", "Based on the results of a doppler ultrasound of her swollen lower extremity, heparin is started", "The patient is then transferred to the general medicine floor for continued management", "Laboratory studies are shown below", "Serum:", "Na+: 137 mEq/L", "K+: 5.5 mEq/L", "Cl-: 100 mEq/L", "HCO3-: 24 mEq/L", "Urea nitrogen: 22 mg/dL", "Ca2+: 5.7 mg/dL", "Creatinine: 1.7 mg/dL", "Glucose: 70 mg/dL"], "s1": [0, 1, 2, 3, 4, 5, 6, 7, 8], "s2": [9, 10, 11, 12, 13, 14, 15, 16, 17, 18]} {"key": 11, "questions": "Which of the following is the most likely cause of this patient's symptoms?", "options": [{"label": "A", "disease": "Erosion into a gastric vessel"}, {"label": "B", "disease": "Inflammation and infection of colonic outpouching"}, {"label": "C", "disease": "Malignancy"}, {"label": "D", "disease": "Submucosal esophageal tear"}, {"label": "E", "disease": "Vascular malformation"}], "answer_idx": "E", "symptoms": ["A 43-year-old man presents to the emergency department with bright red blood in his stool this morning", "He noticed this yesterday as well, but it was much scanter", "The patient is otherwise healthy", "He was celebrating his birthday last night and admits having \"too much to drink\" and vomited profusely overnight", "He currently feels well and is not nauseous", "He has no other significant medical history and does not take any medications", "His temperature is 99.2°F (37.3°C), blood pressure is 110/75 mmHg, pulse is 82/min, respirations are 16/min, and oxygen saturation is 100% on room air", "Physical exam reveals a nontender abdomen with normal bowel sounds", "Rectal exam reveals grossly bloody stool", "Laboratory studies are ordered as shown below", "Hemoglobin: 12 g/dL", "Hematocrit: 36%", "Leukocyte count: 5,500/mm^3 with normal differential", "Platelet count: 179,000/mm^3"], "s1": [0, 1, 3, 4, 8], "s2": [2, 5, 6, 7, 9, 10, 11, 12, 13]} {"key": 12, "questions": "Which of the following is the most likely cause of this patient's symptoms?", "options": [{"label": "A", "disease": "Absence seizures"}, {"label": "B", "disease": "Alcohol intoxication"}, {"label": "C", "disease": "Cannabinoid use"}, {"label": "D", "disease": "Juvenile myoclonic epilepsy"}, {"label": "E", "disease": "West syndrome"}], "answer_idx": "D", "symptoms": ["A 14-year-old boy presents to his pediatrician with a 2-month history of intermittent arm and leg spasms", "He says that the spasms usually occur after he wakes up as he is getting ready for school, and consist of disorganized jerking motions that stop after a few minutes", "He has also noticed that they occur when he stays up late to cram last minute for exams", "He has not noticed any other symptoms", "He has no medical history and takes no medications", "He recently started experimenting with alcohol and marijuana that one of his friends has been stealing from their parents", "His temperature is 98.6°F (37.0°C), blood pressure is 115/70 mmHg, pulse is 70/min, and respirations are 12/min", "Physical exam reveals a healthy child"], "s1": [0, 1, 2], "s2": [3, 4, 5, 6, 7]} {"key": 13, "questions": "Which of the following is the most likely diagnosis?", "options": [{"label": "A", "disease": "Cardiogenic shock"}, {"label": "B", "disease": "Hemorrhagic shock"}, {"label": "C", "disease": "Neurogenic shock"}, {"label": "D", "disease": "Obstructive shock"}, {"label": "E", "disease": "Septic shock"}], "answer_idx": "D", "symptoms": ["A 37-year-old woman presents to the emergency department in cardiac arrest", "She was found to have altered mental status at home, and during transport to the hospital, she became pulseless", "Cardiopulmonary resuscitation is ongoing and 2 intravenous lines are placed", "The patient is obese, and her only medication is an oral contraceptive pill", "She recently underwent Achilles tendon repair", "She otherwise is known to smoke cigarettes on occasion", "Physical exam is notable for a critically ill patient with chest compressions ongoing", "The monitor shows pulseless electrical activity", "Her right lower extremity is in a post-operative splint", "The patient regains her pulse after several rounds of CPR and epinephrine", "A bedside echocardiograph is notable for a dilated and hypokinetic right ventricle with septal bowing into a hyperkinetic left ventricle"], "s1": [0, 1, 2, 6, 7, 9], "s2": [3, 4, 5, 8, 10]} {"key": 14, "questions": "Which of the following personality disorders is the most likely diagnosis?", "options": [{"label": "A", "disease": "Antisocial"}, {"label": "B", "disease": "Avoidant"}, {"label": "C", "disease": "Borderline"}, {"label": "D", "disease": "Dependent"}, {"label": "E", "disease": "Histrionic"}], "answer_idx": "C", "symptoms": ["A 23-year-old woman presents to the emergency room for a self-inflicted laceration of her distal volar forearm", "The patient states she knew her boyfriend was having sexual thoughts about the woman from the grocery store, prompting her decision to cut her own wrist", "In the emergency department, the bleeding has stopped and the patient is currently medically stable", "When interviewing the patient, she is teary and apologizes for her behavior", "She is grateful to you for her care and regrets her actions", "Of note, the patient has presented to the emergency department before for a similar reason when she was struggling with online dating", "The patient states that she struggles with her romantic relationships though she deeply desires them", "Her temperature is 98.6°F (37°C), blood pressure is 112/71 mmHg, pulse is 73/min, and respirations are 14/min", "On physical exam, you note a frightened young woman who is wearing a revealing dress that prominently displays her breasts", "You tell the patient that she will have to stay in the psychiatric emergency department for the night which makes her furious"], "s1": [0, 2, 7], "s2": [1, 3, 4, 5, 6, 8, 9]} {"key": 15, "questions": "Which of the following describes this patient's most likely diagnosis?", "options": [{"label": "A", "disease": "Benign capillary proliferation"}, {"label": "B", "disease": "Edema of the epidermis"}, {"label": "C", "disease": "Healthy dermatologic development"}, {"label": "D", "disease": "Malignant blood vessel proliferation"}, {"label": "E", "disease": "Viral infection"}], "answer_idx": "A", "symptoms": ["A 39-year-old man presents to his doctor for a wellness checkup", "He is concerned about a rash that does not seem to be improving", "He was recently exposed to his grandfather who has vesicular lesions on his skin and is being treated", "He has a family history of skin cancer, colon cancer, and ovarian cancer", "The patient has a medical history of asthma and seasonal allergies", "His temperature is 98.6°F (37.0°C), blood pressure is 137/98 mmHg, pulse is 90/min, respirations are 14/min, and oxygen saturation is 98% on room air", "Physical exam reveals the finding in Figure A"], "s1": [1, 2, 3, 6], "s2": [0, 4, 5]} {"key": 16, "questions": "Which of the following is the most likely diagnosis?", "options": [{"label": "A", "disease": "Cerebrovascular accident"}, {"label": "B", "disease": "Giant cell arteritis"}, {"label": "C", "disease": "Idiopathic intracranial hypertension"}, {"label": "D", "disease": "Meniere disease"}, {"label": "E", "disease": "Migraine"}], "answer_idx": "C", "symptoms": ["A 19-year-old woman presents to her primary care physician with headaches, blurry vision, and tinnitus over the past month", "She started treatment for a skin condition with a new oral medication 2 months prior", "She has not noticed any changes in her complexion as of yet", "Her temperature is 98.2°F (36.8°C), blood pressure 100/65 mmHg, pulse 87/min, and respiratory rate 14/min", "She is oriented to person, place, and time", "Neurological exam reveals papilledema upon examination of both eyes", "Skin examination is notable for the findings in Figure A"], "s1": [0, 5], "s2": [1, 2, 3, 4, 6]} {"key": 17, "questions": "This child is most likely to develop which of the following complications?", "options": [{"label": "A", "disease": "Aortic dissection"}, {"label": "B", "disease": "Medullary thyroid cancer"}, {"label": "C", "disease": "Osteoarthritis"}, {"label": "D", "disease": "Thromboembolic stroke"}, {"label": "E", "disease": "Wilms tumor"}], "answer_idx": "D", "symptoms": ["A 3-year-old boy presents with his parents to a pediatrician for a new patient visit", "The child was recently adopted and little is known about his medical or family history", "The child seems to be doing well, but the patient is much larger than any of the other 3-year-olds in his preschool class", "He eats a varied diet at home and with limited juice and snack foods", "His temperature is 98.6°F (37°C), blood pressure is 101/67 mmHg, pulse is 110/min, and respirations are 24/min", "On physical exam, the patient is in the 73rd percentile for weight, 99th percentile for height, and 86th percentile for head circumference", "He appears mildly developmentally delayed", "He has a fair complexion and tall stature with a prominent sternum", "The patient also has joint hypermobility and hyperelastic skin", "He appears to have poor visual acuity and is referred to an ophthalmologist, who finds downward lens subluxation of the right eye"], "s1": [0, 1, 3, 4, 5], "s2": [2, 6, 7, 8, 9]} {"key": 18, "questions": "Which of the following is the most likely diagnosis in this patient?", "options": [{"label": "A", "disease": "Amyotrophic lateral sclerosis"}, {"label": "B", "disease": "Carpal tunnel syndrome"}, {"label": "C", "disease": "Cervical spondylosis"}, {"label": "D", "disease": "Syringomyelia"}, {"label": "E", "disease": "Transverse myelitis"}], "answer_idx": "C", "symptoms": ["An 84-year-old man presents to the physician with numbness and tingling in the right hand for the last 6 months", "He reports these symptoms started intermittently in the middle finger after returning from a golfing trip but has progressed to being nearly constant", "The thenar eminence, thumb, and other digits are spared", "He reports no other symptoms", "His medical problems include hypertension, hyperlipidemia, hypothyroidism, and chronic neck pain for which he takes amlodipine, hydrochlorothiazide, atorvastatin, levothyroxine, and duloxetine", "Surgical history consists of posterior lumbar fusion and elective left total hip arthroplasty", "He has a 90-pack-year smoking history and quit 10 years ago", "The patient’s temperature is 99.6°F (37.6°C), blood pressure is 134/80 mmHg, pulse is 90/min, and respirations are 16/min", "Physical exam reveals 4/5 strength in elbow extension and wrist flexion on the right", "He has 5/5 strength in the remainder of the motor exam on the right upper extremity and on the left upper extremity", "The triceps reflex is 1+ on the right compared to 2+ on the left", "There is no spasticity", "His neck pain, which radiates down his right arm, is reproduced with axial loading during neck extension with simultaneous rightward rotation and lateral bending"], "s1": [0, 1, 2, 8, 9, 10, 12], "s2": [3, 4, 5, 6, 7, 11]} {"key": 19, "questions": "Which of the following is the most likely diagnosis?", "options": [{"label": "A", "disease": "Cerebral contusion"}, {"label": "B", "disease": "Concussion"}, {"label": "C", "disease": "Diffuse axonal injury"}, {"label": "D", "disease": "Epidural hematoma"}, {"label": "E", "disease": "Subarachnoid hemorrhage"}], "answer_idx": "B", "symptoms": ["A 17-year-old boy presents to the emergency department after a car accident", "He drove his car into a pole while speeding at night", "He has no past medical history and takes no medications", "The patient is unable to offer a history and repetitively asks the nurses what happened", "He knows his name and address and is able to remember other information about himself", "He is complaining of a headache", "His temperature is 97.6°F (36.4°C), blood pressure is 122/74 mmHg, pulse is 90/min, respirations are 12/min, and oxygen saturation is 98% of room air", "Physical exam reveals a confused young man", "Cranial nerves 2-12 are intact and the patient has normal strength and sensation", "He demonstrates a stable gait", "Throughout the exam, he continuously asks what happened to him", "A head CT scan is ordered"], "s1": [0, 1, 2, 6, 8, 9], "s2": [3, 4, 5, 7, 10, 11]} {"key": 20, "questions": "Which of the following is the most likely diagnosis?", "options": [{"label": "A", "disease": "Ductal carcinoma in situ"}, {"label": "B", "disease": "Intraductal papilloma"}, {"label": "C", "disease": "Invasive ductal carcinoma"}, {"label": "D", "disease": "Mammary duct ectasia"}, {"label": "E", "disease": "Paget disease of the breast"}], "answer_idx": "B", "symptoms": ["A 34-year-old nulliparous woman presents to the clinic with 2 weeks of copper-colored discharge from her right nipple", "Her breast has not been tender, and she has not noticed any changes to the skin", "She is otherwise healthy and does not take any medications", "Her temperature is 36.5°C (97.7°F), blood pressure is 110/82 mmHg, pulse is 68/min, respirations are 12/min, and oxygen saturation is 99% on room air", "Exam of the right breast reveals no mass on palpation", "Bloody discharge is expressed from the right nipple", "There is no axillary lymphadenopathy"], "s1": [0, 5], "s2": [1, 2, 3, 4, 6]} {"key": 21, "questions": "Which of the following is the most likely diagnosis?", "options": [{"label": "A", "disease": "Addison disease"}, {"label": "B", "disease": "Cerebral salt wasting"}, {"label": "C", "disease": "Diuretic overuse"}, {"label": "D", "disease": "Primary polydipsia"}, {"label": "E", "disease": "Syndrome of inappropriate anti-diuretic hormone"}], "answer_idx": "B", "symptoms": ["A 55-year-old woman is brought to the emergency department by her husband with a 1 hour history of an unremitting headache", "The headache started suddenly while she was eating dinner and she says it feels like the “worst headache of my life", "” An emergent CT scan of the head without contrast confirms the diagnosis, and a CT angiogram identifies the source of bleeding", "The patient undergoes surgical management of her condition", "On hospital day 3, she is found to be disoriented to person, place, and time", "She also develops nausea and vomiting", "Her medical problems consist of heart failure for which she takes furosemide, spironolactone, and metoprolol, which were continued at admission", "Her temperature is 99.6°F (37.6°C), blood pressure is 100/60 mmHg, pulse is 112/min, and respirations are 16/min", "Physical examination shows poor skin turgor", "Capillary refill time is 4 seconds", "Serum laboratory results are shown below: Na+: 120 mEq/L Cl-: 92 mEq/L K+: 3.9 mEq/L HCO3-: 26 mEq/L BUN: 32 mg/dL Creatinine: 1.0 mg/dL Serum osmolality is 265 mEq/L and urine osmolality is 340 mEq/L", "Urine sodium is 44 mEq/L"], "s1": [0, 1, 2, 3, 4, 5], "s2": [6, 7, 8, 9, 10, 11]} {"key": 22, "questions": "Which of the following is the most likely diagnosis?", "options": [{"label": "A", "disease": "Arrhythmogenic right ventricular dysplasia"}, {"label": "B", "disease": "Brugada syndrome"}, {"label": "C", "disease": "Hypertrophic obstructive cardiomyopathy"}, {"label": "D", "disease": "QT prolongation"}, {"label": "E", "disease": "Wolff Parkinson White syndrome"}], "answer_idx": "A", "symptoms": ["A 7-year-old adopted boy presents to the emergency department after fainting", "He just switched from being home-schooled to public school and joined the basketball team", "He has fainted several times during basketball practices", "He did not sustain any injuries or have any prodromes prior to these episodes", "He is usually unconscious for less than a minute", "He is otherwise healthy and has met all developmental milestones", "He does not currently take any medications", "His temperature is 97.6°F (36.4°C), blood pressure is 104/54 mmHg, pulse is 100/min, respirations are 19/min, and oxygen saturation is 98% on room air", "Physical exam reveals a well-appearing and playful boy", "An ECG is performed as seen in Figure A", "An echocardiogram is ordered and pending"], "s1": [0, 1, 2, 3, 4, 5, 6, 8], "s2": [7, 9, 10]} {"key": 23, "questions": "Which of the following is the most likely diagnosis?", "options": [{"label": "A", "disease": "Genitourinary syndrome of menopause"}, {"label": "B", "disease": "Mixed urinary incontinence"}, {"label": "C", "disease": "Overflow incontinence"}, {"label": "D", "disease": "Stress incontinence"}, {"label": "E", "disease": "Urge incontinence"}], "answer_idx": "E", "symptoms": ["A 48-year-old woman, gravida 3 para 3, presents to the clinic with a 5-month history of intermittent loss of urine", "Her urinary leaking occurs throughout the day and night, and she often has to get up to use the bathroom while asleep", "Exercising does not affect the loss of urine", "She states that she often cannot make it to the bathroom in time once she senses a need to void", "Her last menstrual period was 2 weeks ago", "Her urinalysis is unremarkable, and her postvoid residual volume is 40 mL", "On pelvic examination, there is no dribbling of urine when the patient coughs", "Physical exam is otherwise unremarkable"], "s1": [0, 1, 2, 3], "s2": [4, 5, 6, 7]} {"key": 24, "questions": "Which of the following is the most likely diagnosis?", "options": [{"label": "A", "disease": "Celiac disease"}, {"label": "B", "disease": "Crohn disease"}, {"label": "C", "disease": "Giardia"}, {"label": "D", "disease": "Tropical sprue"}, {"label": "E", "disease": "Whipple disease"}], "answer_idx": "D", "symptoms": ["A 26-year-old woman presents to the clinic with diarrhea, bloating, flatulence, and abdominal cramps", "These symptoms have been going on for 2 months", "She has lost 6 pounds (2.7 kg) over these 2 months", "Her medical history is significant for mild intermittent asthma for which she uses an albuterol inhaler as needed", "She is sexually active with 1 male partner", "She recently returned from South India following a mission trip for 6 weeks", "Her temperature is 98.6°F (37.0°C), blood pressure is 105/70 mmHg, pulse is 95/min, and respirations are 14/min", "On examination, the patient’s skin is pale", "Labs are obtained and show the following results:", "Leukocyte count: 4,500/mm^3", "Hemoglobin: 10.5 g/dL", "Platelets: 110,000/mm^3", "Mean corpuscular volume (MCV): 116 µm^3", "Reticulocyte count: 0.5%", "A biopsy of the small bowel reveals blunting of villi and a mixed infiltration of lymphocytes, plasma cells, and eosinophils"], "s1": [0, 1, 2, 3, 4, 5, 6, 7], "s2": [8, 9, 10, 11, 12, 13, 14]} {"key": 25, "questions": "Which of the following is the most likely underlying etiology for this patient’s chief concern?", "options": [{"label": "A", "disease": "Hypocalcemia"}, {"label": "B", "disease": "Hypomagnesemia"}, {"label": "C", "disease": "Hyponatremia"}, {"label": "D", "disease": "Hypothyroidism"}, {"label": "E", "disease": "Hypovolemia"}], "answer_idx": "B", "symptoms": ["A 33-year-old woman presents to the emergency department with muscle spasms", "She has noticed \"twitching\" in both her upper and lower extremities over the past 3 days that have interfered with her training for a marathon", "She has felt more fatigued over the past month", "She has no significant medical history", "She takes no medication other than whey protein supplements, a multivitamin, and fish oil", "Her temperature is 98.5°F (36.9°C), blood pressure is 103/71 mmHg, pulse is 105/min, respirations are 11/min, and oxygen saturation is 100% on room air", "Physical exam reveals a thin woman with thinning hair", "She appears pale and her mucous membranes are dry", "Her neurological and cardiac exams are unremarkable", "Laboratory studies are ordered as seen below", "Hemoglobin: 10 g/dL Hematocrit: 30% Leukocyte count: 6,100/mm^3 with normal differential Platelet count: 187,500/mm^3 Serum: Na+: 130 mEq/L Cl-: 100 mEq/L K+: 2.3 mEq/L HCO3-: 28 mEq/L BUN: 40 mg/dL Glucose: 79 mg/dL Creatinine: 0.9 mg/dL Ca2+: 8.2 mg/dL Mg2+: 1.8 mg/dL Thyroid stimulating hormone (TSH): 4.0 mIU/L Free T4: 0.4 ng/dL (normal: 0.7-1.53 ng/dL) The patient is given 4 liters of lactated ringer solution and 40 mEq of potassium", "Her repeat whole blood potassium 3 hours later is 2.4 mEq/L, and her repeat BUN is 20 mEq/L", "Another 40 mEq of potassium is administered", "Another repeat whole blood potassium is 2.5 mEq/L", "The patient’s muscle spasms persist"], "s1": [0, 1, 6, 7, 8, 14], "s2": [2, 3, 4, 5, 9, 10, 11, 12, 13]} {"key": 26, "questions": "Which of the following is most likely responsible for the patient’s symptoms?", "options": [{"label": "A", "disease": "Lung adenocarcinoma"}, {"label": "B", "disease": "Mesothelioma"}, {"label": "C", "disease": "Rotator cuff tendinopathy"}, {"label": "D", "disease": "Syringomyelia"}, {"label": "E", "disease": "Tuberculosis"}], "answer_idx": "A", "symptoms": ["A 69-year-old man presents to clinic due to shortness of breath, worsening pain in his right shoulder, and episodes of hemoptysis", "His symptoms began 3 months ago", "He has also lost 18 pounds", "He has a history of coronary artery disease and underwent an uncomplicated coronary angioplasty with stent placement 2 years ago", "He routinely travels both domestically and internationally as a senior shipyard worker and has a 30-pack-year smoking history", "He used to drink 4 cocktails a week", "He has not smoke or drank alcohol in over 7 years", "His temperature is 99.5°F (37.7°C), blood pressure is 140/60 mmHg, pulse is 97/min, and respirations are 13/min", "Physical exam is notable for right pupillary constriction as well as paresthesias in his right fourth and fifth digits", "There is no pain with active shoulder rotation", "His chest imaging is shown in Figure A"], "s1": [0, 2, 8, 10], "s2": [1, 3, 4, 5, 6, 7, 9]} {"key": 27, "questions": "Which of the following describes this patient's most likely diagnosis?", "options": [{"label": "A", "disease": "Benign capillary proliferation"}, {"label": "B", "disease": "Edema of the epidermis"}, {"label": "C", "disease": "Healthy dermatologic development"}, {"label": "D", "disease": "Malignant blood vessel proliferation"}, {"label": "E", "disease": "Viral infection"}], "answer_idx": "A", "symptoms": ["A 39-year-old man presents to his doctor for a wellness checkup", "He is concerned about a rash that does not seem to be improving", "He was recently exposed to his grandfather who has vesicular lesions on his skin and is being treated", "He has a family history of skin cancer, colon cancer, and ovarian cancer", "The patient has a medical history of asthma and seasonal allergies", "His temperature is 98.6°F (37.0°C), blood pressure is 137/98 mmHg, pulse is 90/min, respirations are 14/min, and oxygen saturation is 98% on room air", "Physical exam reveals the finding in Figure A"], "s1": [1, 2, 6], "s2": [0, 3, 4, 5]} {"key": 28, "questions": "Which of the following is the most likely etiology of this patient’s presentation?", "options": [{"label": "A", "disease": "Meconium aspiration syndrome"}, {"label": "B", "disease": "Neonatal respiratory distress syndrome"}, {"label": "C", "disease": "Persistent pulmonary hypertension"}, {"label": "D", "disease": "Transient tachypnea of the newborn"}, {"label": "E", "disease": "Viral pneumonia"}], "answer_idx": "D", "symptoms": ["A newborn boy is evaluated 30 minutes after birth", "He was born at 39 weeks gestation to a 27-year-old primigravid mother via cesarean section for cervical insufficiency", "The pregnancy was complicated by gestational diabetes and the amniotic fluid was clear", "Upon delivery, the patient had strong respiratory effort and a strong cry", "His Apgar scores at 1 and 5 minutes were 7 and 8, respectively", "The patient now is exhibiting increased work of breathing and is progressively more tachypneic", "His birth weight is 3,568 g (7 lb 14 oz)", "His temperature is 99.0°F (37.2°C), blood pressure is 60/44 mmHg, pulse is 146/min, and respirations are 72/min", "On physical exam, the patient is grunting with nasal flaring and subcostal retractions", "Breath sounds are decreased at the bases bilaterally", "The patient has central cyanosis", "His chest radiograph can be seen in Figure A"], "s1": [0, 1, 2, 3, 4, 6, 11], "s2": [5, 7, 8, 9, 10]} {"key": 29, "questions": "Which of the following conditions is most likely to occur in this patient based on his current history and ECG?", "options": [{"label": "A", "disease": "Atrial fibrillation"}, {"label": "B", "disease": "Complete heart block"}, {"label": "C", "disease": "Myocardial infarction"}, {"label": "D", "disease": "Stroke"}, {"label": "E", "disease": "Torsades des pointes"}], "answer_idx": "B", "symptoms": ["A 55-year-old man presents to the emergency department after fainting while moving furniture", "He states that he has been working in a hot, humid attic all day and forgot his water bottle", "He felt dizzy when standing up and then fainted and was unconscious for roughly 1 minute", "He awoke at his neurologic baseline afterwards", "The patient has a history of hypertension being managed by his primary care doctor", "His temperature is 98.7°F (37.1°C), blood pressure is 149/82 mmHg, pulse is 86/min, respirations are 15/min, and oxygen saturation is 99% on room air", "Physical exam reveals a diaphoretic man", "He demonstrates an unremarkable cardiopulmonary exam", "The patient walks with a steady gait and his neurologic exam is non-focal", "An ECG is performed as seen in Figure A", "The patient is given 2 liters of oral fluids and feels back to his baseline"], "s1": [0, 1, 2, 3, 6, 10], "s2": [4, 5, 7, 8, 9]} {"key": 30, "questions": "Which of the following is the most likely diagnosis?", "options": [{"label": "A", "disease": "Duodenal atresia"}, {"label": "B", "disease": "Hirschsprung disease"}, {"label": "C", "disease": "Intestinal malrotation"}, {"label": "D", "disease": "Meconium ileus"}, {"label": "E", "disease": "Pyloric stenosis"}], "answer_idx": "B", "symptoms": ["A 2-day-old boy has an episode of vomiting in the hospital nursery", "The vomitus was described as “bright green” without any traces of blood", "The patient has urinated several times since he was born but has not passed any stool", "He was born at 37 weeks of gestation to a 38-year-old G3P3 woman", "The pregnancy was uncomplicated and the patient’s mother refused all prenatal testing", "The patient’s 2 older siblings are both healthy", "His temperature is 98.6°F (37°C), blood pressure is 67/43 mmHg, pulse is 135/min, and respirations are 34/min", "On physical exam, the patient has upslanting palpebral fissures, epicanthal folds, and a single transverse palmar crease", "His abdomen is non-tender, firm, and distended", "Bowel sounds are hypoactive", "Digital rectal exam evacuates a small amount of stool and flatulence", "A nasogastric tube is placed to decompress the stomach", "The patient’s abdominal radiograph can be seen in Figure A"], "s1": [0, 1, 2, 6, 8, 9, 10, 11], "s2": [3, 4, 5, 7, 12]} {"key": 31, "questions": "Which of the following is the most likely diagnosis?", "options": [{"label": "A", "disease": "Cardiac tamponade"}, {"label": "B", "disease": "Constrictive pericarditis"}, {"label": "C", "disease": "Myocardial infarction"}, {"label": "D", "disease": "Pulmonary embolism"}, {"label": "E", "disease": "Restrictive cardiomyopathy"}], "answer_idx": "B", "symptoms": ["A 55-year-old man presents to the emergency department with worsening dyspnea over the past 48 hours", "He recently had a cold that kept him home from work for the past week", "He has a past medical history of diabetes, obesity, and hypertension", "He had his Achilles tendon repaired 4 weeks ago and he has been less mobile", "His temperature is 99.2°F (37.3°C), blood pressure is 150/85 mmHg, pulse is 82/min, respirations are 16/min, and oxygen saturation is 100% on room air", "Physical exam is notable for dyspnea provoked by walking short distances", "Jugular venous distension is noted on exam", "The patient’s blood pressure is 130/70 mmHg during inspiration", "A bedside echocardiogram demonstrates impaired diastolic filling with normal ventricular contractility", "An ECG is performed as seen in Figure A"], "s1": [0, 1, 2, 3, 4], "s2": [5, 6, 7, 8, 9]} {"key": 32, "questions": "Which of the following is the most likely underlying diagnosis?", "options": [{"label": "A", "disease": "Acute inflammatory demyelinating polyneuropathy"}, {"label": "B", "disease": "Cauda equina syndrome"}, {"label": "C", "disease": "L5/S1 disc herniation"}, {"label": "D", "disease": "Overflow incontinence"}, {"label": "E", "disease": "Spinal stenosis"}], "answer_idx": "B", "symptoms": ["A 65-year-old man presents to the emergency department with low back pain", "The pain started 6 hours ago when he was gardening and carrying a heavy bag", "The pain is rated as 10/10 in severity and radiates down the posterior aspect of the right thigh", "He reports no fevers or chills", "His only medical problem is hypertension for which he takes amlodipine", "The patient is a non-smoker, uses alcohol infrequently, and has never used illicit drugs", "His temperature is 99.6°F (37.6°C), blood pressure is 140/70 mmHg, pulse is 90/min, and respirations are 20/min", "Physical examination reveals 3/5 strength to hip extension, knee flexion and extension, and plantar flexion bilaterally", "Sensation to pinprick is diminished over the posterolateral legs and lateral aspects of both feet", "Ankle and knee reflexes are absent bilaterally", "The patient’s underwear is wet and a bladder scan reveals 800 mL of urine"], "s1": [0, 1, 3, 4, 5, 6], "s2": [2, 7, 8, 9, 10]} {"key": 33, "questions": "Which of the following is the most likely diagnosis?", "options": [{"label": "A", "disease": "Advanced sleep phase disorder"}, {"label": "B", "disease": "Chronic insomnia"}, {"label": "C", "disease": "Delayed sleep phase disorder"}, {"label": "D", "disease": "Normal aging"}, {"label": "E", "disease": "Obstructive sleep apnea"}], "answer_idx": "A", "symptoms": ["A 69-year-old man presents to his primary care physician for trouble sleeping", "He recently retired from working the day shift at a cemetery", "When the patient retired, his goal was to finally be able to go out with his wife", "however, he finds that he is unable to stay awake past 6 pm in the evening", "His wife is disappointed that they cannot do any activities in the evening together", "The patient has tried drinking caffeine but finds that it does not help", "The patient’s wife claims that the patient seems to sleep peacefully, and the patient states he feels rested when he awakes", "The patient has a medical history of irritable bowel syndrome which is managed with fiber supplements", "His temperature is 98.6°F (37°C), blood pressure is 125/83 mmHg, pulse is 87/min, and respirations are 11/min", "The patient’s neurological exam is within normal limits"], "s1": [0, 1, 2, 3, 4, 5, 6], "s2": [7, 8, 9]} {"key": 34, "questions": "Which of the following is the most likely diagnosis?", "options": [{"label": "A", "disease": "Cerebral toxoplasmosis"}, {"label": "B", "disease": "Glioblastoma multiforme"}, {"label": "C", "disease": "Hemorrhagic stroke"}, {"label": "D", "disease": "Metastatic brain tumor"}, {"label": "E", "disease": "Neurosarcoidosis"}], "answer_idx": "B", "symptoms": ["A 66-year-old woman presents to the emergency room after experiencing a tonic-clonic seizure", "She has been having worsening headaches and intermittent nausea over the past 5 months", "The headaches are constant and dull, and they typically worsen when she sneezes or laughs", "Recently, she has become increasingly nauseous and has vomited twice in the past 10 days", "Her medical history is significant for type 2 diabetes mellitus and hypertension for which she takes metformin and losartan", "Her temperature is 98.6°F (37.0°C), blood pressure is 115/70 mmHg, pulse is 70/min, and respirations are 12/min", "She is noted to have papilledema on fundoscopy, but physical exam is otherwise normal", "Her CT scan findings are presented in Figure A"], "s1": [0, 1, 2, 3, 6], "s2": [4, 5, 7]} {"key": 35, "questions": "Which of the following is the most likely diagnosis?", "options": [{"label": "A", "disease": "Left ventricular hypertrophy"}, {"label": "B", "disease": "Non-ST elevation myocardial infarction"}, {"label": "C", "disease": "ST elevation myocardial infarction"}, {"label": "D", "disease": "Torsades des pointes"}, {"label": "E", "disease": "Wolff Parkinson White syndrome"}], "answer_idx": "A", "symptoms": ["A 55-year-old man presents to the emergency department after feeling lightheaded then fainting while he was carrying boxes upstairs", "The patient works on a farm and does not see a doctor often", "He has a history of hypertension but does not take medications routinely", "He has not had any chest pain, dyspnea, or weakness during this time frame and currently feels at his baseline", "He smokes cigarettes occasionally", "His temperature is 98.1°F (36.7°C), blood pressure is 152/93 mmHg, pulse is 86/min, respirations are 15/min, and oxygen saturation is 98% on room air", "Physical exam is unremarkable and the patient walks with a stable gait and no focal weakness", "An ECG is performed as seen in Figure A", "Laboratory studies are ordered as seen below", "Hemoglobin: 12 g/dL", "Hematocrit: 36%", "Leukocyte count: 6,500/mm^3 with normal differential", "Platelet count: 197,000/mm^3", "Serum:", "Na+: 140 mEq/L", "Cl-: 102 mEq/L", "K+: 4.3 mEq/L", "HCO3-: 24 mEq/L", "BUN: 20 mg/dL", "Glucose: 139 mg/dL", "Creatinine: 1.1 mg/dL", "Troponin: < 0.01 ng/mL"], "s1": [0, 1, 2, 3, 4, 6], "s2": [5, 7, 8, 9, 10, 11, 12, 13, 14, 15, 16, 17, 18, 19, 20, 21]} {"key": 36, "questions": "Which of the following is the most likely diagnosis?", "options": [{"label": "A", "disease": "Anterior cruciate ligament tear"}, {"label": "B", "disease": "Apophysitis of the tibial tubercle"}, {"label": "C", "disease": "Patellar fracture"}, {"label": "D", "disease": "Posterior dislocation of the knee"}, {"label": "E", "disease": "Tibial plateau fracture"}], "answer_idx": "E", "symptoms": ["A 27-year-old man presents to the emergency department after he developed pain in his right leg after landing off a ski jump", "The patient is otherwise healthy and does not take any medications", "He states his pain is 10/10. He is requesting medications and is crying out in pain", "His temperature is 98.7°F (37.1°C), blood pressure is 149/85 mmHg, pulse is 103/min, respirations are 15/min, and oxygen saturation is 99% on room air", "Physical exam reveals swelling over his right leg and knee", "The patient cries out in pain with passive motion of the foot at the ankle", "A radiograph is performed as seen in Figure A"], "s1": [0, 1, 6], "s2": [2, 3, 4, 5]} {"key": 37, "questions": "Which of the following is the most likely etiology of this patient’s condition?", "options": [{"label": "A", "disease": "Duodenal atresia"}, {"label": "B", "disease": "Hirschsprung disease"}, {"label": "C", "disease": "Intestinal malrotation"}, {"label": "D", "disease": "Jejunal atresia"}, {"label": "E", "disease": "Meconium ileus"}], "answer_idx": "C", "symptoms": ["A 4-day-old girl presents with her parents to the pediatrician for a routine visit", "She was feeding well until this morning when she had several episodes of vomiting", "The parents describe the vomitus as “bright green” and the patient has shown little interest in feeding since then", "She was born at 36 weeks gestation to a 37-year-old G2P2 mother", "The pregnancy was complicated by gestational diabetes, and all prenatal testing was unremarkable", "The patient has not yet regained her birth weight and is in the 46th percentile for height and the 36th percentile for weight", "The patient’s temperature is 99.2°F (37.3°C), blood pressure is 68/46 mmHg, pulse is 132/min, and respirations are 32/min", "On physical exam, the patient is in mild distress", "She has no dysmorphic features", "Her abdomen is distended and firm without guarding", "Bowel sounds are hypoactive", "The patient’s abdominal radiograph can be seen in Figure A and the patient’s upper gastrointestinal series can be seen in Figure B"], "s1": [0, 3, 4, 5, 6, 8], "s2": [1, 2, 7, 9, 10, 11]} {"key": 38, "questions": "Which of the following is the most likely diagnosis?", "options": [{"label": "A", "disease": "Acute lymphoblastic leukemia"}, {"label": "B", "disease": "Acute myelogenous leukemia"}, {"label": "C", "disease": "Chronic lymphocytic leukemia"}, {"label": "D", "disease": "Chronic myelogenous leukemia"}, {"label": "E", "disease": "Hairy cell leukemia"}], "answer_idx": "C", "symptoms": ["A 72-year-old man presents to his primary care physician for a general checkup", "He states that he has been a bit more fatigued lately but believes it is secondary to poor sleep at his new house", "The patient is otherwise healthy and takes no medications", "His temperature is 98.0°F (36.7°C), blood pressure is 141/90 mmHg, pulse is 82/min, respirations are 16/min, and oxygen saturation is 98% on room air", "Physical exam is notable only for minor pallor but is otherwise unremarkable", "Basic laboratory studies are ordered as seen below", "Hemoglobin: 10 g/dL", "Hematocrit: 30%", "Leukocyte count: 67,500/mm^3", "Platelet count: 119,000/mm^3", "Serum:", "Na: 141 mEq/L", "Cl: 103 mEq/L", "K: 4.0 Eq/L", "HCO3-: 24 mEq/L", "BUN: 22 mg/dL", "Glucose: 109 mg/dL", "Creatinine: 1.2 mg/dL", "Ca: 10.0 mg/dL"], "s1": [0, 1, 2, 3, 4], "s2": [5, 6, 7, 8, 9, 10, 11, 12, 13, 14, 15, 16, 17, 18]} {"key": 39, "questions": "Which of the following is the most likely diagnosis?", "options": [{"label": "A", "disease": "Chalazion"}, {"label": "B", "disease": "Foreign body"}, {"label": "C", "disease": "Hordeolum"}, {"label": "D", "disease": "Ingrown eyelash follicle"}, {"label": "E", "disease": "Meibomian cell carcinoma"}], "answer_idx": "A", "symptoms": ["A 37-year-old machinist presents to his primary care physician with eye problems", "He has felt a mass in his eye that has persisted for the past month", "The patient has a medical history of blepharitis treated with eye cleansing and squamous cell carcinoma of the skin treated with Mohs surgery", "His temperature is 99.5°F (37.5°C), blood pressure is 157/102 mmHg, pulse is 90/min, respirations are 17/min, and oxygen saturation is 98% on room air", "Physical exam is notable for a firm and rubbery nodule palpable inside the patient's left eyelid", "Physical exam does not elicit any pain"], "s1": [3, 5], "s2": [0, 1, 2, 4]} {"key": 40, "questions": "Which of the following is the most likely diagnosis?", "options": [{"label": "A", "disease": "First degree atrioventricular block"}, {"label": "B", "disease": "Second degree atrioventricular block, Mobitz type I"}, {"label": "C", "disease": "Second degree atrioventricular block, Mobitz type II"}, {"label": "D", "disease": "Sinus bradycardia"}, {"label": "E", "disease": "Third degree atrioventricular block"}], "answer_idx": "B", "symptoms": ["A 24-year-old woman presents to clinic for a routine annual exam", "She has generally been feeling well, but notes feeling intermittent palpitations over the past few months", "Her past medical history is unremarkable and she is not currently taking any medications", "She denies drinking alcohol, smoking cigarettes, or using recreational drugs", "She attributes her palpitations to recently drinking more caffeine, but would like to obtain an electrocardiogram since her symptoms are worrisome and affecting her ability to concentrate on her career as a fitness instructor", "Her temperature is 36.9°C (98.4°F), blood pressure is 116/76 mmHg, pulse is 55/min, respirations are 12/min, and oxygen saturation is 98% on room air", "Her electrocardiogram is shown in Figure A"], "s1": [0, 2, 3, 5, 6], "s2": [1, 4]} {"key": 41, "questions": "Which of the following is the most likely diagnosis?", "options": [{"label": "A", "disease": "Basal cell carcinoma"}, {"label": "B", "disease": "Keratoacanthoma"}, {"label": "C", "disease": "Molluscum contagiosum"}, {"label": "D", "disease": "Pyogenic granuloma"}, {"label": "E", "disease": "Verruca vulgaris"}], "answer_idx": "B", "symptoms": ["A 66-year-old man presents to the clinic for evaluation of a skin lesion on his hand", "The lesion has grown rapidly over the previous 2 weeks", "He endorses trauma to the area when he accidentally scraped his hand against a window and notes that there was some bleeding", "His medical history is significant for chronic renal failure for which he received a renal transplant 5 years prior, as well as multiple surgeries for skin cancer", "His current medications include aspirin, atorvastatin, prednisone, tacrolimus, and mycophenolate mofetil", "His temperature is 98.6°F (37°C), blood pressure is 136/91 mmHg, pulse is 82/min, and respirations are 11/min", "Physical exam is notable for a 2 x 2.5 cm nodular lesion of the dorsal hand as seen in Figure A"], "s1": [0, 1, 2, 6], "s2": [3, 4, 5]} {"key": 42, "questions": "Which of the following is the most likely diagnosis?", "options": [{"label": "A", "disease": "Alzheimer disease"}, {"label": "B", "disease": "Mild cognitive impairment"}, {"label": "C", "disease": "Normal aging"}, {"label": "D", "disease": "Pseudodementia"}, {"label": "E", "disease": "Vascular dementia"}], "answer_idx": "E", "symptoms": ["A 72-year-old man with a history of type 2 diabetes mellitus, hypertension, and hyperlipidemia presents to his primary care provider for concerns about his cognitive decline", "His wife has noticed that he has a worsening ability to organize, plan, and exhibit impulse control over the last month", "The patient states that he is able to complete his activities of daily living without assistance, but has some weakness of his left upper and lower extremities that began 3 months prior", "He needs reminders from his wife about daily tasks to complete", "He enjoys spending time with his spouse, playing cards with his friends, and taking daily walks around his neighborhood", "His temperature is 98.2°F (36.8°C), blood pressure is 149/87 mmHg, pulse is 87/min, and respirations are 12/min", "Physical exam reveals an elderly man who is oriented to person, place, and time", "He has 4/5 left-sided weakness of the upper and lower extremities and associated pronator drift of the left upper extremity", "A mini-mental status exam reveals he is able to remember 2 out of 3 words after 4 minutes", "He is able to appropriately draw a clock", "A magnetic resonance imaging study of the brain is shown in Figure A"], "s1": [0, 1, 3, 4, 8, 9], "s2": [2, 5, 6, 7, 10]} {"key": 43, "questions": "Which of the following is the most likely diagnosis?", "options": [{"label": "A", "disease": "Acute lymphoblastic lymphoma"}, {"label": "B", "disease": "Chronic lymphocytic leukemia"}, {"label": "C", "disease": "Chronic myeloid leukemia"}, {"label": "D", "disease": "Leukemoid reaction"}, {"label": "E", "disease": "Multiple myeloma"}], "answer_idx": "D", "symptoms": ["A 59-year-old man presents to his primary care physician for fatigue", "In general, he has been in good health", "however, he recently has experienced weight loss, abdominal pain, and general fatigue", "He has a medical history of anxiety, diabetes, a fracture of his foot sustained when he tripped, and a recent cold that caused him to miss work for 1 week", "His current medications include metformin, insulin, buspirone, vitamin D, calcium, and sodium docusate", "His temperature is 99.5°F (37.5°C), blood pressure is 150/100 mmHg, pulse is 90/min, respirations are 18/min, and oxygen saturation is 98% on room air", "Physical exam reveals a calm gentleman", "A mild systolic murmur is heard in the left upper sternal region", "The rest of the physical exam is within normal limits", "Laboratory values are ordered as seen below", "Hemoglobin: 12 g/dL Hematocrit: 36% Leukocyte count: 66,500/mm^3 with normal differential Platelet count: 177,000/mm^3 Leukocyte alkaline phosphatase: elevated Serum: Na+: 139 mEq/L Cl-: 100 mEq/L K+: 4.3 mEq/L BUN: 20 mg/dL Glucose: 120 mg/dL Creatinine: 1.1 mg/dL Ca2+: 10.9 mEq/L AST: 12 U/L ALT: 10 U/L"], "s1": [0, 2, 5, 7], "s2": [1, 3, 4, 6, 8, 9, 10]} {"key": 44, "questions": "Which of the following is the most likely diagnosis?", "options": [{"label": "A", "disease": "Common fibular nerve compression"}, {"label": "B", "disease": "Herniated disc"}, {"label": "C", "disease": "Plantar fasciitis"}, {"label": "D", "disease": "Tarsal tunnel syndrome"}, {"label": "E", "disease": "Vitamin B12 deficiency"}], "answer_idx": "D", "symptoms": ["A 35-year-old man presents to his primary care physician with pain along the bottom of his foot", "The patient is a long-time runner but states that the pain has been getting worse recently", "When running and at rest he has a burning and aching pain along the bottom of his foot that sometimes turns to numbness", "Taking time off from training does not improve his symptoms", "The patient has a medical history of surgical repair of his Achilles tendon, ACL, and medial meniscus", "He is currently not taking any medications", "The patient lives with his wife and they both practice a vegan lifestyle", "His temperature is 98.6°F (37°C), blood pressure is 114/72 mmHg, pulse is 81/min, and respirations are 12/min", "On physical exam, the patient states that he is currently not experiencing any pain in his foot but rather is experiencing numbness/tingling along the plantar surface of his foot", "Strength is 5/5 and reflexes are 2+ in the lower extremities"], "s1": [0, 1, 2, 3, 4, 8], "s2": [5, 6, 7, 9]} {"key": 45, "questions": "Which of the following is the most likely diagnosis?", "options": [{"label": "A", "disease": "Hyperkalemia"}, {"label": "B", "disease": "Hypertrophic obstructive cardiomyopathy"}, {"label": "C", "disease": "Hypokalemia"}, {"label": "D", "disease": "Myocardial infarction"}, {"label": "E", "disease": "Pericarditis"}], "answer_idx": "D", "symptoms": ["A 50-year-old man presents to the emergency department with chest pain", "He states that the pain is dull in quality, started 30 minutes ago, has been gradually worsening, and is worse with exertion", "He also endorses some shortness of breath", "He also believes the pain is worse when leaning back and improved when leaning forward", "His wife noticed he fainted after the pain started but regained consciousness shortly after", "He has a medical history of diabetes and hypertension for which he is on metformin and losartan", "His temperature is 99.2°F (37.3°C), blood pressure is 130/87 mmHg, pulse is 99/min, respirations are 22/min, and oxygen saturation is 100% on room air", "Physical exam reveals an overweight man", "A normal S1 and S2 are auscultated with clear breath sounds", "An ECG is performed as seen in Figure A"], "s1": [0, 1, 2, 3, 4], "s2": [5, 6, 7, 8, 9]} {"key": 46, "questions": "Which of the following is the most likely diagnosis?", "options": [{"label": "A", "disease": "Acute lymphoblastic leukemia"}, {"label": "B", "disease": "Acute myelogenous leukemia"}, {"label": "C", "disease": "Chronic myelogenous leukemia"}, {"label": "D", "disease": "Leukemoid reaction"}, {"label": "E", "disease": "Parvovirus B19 infection"}], "answer_idx": "A", "symptoms": ["A 5-year-old boy presents to his primary care physician for recurrent colds, fatigue, and fussiness", "Over the past several weeks, the child has been more fatigued and his parents state that “he always seems to be sick", "” They state that sometimes he complains about his bones hurting and they note that he is less playful", "He is up to date on his vaccinations", "His temperature is 102°F (38.9°C), blood pressure is 92/60 mmHg, pulse is 115/min, respirations are 23/min, and oxygen saturation is 99% on room air", "Physical exam is notable for diffuse non-tender lymphadenopathy", "Abdominal exam reveals hepatosplenomegaly", "Laboratory studies are ordered as seen below", "Hemoglobin: 8.0 g/dL", "Hematocrit: 23%", "Leukocyte count: 27,500/mm^3 with 35% lymphoblasts", "Platelet count: 49,000/mm^3", "Serum:", "Na+: 139 mEq/L", "Cl-: 101 mEq/L", "K+: 4.0 Eq/L", "HCO3-: 24 mEq/L", "BUN: 16 mg/dL", "Glucose: 100 mg/dL", "Creatinine: 0.7 mg/dL"], "s1": [0, 1, 2, 3, 4, 5, 6], "s2": [7, 8, 9, 10, 11, 12, 13, 14, 15, 16, 17, 18, 19]} {"key": 47, "questions": "Which of the following is the most likely diagnosis?", "options": [{"label": "A", "disease": "Central retinal artery occlusion"}, {"label": "B", "disease": "Closed angle glaucoma"}, {"label": "C", "disease": "Papilledema"}, {"label": "D", "disease": "Retinal detachment"}, {"label": "E", "disease": "Retinal vein occlusion"}], "answer_idx": "E", "symptoms": ["A 71-year-old woman arrives to clinic with sudden vision loss", "She was in her usual state of health until waking up this morning unable to see out of her right eye", "She denies pain, but reports her right eye “feels funny", "” The patient’s medical history is significant for a previous myocardial infarction, hypertension, and osteoporosis", "Her current medications include aspirin, metoprolol, rosuvastatin, lisinopril, and alendronate", "Her temperature is 98°F (36.7°C), blood pressure is 145/86 mmHg, pulse is 62/min, and respirations are 12/min with an oxygen saturation of 98% on room air", "The patient's pupils are symmetric in size and equally reactive to light with accommodation", "A fundoscopic exam of the right eye is shown in Figure A", "The left optic fundus is unremarkable"], "s1": [0, 1, 2, 7, 8], "s2": [3, 4, 5, 6]} {"key": 48, "questions": "Which of the following is the most likely diagnosis?", "options": [{"label": "A", "disease": "Costochondritis"}, {"label": "B", "disease": "Non-ST elevation myocardial infarction"}, {"label": "C", "disease": "ST elevation myocardial infarction"}, {"label": "D", "disease": "Stable angina"}, {"label": "E", "disease": "Unstable angina"}], "answer_idx": "D", "symptoms": ["A 72-year-old man presents to the clinic for his annual exam", "For the past few months, he has pain in his chest with physical activity", "The pain goes away after he takes a break", "He reports no chest pain while he is resting, and the pain is not worsening", "His past medical history is notable for hypertension, type 2 diabetes mellitus, and hyperlipidemia", "His medications include amlodipine, atorvastatin, and metformin", "His temperature is 36.5°C (97.7°F), blood pressure is 132/80 mmHg, pulse is 74/min, respirations are 14/min, and oxygen saturation is 98% on room air", "He has a regular rate and rhythm, normal S1 and S2, and no murmurs, rubs, or gallops", "His anterior, lateral, and posterior chest are non-tender to palpation", "His electrocardiogram is shown in Figure A", "Troponin I level is < 0.017 ng/mL"], "s1": [0, 1, 2, 3, 4], "s2": [5, 6, 7, 8, 9, 10]} {"key": 49, "questions": "Which of the following is the most likely diagnosis?", "options": [{"label": "A", "disease": "Alzheimer dementia"}, {"label": "B", "disease": "Creutzfeldt-Jakob disease"}, {"label": "C", "disease": "Frontotemporal dementia"}, {"label": "D", "disease": "Normal pressure hydrocephalus"}, {"label": "E", "disease": "Vascular dementia"}], "answer_idx": "A", "symptoms": ["A 76-year-old woman is brought to a primary care physician by her daughter who is concerned about her mother's growing inability to take care of herself", "Initially, she attributed her mother's forgetfulness and word-finding difficulties to normal aging, but over the past few years, her mother's memory has worsened, her mood has grown more irritable, and her mother has been found wandering the neighborhood unsure of how to get home", "Her medical history includes type 2 diabetes mellitus which is well-controlled with metformin", "On exam, her temperature is 98.2°F (36.8°C), blood pressure is 115/82 mmHg, pulse is 73/min, respirations are 12/min, and oxygen saturation is 99% on room air", "Cardiopulmonary exam reveals normal S1 and S2, no murmurs, and clear lungs bilaterally", "She scores 16/30 on the Montreal Cognitive Assessment (MoCA) test"], "s1": [0, 1, 5], "s2": [2, 3, 4]} {"key": 50, "questions": "Which of the following is the most likely diagnosis?", "options": [{"label": "A", "disease": "Bacterial meningitis"}, {"label": "B", "disease": "Fat-soluble vitamin overuse"}, {"label": "C", "disease": "Migraine headache"}, {"label": "D", "disease": "Subarachnoid hemorrhage"}, {"label": "E", "disease": "Viral meningitis"}], "answer_idx": "B", "symptoms": ["A 17-year-old girl presents to the emergency department with a headache", "The patient has had headaches in the past but this is the worst headache of her life", "Her symptoms started yesterday and have been getting progressively worse", "The patient states that the pain is mostly on the left side of her head", "There has been a recent outbreak of measles at the patient’s school and the patient’s mother has been trying to give her daughter medicine to prevent her from getting sick", "Her mother fears that her daughter may have caught measles", "Her temperature is 98.6°F (37°C), blood pressure is 123/74 mmHg, pulse is 85/min, and respirations are 13/min", "On exam, the patient is an obese girl who is clutching her head with the light in the room turned off", "Her neurological exam is within normal limits", "Fundoscopic exam reveals mild bilateral papilledema", "An MRI of the head is obtained and reveals cerebral edema", "A lumbar puncture reveals an increased opening pressure with a normal glucose level"], "s1": [0, 1, 2, 3, 7, 9, 10, 11], "s2": [4, 5, 6, 8]} {"key": 51, "questions": "Which of the following is the most likely etiology of this patient's symptoms?", "options": [{"label": "A", "disease": "Epidural hematoma"}, {"label": "B", "disease": "Intracerebral hemorrhage"}, {"label": "C", "disease": "Ischemic stroke"}, {"label": "D", "disease": "Subarachnoid hemorrhage"}, {"label": "E", "disease": "Subdural hematoma"}], "answer_idx": "B", "symptoms": ["A 77-year-old man presents to the emergency department with a complaint of sudden onset weakness in his right upper extremity", "At home, the patient thought he was simply dehydrated, but he rapidly began to have trouble speaking", "When his wife noted this she brought him into the hospital", "On arrival to the emergency department, the patient is not responding to verbal stimuli and only withdraws his left upper extremity and lower extremity to pain", "His past medical history is notable for hypertension and atrial fibrillation treated with metoprolol, apixaban, and lisinopril", "His temperature is 99.0°F (37.2°C), blood pressure is 170/100 mmHg, pulse is 95/min, and respirations are 16/min", "The patient has a seizure and subsequently demonstrates agonal breathing", "He no longer responds to painful stimuli", "A fingerstick blood glucose is 122 mg/dL"], "s1": [0, 1, 2, 3, 6, 7], "s2": [4, 5, 8]} {"key": 52, "questions": "Which of the following is the most likely etiology of this patient's symptoms?", "options": [{"label": "A", "disease": "Acetaminophen"}, {"label": "B", "disease": "Arsenic"}, {"label": "C", "disease": "Cyanide"}, {"label": "D", "disease": "Iron"}, {"label": "E", "disease": "Lead"}], "answer_idx": "B", "symptoms": ["A 27-year-old man presents to the emergency department with altered mental status", "He has become gradually more confused over the past several days", "His wife also notes he has had diarrhea, nausea and vomiting, and abdominal pain for the past week", "The patient has a history of depression and multiple suicide attempts", "He takes fluoxetine as well as over-the-counter pain medications for a recent muscle strain", "He works in a large industrial compound that manufactures semiconductors", "He exercises regularly and recently has started making his own beer at home", "His temperature is 98.3°F (36.8°C), blood pressure is 107/75 mmHg, pulse is 110/min, respirations are 22/min, and oxygen saturation is 99% on room air", "Physical exam reveals a confused man with a garlic odor on his breath and the finding in Figure A", "Cardiopulmonary exam reveals a rapid heart rate with no murmurs and clear breath sounds"], "s1": [0, 1, 2, 3, 7, 8, 9], "s2": [4, 5, 6]} {"key": 53, "questions": "Which of the following is the most likely diagnosis?", "options": [{"label": "A", "disease": "Ankylosing spondylitis"}, {"label": "B", "disease": "Degenerative spine disease"}, {"label": "C", "disease": "Herniated disc"}, {"label": "D", "disease": "Metastatic tumor"}, {"label": "E", "disease": "Osteomyelitis"}], "answer_idx": "E", "symptoms": ["A 54-year-old man presents to the clinic with 4 weeks of persistently worsening back pain", "The pain is localized to 1 spot in his lower back and is worse with physical activity", "His medical history is pertinent for intravenous drug use", "He has no past surgical history", "His temperature is 36.8°C (98.2°F), blood pressure is 118/90 mmHg, pulse is 92/min, respirations are 13/min, and oxygen saturation is 99% on room air", "On physical exam, he has midline back tenderness to palpation at L2-L3. Laboratory workup reveals an erythrocyte sedimentation rate of 112 mm/h and C-reactive protein of 10 mg/dL"], "s1": [0, 1, 5], "s2": [2, 3, 4]} {"key": 54, "questions": "Which of the following is the most likely diagnosis?", "options": [{"label": "A", "disease": "Acute pancreatitis"}, {"label": "B", "disease": "Aortic dissection"}, {"label": "C", "disease": "Ischemic colitis"}, {"label": "D", "disease": "Perforated viscus"}, {"label": "E", "disease": "Ruptured abdominal aortic aneurysm"}], "answer_idx": "E", "symptoms": ["A 67-year-old man presents to the emergency room reporting sudden-onset abdominal and back pain that began 3 hours ago", "He was eating dinner when he started feeling severe pain in his abdomen that made him lose his appetite", "He denies any diarrhea or hematochezia and says the pain is not affected by movement", "He has a history of hypertension, hyperlipidemia, and depression", "He currently takes amlodipine and atorvastatin and is compliant with his medications", "He has a 45-pack-year smoking history, but he does not drink alcohol or use any illicit drugs", "His temperature is 98.6°F (37°C), his blood pressure is 110/64 mmHg, pulse is 97/min, and respirations are 15/min", "Physical exam is notable for diffuse abdominal tenderness without rigidity or guarding", "During evaluation, the patient becomes diaphoretic and pale and reports that he feels fatigued", "Repeat blood pressure is 87/50 and pulse is 127. Intravenous fluid boluses are administered"], "s1": [0, 1, 2, 7, 8], "s2": [3, 4, 5, 6, 9]} {"key": 55, "questions": "Which of the following is most likely to confirm the diagnosis?", "options": [{"label": "A", "disease": "Abdominal radiograph"}, {"label": "B", "disease": "CT imaging of abdomen and pelvis"}, {"label": "C", "disease": "Flexible sigmoidoscopy"}, {"label": "D", "disease": "Rectal swab for selective anaerobic culture"}, {"label": "E", "disease": "Stool toxin assay"}], "answer_idx": "E", "symptoms": ["A 68-year-old woman presents to the emergency room with several days of frequent, nonbloody, watery stools", "She recently completed a course of clindamycin 2 weeks ago for a tooth infection", "Her medical history is notable for hypertension and hyperlipidemia for which she takes amlodipine and atorvastatin", "She traveled recently and has no sick contacts", "Her temperature is 37.0°C (98.6°F), blood pressure is 118/82 mmHg, pulse is 98/min, respirations are 14/min, and oxygen saturation is 98% on room air", "On exam, her abdomen is soft, non-distended, and non-tender to palpation", "however she has diffuse discomfort upon palpation of the abdomen with no rebound, rigidity, or guarding"], "s1": [0, 1, 3, 6], "s2": [2, 4, 5]} {"key": 56, "questions": "Which of the following is the most likely diagnosis?", "options": [{"label": "A", "disease": "Generalized anxiety disorder"}, {"label": "B", "disease": "Major depressive disorder"}, {"label": "C", "disease": "Panic disorder"}, {"label": "D", "disease": "Premenstrual dysophoric disorder"}, {"label": "E", "disease": "Premenstrual syndrome"}], "answer_idx": "D", "symptoms": ["A 21-year-old woman presents to her primary care physician due to a history of abdominal pain", "She states that she has had recurrent bouts of mood swings, bloating, and abdominal pain that occur together approximately every 4 weeks", "She states that when these symptoms occur, she gets into frequent arguments at work, is unable to concentrate, and sleeps longer than usual", "Furthermore, she has episodes of extreme anxiety during these periods, leading her to take off time from work", "She notes that these symptoms are causing distress in her interpersonal relationships as well", "She has no past medical history and takes no medications", "She experienced menarche at age 12 and has regular periods", "Her temperature is 98.2°F (36.8°C), blood pressure is 100/70 mmHg, pulse is 75/min, and respirations are 12/min", "Physical exam reveals a nontender abdomen", "Pelvic exam reveals a closed cervix with no cervical motion or adnexal tenderness", "Her cardiopulmonary and neurological exams are unremarkable"], "s1": [0, 1, 2, 3, 4], "s2": [5, 6, 7, 8, 9, 10]} {"key": 57, "questions": "Which of the following is the most likely diagnosis?", "options": [{"label": "A", "disease": "Both bone forearm fracture"}, {"label": "B", "disease": "Distal radius fracture"}, {"label": "C", "disease": "Monteggia fracture-dislocation"}, {"label": "D", "disease": "Olecranon fracture"}, {"label": "E", "disease": "Supracondylar humerus fracture"}], "answer_idx": "E", "symptoms": ["A 6-year-old boy is brought to the emergency department by his father for elbow pain", "He was riding his bicycle when he fell onto his outstretched right hand", "He immediately clutched his right elbow in pain and began crying", "He refuses to move his elbow", "He has no medical problems and takes no medications", "His temperature is 98.4°F (36.9°C), blood pressure is 108/64 mmHg, pulse is 100/min, and respirations are 20/min", "On exam, the boy is holding his right wrist with his left hand and cries upon palpation of a region that is 2 cm proximal to his elbow", "The radial head is palpated just distal to the lateral epicondyle", "He is unable to make a pincer with his right thumb and index finger", "When asked to hold a pen between his right thumb and index fingers, he drops it", "The radial pulse is diminished on the right"], "s1": [4, 5, 10], "s2": [0, 1, 2, 3, 6, 7, 8, 9]} {"key": 58, "questions": "Which of the following is the most likely diagnosis?", "options": [{"label": "A", "disease": "Blood vessel narrowing"}, {"label": "B", "disease": "Cardiac dysrhythmia"}, {"label": "C", "disease": "Seizure"}, {"label": "D", "disease": "Stroke"}, {"label": "E", "disease": "Vagal event"}], "answer_idx": "A", "symptoms": ["A 67-year-old man presents with a recurrent episode of syncope", "He states that this has happened to him multiple times, particularly when he changes body position from sitting to standing, exerts himself, or turns his head quickly", "He says that he notes numbness, vertigo, and sometimes even trouble with speaking immediately prior to fainting", "The episodes usually last 1-5 minutes", "When he wakes up, he is mildly confused but typically returns to baseline within a minute", "The patient has a past medical history of diabetes, dyslipidemia, and hypertension", "His temperature is 98.5°F (36.9°C), blood pressure is 153/89 mmHg, pulse is 90/min, respirations are 11/min, and oxygen saturation is 97% on room air", "The patient's neurological exam is unremarkable", "However, when he is asked to stand up and start walking, he experiences the same numbness and tingling and feels like he may faint"], "s1": [0, 1, 2, 3, 4, 8], "s2": [5, 6, 7]} {"key": 59, "questions": "Which of the following is the most likely diagnosis in this patient?", "options": [{"label": "A", "disease": "Exacerbation of elbow arthritis"}, {"label": "B", "disease": "Triceps rupture"}, {"label": "C", "disease": "Olecranon bursitis"}, {"label": "D", "disease": "Lateral collateral ligament tear"}, {"label": "E", "disease": "Anconeus avulsion fracture"}], "answer_idx": "B", "symptoms": ["Figure A is the lateral radiograph of a 44-year-old male who sustained a fall on his outstretched hand while rollerblading", "The patient reports pain localized to the posterior elbow and refuses to attempt elbow extension secondary to pain and swelling"], "s1": [0], "s2": [1]} {"key": 60, "questions": "What is the most likely cause of this presentation?", "options": [{"label": "A", "disease": "Hyperkalemia"}, {"label": "B", "disease": "Iatrogenic hypoparathyroidism"}, {"label": "C", "disease": "Iatrogenic hypothyroidism"}, {"label": "D", "disease": "Multiple endocrine neoplasia"}, {"label": "E", "disease": "Somatization"}], "answer_idx": "B", "symptoms": ["A 46-year-old woman presents to her primary care doctor to request a referral to ophthalmology", "Her vision has been steadily declining over the past 2 years and she thinks she needs a prescription for glasses", "She further complains that her lips and feet feel numb", "Her medical history is notable for medullary thyroid cancer status post total thyroidectomy", "Her periods are regular", "She enjoys a diverse diet and takes levothyroxine", "Her blood pressure is 110/70 mmHg, pulse is 80/min, and respirations are 12/min", "She is alert and oriented", "Her pupils are equal, round, and reactive to light, but appear opacified", "Extraocular movements are intact, albeit slow", "Her visual acuity is decreased bilaterally", "The remainder of her physical exam is unremarkable", "Her basic metabolic panel is shown below: Na+: 139 mEq/L Cl-: 100 mEq/L K+: 4.9 mEq/L HCO3-: 25 mEq/L BUN: 10 mg/dL Glucose: 110 mg/dL Creatinine: 0.8 mg/dL Thyroid-stimulating hormone: 1.5 µU/mL Ca2+: 7.0 mEq/L Phosphorus: 6.5 mEq/L Albumin: 3.6 mg/dL The patient's ECG is shown in Figure A"], "s1": [0, 1, 8, 9, 10], "s2": [2, 3, 4, 5, 6, 7, 11, 12]} {"key": 61, "questions": "Which of the following is the most likely disorder that this patient is suffering from?", "options": [{"label": "A", "disease": "Borderline personality disorder"}, {"label": "B", "disease": "Intermittent explosive disorder"}, {"label": "C", "disease": "Narcissistic personality disorder"}, {"label": "D", "disease": "Paranoid personality disorder"}, {"label": "E", "disease": "Schizoid personality disorder"}], "answer_idx": "D", "symptoms": ["A 27-year-old man presents to his primary care physician for exposure to toxic materials", "The patient states that when he left for work this morning he was certain that he had closed the door to his pantry", "Upon returning home, he saw that the door to his pantry was wide open", "The patient is certain that his neighbors have been tampering with his food and potentially poisoned him", "He further states that he knows they have been trying to break into his house and steal his things", "He has tried multiple times to get them evicted from the building to no avail", "It is for this reason that he is certain that they are trying to get their revenge upon him", "His temperature is 98.6°F (37°C), blood pressure is 115/71 mmHg, pulse is 72/min, and respirations are 12/min", "The physician performs a physical exam and tells the patient that he thinks there is nothing to be concerned about, but that he should call him or come into the office if he experiences any symptoms", "The patient is outraged at this news and requests a competent doctor who is not colluding with his enemies", "He storms out of the office angrily, stating that he deserves the best in medical care"], "s1": [0, 7, 8], "s2": [1, 2, 3, 4, 5, 6, 9, 10]} {"key": 62, "questions": "Which of the following is the most likely diagnosis?", "options": [{"label": "A", "disease": "Brief psychotic disorder"}, {"label": "B", "disease": "Schizoid personality disorder"}, {"label": "C", "disease": "Schizophrenia"}, {"label": "D", "disease": "Schizophreniform disorder"}, {"label": "E", "disease": "Schizotypal personality disorder"}], "answer_idx": "E", "symptoms": ["A 25-year-old man presents to his primary care physician with a chief complaint of \"failing health", "\" He states that he typically can converse with animals via telepathy but is having trouble right now due to the weather", "He has begun taking an assortment of Peruvian herbs to little avail", "Otherwise, he is not currently taking any medications", "The patient lives alone and works in a health food store", "He states that his symptoms have persisted for the past 8 months", "When obtaining the patient's medical history, there are several times he attempts to telepathically connect with the animals in the vicinity", "His temperature is 98.6°F (37°C), blood pressure is 115/71 mmHg, pulse is 72/min, and respirations are 12/min", "On physical exam, you note a healthy young man who is dressed in an all-burlap ensemble"], "s1": [0, 1, 2, 6], "s2": [3, 4, 5, 7, 8]} {"key": 63, "questions": "Which of the following is the most likely underlying diagnosis?", "options": [{"label": "A", "disease": "Cholesterol embolism"}, {"label": "B", "disease": "Heart failure and reinfarction"}, {"label": "C", "disease": "Medication side effect"}, {"label": "D", "disease": "Pancreatitis"}, {"label": "E", "disease": "Renal failure"}], "answer_idx": "A", "symptoms": ["A 55-year-old man presents to the emergency department with chest pain and shortness of breath", "He has a medical history of hypertension, diabetes, and obesity", "His temperature is 98.6°F (37.0°C), blood pressure is 177/118 mmHg, pulse is 127/min, respirations are 11/min, and oxygen saturation is 98% on room air", "An ECG is performed and notable for ST elevation in leads II, III, and aVF", "The patient is treated appropriately and transferred to the medical floor", "On the 2nd day of his hospitalization, the patient has abdominal pain", "His serum lipase is 272 U/L and his creatinine is 1.6 mg/dL", "A physical exam is notable for the finding in Figure A", "He is requesting pain medication for his abdominal pain"], "s1": [0, 1, 2, 3, 4], "s2": [5, 6, 7, 8]} {"key": 64, "questions": "Which of the following is the most likely underlying diagnosis?", "options": [{"label": "A", "disease": "Cholesterol embolism"}, {"label": "B", "disease": "Heart failure and reinfarction"}, {"label": "C", "disease": "Medication side effect"}, {"label": "D", "disease": "Pancreatitis"}, {"label": "E", "disease": "Renal failure"}], "answer_idx": "A", "symptoms": ["A 55-year-old man presents to the emergency department with chest pain and shortness of breath", "He has a medical history of hypertension, diabetes, and obesity", "His temperature is 98.6°F (37.0°C), blood pressure is 177/118 mmHg, pulse is 127/min, respirations are 11/min, and oxygen saturation is 98% on room air", "An ECG is performed and notable for ST elevation in leads II, III, and aVF", "The patient is treated appropriately and transferred to the medical floor", "On the 2nd day of his hospitalization, the patient has abdominal pain", "His serum lipase is 272 U/L and his creatinine is 1.6 mg/dL", "A physical exam is notable for the finding in Figure A", "He is requesting pain medication for his abdominal pain"], "s1": [0, 1, 2, 3, 4], "s2": [5, 6, 7, 8]} {"key": 65, "questions": "Which of the following most appropriately describes the most likely diagnosis?", "options": [{"label": "A", "disease": "IgG in a fish-net pattern"}, {"label": "B", "disease": "Linear IgG along the basement membrane"}, {"label": "C", "disease": "Multi-nucleated giant cells"}, {"label": "D", "disease": "Panniculitis"}, {"label": "E", "disease": "Type IV hypersensitivity reaction"}], "answer_idx": "B", "symptoms": ["A 62-year-old man presents to the emergency department with skin lesions", "He felt itchy recently, then noticed skin lesions that broke out prompting him to come to the emergency department", "He recently went camping and hiking in the woods", "The patient drinks 1-2 beers per day and smokes 1 pack of cigarettes per day", "He is currently sexually active with multiple partners and does not use condoms", "The patient was discharged 1 week ago for pneumonia which was treated with azithromycin", "however, he feels that his symptoms may be returning", "His temperature is 99.5°F (37.5°C), blood pressure is 157/78 mmHg, pulse is 90/min, respirations are 15/min, and oxygen saturation is 98% on room air", "Physical exam reveals the findings in Figure A on the patient's arm", "The skin lesions are thick and do not break when pressure is applied"], "s1": [0, 1, 8, 9], "s2": [2, 3, 4, 5, 6, 7]} {"key": 66, "questions": "Which of the following is the most likely diagnosis?", "options": [{"label": "A", "disease": "Atopic dermatitis"}, {"label": "B", "disease": "Lichen planus"}, {"label": "C", "disease": "Prurigo nodularis"}, {"label": "D", "disease": "Psoriasis"}, {"label": "E", "disease": "Scabies"}], "answer_idx": "B", "symptoms": ["A 45-year-old man presents to his primary care physician with a rash", "He first noticed the rash 2 weeks prior and he endorses intermittent itchiness from the rash", "His medical history is significant for chronic hepatitis C infection, hypertension, type 2 diabetes mellitus, and heart failure", "The patient works as a butcher at a local meat processing shop and is sexually active with several partners", "He does not use barrier protection", "His temperature is 98.6°F (37.0°C), pulse is 90/min, blood pressure is 155/95 mmHg, and respirations are 12/min", "On physical exam, his skin has the finding shown in Figure A", "Fine, lace-like white lines are also noted on the patient’s buccal mucosa"], "s1": [0, 1, 6, 7], "s2": [2, 3, 4, 5]} {"key": 67, "questions": "Which of the following is the most likely diagnosis in this patient?", "options": [{"label": "A", "disease": "Adrenal tumor"}, {"label": "B", "disease": "Congenital adrenal hyperplasia"}, {"label": "C", "disease": "Ovarian hyperthecosis"}, {"label": "D", "disease": "Ovarian tumor"}, {"label": "E", "disease": "Polycystic ovary syndrome"}], "answer_idx": "D", "symptoms": ["A 27-year-old woman presents to her primary care physician with new hair growth on her face and lower abdomen over the last month", "She has started to develop pimples on her face and back over the last several months", "Her last menstrual period was over 3 months ago and her periods have been irregular over the last year", "She has been gaining weight recently", "The patient has a medical history of obesity and prediabetes with a hemoglobin A1c of 6.0% last year", "Her temperature is 98.5°F (36.9°C), pulse is 80/min, blood pressure is 139/88 mmHg, and respirations are 13/min", "Cardiopulmonary exam is unremarkable, and the patient’s abdomen appears slightly distended but exhibits no tenderness to palpation", "The patient’s face has coarse stubble along the jawline and on the upper lip, and there is similar hair along the midline of her lower abdomen", "A pelvic exam reveals mild clitoromegaly, a normal anteverted uterus, and a large left adnexal mass that is mildly tender", "Her laboratory test results are shown below:", "Hemoglobin: 13.9 g/dL", "Leukocyte count: 8,000 cells/mm^3", "Platelet count: 142,000/mm^3", "DHEAS: 73 ug/dL (Normal: 145-395 ug/dL)", "Testosterone: 256 ng/dL", "17-hydroxyprogesterone: 214 ng/dL (Normal: < 200 ng/dL)", "Ultrasound findings are shown in Figure A"], "s1": [0, 1, 2, 3, 7, 8, 13, 14, 15], "s2": [4, 5, 6, 9, 10, 11, 12, 16]} {"key": 68, "questions": "Which of the following is the most likely diagnosis?", "options": [{"label": "A", "disease": "Bacterial meningitis"}, {"label": "B", "disease": "Brain abscess"}, {"label": "C", "disease": "Cytomegalovirus encephalitis"}, {"label": "D", "disease": "Epilepsy"}, {"label": "E", "disease": "Fungal meningitis"}], "answer_idx": "B", "symptoms": ["A 35-year-old man presents to the emergency department with altered mental status", "He was found by his roommate to be disoriented this morning", "He had complained of a right-sided headache for the past 5 days", "He has a history of human immunodeficiency virus (HIV) infection being treated with bictegravir, emtricitabine, and tenofovir", "His last CD4 cell count was 500/mm^3. He does not smoke tobacco and drinks alcohol socially", "His temperature is 104.2°F (40.1°C), blood pressure is 110/70 mmHg, pulse is 110/min, and respirations are 22/min", "Examination reveals 3/5 strength to right shoulder abduction and elbow flexion/extension and 4/5 strength to right hip flexion/extension", "The remainder of the strength exam on the right and left sides is normal", "Neck flexion does not elicit pain, and there are no tongue lacerations", "During the exam, the patient’s eyes are seen to roll backward with repeated flexion/extension of his extremities and loss of urine", "This stops after 1 minute"], "s1": [0, 1, 2, 3, 4], "s2": [5, 6, 7, 8, 9, 10]} {"key": 69, "questions": "Which of the following is the most likely diagnosis?", "options": [{"label": "A", "disease": "Couvade syndrome"}, {"label": "B", "disease": "Delusion of pregnancy"}, {"label": "C", "disease": "Feigned pregnancy"}, {"label": "D", "disease": "Pseudocyesis"}, {"label": "E", "disease": "Recent miscarriage"}], "answer_idx": "D", "symptoms": ["A 27-year-old woman presents for her first prenatal visit at an estimated gestational age of 16 weeks and 4 days by last menstrual period", "She is presenting late to prenatal care due to significant anxiety about her pregnancy, as she and her husband had struggled with infertility for several years prior to conceiving", "She has had nausea and vomiting for about 2 months, tender and swollen breasts, and cravings for foods she typically does not eat", "She took a pregnancy test about 10 weeks ago and was too nervous to read the result immediately", "After 24 hours, she saw that it was positive", "The patient has a past medical history of bulimia nervosa but has not been symptomatic in 2 years", "She also had an appendectomy at age 15 for appendicitis", "She is a Ph", "D", "student and her husband is a physician", "Her temperature is 98.5°F (36.9°C), pulse is 75/min, blood pressure is 122/76 mmHg, and respirations are 13/min", "The patient’s affect is normal and she is pleasant and excited about her pregnancy", "A physical exam is notable for abdominal distension but is otherwise unremarkable", "Ultrasound is shown in Figure A, and urine human chorionic gonadotropin is negative in the office"], "s1": [0, 1, 2, 3, 4, 11, 12], "s2": [5, 6, 7, 8, 9, 10, 13]} {"key": 70, "questions": "Which of the following is the most likely diagnosis?", "options": [{"label": "A", "disease": "Disc herniation"}, {"label": "B", "disease": "Lumbar stenosis"}, {"label": "C", "disease": "Lumbar strain"}, {"label": "D", "disease": "Osteoarthritis"}, {"label": "E", "disease": "Vertebral compression fracture"}], "answer_idx": "C", "symptoms": ["A 42-year-old man presents to the urgent care clinic with low back pain", "He was working on a home improvement project the day prior to presentation when the pain started", "He describes the pain as \"achy and sore", "\" It is not positional and does not radiate", "He denies fevers, chills, paresthesias, and bowel or bladder incontinence", "He has a history of a distal radius fracture 2 years ago from falling off a ladder", "He drinks 3 alcoholic beverages weekly and denies illicit drug use", "The patient’s temperature is 98.4°F (36.9°C), blood pressure is 124/80 mmHg, pulse is 90/min, and respirations are 16/min", "His body mass index (BMI) is 22.4 kg/m^2. There is tenderness to palpation of his paravertebral lumbar region bilaterally", "Perineal and dermatomal sensation is symmetric and intact", "Strength is 5/5 to knee flexion/extension and ankle dorsiflexion/plantarflexion", "Patellar and Achilles reflexes are 2+ bilaterally", "Raising either leg while the patient is in the supine position does not elicit any pain"], "s1": [0, 1, 2, 3, 8], "s2": [4, 5, 6, 7, 9, 10, 11, 12]} {"key": 71, "questions": "Which of the following is the most likely diagnosis?", "options": [{"label": "A", "disease": "Acute lymphoblastic leukemia"}, {"label": "B", "disease": "Acute myelogenous leukemia"}, {"label": "C", "disease": "Chronic lymphocytic leukemia"}, {"label": "D", "disease": "Chronic myelogenous leukemia"}, {"label": "E", "disease": "Multiple myeloma"}], "answer_idx": "D", "symptoms": ["A 57-year-old man presents to the emergency department with several days of malaise, weakness, and night sweats", "Today, he experienced a headache with blurry vision, thus prompting his presentation", "The patient has a history of diabetes and is followed closely by an endocrinologist", "Otherwise, he has lost 10 pounds over the past month", "His temperature is 100°F (37.8°C), blood pressure is 122/90 mmHg, pulse is 84/min, respirations are 16/min, and oxygen saturation is 99% on room air", "Physical exam is notable for a thin man but is otherwise unremarkable", "His visual acuity is 20/100 in both eyes, though he says that he normally has 20/20 vision", "His gait is mildly ataxic as well, but his neurological exam is otherwise nonfocal", "Laboratory studies are ordered as seen below", "Hemoglobin: 10 g/dL", "Hematocrit: 31%", "Leukocyte count: 57,500/mm^3 with 35% blasts", "Platelet count: 109,000/mm^3", "Serum:", "Na+: 139 mEq/L", "Cl-: 100 mEq/L", "K+: 4.1 Eq/L", "HCO3-: 22 mEq/L", "BUN: 20 mg/dL", "Glucose: 99 mg/dL", "Creatinine: 1.1 mg/dL", "Calcium: 10.0 mEq/L"], "s1": [0, 1, 3, 4, 5, 6, 7], "s2": [2, 8, 9, 10, 11, 12, 13, 14, 15, 16, 17, 18, 19, 20, 21]} {"key": 72, "questions": "Which of the following is the most likely diagnosis?", "options": [{"label": "A", "disease": "Adhesive capsulitis"}, {"label": "B", "disease": "Biceps tendinopathy"}, {"label": "C", "disease": "Glenohumeral osteoarthritis"}, {"label": "D", "disease": "Rotator cuff impingement"}, {"label": "E", "disease": "Subacromial bursitis"}], "answer_idx": "A", "symptoms": ["A 65-year-old man presents to his primary care physician for stiffness in his arm", "He has been having trouble combing his hair and reaching objects that are high on the shelf", "The patient has a medical history of type 2 diabetes mellitus, obesity, and hypertension", "His current medications include metformin, insulin, lisinopril, and hydrochlorothiazide", "The patient leads a sedentary life in which he tends to stay home and watch television", "He does not engage in any physical or strenuous activity", "His temperature is 98.6°F (37°C), blood pressure is 138/85 mmHg, pulse is 75/min, and respirations are 12/min", "On physical exam, the patient has decreased passive and active range of motion of his shoulder", "The strength of the patient's upper extremity is 4/5 limited by pain"], "s1": [2, 3, 4, 5, 6], "s2": [0, 1, 7, 8]} {"key": 73, "questions": "Which of the following is the most likely the diagnosis?", "options": [{"label": "A", "disease": "Ectopic implantation of a blastocyst"}, {"label": "B", "disease": "Inflammation of the pancreas"}, {"label": "C", "disease": "Obstruction of blood flow through the hepatic vein"}, {"label": "D", "disease": "Obstruction of the common bile duct by radio-opaque stones"}, {"label": "E", "disease": "Vascular ectasia within the liver"}], "answer_idx": "E", "symptoms": ["A 23-year-old woman presents to the emergency department with severe abdominal pain", "The pain has been dull and progressive, but became suddenly worse while she was exercising", "Her medical history is notable for depression, anxiety, and gonococcal urethritis that was appropriately treated", "She is sexually active and does not use condoms", "She admits to drinking at least 5 standard alcoholic drinks a day", "The patient also recently lost a large amount of weight for a fitness show she planned on entering", "The patient's current medications include oral contraceptive pills, fluoxetine, alprazolam, ibuprofen, acetaminophen, and folate", "Her temperature is 99.5°F (37.5°C), blood pressure is 80/40 mmHg, pulse is 110/minute, and respirations are 15/minute with an oxygen saturation of 96% on room air", "On physical exam, you note an athletic young woman with burly shoulders, a thick neck, and acne on her forehead and back", "On abdominal exam you note diffuse tenderness with 10/10 pain upon palpation of the right upper quadrant", "Intravenous fluids are started and labs are sent", "A urinary ß-hCG has been ordered"], "s1": [0, 1, 7, 9, 10, 11], "s2": [2, 3, 4, 5, 6, 8]} {"key": 74, "questions": "Which of the following is the most likely cause of this patient's symptoms?", "options": [{"label": "A", "disease": "Antigen exposure"}, {"label": "B", "disease": "Drug reaction"}, {"label": "C", "disease": "Infection"}, {"label": "D", "disease": "IV drug use"}, {"label": "E", "disease": "Photosensitivity"}], "answer_idx": "A", "symptoms": ["A 27-year-old woman presents to her primary care physician for an abnormal vaginal discharge", "The patient has engaged in unprotected intercourse with 12 different partners", "She has a medical history of asthma, IV drug abuse, and depression", "She has 1 to 2 alcoholic drinks per day", "She feels ashamed of her behavior and is requesting treatment for her condition and advice for safe sex", "The patient is given antibiotics and is advised on safe sex practices", "The patient presents to the emergency department 3 days later with a complaint of a rash", "She states that every time she has sex a rash has emerged on her skin", "Her temperature is 99.5°F (37.5°C), blood pressure is 127/68 mmHg, pulse is 91/min, respirations are 14/min, and oxygen saturation is 98% on room air", "A physical exam is notable for the finding in Figure A"], "s1": [0, 1, 4, 5], "s2": [2, 3, 6, 7, 8, 9]} {"key": 75, "questions": "Which of the following is the most likely cause of this patient’s symptoms?", "options": [{"label": "A", "disease": "Defective metabolism of methionine"}, {"label": "B", "disease": "Extra copy of sex chromosome"}, {"label": "C", "disease": "Mutation of chromosome 15"}, {"label": "D", "disease": "Mutation of COL5A1 or COL5A2"}, {"label": "E", "disease": "Mutation of RET proto-oncogene"}], "answer_idx": "C", "symptoms": ["A 16-year-old boy presents to an ophthalmologist with blurry vision", "Over the past several months, he has had increasing difficulty seeing the board from the back of the classroom at school", "The patient is otherwise doing well in school and enjoys playing basketball", "His medical history is otherwise significant for scoliosis which is managed by an orthopedic surgeon", "His family history is significant for a mother with type 2 diabetes mellitus and a father who underwent aortic valve replacement last year", "His temperature is 98.6°F (37°C), blood pressure is 115/71 mmHg, pulse is 72/min, and respirations are 12/min", "On physical exam, the patient is tall for his age and has long arms", "He has 20 degrees of thoracic scoliosis, which is stable from previous exams", "On slit-lamp examination, the patient is found to have bilateral upward lens subluxation and is prescribed corrective lenses"], "s1": [0, 1, 2, 5, 8], "s2": [3, 4, 6, 7]} {"key": 76, "questions": "Which of the following is most strongly associated with this patient's condition?", "options": [{"label": "A", "disease": "Cardiac rhabdomyoma"}, {"label": "B", "disease": "Glaucoma"}, {"label": "C", "disease": "Optic glioma"}, {"label": "D", "disease": "Polyostotic fibrous dysplasia"}, {"label": "E", "disease": "Renal cell carcinoma"}], "answer_idx": "A", "symptoms": ["A 1-year-old girl is brought to a neurologist due to increasing seizure frequency over the past 2 months", "She recently underwent a neurology evaluation which revealed hypsarrhythmia on electroencephalography (EEG) with a mix of slow waves, multifocal spikes, and asynchrony", "Her parents have noticed the patient occasionally stiffens and spreads her arms at home", "She was born at 38-weeks gestational age without complications", "She has no other medical problems", "Her medications consist of lamotrigine and valproic acid", "Her temperature is 98.3°F (36.8°C), blood pressure is 90/75 mmHg, pulse is 94/min, and respirations are 22/min", "Physical exam reveals innumerable hypopigmented macules on the skin and an irregularly shaped, thickened, and elevated plaque on the lower back"], "s1": [0, 1, 2, 5], "s2": [3, 4, 6, 7]} {"key": 77, "questions": "Which of the following is the most likely diagnosis?", "options": [{"label": "A", "disease": "Herniated disc"}, {"label": "B", "disease": "Lumbosacral strain"}, {"label": "C", "disease": "Multiple myeloma"}, {"label": "D", "disease": "Spondylolisthesis"}, {"label": "E", "disease": "Vertebral compression fracture"}], "answer_idx": "B", "symptoms": ["A 60-year-old woman presents to the emergency department with back pain after gardening", "Her pain is 7/10 in severity, non-radiating, and not relieved by rest", "She has never experienced this pain in the past and denies fever, night sweats, unintentional weight loss, and bowel or bladder incontinence", "She has hypertension for which she takes hydrochlorothiazide and had a recent asthma flare requiring a prednisone taper", "She does not drink alcohol or smoke", "Her temperature is 98.6°F (37.0°C), blood pressure is 120/80 mmHg, pulse is 90/min, and respirations are 18/min", "Physical exam reveals an uncomfortable middle-aged woman in no acute distress", "There is no tenderness to palpation of the spinous processes", "Flexion of the hip with the knee extended while the patient is supine does not elicit any pain, nor does forced dorsiflexion of the foot at terminal hip extension", "She has 5/5 strength to hip flexion, extension, abduction, and adduction", "knee flexion and extension", "and ankle dorsiflexion and plantarflexion bilaterally", "Bilateral patellar and Achilles reflexes are 2+", "Serum laboratory results are as follows: Hemoglobin: 12.0 g/dL Creatinine: 1.1 mg/dL Ca2+: 10.6 mg/dL"], "s1": [0, 1, 2, 3, 4], "s2": [5, 6, 7, 8, 9, 10, 11, 12, 13]} {"key": 78, "questions": "Which of the following is the most likely cause of this patient's symptoms?", "options": [{"label": "A", "disease": "Cardiac tamponade"}, {"label": "B", "disease": "Cirrhosis"}, {"label": "C", "disease": "Constrictive pericarditis"}, {"label": "D", "disease": "Pulmonary arterial hypertension"}, {"label": "E", "disease": "Restrictive cardiomyopathy"}], "answer_idx": "C", "symptoms": ["A 62-year-old man arrives at the emergency room complaining of chest pain and difficulty breathing", "He reports that the dyspnea started 2 months ago after he had the flu", "At first, the difficulty breathing occurred whenever he went up and down 1 flight of stairs, but the dyspnea progressively worsened since then", "This morning, he developed chest pain and difficulty breathing while sitting at the kitchen table", "The patient’s medical history is significant for hypertension, type 2 diabetes mellitus, and rheumatoid arthritis", "His medications include aspirin, lisinopril, metformin, and sulfasalazine", "His temperature is 97°F (36.1°C), blood pressure is 130/78 mmHg, pulse is 88/min, and respirations are 14/min with an oxygen saturation of 97% O2 on room air", "On physical exam, jugular venous distension during both inspiration and expiration, mild abdominal distention, and 2+ bilateral lower extremity edema is noted", "A chest radiograph is obtained and the result is shown in Figure A"], "s1": [0, 3, 1, 2, 7], "s2": [4, 5, 6, 8]} {"key": 79, "questions": "Which of the following is the most likely explanation for this patient's presentation?", "options": [{"label": "A", "disease": "Dermatomyositis"}, {"label": "B", "disease": "Hypokalemia"}, {"label": "C", "disease": "Hypothyroidism"}, {"label": "D", "disease": "Inclusion body myositis"}, {"label": "E", "disease": "Polymyositis"}], "answer_idx": "D", "symptoms": ["A 57-year-old woman presents to her primary care physician with weakness for the past 6 months with symptoms that have worsened recently", "She feels fatigued, depressed, and has gained 10 pounds which she attributes to feeling too weak to go to the gym", "She was recently treated for an ear infection with an antibiotic which she completed 3 weeks ago, and experienced a rash which she believes was an allergic reaction to her treatment", "The patient has a medical history of diabetes mellitus that is well-controlled with insulin and metformin", "She has been admitted multiple times for hypoglycemia secondary to missing meals", "Her temperature is 99.5°F (37.5°C), blood pressure is 127/68 mmHg, pulse is 80/min, respirations are 17/min, and oxygen saturation is 98% on room air", "Physical exam is notable for 2/5 strength in the patient's upper and lower extremities, sparse fine hair on her body, and a rash on her face and arms", "Laboratory values are ordered as seen below", "Serum: Na+: 139 mEq/L K+: 3.3 mEq/L Cl-: 100 mEq/L HCO3-: 25 mEq/L BUN: 20 mg/dL Glucose: 70 mg/dL Creatinine: 1.1 mg/dL Ca2+: 10.2 mg/dL AST: 12 U/L ALT: 10 U/L A muscle biopsy is obtained as seen in Figure A"], "s1": [0, 1, 3, 4, 6], "s2": [2, 5, 7, 8]} {"key": 80, "questions": "Which of the following is the most likely diagnosis?", "options": [{"label": "A", "disease": "Cecal volvulus"}, {"label": "B", "disease": "Ileus"}, {"label": "C", "disease": "Ogilvie syndrome"}, {"label": "D", "disease": "Sigmoid volvulus"}, {"label": "E", "disease": "Small bowel obstruction"}], "answer_idx": "A", "symptoms": ["A 24-year-old man presents to the emergency department with abdominal pain, nausea, and vomiting", "The symptoms began towards the end of running a marathon and persisted after finishing the race", "The patient is otherwise healthy and has no other medical problems", "His temperature is 98.0°F (36.7°C), blood pressure is 105/71 mmHg, pulse is 113/min, respirations are 19/min, and oxygen saturation is 98% on room air", "Physical exam reveals a distended and tender abdomen", "A radiograph is performed as seen in Figure A"], "s1": [0, 1, 2, 4], "s2": [3, 5]} {"key": 81, "questions": "Which of the following is the most likely diagnosis?", "options": [{"label": "A", "disease": "Actinic keratosis"}, {"label": "B", "disease": "Basal cell carcinoma"}, {"label": "C", "disease": "Melanoma"}, {"label": "D", "disease": "Poor wound healing secondary to diabetes"}, {"label": "E", "disease": "Squamous cell carcinoma"}], "answer_idx": "E", "symptoms": ["A 69-year-old man presents to his primary care physician for a general checkup", "The patient works as a farmer and gardens in his spare time", "He has a medical history of poorly managed type 2 diabetes mellitus and irritable bowel syndrome", "His current medications include lisinopril, metformin, insulin, fiber supplements, and a multivitamin", "On physical exam, there is a calm, obese gentleman", "Inspection of the patient's scalp reveals the finding shown in Figure A", "The patient is sent home and instructed to keep a blood glucose journal", "Three months later, the patient returns with his glucose journal", "In addition, he complains of a cut on his scalp that is not seeming to heal", "The patient's glucose journal reveals an average blood glucose of 175 mg/dL", "His temperature is 98.8°F (37.1°C), blood pressure is 134/86 mmHg, pulse is 80/min, and respirations are 13/min", "On inspection of the patient's scalp, the finding in Figure B is noted"], "s1": [0, 1, 2, 3, 4, 6, 7, 9, 10], "s2": [5, 8, 11]} {"key": 82, "questions": "Which of the following is the most likely diagnosis?", "options": [{"label": "A", "disease": "Alzheimer disease"}, {"label": "B", "disease": "Creutzfeldt-Jacob disease"}, {"label": "C", "disease": "Dementia with Lewy bodies"}, {"label": "D", "disease": "Frontotemporal dementia"}, {"label": "E", "disease": "Subcortical leukoencephalopathy"}], "answer_idx": "B", "symptoms": ["A 57-year-old man is brought to a neurologist by his wife for abnormal behavior", "He was normal until around 3 months ago when he started forgetting important meetings and misplacing items around the house", "One month ago, the patient was fired from his job as a software engineer after attempting to grope a coworker", "The patient expresses frustration that he is no longer able to concentrate enough to enjoy his hobby of solving puzzles", "His medical history is significant for gastroesophageal reflux disease for which he takes omeprazole", "He has no family history of neurologic disease", "His temperature is 98.6°F (37.0°C), blood pressure is 120/75 mmHg, pulse is 70/min, and respirations are 16/min", "During the examination, the patient’s cell phone rings unexpectedly causing the patient to suddenly jerk his arms and legs", "Further diagnostic investigation is performed which reveals an elevated 14-3-3 protein on cerebrospinal fluid analysis"], "s1": [0, 1, 2, 3, 8], "s2": [4, 5, 6, 7]} {"key": 83, "questions": "Which of the following is the most likely etiology of this patient’s presentation?", "options": [{"label": "A", "disease": "Congenital enzyme deficiency"}, {"label": "B", "disease": "Hypertrophy of sphincter muscle"}, {"label": "C", "disease": "Non-IgE-mediated immunologic reaction"}, {"label": "D", "disease": "Passage of gastric contents into esophagus"}, {"label": "E", "disease": "Telescoping of the intestine"}], "answer_idx": "C", "symptoms": ["A 6-week-old girl presents with her parents to the pediatrician for blood-streaked diapers", "The patient is breastfed every 2-3 hours and voids 10-12 times daily", "The patient previously had several soft stools per day that ranged in color from mustard yellow to dark green", "Over the past 2 days, the patient’s stools have become looser and streaked with blood", "The patient has also regurgitated several of her feedings in the last few days", "Neither the vomit nor bloody stools have seemed to bother the patient", "The patient’s weight continues to trend along the 50th percentile", "The patient’s temperature is 98.4°F (36.9°C), blood pressure is 66/51 mmHg, pulse is 127/min, and respirations are 26/min", "On physical exam, the patient appears well nourished and is cooing", "Her abdomen is soft and non-tender", "Physical exam reveals the finding seen in Figure A"], "s1": [0, 3, 4, 5, 10], "s2": [1, 2, 6, 7, 8, 9]} {"key": 84, "questions": "Which of the following is the most likely diagnosis?", "options": [{"label": "A", "disease": "Cerebellar stroke"}, {"label": "B", "disease": "Essential tremor"}, {"label": "C", "disease": "Huntington disease"}, {"label": "D", "disease": "Lacunar stroke"}, {"label": "E", "disease": "Parkinson disease"}], "answer_idx": "E", "symptoms": ["A 68-year-old man presents for evaluation by a neurologist for an involuntary hand tremor that started approximately 3 months ago", "The shaking improves when he reaches for objects but returns when he is not using his hand", "His temperature is 98.6°F (37.0°C), blood pressure is 115/70 mmHg, pulse is 70/min, and respirations are 12/min", "His left hand exhibits a 4-6 Hz tremor when resting on his lap", "The tremor extinguishes with voluntary movement", "In addition, his left upper extremity has increased tone with passive range of motion"], "s1": [2], "s2": [0, 1, 3, 4, 5]} {"key": 85, "questions": "Which of the following is the most likely diagnosis?", "options": [{"label": "A", "disease": "Tetralogy of Fallot"}, {"label": "B", "disease": "Total anomalous pulmonary venous return"}, {"label": "C", "disease": "Transposition of the great vessels"}, {"label": "D", "disease": "Tricuspid valve atresia"}, {"label": "E", "disease": "Truncus arteriosus"}], "answer_idx": "C", "symptoms": ["A newborn boy is assessed in the newborn nursery due to central cyanosis that does not respond to oxygen therapy", "The patient was born to a G1P1 mother at 39 weeks gestation", "The mother had diabetes that was managed with exercise and insulin during the pregnancy", "The mother has never been vaccinated and did not follow routine prenatal care", "The newborn's temperature is 99.5°F (37.5°C), blood pressure is 70/30 mmHg, pulse is 160/min, respirations are 27/min, and oxygen saturation is 80% on room air", "The initial workup of the patient includes a chest radiograph seen in Figure A as well as lab values as seen below", "Hemoglobin: 14 g/dL Hematocrit: 42% Leukocyte count: 6,500/mm^3 with normal differential Platelet count: 197,000/mm^3 Serum: Na+: 139 mEq/L Cl-: 101 mEq/L K+: 4.3 mEq/L HCO3-: 24 mEq/L BUN: 12 mg/dL Glucose: 99 mg/dL Creatinine: 0.5 mg/dL Ca2+: 10.0 mg/dL AST: 12 U/L ALT: 10 U/L An electrocardiogram is obtained as seen in Figure B"], "s1": [0, 4, 5, 6], "s2": [1, 2, 3]} {"key": 86, "questions": "Which of the following is the most likely underlying etiology of this patient's symptoms?", "options": [{"label": "A", "disease": "Adhesions"}, {"label": "B", "disease": "Enteric nervous system damage"}, {"label": "C", "disease": "Impacted stool"}, {"label": "D", "disease": "Norovirus"}, {"label": "E", "disease": "Twisting of the bowel"}], "answer_idx": "A", "symptoms": ["A 55-year-old man presents to the emergency department with nausea and vomiting for the past week", "He decided to come in when his symptoms worsened", "He feels that his symptoms are exacerbated with large fatty meals and when he drinks alcohol", "His wife recently returned from a cruise with symptoms of vomiting and diarrhea", "The patient has a medical history of poorly managed diabetes, constipation, anxiety, dyslipidemia, and hypertension", "His temperature is 99.5°F (37.5°C), blood pressure is 197/128 mmHg, pulse is 100/min, respirations are 17/min, and oxygen saturation is 95% on room air", "Physical exam reveals a systolic murmur heard loudest along the left upper sternal border", "Abdominal exam reveals an obese, tympanitic and distended abdomen with a 3 cm scar in the right lower quadrant", "Vascular exam reveals weak pulses in the lower extremities"], "s1": [0, 1, 2, 3, 7], "s2": [4, 5, 6, 8]} {"key": 87, "questions": "Which of the following is the most likely diagnosis?", "options": [{"label": "A", "disease": "Overflow incontinence"}, {"label": "B", "disease": "Pelvic organ prolapse"}, {"label": "C", "disease": "Stress urinary incontinence"}, {"label": "D", "disease": "Urethral diverticulum"}, {"label": "E", "disease": "Vesicovaginal fistula"}], "answer_idx": "D", "symptoms": ["A 52-year-old G3P3 woman presents to clinic with a 2-year history of urinary incontinence", "She has had frequent, involuntary loss of urine over the last 2 years but presented today because of 10 days of dysuria", "She has been treated for 2 urinary tract infections over the past 6 months", "She does not endorse any loss of urine with coughing or laughter", "She has no chronic medical illnesses and no surgical history", "Her temperature is 99.5°F (37.5°C), blood pressure is 120/80 mmHg, pulse is 92/min, and respirations are 15/min", "Her BMI is 30 kg/m^2. On pelvic examination, the vagina is dry and atrophic with a 2-cm tender, palpable anterior vaginal mass", "The mass does not change with Valsava maneuver", "Her postvoid residual volume is 60 mL"], "s1": [0, 1, 2, 3, 4], "s2": [5, 6, 7, 8]} {"key": 88, "questions": "Which of the following is the most likely diagnosis?", "options": [{"label": "A", "disease": "Bleeding and infected vessel of Kiesselbach plexus"}, {"label": "B", "disease": "Foreign body obstruction"}, {"label": "C", "disease": "Nasopharyngeal carcinoma"}, {"label": "D", "disease": "Septal perforation"}, {"label": "E", "disease": "Sinusitis with bacterial superinfection"}], "answer_idx": "B", "symptoms": ["A 7-year-old boy is brought to the emergency department by his parents for worsening symptoms of previously diagnosed acute sinusitis", "Initially, the pediatrician prescribed decongestants and sent the patient home", "Since then, the patient has developed a nasal discharge with worsening pain", "The patient has a medical history of asthma which is well-controlled with albuterol", "His temperature is 99.5°F (37.5°C), blood pressure is 90/48 mmHg, pulse is 124/min, respirations are 17/min, and oxygen saturation is 98% on room air", "The cardiopulmonary exam is within normal limits", "Inspection of the patient's nose reveals a unilateral purulent discharge mixed with blood", "The rest of the patient's exam is within normal limits"], "s1": [0, 1, 2, 6], "s2": [3, 4, 5, 7]} {"key": 89, "questions": "Which of the following is the most likely diagnosis?", "options": [{"label": "A", "disease": "Chalazion"}, {"label": "B", "disease": "Foreign body"}, {"label": "C", "disease": "Hordeolum"}, {"label": "D", "disease": "Ingrown eyelash follicle"}, {"label": "E", "disease": "Meibomian cell carcinoma"}], "answer_idx": "A", "symptoms": ["A 37-year-old machinist presents to his primary care physician with eye problems", "He has felt a mass in his eye that has persisted for the past month", "The patient has a medical history of blepharitis treated with eye cleansing and squamous cell carcinoma of the skin treated with Mohs surgery", "His temperature is 99.5°F (37.5°C), blood pressure is 157/102 mmHg, pulse is 90/min, respirations are 17/min, and oxygen saturation is 98% on room air", "Physical exam is notable for a firm and rubbery nodule palpable inside the patient's left eyelid", "Physical exam does not elicit any pain"], "s1": [0, 1, 4, 5], "s2": [2, 3]} {"key": 90, "questions": "Which of the following is the most likely etiology of this patient's current symptoms?", "options": [{"label": "A", "disease": "Atypical pneumonia"}, {"label": "B", "disease": "Chronic obstructive pulmonary disease"}, {"label": "C", "disease": "Community acquired pneumonia"}, {"label": "D", "disease": "Medication administration"}, {"label": "E", "disease": "Poor cardiac function"}], "answer_idx": "D", "symptoms": ["A 55-year-old man with a history of polysubstance abuse is found down in his home acutely obtunded", "Emergency medical services administered naloxone which immediately woke the patient up and improved his oxygen saturation from 40% to 90%", "However, they noted subsequently that the patient demonstrated increased work of breathing and had crackles and wheezing on pulmonary exam", "The patient arrives alert and oriented claiming he feels short of breath", "He denies any chest pain or other symptoms aside from shortness of breath", "He has a history of cocaine-induced cardiomyopathy and myocardial infarction from cocaine use 2 years ago", "The patient has a 40 pack-year smoking history", "His temperature is 96.0°F (35.6°C), blood pressure is 120/68 mmHg, pulse is 102/min, respirations are 26/min, and oxygen saturation is 88% on room air", "Physical exam reveals increased work of breathing, pulmonary crackles, and wheezing", "A chest radiograph is performed as seen in Figure A"], "s1": [0, 1, 2, 3, 4, 8, 9], "s2": [5, 6, 7]} {"key": 91, "questions": "Which of the following is the most likely diagnosis for the patient?", "options": [{"label": "A", "disease": "Lymphogranuloma venereum"}, {"label": "B", "disease": "Herpes"}, {"label": "C", "disease": "Chancroid"}, {"label": "D", "disease": "Syphilis"}], "answer_idx": "A", "symptoms": ["A 22-year-old man presented with complaints of painful lesions on his penis and swelling in the left groin that started 10 days ago", "He denied fever, chills, night sweats, rashes, dysuria, discharge, testicular pain, or proctitis", "His female partner had been diagnosed with chlamydia one year earlier but he has not undergone evaluation", "Multiple small, nontender scabbed lesions are identified in the bilateral scrotal area and on the shaft of penis", "The right inguinal lymph node was tender and swollen."], "s1": [0, 3, 4], "s2": [1, 2]} {"key": 92, "questions": "Which of the following is the most likely diagnosis for the patient?", "options": [{"label": "A", "disease": "Acrodermatitis continua of Hallopeau"}, {"label": "B", "disease": "Herpetic whitlow"}, {"label": "C", "disease": "Paronychia"}, {"label": "D", "disease": "Acute contact dermatitis"}], "answer_idx": "D", "symptoms": ["A 51-year-old white man presents to the clinic with a 2-year history of a painful, recurrent rash on his fingers", "He initially noticed redness, irritation, and pustules around his nails that he thought were caused by an infection", "However, the rash progressed to affect the nail bed, eventually destroying his nails", "On examination, well-demarcated, scaly plaques with associated pustules and nail atrophy are present on the right second and third distal digits", "He is otherwise healthy."], "s1": [0, 1, 2, 3], "s2": [4]} {"key": 93, "questions": "Which of the following is the most likely diagnosis for the patient?", "options": [{"label": "A", "disease": "Varicella"}, {"label": "B", "disease": "Gianotti-Crosti syndrome"}, {"label": "C", "disease": "Langerhans cell histiocytosis"}, {"label": "D", "disease": "Pityriasis lichenoides et varioliformis acuta"}], "answer_idx": "D", "symptoms": ["An 8-year-old male child is brought to the clinic for the third time in 4 months presenting with red-brown papules with an overlying mica-like scale on the ventral side of his arms and legs", "His medical and family histories are otherwise unremarkable", "His mother states that when the macules and papules go away, the skin where the lesions were previously located is typically lighter than before."], "s1": [0, 2], "s2": [1]} {"key": 94, "questions": "Which of the following is the most likely diagnosis for the patient?", "options": [{"label": "A", "disease": "Perioral dermatitis"}, {"label": "B", "disease": "Acne vulgaris"}, {"label": "C", "disease": "Allergic contact dermatitis"}, {"label": "D", "disease": "Facial demodicosis"}], "answer_idx": "A", "symptoms": ["A 24-year-old white female graduate student presents with erythematous, papulopustular patches of skin in a muzzle-like distribution surrounding the mouth, chin, and glabellar region", "The lesions are tender to palpation, associated with a stinging and burning sensation, and are aggravated by exfoliating facial washes", "Two-year treatment history includes a variety of topical antibiotics, azelaic acid cream, retinoid agents, benzoyl peroxide preparations, topical corticosteroids, and oral tetracycline for cyclic recurrences of outbreaks", "These treatment methods were previously successful with complete resolution of each outbreak; however, at present the lesions are persisting despite treatment."], "s1": [0], "s2": [1, 2, 3]} {"key": 95, "questions": "Which of the following is the most likely diagnosis for the patient?", "options": [{"label": "A", "disease": "Halo nevus"}, {"label": "B", "disease": "Melanoma"}, {"label": "C", "disease": "Vitiligo"}, {"label": "D", "disease": "Dysplastic nevus"}], "answer_idx": "A", "symptoms": ["A 20-year-old woman presents to the clinic with a circular hypopigmented lesion on her right cheek", "The patient stated that she used to have a mole in the same location", "Over time she noticed a white area around the mole that enlarged to the current size of the lesion", "After a few months she noticed the mole in the center of the lesion had disappeared", "On further questioning, she denies any personal or family history of skin cancer."], "s1": [0, 4], "s2": [1, 2, 3]} {"key": 96, "questions": "Which of the following is the most likely diagnosis for the patient?", "options": [{"label": "A", "disease": "Lichen sclerosis"}, {"label": "B", "disease": "Corticosteroid-induced atrophy"}, {"label": "C", "disease": "Morphea"}, {"label": "D", "disease": "Pityriasis rosea"}], "answer_idx": "B", "symptoms": ["A 29-year-old woman with a history of acne presents to the clinic with 3 discrete, atrophic, blue-tinged plaques on her face", "The plaques were located on her forehead and cheeks bilaterally", "The patient states the lesions had been present for several months and had slowly worsened until they stabilized at their current appearance", "The patient reported that the lesions occurred after she went to a physician for “acne treatment.” The patient has no other medical conditions or relevant social history."], "s1": [0, 3], "s2": [1, 2]} {"key": 97, "questions": "Which of the following is the most likely diagnosis for the patient?", "options": [{"label": "A", "disease": "Tinea corporis"}, {"label": "B", "disease": "Mycosis fungoides"}, {"label": "C", "disease": "Nummular Eczema"}, {"label": "D", "disease": "Psoriasis"}], "answer_idx": "C", "symptoms": ["A 22-year-old woman complains of an itchy rash on her leg for 4 weeks", "She is a collegiate swimmer and had a prior ringworm infection during her freshman year", "She has been using over-the-counter antifungal cream for the past 3 weeks with no relief", "Multiple round, erythematous, scaling lesions are present on her right leg, some of which are vesicular and weeping", "Potassium hydroxide (KOH) test is negative for a dermatophytic infection."], "s1": [0, 3], "s2": [1, 2, 4]} {"key": 98, "questions": "Which of the following is the most likely diagnosis for the patient?", "options": [{"label": "A", "disease": "Subungual hematoma"}, {"label": "B", "disease": "Melanonychia"}, {"label": "C", "disease": "Subungual melanoma"}, {"label": "D", "disease": "Nummular eczema"}], "answer_idx": "B", "symptoms": ["A 62-year-old African American man presents to establish care and to undergo skin examination, which he reports receiving regularly since his sister was diagnosed with melanoma", "Examination reveals 3-mm hyperpigmented horizontal bands across the nailbeds of all fingernails, which he reports have been present since his 20s", "He denies experiencing any other nail changes or pain", "He takes no medications, and he has no history of skin cancer or suspicious moles", "He has no relevant medical history."], "s1": [0, 3, 4], "s2": [1, 2]} {"key": 99, "questions": "Which of the following is the most likely diagnosis for the patient?", "options": [{"label": "A", "disease": "Pemphigus vegetans"}, {"label": "B", "disease": "Acrodermatitis continua supperativa"}, {"label": "C", "disease": "Proderma gangraneosum"}, {"label": "D", "disease": "Cutaneous horn"}], "answer_idx": "D", "symptoms": ["A 60-year-old man presents to the clinic with a 1-cm long horn-like projection coming from the skin on the superior aspect of the skull just lateral to midline", "The structure is yellowish-white in color and feels very firm to the touch", "He states that he noticed a small, hard lesion a few months ago and that it has grown progressively larger from the time of initial presentation to now", "The patient denies any pain at rest but states that he experiences pain when the lesion is struck accidentally."], "s1": [0, 2], "s2": [1, 3]} {"key": 100, "questions": "Which of the following is the most likely diagnosis for the patient?", "options": [{"label": "A", "disease": "Proderma gangraneosum"}, {"label": "B", "disease": "Pemphigus vegetans"}, {"label": "C", "disease": "Bowen’s disease"}, {"label": "D", "disease": "Squamous cell carcinoma"}], "answer_idx": "B", "symptoms": ["A 43-year-old African American woman presents to the clinic with irritative lesions in the folds of her abdomen and in her genital region as well as painful ulcers in her oral mucosa", "The patient reports noticing some flaccid blisters in addition to the lesions she presents with", "On examination, erythematous, vegetative lesions are found", "She states that the affected areas are painful but not pruritic", "She has a previous history of herpes simplex virus (HSV)."], "s1": [0, 3, 4], "s2": [1, 2]} {"key": 101, "questions": "Which of the following is the most likely diagnosis for the patient?", "options": [{"label": "A", "disease": "Bullosis diabeticorum"}, {"label": "B", "disease": "Subepidermal bullae"}, {"label": "C", "disease": "Bullous leukocytoclastic vasculitis"}, {"label": "D", "disease": "Pressure bullae (coma bullae)"}], "answer_idx": "D", "symptoms": ["A 20-year-old male with a medical history of diabetes mellitus type 1 presents with ecchymosis and bullous lesions on his back, posterior legs, and buttocks", "He was playing football when he ran into another player and became unconscious", "He was immobile in the hospital for 3 days before he regained consciousness", "He has about 7 tense blisters with areas of surrounding erythema on the back, buttocks, and posterior thighs."], "s1": [0, 3], "s2": [1, 2]} {"key": 102, "questions": "Which of the following is the most likely diagnosis for the patient?", "options": [{"label": "A", "disease": "X-linked ichthyosis"}, {"label": "B", "disease": "Ichthyosis vulgaris"}, {"label": "C", "disease": "Lamellar ichthyosis"}, {"label": "D", "disease": "Epidermolytic hyperkeratosis"}], "answer_idx": "B", "symptoms": ["A 6-year-old Caucasian boy is brought to the clinic by his parents for dry, scaly skin that has persisted for the past 2 years despite the use of commercial moisturizers", "His scaly skin is especially notable over his lower extremities, back, and elbows", "The patient denies any pain over the affected areas and reports only a mild itch that is soothed with cold water or lotions", "The parents report that he had a similar condition as an infant throughout his entire body and that he has siblings who also have persistently dry skin", "His parents have tried exfoliating the scales with moderate success, but the scales recur shortly thereafter."], "s1": [0, 1, 2], "s2": [3, 4]} {"key": 103, "questions": "Which of the following is the most likely diagnosis for the patient?", "options": [{"label": "A", "disease": "Pityriasis rosea"}, {"label": "B", "disease": "Secondary syphilis"}, {"label": "C", "disease": "Guttate psoriasis"}, {"label": "D", "disease": "Small plaque parapsoriasis"}], "answer_idx": "A", "symptoms": ["A 7-year-old healthy boy is brought to the dermatology clinic by his parents after having had a 3-week history of a rash", "The patient’s rash was slightly pruritic, and it involved predominantly the trunk, starting as one larger pink patch with fine scale and subsequently spreading with smaller similar macules and patches", "The boy’s palms and soles were spared of the rash."], "s1": [0], "s2": [1, 2]} {"key": 104, "questions": "Which of the following is the most likely diagnosis for the patient?", "options": [{"label": "A", "disease": "Rosacea"}, {"label": "B", "disease": "Acne vulgaris"}, {"label": "C", "disease": "Acanthosis nigricans"}, {"label": "D", "disease": "Alopecia"}], "answer_idx": "B", "symptoms": ["A 24-year-old Caucasian woman presented with intermittent, inflammatory papulopustules and nodules on the face, neck, chest, and back", "The lesions would appear 1 week premenstrually and resolve 1 week postmenstrually with normal menstrual cycles", "She has a history of adolescent acne that is only responsive to isotretinoin 6 years ago", "Examination revealed scarring and postinflammatory hyperpigmentation diffusely on her face."], "s1": [0, 2], "s2": [1, 3]} {"key": 105, "questions": "Which of the following is the most likely diagnosis for the patient?", "options": [{"label": "A", "disease": "Severe recessive dystrophic EB"}, {"label": "B", "disease": "Junctional EB"}, {"label": "C", "disease": "Sweet syndrome"}, {"label": "D", "disease": "EBS-Dowling Meara"}], "answer_idx": "D", "symptoms": ["A 15-month-old boy presents with a history of skin blistering since birth, primarily on his hands, feet, and the perioral face", "Blisters and erosions also involve the trunk, some in an annular configuration", "There is no family history of neonatal blistering or skin fragility", "Over time, the patient has had a decreased number of new vesicles and bullae associated with trauma and friction but has developed thicker skin on the soles of the feet and dystrophic nails."], "s1": [0, 1], "s2": [2, 3]} {"key": 106, "questions": "Which of the following is the most likely diagnosis for the patient?", "options": [{"label": "A", "disease": "Bacterial sepsis"}, {"label": "B", "disease": "Sweet syndrome"}, {"label": "C", "disease": "Cellulitis"}, {"label": "D", "disease": "Erysipelas"}], "answer_idx": "B", "symptoms": ["A 71-year-old Caucasian man with myelodysplastic syndrome and on chemotherapy with azacitidine presented to the hospital with 2 days of high fever and 4 days of worsening skin lesions", "The lesions appeared on the elbows as “blood blisters,” which grew larger and more painful", "He denied any other new medications", "He was placed on broad spectrum antibiotics by infectious disease physicians out of concern for ecthyma gangrenosum, and dermatology was consulted", "On examination, violaceous, edematous plaques with a central pseudovesicle were seen on the bilateral elbows", "Punch biopsies were performed for tissue culture and histology."], "s1": [0, 1, 4], "s2": [2, 3, 5]} {"key": 107, "questions": "Which of the following is the most likely diagnosis for the patient?", "options": [{"label": "A", "disease": "Keratolysis exfoliativa"}, {"label": "B", "disease": "Dyshidrotic eczema"}, {"label": "C", "disease": "Palmar plantar pustulosis"}, {"label": "D", "disease": "Tinea"}], "answer_idx": "D", "symptoms": ["A 37-year-old man without a significant past medical history presents to the dermatology clinic with an itchy rash on his right hand that he has had for about 4 months", "He had tried numerous over-the-counter hand creams and hydrocortisone cream with no improvement", "Additional questioning revealed a 1-year-history of dry, itchy feet", "As a construction worker, he wears occlusive, steel-toed work boots and often wears gloves", "He is also right-handed", "Examination of his hands reveals a normal left hand and a scaly right palm", "Diffuse, fine, dry white scale was present on the plantar aspect of both feet."], "s1": [0, 1, 5], "s2": [2, 3, 4, 6]} {"key": 108, "questions": "Which of the following is the most likely diagnosis for the patient?", "options": [{"label": "A", "disease": "Scleroderma (systemic sclerosis)"}, {"label": "B", "disease": "Morphea"}, {"label": "C", "disease": "Lichen sclerosus"}, {"label": "D", "disease": "Lipodermatosclerosis"}], "answer_idx": "B", "symptoms": ["The patient is a 46-year-old woman who presents at your primary care office for an examination with a single lesion that appears on her left arm", "During the examination, it is revealed that the patient has a large sclerotic, indurated, hyperpigmented, anhidrotic, and hairless plaque on the right dorsal forearm", "The patient states that these lesions had previously been slightly pink for several months before they had turned brown in color and had become “hard.” The patient has type 2 diabetes, but she has no other medical problems", "In addition, she has no relevant social or family history."], "s1": [0, 1, 2], "s2": [3]} {"key": 109, "questions": "Which of the following is the most likely diagnosis for the patient?", "options": [{"label": "A", "disease": "Cutis laxa"}, {"label": "B", "disease": "Mid-dermal elastolysis"}, {"label": "C", "disease": "Post-inflammatory elastolysis"}, {"label": "D", "disease": "Anetoderma associated with SLE"}], "answer_idx": "D", "symptoms": ["A 38-year-old African American woman was referred to dermatology because of spots on her torso and arms", "The lesions were asymptomatic, but their development, cause, and cosmesis were of concern", "No benefit was seen with the use of OTC remedies such as hydroxycortisone 1% cream or vitamin E oil", "Her medical history was significant for systemic lupus erythematosus, well controlled on hydroxychloroquine", "Physical examination revealed numerous, well-circumscribed, 0.5- to 2-cm, hypopigmented, round-to-oval macules of wrinkled skin on her anterior chest wall, back, abdomen, and bilateral upper arms", "Invagination of the lesions was noted with palpation", "A punch biopsy was performed."], "s1": [0, 1, 4, 5], "s2": [2, 3, 6]} {"key": 110, "questions": "Which of the following is the most likely diagnosis for the patient?", "options": [{"label": "A", "disease": "Type I minocycline hyperpigmentation"}, {"label": "B", "disease": "Type II minocycline hyperpigmentation"}, {"label": "C", "disease": "Immunobullous disease"}, {"label": "D", "disease": "Type III minocycline hyperpigmentation"}], "answer_idx": "A", "symptoms": ["The patient is a 21-year-old male who presents for continued acne therapy", "He has had severe acne on his face, chest, and back for years", "He is happy with his current acne regimen of topical tretinoin, topical clindamycin, and oral minocycline", "He has been on the regimen for years", "On examination, you notice significant blue-black discoloration confined to sites of previous acne scars", "The patient has no other medical problems and no relevant social or family history."], "s1": [0, 1, 2, 3], "s2": [4, 5]} {"key": 111, "questions": "Which of the following is the most likely diagnosis for the patient?", "options": [{"label": "A", "disease": "Myiasis"}, {"label": "B", "disease": "Pyoderma"}, {"label": "C", "disease": "Foreign body granuloma"}, {"label": "D", "disease": "Cutaneous leishmaniasis"}], "answer_idx": "D", "symptoms": ["A 53-year-old woman presents with a rash on her left lower extremity that has been present for several weeks", "The patient is a native of Brazil and immigrated to the United States in the past month", "She states that the rash is painless and does not itch", "On examination, the patient has multiple, raised, hyperpigmented papules on the left lower extremity", "There is a large, atrophic, hypopigmented patch on her lower shin, at the site of a previous large ulcer", "The superior lesion is the newest, and is beginning to ulcerate", "She has no other medical problems."], "s1": [1, 6], "s2": [0, 2, 3, 4, 5]} {"key": 112, "questions": "Which of the following is the most likely diagnosis for the patient?", "options": [{"label": "A", "disease": "Eczema"}, {"label": "B", "disease": "Scabies"}, {"label": "C", "disease": "Allergic contact dermatitis"}, {"label": "D", "disease": "Localized pruritus"}], "answer_idx": "B", "symptoms": ["A 6-month-old infant presents with 2 weeks duration of erythematous papules on the soles of both feet and in the toe web spaces", "On examination, a single burrow is visible on the left sole", "Multiple family members also complain of a pruritic rash that has been bothering them for several months", "The family members have tried multiple over-the-counter creams and remedies without success", "The patient has no other medical problems or relevant family history."], "s1": [0, 1, 2], "s2": [3, 4]} {"key": 113, "questions": "Which of the following is the most likely diagnosis for the patient?", "options": [{"label": "A", "disease": "Zosteriform herpes simplex"}, {"label": "B", "disease": "Contact dermatitis"}, {"label": "C", "disease": "Herpes zoster"}, {"label": "D", "disease": "Scabies"}], "answer_idx": "C", "symptoms": ["A 42-year-old man presents with a four-day history of experiencing headache, malaise, and stabbing right-sided headache", "Two days after his initial symptoms appeared, he developed a rash over the area of pain", "He reports that he went hiking through the Texas hill country prior to becoming ill", "The patient is otherwise in good health and has an unremarkable medical history", "Physical examination reveals unilateral erythematous, thin, raised plaques over the right forehead", "In addition, he has no relevant social or family history."], "s1": [0, 1, 4], "s2": [2, 3, 5]} {"key": 114, "questions": "Which of the following is the most likely diagnosis for the patient?", "options": [{"label": "A", "disease": "Lichen planus"}, {"label": "B", "disease": "Trachyonychia"}, {"label": "C", "disease": "Psoriasis"}, {"label": "D", "disease": "Brittle nails"}], "answer_idx": "B", "symptoms": ["The patient is a 12-year-old Hispanic girl who presents with a 6-month history of nail dystrophy involving all of her fingernails", "On examination, all 10 of her fingernails exhibit longitudinal ridging, pitting, fragility, thinning, and distal notching", "The patient’s mother is very concerned about the cosmesis of her daughter’s nails", "The patient has no systemic symptoms", "On review of systems, the patient’s mother noted that her daughter has started to develop circular patches of hair loss that appear to resolve on their own", "The patient has no relevant social or family history and does not take any medications."], "s1": [0, 1, 2], "s2": [3, 4, 5]} {"key": 115, "questions": "Which of the following is the most likely diagnosis for the patient?", "options": [{"label": "A", "disease": "Juvenile xanthogranuloma"}, {"label": "B", "disease": "Epidermal nevus"}, {"label": "C", "disease": "Syringocystadenoma papilliferum."}, {"label": "D", "disease": "Nevus sebaceous"}], "answer_idx": "D", "symptoms": ["A 1-year-old boy presents with a single, L-shaped, yellow-orange hairless patch over the left preauricular skin", "The patient’s mother notes that he has never grown hair in that location", "The mother is concerned because the area used to be smooth, but seems to be changing", "The patient has a history of prematurity and low birthweight but is otherwise healthy", "The patient has no relevant social or family history."], "s1": [0, 1, 2], "s2": [3, 4]} {"key": 116, "questions": "Which of the following is the most likely diagnosis for the patient?", "options": [{"label": "A", "disease": "Bullous erythema multiforme"}, {"label": "B", "disease": "Stevens-Johnson syndrome"}, {"label": "C", "disease": "Bullous impetigo"}, {"label": "D", "disease": "Autoimmune bullous dermatoses"}], "answer_idx": "C", "symptoms": ["An 8-year-old African-American boy presents with a 3-day history of fever, pain, and a rash on his trunk", "On examination, the patient has several fragile, bullous lesions on his trunk, many of which have eroded, exposing erythematous, annular lesions", "The patient is febrile and shaking in the office, and he appears to be in pain", "He has no other relevant social or family history and does not take any medications", "His mother is very concerned about her son."], "s1": [0, 1, 2], "s2": [3, 4]} {"key": 117, "questions": "Which of the following is the most likely diagnosis for the patient?", "options": [{"label": "A", "disease": "Raynaud disease"}, {"label": "B", "disease": "Cryoglobulinemia,"}, {"label": "C", "disease": "Perniosis"}, {"label": "D", "disease": "Chronic myelomonocytic leukemia"}], "answer_idx": "C", "symptoms": ["A 13-year-old boy presents to the dermatology clinic concerned about the development of red, itchy, and sometimes painful bumps on his fingers", "He first noted seeing the lesions 2 months prior, in early December", "Although the initial lesions have since resolved, new lesions appeared after a skiing trip the previous weekend", "The patient is otherwise healthy, with no significant past medical history, and he does not take any medications", "Physical examination reveals erythematous, tender papules on the lateral and dorsal aspects of the distal fingers."], "s1": [0, 4], "s2": [1, 2, 3]} {"key": 118, "questions": "Which of the following is the most likely diagnosis for the patient?", "options": [{"label": "A", "disease": "Prurigo nodularis"}, {"label": "B", "disease": "Lichen amyloidosis"}, {"label": "C", "disease": "Pretibial myxedema"}, {"label": "D", "disease": "Hypertrophic lichen planus"}], "answer_idx": "B", "symptoms": ["A 52-year-old Hispanic woman presents with a 3-year history of a very pruritic rash over her shins bilaterally", "On examination, both shins are covered in hyperpigmented papules that have coalesced to form thin, rippled plaques", "The patient is very concerned about the appearance of her legs and the intense pruritus in the areas of the rash", "The patient has no other medical problems and has no systemic symptoms", "She has no relevant social or family history and does not take any medications."], "s1": [0, 1, 2], "s2": [3, 4]} {"key": 119, "questions": "Which of the following is the most likely diagnosis for the patient?", "options": [{"label": "A", "disease": "Acral nevi"}, {"label": "B", "disease": "Lentigines"}, {"label": "C", "disease": "Acral melanoma"}, {"label": "D", "disease": "Tinea nigra"}], "answer_idx": "D", "symptoms": ["The patient is a 13-year-old adolescent girl who presents with a 1-month history of a dark lesion on her finger", "She states that the lesion does not bother her, but she came to the clinician because her mother was very concerned about the enlarging area of hyperpigmentation", "The patient has no past medical history, no relevant family or social history, and is otherwise in good health", "On examination, the rash appears to be a 1 x 1 cm circular patch of hyperpigmentation with overlying scale on the middle finger", "During the past month, the rash has grown in size but remains asymptomatic."], "s1": [0, 2], "s2": [1, 3, 4]} {"key": 120, "questions": "Which of the following is the most likely diagnosis for the patient?", "options": [{"label": "A", "disease": "Grover disease"}, {"label": "B", "disease": "Scabies"}, {"label": "C", "disease": "Bullous pemphigoid"}, {"label": "D", "disease": "Pemphigus vulgaris"}], "answer_idx": "A", "symptoms": ["A 72-year-old man presents to the clinic with an intensely itchy rash on his abdomen and chest that appeared 2 months prior", "He denies starting any new medications prior to the onset of the eruption and any systemic symptoms", "Although he traveled and stayed in hotels recently, he reports that his wife does not exhibit any similar symptoms", "Physical examination reveals numerous discrete papules, crusted papules, and papulovesicles on the chest, abdomen, and mid-back", "His upper and lower extremities, face, and groin are unaffected."], "s1": [0, 3], "s2": [1, 2, 4]} {"key": 121, "questions": "Which of the following is the most likely diagnosis for the patient?", "options": [{"label": "A", "disease": "Macular amyloidosis"}, {"label": "B", "disease": "Lichen amyloidosis"}, {"label": "C", "disease": "Prurigo nodularis"}, {"label": "D", "disease": "Paresthetica"}], "answer_idx": "A", "symptoms": ["The patient is a 38-year-old Hispanic woman who presents with a 1-year history of a very pruritic rash over her interscapular area", "On examination, she has a circular area of slightly hyperpigmented macules in the interscapular region", "The patient complains of pruritus but no pain in the area of the rash", "The patient has no other medical problems and has no systemic symptoms", "She has no relevant social or family history and does not take any medications."], "s1": [0, 1, 2], "s2": [3, 4]} {"key": 122, "questions": "Which of the following is the most likely diagnosis for the patient?", "options": [{"label": "A", "disease": "Hailey-Hailey disease"}, {"label": "B", "disease": "Grover disease"}, {"label": "C", "disease": "Darier disease"}, {"label": "D", "disease": "Epidermodysplasia verruciformis"}], "answer_idx": "C", "symptoms": ["The patient is a 19-year-old woman who presents with several years of a worsening rash", "The rash appears as erythematous, hyperkeratotic thin plaques in a seborrheic distribution", "The patient states that the rash is very irritating and sometimes pruritic", "The patient’s main complaint is that the rash exudes a malodor", "The patient states that her father has a similar rash that he chooses not to treat."], "s1": [0, 4], "s2": [1, 2, 3]} {"key": 123, "questions": "Which of the following is the most likely diagnosis for the patient?", "options": [{"label": "A", "disease": "Intertrigo"}, {"label": "B", "disease": "Darier disease"}, {"label": "C", "disease": "Pemphigus vegetans"}, {"label": "D", "disease": "Hailey-Hailey disease"}], "answer_idx": "D", "symptoms": ["A 35-year-old woman is referred for evaluation and management of a rash involving both axillae", "She reports recurring episodes of the same rash beginning in her early 20s", "Lesions develop and involute spontaneously", "Areas affected included the axillae, neck, and groin", "Historically, outbreaks are more common and worse during the summer, and after vigorous exercise", "Both her father and brother have the same problem; her brother saw a specialist, but forgot what he was told", "Otherwise, she is healthy and taking no medications", "Examination reveals moist, red, crusted, and malodorous plaques with fissures in both axillae."], "s1": [0, 1, 3, 4, 7], "s2": [2, 5, 6]} {"key": 124, "questions": "Which of the following is the most likely diagnosis for the patient?", "options": [{"label": "A", "disease": "Staphylococcal infection"}, {"label": "B", "disease": "Herpes"}, {"label": "C", "disease": "Epidermolysis bullosa"}, {"label": "D", "disease": "Bullous impetigo"}], "answer_idx": "C", "symptoms": ["On day-of-life 2, a 3-day-old boy who was born with no visible skin problems begins to develop erosions and bullae over both the palmar and plantar surfaces", "These bullae are localized to areas of friction or trauma (eg, where electrocardiogram leads were placed)", "The mother’s pregnancy was normal, and the infant had been born without complications", "Aside from the development of bullae, the infant has no evidence of any other medical problems", "The infant’s parents are healthy and have no medical problems", "No one in the infant’s family has had similar dermatologic findings."], "s1": [0, 1], "s2": [2, 3, 4, 5]} {"key": 125, "questions": "Which of the following is the most likely diagnosis for the patient?", "options": [{"label": "A", "disease": "Dysplastic nevi"}, {"label": "B", "disease": "Basal cell carcinoma"}, {"label": "C", "disease": "Dermatofibroma"}, {"label": "D", "disease": "Keloids"}], "answer_idx": "C", "symptoms": ["A 44-year-old woman presents for her annual full skin examination", "She has a specific concern regarding a lesion on her left lower leg", "She recalls having a mosquito bite in that area a few years earlier, but it never went away", "The lesion is bothersome only when she shaves her legs and nicks it; otherwise, there is no associated pain or tenderness", "Physical examination reveals a firm, flat, pink and brown nodule located on the left anterior proximal shin", "The lesion dimples inward when pressure is applied."], "s1": [0], "s2": [1, 2, 3, 4, 5]} {"key": 126, "questions": "Which of the following is the most likely diagnosis for the patient?", "options": [{"label": "A", "disease": "Squamous cell carcinoma in situ"}, {"label": "B", "disease": "Extramammary Paget disease"}, {"label": "C", "disease": "Langerhans cell histiocytosis"}, {"label": "D", "disease": "Amelanotic melanoma"}], "answer_idx": "B", "symptoms": ["A 79-year-old white man with an extensive history of nonmelanoma skin cancer presents for a routine skin examination", "During his visit, he asks to have a groin rash checked", "For 1 to 2 years, he has been using over-the-counter and prescription antifungal and antibiotic creams for a presumed “jock itch.” Recently, he has developed more discomfort and reports episodic, profuse bleeding from the area", "Examination reveals a large erythematous, eroded plaque on the posterior right scrotum extending anteriorly to the right upper thigh", "A nodule is present within the plaque", "The left scrotum is minimally involved, and there is no adenopathy."], "s1": [0], "s2": [1, 2, 3, 4, 5]} {"key": 127, "questions": "Which of the following is the most likely diagnosis for the patient?", "options": [{"label": "A", "disease": "Lichen sclerosus et atrophicus"}, {"label": "B", "disease": "Lichen nitidus"}, {"label": "C", "disease": "Tinea versicolor"}, {"label": "D", "disease": "Vitiligo"}], "answer_idx": "A", "symptoms": ["A 61-year-old woman presents concerned about a white discoloration on her left breast that she first noticed 6 months earlier", "It itches at times, and she is worried it might be cancerous", "She showed it to her gynecologist at her annual visit, and he referred her to the dermatology clinic", "Physical examination of the left breast reveals a 10-cm white, smooth, porcelain-like plaque", "There is no associated erythema and no noticeable scale."], "s1": [0, 1, 2], "s2": [3, 4]} {"key": 128, "questions": "Which of the following is the most likely diagnosis for the patient?", "options": [{"label": "A", "disease": "Scleredema"}, {"label": "B", "disease": "Scleroderma"}, {"label": "C", "disease": "Scleromyxedema"}, {"label": "D", "disease": "Nephrogenic systemic fibrosis"}], "answer_idx": "C", "symptoms": ["A 56-year-old man presents with diffuse pruritic papules involving the upper body, with nose and forehead thickening", "Over the past 2 years, he has been evaluated by multiple specialists for dysphagia, reflux, chronic kidney disease, hypertension, and dyspnea on exertion", "Examination results are significant for generalized firm, fleshy, dome-shaped papules of several millimeters in diameter, on his forehead, ears, neck, trunk, back, proximal upper extremities, and dorsal hands, with forehead, glabella, and nose induration", "Relevant laboratory findings include monoclonal gammopathy on serum immunofixation, consisting of immunoglobulin (Ig) G_ and IgA_, and unremarkable thyroid panel findings"], "s1": [0, 2], "s2": [1, 3]} {"key": 129, "questions": "Which of the following is the most likely diagnosis for the patient?", "options": [{"label": "A", "disease": "Bullous impetigo"}, {"label": "B", "disease": "Toxicodendron dermatitis"}, {"label": "C", "disease": "Shingles"}, {"label": "D", "disease": "Phytophotodermatitis"}], "answer_idx": "B", "symptoms": ["A 34-year-old man presents with an itchy rash, primarily on his arms, but also, to a lesser extent, on his face", "He denies any fever or malaise, and his medical history is unremarkable", "He does not take any medications and notes that he spent the previous weekend doing yardwork on his property, and the rash appeared 4 days later", "Physical examination reveals several linear arrangements of papulovesicles on the dorsal and ventral aspects of his forearms with surrounding erythema."], "s1": [0, 3], "s2": [1, 2]} {"key": 130, "questions": "Which of the following is the most likely diagnosis for the patient?", "options": [{"label": "A", "disease": "Median rhomboid glossitis"}, {"label": "B", "disease": "Lichen planus"}, {"label": "C", "disease": "Secondary syphilis"}, {"label": "D", "disease": "Geographic tongue"}], "answer_idx": "A", "symptoms": ["A 62-year-old white man with a past medical history of latent tuberculosis, vitiligo, and psoriasis presents for follow-up for treatment of psoriasis", "The patient’s psoriasis has proven refractory to numerous treatments over the years, so he began treatment with secukinumab, a monoclonal antibody against interleukin-17A", "After 2 months of therapy, the patient reports new-onset tongue pain", "Physical examination demonstrates an erythematous, edematous, diamond-shaped plaque on the posterior dorsum of the patient’s tongue."], "s1": [0, 1], "s2": [2, 3]} {"key": 131, "questions": "Which of the following is the most likely diagnosis for the patient?", "options": [{"label": "A", "disease": "Impetigo"}, {"label": "B", "disease": "Eczema herpeticum"}, {"label": "C", "disease": "Varicella zoster virus infection"}, {"label": "D", "disease": "Animal pox infection"}], "answer_idx": "B", "symptoms": ["A 13-month-old toddler with spreading eczema on his trunk and arms is brought in by his mother", "She is applying moisturizer and a cortisone cream", "However, 2 days earlier, a wound appeared on his left hand that appears to be infected", "Since then, more areas of raw, oozing skin have developed", "Examination reveals lichenified scaling plaques scattered on the trunk and posterior knees and arms, and his left arm has monomorphic crusted vesicles and ulcerations on an erythematous base."], "s1": [0, 1, 4], "s2": [2, 3]} {"key": 132, "questions": "Which of the following is the most likely diagnosis for the patient?", "options": [{"label": "A", "disease": "Schwannomas"}, {"label": "B", "disease": "Synovial sarcoma"}, {"label": "C", "disease": "Plexiform neurofibroma"}, {"label": "D", "disease": "Eczema herpeticum"}], "answer_idx": "C", "symptoms": ["A 30-year-old healthy Hispanic woman presents with a slowly progressive facial deformity present since early childhood", "She denies any new lesions during this time, any history of trauma to the area, and any family history of similar skin lesions", "Examination reveals a bulging, deformed dermal mass with a “bag-of-worms” appearance involving the upper right quadrant of her face, with hypertrophy of the overlying skin and soft tissues", "Neurofibromas and café-au-lait macules are not observed."], "s1": [0, 1], "s2": [2, 3]} {"key": 133, "questions": "Which of the following is the most likely diagnosis for the patient?", "options": [{"label": "A", "disease": "Cystic pleomorphic adenomas"}, {"label": "B", "disease": "Warthin tumors"}, {"label": "C", "disease": "Mucous retention cysts"}, {"label": "D", "disease": "Hidrocystomas"}], "answer_idx": "D", "symptoms": ["A healthy 45-year-old woman presents with a several-month history of a small papule at the corner of her right eye", "The lesion is asymptomatic and is not enlarging, but she has cosmetic concerns", "She denies diplopia, vision loss, or any other lesions on her face and body", "Examination reveals a dome-shaped, smooth, translucent papule filled with watery fluid located below the lateral canthus of her right eye."], "s1": [0, 1], "s2": [2, 3]} {"key": 134, "questions": "Which of the following is the most likely diagnosis for the patient?", "options": [{"label": "A", "disease": "Chilblains"}, {"label": "B", "disease": "Cutaneous lupus erythematosus"}, {"label": "C", "disease": "Equestrian perniosis"}, {"label": "D", "disease": "Cutaneous sarcoidosis"}], "answer_idx": "C", "symptoms": ["A 32-year-old white woman who has never smoked presents with pruritic, tender, erythematous, violaceous patches with eczema-like blisters on her upper, lateral thighs", "In 2012, she began traveling annually from Pennsylvania to South Carolina, from January through April, where she works with and rides horses daily from morning to evening", "Her symptoms began in her first year in South Carolina and appear consistently from January through March", "Her lesions appear initially in mid-January as a red dot and progress by mid-February", "They resolve spontaneously in March."], "s1": [0], "s2": [1, 2, 3, 4]} {"key": 135, "questions": "Which of the following is the most likely diagnosis for the patient?", "options": [{"label": "A", "disease": "Acquired digital fibrokeratoma (ADFK)"}, {"label": "B", "disease": "Pyogenic granuloma"}, {"label": "C", "disease": "Necrobiosis lipoidica"}, {"label": "D", "disease": "Cutaneous horn"}], "answer_idx": "A", "symptoms": ["A healthy 30-year-old man presents with a 3-year history of an asymptomatic, fleshy papule at the base of his great toe", "He denies any new lesions during this time, a history of trauma to the area, or a family history of similar skin lesions", "He was treated previously by another dermatologist with cryotherapy; however, he returns to the clinic with recurrence of the lesion", "Examination reveals a pedunculated, flesh-colored, well-demarcated tumor on the plantar surface of the base of his great toe."], "s1": [0, 3], "s2": [1, 2]} {"key": 136, "questions": "Which of the following is the most likely diagnosis for the patient?", "options": [{"label": "A", "disease": "Necrobiotic xanthogranuloma (NXG)"}, {"label": "B", "disease": "Acquired digital fibrokeratoma (ADFK)"}, {"label": "C", "disease": "Diabetic dermopathy"}, {"label": "D", "disease": "Necrobiosis lipoidica"}], "answer_idx": "D", "symptoms": ["A 45-year-old woman with insulin-dependent diabetes mellitus presents with a sore on her right leg", "She describes a long-standing history of asymptomatic lesions on her shins but had only recently injured her leg, causing the sore, which is slow to heal", "Despite a 15-year history of diabetes, she has no systemic complications", "She has large, well-circumscribed plaques on her anterior shins bilaterally, which are yellow-brown with telangiectasias and minimally violaceous borders."], "s1": [0, 1], "s2": [2, 3]} {"key": 137, "questions": "Which of the following is the most likely diagnosis for the patient?", "options": [{"label": "A", "disease": "Seborrheic dermatitis"}, {"label": "B", "disease": "Cutaneous Langerhans cell histiocytosis"}, {"label": "C", "disease": "Prurigo nodularis"}, {"label": "D", "disease": "Atopic dermatitis"}], "answer_idx": "B", "symptoms": ["A 60-year-old man presents with rapidly growing lesions on his face, trunk, axillae, flexural areas, and groin", "The lesions appeared 3 months earlier in his perianal area and extended rapidly to other areas", "Previously, he was treated with topical agents for atopic dermatitis and seborrheic dermatitis without improvement", "Examination reveals disseminated violaceous papules and plaques on the face, trunk, axillae, and perianal area; some lesions were confluent and ulcerated", "Yellowish-red to skin-colored papules are present on the scalp", "He has no palpable lymphadenopathy or organomegaly."], "s1": [0, 1, 2, 3], "s2": [4, 5]} {"key": 138, "questions": "Which of the following is the most likely diagnosis for the patient?", "options": [{"label": "A", "disease": "neuroma"}, {"label": "B", "disease": "Angiolipoma"}, {"label": "C", "disease": "Glomus tumor"}, {"label": "D", "disease": "Spiradenoma"}], "answer_idx": "D", "symptoms": ["A 13-year-old girl presents with a cystic lesion that has been on her forearm for several years", "Concerned about the lesion, she reports that it may have grown slightly larger in the past year and it is intermittently painful", "More recently, a second similar lesion has appeared in close proximity", "She denies any other suspicious lesions elsewhere on her skin", "Her family history is negative for skin cancer, and she reports no other medical conditions", "Physical examination reveals two well-defined 6-mm nodules that are rubbery, slightly tender, and pink on the right proximal ventral forearm", "The overlying skin is intact, and there is no associated adenopathy."], "s1": [0, 1, 2, 5], "s2": [3, 4, 6]} {"key": 139, "questions": "Which of the following is the most likely diagnosis for the patient?", "options": [{"label": "A", "disease": "Adult-onset xanthogranuloma"}, {"label": "B", "disease": "Necrobiotic xanthogranuloma"}, {"label": "C", "disease": "Erdheim-Chester disease"}, {"label": "D", "disease": "Adult-onset asthma with periocular xanthogranuloma"}], "answer_idx": "B", "symptoms": ["A 62-year-old man with well-controlled rosacea, a monoclonal gammopathy of undetermined significance, and chronic lymphocytic leukemia (CLL; stable on no current therapy) was seen in the clinic for a full-body skin check", "Results of his examination were unremarkable except for the presence of periorbital, yellow-brown plaques with violaceous borders involving the upper and lower eyelids, bilaterally", "There was induration, but no warmth or tenderness, of the periorbital plaques", "He first noticed these lesions around the time he was diagnosed with CLL; initially they were more discrete yellow papules that developed into plaques over time", "They are completely asymptomatic."], "s1": [0, 3], "s2": [1, 2, 4]} {"key": 140, "questions": "Which of the following is the most likely diagnosis for the patient?", "options": [{"label": "A", "disease": "Noonan Syndrome"}, {"label": "B", "disease": "McCune-Albright Syndrome"}, {"label": "C", "disease": "DNA Repair Syndromes"}, {"label": "D", "disease": "Neurofibromatosis type 1"}], "answer_idx": "D", "symptoms": ["A healthy 16-year-old girl presents with six tan patches on her trunk and upper arms, freckles in her axillae, and a reddish discoloration under the tip of her thumbnail", "The tan patches have been present since birth but have enlarged over the past 4 years", "She reports paroxysmal pain and cold sensitivity involving the discolored portion of her thumb", "Examination reveals six tan-brown macules with smooth borders ranging from 16 to 20 mm, axillary freckling, and a tender red-blue nodule under the thumbnail."], "s1": [0, 1], "s2": [2, 3]} {"key": 141, "questions": "Which of the following is the most likely diagnosis for the patient?", "options": [{"label": "A", "disease": "Nevus comedonicus"}, {"label": "B", "disease": "Trichofolliculoma"}, {"label": "C", "disease": "Familial dyskeratotic comedones"}, {"label": "D", "disease": "Segmental acne"}], "answer_idx": "A", "symptoms": ["A healthy 15-year-old girl presents with a solitary, asymptomatic lesion on her back", "The lesion has been present since birth but has enlarged over the past 2 years", "She denies association with previous trauma and irritation", "Examination reveals multiple, comedo-like openings filled with dark lamellate keratinocyte plugs grouped in a linear distribution over a slightly hypotrophic area of 3 cm by 7 cm", "Dermoscopy revealed numerous circular and barrel-shaped homogenous areas in dark-brown shades with remarkable keratin plugs."], "s1": [0, 1, 2], "s2": [3, 4]} {"key": 142, "questions": "Which of the following is the most likely diagnosis for the patient?", "options": [{"label": "A", "disease": "Amelanotic melanoma"}, {"label": "B", "disease": "Kaposi sarcoma"}, {"label": "C", "disease": "Pyogenic granuloma"}, {"label": "D", "disease": "Bacillary angiomatosis"}], "answer_idx": "C", "symptoms": ["A 28-year-old healthy white woman with no significant past medical history presents with an enlarging papule on her right lateral neck", "She describes a 1- to 2-mm pink papule that grew to approximately 10 mm within the past week", "With minimal rubbing or trauma, the papule intermittently bleeds", "It is otherwise nontender and nonpruritic", "She has no history of skin cancer and is taking oral contraceptives", "Physical examination shows a pink, friable papule on a stalk on the right lateral neck", "There are no other similar lesions seen on examination."], "s1": [0, 4, 6], "s2": [1, 2, 3, 5]} {"key": 143, "questions": "Which of the following is the most likely diagnosis for the patient?", "options": [{"label": "A", "disease": "Solitary mastocytoma"}, {"label": "B", "disease": "Pseudomonas nail infection"}, {"label": "C", "disease": "Epidermolysis bullosa"}, {"label": "D", "disease": "Bullous arthropod bites"}], "answer_idx": "A", "symptoms": ["A healthy 12-month-old infant girl presents with a yellowish-orange papule on the right upper extremity", "Her mother first noticed this lesion approximately 6 months earlier", "It has never bled, ulcerated, or formed blisters", "The infant does not scratch the lesion nor appears otherwise bothered by it", "However, when the lesion is rubbed, it appears to swell and turn bright red", "On physical examination, there is a soft, oval-shaped, yellowish papule with a peripheral rim of erythema on the right extensor forearm."], "s1": [0, 1, 2, 3], "s2": [4, 5]} {"key": 144, "questions": "Which of the following is the most likely diagnosis for the patient?", "options": [{"label": "A", "disease": "Subungual hematomas"}, {"label": "B", "disease": "Glomus tumors"}, {"label": "C", "disease": "Pseudomonas nail infection"}, {"label": "D", "disease": "Solitary mastocytoma"}], "answer_idx": "C", "symptoms": ["An otherwise healthy 60-year-old dishwasher presents with asymptomatic green discoloration of his right middle fingernail over a period of 2 months", "The discoloration began from the distal margin", "He denies wearing gloves while working and reports a tendency to pick at his nails when nervous", "On examination, distal onycholysis and green discoloration are present exclusively on the right middle nail plate starting from the distal edge without signs of paronychia", "Dermoscopy reveals pigment localization under the nail plate."], "s1": [2], "s2": [0, 1, 3, 4]} {"key": 145, "questions": "Which of the following is the most likely diagnosis for the patient?", "options": [{"label": "A", "disease": "Tinea incognito"}, {"label": "B", "disease": "Eczema"}, {"label": "C", "disease": "Cutaneous lupus"}, {"label": "D", "disease": "Levamisole-induced antineutrophil cytoplasmic antibody vasculitis"}], "answer_idx": "A", "symptoms": ["A 43-year-old elementary school teacher presents with a rash that began on his left thigh approximately 4 months earlier; the rash has since spread", "It is mildly itchy but has no associated pain or bleeding", "His past medical history is significant for allergic rhinitis and atopic dermatitis", "Suspecting eczema, he has been applying an over-the-counter hydrocortisone 1% cream as well as a moisturizer, which transiently alleviates the itch", "He has not changed laundry or hygiene products", "On physical examination, there are slightly erythematous, minimally scaly patches scattered over his left hip and buttock."], "s1": [0, 4, 5], "s2": [1, 2, 3]} {"key": 146, "questions": "Which of the following is the most likely diagnosis for the patient?", "options": [{"label": "A", "disease": "Disseminated intravascular coagulation"}, {"label": "B", "disease": "Warfarin-induced skin necrosis"}, {"label": "C", "disease": "Tinea incognito"}, {"label": "D", "disease": "Levamisole-induced antineutrophil cytoplasmic antibody vasculitis"}], "answer_idx": "D", "symptoms": ["A 50-year-old woman presents with a 5-day history of progressive, painful rash involving her face, trunk, extremities, nasal tip, and ears, in addition to fever and arthralgia", "She admits to a history of polysubstance use and states that her last use was a week prior to symptom onset", "She denies a history of autoimmune disease or recent travel", "Examination reveals retiform purpura with areas of necrosis and bullae formation", "Urine toxicology screening is positive for cocaine, and laboratory findings include leukopenia and an elevated erythrocyte sedimentation rate (70 mm/h)."], "s1": [0, 3], "s2": [1, 2, 4]} {"key": 147, "questions": "Which of the following is the most likely diagnosis for the patient?", "options": [{"label": "A", "disease": "Acrodermatitis enteropathica-like eruption (AELE)"}, {"label": "B", "disease": "Acrodermatitis enteropathica"}, {"label": "C", "disease": "Atopic dermatitis"}, {"label": "D", "disease": "AELE unrelated to zinc deficiency"}], "answer_idx": "B", "symptoms": ["A 7-month-old male who had been born prematurely presents with a 2-month history of diarrhea, failure to thrive, alopecia, and rash involving the perioral, perianal, and acral areas", "He had been treated unsuccessfully for presumed atopic dermatitis with topical corticosteroids", "He was exclusively breastfed until 2 months ago, when he was transitioned to formula", "Examination reveals an irritable, thin infant with sharply demarcated red, inflamed, scaly plaques and erosions around his mouth, anus, eyes, hands, and feet", "Diffuse hair loss is noted on the scalp, eyebrows, and eyelashes", "Paronychia and nail dystrophy are noted on his upper and lower extremities."], "s1": [0, 2, 4], "s2": [1, 3, 5]} {"key": 148, "questions": "Which of the following is the most likely diagnosis for the patient?", "options": [{"label": "A", "disease": "Pilomatricoma"}, {"label": "B", "disease": "Nodulocystic basal cell carcinoma"}, {"label": "C", "disease": "Squamous cell carcinoma"}, {"label": "D", "disease": "Pilar cyst"}], "answer_idx": "D", "symptoms": ["A 64-year-old woman presents with a 6-month history of a slowly enlarging bump near her left eyebrow", "The patient says it appeared suddenly and became a nuisance, as well as cosmetically unacceptable as it enlarged", "She denies any pain, tenderness, or oozing, and denies any other similar lesions elsewhere", "Her pertinent dermatologic medical and family history are unremarkable", "Physical examination reveals a 6-mm nontender subcutaneous mobile nodule on the left lateral eyebrow", "There is no overlying ulceration or central punctum, and no discharge is noted", "No other suspicious lesions are observed on examination of the scalp and face, aside from mild actinic damage."], "s1": [0, 1, 2, 3, 6], "s2": [4, 5]} {"key": 149, "questions": "Which of the following is the most likely diagnosis for the patient?", "options": [{"label": "A", "disease": "Schöpf-Schulz-Passarge syndrome"}, {"label": "B", "disease": "Witkop tooth and nail syndrome"}, {"label": "C", "disease": "Trichodento-osseous syndrome"}, {"label": "D", "disease": "Hypohidrotic ectodermal dysplasia"}], "answer_idx": "D", "symptoms": ["A 30-year-old Hispanic woman presents with her 1-year-old son, concerned that he has only small amounts of thin, wispy hair on his head and has none anywhere else on his body", "She notes that he does not yet have any teeth, but has a large forehead and ears", "He has been irritable and crying for the past day", "She reveals a personal history of very high body temperature at times during the summer and says she customarily stays indoors with the air conditioning on, especially in the summer", "On physical examination, she has sparse hair on her scalp and eyebrows, as well as eczematous patches on her bilateral lower extremities."], "s1": [0, 1, 4], "s2": [2, 3]} {"key": 150, "questions": "Which of the following is the most likely diagnosis for the patient?", "options": [{"label": "A", "disease": "Nipple inversion secondary to neglected malignancy"}, {"label": "B", "disease": "Periductal mastitis"}, {"label": "C", "disease": "Duct ectasia"}, {"label": "D", "disease": "Congenital nipple inversion"}], "answer_idx": "A", "symptoms": ["A 48-year-old premenopausal woman, who has had one pregnancy and has delivered once, presents with a 15-month history of left breast deformity", "The patient began having pain in the region 2 months ago", "A review of systems is negative for weight loss, fever, or nipple discharge", "Her medical history is otherwise unremarkable, and her family history is noncontributory", "Physical examination is notable for extensive left nipple inversion and retraction, a large underlying mass in the central breast, edema and ulceration of the overlying skin of the breast, and matted ipsilateral axillary nodes", "There is no erythema of the skin", "The right breast is unremarkable."], "s1": [0, 4, 1], "s2": [2, 3, 5, 6]} {"key": 151, "questions": "Which of the following is the most likely diagnosis for the patient?", "options": [{"label": "A", "disease": "Mycosis fungoides"}, {"label": "B", "disease": "Dyshidrosis"}, {"label": "C", "disease": "Sézary syndrome"}, {"label": "D", "disease": "Shoe dermatitis"}], "answer_idx": "D", "symptoms": ["A 25-year-old man with no remarkable past medical history presents with an itchy rash on the feet", "Symptoms began 2 months ago, starting over the dorsal toes with redness, scaling, and itching", "The eruption spread over the dorsum of the feet and progressed to weeping and crusting", "He reports recently buying new shoes for work as a chef, which requires 15 hours daily of standing", "Due to profuse sweating, his socks become soaked daily", "Examination reveals symmetric erythematous plaques over the bilateral dorsum of both feet with fissures, scaling, and lichenification", "No blisters or vesicles are present, but there are scattered crusted erosions."], "s1": [3, 4], "s2": [0, 1, 2, 5, 6]} {"key": 152, "questions": "Which of the following is the most likely diagnosis for the patient?", "options": [{"label": "A", "disease": "Marshall syndrome"}, {"label": "B", "disease": "Perifollicular elastolysis"}, {"label": "C", "disease": "Mid-dermal elastolysis"}, {"label": "D", "disease": "Anetoderma"}], "answer_idx": "C", "symptoms": ["A 55-year-old black man presents for evaluation of fine wrinkles that had developed over the past year", "Physical examination reveals diffuse areas offine wrinkling that run parallel to lines of cleavage on his neck, chest, axilla, trunk, back, and upper arms bilaterally", "The affected skin has normal pigmentation and lacks erythema, induration, and atrophy", "Lateral tension obliterates the wrinkling", "The patient never notes any inflammation, itching, or pain", "There are no signs or symptoms of systemic involvement", "He denies sunbathing and denies any history of skin disorders or other significant health problems."], "s1": [0, 1, 2, 3], "s2": [4, 5, 6]} {"key": 153, "questions": "Which of the following is the most likely diagnosis for the patient?", "options": [{"label": "A", "disease": "Ulcers"}, {"label": "B", "disease": "Hematomas"}, {"label": "C", "disease": "Acral lentiginous melanoma"}, {"label": "D", "disease": "Dysplastic nevus"}], "answer_idx": "C", "symptoms": ["A 26-year-old African American man with no significant past medical history presents with a growth on his left foot that had been increasing in size for 1 year", "He initially thought it was a wart but had concerns when it continued to grow and became painful", "Examination reveals an exophytic, brown, and mildly tender nodule, 3cm in diameter, on his left medial heel and instep", "The overlying skin is crusted and friable", "His nails and hair show no abnormalities", "There is no associated lymphadenopathy", "The remainder of the examination is unremarkable."], "s1": [0, 1, 2, 3], "s2": [4, 5, 6]} {"key": 154, "questions": "Which of the following is the most likely diagnosis for the patient?", "options": [{"label": "A", "disease": "Hydroxychloroquine-induced hyperpigmentation"}, {"label": "B", "disease": "Drug-induced pigmentation"}, {"label": "C", "disease": "Postinflammatory hyperpigmentation"}, {"label": "D", "disease": "Fixed drug eruption"}], "answer_idx": "A", "symptoms": ["An African American woman, aged 50 years, presents with painless “bruises” on her leg", "She first noticed bruises on both legs a year ago and reports that they have since become larger", "Physical examination reveals blue-black macules and patches on both shins", "The macules are asymptomatic, and the remainder of the physical examination is within normal limits", "A venous stasis ulcer is seen on her right leg", "She says she has a 10-year history of discoid and systemic lupus erythematosus treated with hydroxychloroquine and does not recall any history of trauma to her legs."], "s1": [0, 1, 2, 3], "s2": [4, 5]} {"key": 155, "questions": "Which of the following is the most likely diagnosis for the patient?", "options": [{"label": "A", "disease": "Bullous impetigo"}, {"label": "B", "disease": "Psoriasis"}, {"label": "C", "disease": "Tinea corporis"}, {"label": "D", "disease": "Nummular dermatitis"}], "answer_idx": "D", "symptoms": ["A man, aged 68 years, with no known history of atopy presents with a 4-month history of an itchy and burning rash on the legs that has been recurring every winter for the past 3 years", "He notes that he had dry skin at baseline and did not use emollients, but he did try hydrocortisone 1% cream without improvement", "Examination revealed ovoid, erythematous, well-marginated plaques on the lower legs and dorsal feet with variable scaling and focal weeping and crusting", "There were no nail changes, and the rest of his skin examination was notable only for xerosis."], "s1": [0, 1], "s2": [2, 3]} {"key": 156, "questions": "Which of the following is the most likely diagnosis for the patient?", "options": [{"label": "A", "disease": "Laugier-Hunziker syndrome"}, {"label": "B", "disease": "Peutz-Jeghers syndrome"}, {"label": "C", "disease": "Bandler syndrome"}, {"label": "D", "disease": "McCune-Albright syndrome"}], "answer_idx": "A", "symptoms": ["An elderly man presents with asymptomatic hyperpigmented, confluent, brown macules on his upper and lower lips, along with hyperpigmented macules on his hands", "He also has diffuse blue-black bilateral melanonychia with nail dystrophy on his toenails, and his fingernails contain longitudinal melanonychia with hyperpigmentation of the proximal nail beds", "He denies any family history of mucocutaneous pigmentation or gastrointestinal or systemic symptoms", "In the last 5 years, results of a colonoscopy, upper gastrointestinal endoscopy, and blood work have all been within normal limits."], "s1": [0, 1], "s2": [2, 3]} {"key": 157, "questions": "Which of the following is the most likely diagnosis for the patient?", "options": [{"label": "A", "disease": "Pellagra"}, {"label": "B", "disease": "Polymorphous light eruption"}, {"label": "C", "disease": "Photoallergic skin reaction"}, {"label": "D", "disease": "Phototoxic skin reaction"}], "answer_idx": "A", "symptoms": ["A man, aged 42 years, presents with a 6-month history of a painful, scaly skin rash", "The eruption is symmetrically distributed over photosensitive areas, affecting his face, neck, upper back, forearms, and hands", "The dorsal surfaces of his hands and forearms exhibit a dusky brown coloration over rough and cracked skin", "He complains of itching and burning and reports intermittent vomiting and diarrhea", "He admits to heavy alcohol intake for the last 2 decades and denies taking any medications", "Neuropsychiatric evaluation reveals short-term memory loss, speech problems, and poor concentration."], "s1": [0, 1, 2, 3], "s2": [4, 5]} {"key": 158, "questions": "Which of the following is the most likely diagnosis for the patient?", "options": [{"label": "A", "disease": "Acanthosis nigricans"}, {"label": "B", "disease": "Darier disease"}, {"label": "C", "disease": "Chronic eczema"}, {"label": "D", "disease": "Nipple hyperkeratosis"}], "answer_idx": "D", "symptoms": ["A 68-year-old white man presents with 4 weeks of progressively worsening thickening of the skin and plaques on the bilateral nipple and areola", "He has no personal or family history of warts or epidermal nevi and has not had hormonal therapy", "On examination, he has hyperpigmented, verrucous plaques bilaterally, with no associated erythema or lymphadenopathy", "Skin biopsy shows hyperkeratosis with papillomatosis", "Corticosteroid treatment failed, but there was some minor improvement with calcipotriol after several weeks of use."], "s1": [0, 2, 3], "s2": [1, 4]} {"key": 159, "questions": "Which of the following is the most likely diagnosis for the patient?", "options": [{"label": "A", "disease": "Lichen striatus"}, {"label": "B", "disease": "Lichen planus"}, {"label": "C", "disease": "Lichen nitidus"}, {"label": "D", "disease": "Guttate lichen sclerosus"}], "answer_idx": "C", "symptoms": ["A 4-year-old white male presents with a 3-month history of an asymptomatic papular rash on his back, abdomen, chest, upper arms, and ankles", "His medical and family histories are otherwise unremarkable", "On examination, the grouped papules are shiny, monomorphic, skin-colored, and pinhead-sized", "The papules exhibit koebnerization but no erythema or tenderness", "His nails, hair, oral mucosa, and genitals are normal."], "s1": [0, 2, 3], "s2": [1, 4]} {"key": 160, "questions": "Which of the following is the most likely diagnosis for the patient?", "options": [{"label": "A", "disease": "Lichen planus"}, {"label": "B", "disease": "Herpes simplex viral lesions"}, {"label": "C", "disease": "Guttate psoriasis"}, {"label": "D", "disease": "Scabies infestation"}], "answer_idx": "A", "symptoms": ["An African-American man, aged 35 years, presents with a 3-month history of pruritic penile lesions", "Examination reveals an arcuate pattern of raised violaceous-to-flesh-colored papules with central clearing on the glans penis and a few scattered, raised violaceous-to-flesh-colored polygonal papules of the glans penis and distal penile shaft", "Results of the remainder of the skin examination are normal, and there is no inguinal lymphadenopathy", "His sexual history includes no previous sexually transmitted diseases and no new sexual partners within the past year."], "s1": [0, 1], "s2": [2, 3]} {"key": 161, "questions": "Which of the following is the most likely diagnosis for the patient?", "options": [{"label": "A", "disease": "Systemic lupus erythematosus"}, {"label": "B", "disease": "Dermatomyositis"}, {"label": "C", "disease": "Chemical burn"}, {"label": "D", "disease": "Drug-induced photosensitivity"}], "answer_idx": "D", "symptoms": ["A 50-year-old woman presents with darkening skin that started several weeks ago", "She recently went to the beach and developed severe redness in sun-exposed areas within a few hours of being out in the sun", "She did not wear sunscreen and says she usually does not burn in the sun", "The affected areas have since darkened", "She started hydrochlorothiazide for hypertension 4 months ago", "On physical examination, hyperpigmentation is noted on the dorsal hands, proximal forearms, dorsal feet with sparing of skin covered by her sandal straps, “V” of her neck, cheeks, and forehead."], "s1": [1, 2, 5], "s2": [0, 3, 4]} {"key": 162, "questions": "Which of the following is the most likely diagnosis for the patient?", "options": [{"label": "A", "disease": "Synovial cyst"}, {"label": "B", "disease": "Ganglion cyst"}, {"label": "C", "disease": "Pigmented villonodular synovitis"}, {"label": "D", "disease": "Lipoma"}], "answer_idx": "B", "symptoms": ["A 61-year-old woman presents with a 7-month history of a bump on the front of her left knee", "The lesion is slightly tender but otherwise is asymptomatic", "She is concerned that the lesion may be cancerous", "She has no other medical problems and takes vitamin D daily", "On physical examination, a firm subcutaneous tumor is noted on the left knee measuring 8 cm without overlying skin changes", "She has no other similar lesions elsewhere on her body."], "s1": [0, 1, 4], "s2": [2, 3, 5]} {"key": 163, "questions": "Which of the following is the most likely diagnosis for the patient?", "options": [{"label": "A", "disease": "Pityriasis rosea"}, {"label": "B", "disease": "Tinea corporis"}, {"label": "C", "disease": "Vitiligo"}, {"label": "D", "disease": "Tinea versicolor"}], "answer_idx": "D", "symptoms": ["A 65-year-old man presents with a recurring rash on his back", "He has had the rash several times before, most often during the summer, and it was treated successfully with an antifungal shampoo prescribed by a prior dermatologist", "The rash is not itchy or painful", "He has a history of depression and hypertension", "On physical examination, hyperpigmented macules coalescing into patches with very fine white scale are found", "The lesions are scattered primarily on his upper chest and upper back."], "s1": [0, 1, 2, 4, 5], "s2": [3]} {"key": 164, "questions": "Which of the following is the most likely diagnosis for the patient?", "options": [{"label": "A", "disease": "GGXC-mediated coagulation factor deficiency"}, {"label": "B", "disease": "ENPP1-mediated premature arterial calcification"}, {"label": "C", "disease": "Pseudoxanthoma elasticum"}, {"label": "D", "disease": "Ehlers-Danlos syndrome"}], "answer_idx": "C", "symptoms": ["A 22-year-old woman presents for evaluation of progressive skin changes on her neck and arms beginning sometime around puberty", "She tried various over-the-counter anti-itch, antiaging, and eczema creams that were ineffective", "She has no personal or family history of skin conditions and is otherwise healthy", "Review of systems is negative for skin laxity, joint hypermobility, or vision changes", "Physical examination reveals numerous nontender, noninflammatory, yellow, 2- to 5-mm papules that coalesce into reticulated plaques on the nape of her neck and bilateral antecubital fossa", "A punch biopsy demonstrates fragmentation and calcification of elastic fibers."], "s1": [0, 1, 2, 3], "s2": [4, 5]} {"key": 165, "questions": "Which of the following is the most likely diagnosis for the patient?", "options": [{"label": "A", "disease": "Morphea"}, {"label": "B", "disease": "Stiff-skin syndrome"}, {"label": "C", "disease": "Systemic sclerosis"}, {"label": "D", "disease": "Scleredema"}], "answer_idx": "C", "symptoms": ["A 47-year-old otherwise healthy woman presents to the clinic with several concerns", "Over the past several months, she has experienced thickening of her skin around her hands, forearms, and face", "Her fingers turn blue and then pale when she is exposed to cold", "Additional recent symptoms include fatigue and gastroesophageal reflux", "She takes no medications and has no family history of autoimmune disease", "On physical examination, the patient has skin thickening on the hands, forearms, and face", "She also has hypopigmented macules on the hands and lower legs."], "s1": [0, 3, 4], "s2": [1, 2, 5, 6]} {"key": 166, "questions": "Which of the following is the most likely diagnosis for the patient?", "options": [{"label": "A", "disease": "Tinea versicolor"}, {"label": "B", "disease": "Vitiligo"}, {"label": "C", "disease": "Nevus depigmentosus"}, {"label": "D", "disease": "Postinflammatory hypomelanosis"}], "answer_idx": "B", "symptoms": ["A 15-year-old Black man presents with a 1-year history of light spots on his back", "The rash has grown slowly, with several new lesions appearing near the first one", "It is not itchy or painful", "He has not tried any treatments for this condition", "He has no relevant medical history and none of his family members have similar skin findings", "On examination, there are several well-demarcated depigmented macules and patches on his lateral back."], "s1": [0, 1, 2, 5], "s2": [3, 4]} {"key": 167, "questions": "Which of the following is the most likely diagnosis for the patient?", "options": [{"label": "A", "disease": "Bullous pemphigoid"}, {"label": "B", "disease": "Pemphigus vulgaris"}, {"label": "C", "disease": "Secondary syphilis"}, {"label": "D", "disease": "Pityriasis lichenoides et varioliformis acuta (PLEVA)"}], "answer_idx": "C", "symptoms": ["A 60-year-old Black woman presents with a 30-day history of a nonitchy rash", "The rash is located on her face, trunk, and extremities", "She reports that she has tried topical steroids on the rash, which did not help", "The patient has no other medical conditions and takes no medications", "She notes that she had unprotected sex with a new partner about 4 months prior to presentation", "On physical examination, the patient has many scattered erythematous scaly papules on her face, abdomen, back, and upper and lower extremities including the palms and soles."], "s1": [0, 1, 2, 5], "s2": [3, 4]} {"key": 168, "questions": "Which of the following is the most likely diagnosis for the patient?", "options": [{"label": "A", "disease": "Basal cell carcinoma"}, {"label": "B", "disease": "Acanthoma fissuratum"}, {"label": "C", "disease": "Chondrodermatitis nodularis helicis"}, {"label": "D", "disease": "Lichen simplex chronicus"}], "answer_idx": "B", "symptoms": ["A 64-year-old man with a history of numerous nonmelanoma skin cancers presents with a lesion on the left side of his nose that was first noticed about 6 months ago and is slowly growing", "The lesion is not painful or itchy", "The patient has not tried any medications on the lesion", "The patient wears eyeglasses and the nose pad of his eyeglasses rubs on the lesion", "On physical examination, an erythematous papule is found on the left superior nasal sidewall with a central depression."], "s1": [0, 4], "s2": [1, 2, 3]} {"key": 169, "questions": "Which of the following is the most likely diagnosis for the patient?", "options": [{"label": "A", "disease": "Schnitzler syndrome"}, {"label": "B", "disease": "Urticarial vasculitis"}, {"label": "C", "disease": "Systemic lupus erythematosus"}, {"label": "D", "disease": "Photodermatitis"}], "answer_idx": "B", "symptoms": ["A 54-year-old woman with a history of depression presents to the clinic with a 1-year history of an intermittent rash", "She was hospitalized for COVID-19 approximately 1 year ago and soon after developed a rash all over her body", "The lesions persist for several days and have a burning sensation", "The patient has no associated joint pain or other systemic symptoms", "She tried antihistamines for the rash with no symptomatic relief", "On examination, erythematous wheals are evident on her arms and legs as well as her back and abdomen."], "s1": [0, 3], "s2": [1, 2, 4, 5]} {"key": 170, "questions": "Which of the following is the most likely diagnosis for the patient?", "options": [{"label": "A", "disease": "Dermatitis"}, {"label": "B", "disease": "Acne rosacea"}, {"label": "C", "disease": "Seborrheic dermatitis"}, {"label": "D", "disease": "Systemic lupus erythematosus"}], "answer_idx": "D", "symptoms": ["A 42-year-old Hispanic man with no prior medical history presents with a 2-month history of a rash on his face, chest, and arms", "He reports that the rash gets worse after sun exposure and he has associated joint pain", "He has taken over-the-counter nonsteroidal anti-inflammatory drugs (NSAIDs) for the joint pain and topical steroids for the rash with no symptomatic relief", "Physical examination reveals erythematous patches over the malar cheeks and dorsal nose sparing the nasolabial fold and erythematous macules on the forearms and chest."], "s1": [0, 3], "s2": [1, 2]} {"key": 171, "questions": "Which of the following is the most likely diagnosis for the patient?", "options": [{"label": "A", "disease": "Lymphomatoid papulosis"}, {"label": "B", "disease": "Pityriasis lichenoides chronica"}, {"label": "C", "disease": "Lichen planus"}, {"label": "D", "disease": "Guttate psoriasis"}], "answer_idx": "B", "symptoms": ["A 6-year-old girl presents with a 1-year history of a waxing and waning rash on her limbs and face", "The rash starts as red scaly bumps that fade over several weeks leaving smooth white spots", "Every few months, the patient gets several new red bumps", "The rash is not itchy or painful", "Her parents have tried topical steroid creams that did not help clear the rash", "Examination reveals scattered erythematous scaly papules on both legs and several hypopigmented macules on her legs, arms, and face."], "s1": [3, 4], "s2": [0, 1, 2, 5]} {"key": 172, "questions": "Which of the following is the most likely diagnosis for the patient?", "options": [{"label": "A", "disease": "Tinea capitis"}, {"label": "B", "disease": "Androgenetic alopecia"}, {"label": "C", "disease": "Alopecia areata"}, {"label": "D", "disease": "Trichotillomania"}], "answer_idx": "C", "symptoms": ["A 15-year-old girl presents with patches of hair loss on her scalp that have been present for the last 6 months", "She reports that her scalp is not itchy or painful and denies any recent illnesses, diet changes, or stressful life events", "She does not have any other medical conditions and takes no medications", "Her uncle had a similar condition that started when he was in his 20s, but no one else in the family has experienced hair loss", "The patient says she has not tried any treatments for the condition", "On examination, she has several well-demarcated patches of hair loss on her frontal and parietal scalp, with no perifollicular erythema or scale."], "s1": [0, 3, 5], "s2": [1, 2, 4]} {"key": 173, "questions": "Which of the following is the most likely diagnosis for the patient?", "options": [{"label": "A", "disease": "Amelanotic melanoma"}, {"label": "B", "disease": "Squamous cell carcinoma"}, {"label": "C", "disease": "Bacillary angiomatosis"}, {"label": "D", "disease": "Lobular capillary hemangioma"}], "answer_idx": "D", "symptoms": ["A 65-year-old man of Middle Eastern descent presents with a tender, weeping mass on his right palm", "Since appearing 8 months ago, the lesion has enlarged into a 0.9-cm, pink pedunculated nodule with abundant serous crust", "The patient has no significant history of trauma", "His medical history is significant for atrial fibrillation, hypertension, coronary artery disease, congestive heart failure, and lung cancer, which was treated with right lobectomy", "It is unclear what medications were used to treat his lung cancer", "His current medications include metoprolol, lisinopril, allopurinol, aspirin, and digoxin", "The patient also presents with additional pink, scaly plaques on his right ankle, consistent with nummular eczema."], "s1": [0, 1, 6], "s2": [2, 3, 4, 5]} {"key": 174, "questions": "Which of the following is the most likely diagnosis for the patient?", "options": [{"label": "A", "disease": "Fungal infection"}, {"label": "B", "disease": "Cutaneous lupus"}, {"label": "C", "disease": "Cutaneous sarcoidosis"}, {"label": "D", "disease": "Rosacea"}], "answer_idx": "C", "symptoms": ["A 33-year-old Black man presents to a dermatology clinic with a 5-month history of asymptomatic lesions on his arms and trunk", "He is otherwise healthy with no personal or family history of skin disease", "He has no history of fever, cough, shortness of breath, or joint pain", "The patient has not tried any treatments for these lesions", "On examination, there are several smooth, well-demarcated hypopigmented plaques on his upper arms and left flank", "He has no similar lesions elsewhere on his body."], "s1": [0, 4], "s2": [1, 2, 3, 5]} {"key": 175, "questions": "Which of the following is the most likely diagnosis for the patient?", "options": [{"label": "A", "disease": "Sarcoidosis"}, {"label": "B", "disease": "Localized granuloma annulare"}, {"label": "C", "disease": "Tinea corporis"}, {"label": "D", "disease": "Lupus"}], "answer_idx": "B", "symptoms": ["A 61-year-old man presents with a 7-month history of lesions on his hands and arms", "His medical history includes depression, hypertension, and hyperlipidemia", "He has no personal or family history of skin problems", "His skin lesions are not painful or itchy, and he is not bothered by their appearance", "He has not tried any treatments for the lesions", "Physical examination reveals a number of pink, annular plaques with smooth raised borders on the patient’s dorsal forearms and hands", "On close inspection, small discrete papules are seen within the plaques."], "s1": [1, 2, 4], "s2": [0, 3, 5, 6]} {"key": 176, "questions": "Which of the following is the most likely diagnosis for the patient?", "options": [{"label": "A", "disease": "Lichen planus"}, {"label": "B", "disease": "Psoriasis"}, {"label": "C", "disease": "Discoid lupus"}, {"label": "D", "disease": "Pityriasis rosea"}], "answer_idx": "A", "symptoms": ["A 32-year-old woman presents with an itchy rash that began 3 months ago", "She has a history of anxiety but otherwise no known medical conditions", "She has tried over-the-counter hydrocortisone cream to treat the rash, finding some relief from the itchiness, but the rash continues to spread", "She has no known allergies and has not changed her skin care products or medications", "On examination, the patient has dark purple and gray flat-topped papules with fine gray-white streaks on her flexural wrists, dorsal hands, elbows, ankles, and dorsal feet."], "s1": [0, 2, 4], "s2": [1, 3]} {"key": 177, "questions": "Which of the following is the most likely diagnosis for the patient?", "options": [{"label": "A", "disease": "Psoriasis"}, {"label": "B", "disease": "Chronic paronychia"}, {"label": "C", "disease": "Herpetic whitlow"}, {"label": "D", "disease": "Granuloma annulare"}], "answer_idx": "B", "symptoms": ["A 35-year-old woman presents to the dermatology clinic with inflammation of her fingernails that first appeared 5 months ago and recently worsened", "She reports wearing acrylic nails for the last year, removing them just before the clinic visit", "On examination, the patient has erythema and swelling of the lateral nail folds and hyperpigmentation of the proximal nail folds of all fingers on her right hand", "Examination also reveals nail dystrophy, onycholysis, and an absence of cuticles", "She has not tried any treatments for this condition."], "s1": [0, 4], "s2": [1, 2, 3]} {"key": 178, "questions": "Which of the following is the most likely diagnosis for the patient?", "options": [{"label": "A", "disease": "Gangrene"}, {"label": "B", "disease": "Frostbite"}, {"label": "C", "disease": "Pseudoainhum"}, {"label": "D", "disease": "Hair-thread tourniquet syndrome"}], "answer_idx": "C", "symptoms": ["A 42-year-old woman was admitted to the hospital for complications related to type 2 diabetes", "Her inpatient team noticed that her right fifth finger was swollen, red, and appeared to have a fissure", "The patient stated that although the fissure was not present since birth, her finger has looked this way for “a while” and typically did not bother her", "On examination, she had hyperkeratotic plaques on some of her fingers", "She denied trauma to the site of injury and her family and travel history were unremarkable."], "s1": [0], "s2": [1, 2, 3, 4]} {"key": 179, "questions": "Which of the following is the most likely diagnosis for the patient?", "options": [{"label": "A", "disease": "Maculopapular drug eruption"}, {"label": "B", "disease": "IgA-mediated small vessel vasculitis"}, {"label": "C", "disease": "Scurvy"}, {"label": "D", "disease": "Pigmented purpuric dermatoses"}], "answer_idx": "B", "symptoms": ["A 50-year-old woman presents with a crop of lesions that recently appeared on her buttocks and legs", "On examination, the red macules and papules range in size from a few millimeters to a few centimeters; when pressed, the lesions do not blanch or partially blanch", "The patient reports joint pain over her ankles and knees, along with malaise", "When questioned, she acknowledges having had a “cold” a few weeks earlier, for which her primary care provider prescribed an antibiotic", "Direct immunofluorescence of the lesions reveals immunoglobulin A (IgA) deposits around the small vessels in the papillary dermis."], "s1": [0, 1, 4], "s2": [2, 3]} {"key": 180, "questions": "Which of the following is the most likely diagnosis for the patient?", "options": [{"label": "A", "disease": "Traumatic lesions"}, {"label": "B", "disease": "Acral lentiginous melanoma"}, {"label": "C", "disease": "Tinea nigra"}, {"label": "D", "disease": "Calluses"}], "answer_idx": "B", "symptoms": ["A 70-year-old White man presents for evaluation of a recent foot injury", "The patient states that he hit his foot at home and noticed blood on the floor; because of a previous injury, he was unable to view his sole", "A large (several cm), irregularly shaped plaque is seen on the mid-anterior volar foot", "The lesion is brown and black, with central red discoloration and overlying yellow-white scale."], "s1": [0, 1], "s2": [2, 3]} {"key": 181, "questions": "Which of the following is the most likely diagnosis for the patient?", "options": [{"label": "A", "disease": "Nocardiosis"}, {"label": "B", "disease": "Chromomycosis"}, {"label": "C", "disease": "Histoplasmosis"}, {"label": "D", "disease": "Blastomycosis"}], "answer_idx": "D", "symptoms": ["A 35-year-old construction worker presents with a well-circumscribed, verrucous plaque on his left upper arm", "The lesion has been present for a few months, but recently has increased in size and become painful", "He reports that a cough preceded his skin lesion, but otherwise has no relevant medical history", "A large verrucous and ulcerating lesion with pustules is seen", "On palpation, a purulent discharge is expressed."], "s1": [2], "s2": [0, 1, 3, 4]} {"key": 182, "questions": "Which of the following is the most likely diagnosis for the patient?", "options": [{"label": "A", "disease": "Vascular hemorrhage"}, {"label": "B", "disease": "Ataxia-telangiectasia"}, {"label": "C", "disease": "Hereditary hemorrhagic telangiectasia"}, {"label": "D", "disease": "Bloom syndrome"}], "answer_idx": "C", "symptoms": ["A 45-year-old woman presents with enlarging blood vessels on her tongue that recently have increased in size and number", "Numerous 1- to 2-mm discrete red macules and papules are seen on her tongue, oral mucosa, nose, lower lip, and fingertips", "She has had frequent nosebleeds since childhood as well as iron-deficiency anemia that has not responded to iron supplementation", "Her father, who died at the age of 50, had similar symptoms."], "s1": [0, 1], "s2": [2, 3]} {"key": 183, "questions": "Which of the following is the most likely diagnosis for the patient?", "options": [{"label": "A", "disease": "Becker nevus"}, {"label": "B", "disease": "Café au lait macules"}, {"label": "C", "disease": "Congenital smooth muscle hamartomas"}, {"label": "D", "disease": "Congenital melanocytic nevi"}], "answer_idx": "A", "symptoms": ["A 12-year-old boy presents to the dermatology clinic with a 2-year history of a brown patch on his chest", "The 8-cm patch had been asymptomatic, but over the past 6 months, it has become darker, with hair and acne vulgaris developing within the lesion", "The patient is self-conscious about the lesion’s appearance and wishes to learn if it can be treated to lessen the hyperpigmentation."], "s1": [0, 1], "s2": [2]} {"key": 184, "questions": "Which of the following is the most likely diagnosis for the patient?", "options": [{"label": "A", "disease": "Addison disease"}, {"label": "B", "disease": "Fixed-drug eruption"}, {"label": "C", "disease": "Macular amyloidosis"}, {"label": "D", "disease": "Erythema dyschromicum perstans"}], "answer_idx": "D", "symptoms": ["A 28-year-old Black man presents with a 2-year history of dyspigmentation of his back and chest", "The patient notes that the lesions initially had red borders but these have faded over the past several months", "Physical examination reveals circular, ashy-gray macules distributed diffusely over his back and chest", "The patient denies pain or pruritus and takes nomedication", "He notes that he recently traveled to Mexico", "He has been relatively unconcerned about the lesions and only sought medical care to rule out cancer."], "s1": [0, 2, 3], "s2": [1, 4, 5]} {"key": 185, "questions": "Which of the following is the most likely diagnosis for the patient?", "options": [{"label": "A", "disease": "Morphea"}, {"label": "B", "disease": "Systemic sclerosis"}, {"label": "C", "disease": "Erythema migrans"}, {"label": "D", "disease": "Post-inflammatory hyperpigmentation"}], "answer_idx": "A", "symptoms": ["A 45-year-old woman presents with oval-shaped lesions on her arms that she first noted a few months ago", "The lesions began as an area of discrete erythema, progressed to induration, and then became dark, smooth, and shiny", "She has a family history of rheumatoid arthritis", "On examination, the patient has multiple hyperpigmented and hypopigmented indurated plaques with a shiny surface; her nails and fingertips are normal, and autoantibody tests were negative."], "s1": [2], "s2": [0, 1, 3]} {"key": 186, "questions": "Which of the following is the most likely diagnosis for the patient?", "options": [{"label": "A", "disease": "Palmar lichen planus"}, {"label": "B", "disease": "Melanocytic nevi"}, {"label": "C", "disease": "Lentigines"}, {"label": "D", "disease": "Tinea nigra"}], "answer_idx": "D", "symptoms": ["An 8-year-old boy presents with a 2-month history of an asymptomatic rash on the dorsum of his right hand", "He and his family have recently moved from a coastal region in Mexico", "Medical and family history are otherwise unremarkable", "On examination, he has a 2-cm pigmented macular lesion with an irregular outline centrally located on the dorsal aspect of his right hand", "The remainder of the physical examination is normal", "Dermoscopy rules out a nevoid lesion, and septated filaments are observed on potassium hydroxide (KOH) test."], "s1": [0, 3, 5], "s2": [1, 2, 4]} {"key": 187, "questions": "Which of the following is the most likely diagnosis for the patient?", "options": [{"label": "A", "disease": "Leishmaniasis"}, {"label": "B", "disease": "Ulceroglandular tularemia"}, {"label": "C", "disease": "Cutaneous anthrax"}, {"label": "D", "disease": "Leprosy"}], "answer_idx": "A", "symptoms": ["A 53-year-old man presents to the clinic with ulcerated, scarred, and hypertrophic plaques on both lower extremities", "He is a refugee from Sudan who arrived in the United States a month earlier", "He reports that the lesions started as red bumps that slowly expanded and developed a raised border", "Several of his family members in Sudan have had similar lesions in the past, and he is concerned that he may have developed the same condition."], "s1": [0, 2], "s2": [1, 3]} {"key": 188, "questions": "Which of the following is the most likely diagnosis for the patient?", "options": [{"label": "A", "disease": "Pemphigus foliaceus"}, {"label": "B", "disease": "Photolichenoid drug eruption"}, {"label": "C", "disease": "Discoid lupus erythematosus"}, {"label": "D", "disease": "Subacute cutaneous lupus erythematosus"}], "answer_idx": "D", "symptoms": ["A 48-year-old white man presents to the dermatology clinic with papulosquamous cutaneous lesions that recently appeared on his chest, arms, and the dorsal aspects of his hands", "The patient has a family history of autoimmune disorders, including systemic lupus erythematosus (SLE)", "Physical examination reveals that the lesions lack induration and do not appear to be leaving scars", "Biopsy of the lesion is performed, revealing mild inflammatory infiltrate", "Direct immunofluorescence shows a granular deposition at the dermal-epidermal junction", "Serology indicates that the patient is positive for anti-Ro/SSA (Sjögren-syndrome-related antigen A)."], "s1": [0, 2, 3], "s2": [1, 4, 5]} {"key": 189, "questions": "Which of the following is the most likely diagnosis for the patient?", "options": [{"label": "A", "disease": "Amyloidosis cutis dyschromica"}, {"label": "B", "disease": "Dowling-Degos disease"}, {"label": "C", "disease": "Confluent and reticulated papillomatosis"}, {"label": "D", "disease": "Prurigo pigmentosa"}], "answer_idx": "C", "symptoms": ["A 15-year-old white adolescent presents to the dermatology clinic with an asymptomatic rash on his upper back that has persisted for 4 months", "Physical examination reveals hyperpigmented scaly papules coalescing into confluent plaques with peripheral reticulated papillomatosis", "The boy is overweight but has no history of any other medical conditions."], "s1": [0, 1], "s2": [2]} {"key": 190, "questions": "Which of the following is the most likely diagnosis for the patient?", "options": [{"label": "A", "disease": "Brittle nails"}, {"label": "B", "disease": "Onychomycosis"}, {"label": "C", "disease": "Senile nails"}, {"label": "D", "disease": "Trachyonychia"}], "answer_idx": "D", "symptoms": ["An 8-year-old boy presents to the clinic for evaluation of a progressively worsening nail dystrophy that began a few months ago", "The patient’s mother denies a family history of skin diseases and reports that her son has no history of medical problems", "On physical examination, all of the boy’s fingernails and toenails show hyperkeratosis, longitudinal ridging, loss of luster, and roughness", "His hair, skin, and oral mucosa are unremarkable", "Nail biopsy reveals spongiotic changes and exocytosis of inflammatory cells into the nail epithelia", "The results of a fungal culture are negative."], "s1": [0, 2, 4], "s2": [1, 3, 5]} {"key": 191, "questions": "Which of the following is the most likely diagnosis for the patient?", "options": [{"label": "A", "disease": "Androgenetic alopecia"}, {"label": "B", "disease": "Central centrifugal cicatricial alopecia"}, {"label": "C", "disease": "Telogen effluvium"}, {"label": "D", "disease": "Alopecia areata"}], "answer_idx": "A", "symptoms": ["A 54-year-old woman presents with 2 years of gradual hair thinning", "She says that her mother also experienced hair thinning starting in her 50s", "She denies scalp itch", "She denies clumps of hair coming out in the shower or on her pillow", "She denies any preceding stressful life events", "She denies wearing her hair in tight braids", "She denies loss of eyebrows or eyelashes", "Physical examination reveals frontotemporal hair thinning without scarring."], "s1": [0, 1, 7], "s2": [2, 3, 4, 5, 6]} {"key": 192, "questions": "Which of the following is the most likely diagnosis for the patient?", "options": [{"label": "A", "disease": "Androgenetic alopecia"}, {"label": "B", "disease": "Telogen effluvium"}, {"label": "C", "disease": "Alopecia areata"}, {"label": "D", "disease": "Syphilitic alopecia"}], "answer_idx": "B", "symptoms": ["A 20-year-old man presents with 2 months of hair loss", "He says that approximately 8 months ago, he was involved in a motor vehicle accident which resulted in several fractures and a difficult recovery", "He has noticed clumps of hair coming out in the shower and on his pillow", "He denies scalp itch", "He denies any recent infections or genital sores", "Physical exmamination reveals diffuse hair thinning on the scalp without scarring."], "s1": [0, 2, 3, 5], "s2": [1, 4]} {"key": 193, "questions": "Which of the following is the most likely diagnosis for the patient?", "options": [{"label": "A", "disease": "Subcutaneous panniculitis-like T-cell lymphoma"}, {"label": "B", "disease": "Erythema induratum"}, {"label": "C", "disease": "Erythema nodosum"}, {"label": "D", "disease": "Lupus profundus"}], "answer_idx": "C", "symptoms": ["A 45-year-old woman presents with 3 months of tender bumps on the shins", "She says that 4 months ago, she was hospitalized for abdominal pain and was found to be positive for hepatitis B", "She denies ever having tuberculosis", "She denies itching or scratching at the bumps", "She denies any rash on the trunk or extremities", "Physical examination reveals tender, subcutaneous nodules on the bilateral shins."], "s1": [1, 2, 4], "s2": [0, 3, 5]} {"key": 194, "questions": "Which of the following is the most likely diagnosis for the patient?", "options": [{"label": "A", "disease": "Squamous cell carcinoma"}, {"label": "B", "disease": "Melanoma"}, {"label": "C", "disease": "Atypical fibroxanthoma"}, {"label": "D", "disease": "Basal cell carcinoma"}], "answer_idx": "D", "symptoms": ["A 70-year-old man presents with 1 year of a non-healing growth on the right cheek", "He says he first noticed it when he accidentally bumped into it and it bled easily", "He denies any preceding trauma to the lesion", "Physical examination reveals a pearly, eroded telangiectatic papule on the right cheek."], "s1": [0, 3], "s2": [1, 2]} {"key": 195, "questions": "Which of the following is the most likely diagnosis for the patient?", "options": [{"label": "A", "disease": "Allergic contact dermatitis"}, {"label": "B", "disease": "Juvenile spring eruption"}, {"label": "C", "disease": "Juvenile dermatomyositis"}, {"label": "D", "disease": "Seborrheic dermatitis"}], "answer_idx": "B", "symptoms": ["An 8-year-old male presents with 1 week of itchy blisters on the ears", "He just recently returned from spring break with his family", "He denies using any particular products on the ears before the rash", "He says that this has occurred yearly for the last several years, always around the same time", "He denies proximal muscle weakness", "He denies flaking in the scalp", "Physical examination reveals tiny vesicles atop erythematous patches on the superior helices."], "s1": [0, 6], "s2": [1, 2, 3, 4, 5]} {"key": 196, "questions": "Which of the following is the most likely diagnosis for the patient?", "options": [{"label": "A", "disease": "Scabies"}, {"label": "B", "disease": "Cholestatic pruritus"}, {"label": "C", "disease": "Renal pruritus"}, {"label": "D", "disease": "Hodgkin lymphoma"}], "answer_idx": "D", "symptoms": ["A 67-year-old male presents with unrelenting itch present over the last 6 months", "He endorses weight loss and fatigue", "He denies any liver or kidney disease", "He denies exacerbation of itch with water", "A KOH scraping is performed which is negative for scabies", "Physical examination is significant for numerous punctate and linear erosions consistent with excoriations as well as diffuse shoddy lymphadenopathy."], "s1": [0, 3, 4], "s2": [1, 2, 5]} {"key": 197, "questions": "Which of the following is the most likely diagnosis for the patient?", "options": [{"label": "A", "disease": "Secondary syphilis"}, {"label": "B", "disease": "Keratoderma climactericum"}, {"label": "C", "disease": "Pustular psoriasis"}, {"label": "D", "disease": "Keratoderma blenorrhagicum"}], "answer_idx": "D", "symptoms": ["A 34-year-old woman presents with scaly lesions on the palms and soles", "Lesions have been present for about a month", "She denies recent genital sores or history of sexually transmitted diseases", "She denies any family history of similar lesions on the palms or soles", "She denies exposure to arsenic or well water", "She denies being pregnant", "She is otherwise healthy aside from a recent bout of stomach flu which she thinks she got from a bad piece of chicken", "Physical examination reveals discrete scaly papules on the palms and soles."], "s1": [0, 7], "s2": [1, 2, 3, 4, 5, 6]} {"key": 198, "questions": "Which of the following is the most likely diagnosis for the patient?", "options": [{"label": "A", "disease": "Erythrasma"}, {"label": "B", "disease": "Hailey-Hailey Disease"}, {"label": "C", "disease": "Inverse psoriasis"}, {"label": "D", "disease": "Acanthosis nigricans"}], "answer_idx": "A", "symptoms": ["A 25-year-old male presents with 6 months of discoloration in the underarms", "He denies any blisters or erosions in the areas", "He denies any history of diabetes, insulin resistance, or weight gain", "He denies itch", "He denies discoloration only with sweating", "Physical examination reveals reddish-brown patches in the underarms; Wood's lamp reveals coral red fluorescence."], "s1": [2, 3], "s2": [0, 1, 4, 5]} {"key": 199, "questions": "Which of the following is the most likely diagnosis for the patient?", "options": [{"label": "A", "disease": "Morbilliform drug eruption"}, {"label": "B", "disease": "Stevens-Johnson Syndrome"}, {"label": "C", "disease": "Drug reaction with eosinophilia and systemic symptoms"}, {"label": "D", "disease": "Lichenoid drug eruption"}], "answer_idx": "C", "symptoms": ["A 72-year-old woman is seen by the Dermatology consult service for a rash which has developed during a prolonged 6-week hospitalization following a hemorrhagic stroke", "She is currently intubated, so history is limited, but per chart review, nurses documented the rash starting 5 days ago", "She has been experiencing high fevers as well as unexplained transaminitis over the last few days", "She has taken atorvastatin and pantoprazole for years, including during the hospitalization", "Following the stroke, she was started on carbamazepine for seizure prophylaxis", "She contracted MRSA pneumonia 2 weeks ago and has been on vancomycin since then", "She has also been on cefepime and micafungin for the last 5 days given the fevers and transaminitis", "Physical examination is significant for diffuse erythematous patches extending over the face, chest, back, and extremities, as well as facial edema."], "s1": [0, 1, 7], "s2": [2, 3, 4, 5, 6]} {"key": 200, "questions": "Which of the following is the most likely diagnosis for the patient?", "options": [{"label": "A", "disease": "Langerhans cell histiocytosis"}, {"label": "B", "disease": "Jacquet's dermatitis"}, {"label": "C", "disease": "Eczema coxsackium"}, {"label": "D", "disease": "Acrodermatitis enteropathica"}], "answer_idx": "D", "symptoms": ["A 2-month-old male infant presents with a rash in the diaper area", "It has been present for the last month", "He was seen by his pediatrician and was given topical triamcinolone as well as zinc oxide paste which did not help", "His mom says that she was unable to make enough breastmilk and so has recently been feeding him from a new formula", "Physical exam reveals erythematous erosions in the diaper area, as well as peri-oral scaling and a shiny tongue", "There is no rash on the scalp."], "s1": [0, 4, 5], "s2": [1, 2, 3]} {"key": 201, "questions": "Which of the following is the most likely diagnosis for the patient?", "options": [{"label": "A", "disease": "Atopic dermatitis"}, {"label": "B", "disease": "Psoriasis vulgaris"}, {"label": "C", "disease": "Lichen planus"}, {"label": "D", "disease": "Seborrheic dermatitis"}], "answer_idx": "B", "symptoms": ["A 25-year-old male presents with 2 years of pruriginous erythematous plaques with silver scales on elbows, knees and buttocks", "He says that his father is known for hand and back arthritis", "He denies personal and family history of dermatology conditions and atopy", "Physical exam reveals pitting on all 10 fingernails."], "s1": [0, 2], "s2": [1, 3]} {"key": 202, "questions": "Which of the following is the most likely diagnosis for the patient?", "options": [{"label": "A", "disease": "Venous ulcer"}, {"label": "B", "disease": "Arterial ulcer"}, {"label": "C", "disease": "Traumatic wound"}, {"label": "D", "disease": "Pyoderma grangrenosum"}], "answer_idx": "D", "symptoms": ["A 60-year-old female presents with a non-healing painful ulcer on her right leg for 4 months", "She says she first noticed it after she accidentally bumped into a chair", "She is known for rheumatoid arthritis and hypertension, and has taken methotrexate and bisoprolol for years", "According to the patient, oral antibiotics and wound debridement have only increase the size of the ulcer", "Physical exam reveals a 5x5 cm fibrinous ulcer with violaceous undermined borders on her right shin."], "s1": [1, 2], "s2": [0, 3, 4]} {"key": 203, "questions": "Which of the following is the most likely diagnosis for the patient?", "options": [{"label": "A", "disease": "IgA vasculitis"}, {"label": "B", "disease": "Bacterial endocarditis"}, {"label": "C", "disease": "Meningococcemia"}, {"label": "D", "disease": "Thrombocytopenic purpura"}], "answer_idx": "A", "symptoms": ["A 10-year-old male presents with non-blanchable palpable purple papules on both legs for a week", "Sore throat preceded the skin lesions", "His parents deny any other history of infectious diseases or new medications", "The boy complains about abdominal pain and trouble walking", "His vitals are stable", "Physical exam reveals red and swollen ankles on top of palpable purpura on both legs."], "s1": [0, 1, 3, 5], "s2": [2, 4]} {"key": 204, "questions": "Which of the following is the most likely diagnosis for the patient?", "options": [{"label": "A", "disease": "Stevens-Johnson Syndrome"}, {"label": "B", "disease": "Erythema multiforme major"}, {"label": "C", "disease": "Herpetic gingivostomatitis"}, {"label": "D", "disease": "Mycoplasma pneumoniae-induced rash and mucositis"}], "answer_idx": "D", "symptoms": ["A 12-year-old female presents with hemorrhagic crusts on the lips and erosions of the oral, genital, and conjunctival mucosa for 2 days", "According to the parents, there was a pneumonia outbreak at school", "The patient also had a high fever and cough a week before the onset of cutaneous lesions", "The parents deny any new medication and a history of allergies", "The physical exam reveals no other skin findings."], "s1": [0, 3, 4], "s2": [1, 2]} {"key": 205, "questions": "Which of the following is the most likely diagnosis for the patient?", "options": [{"label": "A", "disease": "Multiple endocrine neoplasia type 1"}, {"label": "B", "disease": "Birt-Hogg-Dube syndrome"}, {"label": "C", "disease": "Tuberous sclerosis complex"}, {"label": "D", "disease": "Neurofibromatosis type 1"}], "answer_idx": "C", "symptoms": ["A 16-year-old male presents with an increased number of perinasal dome-shaped smooth flesh-colored papules", "He is known to have epilepsy", "He denies family history of cancer", "The physical exam reveals hypopigmented macules on his back, three café-au-lait patches, and flesh-colored papules emerging from nail folds", "A biopsy of the facial lesions shows dermal proliferation of fibroblasts in a collagenous stroma associated with increased dilated blood vessels."], "s1": [1, 2], "s2": [0, 3, 4]} {"key": 206, "questions": "Which of the following is the most likely diagnosis for the patient?", "options": [{"label": "A", "disease": "Actinic keratosis"}, {"label": "B", "disease": "Psoriasis vulgaris"}, {"label": "C", "disease": "Sarcoidosis"}, {"label": "D", "disease": "Discoid lupus"}], "answer_idx": "D", "symptoms": ["A 40-year-old female complains about squamous papules on her ears for 10 years", "These lesions responded to topical corticosteroids but recurs when the treatment is stopped", "She denies any systemic symptoms", "Physical exam reveals erythematous scaly papules with signs of atrophy and scarring, as well as cicatricial alopecia", "Skin biopsy shows lichenoid interface dermatitis with hyperkeratosis and follicular plugging."], "s1": [2], "s2": [0, 1, 3, 4]} {"key": 207, "questions": "Which of the following is the most likely diagnosis for the patient?", "options": [{"label": "A", "disease": "Atopic dermatitis"}, {"label": "B", "disease": "Lichen planus"}, {"label": "C", "disease": "Lichenoid drug eruption"}, {"label": "D", "disease": "Prurigo nodular"}], "answer_idx": "B", "symptoms": ["A 30-year-old female presents with itchy skin lesions on her arms and legs", "She denies family history of atopy and psoriasis", "She does not take any medication and has no risk factors for hepatitis", "Review of systems came back negative", "Physical exam reveals flat purple papules and plaques on her wrists and shins as well as white plaques with with lacy patterns in the oral mucosa."], "s1": [1, 2, 3], "s2": [0, 4]} {"key": 208, "questions": "Which of the following is the most likely diagnosis for the patient?", "options": [{"label": "A", "disease": "Systemic lupus erythematosus"}, {"label": "B", "disease": "Dermatomyositis"}, {"label": "C", "disease": "Rosacea"}, {"label": "D", "disease": "Seborrheic dermatitis"}], "answer_idx": "A", "symptoms": ["A 40-year-old female presents with a recurrent skin eruption on her nose and cheeks for 2 years", "These lesions appears usually in the summer following excessive sun exposure, and disappears without scarring", "She has complains about arthritis and chronic fatigue", "She denies muscle weakness", "Physical exam reveals butterfly-shaped erythematous plaques over the malar area, non-scarring alopecia and oral ulcers."], "s1": [0, 1, 4], "s2": [2, 3]} {"key": 209, "questions": "Which of the following is the most likely diagnosis for the patient?", "options": [{"label": "A", "disease": "IgA vasculitis"}, {"label": "B", "disease": "Thrombocytopenic purpura"}, {"label": "C", "disease": "Chronic liver disease"}, {"label": "D", "disease": "Scurvy"}], "answer_idx": "D", "symptoms": ["A 22-year-old male presents with a follicular eruption that has been present for 2 weeks", "He has a history of anorexia nervosa and alcoholism", "There is no identifiable preceding incident related to the eruption", "He denies a history of atopy, drug use, and new medications", "A review of systems shows positive findings for joint pain and bleeding gums", "Physical exam reveals ecchymosis and purpuric follicular papules with corkscrew-like hair on his legs and arms."], "s1": [0, 2, 3], "s2": [1, 4, 5]} {"key": 210, "questions": "Which of the following is the most likely diagnosis for the patient?", "options": [{"label": "A", "disease": "Congenital melanocytic nevus"}, {"label": "B", "disease": "Spitz nevus"}, {"label": "C", "disease": "Mastocytoma"}, {"label": "D", "disease": "Xanthogranuloma"}], "answer_idx": "C", "symptoms": ["A 5-year-old female presents with a plaque on her left leg", "The lesion appeared a few months after her birth", "It has been increasing in size proportionally to her growth", "The parents mentions that the skin lesion sometimes becomes irritated, erythematous and itchy when the patient plays with it or when she gets sick", "Physical exam reveals a red-brown well delimited plaque on her left leg without any noticeable erythema or epidermal changes."], "s1": [0, 4], "s2": [1, 2, 3]} {"key": 211, "questions": "Which of the following is the most likely diagnosis for the patient?", "options": [{"label": "A", "disease": "Fragile X Syndrome"}, {"label": "B", "disease": "Neurofibromatosis Type I"}, {"label": "C", "disease": "Neurofibromatosis Type II"}, {"label": "D", "disease": "Ehlers Danlos Syndrome"}], "answer_idx": "B", "symptoms": ["A 12 year old male presents to clinic with concerns for new red bumps developing on the chest, arms, and legs over the past few months", " Patient states the lesions begin as small bumps that slowly grow in size and persist", "He denies any pain, itch, or symptoms associated with the lesions", "He has not tried any treatment and denies any recent travel", " The patient's mother states the patient also has a few light brown patches on the body, and some of her other family members had similar spots", "The patient is currently receiving extra classes for learning disability, and recently was evaluated by orthopaedics for spinal irregularities", " On physical examination, multiple skin colored papules that are able to be pushed in with a finger are spread across the chest, arms, and bilateral lower extremities", "Seven light tan-brown ovoid uniform patches are scattered throughout the trunk and legs", "The bilateral axilla are noted to have few scattered ovoid tan brown macules."], "s1": [0, 1, 2, 3, 6], "s2": [4, 5, 7, 8]} {"key": 212, "questions": "Which of the following is the most likely diagnosis for the patient?", "options": [{"label": "A", "disease": "Rombo syndrome"}, {"label": "B", "disease": "Nicolau-Balus syndrome"}, {"label": "C", "disease": "Bazex-Dupre-Christol syndrome"}, {"label": "D", "disease": "Conradi-Hünermann-Happle syndrome"}], "answer_idx": "D", "symptoms": ["A 6 month old male presents to dermatology clinic for evaluation of skin depressions on the arms and legs present for a few months", "The patient's mother denies any trauma that may have incited the depressions", "Lesions are not painful or itchy to the patient", "Mother denies any recent travel or treatments", " Past medical history is relevant for full term prenatal course", "At birth, patient was noted to be red across the entire body with thin flaky yellow skin attached to the back and chest which slowly healed", "Recently, patient was also evaluated by orthopaedics and completed a hand x-ray that showed epiphyseal stippling", " Physical examination is remarkable for multiple follicular based depressions a curvilinear distribution on the hands, legs, and feet on the hands, legs, and feet", " What is the most likely diagnosis?"], "s1": [0, 2, 3, 7], "s2": [1, 4, 5, 6, 8]} {"key": 213, "questions": "Which of the following is the most likely diagnosis for the patient?", "options": [{"label": "A", "disease": "Keloid"}, {"label": "B", "disease": "Dermatofibroma"}, {"label": "C", "disease": "Dermatofibrosarcoma protuberans"}, {"label": "D", "disease": "Squamous cell carcinoma"}], "answer_idx": "C", "symptoms": ["A 34 year old woman presents to clinic complaining of a painful, red spot on the left shoulder that developed about 4 years ago", "She denies any trauma that may have incited it", "It has been evaluated by multiple dermatologists and treated with intralesional corticosteroid injections, but it continues to enlargen and has become more raised", " Physical examination reveals a multilobular violaceous, poorly demarcated firm plaque that is tender to palpation on the left shoulder", "Histopathologic analysis of the lesion reveals lesional cells that stain positive for CD34 and negative for Factor 13a."], "s1": [0, 1, 2], "s2": [3, 4]} {"key": 214, "questions": "Which of the following is the most likely diagnosis for the patient?", "options": [{"label": "A", "disease": "Steven Johnson Syndrome"}, {"label": "B", "disease": "Fixed Drug Eruption"}, {"label": "C", "disease": "Morbilliform Drug Eruption"}, {"label": "D", "disease": "Erythema migrans"}], "answer_idx": "B", "symptoms": ["A 45-year-old-man with a past medical history of knee pain and hypertension complains of dark spot on the lower lip that develop whenever he takes certain oral medications", "He states the rash has been ongoing for several years, and usually begins as a pink spot with a darker brown-black center", "He denies any bleeding or ulceration at the site", " Current medications includes naproxen, multivitamins, and metoprolol", " Physical examination reveals a well demarcated red-purple patch with a darker red-violaceous center", "The patch is non-painful."], "s1": [0, 3], "s2": [1, 2, 4, 5]} {"key": 215, "questions": "Which of the following is the most likely diagnosis for the patient?", "options": [{"label": "A", "disease": "Intradermal nevus"}, {"label": "B", "disease": "Solar lentigo"}, {"label": "C", "disease": "Seborrheic keratosis"}, {"label": "D", "disease": "Melanoma"}], "answer_idx": "D", "symptoms": ["A 67-year-old-male farmer presents complaining of a growing mole on the right side of his cheek", " He states the mole was present for several years, but then started growing rapidly on one side a few weeks ago", "Sometimes the mole will bleed on its own", "He tries to avoid picking at the area", " Physical examination shows tan-brown ovoid irreuglarly shaped hyperpigmented macule with focal ulcearion", "Dermoscopy demonstrates a blue-gray veil in the center of the lesion."], "s1": [0, 1, 3], "s2": [2, 4, 5]} {"key": 216, "questions": "Which of the following is the most likely diagnosis for the patient?", "options": [{"label": "A", "disease": "Psoriasis"}, {"label": "B", "disease": "Atopic Dermatitis"}, {"label": "C", "disease": "Vitiligo"}, {"label": "D", "disease": "Tinea versicolor"}], "answer_idx": "C", "symptoms": ["A 13-year-old girl presents to the dermatology clinic with a light patch over both knees that the mom noticed a few months ago", "The spots have been growing in size", "The patient denies any pain at the site, and thinks she may have bumped her knee that caused the light patch", " Physical examination reveals well demarcated depigmented irregularly shaped patches on the bilateral anterior knees."], "s1": [0, 2], "s2": [1, 3]} {"key": 217, "questions": "Which of the following is the most likely diagnosis for the patient?", "options": [{"label": "A", "disease": "Erythema migrans"}, {"label": "B", "disease": "Tinea corporis"}, {"label": "C", "disease": "Psoriasis "}, {"label": "D", "disease": "Syphilis "}], "answer_idx": "A", "symptoms": ["A 45-year-old man with a past medical history of hypertension presents with concerns of a rash on his left lower back that his wife noticed two days ago", "The rash has been progressively enlargened", "Patient denies any pain or any prior treatments", "He thinks the surrounding skin is warm and may be infected, but he denies any drainage", " He reports that they went hiking eight days ago in Rhode Island", " Physical exam demonstrates a non-scaly large annular, pink-red patch on the left lower back with a bright pink center and clear central clearing."], "s1": [0, 1, 4], "s2": [2, 3, 5]} {"key": 218, "questions": "Which of the following is the most likely diagnosis for the patient?", "options": [{"label": "A", "disease": "Morbilliform eruption"}, {"label": "B", "disease": "Acute Generalized Exanthematous Pustulosis"}, {"label": "C", "disease": "Steven Johnsons Syndrome"}, {"label": "D", "disease": "Drug-induced hypersensitivity syndrome"}], "answer_idx": "B", "symptoms": ["A 54-year-old female is hospitalized for a left lower leg cellulitis and treated with ampicillin", "On day 4 of hospitalization, she developed a rash on the neck, under the arms that start as small yellow bumps but rapidly spread", "She denies any new lotions or creams", " She denies any any personal or family history of psoriasis", " Physical examination reveals multiple studded monomorphic yellow pustules on erythematous bright pink bases that coalesce into plaques across the flexural neck, bilateral axilla, and inframmary folds."], "s1": [0, 2, 3], "s2": [1, 4]} {"key": 219, "questions": "Which of the following is the most likely diagnosis for the patient?", "options": [{"label": "A", "disease": "Morbilliform eruption"}, {"label": "B", "disease": "Acute Generalized Exanthematous Pustulosis"}, {"label": "C", "disease": "Steven Johnsons Syndrome"}, {"label": "D", "disease": "Drug-induced hypersensitivity syndrome"}], "answer_idx": "C", "symptoms": ["A 30 year old male is admitted to the hospital for a rash that developed on the chest and then spread across the back, face, and legs", "He recently started lamotrigine two weeks ago for seizure prevention", "He denies any other recent medication changes", "He endorses significant pain at the site of the rash and also pain with urination", " Physical examination reveals diffuse pink erythematous patches of the chest, back with few scattered dusky central patches", "Applying pressure with a finger and moving laterally results in separation of the skin on many areas of the back", "A thin erosion is appreciated on the glans penis."], "s1": [0, 1, 2, 4, 5], "s2": [3, 6]} {"key": 220, "questions": "Which of the following is the most likely diagnosis for the patient?", "options": [{"label": "A", "disease": "Pemphigus foliaceous"}, {"label": "B", "disease": "Pemphigus vulgaris"}, {"label": "C", "disease": "Bullous pemphigoid"}, {"label": "D", "disease": "Bullous arthropod assault"}], "answer_idx": "B", "symptoms": ["A 66-year-old woman with a past medical history of hypertension and diabetes mellitus type 2 presents to clinic with a mildly painful, pruritic peeling skin on the chest and back that began 4 weeks ago", "She denies any new medications, new topical creams or lotions, or laundry detergents", "She has been applying vaseline to the spots", "She also thinks she may have a similar spot on the scalp", "She denies any similar spots in the mouth, and denies forming any blisters on the skin", " Physical exam demonstrates multiple thinly eroded pink-red oval plaques on the chest, scalp, back with yellow brown corn-flake like scale", "Ocular and oral mucosa are normal."], "s1": [0, 1, 2, 3], "s2": [4, 5, 6]} {"key": 221, "questions": "Which of the following is the most likely diagnosis for the patient?", "options": [{"label": "A", "disease": "Dermatofibrosarcoma protuberans"}, {"label": "B", "disease": "Basal cell carcinoma"}, {"label": "C", "disease": "Hypertrophic scar"}, {"label": "D", "disease": "Keloid"}], "answer_idx": "D", "symptoms": ["A 21-year-old African American woman presents to clinic with rough, firm spots on both earlobes that developed three months ago when she underwent ear piercings", "She has tried over the counter hydrocortisone without any improvement", " She notes that she has a history of multiple similar spots that developed after she injured herself, and multiple family members have similar spots", " On physical exam, thick violcaeous firm plaques are noted at the sites of ear piercings that extend beyond margin of the scar."], "s1": [1, 2], "s2": [0, 3]} {"key": 222, "questions": "Which of the following is the most likely diagnosis for the patient?", "options": [{"label": "A", "disease": "Impetigo"}, {"label": "B", "disease": "Hidradenitis suppurativa "}, {"label": "C", "disease": "Inflamed sebacous cysts"}, {"label": "D", "disease": "Abscesses"}], "answer_idx": "B", "symptoms": ["A 35-year-old man with a past medical history of obesity and tobacco use presents to clinic with a seven month history of boils under the arms and in the groin", "He reports that the boils come up twice a week under the arms, and once a week in the groin", "The individual boils are painful and will often drain foul odorous yellow green material", "Many of the boils have been drained in the emergency room", " Physical examination reveals multiple painful, tender subcutaneous nodules draining yellow-white malodorous pus in the bilateral axilla and inguinal folds", "Scattered double headed comedones are present with few sinus tracts noted."], "s1": [0, 1], "s2": [2, 3, 4, 5]} {"key": 223, "questions": "Which of the following is the most likely diagnosis for the patient?", "options": [{"label": "A", "disease": "Psoriasis"}, {"label": "B", "disease": "Seborrheic dermatitis"}, {"label": "C", "disease": "Rosacea"}, {"label": "D", "disease": "Acne vulgaris"}], "answer_idx": "B", "symptoms": ["A 45-year-old man presents to the primary care clinic with a red, itchy rash on the eyebrows and around the nose that has been ongoing for eight months", "He has tried moisturizers to the areas, but the flakiness returns a few days later", "He also reports he has dandruff in his scalp", "He denies any new facial products or using any other creams or lotions", " Physical examination reveals greasy yellow scale on poorly demarcated erythematous patches on the eyebrows and bilateral nasal alar creases", "Mild white scale is appreciated diffusely across the scalp."], "s1": [0, 3], "s2": [1, 2, 4, 5]} {"key": 224, "questions": "Which of the following is the most likely diagnosis for the patient?", "options": [{"label": "A", "disease": "Acrochordon"}, {"label": "B", "disease": "Confluent and reticulated papillomatosis "}, {"label": "C", "disease": "Acanthosis nigricans"}, {"label": "D", "disease": "Tinea versicolor"}], "answer_idx": "C", "symptoms": ["A 24-year-old obese woman presents to the dermatology clinic with darkening, rough skin around the neck for six months", "The rash is not painful or itchy", "She has tried to clean the area with over the counter shampoo and rubbing alcohol without improvement", "She denies any rash elsewhere on the body", "The rash always begins as dark patches, and she denies any blistering or redness before the dark spots develop", " Physical examination reveals thick, velvety ropey hyperpigmented rough, poorly demarcated plaques circumferentially around the neck."], "s1": [0, 5], "s2": [1, 2, 3, 4]} {"key": 225, "questions": "Which of the following is the most likely diagnosis for the patient?", "options": [{"label": "A", "disease": "Melasma"}, {"label": "B", "disease": "Acne vulgaris"}, {"label": "C", "disease": "Medication induced hyperpigmentation"}, {"label": "D", "disease": "Dermatosis papulosa nigra"}], "answer_idx": "D", "symptoms": ["A 65-year-old African American man presents to dermatology clinic for multiple non-painful, non-itchy brown spots on the face", "He notes they started developing when he was 42 years old, and they have increased in number and size", "Sometimes he tries to remove them with a needle, but they will bleed", " Physical exam reveals multiple dark, flat topped ovoid papules, and few pedunculated brown-black papules on the bilateral zygomatic cheeks."], "s1": [0, 3], "s2": [1, 2]} {"key": 226, "questions": "Which of the following is the most likely diagnosis for the patient?", "options": [{"label": "A", "disease": "Herpes virus infection"}, {"label": "B", "disease": "Basal cell carcinoma"}, {"label": "C", "disease": "Molluscum contagiosum"}, {"label": "D", "disease": "Fibrous papule"}], "answer_idx": "C", "symptoms": ["A 5-year-old boy presents to the pediatrician office with complaints of multiple spots that began on the abdomen and spread to the arms and legs", "The spots are not itchy or painful, unless he scratches them", "Sometimes the skin surronding the lesion will become very red and painful, but then the lesion and redness fade", "The parents report they have tried topical corticosteroids but the lesions do not go away", " Mom endorses a history of atopic dermatitis in her sister and herself", " Physical exam reveals multiple dome shaped skin colored to light pink papules with central umbilication and yellow core across the chest and back."], "s1": [4, 3], "s2": [0, 1, 2, 5]} {"key": 227, "questions": "Which of the following is the most likely diagnosis for the patient?", "options": [{"label": "A", "disease": "Allergic contact dermatitis"}, {"label": "B", "disease": "Dermatomyositis"}, {"label": "C", "disease": "Lupus erythematosus"}, {"label": "D", "disease": "Rheumatoid arthritis"}], "answer_idx": "B", "symptoms": ["A 65-year-old female presents to the dermatology clinic for evaluation of eyelid redness and neck rash ongoing for four months that began after she started a new body lotion", "She reports increasing fatigue when climbing stairs and lifting heavy items over her head", "She has not tried any topical treatments", "She denies any recent medication changes, fevers, chills, or recent travel", " Physical examination reveals poorly demarcated patchy pink erythema affecting the periocular areas with a bright pink poorly demarcated patch diffusely across the upper back and shoulders in the shape of a V", "The bilateral dorsal hands reveals thin pink rough plaques on the knuckle joints."], "s1": [1, 3], "s2": [0, 2, 4, 5]} {"key": 228, "questions": "Which of the following is the most likely diagnosis for the patient?", "options": [{"label": "A", "disease": "Xanthelasma"}, {"label": "B", "disease": "Necrobiotic xanthogranuloma (NXG) "}, {"label": "C", "disease": "Necrobiosis lipoidica diabeticorum"}, {"label": "D", "disease": "Hidrocystoma "}], "answer_idx": "B", "symptoms": ["A 53-year-old man with a past medical history of multiple myeloma presents to dermatology clinic with multiple yellow, irritated spots under the eyes", "He reports the lesions developed about four months ago, beginning as a red spot that will enlargen, bleed, and then scab over with yellow material", "He denies any new facial creams, topical treatments, or new medications", "He does not have any new dogs or cats", " Physical examination is remarkable for hepatosplenomegaly and on the lower eyelids bilaterally are yellow-pink thinly ulcerated plaques with yellow crust."], "s1": [0, 1, 4], "s2": [2, 3]} {"key": 229, "questions": "Which of the following is the most likely diagnosis for the patient?", "options": [{"label": "A", "disease": "Scabies"}, {"label": "B", "disease": "Poison ivy allergic contact dermatitis"}, {"label": "C", "disease": "Irritant contact dermatitis"}, {"label": "D", "disease": "Nummular eczema"}], "answer_idx": "B", "symptoms": ["A 34-year-old woman presents with complaints of a severely itchy rash affecting both lower legs that developed three days ago after returning from camping six days ago", "She reports during the camping trip they often hiked for several hours a day, and she wore shorts due to the humid, hot weather", "When the rash began, it began as small red bumps that formed larger fluid filled bumps", "She has tried over the counter cortisone that helps with the itch a little bit", "She denies any new topical lotions, laundry detergents, or body washes", " Physical exam reveals multiple streaks of yellow vesicles on bright pink erythematous bases in a linear distribution on the bilateral lower extremities."], "s1": [0, 2, 5], "s2": [1, 3, 4]} {"key": 230, "questions": "Which of the following is the most likely diagnosis for the patient?", "options": [{"label": "A", "disease": "Keratoacanthoma"}, {"label": "B", "disease": "Dermatofibroma"}, {"label": "C", "disease": "Keratosis pilaris"}, {"label": "D", "disease": "Fibrous papule"}], "answer_idx": "B", "symptoms": ["A 22-year-old female presents with a brown rough spot on the left shin which she noted when shaving her leg", "She thinks it may have been there for a few weeks", "She denies any pain, pruritus, or spontaneous bleeding from the site", "She cannot recall any trauma that may have caused the lesion", " She denies any personal or family history of skin cancer", " Physical exam reveals a thin, firm brown dome shaped papule on the left shin that invaginates upon pinching."], "s1": [0, 1, 2, 3, 4], "s2": [5]} {"key": 231, "questions": "Which of the following is the most likely diagnosis?", "options": [{"label": "A", "disease": "Cyclic vomiting syndrome"}, {"label": "B", "disease": "Gastroenteritis"}, {"label": "C", "disease": "Hypertrophic pyloric stenosis"}, {"label": "D", "disease": "Gastroesophageal reflux disease"}], "answer_idx": "A", "symptoms": ["A 5-year-old girl is brought to the emergency department by her mother because of multiple episodes of nausea and vomiting that last about 2 hours.", "During this period, she has had 6–8 episodes of bilious vomiting and abdominal pain.", "The vomiting was preceded by fatigue.", "The girl feels well between these episodes.", "She has missed several days of school and has been hospitalized 2 times during the past 6 months for dehydration due to similar episodes of vomiting and nausea.", "The patient has lived with her mother since her parents divorced 8 months ago.", "Her immunizations are up-to-date.", "She is at the 60th percentile for height and 30th percentile for weight.", "She appears emaciated.", "Her temperature is 36.8°C (98.8°F), pulse is 99/min, and blood pressure is 82/52 mm Hg.", "Examination shows dry mucous membranes.", "The lungs are clear to auscultation.", "Abdominal examination shows a soft abdomen with mild diffuse tenderness with no guarding or rebound.", "The remainder of the physical examination shows no abnormalities."], "s1": [0, 1, 2, 3, 4, 5], "s2": [6, 7, 8, 9, 10, 11, 12, 13]} {"key": 232, "questions": "Which of the following is the most likely diagnosis?", "options": [{"label": "A", "disease": "Exertional heat stroke"}, {"label": "B", "disease": "Neuroleptic malignant syndrome"}, {"label": "C", "disease": "Non-exertional heat stroke"}, {"label": "D", "disease": "Septic shock"}], "answer_idx": "C", "symptoms": ["A 67-year-old man presents to the emergency department with a fever and altered mental status.", "The patient has a history of Alzheimer dementia and is typically bed bound.", "His son found him confused with a warm and flushed complexion thus prompting his presentation.", "The patient has a past medical history of dementia, diabetes, and hypertension and typically has a visiting home nurse come to administer medications.", "Prior to examination, he is given haloperidol and diphenhydramine as he is combative and will not allow the nurses near him.", "His temperature is 102.9°F (39.4°C), blood pressure is 104/64 mmHg, pulse is 170/min, respirations are 22/min, and oxygen saturation is 100% on room air.", "Physical exam is notable for dry and flushed skin and a confused man.", "There is no skin breakdown, and flexion of the patient’s neck elicits no discomfort.", "Laboratory studies are drawn as seen below. | | Hemoglobin: 15 g/dL | Hematocrit: 45% | Leukocyte count: 4,500/mm^3 with normal differential | Platelet count: 227,000/mm^3 | | Serum: | Na+: 139 mEq/L | Cl-: 100 mEq/L | K+: 4.3 mEq/L | HCO3-: 24 mEq/L | BUN: 30 mg/dL | Glucose: 97 mg/dL | Creatinine: 1.5 mg/dL | Ca2+: 10.2 mg/dL | AST: 12 U/L | ALT: 10 U/L | | Urine: | Color: Yellow | Bacteria: Absent | Nitrites: Negative | Red blood cells: Negative | | An initial chest radiograph is unremarkable.", "The patient is given 3 liters of Ringer's lactate and an electric fan to cool off.", "Two hours later, his temperature is 99°F (37.2°C), blood pressure is 154/94 mmHg, pulse is 100/min, respirations are 17/min, and oxygen saturation is 100% on room air.", "The patient’s mental status is at the patient’s baseline according to the son."], "s1": [0, 1, 2, 3, 6, 7], "s2": [4, 5, 8, 9, 10, 11]} {"key": 233, "questions": "Which of the following represents the most likely etiology of her anemia.", "options": [{"label": "A", "disease": "Vitamin B12 deficiency"}, {"label": "B", "disease": "Folate deficiency"}, {"label": "C", "disease": "Iron deficiency"}, {"label": "D", "disease": "Intravascular hemolysis"}], "answer_idx": "C", "symptoms": ["A 41-year-old woman presents to her primary care physician with complaints of fatigue and weakness.", "She denies any personal history of blood clots or bleeding problems in her past, but she says that her mother has had to be treated for breast cancer recently and is starting to wear her down.", "Her past medical history is significant for preeclampsia, hypertension, polycystic ovarian syndrome, and hypercholesterolemia.", "She currently smokes 1 pack of cigarettes per day, drinks a glass of wine per day, and currently denies any illicit drug use.", "Her vital signs include: temperature, 36.7°C (98.0°F); blood pressure, 126/74 mm Hg; heart rate, 111/min; and respiratory, rate 23/min.", "On physical examination, her pulses are bounding and irregular, complexion is pale, but breath sounds remain clear.", "On examination, the physician finds diffuse skin pallor and orders a complete blood count.", "Her laboratory data demonstrate a hematocrit of 27.1%, MCV of 79 fL, and a reticulocyte count of 2.0%.", "The patient is diagnosed with anemia."], "s1": [0, 1, 2, 3], "s2": [4, 5, 6, 7, 8]} {"key": 234, "questions": "What is the most likely cause of his condition?", "options": [{"label": "A", "disease": "Monomorphic ventricular tachycardia"}, {"label": "B", "disease": "Mitral regurgitation"}, {"label": "C", "disease": "Third-degree heart block"}, {"label": "D", "disease": "Acute pericarditis"}], "answer_idx": "A", "symptoms": ["A 56-year-old man with known coronary artery disease presents to the emergency department complaining of chest discomfort and palpitations for 2 hours.", "On arrival, the vital signs include blood pressure 122/76 mm Hg, heart rate 180/min, respiratory rate 22/min, temperature 37.0℃ (98.6℉), and blood oxygen saturation (SpO2) 98% on room air.", "A 12-lead electrocardiogram demonstrated ST-segment elevation in the anterolateral leads.", "The troponin level was 0.8 ng/mL (normal 0–0.4 ng/mL).", "The patient declined primary percutaneous intervention and was treated with antifibrinolytics in the coronary care unit.", "After 1 hour of treatment, the patient loses consciousness and the blood pressure falls to 60/40 mm Hg.", "Cardiac monitoring shows the electrocardiogram (ECG) pattern in lead 2 shown below."], "s1": [0, 1, 2, 3, 4], "s2": [5, 6]} {"key": 235, "questions": "Which of the following is the most likely diagnosis in this patient?", "options": [{"label": "A", "disease": "Reactive attachment disorder"}, {"label": "B", "disease": "Conduct disorder"}, {"label": "C", "disease": "Antisocial personality disorder"}, {"label": "D", "disease": "Oppositional defiant disorder"}], "answer_idx": "D", "symptoms": ["A 16-year-old female high school student is brought to the physician by her parents for her repeated behavioral problems at home and school during the past 10 months.", "Her teachers describe her behavior as uncooperative and disruptive as she persistently refuses to answer questions, insults her teachers, and annoys her classmates on a daily basis.", "At home, her parents try to address her frequent violations of curfew, but attempts at discussing the issue often result in their daughter losing her temper and screaming at her parents.", "Her grades have deteriorated over the past year.", "She has no history of psychiatric illness.", "On questioning, the patient refuses to answer and frequently disrupts the physician’s conversation with the parents."], "s1": [0, 1, 2, 5], "s2": [3, 4]} {"key": 236, "questions": "What is the most likely etiology of this patient’s presentation?", "options": [{"label": "A", "disease": "Patent processus vaginalis"}, {"label": "B", "disease": "Collection of fluid in the tunica vaginalis"}, {"label": "C", "disease": "Infection of the epididymis"}, {"label": "D", "disease": "Neoplasm of the testicle"}], "answer_idx": "A", "symptoms": ["A 4-year-old boy presents to the office with his mother.", "She states that the patient has been complaining of pain in his scrotum with swelling, abdominal pain, and nausea for the last 2 or 3 days.", "On exam, the abdomen is soft and nontender to palpation.", "The right scrotal sac is mildly enlarged without erythema.", "A tender mass is palpated in the right scrotal area.", "The mass does not transilluminate when a penlight is applied.", "The patient is afebrile and all vital signs are stable."], "s1": [0, 6], "s2": [1, 2, 3, 4, 5]} {"key": 237, "questions": "What is the most likely diagnosis?", "options": [{"label": "A", "disease": "Adjustment disorder with depressive features"}, {"label": "B", "disease": "Bipolar disorder"}, {"label": "C", "disease": "Cyclothymia"}, {"label": "D", "disease": "Dysthymia"}], "answer_idx": "D", "symptoms": ["A 59-year-old man presents to his primary care physician, accompanied by his wife, who requests treatment for his “chronic pessimism.” The patient admits to feeling tired and “down” most of the time for the past several years but insists that it is “just part of getting old.” His wife believes that he has become more self-critical and less confident than he used to be.", "Neither the patient nor his wife can identify any stressors or triggering events.", "He has continued to work as a librarian at a nearby college during this time and spends time with friends on the weekends.", "He sleeps 7 hours per night and eats 3 meals per day.", "He denies suicidal ideation or periods of elevated mood, excessive irritability, or increased energy.", "Physical exam reveals a well-dressed, well-groomed man without apparent abnormality.", "Basic neurocognitive testing and labs (CBC, BMP, TSH, cortisol, testosterone, and urine toxicology) are within normal limits."], "s1": [0, 1, 4], "s2": [2, 3, 5, 6]} {"key": 238, "questions": "Which of the following is the most likely diagnosis?", "options": [{"label": "A", "disease": "Infectious colitis"}, {"label": "B", "disease": "Bowel adhesions"}, {"label": "C", "disease": "Volvulus"}, {"label": "D", "disease": "Acute diverticulitis"}], "answer_idx": "C", "symptoms": ["ِA 62-year-old man comes to the emergency department because of colicky pain in the lower abdomen and abdominal distension for the past 12 hours.", "He has chronic constipation for which he takes lactulose and senna laxatives.", "His temperature is 37.1°C (98.7°F), blood pressure is 110/60 mm Hg and pulse is 85/min.", "On physical examination, there is diffuse abdominal distension and tenderness, and bowel sounds are faint.", "His plain abdominal radiograph is shown."], "s1": [2, 4], "s2": [0, 1, 3]} {"key": 239, "questions": "What is the most likely explanation for the patient’s presentation?", "options": [{"label": "A", "disease": "Child abuse"}, {"label": "B", "disease": "Defective type 1 collagen gene"}, {"label": "C", "disease": "Low levels of phosphate"}, {"label": "D", "disease": "Low levels of vitamin D"}], "answer_idx": "B", "symptoms": ["A 17-month-old girl was brought to the emergency department by her mom following a fall.", "The mom reports that the patient was playing in the playground when she tripped and fell onto the mulch.", "She had an uncomplicated birth history and has been meeting developmental goals except for language delay, for which she is to receive a hearing test for further evaluation next week.", "Physical exam reveals bruising along the left lateral thigh, knee, and elbow; all lab tests are within normal limits.", "Radiograph shows a fracture of the olecranon.", "Serum chemistry and liver panels were within normal limits."], "s1": [0, 1, 3, 4], "s2": [2, 5]} {"key": 240, "questions": "Which of the following is the most likely underlying cause of this patient's findings?", "options": [{"label": "A", "disease": "Fracture of the femoral neck"}, {"label": "B", "disease": "Inflammation of the hip synovium"}, {"label": "C", "disease": "Abnormal development of the acetabulum"}, {"label": "D", "disease": "Displacement of the femoral epiphysis"}], "answer_idx": "C", "symptoms": ["A 3855-g (8-lb 8-oz) newborn is examined shortly after birth.", "She was delivered at 40 weeks' gestation by cesarean delivery because of breech presentation.", "Pregnancy was otherwise uncomplicated.", "Physical examination shows asymmetric thigh creases.", "The left leg is shorter than the right leg and positioned in external rotation."], "s1": [0, 1, 2], "s2": [3, 4]} {"key": 241, "questions": "Which of the following is the most likely cause of this patient's anemia?", "options": [{"label": "A", "disease": "Vitamin B12 deficiency"}, {"label": "B", "disease": "Zinc deficiency"}, {"label": "C", "disease": "Gestational anemia"}, {"label": "D", "disease": "Thalassemia trait"}], "answer_idx": "D", "symptoms": ["A 27-year-old G1P0 woman at 9 weeks estimated gestational age presents for a prenatal visit.", "She has no current complaints and takes no medications.", "She is vegetarian and emigrated from Nepal 7 years ago.", "She does not use tobacco, alcohol or recreational drugs.", "The patient’s vital signs include: blood pressure 111/95 mm Hg, temperature 36.7°C (98.6°F), pulse 88/min.", "Laboratory results are significant for the following: | Hemoglobin 10.2 g/dL | Erythrocyte count 5.5 million/mm3 | Mean corpuscular volume 65 μm3 | Mean corpuscular hemoglobin 21 pg/cell | Red cell distribution width 13.5% (ref: 11.5–14.5%) | Serum ferritin 170 ng/mL | ."], "s1": [0, 1, 2, 3], "s2": [4, 5]} {"key": 242, "questions": "Which of the following is the most likely cause of this patient's symptoms?", "options": [{"label": "A", "disease": "Cardiac tamponade"}, {"label": "B", "disease": "Constrictive pericarditis"}, {"label": "C", "disease": "Reinfarction"}, {"label": "D", "disease": "Dressler syndrome"}], "answer_idx": "D", "symptoms": ["A 55-year-old man comes to the emergency department because of a dry cough and severe chest pain beginning that morning.", "Two months ago, he was diagnosed with inferior wall myocardial infarction and was treated with stent implantation of the right coronary artery.", "He has a history of hypertension and hypercholesterolemia.", "His medications include aspirin, clopidogrel, atorvastatin, and enalapril.", "His temperature is 38.5°C (101.3°F), pulse is 92/min, respirations are 22/min, and blood pressure is 130/80 mm Hg.", "Cardiac examination shows a high-pitched scratching sound best heard while sitting upright and during expiration.", "The remainder of the examination shows no abnormalities.", "An ECG shows diffuse ST elevations.", "Serum studies show a troponin I of 0.2 ng/mL (N < 0.01)."], "s1": [0, 4, 5, 7], "s2": [1, 2, 3, 6, 8]} {"key": 243, "questions": "Of the following organisms, infection with which one is most consistent with her symptoms?", "options": [{"label": "A", "disease": "Plasmodium falciparum"}, {"label": "B", "disease": "Schistosoma mansoni"}, {"label": "C", "disease": "Mycobacterium tuberculosis"}, {"label": "D", "disease": "Trypanosoma brucei rhodesiense"}], "answer_idx": "B", "symptoms": ["A 22-year old active duty soldier presents to your clinic with complaints of intense fatigue, fevers, abdominal pain, and a nonproductive cough.", "Her symptoms began a few days ago, but she returned from a tour of duty in Malawi and Mozambique four weeks ago.", "She endorses sleeping outside, using minimal bug spray, and swimming in rivers during her tour.", "On physical exam, her temperature is 101.5, and she appears exhausted.", "She has a hive-like rash scattered over her body.", "Her abdominal exam is notable for hepatosplenomegaly, and her lung exam is notable for scattered crackles.", "Her CBC with diff is remarkable for marked eosinophilia, and she has an elevated IgE."], "s1": [0, 1, 2, 3], "s2": [4, 5, 6]} {"key": 244, "questions": "Which of the following is the primary cause of the patient’s lab results?", "options": [{"label": "A", "disease": "Hematologic malignancy"}, {"label": "B", "disease": "Immune-mediated destruction"}, {"label": "C", "disease": "Mechanical shearing"}, {"label": "D", "disease": "Viral infection"}], "answer_idx": "B", "symptoms": ["A 46-year-old woman presents to the emergency department with progressive dyspnea and chest pain.", "She reports that her symptoms started 1 week ago and have gotten progressively worse.", "The chest pain is left-sided and is exacerbated by coughing or deep breaths.", "She also endorses a 6-month history of joint pains involving her knees, elbows, and digits.", "She does not have a significant medical or surgical history.", "She takes ibuprofen as needed.", "She works as a pre-school teacher.", "The patient’s temperature is 99°F (37.2°C), blood pressure is 120/78 mmHg, pulse is 89/min, and respirations are 17/min with an oxygen saturation of 93% on room air.", "On physical examination, a friction rub upon inspiration/expiration and crackles are appreciated at the base of the left lung.", "She has an erythematous rash that spans the bilateral cheeks and nose.", "There are also scattered ecchymoses on her arms and legs.", "A chest radiograph shows a small left-sided pleural effusion.", "A complete blood count is obtained, as shown below: | | Hemoglobin: 9 g/dL | Hematocrit: 28% | Leukocyte count: 1,500/mm^3 with normal differential | Platelet count: 80,000/mm^3 | | A urinalysis shows elevated protein levels.", "Serologic antibodies are pending."], "s1": [0, 1, 2, 7, 8, 11], "s2": [3, 4, 5, 6, 9, 10, 12, 13]} {"key": 245, "questions": "What is the most likely diagnosis?", "options": [{"label": "A", "disease": "Giardiasis"}, {"label": "B", "disease": "Irritable bowel syndrome (IBS)"}, {"label": "C", "disease": "Traveler’s diarrhea due to Norovirus"}, {"label": "D", "disease": "Traveler’s diarrhea due to ETEC"}], "answer_idx": "D", "symptoms": ["A 31-year-old male presents with a 2-day history of watery diarrhea that began 8 days after his arrival from Africa.", "None of the other members of his family became ill.", "He reports symptoms of malaise, anorexia, and abdominal cramps followed by watery diarrhea.", "He denies tenesmus, urgency, and bloody diarrhea.", "His temperature is 98.6°F (37°C), respiratory rate is 15/min, pulse is 107/min, and blood pressure is 90/68 mm Hg.", "A physical examination is performed and is within normal limits.", "Intravenous fluids are started, and a stool sample is sent to the lab, which comes out to be negative for any ova/parasites, blood cells, or pus cells."], "s1": [0, 2, 3, 6], "s2": [1, 4, 5]} {"key": 246, "questions": "What is the most logical diagnostic test for this patient?", "options": [{"label": "A", "disease": "Echocardiography"}, {"label": "B", "disease": "Holter monitoring"}, {"label": "C", "disease": "Lung biopsy"}, {"label": "D", "disease": "Transbronchial biopsy"}], "answer_idx": "A", "symptoms": ["A 25-year-old woman complains of dyspnea and mild chest pain on exertion, which increases gradually with continued exertion.", "She had similar symptoms last year and her medical record included the following arterial blood gas findings: | pH 7.51 | pO2 77 mm Hg | pCO2 32 mm Hg | An ECG last year demonstrated a right axis deviation.", "The current chest X-ray showed enlarged pulmonary arteries but no parenchymal infiltrates.", "A lung perfusion scan revealed a low probability for pulmonary thromboembolism.", "The current ECG showed right heart strain but no evidence of primary cardiac disease."], "s1": [0, 3], "s2": [1, 2, 4]} {"key": 247, "questions": "Which of the following is the most likely diagnosis?", "options": [{"label": "A", "disease": "Rubella"}, {"label": "B", "disease": "Allergic drug reaction"}, {"label": "C", "disease": "Scarlet fever"}, {"label": "D", "disease": "Erythema infectiosum"}], "answer_idx": "A", "symptoms": ["A 4-year-old boy is brought to the physician because of a progressive rash for 2 days.", "The rash started behind the ears and now involves the trunk and extremities.", "Over the past 4 days, he has had mild sore throat, red, itchy eyes, and headache.", "He was born at term and has been healthy except for recurrent episodes of tonsillitis and occasional asthma attacks.", "Two weeks ago, he was treated for tonsillitis that resolved with penicillin therapy.", "He immigrated with his family from Brazil 3 weeks ago.", "His immunization status is unknown.", "The patient appears weak and lethargic.", "His temperature is 38°C (100.4°F), pulse is 100/min, and blood pressure is 100/60 mm Hg.", "Examination shows postauricular and suboccipital lymphadenopathy.", "There is a non-confluent, pink maculopapular rash over the torso and extremities.", "His palms and soles show no abnormalities."], "s1": [0, 1, 2, 10, 11], "s2": [3, 4, 5, 6, 7, 8, 9]} {"key": 248, "questions": "The child likely has:", "options": [{"label": "A", "disease": "Severe combined immunodeficiency syndrome"}, {"label": "B", "disease": "DiGeorge syndrome"}, {"label": "C", "disease": "Isolated IgA deficiency"}, {"label": "D", "disease": "Common variable immunodeficiency"}], "answer_idx": "A", "symptoms": ["A 3-year-old Cuban-American male has a history of recurrent Pseudomonas and Candida infections.", "Laboratory analysis reveals no electrolyte abnormalities.", "Examination of his serum shows decreased levels of IgG and CT scan reveals the absence of a thymus."], "s1": [0, 2], "s2": [1]} {"key": 249, "questions": "Which of the following is the most likely diagnosis?", "options": [{"label": "A", "disease": "Cerebral venous thrombosis"}, {"label": "B", "disease": "Hypertensive emergency"}, {"label": "C", "disease": "Subarachnoid hemorrhage"}, {"label": "D", "disease": "Viral meningitis"}], "answer_idx": "A", "symptoms": ["A 38-year-old woman comes to the emergency department because of progressive headache, blurry vision, and nausea for 1 day.", "Four days ago, she was diagnosed with a right middle ear infection.", "She appears lethargic.", "Her temperature is 39.1°C (102.3°F), and blood pressure is 148/95 mm Hg.", "Ophthalmologic examination shows bilateral swelling of the optic disc.", "The corneal reflex in the right eye is absent.", "Sensation to touch is reduced on the upper right side of the face.", "Serum studies show increased concentrations of fibrin degradation products."], "s1": [0, 1, 2, 3, 5, 6], "s2": [4, 7]} {"key": 250, "questions": "Which of the following is the most likely underlying cause of the patient's condition?\"", "options": [{"label": "A", "disease": "Glycoprotein Ib deficiency"}, {"label": "B", "disease": "Vitamin K deficiency"}, {"label": "C", "disease": "Von Willebrand factor deficiency"}, {"label": "D", "disease": "ADAMTS13 deficiency"}], "answer_idx": "A", "symptoms": ["A 7-year-old girl is brought to the physician for evaluation of recurrent epistaxis.", "Her mother reports that she bruises easily while playing.", "Her pulse is 89/min and blood pressure is 117/92 mm Hg.", "Examination shows multiple bruises in the upper and lower extremities.", "Laboratory studies show: | Platelet count 100,000/mm3 | Prothrombin time 12 seconds | Partial thromboplastin time 33 seconds | Bleeding time 13 minutes | A peripheral blood smear shows enlarged platelets.", "Ristocetin assay shows no platelet aggregation."], "s1": [2], "s2": [0, 1, 3, 4, 5]} {"key": 251, "questions": "Which of the following disorders does this patient potentially suffer from?", "options": [{"label": "A", "disease": "Schizoid personality disorder"}, {"label": "B", "disease": "Narcissistic personality disorder"}, {"label": "C", "disease": "Obsessive compulsive personality disorder"}, {"label": "D", "disease": "Paranoid personality disorder"}], "answer_idx": "C", "symptoms": ["A 30-year-old male biology graduate student was dismissed from his PhD program after 8 years because he was not able to produce a thesis, claiming that his data was never exactly how he wanted it.", "He would spend weeks planning out a simple experiment, since everything had to be just right.", "For many experiments, he would start over because he felt he went out of order in adding the reagents to his media for his cells.", "He has had similar problems in his undergraduate courses, often failing to complete his assignments on time because he had to revise them until they were perfect."], "s1": [0], "s2": [1, 2, 3]} {"key": 252, "questions": "Arterial blood gas analysis on room air shows: | pH 7.56 | PCO2 40 mm Hg | PO2 94 mm Hg | HCO3- 30 mEq/L | Measurement of which of the following is the most appropriate next step in diagnosis?\"", "options": [{"label": "A", "disease": "Serum osmolal gap"}, {"label": "B", "disease": "Serum anion gap"}, {"label": "C", "disease": "Urine albumin to urine creatinine ratio"}, {"label": "D", "disease": "Urine chloride concentration"}], "answer_idx": "D", "symptoms": ["A 24-year-old primigravid woman at 8 weeks' gestation is brought to the emergency department because of a 5-day history of nausea and vomiting.", "She has not been able to tolerate much food or drink.", "Her symptoms are worse in the morning.", "She has tried multiple oral antiemetics with limited relief.", "She has not had fevers, chills, abdominal pain, urinary symptoms, or diarrhea.", "She appears tired.", "Her temperature is 37°C (98.6°F), pulse is 105/min, and blood pressure is 108/60 mm Hg.", "Examination shows dry mucous membranes and cool extremities, with delayed capillary refill time."], "s1": [0, 1, 2, 3], "s2": [4, 5, 6, 7]} {"key": 253, "questions": "Which of the following is the most likely cause of these findings?\"", "options": [{"label": "A", "disease": "Malignant biliary tract obstruction"}, {"label": "B", "disease": "Acetaminophen-induced liver injury"}, {"label": "C", "disease": "Hemolysis"}, {"label": "D", "disease": "Chronic pancreatitis"}], "answer_idx": "A", "symptoms": ["A 64-year-old man comes to the physician because of a 2-week history of intermittent epigastric discomfort.", "He reports that his urine has been very dark, and his stools have been pale for the past week.", "His appetite has decreased, and he has had a 4.5-kg (10-lb) weight loss during this period.", "He has smoked 1 pack of cigarettes daily for 30 years.", "He drinks a few shots of vodka daily.", "He has no history of severe illness.", "He has chronic left knee pain, for which he takes acetaminophen.", "Vital signs are within normal limits.", "Examination shows jaundice of the skin and scleral icterus.", "There are scratch marks on the extremities.", "Abdominal examination shows a nontender, palpable mass in the right upper quadrant.", "The remainder of the examination is normal.", "Laboratory studies show: | Hemoglobin 11.6 g/dL | Leukocyte count 8,700/mm3 | Platelet count 172,000/mm3 | Serum | Urea nitrogen 17 mg/dL | Creatinine 1.1 mg/dL | Bilirubin | Total 6 mg/dL | Direct 5.2 mg/dL | Alkaline phosphatase 220 IU/L | Ultrasonography shows dilated extrahepatic and pancreatic ducts and a distended gall bladder."], "s1": [0, 1, 2, 8, 9, 10, 12], "s2": [3, 4, 5, 6, 7, 11]} {"key": 254, "questions": "Which of the following is the most likely diagnosis?", "options": [{"label": "A", "disease": "Hypertensive urgency"}, {"label": "B", "disease": "Hypertensive emergency"}, {"label": "C", "disease": "Aortic aneurysm"}, {"label": "D", "disease": "Malignant hypertension"}], "answer_idx": "B", "symptoms": ["A 55-year-old African American man presents to the emergency department with central chest pressure.", "His symptoms started 1 day before.", "The pain was initially intermittent in nature but has become constant and radiates to his jaw and left shoulder.", "He also complains of some difficulty breathing.", "The patient was diagnosed with essential hypertension 1 year ago, but he is not taking any medications for it.", "The patient denies smoking, alcohol, or drug use.", "Family history is unremarkable.", "His blood pressure is 230/130 mm Hg in both arms, the temperature is 36.9°C (98.4°F), and the pulse is 90/min.", "ECG shows diffuse T wave inversion and ST depression in lateral leads.", "Laboratory testing is significant for elevated troponin."], "s1": [0, 2, 3, 4, 7, 8, 9], "s2": [1, 5, 6]} {"key": 255, "questions": "Which of the following is the most likely diagnosis in this patient?", "options": [{"label": "A", "disease": "Joint hypermobility syndrome"}, {"label": "B", "disease": "Sturge-Weber syndrome"}, {"label": "C", "disease": "Ehlers-Danlos syndrome"}, {"label": "D", "disease": "Marfan syndrome"}], "answer_idx": "D", "symptoms": ["A 19-year-old female college student comes to the physician for a sudden loss of visual acuity of her right eye.", "She noticed that she was unable to read the time on the alarm clock when she woke up in the morning.", "When she closes her right eye, she is able to see sharply.", "When she closes her left eye, she has blurry double vision.", "She does not recall trauma to her eye but has been working long nights on her honors thesis.", "She has a history of occasional shoulder luxation.", "She is 180 cm (5 ft 11 in) tall, and weighs 62 kg (136 lbs); her BMI is 19.1 kg/m2.", "Her vital signs are within normal limits.", "On physical examination, slender and long fingers are noted.", "She has several flat, demarcated brownish nevi on her left cheek.", "Ocular examination shows upward temporal subluxation of her right lens."], "s1": [0, 1, 2, 3, 4, 10], "s2": [5, 6, 7, 8, 9]} {"key": 256, "questions": "Which of the following is the most likely underlying cause of these findings?\"", "options": [{"label": "A", "disease": "Parvovirus B19"}, {"label": "B", "disease": "Medication-induced hemolysis"}, {"label": "C", "disease": "Defect in erythrocyte membrane proteins"}, {"label": "D", "disease": "Hemolytic crisis"}], "answer_idx": "A", "symptoms": ["A 14-year-old boy is brought to the emergency department because of a 2-day history of fatigue.", "He reports that during this time he has had occasional palpitations and shortness of breath.", "He has sickle cell disease.", "Current medications include hydroxyurea and folic acid.", "He appears fatigued.", "His temperature is 38.3°C (100.9°F), pulse is 120/min, respirations are 24/min, and blood pressure is 112/74 mm Hg.", "Examination shows pale conjunctivae.", "Cardiac examination shows a midsystolic ejection murmur.", "Laboratory studies show: | Hemoglobin 6.4 g/dl | Leukocyte count 6,000/mm3 | Platelet count 168,000/mm3 | Mean corpuscular volume 84 μm3 | Reticulocyte count 0.1% | ."], "s1": [1, 5, 7], "s2": [0, 2, 3, 4, 6, 8]} {"key": 257, "questions": "Which of the following is the most likely diagnosis?", "options": [{"label": "A", "disease": "Benign paroxysmal positional vertigo"}, {"label": "B", "disease": "Persistent postural-perceptual dizziness"}, {"label": "C", "disease": "Meniere disease"}, {"label": "D", "disease": "Acoustic neuroma"}], "answer_idx": "A", "symptoms": ["A 64-year-old woman comes to the physician because of several episodes of dizziness during the last month.", "The episodes last between 30–40 seconds and during the episodes she feels as though she is spinning.", "Episodes usually occur immediately after lying down or suddenly sitting up from a reclined position.", "They also sometimes happen when she suddenly turns her head to look at something.", "She has no nausea.", "Last month, she had an upper respiratory tract infection that resolved spontaneously.", "She has hypertension treated with hydrochlorothiazide.", "Otoscopic examination shows no abnormalities.", "There is no lateralization when a vibrating 512 Hz tuning fork is placed in the middle of the forehead."], "s1": [0, 1, 2, 3, 4], "s2": [5, 6, 7, 8]} {"key": 258, "questions": "Which of the following is the most likely diagnosis in this patient?", "options": [{"label": "A", "disease": "Illness anxiety disorder"}, {"label": "B", "disease": "Somatic symptom disorder"}, {"label": "C", "disease": "Factitious disorder"}, {"label": "D", "disease": "Munchausen syndrome"}], "answer_idx": "A", "symptoms": ["A 45-year-old man presents to the clinic for the third time in the past 3 weeks asking for an MRI of his lower back.", "He had initially gone to the gym as advised by his wife and had started doing some back exercises.", "The next day, he developed soreness in his lower back and spent the whole day online trying to figure out what was wrong with him.", "He has been to the emergency department a few times and subsequently has been redirected to the clinic where each time he was thoroughly examined and investigated for all the possible causes.", "The patient still exhibits worry and would like to get an MRI to make sure everything is ‘okay.’ ."], "s1": [1, 2], "s2": [0, 3, 4]} {"key": 259, "questions": "Which of the following is the most likely diagnosis?", "options": [{"label": "A", "disease": "Fat embolism"}, {"label": "B", "disease": "Thyrotoxic crisis"}, {"label": "C", "disease": "Malignant hyperthermia"}, {"label": "D", "disease": "Meningitis\n\""}], "answer_idx": "A", "symptoms": ["Shortly after undergoing a bipolar prosthesis for a displaced femoral neck fracture of the left hip acquired after a fall the day before, an 80-year-old woman suddenly develops dyspnea.", "The surgery under general anesthesia with sevoflurane was uneventful, lasting 98 minutes, during which the patient maintained oxygen saturation readings of 96–100% on 8 L of oxygen.", "She has a history of hypertension, osteoporosis, and osteoarthritis of her right knee.", "Her medications include ramipril, naproxen, ranitidine, and a multivitamin.", "She appears cyanotic, drowsy, and is oriented only to person.", "Her temperature is 38.6°C (101.5°F), pulse is 135/minute, respirations are 36/min, and blood pressure is 155/95 mm Hg.", "Pulse oximetry on room air shows an oxygen saturation of 81%.", "There are several scattered petechiae on the anterior chest wall.", "Laboratory studies show a hemoglobin concentration of 10.5 g/dL, a leukocyte count of 9,000/mm3, a platelet count of 145,000/mm3, and a creatine kinase of 190 U/L.", "An ECG shows sinus tachycardia."], "s1": [0, 4, 5, 6, 7, 9], "s2": [1, 2, 3, 8]} {"key": 260, "questions": "Which of the following is the most likely etiology of the patient's presentation?", "options": [{"label": "A", "disease": "Breastfeeding jaundice"}, {"label": "B", "disease": "Breast milk jaundice"}, {"label": "C", "disease": "Glucose-6-phosphate dehydrogenase deficiency"}, {"label": "D", "disease": "Hepatitis A"}], "answer_idx": "A", "symptoms": ["A 4-day-old, full-term infant born by uncomplicated vaginal delivery is brought to a pediatrician by his mother, who notes that her son's skin appears yellow.", "She reports that the child cries several times per day, but sleeps 7-8 hours at night, uninterrupted.", "She has been breastfeeding exclusively and feels that he has been feeding well.", "He has been having a bowel movement on average once every 3 days."], "s1": [0], "s2": [1, 2, 3]} {"key": 261, "questions": "Which of the following is the most likely diagnosis?", "options": [{"label": "A", "disease": "Acute lymphoid leukemia"}, {"label": "B", "disease": "Chronic myeloid leukemia"}, {"label": "C", "disease": "Hairy cell leukemia"}, {"label": "D", "disease": "Burkitt lymphoma"}], "answer_idx": "B", "symptoms": ["A 60-year-old man presents to the physician for a follow-up examination.", "During the previous visit, splenomegaly was detected on the abdominal exam, which has been confirmed by abdominal ultrasound.", "He has no complaints other than fatigue for several months.", "There is no history of serious illness.", "His only medication is acetaminophen for knee pain.", "The temperature is 36.7℃ (98.1℉), pulse is 68, respirations are 12/min, and blood pressure is 125/70 mm Hg.", "On physical examination, the spleen size on percussion is 16 cm (6.2 in).", "Otherwise, the physical examination shows no abnormalities.", "The laboratory test results are as follows: | Hemoglobin 10 g/dL | Mean corpuscular volume 90 μm3 | Leukocyte count 80,000/mm3 | Platelet count 450,000/mm3 | The peripheral blood smear shows a predominance of neutrophils and the presence of band cells, myelocytes, promyelocytes, and blasts (< 5%).", "The cytogenetic study shows t(9;22)."], "s1": [0, 2, 3, 4, 5, 7], "s2": [1, 6, 8, 9]} {"key": 262, "questions": "Which of the following is the most likely cause of this patient’s recent condition?", "options": [{"label": "A", "disease": "Adenocarcinoma"}, {"label": "B", "disease": "Bronchoalveolar carcinoma"}, {"label": "C", "disease": "Large-cell carcinoma"}, {"label": "D", "disease": "Squamous cell carcinoma"}], "answer_idx": "D", "symptoms": ["A 75-year-old woman is brought to the emergency department because of vomiting, abdominal pain, and constipation for the past 2 days.", "She has urinated frequently over the past week and has become excessively thirsty during this time.", "She has no significant past medical history.", "She is a 30-pack-year smoker.", "She appears confused.", "Her blood pressure is 95/70 mm Hg, pulse is 110/min, respirations are 25/min, and temperature is 37.1°C (98.8°F).", "She has dry oral mucosa.", "Lung auscultation shows rhonchi and wheezing localized to the right middle lobe area.", "An electrocardiogram (ECG) shows a shortened QT interval.", "A computed tomography scan is shown."], "s1": [0, 1, 2, 4, 5, 6, 8], "s2": [3, 7, 9]} {"key": 263, "questions": "What is the most likely diagnosis?", "options": [{"label": "A", "disease": "Acute viral rhinosinusitis"}, {"label": "B", "disease": "Acute bacterial rhinosinusitis"}, {"label": "C", "disease": "Allergic rhinitis"}, {"label": "D", "disease": "Chronic rhinosinusitis"}], "answer_idx": "B", "symptoms": ["A 30-year-old previously healthy male presents to your office with complaints of facial pressure and rhinorrhea for the past 3 weeks.", "The patient reports that several weeks prior, he had a “common cold” which resolved.", "However, he has since developed worsening facial pressure, especially over his cheeks and forehead.", "He reports over 1 week of green tinged rhinorrhea.", "His temperature is 100.1 deg F (37.8 deg C), blood pressure is 120/70 mmHg, pulse is 85/min, and respirations are 15/min.", "Nasal exam reveals edematous turbinates and purulent discharge."], "s1": [0, 1, 2, 3], "s2": [4, 5]} {"key": 264, "questions": "Which of the following is the most likely diagnosis?", "options": [{"label": "A", "disease": "Asbestosis"}, {"label": "B", "disease": "Hypersensitivity pneumonitis"}, {"label": "C", "disease": "Asthma"}, {"label": "D", "disease": "Chronic bronchitis"}], "answer_idx": "D", "symptoms": ["A 48-year-old man comes to the physician because of a 4-month history of persistent cough productive of white phlegm.", "He coughs consistently throughout the day, and he has not noticed any particular triggers that make it worse.", "He has also recently experienced some shortness of breath with exertion.", "He has not had any weight loss, fever, or chills.", "He had similar symptoms last year, which lasted about 6 months.", "He has hypertension, for which he takes amlodipine.", "He has worked in construction for the past 10 years.", "He has smoked a pack of cigarettes per day for 30 years.", "Vital signs are within normal limits.", "Examination shows an end-expiratory wheeze throughout all lung zones."], "s1": [0, 1, 2, 4, 9], "s2": [3, 5, 6, 7, 8]} {"key": 265, "questions": "Which of the following is the most likely diagnosis?", "options": [{"label": "A", "disease": "Hodgkin lymphoma"}, {"label": "B", "disease": "Hepatoblastoma"}, {"label": "C", "disease": "Nephroblastoma"}, {"label": "D", "disease": "Neuroblastoma"}], "answer_idx": "D", "symptoms": ["A 2-year-old girl is brought to the physician because of abdominal distention and twitching of her feet, which her mother noticed a week ago.", "The patient has also had a low-grade fever for 5 days.", "She has not had a bowel movement in 3 days.", "She was born at term and has been healthy since.", "She is at the 40th percentile for height and 20th percentile for weight.", "Her temperature is 38.1°C (100.6°F), pulse is 128/min, and blood pressure is 135/82 mm Hg.", "Examination shows an irregular palpable mass that crosses the midline of the abdomen.", "The liver is palpated 3 cm below the right costal margin.", "There are erratic movements of the eyes.", "A 24-hour urine collection shows elevated homovanillic acid and vanillylmandelic acid levels."], "s1": [1, 3, 4, 5], "s2": [0, 2, 6, 7, 8, 9]} {"key": 266, "questions": "Which of the following is the most probable cause of the newborn’s condition?", "options": [{"label": "A", "disease": "Microangiopathy"}, {"label": "B", "disease": "Hyperinsulinemia"}, {"label": "C", "disease": "Hyperglycemia"}, {"label": "D", "disease": "Polycythemia"}], "answer_idx": "B", "symptoms": ["A 29-year-old G3P2 undergoes a cesarean section at 38 weeks gestation and delivers a boy weighing 4570 g with Apgar scores of 5 and 8 at 1 and 5 minutes, respectively.", "The current pregnancy was complicated by gestational diabetes with poor glycemic control.", "The newborn’s heart rate is 122/min, the respiratory rate is 31/min, and the temperature 36.4℃ (97.5℉).", "On examination, the newborn is pale, lethargic, diaphoretic, and has poor muscular tone.", "The liver is 2 cm below the right costal margin."], "s1": [0, 1], "s2": [2, 3, 4]} {"key": 267, "questions": "Of the following options, which underlying condition could cause this reaction?", "options": [{"label": "A", "disease": "Bruton's agammaglobulinemia"}, {"label": "B", "disease": "IgA deficiency"}, {"label": "C", "disease": "Hyper-IgM syndrome"}, {"label": "D", "disease": "Hyper-IgD syndrome"}], "answer_idx": "B", "symptoms": ["A 4-year-old boy is referred to the transfusion clinic with malaise, fever, bilateral conjunctivitis, erythema of the oral mucosa, macular rash of the trunk, and inguinal lymphadenopathy following several days of loose stool.", "The boy was born at 39 weeks gestation via spontaneous vaginal delivery.", "He is up to date on all vaccines and is meeting all developmental milestones.", "Kawasaki’s disease is suspected and therapy with intravenous immunoglobulin and aspirin is initiated.", "Later that day, the patient develops trouble breathing, facial flushing, and rapidly evolving pruritic abdominal rash.", "He is rushed to the emergency department where his blood pressure is 85/48 mm Hg, heart rate is 120/min, respiratory rate is 26/min, and temperature is 37.0°C (98.6°F).", "On physical examination, the patient has severe facial edema and severe stridor, which can be heard without a stethoscope.", "A complete blood count is normal."], "s1": [0, 3, 4], "s2": [1, 2, 5, 6, 7]} {"key": 268, "questions": "Which of the following is the most likely explanation for this patient's presentation?", "options": [{"label": "A", "disease": "Abdominal aortic aneurysm repair complication"}, {"label": "B", "disease": "Atrial fibrillation"}, {"label": "C", "disease": "Obstruction of the small bowel"}, {"label": "D", "disease": "Ruptured appendix"}], "answer_idx": "A", "symptoms": ["A 59-year-old man presents to the emergency department with abdominal pain.", "He states that it seemed to come on abruptly, though he does endorse feeling pain the past several days.", "He has been experiencing bloody diarrhea as well as a subjective fever.", "His past medical history is notable for an ST elevation myocardial infarction (STEMI) treated 1 year ago, an abdominal aortic aneurysm (AAA) treated 1 week ago, chronic obstructive pulmonary disease (COPD), asthma, obesity, hypertension, stroke treated 5 years ago, an atrial arrhythmia, diabetes, constipation, and post-traumatic stress disorder (PTSD).", "His temperature is 102.0°F (39°C), blood pressure is 197/128 mmHg, pulse is 135/min, respirations are 22/min, and oxygen saturation is 92% on room air.", "On physical exam, you note an obese man in distress.", "Cardiopulmonary exam is notable for bilateral wheezes and a systolic murmur.", "There is bilateral lower extremity pitting edema and venous stasis ulcers.", "Abdominal exam reveals diffuse tenderness which the patient describes as 10/10 in severity.", "Laboratory values are ordered as seen below. | | Hemoglobin: 14 g/dL | Hematocrit: 42% | Leukocyte count: 15,500/mm^3 with normal differential | Platelet count: 199,000/mm^3 | | Serum: | Na+: 139 mEq/L | Cl-: 100 mEq/L | K+: 4.4 mEq/L | HCO3-: 22 mEq/L | BUN: 32 mg/dL | Glucose: 189 mg/dL | Creatinine: 1.9 mg/dL | Ca2+: 10.2 mg/dL | Lactate: 4.5 mg/dL | AST: 12 U/L | ALT: 10 U/L | | ."], "s1": [0, 1, 2, 8], "s2": [3, 4, 5, 6, 7, 9]} {"key": 269, "questions": "Which of the following is the most likely diagnosis?", "options": [{"label": "A", "disease": "Acute pericarditis"}, {"label": "B", "disease": "Aortic stenosis"}, {"label": "C", "disease": "Infective endocarditis"}, {"label": "D", "disease": "Myocardial infarction"}], "answer_idx": "A", "symptoms": ["A 44-year-old man presents to the emergency department due to sudden chest pain and difficulty breathing for the past 3 hours.", "The pain is felt in the retrosternal area and radiates up to his left shoulder and arm; it worsens on inspiration, and is relieved when he is leaning forward.", "His heart rate is 61/min, respiratory rate is 16/min, temperature is 36.5°C (97.7°F), and blood pressure is 115/78 mm Hg.", "Physical examination shows no abnormalities.", "Pericardial friction rub is heard on auscultation.", "Laboratory results show elevated erythrocyte sedimentation rate (ESR) and C-reactive protein (CRP) levels.", "An ECG is performed."], "s1": [0, 1, 4, 5], "s2": [2, 3, 6]} {"key": 270, "questions": "Which of the following is the most likely underlying diagnosis in this patient?", "options": [{"label": "A", "disease": "Glioblastoma multiforme"}, {"label": "B", "disease": "Infiltrating ductal carcinoma of the breast"}, {"label": "C", "disease": "Primary seizure disorder"}, {"label": "D", "disease": "Small cell carcinoma of the lung"}], "answer_idx": "D", "symptoms": ["A 65-year-old woman presents to the emergency department with a new-onset seizure.", "The patient was at home watching television when she suddenly began jerking her limbs and not responding according to her partner.", "The patient works as a gardener is often dehydrated but drinks large volumes of water to compensate for her work.", "She does not wear protective gear or sunscreen while working.", "She has smoked 1.5 packs of cigarettes per day for the past 40 years.", "She drinks liquor before bed every night.", "The patient has a past medical history of hypertension and diabetes and recently lost 9 pounds.", "Physical exam is within normal limits."], "s1": [0, 1, 6, 7], "s2": [2, 3, 4, 5]} {"key": 271, "questions": "Which of the following is the most likely underlying cause of this patient's symptoms?\"", "options": [{"label": "A", "disease": "Adrenal carcinoma"}, {"label": "B", "disease": "Hypothyroidism"}, {"label": "C", "disease": "Small cell lung cancer"}, {"label": "D", "disease": "Adrenal adenoma"}], "answer_idx": "C", "symptoms": ["A 45-year-old woman comes to the physician because of fatigue and irregular menstrual cycles for the past year.", "She also complains of recurrent sinus infections.", "During the past 6 months, she has had increased urinary frequency and swelling of her feet.", "She has also had difficulty lifting her 3-year-old niece for the past 3 weeks.", "She was recently diagnosed with depression.", "She works as a medical assistant.", "The patient has smoked one half-pack of cigarettes daily for 25 years and drinks four beers on the weekends.", "Her only medication is escitalopram.", "She is 160 cm (5 ft 3 in) tall and weighs 79 kg (175 lb); BMI is 31 kg/m2.", "She appears tired.", "Her temperature is 37°C (98.6°F), pulse is 80/min, respirations are 18/min, and blood pressure is 140/82 mm Hg.", "Physical examinations shows neck obesity and an enlarged abdomen.", "Examination of the skin shows multiple bruises on her arms and legs.", "There is generalized weakness and atrophy of the proximal muscles.", "Laboratory studies show: | Serum | Na+ 150 mEq/L | K+ 3.0 mEq/L | Cl- 103 mEq/L | HCO3- 30 mEq/L | Urea nitrogen 19 mg/dL | Creatinine 0.9 mg/dL | Glucose 136 mg/dL | A 1 mg overnight dexamethasone suppression test shows a serum cortisol of 167 nmol/L (N < 50) and a 24-hour urinary cortisol is 425 μg (N < 300 μg).", "Serum ACTH is 169 pg/mL (N = 7–50).", "Subsequently, a high-dose dexamethasone suppression test shows a serum cortisol level of 164 nmol/L (N < 50)."], "s1": [0, 1, 4, 5, 6, 7, 8, 10], "s2": [2, 3, 9, 11, 12, 13, 14, 15, 16]} {"key": 272, "questions": "Which of the following is the most likely etiology of this patient’s condition?", "options": [{"label": "A", "disease": "Fibromyalgia"}, {"label": "B", "disease": "Oral contraceptives"}, {"label": "C", "disease": "Protein C deficiency"}, {"label": "D", "disease": "Protein S deficiency"}], "answer_idx": "B", "symptoms": ["A 43-year-old woman presents with left calf pain.", "She denies any previous episodes of such pain.", "Past medical history is significant for cervical carcinoma treated with cryosurgery with no evidence of recurrence.", "Current medications are an estrogen-containing oral contraceptive and a multivitamin.", "The patient reports a 40-pack-year smoking history, but no alcohol or recreational drug use.", "The patient is afebrile and her vital signs are within normal limits.", "On physical examination, her left calf is swollen, erythematous, and tender to palpation.", "A D-dimer level is positive."], "s1": [1, 2, 3, 4, 5], "s2": [0, 6, 7]} {"key": 273, "questions": "Which of the following is the most likely diagnosis?", "options": [{"label": "A", "disease": "Anterior uveitis"}, {"label": "B", "disease": "Normal post-operative visual adaptations"}, {"label": "C", "disease": "Seasonal allergies"}, {"label": "D", "disease": "Sympathetic ophthalmia"}], "answer_idx": "D", "symptoms": ["A 47-year-old man presents to his ophthalmologist for abnormal vision.", "The patient was discharged from the hospital 2 weeks ago after a motor vehicle accident where he was given 1L of blood.", "He also underwent several surgical procedures including enucleation of the right globe and exploratory abdominal laparotomy to stop internal bleeding.", "Currently, he states that he has noticed decreased vision in his left eye.", "The patient has a past medical history of poorly controlled diabetes, asthma, and atopic dermatitis.", "His temperature is 98.9°F (37.2°C), blood pressure is 167/108 mmHg, pulse is 90/min, respirations are 14/min, and oxygen saturation is 97% on room air.", "Physical exam is notable for 20/200 vision in the patient's left eye, which is worse than usual.", "The patient's left eye appears red and irritated."], "s1": [0, 3, 6, 7], "s2": [1, 2, 4, 5]} {"key": 274, "questions": "Which of the following is the most likely etiology of this patient's symptoms?", "options": [{"label": "A", "disease": "Alcohol use"}, {"label": "B", "disease": "Folate deficiency"}, {"label": "C", "disease": "Iron deficiency"}, {"label": "D", "disease": "Vitamin B12 deficiency"}], "answer_idx": "D", "symptoms": ["A 29-year-old woman presents to her primary care physician with worsening fatigue and lightheadedness over the past several months.", "She states that she has felt easily fatigued and has experienced several falls during this time frame as well.", "She drinks 5 to 8 drinks per day and works as a waitress.", "Her temperature is 98.2°F (36.8°C), blood pressure is 114/64 mmHg, pulse is 98/min, respirations are 12/min, and oxygen saturation is 98% on room air.", "Laboratory studies are ordered as seen below. | | Hemoglobin: 9.0 g/dL | Hematocrit: 29% | Leukocyte count: 6,700/mm^3 with normal differential | Platelet count: 199,400/mm^3 | Methylymalonic acid: 1.0 umol/L (normal < 0.40 umol/L) | | ."], "s1": [0, 1, 3], "s2": [2, 4]} {"key": 275, "questions": "What is the most likely cause of her symptoms?", "options": [{"label": "A", "disease": "Vascular abnormality"}, {"label": "B", "disease": "Seizure"}, {"label": "C", "disease": "Demyelinating disease"}, {"label": "D", "disease": "Impaired dopaminergic neurons"}], "answer_idx": "A", "symptoms": ["A 63-year-old woman with a history of adult polycystic kidney disease and COPD presents to her optometrist after having difficulty watching a movie.", "She has no other complaints.", "She is accompanied by her daughter, who notes that her mother hasn’t been able to look at her since the previous day.", "On exam, her right eye has full range of motion, but her left eye is fixed in a down and outward position."], "s1": [0, 1], "s2": [2, 3]} {"key": 276, "questions": "Which of the following is the most likely diagnosis in this patient?", "options": [{"label": "A", "disease": "Iron deficiency anemia"}, {"label": "B", "disease": "Beta-thalassemia trait"}, {"label": "C", "disease": "Alpha-thalassemia trait"}, {"label": "D", "disease": "Folate deficiency anemia"}], "answer_idx": "C", "symptoms": ["A 23-year-old African American G1P0 woman at 18 weeks estimated gestational age presents with fatigue and lethargy.", "She says the symptoms onset gradually about 1 week ago and have progressively worsened.", "She has been otherwise healthy.", "Past medical history is unremarkable.", "She denies smoking, drug or alcohol use.", "The patient is afebrile and vital signs are within normal limits.", "Her physical examination is unremarkable.", "Her blood count shows the following: | Hemoglobin 10.8 g/dL | Red blood cell count 5.7 million/mm3 | Mean corpuscular volume (MCV) 76 μm3 | Red cell distribution width 12.4% (ref: 11.5–14.5%) | She is empirically started on ferrous gluconate tablets.", "Five weeks later, she shows no improvement in her hemoglobin level.", "Hemoglobin electrophoresis is normal."], "s1": [0, 1, 2, 3, 4, 5, 6], "s2": [7, 8, 9]} {"key": 277, "questions": "Which of the following is the most likely diagnosis in this patient?", "options": [{"label": "A", "disease": "Follicular carcinoma"}, {"label": "B", "disease": "Papillary carcinoma"}, {"label": "C", "disease": "Anaplastic carcinoma"}, {"label": "D", "disease": "Follicular adenoma"}], "answer_idx": "B", "symptoms": ["A 49-year-old woman otherwise healthy presents to the outpatient department with swelling of the neck.", "Family history is negative for any thyroid disorders.", "Physical examination shows a nontender thyroid gland with a nodule on the right side.", "The thyroid gland is mobile on deglutition.", "Cervical lymphadenopathy is present.", "Fine needle aspiration and cytology show empty appearing nuclei with central clearing, nuclear grooves and branching structures interspersed with calcific bodies."], "s1": [0, 1], "s2": [2, 3, 4, 5]} {"key": 278, "questions": "What is the most likely diagnosis?", "options": [{"label": "A", "disease": "Social phobia"}, {"label": "B", "disease": "Agoraphobia"}, {"label": "C", "disease": "Posttraumatic stress disorder"}, {"label": "D", "disease": "Generalized anxiety disorder"}], "answer_idx": "D", "symptoms": ["A 43-year-old woman presents to a new primary care physician complaining of anxiety.", "She has been worrying non-stop recently about the possibility that her husband will lose his job as a teacher.", "Her husband, who is present, assures the physician that his job is entirely secure and states that she has \"fretted\" for their entire marriage, though the exact topic causing her anxiety varies over time.", "She also worries excessively about everyday tasks, such as whether she will catch the train on time and whether their house in Southern California is sufficiently earthquake-proof.", "She has no way to overcome these worries.", "She endorses other symptoms including poor sleep (associated with racing thoughts about her various worries), fatigue, and impaired concentration at work, all of which have been present for at least the past year.", "Her vital signs are within normal limits and there are no abnormalities on physical exam."], "s1": [0, 1, 2, 3, 4, 5], "s2": [6]} {"key": 279, "questions": "Which of the following is the most probable cause of his complaints?", "options": [{"label": "A", "disease": "Osteomyelitis"}, {"label": "B", "disease": "Osteosarcoma"}, {"label": "C", "disease": "Ewing’s sarcoma"}, {"label": "D", "disease": "Chondrosarcoma"}], "answer_idx": "B", "symptoms": ["A 13-year-old boy presents to the clinic for evaluation of pain, swelling, and redness in the left knee.", "This began a month ago and has been progressively worsening.", "He thinks his pain is due to playing soccer.", "He is frustrated that the pain does not respond to Tylenol.", "History is non-contributory.", "The vital signs are unremarkable.", "On examination, there is tenderness and warmth present over the knee with limitation of movement.", "The laboratory values show a normal white blood cell (WBC) count and elevated alkaline phosphatase.", "A radiograph of the knee shows mixed lytic and blastic lesions."], "s1": [2, 3, 4, 5], "s2": [0, 1, 6, 7, 8]} {"key": 280, "questions": "What is the most likely diagnosis?", "options": [{"label": "A", "disease": "Squamous cell carcinoma"}, {"label": "B", "disease": "Keratoacanthoma"}, {"label": "C", "disease": "Verrucous carcinoma"}, {"label": "D", "disease": "Basal cell carcinoma"}], "answer_idx": "D", "symptoms": ["A 68-year-old male presents to your office for his annual physical exam.", "He has no complaints at this time and his chronic hypertension is well controlled.", "You notice a .5cm papule on the patient's eyelid that has a pink pearly appearance, rolled borders, and overlying telangiectasias.", "On further questioning, you find out the patient was a door-to-door salesman and spent a lot of time outdoors, and he did not wear sunscreen.", "He has fair skin and blonde hair.", "The patient states that he first noticed the lesion about 6 months prior, and it has grown slightly since then."], "s1": [0, 1, 3, 4], "s2": [2, 5]} {"key": 281, "questions": "What is the most likely diagnosis for this patient?", "options": [{"label": "A", "disease": "Acute pericarditis"}, {"label": "B", "disease": "Left ventricular free wall rupture"}, {"label": "C", "disease": "Papillary muscle rupture"}, {"label": "D", "disease": "Pulmonary embolism"}], "answer_idx": "B", "symptoms": ["An obese 50-year-old woman presents to the emergency department of a rural hospital due to chest pain for the last 12 hours.", "The pain is substernal, crushing in nature, radiating into her neck, and accompanied by profuse sweating.", "Her history is significant for hypertension, and she admits poor compliance with her medications.", "An ECG done at the clinic confirmed the presence of STEMI, and the patient was given aspirin.", "The ambulance was in an accident on the way to the hospital and by the time the patient reached the angiography suite, the cardiologist decided intervention should be delayed due to the late presentation.", "Also, the patient’s pain has improved.", "The patient is admitted and is stable for 3 nights with no dyspnea or edema.", "Her blood pressure is kept under control while in the hospital.", "On the fourth night, the patient becomes quite agitated and soon thereafter she is unresponsive.", "Her pulse cannot be palpated.", "She has an elevated JVP and upon auscultation, heart sounds are distant."], "s1": [0, 1, 2, 3, 4, 8, 9], "s2": [5, 6, 7, 10]} {"key": 282, "questions": "Which of the following best explains the findings in this patient?", "options": [{"label": "A", "disease": "Atrophy and fibrosis of the esophageal smooth muscle"}, {"label": "B", "disease": "Metaplastic transformation of esophageal mucosa"}, {"label": "C", "disease": "Neoplastic proliferation of squamous epithelium"}, {"label": "D", "disease": "Eosinophilic infiltration of the esophageal walls\n\""}], "answer_idx": "C", "symptoms": ["A 68-year-old man comes to the physician because of a 4-month history of bad breath and progressive difficulty swallowing solid food.", "Physical examination shows no abnormalities.", "An upper endoscopy is performed and a photomicrograph of a biopsy specimen obtained from the mid-esophagus is shown."], "s1": [0], "s2": [1, 2]} {"key": 283, "questions": "Which of the following is the most appropriate next step in diagnosis?", "options": [{"label": "A", "disease": "Serum TSH levels"}, {"label": "B", "disease": "Acetylcholine receptor antibody testing"}, {"label": "C", "disease": "Serum potassium levels"}, {"label": "D", "disease": "Temporal artery biopsy"}], "answer_idx": "A", "symptoms": ["A 43-year-old woman comes to the physician because of a 2-month history of progressive muscle pain and stiffness that worsens with exercise.", "She also has difficulty climbing stairs, getting out of chairs, and putting things on shelves or in cupboards.", "She has had constipation, occasional headaches, and a 9-kg (20-lb) weight gain during the past year.", "She has hypertension controlled with atenolol and amlodipine.", "She has used calamine lotion for the past 6 months for dry skin.", "Her pulse is 80/min and her blood pressure is 138/76 mm Hg.", "Physical examination shows weakness of the proximal muscle groups.", "She has delayed tendon reflex relaxation, with a mounding of the muscle surface occurring just before relaxation.", "Creatine kinase level is 3,120 U/L."], "s1": [0, 1, 6, 7, 8], "s2": [2, 3, 4, 5]} {"key": 284, "questions": "Which of the following is the most likely diagnosis?", "options": [{"label": "A", "disease": "Alopecia areata"}, {"label": "B", "disease": "Secondary syphilis"}, {"label": "C", "disease": "Tinea capitis"}, {"label": "D", "disease": "Trichotillomania"}], "answer_idx": "A", "symptoms": ["A 40-year-old woman presents to the clinic for hair loss.", "She was in her usual state of health until earlier this month when she started noticing more and more hair on her pillow in the morning.", "The problem has only been getting worse since then; she's even started pulling out clumps of hair when she shampoos in the morning.", "She has no other symptoms, and her past medical history is unremarkable.", "The physical exam is notable for smooth, circular, non-scarring, hairless patches across her scalp.", "A firm tug on a bundle of hair causes almost all of the hair to be removed."], "s1": [3], "s2": [0, 1, 2, 4, 5]} {"key": 285, "questions": "Which of the following is the most likely diagnosis?", "options": [{"label": "A", "disease": "Cavernous hemangioma"}, {"label": "B", "disease": "Kaposi sarcoma"}, {"label": "C", "disease": "Angiosarcoma"}, {"label": "D", "disease": "Cholangiocarcinoma\n\""}], "answer_idx": "C", "symptoms": ["A previously healthy 75-year-old man comes to the physician with a 6-month history of fatigue, weight loss, and abdominal pain.", "He drinks 2 oz of alcohol on the weekends and does not smoke.", "He is retired but previously worked in a factory that produces plastic pipes.", "Abdominal examination shows right upper quadrant tenderness; the liver edge is palpable 2 cm below the ribs.", "A liver biopsy specimen shows pleomorphic spindle cells that express PECAM-1 on their surface."], "s1": [0, 3, 4], "s2": [1, 2]} {"key": 286, "questions": "Which of the following is the most likely diagnosis?", "options": [{"label": "A", "disease": "Staphylococcal scalded skin syndrome"}, {"label": "B", "disease": "Juvenile idiopathic arthritis"}, {"label": "C", "disease": "Granulomatosis with polyangiitis"}, {"label": "D", "disease": "Kawasaki disease\n\""}], "answer_idx": "D", "symptoms": ["A 5-year-old boy is brought to the physician because of high-grade fever and generalized fatigue for 5 days.", "Two days ago, he developed a rash on his trunk.", "He returned from a family hiking trip to Montana 1 week ago.", "His immunization records are unavailable.", "His temperature is 39.8°C (103.6°F), pulse is 111/min, and blood pressure is 96/60 mm Hg.", "Examination shows injection of the conjunctivae bilaterally.", "The tongue and pharynx are erythematous.", "Tender cervical lymphadenopathy is present on the left.", "There is a macular rash over the trunk and extremities.", "Bilateral knee joints are swollen and tender; range of motion is limited by pain.", "Cardiopulmonary examination shows no abnormalities."], "s1": [0, 1, 2, 4, 8, 10], "s2": [3, 5, 6, 7, 9]} {"key": 287, "questions": "Which of the following is the most likely predisposing factor for this patient’s current condition?", "options": [{"label": "A", "disease": "Periodontal disease"}, {"label": "B", "disease": "Colon cancer"}, {"label": "C", "disease": "Valve replacement"}, {"label": "D", "disease": "Urinary tract infection"}], "answer_idx": "B", "symptoms": ["A 59-year-old man is brought to the emergency department by his wife because of fever, chills, night sweats, and generalized fatigue for 2 weeks.", "His temperature is 39.1°C (102.4°F).", "He appears ill.", "Physical examination shows a grade 3/6 mid-diastolic murmur at the left sternal border, and crackles at both lung bases.", "A transesophageal echocardiography shows a 12 mm vegetation on the aortic valve.", "Blood cultures show gram-positive, catalase-negative, gamma-hemolytic cocci in chains that are unable to grow in a 6.5% NaCl medium."], "s1": [0, 1, 2], "s2": [3, 4, 5]} {"key": 288, "questions": "Which of the following is the most likely explanation for this patient's symptoms?\"", "options": [{"label": "A", "disease": "Somatic symptom disorder"}, {"label": "B", "disease": "Unipolar major depression"}, {"label": "C", "disease": "Illness anxiety disorder"}, {"label": "D", "disease": "Impaired glucose homeostasis"}], "answer_idx": "C", "symptoms": ["A 52-year-old woman comes to the physician because of mild fatigue and dizziness for the past 2 days.", "She has not been to work since yesterday due to her symptoms.", "She says she has \"\"very high blood sugar” and has had similar episodes often in the past 2 years, for which she has visited multiple doctors around the city.", "She has also purchased a home glucose monitoring device, which she uses daily.", "Since the symptoms began, she has become socially withdrawn and spends much of her time at home researching diabetes on the internet.", "One week ago, she took a day off work because of her symptoms.", "She is not on any treatment.", "She has had 3 laboratory test reports that all show normal fasting and post-meal blood glucose levels.", "Her father and brother have diabetes mellitus type 2.", "She appears anxious.", "Vital signs are normal.", "Physical examination shows no abnormalities.", "Random serum glucose is 128 mg/dL."], "s1": [0, 1, 5, 9, 4], "s2": [2, 3, 6, 7, 8, 10, 11, 12]} {"key": 289, "questions": "Which of the following is the most likely diagnosis?", "options": [{"label": "A", "disease": "Cerumen impaction"}, {"label": "B", "disease": "Ototoxicity"}, {"label": "C", "disease": "Vestibulocochlear nerve damage"}, {"label": "D", "disease": "Presbycusis"}], "answer_idx": "D", "symptoms": ["A 78-year-old man comes to the physician for evaluation of progressive hearing loss in both ears over the past year.", "He has difficulties understanding conversations in crowded places and when more than one person talks at the same time.", "He has no dizziness, ear discharge, ringing noise, or ear pain.", "He has a history of hypertension, hypercholesterolemia, and type 2 diabetes mellitus.", "Medications include enalapril, metformin, and atorvastatin.", "Vital signs are within normal limits.", "Otoscopic examination shows pearly gray, translucent tympanic membranes with a normal light reflex.", "A vibrating 512 Hz tuning fork is placed on the left mastoid process.", "Once the patient no longer hears a tone, the fork is held over the ipsilateral ear and the patient reports to hear the tone again.", "The same test is repeated on the right side with similar results.", "There is no lateralization when a vibrating 512 Hz tuning fork is placed in the middle of the forehead."], "s1": [0, 1, 2, 3, 4, 5], "s2": [6, 7, 8, 9, 10]} {"key": 290, "questions": "Which of the following is the most likely diagnosis?", "options": [{"label": "A", "disease": "Traumatic subungual hemorrhage"}, {"label": "B", "disease": "Malignant melanoma"}, {"label": "C", "disease": "Onychomycosis"}, {"label": "D", "disease": "Squamous cell carcinoma"}], "answer_idx": "B", "symptoms": ["A 65-year-old Asian woman comes to the physician for a routine health maintenance examination.", "On questioning, she has had occasional night sweats during the past 2 months.", "She has not had fevers or weight loss.", "Seven months ago, she had an acute myocardial infarction and was treated with percutaneous coronary intervention.", "She has hypertension, hyperlipidemia, and gastroesophageal reflux disease.", "She has smoked one pack of cigarettes daily for 37 years.", "Current medications include aspirin, atorvastatin, ramipril, metoprolol, and esomeprazole.", "She is 178 cm (5 ft 10 in) tall and weighs 89 kg (207 lbs); BMI is 29.7 kg/m2.", "Her temperature is 37.4°C (99.3°F), pulse is 84/min, respirations are 18/min, and blood pressure is 145/80 mm Hg.", "The lungs are clear to auscultation.", "Cardiac examination shows an S4.", "There is a nontender skin lesion near the right large toenail.", "A photograph of the lesion is shown.", "The remainder of the examination shows no abnormalities."], "s1": [0, 1, 2, 5, 7, 8, 9, 13], "s2": [3, 4, 6, 10, 11, 12]} {"key": 291, "questions": "What is the most likely cause of this patient’s symptoms?", "options": [{"label": "A", "disease": "Hypothyroidism"}, {"label": "B", "disease": "Acute intermittent porphyria"}, {"label": "C", "disease": "Amyotrophic lateral sclerosis"}, {"label": "D", "disease": "Multiple sclerosis"}], "answer_idx": "A", "symptoms": ["A 22-year-old female software analyst presents to a medical clinic for evaluation of tingling and numbness in both hands for the past 2 months.", "Her symptoms are usually aggravated by the end of the work day and absent on most off days.", "She has been a type I diabetic for 2 years and is currently on insulin.", "She admits to being sexually active but has had irregular periods for the past 3 months.", "A urine pregnancy test is negative."], "s1": [0, 1, 2], "s2": [3, 4]} {"key": 292, "questions": "Which of the following is the most likely diagnosis?", "options": [{"label": "A", "disease": "Gonococcal arthritis"}, {"label": "B", "disease": "Parvovirus B19"}, {"label": "C", "disease": "Rheumatoid arthritis"}, {"label": "D", "disease": "Transient synovitis"}], "answer_idx": "B", "symptoms": ["A 27-year-old woman presents to the emergency department for pain in multiple joints.", "She states that she has had symmetric joint pain that started yesterday and has been worsening.", "It is affecting her wrists, elbows, and shoulders.", "She also endorses a subjective fever and some fatigue but denies any other symptoms.", "The patient works as a schoolteacher and is generally healthy.", "She is currently sexually active with 2 male partners and uses condoms occasionally.", "Her temperature is 100°F (37.8°C), blood pressure is 122/85 mmHg, pulse is 88/min, respirations are 14/min, and oxygen saturation is 99% on room air.", "Her laboratory values are within normal limits.", "Physical exam is notable for joint stiffness and pain in particular of the metacarpophalangeal (MCP), proximal interphalangeal (PIP), and the wrist.", "The patient is discharged with ibuprofen.", "Four weeks later, the patient follows up at her primary doctor and notes that her symptoms have improved and she is no longer taking any medications for symptom control."], "s1": [0, 1, 2, 3, 8], "s2": [4, 5, 6, 7, 9, 10]} {"key": 293, "questions": "Which of the following is the most likely cause of this patient's symptoms?", "options": [{"label": "A", "disease": "Bronchiectasis"}, {"label": "B", "disease": "Adenocarcinoma in situ"}, {"label": "C", "disease": "Tuberculosis"}, {"label": "D", "disease": "Aspiration pneumonia"}], "answer_idx": "D", "symptoms": ["A 57-year-old man with HIV and GERD comes to the emergency department because of productive cough with malodorous phlegm and night sweats for the past week.", "He has smoked 1 pack of cigarettes daily for 30 years and he drinks 8–10 beers daily.", "His temperature is 38.9°C (102.0°F).", "Physical examination shows coarse crackles and dullness to percussion at the right lung base.", "Scattered expiratory wheezing is heard throughout both lung fields.", "The CD4+ T-lymphocyte count is 280/mm3 (N ≥ 500).", "An x-ray of the chest is shown."], "s1": [0, 1, 5], "s2": [2, 3, 4, 6]} {"key": 294, "questions": "Which of the following pathologies most likely underlies this patient’s condition?", "options": [{"label": "A", "disease": "Demyelination of neurons in various parts of the brain"}, {"label": "B", "disease": "Degeneration of the dorsal columns of the spinal cord"}, {"label": "C", "disease": "Infarction of the posterior limb of the internal capsule"}, {"label": "D", "disease": "Loss of the pyramidal tract neurons and anterior horn cells of the spinal cord"}], "answer_idx": "D", "symptoms": ["A 55-year-old man presents with worsening weakness of the left hand and progressive difficulty in walking for the last 2 months.", "He says he was previously healthy and active, hiking every weekend, but because of his increasing weakness and incoordination while walking, he stopped his weekly hiking excursion.", "His past medical history is unremarkable.", "Current medications are a daily multivitamin and occasional acetaminophen for headaches.", "He has no significant family history.", "The patient denies smoking, recreational drug use, or drinking.", "His vitals include: pulse 70/min, respirations 14/min, temperature 37.0°C (98.6°F), and blood pressure 130/80 mm Hg.", "Physical examination reveals fasciculations in the tongue and distal extremities.", "The left forearm shows significant atrophy, and mild lower limb muscle atrophy is also seen bilaterally.", "Hyporeflexia (1+) is present in the left arm, while hyperreflexia (3+) is seen in the legs bilaterally.", "Babinski’s sign is positive bilaterally.", "The patient's speech is notably slow but no evidence of productive or expressive aphasia is present.", "The cranial nerves are intact."], "s1": [0, 1, 7, 8, 9, 10, 11], "s2": [2, 3, 4, 5, 6, 12]} {"key": 295, "questions": "Which of the following is the most likely diagnosis?", "options": [{"label": "A", "disease": "Obsessive-compulsive personality disorder"}, {"label": "B", "disease": "Obsessive-compulsive disorder"}, {"label": "C", "disease": "Schizoid personality disorder"}, {"label": "D", "disease": "Adjustment disorder with mixed anxiety and depressed mood\n\""}], "answer_idx": "A", "symptoms": ["A 21-year-old man comes to the physician because of nervousness and difficulty sleeping over the past 2 weeks.", "Three months ago, he started a new team project at his job.", "He says that he has trouble falling asleep at night because he thinks about the project.", "He insists on checking the work of his teammates and spends several hours each week making elaborate lists and schedules.", "He refuses to let others take on more tasks because he thinks that only he can complete them satisfactorily.", "The project deadline has already been postponed once.", "He says, “My teammates never understand how important it is to do things the right way.” Ever since high school, he has used the majority of his time to plan out assignments and does not have time to regularly participate in social activities."], "s1": [0, 1, 5], "s2": [2, 3, 4, 6]} {"key": 296, "questions": "Which of the following is the most likely cause of her current condition?", "options": [{"label": "A", "disease": "Fat globules entering the circulation"}, {"label": "B", "disease": "Rupture of a coronary plaque"}, {"label": "C", "disease": "Damage to the alveolar capillary endothelium"}, {"label": "D", "disease": "Bacterial invasion of the pulmonary parenchyma"}], "answer_idx": "A", "symptoms": ["Two days after undergoing internal fixation of a displaced right femoral fracture sustained during a motor vehicle accident, a 34-year-old woman develops sudden shortness of breath.", "On examination, she is oriented only to person.", "Her temperature is 38.3°C (100.9°F), pulse is 122/min, respiratory rate is 31/min, and blood pressure is 152/90 mm Hg.", "Pulse oximetry on room air shows an oxygen saturation of 83%.", "There are several scattered petechiae on the anterior chest wall.", "Her hemoglobin concentration is 10.3 g/dL, leukocyte count is 9,500/mm3, and platelet count is 140,000/mm3."], "s1": [0, 2, 3], "s2": [1, 4, 5]} {"key": 297, "questions": "Which of the following is the most likely diagnosis?", "options": [{"label": "A", "disease": "Lymphadenopathy"}, {"label": "B", "disease": "Branchial cleft cyst"}, {"label": "C", "disease": "Laryngocele"}, {"label": "D", "disease": "Infantile hemangioma"}], "answer_idx": "B", "symptoms": ["A 15-year-old girl comes to the physician for the evaluation of a painless swelling under the left lower jaw for 5 months.", "Her 18-year-old sister has a history of acute lymphoblastic leukemia that has been in remission for 11 years.", "The patient's temperature is 37°C (98.6°F) Physical examination shows a 3 × 2 cm swelling on the left side, 4 cm below the base of the mandible and anterior to the sternocleidomastoid muscle.", "The swelling is soft and fluctuant.", "It does not move with swallowing, and forced exhalation against a closed glottis does not increase the size of the swelling.", "Ultrasound shows a round mass with uniform low echogenicity and no internal septations."], "s1": [0, 1], "s2": [2, 3, 4, 5]} {"key": 298, "questions": "Which of the following is the most likely cause of this patient’s current condition?", "options": [{"label": "A", "disease": "Infection with Staphylococcus aureus"}, {"label": "B", "disease": "Infection with Aspergillus fumigatus"}, {"label": "C", "disease": "Foreign body aspiration"}, {"label": "D", "disease": "Granulomatosis with polyangiitis"}], "answer_idx": "A", "symptoms": ["A 70-year-old man is brought to the emergency department by staff of the group home where he resides because of fever, a productive cough, and blood-tinged sputum for 2 days.", "The staff report that he developed myalgia, headache, and rhinorrhea 7 days ago, which improved initially before worsening again.", "He has hypertension and Alzheimer disease.", "Current medications include enalapril and donepezil.", "On arrival, he is obtunded.", "His temperature is 39.8°C (103.6°F), pulse is 120/min, respirations are 22/min and shallow, and blood pressure is 100/60 mm Hg.", "Pulse oximetry on 10 L/min of oxygen via nasal cannula shows an oxygen saturation of 92%.", "Examination shows scattered crackles and rhonchi throughout both lung fields.", "An x-ray of the chest shows bilateral lung opacities and multiple small, thin-walled cystic spaces within the basal pulmonary parenchyma."], "s1": [0, 1, 4, 5, 6, 7, 8], "s2": [2, 3]} {"key": 299, "questions": "Which of the following is the most likely underlying cause of this patient's symptoms?", "options": [{"label": "A", "disease": "Loss of sympathetic vascular tone"}, {"label": "B", "disease": "Obstructive thrombus in the pulmonary artery"}, {"label": "C", "disease": "Intravascular volume depletion"}, {"label": "D", "disease": "Type I hypersensitivity reaction"}], "answer_idx": "A", "symptoms": ["A 28-year-old man is scheduled for skin grafting of the right leg.", "Four weeks earlier, he underwent open reduction and internal fixation of an open right tibial fracture.", "The postoperative period at that time was complicated by necrosis of the overlying skin.", "In the operating room, he is placed on continuous hemodynamic monitoring and anesthetized with intrathecal bupivacaine injected into the L3–L4 spinal interspace.", "Thirty minutes later, he has severe shortness of breath and lightheadedness and loses consciousness.", "His temperature is 37.1°C (98.8°F), pulse is 38/min, respirations are 24/min, and palpable systolic blood pressure is 58 mmHg.", "The skin is diaphoretic and flushed.", "He withdraws the upper extremities to painful stimuli above the navel.", "The lungs are clear to auscultation.", "Heart sounds are not audible."], "s1": [0, 1, 2, 3], "s2": [4, 5, 6, 7, 8, 9]} {"key": 300, "questions": "Which of the following is the most likely underlying cause of this patient's symptoms?", "options": [{"label": "A", "disease": "Inadequate hypothalamic response"}, {"label": "B", "disease": "Metabolic acidosis with ketosis"}, {"label": "C", "disease": "Elevation of serum thyroxine levels"}, {"label": "D", "disease": "Central dopamine receptor blockade"}], "answer_idx": "A", "symptoms": ["A 73-year-old man is brought to the emergency department 30 minutes after he lost consciousness for 5 minutes while watching a game of summer league football in a seat from the stands in the afternoon.", "On arrival, he is lethargic and oriented only to person.", "Three weeks ago, he was treated for hepatitis A.", "He has type 2 diabetes mellitus and hypothyroidism.", "Current medications include metformin and levothyroxine.", "His temperature is 41.5 °C (106.7 °F), pulse is 106/min, respirations are 26/min, and blood pressure is 128/70 mm Hg.", "Examination shows hot, dry skin.", "The pupils are equal and reactive to light.", "Neurologic examination shows no focal findings.", "Fundoscopy shows proliferative diabetic retinopathy.", "Cardiopulmonary examination shows no abnormalities.", "His abdomen is soft and nontender.", "There is no costovertebral angle tenderness.", "His serum glucose concentration is 160 mg/dL.", "An ECG shows a normal sinus rhythm."], "s1": [0, 1, 5, 6], "s2": [2, 3, 4, 7, 8, 9, 10, 11, 12, 13, 14]} {"key": 301, "questions": "Which of the following is the most likely diagnosis?", "options": [{"label": "A", "disease": "Patent ductus arteriosus"}, {"label": "B", "disease": "Benign heart murmur"}, {"label": "C", "disease": "Atrial septal defect"}, {"label": "D", "disease": "Pulmonary valve stenosis"}], "answer_idx": "C", "symptoms": ["A 3-year-old boy is brought to the physician for a well-child examination.", "He feels well.", "He was born at 38 weeks' gestation and weighed 2766 g (6 lb 2 oz).", "He now weighs 14 kg (31 lbs).", "There is no personal or family history of serious illness.", "His immunizations are up-to-date.", "He is at 60th percentile for height and 55th percentile for weight.", "Vital signs are within normal limits.", "The lungs are clear to auscultation.", "A grade 3/6 systolic ejection murmur is heard along the upper left sternal border.", "S2 is widely split and does not vary with respiration.", "There is a grade 2/6 mid-diastolic murmur along the lower left sternal border.", "The abdomen is soft and nontender; there is no organomegaly."], "s1": [0, 1, 2, 3, 4, 5, 6, 7, 8, 12], "s2": [9, 10, 11]} {"key": 302, "questions": "Which of the following is the most likely diagnosis in this patient?", "options": [{"label": "A", "disease": "Paranoid personality disorder"}, {"label": "B", "disease": "Schizophrenia"}, {"label": "C", "disease": "Drug abuse"}, {"label": "D", "disease": "Brain tumor"}], "answer_idx": "B", "symptoms": ["A 19-year-old man is brought to a psychiatrist by his mother.", "The patient’s mother has become increasingly concerned about him since his high school graduation.", "Although she says that he had been an excellent student in high school, his grades have begun slipping in his second year in college, and he no longer socializes with friends.", "When she asked him why he no longer talks to his friends, she states that he told her, “They already know where I am.", "They can see me.", "They watch me from my bedroom window, so they already know what I’m doing.", "We don’t need to talk.” She found this to be bizarre, as nearly all of his friends live hours away.", "The patient states that he feels fine and doesn’t understand why everyone thinks he is crazy.", "The psychiatrist notices reduced spontaneous movements while the patient is speaking and that he speaks in a monotone with a voice that is soft in volume.", "He also appears suspicious and defensive, asking the psychiatrist “why are you asking me that?” after each evaluative question.", "His mood is generally flat with little reactivity or engagement during the course of his interview, but he denies any symptoms of depression or psychosis.", "No significant past medical history.", "Family history is not available as he was adopted."], "s1": [0, 1, 2, 11, 12], "s2": [3, 4, 5, 6, 7, 8, 9, 10]} {"key": 303, "questions": "What disease is the child suffering from?", "options": [{"label": "A", "disease": "X-linked agammaglobulinemia"}, {"label": "B", "disease": "Isolated IgA deficiency"}, {"label": "C", "disease": "Severe combined immunodeficiency"}, {"label": "D", "disease": "DiGeorge syndrome"}], "answer_idx": "C", "symptoms": ["A 2-year-old Caucasian male presents with recurrent diaper rash and oral thrush.", "He has also experienced recurrent episodes of otitis media from repeat Streptoccous pneumoniae.", "He notably has reduced T and B cell counts."], "s1": [0], "s2": [1, 2]} {"key": 304, "questions": "Which of the following is most likely this patient’s diagnosis?", "options": [{"label": "A", "disease": "Substance use-related aggression"}, {"label": "B", "disease": "Conduct disorder"}, {"label": "C", "disease": "Antisocial personality disorder"}, {"label": "D", "disease": "Borderline personality disorder"}], "answer_idx": "C", "symptoms": ["A 24-year-old man presents to the office for a physical exam and drug screening requested by his parole officer.", "He has a history of multiple run-ins with the law, including theft, destruction of property, and assault and battery.", "Patient confesses that he has always been the 'problem child' in his home and always got into trouble with authority figures.", "Past medical records reveal testimonies from his mother describing episodes of physically aggressive behavior in school starting at before 12.", "Although a single parent, his mother states there were no stressful changes or issues at home.", "Today's urine toxicology screens are negative."], "s1": [0, 5], "s2": [1, 2, 3, 4]} {"key": 305, "questions": "Which of the following is most likely the diagnosis?", "options": [{"label": "A", "disease": "Benign paroxysmal positional vertigo"}, {"label": "B", "disease": "Meniere disease"}, {"label": "C", "disease": "Migraine"}, {"label": "D", "disease": "Transient ischemic attack"}], "answer_idx": "B", "symptoms": ["A 56-year-old woman presents to her primary care physician for dizziness.", "She says that her symptoms began approximately 1 month prior to presentation and are associated with nausea and ringing of the ears.", "She describes her dizziness as a spinning sensation that has a start and end and lasts approximately 25 minutes.", "This dizziness is severe enough that she cannot walk or stand.", "Her symptoms are fluctuating, and she denies any tunnel vision or feelings of fainting.", "However, she has abruptly fallen to the ground in the past and denied losing consciousness.", "Medical history is significant for a migraine with aura treated with zolmitriptan, type 2 diabetes managed with metformin, and hypertension managed with lisinopril.", "Her blood pressure is 125/75 mmHg, pulse is 88/min, and respirations are 16/min.", "On physical exam, when words are whispered into her right ear, she is unable to repeat the whispered words aloud; however, this is normal in the left ear.", "Air conduction is greater than bone conduction with Rinne testing, and the sound of the tuning fork is loudest in the left ear on Weber testing."], "s1": [0, 1, 2, 3, 4, 5], "s2": [6, 7, 8, 9]} {"key": 306, "questions": "Which of the following is the most likely cause of the patient’s condition?", "options": [{"label": "A", "disease": "Microdeletion of the long arm of chromosome 22"}, {"label": "B", "disease": "Microdeletion of the long arm of chromosome 7"}, {"label": "C", "disease": "Microdeletion of the short arm of chromosome 5"}, {"label": "D", "disease": "Adenosine deaminase deficiency"}], "answer_idx": "A", "symptoms": ["A 42-year-old primigravida woman goes into labor at 37 weeks.", "After several hours of labor, a boy is born with multiple physical abnormalities including cleft palate, micrognathia, and low-set ears.", "Imaging reveals an absence of the thymic shadow."], "s1": [0], "s2": [1, 2]} {"key": 307, "questions": "Which of the following is the most likely cause of this patient's disorder?", "options": [{"label": "A", "disease": "Bilateral fibromuscular dysplasia"}, {"label": "B", "disease": "Conn syndrome"}, {"label": "C", "disease": "Renin secreting tumor"}, {"label": "D", "disease": "Unilateral fibromuscular dysplasia"}], "answer_idx": "A", "symptoms": ["A 25-year-old woman presents to her primary care physician for her annual physical exam.", "Her prior medical history is significant for seasonal allergies and a broken arm at age 12 that was treated in a long arm cast.", "She has not had any major illnesses in the previous year and does not currently have any major complaints.", "On physical exam her blood pressure is found to be 152/95 mmHg.", "Laboratory findings reveal elevated levels of renin and aldosterone.", "Given her elevated blood pressure, she is prescribed captopril; however, 1 week later she presents to the emergency department with hypertensive urgency.", "At that point captopril is immediately stopped."], "s1": [0, 1, 2], "s2": [3, 4, 5, 6]} {"key": 308, "questions": "Which of the following is the most likely diagnosis in this patient?", "options": [{"label": "A", "disease": "Chiari II malformation"}, {"label": "B", "disease": "Chiari III malformation"}, {"label": "C", "disease": "Dandy-Walker malformation"}, {"label": "D", "disease": "Klippel Feil syndrome"}], "answer_idx": "C", "symptoms": ["A 14-month-old girl is brought to the pediatrician after her parents noticed her being increasingly irritable with frequent vomiting for the past 2 weeks.", "Parents deny any history of fever.", "They recently immigrated to the country and, unfortunately, the mother did not receive prenatal care during pregnancy due to a lack of health insurance.", "Physical examination is unremarkable except for a head circumference over 2 standard deviations larger than the mean for her age, and delay in meeting motor developmental milestones.", "A magnetic resonance imaging (MRI) of her brain shows an enlargement of the posterior fossa, cystic dilation of the 4th ventricle, and hypoplasia of the cerebellar vermis (see image)."], "s1": [0, 1], "s2": [2, 3, 4]} {"key": 309, "questions": "What is the most likely cause of her recent mental impairment?", "options": [{"label": "A", "disease": "Delirium"}, {"label": "B", "disease": "Substance abuse"}, {"label": "C", "disease": "Brain metastasis"}, {"label": "D", "disease": "Delirium tremens"}], "answer_idx": "A", "symptoms": ["A 67-year-old woman with a recently found hepatic carcinoma has been an inpatient for more than a week.", "During morning rounds, she was abnormally disoriented to time and place along with a shortened attention span.", "The overnight resident reports that she was coming in and out of this state for the past couple nights.", "The patient was known to drink often, but reported that she has not consumed alcohol in weeks leading up to her admission.", "She is not jaundiced and has no metabolic imbalance upon laboratory tests."], "s1": [0, 3, 4], "s2": [1, 2]} {"key": 310, "questions": "Which of the following is the most likely cause of this patient's symptoms?", "options": [{"label": "A", "disease": "Dislodged otoliths"}, {"label": "B", "disease": "Acoustic neuroma"}, {"label": "C", "disease": "Endolymphatic hydrops"}, {"label": "D", "disease": "Posterior inferior cerebellar artery infarction"}], "answer_idx": "C", "symptoms": ["A 51-year-old man comes to the physician because of recurrent episodes of dizziness, tinnitus, and hearing loss on the left side for 6 weeks.", "These episodes last for hours at a time and are associated with the sensation that the room is spinning.", "He has no history of major medical illness and takes no medications.", "Examination shows horizontal nystagmus to the left.", "Weber test shows lateralization to the right ear.", "The Rinne test is positive bilaterally."], "s1": [0, 1, 3], "s2": [2, 4, 5]} {"key": 311, "questions": "Which of the following is the most likely diagnosis for this patient?", "options": [{"label": "A", "disease": "DiGeorge syndrome"}, {"label": "B", "disease": "Hereditary angioedema"}, {"label": "C", "disease": "Paroxysmal nocturnal hemoglobinuria"}, {"label": "D", "disease": "Chediak-Higashi syndrome"}], "answer_idx": "B", "symptoms": ["A 22-year-old woman presents with an episodic history of swelling of the face, hands, and neck and a single episode of difficulty swallowing.", "She also has a history of frequent, severe abdominal pain.", "At the time of her visit, her blood pressure is 126/84 mm Hg, heart rate is 82/min, and respiratory rate is 15/min.", "Physical examination is unremarkable except for a swollen right hand as shown in the image below.", "Imaging studies are normal and do not reveal any cause of her pain.", "Her C1 inhibitor level is less than 6% of the reference value."], "s1": [0, 1, 5], "s2": [2, 3, 4]} {"key": 312, "questions": "Which of the following is the most likely underlying cause of this patient's condition?", "options": [{"label": "A", "disease": "Left-sided systolic heart failure"}, {"label": "B", "disease": "Ventricular septal defect"}, {"label": "C", "disease": "Chronic occlusion of the pulmonary vessels"}, {"label": "D", "disease": "Chronic hypoxic vasoconstriction"}], "answer_idx": "D", "symptoms": ["A 57-year-old man with chronic obstructive pulmonary disease comes to the emergency department because of leg swelling for 2 weeks.", "He has smoked 2 packs of cigarettes daily for the past twenty years.", "His vital signs are within normal limits.", "Physical examination shows jugular venous distention, hepatomegaly, and pitting edema of both lower extremities.", "Cardiac examination shows a regular heart rate and normal heart sounds.", "Auscultation of the lungs shows scattered wheezing without crackles.", "Left ventricular ejection fraction is 60%."], "s1": [0, 3, 5], "s2": [1, 2, 4, 6]} {"key": 313, "questions": "What is the most likely diagnosis?", "options": [{"label": "A", "disease": "Dactylitis"}, {"label": "B", "disease": "Avascular necrosis"}, {"label": "C", "disease": "Osteomyelitis"}, {"label": "D", "disease": "Thrombophlebitis"}], "answer_idx": "C", "symptoms": ["A young Mediterranean teen brings her 4-year-old little brother to the Emergency Room because of a high temperature.", "Their parents are on their way to the hospital, but, in the meantime, she provides some of the history.", "She explains that he has been running a fever and limping for the past week.", "This morning, she had trouble awakening him, and she noticed some swelling and redness around his right ankle.", "In terms of his past medical history, she knows he has something wrong with his blood.", "He had to receive extra vaccinations as an infant, and he takes an antibiotic everyday.", "On exam in the ED, his temperature is 102.4 deg F (39.1 deg C), blood pressure is 90/60 mmHg, pulse is 123/min, and respirations are 22/min.", "He is lethargic, and his exam is noteworthy for erythema and edema in an area surrounding his distal shin."], "s1": [0, 1, 2, 4, 5], "s2": [3, 6, 7]} {"key": 314, "questions": "What is the most likely cause of this patient’s condition?", "options": [{"label": "A", "disease": "Adenosine deaminase deficiency"}, {"label": "B", "disease": "Hypoxanthine-guanine phosphoribosyl transferase (HGPRT) deficiency"}, {"label": "C", "disease": "HIV infection"}, {"label": "D", "disease": "Purine nucleoside phosphorylase deficiency"}], "answer_idx": "A", "symptoms": ["An 8-month-old boy is brought to the office by his mother for recurrent infections.", "Over the past 2 months, the boy had multiple visits to the urgent care clinic for respiratory, ear, and skin infections.", "His mother is concerned about the health of her child.", "Currently, the child had a runny nose for the last 2 days.", "There is no fever but the mother adds that the boy is not eating very well.", "His mother denies any history of infection during her pregnancy and was tested negative for HIV.", "The patient’s heart rate is 90/min, respiratory rate is 14/min, and temperature is 36.7°C (98.0°F).", "On physical exam, there are decreased lung sounds in the left lower lobe.", "A chest X-ray reveals an absent thymic shadow and fails to show any lung pathology.", "There is no history of similar symptoms in the families of either parent."], "s1": [0, 1, 2, 3, 4, 5, 9], "s2": [6, 7, 8]} {"key": 315, "questions": "Which of the following is the most likely diagnosis?", "options": [{"label": "A", "disease": "Femoral artery dissection"}, {"label": "B", "disease": "Femoral artery pseudoaneurysm"}, {"label": "C", "disease": "Femoral artery thrombosis"}, {"label": "D", "disease": "Femoral abscess"}], "answer_idx": "B", "symptoms": ["A 56-year-old man comes to the physician for a routine health maintenance examination.", "He has had mild pain in his left groin for 2 weeks.", "He was admitted to the hospital 1 month ago for myocardial infarction.", "Cardiac catheterization and angiography showed occlusion of the left anterior descending artery and he underwent placement of 2 stents.", "He has hypertension and hypercholesterolemia.", "There is no family history of serious illness.", "He has smoked 2 packs of cigarettes daily for 30 years.", "Current medications include aspirin, clopidogrel, rosuvastatin, and enalapril.", "His temperature is 36.7°C (98°F), pulse is 88/min, and blood pressure is 130/84 mm Hg.", "Examination shows a 3-cm (1.2-in), tender, pulsatile mass in the left thigh, below the inguinal ligament.", "There is mild erythema of the overlying skin.", "A loud murmur is heard on auscultation of the mass.", "Cardiopulmonary examination shows no abnormalities.", "There is no edema in the lower limbs.", "Femoral and pedal pulses are palpable bilaterally.", "Neurologic examination shows no focal findings."], "s1": [0, 2, 3, 4, 5, 6, 7, 8, 12, 13, 14, 15], "s2": [1, 9, 10, 11]} {"key": 316, "questions": "Which of the following is the most appropriate next step in diagnosis?", "options": [{"label": "A", "disease": "Intravenous pyelography"}, {"label": "B", "disease": "Dynamic renal scintigraphy"}, {"label": "C", "disease": "Urodynamic testing"}, {"label": "D", "disease": "Voiding cystourethrography"}], "answer_idx": "D", "symptoms": ["A 1-year-old girl is brought to the physician because of fever and crying while passing urine for 2 days.", "She was born at term and has been healthy since.", "Three months ago, she was treated for a urinary tract infection with oral cefixime.", "Her temperature is 39°C (102.2°F), pulse is 144/min, and blood pressure is 85/40 mm Hg.", "Physical examination shows no abnormalities.", "Her leukocyte count is 14,000/mm3.", "Urine dipstick shows leukocyte esterase and nitrites; urinalysis shows WBCs and gram-negative rods.", "Urine culture results are pending.", "Renal ultrasonography shows hydronephrosis of the left kidney.", "Empirical antimicrobial therapy is initiated, following which the patient's symptoms improve."], "s1": [0, 1, 8], "s2": [2, 3, 4, 5, 6, 7, 9]} {"key": 317, "questions": "Which of the following is the most likely cause of this patient’s condition?", "options": [{"label": "A", "disease": "Renal artery stenosis"}, {"label": "B", "disease": "Essential hypertension"}, {"label": "C", "disease": "Coarctation of the aorta"}, {"label": "D", "disease": "Primary aldosteronism"}], "answer_idx": "A", "symptoms": ["An otherwise healthy 27-year-old woman presents to the emergency department because of worsening headaches and nosebleeds.", "Her blood pressure is 185/115 mm Hg, pulse is 88/min and temperature is 36.9°C (98.4°F).", "She denies the use of nicotine, alcohol, or illicit drugs.", "Family history is irrelevant.", "Physical examination is only positive for abdominal bruits.", "She is given a calcium-channel blocker and a thiazide diuretic, but her blood pressure did not respond adequately.", "Aldosterone-to-renin ratio is < 20."], "s1": [0, 1, 4, 5, 6], "s2": [2, 3]} {"key": 318, "questions": "Which of the following is the most likely diagnosis?", "options": [{"label": "A", "disease": "Mycetoma"}, {"label": "B", "disease": "Sporotrichosis"}, {"label": "C", "disease": "Entomophthoromycoses"}, {"label": "D", "disease": "Chromoblastomycosis"}], "answer_idx": "D", "symptoms": ["A 50-year-old farmer from Jamaica presents to his physician with a wart-like lesion on his left foot.", "He is a plantain farmer and often farms barefoot because of the pleasant climate.", "Physical examination reveals warty, cutaneous nodules that resemble the florets of cauliflower.", "On a KOH preparation, irregular, dark brown, yeast-like bodies with septae are noted.", "Culture on Sabouraud agar reveals sclerotic bodies."], "s1": [0, 1], "s2": [2, 3, 4]} {"key": 319, "questions": "What was the most likely diagnosis of this patient?", "options": [{"label": "A", "disease": "Follicular cyst"}, {"label": "B", "disease": "Endometrioma"}, {"label": "C", "disease": "Corpus luteum cyst"}, {"label": "D", "disease": "Dermoid cyst"}], "answer_idx": "C", "symptoms": ["A mother brings her 17-year-old daughter to your office because of a recent onset dull unilateral pelvic pain that started last week.", "In addition, the daughter also complains of constipation for which she increased her fiber intake, but the dietary changes are not helping her.", "She began menstruating at the age of 13, and her menstrual cycles are regular (about 28 days).", "On physical examination, the patient has a large, non-tender, mobile adnexal mass in the left lower quadrant.", "The pelvic examination is normal except for the presence of the adnexal mass.", "The patient is advised to have an outpatient ultrasound, and then return to the clinic next week.", "After 2 weeks, you learned that the patient visited the emergency department for a sudden onset of abdominal pain that required observation after an abdominal ultrasound was performed.", "The ultrasound showed a large simple cystic mass with increased internal echoes, having a diameter of 10 cm and hyperechogenic vascular walls on the left ovary with a small intraperitoneal collection in the pouch of Douglas."], "s1": [0, 1, 2, 5], "s2": [3, 4, 6, 7]} {"key": 320, "questions": "Which of the following is the most likely cause of the patient's symptoms?", "options": [{"label": "A", "disease": "Squamous cell lung carcinoma"}, {"label": "B", "disease": "Small cell lung cancer"}, {"label": "C", "disease": "Lung adenocarcinoma"}, {"label": "D", "disease": "Pulmonary metastases"}], "answer_idx": "D", "symptoms": ["A 65-year-old man comes to the physician because of progressive shortness of breath and a worsening cough for the past month.", "He also reports occasional bloody sputum.", "He has lost about 7 kg (15.4 lb) of weight over the past 4 months despite having no change in appetite.", "He has smoked 2 packs of cigarettes daily for 15 years but stopped smoking at the age of 55.", "Physical examination shows reduced breath sounds throughout both lungs.", "An x-ray of the chest is shown."], "s1": [2, 3], "s2": [0, 1, 4, 5]} {"key": 321, "questions": "Which of the following is the most likely underlying cause of this patient's shoulder pain?", "options": [{"label": "A", "disease": "Infection of the joint space"}, {"label": "B", "disease": "Crystal deposition within the joint"}, {"label": "C", "disease": "Infarction of the bone trabeculae"}, {"label": "D", "disease": "Infection of the bone"}], "answer_idx": "C", "symptoms": ["A 13-year-old African-American girl is brought to the physician for right shoulder pain that has worsened over the past month.", "She has had many episodes of joint and bone pain and recurrent painful swelling in her hands and feet.", "Physical examination shows tenderness of the right anterior humerus without swelling or skin changes.", "Active and passive range of motion of the right shoulder is decreased and there is pain with movement.", "The leukocyte count is 4600/mm3.", "An x-ray of the right shoulder shows subchondral lucency of the humeral head with sclerosis and joint space narrowing."], "s1": [0, 2, 3], "s2": [1, 4, 5]} {"key": 322, "questions": "Which of the following is the most likely diagnosis?", "options": [{"label": "A", "disease": "Adenomyosis"}, {"label": "B", "disease": "Cervical cancer"}, {"label": "C", "disease": "Endometriosis"}, {"label": "D", "disease": "Leiomyoma"}], "answer_idx": "D", "symptoms": ["A 43-year-old woman presents to the clinic complaining of “spotting.” She states that over the past 5 months she has had intermenstrual bleeding.", "Additionally, her menses last more than a week with the first 4 days consisting of heavy bleeding.", "She denies abdominal pain but reports feeling an increasing “pressure-like\" discomfort.", "She denies dysuria, dyspareunia, or dyschezia.", "Prior to 5 months ago, her menstrual periods were only 4 days long and consisted of moderate bleeding.", "Her periods normally occur every 28 days.", "She is frustrated because she feels like she needs to wear a pad all the time.", "The patient delivered 3 healthy children vaginally without complications, followed by a tubal ligation.", "Her last menstrual period was 1 week ago.", "She is sexually active with her husband and denies a history of sexually transmitted diseases.", "She has never had an abnormal pap smear.", "Her last pap smear and test for human papillomavirus was 4 years ago.", "The patient has hypothyroidism treated with levothyroxine.", "On physical examination, the thyroid is normal in size without masses or tenderness.", "Pelvic examination reveals a firm, mobile, enlarged, irregularly-shaped uterus without adnexal tenderness.", "Speculum exam is unremarkable."], "s1": [0, 1, 2, 4, 5, 6, 7, 8, 9, 10, 11, 14, 15], "s2": [3, 12, 13]} {"key": 323, "questions": "Which of the following is the most likely underlying condition?", "options": [{"label": "A", "disease": "Adenocarcinoma of the sigmoid colon"}, {"label": "B", "disease": "Cutaneous malignant melanoma"}, {"label": "C", "disease": "Multiple endocrine neoplasia"}, {"label": "D", "disease": "Squamous cell carcinoma of the esophagus\n\""}], "answer_idx": "B", "symptoms": ["A 56-year-old woman comes to the emergency department because of severe abdominal pain associated with nausea and vomiting for 12 hours.", "The pain extends from the epigastrium to the right upper quadrant and radiates to the right scapula.", "The patient has gastroesophageal reflux disease.", "She underwent total thyroidectomy for papillary thyroid carcinoma 3 years ago.", "She has smoked one pack of cigarettes daily for 35 years.", "Current medications include levothyroxine, omeprazole, and a multivitamin.", "The patient appears uncomfortable.", "Her temperature is 38.1°C (100.6°F), pulse is 93/min, and blood pressure is 140/85 mm Hg.", "Abdominal examination shows tenderness to deep palpation in the right upper quadrant.", "The patient's leukocyte count is 10,300/mm3.", "Abdominal ultrasonography shows thickening of the gallbladder wall, without dilatation of the intra- and extrahepatic bile ducts.", "Laparoscopic cholecystectomy is performed.", "Pathological examination shows acute cholecystitis.", "There is an ulcerated 1 x 1.5-cm mass in the fundus of the gallbladder with invasion of the gallbladder wall and lymphatic vessels."], "s1": [0, 1, 6, 7, 8, 9, 10, 11, 12], "s2": [2, 3, 4, 5, 13]} {"key": 324, "questions": "Which of the following is the most likely cause of this patient's symptoms?\"", "options": [{"label": "A", "disease": "Cerebral salt wasting syndrome"}, {"label": "B", "disease": "Syndrome of inappropriate antidiuretic hormone"}, {"label": "C", "disease": "Primary polydipsia"}, {"label": "D", "disease": "Mineralocorticoid deficiency"}], "answer_idx": "B", "symptoms": ["A 58-year-old woman is brought to the emergency department by her husband because of increasing confusion and generalized fatigue over the past week.", "During this period, she has had muscle cramps, headaches, nausea, and vomiting.", "Six days ago, she underwent resection of a parasagittal meningioma.", "She tolerated the procedure well without complications and was discharged two days ago.", "She has hypercholesterolemia and hypertension.", "Current medications include oxycodone, acetaminophen, atorvastatin, and hydrochlorothiazide.", "She is oriented only to person and place.", "Her temperature is 37°C (98.6°F), pulse is 89/min, respiratory rate is 14/min, and blood pressure is 142/88 mm Hg.", "Pulse oximetry on room air shows an oxygen saturation of 97%.", "The remainder of the examination shows no abnormalities.", "Serum studies show: | Serum | Na+ 126 mEq/L | K+ 3.6 mEq/L | Cl- 107 mEq/L | Urea nitrogen 16 mg/dL | Glucose 87 mg/dL | Creatinine 0.9 mg/dL | Osmolality 255 mOsmol/kg H2O | Urine | Osmolality 523 mOsmol/kg H2O | Sodium 44 mEq/L | ."], "s1": [0, 1, 6, 7, 8, 10], "s2": [2, 3, 4, 5, 9]} {"key": 325, "questions": "This patient most likely presents with clinical features of which disease?", "options": [{"label": "A", "disease": "Ankylosing spondylitis"}, {"label": "B", "disease": "Reactive arthritis"}, {"label": "C", "disease": "Osteoma"}, {"label": "D", "disease": "Fibromyalgia"}], "answer_idx": "A", "symptoms": ["A 24-year-old man presents to his primary care physician with heel pain that has been persistent for 3 weeks now.", "He is bothered by the feeling of significant pressure and swelling in his ankle.", "He is also having difficulty walking due to pain in his hip.", "Upon physical examination, there is evidence of swelling and warmth over his Achilles tendon.", "Imaging of his sacroiliac joint reveals soft tissue involvement and narrowing of the joint spaces.", "This patient also appears to have a low to moderate kyphosis of his spine.", "He has no family history of any bone or rheumatological diseases."], "s1": [0, 1, 3], "s2": [2, 4, 5, 6]} {"key": 326, "questions": "Which of the following is associated with the most likely diagnosis?", "options": [{"label": "A", "disease": "Adrenal adenoma"}, {"label": "B", "disease": "Decreased ACTH level"}, {"label": "C", "disease": "Enlargement of the facial bones"}, {"label": "D", "disease": "History of smoking"}], "answer_idx": "D", "symptoms": ["A 48-year-old man presents to his primary care physician concerned about his weight.", "He states he has been gaining weight though does not feel he has changed his diet.", "The patient also states he has felt fatigued lately and not himself.", "He denies taking any medications or using any illicit substances at baseline.", "His temperature is 99.2°F (37.3°C), blood pressure is 177/108 mmHg, pulse is 100/min, respirations are 18/min, and oxygen saturation is 98% on room air.", "Physical exam is notable for an obese man with striae on his abdomen.", "A 24-hour urine free cortisol is collected and demonstrates an elevated cortisol level; however, a high dose of dexamethasone when administered results in no attenuation of cortisol release.", "A CT scan of the abdomen is within normal limits."], "s1": [0, 1, 3, 4, 7], "s2": [2, 5, 6]} {"key": 327, "questions": "What is the most likely cause of this lab abnormality in this patient?", "options": [{"label": "A", "disease": "Trisomy 13"}, {"label": "B", "disease": "Trisomy 18"}, {"label": "C", "disease": "Trisomy 21"}, {"label": "D", "disease": "Inaccurate gestational age"}], "answer_idx": "D", "symptoms": ["A 24-year-old woman who is 16 weeks pregnant comes to your office requesting a referral for an abortion.", "Upon questioning she states that she is scared that her baby will be \"deformed\" because on routine screening she was found to have an elevated level of alpha-fetoprotein."], "s1": [0], "s2": [1]} {"key": 328, "questions": "Of the following options, which is the most likely diagnosis?", "options": [{"label": "A", "disease": "Erysipelas"}, {"label": "B", "disease": "Facial lymphedema"}, {"label": "C", "disease": "Drug-induced angioedema"}, {"label": "D", "disease": "Contact dermatitis"}], "answer_idx": "C", "symptoms": ["A 63-year-old African American man with a history of hypertension and dyslipidemia presents to the emergency department with facial swelling and difficulty breathing.", "Symptoms began suddenly that morning and continued to worsen.", "He started taking lisinopril several weeks ago.", "His blood pressure is 110/74 mm Hg, heart rate is 94/min, and respiratory rate is 20/min.", "Physical examination is notable for swelling of his lips and inspiratory stridor.", "Clinical lab results suggest a normal C1 esterase inhibitor level."], "s1": [0, 4], "s2": [1, 2, 3, 5]} {"key": 329, "questions": "What is the most likely diagnosis?", "options": [{"label": "A", "disease": "Schizoaffective disorder"}, {"label": "B", "disease": "Major depression"}, {"label": "C", "disease": "Schizophrenia"}, {"label": "D", "disease": "Schizophreniform disorder"}], "answer_idx": "C", "symptoms": ["A 20-year-old college student is brought to his primary care physician by his parents over the New Years holiday because they are concerned about his behavior since right after high school graduation in May the spring before.", "Prior to this year, he was generally an outgoing and social young man, who now reportedly has been spending the majority of his time alone in his dorm room and room at home, where his mother and father have overheard him talking to other people when he has no guests.", "When asked if anything was bothering him at school, he reports that at night the voice of the school's mascot encourages him to save the school from the large oak grove that will soon takeover the student union building by setting the trees on fire for all to see.", "On exam, his appears to be otherwise healthy and his urine toxicology screen is negative."], "s1": [0, 3], "s2": [1, 2]} {"key": 330, "questions": "Which of the following is the most likely diagnosis?", "options": [{"label": "A", "disease": "Squamous cell carcinoma"}, {"label": "B", "disease": "Aphthous stomatitis"}, {"label": "C", "disease": "Psoriasis"}, {"label": "D", "disease": "Seborrheic keratosis"}], "answer_idx": "A", "symptoms": ["A 60-year-old man comes to the physician because of a 3-month history of a slowly enlarging painless ulcer on his lower lip.", "One year ago he had a painful rash near his upper lip that resolved completely with acyclovir therapy.", "He has a 10-year history of type 2 diabetes mellitus treated with metformin.", "He has worked in construction for the past 33 years.", "He has smoked one pack of cigarettes daily for 35 years.", "Examination shows a 0.5 x 0.5 cm nontender ulcer with everted edges on the midline of his lower lip.", "The erythematous ulcer bleeds upon palpation.", "He has numerous moles of varying sizes all over his body.", "There are no palpable cervical lymph nodes."], "s1": [0, 5, 6, 8], "s2": [1, 2, 3, 4, 7]} {"key": 331, "questions": "Which of the following is the patient most at risk for developing?", "options": [{"label": "A", "disease": "Ataxia"}, {"label": "B", "disease": "Atopic dermatitis"}, {"label": "C", "disease": "Blurry vision"}, {"label": "D", "disease": "Scant tonsils"}], "answer_idx": "C", "symptoms": ["A 3-year-old girl is brought to the pediatrician by her father for fever and cough.", "The patient’s father states that she had a sore throat and runny nose 2 weeks ago.", "Then last night she developed a productive cough and a fever.", "The father is worried that this is pneumonia again, and reports that she has been hospitalized 5 times already with pneumonia.", "Her medical history is also significant for chronic diarrhea.", "The father reports that the patient has 2 older brothers who are both healthy.", "The patient’s temperature is 102°F (38.9°C), blood pressure is 102/60 mmHg, pulse is 110/min, and respirations are 28/min with an oxygen saturation of 94% on room air.", "On physical examination, decreased breath sounds are appreciated on the right.", "A chest radiography shows consolidation in the right upper lobe, consistent with pneumonia.", "An antibody panel and flow cytometry are obtained in the setting of the patient’s recurrent infections.", "The results show a normal lymphocyte count with low levels of IgA, IgG, and IgE and elevated levels of IgM."], "s1": [0, 1, 2, 6, 7, 8], "s2": [3, 4, 5, 9, 10]} {"key": 332, "questions": "Which of the following is the most likely etiology of his bruises?", "options": [{"label": "A", "disease": "Platelet aggregation inhibition"}, {"label": "B", "disease": "Intravascular coagulation with fibrinolysis"}, {"label": "C", "disease": "Nonaccidental trauma"}, {"label": "D", "disease": "Vitamin K deficiency"}], "answer_idx": "C", "symptoms": ["An 82-year-old man with Alzheimer disease is brought to the physician by his daughter because of extensive bruising on his arms.", "He lives in an assisted living facility.", "Physical examination shows large ecchymoses across both forearms and upper arms in varied stages of healing.", "His hemoglobin concentration is 11.7 g/dL, mean corpuscular volume is 78 μm3, and platelet count is 180,000/mm3.", "Coagulation studies are within normal limits."], "s1": [0, 1], "s2": [2, 3, 4]} {"key": 333, "questions": "Which of the following best describe the primary disorder in this patient?", "options": [{"label": "A", "disease": "Obstruction of the lumen of the appendix"}, {"label": "B", "disease": "Inflammation of the renal parenchyma, calyces and pelvis"}, {"label": "C", "disease": "Herniation of mucosa and submucosa through the muscular layer of the colon"}, {"label": "D", "disease": "Inflammation of the upper genital tract"}], "answer_idx": "C", "symptoms": ["A 57-year-old woman presents to the emergency department with acute pain in the left lower abdomen associated with nausea and vomiting for the past 24 hours.", "Prior to this episode, the patient did not have any significant gastrointestinal (GI) problems except for occasional constipation and indigestion after heavy meals.", "She has had hypertension and hypercholesterolemia for the past 7 years.", "Her family history is negative for GI disorders.", "Vital signs include a temperature of 38.0°C (100.4°F), blood pressure of 120/80 mm Hg, and pulse of 85/min.", "On physical examination, there is tenderness in the left lower abdominal quadrant.", "Abdominal CT scan shows thickening of the bowel wall and streaky mesenteric fat."], "s1": [0, 1, 5, 6], "s2": [2, 3, 4]} {"key": 334, "questions": "Which of the following is the most likely diagnosis?", "options": [{"label": "A", "disease": "Cluster headache"}, {"label": "B", "disease": "Trigeminal neuralgia"}, {"label": "C", "disease": "Postherpetic neuralgia"}, {"label": "D", "disease": "Temporal arteritis"}], "answer_idx": "B", "symptoms": ["A 35-year-old woman comes to the emergency department because of a 3-day history of pain in the left cheek.", "The pain occurs every few hours, lasts 30–60 seconds, and is aggravated by chewing and brushing.", "She has a history of pain with a vesicular rash in the right axillary area one year ago.", "She had an upper respiratory infection 2 weeks ago.", "Physical examination shows no abnormalities."], "s1": [0, 1, 2], "s2": [3, 4]} {"key": 335, "questions": "What is the most likely diagnosis?", "options": [{"label": "A", "disease": "Subdural hematoma"}, {"label": "B", "disease": "Epidural hematoma"}, {"label": "C", "disease": "Subgaleal hematoma"}, {"label": "D", "disease": "Caput succedaneum"}], "answer_idx": "B", "symptoms": ["A 35-year-old is brought into the emergency room after a bicycle vs motored vehicle accident.", "He was unconscious after hitting the side of his head on the road, but recovered consciousness after a few minutes.", "He started to ride his bicycle again, but then developed a severe headache with nausea and vomiting, at which time he called EMS.", "While in the hospital, the patient is confused and cannot provide a history.", "He quickly loses consciousness and his CT scan is displayed in Figure 1."], "s1": [0], "s2": [1, 2, 3, 4]} {"key": 336, "questions": "What is the diagnosis in this patient?", "options": [{"label": "A", "disease": "Acute pancreatitis"}, {"label": "B", "disease": "Chronic pancreatitis"}, {"label": "C", "disease": "Acute cholangitis"}, {"label": "D", "disease": "Primary sclerosing cholangitis"}], "answer_idx": "C", "symptoms": ["A 42-year-old female presents to the emergency department with two days of severe abdominal pain and fever.", "The patient has a history of gallstones, for which she was scheduled to have a cholecystectomy in two weeks.", "On physical exam, her abdomen is tender to palpation over the epigastrium, without rebound or guarding.", "The patient is noticeably jaundiced.", "Laboratory evaluation is notable for a leukocytosis and a total bilirubin of 6.4 mg/dL."], "s1": [0, 1], "s2": [2, 3, 4]} {"key": 337, "questions": "What is the most likely cause of this patient’s high blood pressure?", "options": [{"label": "A", "disease": "Atherosclerosis"}, {"label": "B", "disease": "Adult polycystic kidney disease"}, {"label": "C", "disease": "Fibromuscular hyperplasia"}, {"label": "D", "disease": "Pheochromocytoma"}], "answer_idx": "C", "symptoms": ["A 25-year-old arrives at a gynecology clinic for a consult as she and her husband plan to conceive a baby soon.", "She is currently taking no medications, doesn’t smoke, and drinks alcohol socially.", "Her father passed away at the age of 51 due to heart failure.", "Her vital signs include blood pressure 150/100 mm Hg, heart rate 67/min, respiratory rate 16/min, and temperature 36.7°C (98.0°F).", "Physical examination is unremarkable.", "When asked about her high blood pressure, the patient says that her primary care physician once told her that her blood pressure was elevated on most of her visits; the physician emphasized that she should exercise and control her diet, but she never followed up with him as she always felt good.", "A number of tests are run to find out the etiology of her elevated blood pressure.", "Everything is within the normal range except for a renal angiogram shown in the picture."], "s1": [0, 1, 2, 4], "s2": [3, 5, 6, 7]} {"key": 338, "questions": "Which of the following is the most likely diagnosis?", "options": [{"label": "A", "disease": "H. pylori infection"}, {"label": "B", "disease": "Intestinal type gastric adenocarcinoma"}, {"label": "C", "disease": "NSAID induced peptic ulcer disease"}, {"label": "D", "disease": "Zollinger-Ellison syndrome"}], "answer_idx": "D", "symptoms": ["A 53-year-old man presents to the office complaining of persistent abdominal pain.", "He states that he has gained 4.5 kg (10.0 lb) because the pain is only relieved by eating.", "The patient also admits that he has had numerous loose, foul-smelling stools over the last few weeks with associated nausea.", "He has taken some over the counter antacids without relief or change in symptoms.", "His blood pressure is 132/76 mm Hg, respiratory rate is 14/min, and heart rate is 75/min.", "Physical examination reveals mild tenderness to palpation of the upper abdomen."], "s1": [0, 1, 2, 3, 5], "s2": [4]} {"key": 339, "questions": "Which of the following etiologies is the most likely cause of this patient’s neurologic findings?", "options": [{"label": "A", "disease": "Brown-Séquard syndrome"}, {"label": "B", "disease": "Spinal cord compression"}, {"label": "C", "disease": "Anterior cord syndrome"}, {"label": "D", "disease": "Spinal epidural hematoma"}], "answer_idx": "C", "symptoms": ["A 56-year-old man presents with severe back pain for the past hour.", "He says the pain started about 1 hour ago and has not improved.", "The patient describes the pain as sharp, 10/10, and located between his scapula.", "Past medical history is significant for hypertension, which he is noncompliant with medication.", "His vital signs are a blood pressure of 180/95 mm Hg and a pulse of 108/min.", "On physical examination, there is a loss of pain and temperature sensation in the lower trunk and extremities bilaterally.", "Strength is 4 out of 5 in all muscle groups in the lower extremities bilaterally.", "A contrast CT of the chest reveals findings consistent with a type B aortic dissection."], "s1": [0, 1, 2, 3, 4], "s2": [5, 6, 7]} {"key": 340, "questions": "What is the most likely diagnosis?", "options": [{"label": "A", "disease": "Angelman syndrome"}, {"label": "B", "disease": "Rett syndrome"}, {"label": "C", "disease": "Prader-Willi syndrome"}, {"label": "D", "disease": "McCune-Albright syndrome"}], "answer_idx": "B", "symptoms": ["A 4-year-old girl is seen by her pediatrician for developmental delay.", "One year prior, the patient was able to ride a tricycle, stack 3 blocks, and speak in short sentences.", "Now, she is unable to feed herself and has recently started to point to objects she wants rather than asking.", "Physical exam reveals a well nourished child sitting in a stroller wringing her hands.", "Vital signs are normal.", "The patient's mother reports that her 7-year-old son is doing well, and that they have no family history of mental retardation or other cognitive disorders."], "s1": [0, 1, 2, 3], "s2": [4, 5]} {"key": 341, "questions": "Which of the following is the most likely diagnosis?", "options": [{"label": "A", "disease": "Uterine rupture"}, {"label": "B", "disease": "Placenta previa"}, {"label": "C", "disease": "Placenta accreta"}, {"label": "D", "disease": "Abruptio placentae\n\""}], "answer_idx": "C", "symptoms": ["A 37-year-old woman, gravida 4, para 3, at 35 weeks' gestation is admitted to the hospital in active labor.", "She has had no prenatal care.", "Her three children were delivered by lower segment transverse cesarean section.", "The patient appears in acute distress.", "Her pulse is 98/min, respirations are 15/min, and blood pressure is 130/80 mm Hg.", "The abdomen is nontender and contractions are felt.", "The fetus is in a cephalic presentation.", "The fetal heart rate is reactive with no decelerations.", "One hour following vaginal delivery, the placenta is not delivered.", "Manual separation of the placenta leads to profuse vaginal bleeding.", "A firm, nontender uterine fundus is palpated at the level of the umbilicus."], "s1": [0, 1, 2, 5, 6, 7], "s2": [3, 4, 8, 9, 10]} {"key": 342, "questions": "Which of the following is the most likely cause of this patient's current symptoms?", "options": [{"label": "A", "disease": "Decreased endolymph resorption"}, {"label": "B", "disease": "Suppurative labyrinthitis"}, {"label": "C", "disease": "Medication toxicity"}, {"label": "D", "disease": "Delirium"}], "answer_idx": "C", "symptoms": ["Six days after admission to the hospital for treatment of infective endocarditis, a 64-year-old woman develops persistent ringing in both ears, lightheadedness, and nausea.", "When she turns her head, she sees the light on the ceiling swinging from side to side.", "She has congestive heart disease, hypertension, and coronary artery disease.", "She was treated for a cerebrovascular accident 7 years ago and has no residual deficits.", "Current medications include intravenous vancomycin and gentamicin, as well as oral aspirin, atenolol, furosemide, and lisinopril.", "Her vital signs are within normal limits.", "Cardiac examination shows a grade 3/6 holosystolic murmur along the left lower sternal border.", "While the patient fixates on a target, administration of brisk, horizontal head rotations to both sides results in corrective saccades to refixate back to the target.", "She has an unsteady gait.", "Muscle strength and sensation are normal."], "s1": [0, 1, 7, 8], "s2": [2, 3, 4, 5, 6, 9]} {"key": 343, "questions": "Which of the following is the most likely underlying cause of this patient's symptoms?", "options": [{"label": "A", "disease": "Graves disease"}, {"label": "B", "disease": "Exogenous levothyroxine use"}, {"label": "C", "disease": "Granulomatous thyroiditis"}, {"label": "D", "disease": "Iodine exposure"}], "answer_idx": "B", "symptoms": ["A 75-year-old woman is brought to the physician by her daughter because of a 1-month history of fatigue and a 3.5-kg (7-lb) weight loss.", "The patient has a history of hypertension, hyperlipidemia, and Alzheimer's dementia.", "She lives with her daughter and requires assistance with all of her banking and meal preparation.", "Her daughter has hypothyroidism but there is no other history of serious illness in the family.", "She does not remember the names of her medications but says she takes them every day.", "Her pulse is 114/min and blood pressure is 148/101 mm Hg.", "Physical examination shows warm skin and a tremor.", "The thyroid gland is not palpable.", "The serum thyroid stimulating hormone level is undetectable.", "Histological examination of a thyroid biopsy shows follicular atrophy."], "s1": [0, 1, 2, 3, 4], "s2": [5, 6, 7, 8, 9]} {"key": 344, "questions": "What is the most likely explanation for this patient’s presentation?", "options": [{"label": "A", "disease": "Hereditary defect of amino acid transporter in the kidney"}, {"label": "B", "disease": "Infection with urease positive bacteria"}, {"label": "C", "disease": "Presence of antibodies to the glomerular basement membrane"}, {"label": "D", "disease": "Thinning and splitting of the glomerular basement membrane"}], "answer_idx": "A", "symptoms": ["A 23-year-old man comes to the emergency department complaining of 10/10 abdominal pain.", "He describes the pain as sharp, stabbing, intermittent, and concentrated at the left lower quadrant.", "It started about 3 hours ago with no obvious precipitating factor.", "He reports multiple similar episodes in the past but they have all resolved within 1 hour or so.", "He denies fever, weight changes, headaches, nausea/vomiting, or gastrointestinal changes but endorses “red-colored urine” for the past day.", "He just returned from an extensive hiking trip in Colorado.", "A radiograph of the abdomen was unremarkable."], "s1": [0, 1, 2, 3, 4], "s2": [5, 6]} {"key": 345, "questions": "Which of the following is the most likely cause of these findings?\"", "options": [{"label": "A", "disease": "Granular subepithelial immune complex deposition"}, {"label": "B", "disease": "Systemic amyloid protein aggregation"}, {"label": "C", "disease": "Anti-type IV collagen antibody formation"}, {"label": "D", "disease": "DNA-anti-DNA antibody immune complex formation"}], "answer_idx": "A", "symptoms": ["A 6-year-old girl is brought to the physician because of generalized fatigue and dark urine for 4 days.", "Four weeks ago, she was treated with cephalexin for a skin infection.", "Her temperature is 37°C (98.6°F) and blood pressure is 132/89 mm Hg.", "Physical examination shows 1+ pretibial edema bilaterally.", "Her serum creatinine is 1.7 mg/dL.", "Urine studies show: | Blood 2+ | Protein 2+ | RBC 12–14/hpf with dysmorphic features | RBC casts numerous | ."], "s1": [0, 1, 2], "s2": [3, 4, 5]} {"key": 346, "questions": "Which of the following is the most likely diagnosis?", "options": [{"label": "A", "disease": "Labial adhesions"}, {"label": "B", "disease": "Androgen insensitivity"}, {"label": "C", "disease": "Endometriosis"}, {"label": "D", "disease": "Imperforate hymen"}], "answer_idx": "D", "symptoms": ["A 15-year-old girl is brought to the physician by her mother because of a 3-day history of lower abdominal pain.", "Over the past 9 months, she has had multiple similar episodes of abdominal pain, each lasting for 4–5 days.", "Menarche has not yet occurred.", "Examination shows suprapubic tenderness to palpation.", "Pubic hair and breast development are Tanner stage 4.", "Pelvic examination shows bulging, bluish tissue 1 cm inside the vaginal introitus."], "s1": [1, 2, 4], "s2": [0, 3, 5]} {"key": 347, "questions": "Which of the following is the most likely explanation of the patient's laboratory findings?\"", "options": [{"label": "A", "disease": "Salicylate toxicity"}, {"label": "B", "disease": "Diabetic ketoacidosis"}, {"label": "C", "disease": "Recurrent vomiting"}, {"label": "D", "disease": "Adrenal insufficiency"}], "answer_idx": "A", "symptoms": ["A 21-year-old man is brought to the emergency department by the police because of altered mental status.", "The police found him covering his ears and shouting near a highway rest area.", "Upon questioning, he was unable to look directly at the police and answer coherently.", "On the way to the hospital, he reported feeling nauseous and vomited twice.", "His temperature is 38.2°C (100.8°F), pulse is 100/min, respirations are 28/min, and blood pressure is 110/77 mm Hg.", "He is admitted to the hospital for evaluation.", "Three hours after admission, he has a tonic-clonic seizure.", "Laboratory studies show: | Serum | Na+ 140 mEq/L | Cl- 102 mEq/L | Arterial blood gas analysis on room air: | On admission Three hours later | pH 7.47 7.39 | PaCO2 24 mm Hg 31 mm Hg | PO2 84 mm Hg 82 mm Hg | HCO3- 20 mEq/L 18 mEq/L | ."], "s1": [0, 1, 2, 3, 6], "s2": [4, 5, 7]} {"key": 348, "questions": "Which of the following is the most likely explanation for this patient's symptoms?", "options": [{"label": "A", "disease": "Infection of the dermis and subcutaneous tissue"}, {"label": "B", "disease": "Infection of the bone"}, {"label": "C", "disease": "Infarction of the bone trabeculae"}, {"label": "D", "disease": "Loss of bone mineral density"}], "answer_idx": "C", "symptoms": ["A 16-year-old African-American boy is brought to the physician because of a 2-month history of progressive right shoulder pain.", "He has had many episodes of joint and bone pain in the past, and as a child, had recurrent painful swelling of his hands and feet.", "His brother had a stroke at 6 years of age and now has an intellectual disability.", "The patient is at the 60th percentile for height and 55th percentile for weight.", "His vital signs are within normal limits.", "Physical examination shows tenderness of the right anterior humerus without noticeable swelling or skin changes.", "Active and passive range of motion of the right shoulder is decreased and there is pain with movement.", "The leukocyte count is 4600/mm3.", "An x-ray of the right shoulder shows subchondral lucency of the humeral head with sclerosis and joint space narrowing."], "s1": [0, 1, 2], "s2": [3, 4, 5, 6, 7, 8]} {"key": 349, "questions": "Which of the following is the most likely cause of this patient's symptoms?", "options": [{"label": "A", "disease": "Deficiency of C1 esterase inhibitor"}, {"label": "B", "disease": "Scombroid poisoning"}, {"label": "C", "disease": "Nonimmunologic release of histamine"}, {"label": "D", "disease": "Impaired breakdown of kinins"}], "answer_idx": "D", "symptoms": ["A 50-year-old woman comes to the emergency department after waking up with facial swelling and with difficulties swallowing.", "She was recently diagnosed with hypertension and started on an antihypertensive drug.", "She follows a strict vegetarian diet.", "Her pulse is 110/min and blood pressure is 135/85 mm Hg.", "Physical examination shows marked edematous swelling of the face, lips, and tongue.", "There is no rash.", "Serum C4 levels are within the reference range."], "s1": [2, 3, 5], "s2": [0, 1, 4, 6]} {"key": 350, "questions": "Which of the following is the most likely etiology of liver damage in this patient?", "options": [{"label": "A", "disease": "α1-antitrypsin (AAT) deficiency"}, {"label": "B", "disease": "Glucose-6-phosphate dehydrogenase deficiency (G6PD deficiency)"}, {"label": "C", "disease": "Dubin–Johnson syndrome"}, {"label": "D", "disease": "Hepatitis C"}], "answer_idx": "A", "symptoms": ["A 42-year-old man presents with acute onset jaundice.", "Past medical history is significant for COPD diagnosed 4 years ago, managed medically, and admission for acute pancreatitis 6 months ago.", "The patient denies any history of smoking, alcohol use or recreational drugs."], "s1": [0], "s2": [1, 2]} {"key": 351, "questions": "Which of the following is the most likely diagnosis?", "options": [{"label": "A", "disease": "Bacterial vaginosis"}, {"label": "B", "disease": "Rectovaginal fistula"}, {"label": "C", "disease": "Vaginal melanoma"}, {"label": "D", "disease": "Vesicovaginal fistula"}], "answer_idx": "B", "symptoms": ["A 31-year-old gravida 2 para 2 woman presents to her primary care physician for follow up.", "Two weeks ago, she gave birth via vaginal delivery to a 9.5 lb (4.3 kg) male infant.", "The delivery was complicated by a vaginal laceration that required extensive suturing once the infant was delivered.", "Immediately after delivery of the placenta she experienced intense shaking and chills that resolved within 1 hour.", "She has felt well since the delivery but admits to 6 days of malodorous smelling vaginal discharge that is tan in color.", "She has a history of vaginal candidiasis and is worried that it may be recurring.", "Her temperature is 98.8°F (37.1°C), blood pressure is 122/73 mmHg, pulse is 88/min, respirations are 16/min, and BMI is 33 kg/m^2.", "Speculum exam reveals a 1.5 cm dark red, velvety lesion on the posterior vaginal wall with a tan discharge.", "The pH of the discharge is 6.4."], "s1": [0, 1, 2, 3, 5, 6], "s2": [4, 7, 8]} {"key": 352, "questions": "Which of the following is the most likely cause of this patient’s respiratory symptoms?", "options": [{"label": "A", "disease": "Lobar lung consolidation"}, {"label": "B", "disease": "Aspiration of stomach contents"}, {"label": "C", "disease": "Occlusion of the pulmonary vessels by fat globules"}, {"label": "D", "disease": "Spontaneous pneumothorax"}], "answer_idx": "C", "symptoms": ["A 50-year-old male presents to the emergency room after getting hit by a car while biking.", "He complains of severe pain in both thighs.", "His thighs appear swollen, bruised, and angulated.", "Radiographic imaging demonstrates bilateral femoral shaft fractures.", "He subsequently undergoes surgical fixation of his fractures.", "Initially he has an uneventful post-operative course; however, on post-op day 4 he becomes confused and develops chest pain, tachypnea, and dyspnea.", "A new petechial rash is seen over his chest and neck.", "Electrocardiogram reveals normal sinus rhythm."], "s1": [0, 1, 2, 3, 4], "s2": [5, 6, 7]} {"key": 353, "questions": "Which of the following is the most likely etiology of this patient’s presentation?", "options": [{"label": "A", "disease": "Coronary atherosclerosis"}, {"label": "B", "disease": "Spontaneous rupture of the aorta"}, {"label": "C", "disease": "Spontaneous rupture of the mitral chordae tendinae"}, {"label": "D", "disease": "Spontaneous separation in the arterial wall"}], "answer_idx": "D", "symptoms": ["A 36-year-old male is brought to the emergency department by fire and rescue after being found down in his apartment by his wife.", "His wife reports that the patient had been complaining of chest pain for the last few hours but refused to go to the hospital.", "She went to the grocery store for about half an hour and found her husband unresponsive on the ground when she returned home.", "The patient’s wife reports that his past medical history is significant for a “heart murmur” and that the patient’s father died at age 32 for unknown reasons.", "In the trauma bay, the patient’s temperature is 98.8°F (37.1°C), blood pressure is 88/41 mmHg, pulse is 116/min, and respirations are 12/min.", "On physical exam, the patient has a Glascow Coma Score (GCS) of 7.", "He has a tall stature with long limbs and long, slender fingers.", "He is also noted to have a pectus deformity.", "On cardiac exam, the patient has a diastolic decrescendo murmur at the left third intercostal space.", "He has weak brachial and femoral pulses. | | ."], "s1": [0, 1, 2, 4, 5, 9], "s2": [3, 6, 7, 8]} {"key": 354, "questions": "Which of the following is the most likely diagnosis?", "options": [{"label": "A", "disease": "Schizoaffective disorder"}, {"label": "B", "disease": "Schizotypal personality disorder"}, {"label": "C", "disease": "Delusional disorder"}, {"label": "D", "disease": "Mood disorder with psychotic features"}], "answer_idx": "A", "symptoms": ["A 28-year-old woman comes to the physician with her mother because of a 1-week history of feeling unusually energetic.", "The mother describes her state as “hyper” and reports that she often paces around her room.", "During this period, the patient has not slept.", "She had similar episodes 2 months and 5 months ago.", "The mother also states that, for the past 8 months, the patient has believed she is a famous singer and is going on a world tour next year.", "The patient does not have a partner and has only a few friends.", "She was working as a dental technician until 6 months ago, when she started to hear voices telling her to quit.", "She has not held a job since then.", "She does not use illicit drugs.", "Vital signs are within normal limits.", "Physical and neurological examinations show no abnormalities.", "On mental status examination, her speech is pressured, but she suddenly stops talking in the middle of sentences and does not finish them.", "She occasionally directs her attention to empty corners in the room, as if she were listening to someone."], "s1": [0, 1, 2, 3, 4, 11], "s2": [5, 6, 7, 8, 9, 10, 12]} {"key": 355, "questions": "What could be causing this patient’s newly-discovered dermatologic change?", "options": [{"label": "A", "disease": "Gastric adenocarcinoma"}, {"label": "B", "disease": "Diabetes insipidus"}, {"label": "C", "disease": "Diabetes mellitus"}, {"label": "D", "disease": "Barrett’s esophagus"}], "answer_idx": "A", "symptoms": ["A 32-year-old obese man visits your dermatology clinic after his barber saw a darkened area on the back of his neck.", "He has not seen his primary care physician in many years and does not know if he has diabetes.", "A rapid blood glucose test showed a normal glucose level.", "Otherwise, he reports recent loss of appetite with slight weight loss."], "s1": [0, 3], "s2": [1, 2]} {"key": 356, "questions": "Which of the following is the most likely cause of this patient's symptoms?", "options": [{"label": "A", "disease": "Idiopathic intracranial hypertension"}, {"label": "B", "disease": "Open-angle glaucoma"}, {"label": "C", "disease": "Central retinal artery occlusion"}, {"label": "D", "disease": "Diabetic retinopathy\n\""}], "answer_idx": "D", "symptoms": ["A 59-year-old man comes to the physician because of a 6-month history of progressive blurry vision in both eyes.", "His vision has not improved with the use of reading glasses.", "His blood pressure is 155/98 mm Hg.", "Physical examination shows his visual acuity to be 20/80 in the right eye and 20/60 in the left eye.", "A photograph of the fundoscopic examination of the right eye is shown."], "s1": [2], "s2": [0, 1, 3, 4]} {"key": 357, "questions": "Which of the following is the most likely diagnosis?", "options": [{"label": "A", "disease": "Giant cell tumor"}, {"label": "B", "disease": "Fibrosarcoma"}, {"label": "C", "disease": "Plasmacytoma"}, {"label": "D", "disease": "Chondrosarcoma"}], "answer_idx": "D", "symptoms": ["A 56-year-old man comes to the physician because of a 2-month history of worsening pain in his left buttock.", "The pain is described as a deep, dull ache that is worse at night.", "He does not report any recent trauma.", "He has hyperlipidemia.", "He has smoked one pack of cigarettes daily for the past 30 years and drinks one beer every night.", "He does not use illicit drugs.", "His only medication is simvastatin.", "Physical examination shows tenderness over the left gluteal region.", "Neurologic examination shows no focal findings.", "An x-ray of the pelvis shows a 4-cm, poorly defined, osteolytic lesion in the left ilium with a moth-eaten pattern of bone destruction, multiple well-defined sclerotic lesions, and an aggressive periosteal reaction.", "The remainder of the physical examination, including rectal examination, shows no abnormalities.", "Chest x-ray and abdominal ultrasonography show no abnormalities."], "s1": [0, 1, 7, 9], "s2": [2, 3, 4, 5, 6, 8, 10, 11]} {"key": 358, "questions": "Which of the following disorders is most consistent with this patient's presentation?", "options": [{"label": "A", "disease": "Conversion disorder"}, {"label": "B", "disease": "Factitious disorder"}, {"label": "C", "disease": "Malingering"}, {"label": "D", "disease": "Somatic symptom disorder"}], "answer_idx": "D", "symptoms": ["A 23-year-old woman presents to the emergency department because of nausea.", "She has been experiencing severe intermittent nausea and diarrhea for the last year and refuses to leave the emergency department until she is told what is causing her symptoms.", "She hates hospitals but has reluctantly undergone numerous workups and imaging studies in order to discern what is wrong.", "All of the studies have been normal.", "She says that these symptoms have been causing her severe distress and is impairing her ability to perform her job.", "She says that she feels as if the nausea is worse when she is trying to work but says that she will continue to work as an insurance agent despite the physical discomfort.", "She has started restricting her diet to only graham crackers because she thinks that helps her symptoms.", "She has also started avoiding eating lunch because of the nausea."], "s1": [0, 1, 3, 6, 7], "s2": [2, 4, 5]} {"key": 359, "questions": "These findings support a diagnosis of which of the following?", "options": [{"label": "A", "disease": "Chediak-Higashi syndrome"}, {"label": "B", "disease": "DiGeorge syndrome"}, {"label": "C", "disease": "Severe combined immunodeficiency (SCID)"}, {"label": "D", "disease": "Wiskott-Aidrich syndrome"}], "answer_idx": "B", "symptoms": ["A 4-month-old adopted infant is brought to the pediatrician because of feeding problems and recurrent fungal and bacterial infections.", "Her family has almost no background information about the infant.", "They report that she is a little fragile and looks different compared to other infants her age.", "Physical evaluation reveals a cleft lip with intact hard and soft palate and mild jaw malformation.", "An echocardiogram reveals an interrupted aortic arch.", "Further investigation reveals leukopenia and mild hypocalcemia."], "s1": [0, 1, 2], "s2": [3, 4, 5]} {"key": 360, "questions": "Which of the following is the most likely diagnosis?", "options": [{"label": "A", "disease": "Nummular dermatitis"}, {"label": "B", "disease": "Psoriasis"}, {"label": "C", "disease": "Tinea capitis"}, {"label": "D", "disease": "Trichotillomania"}], "answer_idx": "C", "symptoms": ["A 7-year-old boy is brought in to your clinic by his mother, who is complaining of her son’s “dry scalp and hair loss.” She reports a scaling spot on his scalp developed around 2 months ago, soon after moving to a new school, which then became a larger bald spot.", "She denies that anyone in the household is exhibiting similar symptoms, but does recall a note that came home from school warning of a lice outbreak.", "Thus far, she has only tried daily baths with little improvement.", "The patient denies pain or itchiness, but the mother reports she does notice him scratching his scalp.", "The mother also reports that since her son has started at the new school, she has noticed an increase in his tics including excessive blinking and clearing of his throat.", "The patient's medical history is significant for asthma, allergic rhinitis, and attention deficient hyperactivity disorder.", "His current medications include fluticasone, salmeterol, loratadine, and methylphenidate.", "Family history includes obsessive compulsive disorder in his older sister.", "The mother also reports her father had a debilitating skin and joint disorder.", "On examination, you note swollen lymph glands at the back of the patient's ears and neck, and there are residual black dots within a patch of alopecia."], "s1": [0, 1, 2, 3, 9], "s2": [4, 5, 6, 7, 8]} {"key": 361, "questions": "Which of the following best describes the symptoms this patient is expressing?", "options": [{"label": "A", "disease": "Brief psychotic disorder"}, {"label": "B", "disease": "Major depressive disorder"}, {"label": "C", "disease": "Normal grief reaction"}, {"label": "D", "disease": "Post-traumatic stress disorder"}], "answer_idx": "C", "symptoms": ["A 52-year-old man visits his psychiatrist saying he feels sad most of the time.", "5 months ago, he lost his daughter in a motor vehicle accident and since then has been unable to cope with this loss.", "He spends most of his day thinking about her and this often distracts him from his daily activities.", "He also admits to hearing his daughter’s voice in his sleep and sometimes has dreams about her.", "During the day, he sometimes gets brief glimpses of her.", "He believes this is because he was not able to do anything to help her during the last moments of her life.", "Other than this, he is able to get along his daily life and has never contemplated suicide.", "But he often wishes he could be close to his daughter again."], "s1": [0, 6], "s2": [1, 2, 3, 4, 5, 7]} {"key": 362, "questions": "Which of the following is the most likely diagnosis?", "options": [{"label": "A", "disease": "Gambling disorder"}, {"label": "B", "disease": "Alcohol withdrawal"}, {"label": "C", "disease": "Bipolar II disorder"}, {"label": "D", "disease": "Adjustment disorder"}], "answer_idx": "A", "symptoms": ["A 48-year-old man is brought to the physician by his wife because he has become increasingly restless and anxious.", "His wife says that he has seemed sad and irritable for the past 4 months.", "He lost his job as a store clerk because he often lied to avoid work, and spent most of his days on the phone placing bets on football games.", "He avoids all family-related events because he has borrowed money from most of his relatives.", "He is hopeful that his symptoms will improve once he is able to win some of his money back.", "He previously smoked 8–10 cigarettes daily but quit 1 month ago.", "He drinks beer on the weekends.", "On examination, he is uncooperative and dismissive.", "He is mildly tremulous.", "Neurologic examination is normal.", "He is oriented to person, place, and time."], "s1": [0, 1, 2, 3, 4], "s2": [5, 6, 7, 8, 9, 10]} {"key": 363, "questions": "Which of the following is the most likely diagnosis?", "options": [{"label": "A", "disease": "Sporotrichosis"}, {"label": "B", "disease": "Basal cell carcinoma"}, {"label": "C", "disease": "Actinic keratosis"}, {"label": "D", "disease": "Seborrheic keratosis\n\""}], "answer_idx": "C", "symptoms": ["A 66-year-old man comes to the physician with a 4-month history of recurrent crusty lesions on the dorsal surface of his hands.", "When he scrapes off the crusts, they reappear after a few days.", "The lesions are mildly pruritic and occasionally burn.", "He works as a landscaper.", "A photograph of his right hand is shown."], "s1": [0, 2], "s2": [1, 3, 4]} {"key": 364, "questions": "Which of the following best describes the most likely diagnosis?", "options": [{"label": "A", "disease": "Dehydration"}, {"label": "B", "disease": "Dopamine blockade"}, {"label": "C", "disease": "Inadequate evaporative cooling"}, {"label": "D", "disease": "Infection"}], "answer_idx": "B", "symptoms": ["A 24-year-old man is brought to the emergency department for bowling at a local bowling alley while inappropriately dressed.", "The patient refused to speak with police and was seen conversing with inanimate objects in the back of the police car.", "The patient states that his clothes were contaminated by assassins who have been tracking him for years.", "The patient refuses to answer any further questions, and continues to whisper toward someone else.", "The patient has a past medical history of homelessness, alcohol abuse, marijuana use, and cocaine abuse.", "Physical exam is notable for a healthy young man who seems mistrustful of all hospital personnel.", "The patient is started on a medication and is transferred to the psychiatric inpatient unit.", "The patient’s condition improves, but he still often speaks of a contamination that is on his clothes and skin that is coming from external parties.", "The patient is started on a new medication.", "He is then sent to group therapy, followed by physical activity in the yard.", "The physician is called over when the patient is found down in the recreation yard while he was exercising.", "The patient is not responsive, and his limbs seem to resist passive force.", "His temperature is 104°F (40°C), pulse is 112/min, blood pressure is 140/90 mmHg, respirations are 18/min, and oxygen saturation is 95% on room air."], "s1": [0, 1, 2, 3, 5, 6, 7, 8, 9], "s2": [4, 10, 11, 12]} {"key": 365, "questions": "Which of the following is the most likely diagnosis?", "options": [{"label": "A", "disease": "Alzheimer dementia"}, {"label": "B", "disease": "Lewy body dementia"}, {"label": "C", "disease": "Normal pressure hydrocephalus"}, {"label": "D", "disease": "Vascular dementia"}], "answer_idx": "D", "symptoms": ["A 77-year-old man presents to the emergency department with a change in his mental status.", "The patient’s daughter states that he can cook and clean up after himself and only needs help paying bills.", "Yesterday the patient seemed fine; however, this morning she noticed that he struggled to perform simple tasks and was forgetful while cooking, leaving the stove on.", "They recently returned from a trip to Mexico and admit to drinking unfiltered water and eating local cuisines while abroad.", "The patient has a past medical history of hypertension and diabetes.", "His temperature is 97.5°F (36.4°C), blood pressure is 187/118 mmHg, pulse is 90/min, respirations are 16/min, and oxygen saturation is 100% on room air."], "s1": [0, 2, 4], "s2": [1, 3, 5]} {"key": 366, "questions": "What is the most likely diagnosis?", "options": [{"label": "A", "disease": "Avoidant personality disorder"}, {"label": "B", "disease": "Major depressive disorder"}, {"label": "C", "disease": "Schizoid personality disorder"}, {"label": "D", "disease": "Schizotypal personality disorder"}], "answer_idx": "C", "symptoms": ["A 37-year-old man presents to his primary care physician with a chief complaint of diarrhea.", "The patient states that his symptoms have worsened recently.", "He works as a farmer and is dressed in tattered and old denim.", "He lives alone, and this is the first time he has come to town in years.", "The patient’s diet is mostly dairy products and some meat products.", "He denies any systemic symptoms and states that diarrhea tends to occur right after a meal.", "He also claims that at times he struggles to fall asleep, and sometimes he feels he lacks energy.", "The patient is instructed to avoid consuming dairy products for a week and to return with a report of his symptoms.", "As you set up the return appointment with this patient you notice that he is rather emotionless.", "He also states that he lacks anybody that he can list as an emergency contact so he leaves that section of his medical forms blank."], "s1": [0, 1, 4, 5, 7], "s2": [2, 3, 6, 8, 9]} {"key": 367, "questions": "Which of the following is the most appropriate next step in diagnosis?\"", "options": [{"label": "A", "disease": "Serum parathyroid-related protein level"}, {"label": "B", "disease": "Urine and serum protein electrophoresis"}, {"label": "C", "disease": "Serum parathyroid hormone level"}, {"label": "D", "disease": "X-ray of the chest"}], "answer_idx": "C", "symptoms": ["A 68-year-old man comes to the physician because of constipation and decreased appetite during the past several months.", "In the past 2 weeks, he has also noticed an increasing frequency of urination.", "He has not had any dysuria or flank pain.", "He has a history of hypertension, calcium pyrophosphate crystal arthritis, and alcoholic cirrhosis.", "Current medications include hydrochlorothiazide, rifaximin, lactulose, and naproxen as needed for the joint pain.", "He quit drinking 5 years ago.", "He does not smoke cigarettes.", "His vital signs are within normal limits.", "Physical examination shows decreased bowel sounds but is otherwise unremarkable.", "Serum studies show: | Creatinine 1.1 mg/dL | Calcium 11.8 mg/dL | Phosphorus 2.1 mg/dL | Magnesium 1.6 mg/dL | Total protein 6 g/dL | Albumin 3.1 g/dL | AST 53 U/L | ALT 43 U/L | ."], "s1": [0, 1, 2, 8, 9], "s2": [3, 4, 5, 6, 7]} {"key": 368, "questions": "Which of the following is the most likely etiology of this patient’s symptoms?", "options": [{"label": "A", "disease": "Decrease in beta-oxidation"}, {"label": "B", "disease": "Increase in gluconeogenesis"}, {"label": "C", "disease": "Increase in beta-oxidation"}, {"label": "D", "disease": "Increase in glycolysis"}], "answer_idx": "A", "symptoms": ["A 2-year-old boy was rushed to the emergency department in the early morning by his mother because he was unresponsive to deep pain stimuli.", "The patient’s mother said that he has been suffering from a fever, cough and runny nose for the last 10 days.", "2 days ago, they visited a free clinic and received medication.", "When she gave him the medication yesterday, the patient had 3 episodes of vomiting and his condition rapidly worsened.", "This morning, the patient did not wake up, even after multiple attempts.", "On physical examination, the patient is febrile, hypotensive, and tachycardic.", "Peripheral pulses are 1+, and his extremities are pale and cool to touch.", "Pupils are equal and slowly reactive to light.", "Significant hepatomegaly is noted."], "s1": [0, 1, 2, 3, 4], "s2": [5, 6, 7, 8]} {"key": 369, "questions": "Which of the following is the most likely cause of these findings?", "options": [{"label": "A", "disease": "Fragile X syndrome"}, {"label": "B", "disease": "Trisomy 18"}, {"label": "C", "disease": "Fetal alcohol syndrome"}, {"label": "D", "disease": "Trisomy 13"}], "answer_idx": "D", "symptoms": ["A 2200-g (4-lb 14-oz) newborn is delivered at term to a 37-year-old primigravid woman.", "Apgar scores are 6 and 8 at 1 and 5 minutes, respectively.", "Examination in the delivery room shows that he is at the 2nd percentile for head circumference and at the 10th percentile for length.", "He has a small jaw, small eyes, and low-set ears.", "There is a keyhole-shaped defect in the iris of the left eye.", "He has seven fingers on each hand.", "There is an opening in the roof of the mouth extending into the soft and hard palate bilaterally.", "Cleft lip and cleft palate are present.", "A 3/6 holosystolic murmur is heard at the left lower sternal border.", "The heels are prominent, with convex-shaped deformity of the plantar surface of the feet."], "s1": [0, 1, 2, 3, 9], "s2": [4, 5, 6, 7, 8]} {"key": 370, "questions": "Of the following, which is the most likely cause of the patient’s symptoms:", "options": [{"label": "A", "disease": "Pleural effusion"}, {"label": "B", "disease": "Superior sulcus tumor"}, {"label": "C", "disease": "Pulmonary embolus"}, {"label": "D", "disease": "Mass in the thoracic region of the spinal cord"}], "answer_idx": "B", "symptoms": ["A 72-year-old presents to the Emergency Room with hemoptysis.", "He has smoked 1 pack of cigarettes a day for the past 30 years.", "Physical examination of the face is shown in Image A and is accompanied by left upper extremity edema."], "s1": [0, 1], "s2": [2]} {"key": 371, "questions": "Which of the following diagnostic tests would be the most appropriate?", "options": [{"label": "A", "disease": "Protein C"}, {"label": "B", "disease": "Factor-V-Leiden"}, {"label": "C", "disease": "Lupus anticoagulant"}, {"label": "D", "disease": "Protein S"}], "answer_idx": "C", "symptoms": ["A 34-year-old woman presents to the office due to a pulmonary embolism confirmed by computed tomography (CT) chest angiogram, which she was recently hospitalized for.", "She does not have any recent surgical trauma or travel history.", "She complains of having episodes where her fingers become white and then blue, and they are usually accompanied by pain in the same region.", "These episodes tend to happen more frequently during winter and have been ongoing for about 2 years now.", "She also had a spontaneous abortion at 12 weeks of gestation last year.", "She does not have a family history of venous thromboembolic disease.", "The vital signs include heart rate 76/min, respiratory rate 18/min, temperature 37.2°C (99.0°F), and blood pressure 120/80 mm Hg.", "Physical examination is non-contributory.", "The complete blood count (CBC) results are as follows: | Hemoglobin 14.2 g/dL | Hematocrit 39% | Leukocyte count 4,950/mm3 | Neutrophils 59% | Bands 3% | Eosinophils 1% | Basophils 0% | Lymphocytes 31% | Monocytes 4% | Platelet count 120,000/mm3 | The coagulation test results are as follows: | Partial thromboplastin time (activated) 56 seconds | Prothrombin time 15 seconds | Reticulocyte count 1.0% | Thrombin time < 2 seconds deviation from control | ."], "s1": [0, 1, 4, 5, 8], "s2": [2, 3, 6, 7]} {"key": 372, "questions": "Which of the following is the most likely underlying diagnosis?", "options": [{"label": "A", "disease": "Ascending cholangitis"}, {"label": "B", "disease": "Cholangiocarcinoma"}, {"label": "C", "disease": "Cirrhosis"}, {"label": "D", "disease": "Colon cancer"}], "answer_idx": "B", "symptoms": ["A 33-year-old man presents to his primary care physician for fatigue, weight loss, and diffuse pruritus.", "The patient has a past medical history of multiple admissions for severe abdominal pain and bloody diarrhea as well as several episodes of ascending cholangitis which were successfully treated.", "He is currently a member of Alcoholics Anonymous.", "His temperature is 98.2°F (36.8°C), blood pressure is 124/75 mmHg, pulse is 100/min, respirations are 13/min, and oxygen saturation is 99% on room air.", "Urinalysis is notable only for a dark urine sample with no red blood cells present.", "A rectal exam reveals pale stool and grossly visible blood.", "Physical exam is otherwise notable for pruritis, jaundice, and scleral icterus."], "s1": [0, 4, 6], "s2": [1, 2, 3, 5]} {"key": 373, "questions": "Which of the following is the most likely diagnosis?", "options": [{"label": "A", "disease": "Placenta percreta"}, {"label": "B", "disease": "Placenta previa"}, {"label": "C", "disease": "Placental abruption"}, {"label": "D", "disease": "Uterine rupture"}], "answer_idx": "B", "symptoms": ["A 28-year-old G2P1 woman at 29 weeks gestation presents to the obstetrician after noticing red spots on her undergarments over the past week.", "Her vaginal bleeding has not been painful; however, she is concerned that it has persisted.", "Her previous child was born by cesarean section and she is currently taking folate and a multivitamin.", "She endorses feeling fetal movements.", "Her temperature is 98.9°F (37.2°C), blood pressure is 120/84 mmHg, pulse is 88/min, respirations are 17/min, and oxygen saturation is 99% on room air.", "Physical exam is notable for a gravid uterus and non-tender abdomen.", "Speculum exam reveals a closed cervical os and a small amount of blood."], "s1": [0, 1, 6], "s2": [2, 3, 4, 5]} {"key": 374, "questions": "Which of the following is the most likely cause of this patient's physical findings?", "options": [{"label": "A", "disease": "Cerebellar lesion"}, {"label": "B", "disease": "Inferior frontal gyrus lesion"}, {"label": "C", "disease": "Upper motor neuron lesion"}, {"label": "D", "disease": "Thalamus lesion"}], "answer_idx": "C", "symptoms": ["A 57-year-old woman comes to the emergency department 1 hour after experiencing a distressing 10-minute episode of stuttering during a business meeting.", "She has hypertension.", "She has smoked 2 packs of cigarettes daily for 30 years.", "Current medications include fosinopril and hydrochlorothiazide.", "Her temperature is 37°C (98.6°F), pulse is 90/min, respirations are 16/min, and blood pressure is 134/88 mm Hg.", "She is oriented to person, place, and time.", "Her speech is clear.", "When she is asked to hold both arms fully extended at shoulder level with palms upwards and eyes closed, her right palm turns inwards and downwards."], "s1": [0, 7], "s2": [1, 2, 3, 4, 5, 6]} {"key": 375, "questions": "Which of the following is the most likely etiology of this patient's condition?", "options": [{"label": "A", "disease": "Vitamin D intoxication"}, {"label": "B", "disease": "Familial hypocalciuric hypercalcaemia"}, {"label": "C", "disease": "Solitary parathyroid adenoma"}, {"label": "D", "disease": "Thyrotoxicosis"}], "answer_idx": "C", "symptoms": ["At a routine exam, a 68-year-old female patient is discovered to have a serum calcium level of 11.5 mg/dL.", "Follow-up laboratory tests show a high parathyroid hormone with low phosphorus and mildly elevated alkaline phosphatase.", "24-hour urine calcium level is elevated.", "Review of symptoms includes complaints of fatigue, constipation, and diffuse bone pain for which she takes vitamin D.", "Past medical history is significant for type 2 diabetes mellitus for 25 years and essential hypertension for 15 years.", "The patient has a history of kidney stones.", "Family history is irrelevant."], "s1": [0, 1, 2, 3, 5], "s2": [4, 6]} {"key": 376, "questions": "Which of the following is the most likely underlying pathology?", "options": [{"label": "A", "disease": "Peripheral arterial disease"}, {"label": "B", "disease": "Vasospasm"}, {"label": "C", "disease": "Coarctation of the aorta"}, {"label": "D", "disease": "Tetralogy of Fallot"}], "answer_idx": "C", "symptoms": ["An 11-year-old girl presents to her pediatrician with complaints of headache and rapid leg fatigue with exercise.", "Her vital signs are: HR 77, BP in left arm 155/100, RR 14, SpO2 100%, T 37.0 C.", "On palpation, her left dorsalis pedis pulse is delayed as compared to her left radial pulse.", "Her lower extremities feel slightly cool.", "Blood pressure obtained on the left lower extremity is 120/80."], "s1": [0, 1], "s2": [2, 3, 4]} {"key": 377, "questions": "Which of the following is the most likely diagnosis?", "options": [{"label": "A", "disease": "Bartholin gland cyst"}, {"label": "B", "disease": "Folliculitis"}, {"label": "C", "disease": "Granuloma inguinale"}, {"label": "D", "disease": "Vulvar cancer"}], "answer_idx": "A", "symptoms": ["A 34-year-old woman comes to the gynecologist complaining of vaginal swelling and discomfort.", "She states that over the past 2 weeks she has noticed swelling of her right labia.", "The swelling continued to grow in size and now causes her discomfort when walking.", "She denies abnormal discharge or bleeding from the swelling or from the vulva.", "She normally shaves the area, but with the recent pain, she has been unable to.", "She denies any history of known trauma.", "The patient is relatively healthy and takes only a multivitamin.", "She never received the vaccine for human papillomavirus (HPV).", "On physical exam, there is a fluctuant, swelling of the right labia, most prominently at the posterior vestibule."], "s1": [0, 1, 2, 3, 8], "s2": [4, 5, 6, 7]} {"key": 378, "questions": "Which of the following was the most likely cause of this patient's symptoms?", "options": [{"label": "A", "disease": "Oligodendroglioma"}, {"label": "B", "disease": "Glioblastoma"}, {"label": "C", "disease": "Schwannoma"}, {"label": "D", "disease": "Meningioma"}], "answer_idx": "B", "symptoms": ["A 56-year-old woman is brought to the emergency department by her husband 30 minutes after a generalized tonic-clonic seizure.", "Over the past 3 months, she has had recurrent headaches and worsening early-morning vomiting.", "She has no personal or family history of a seizure disorder.", "An MRI of the brain shows a central mass; surgical resection is planned.", "One day before the surgery, the patient has another generalized seizure and, despite resuscitative efforts, she dies.", "Gross examination of the brain at autopsy is shown."], "s1": [0, 2, 4], "s2": [1, 3, 5]} {"key": 379, "questions": "This patient’s history is most consistent with which of the following conditions?", "options": [{"label": "A", "disease": "Avoidant personality disorder"}, {"label": "B", "disease": "Dependent personality disorder"}, {"label": "C", "disease": "Histrionic personality disorder"}, {"label": "D", "disease": "Paranoid personality disorder"}], "answer_idx": "B", "symptoms": ["A 29-year-old woman presents to her primary care provider for a normal check-up at the prompting of her mother.", "When asked how she is doing, she replies that she feels depressed.", "Upon further questioning, she states that she feels “worthless” now that her ex-boyfriend left her 2 days ago.", "She has been sitting in his bed all day trying to call him but he is not answering her calls.", "She says that she will never find another man like him and feels like she should just “disappear” rather than continue living without him.", "She also reports that she can hear his voice when he’s not around, especially at night before she goes to bed.", "She denies having any other friends and has some social anxiety, particularly in large groups of people.", "A review of her medical record indicates that she has endured several episodes of verbal and physical abuse from her prior relationships which she suggested “were all her fault.” She has had 4 prior serious relationships and reports that she has not been single for more than a few days in the last 10 years.", "On exam, she is unkempt with poor self-care."], "s1": [0, 3, 8], "s2": [1, 2, 4, 5, 6, 7]} {"key": 380, "questions": "Which of the following is the most likely underlying etiology of this patient's condition?", "options": [{"label": "A", "disease": "Acute pyelonephritis"}, {"label": "B", "disease": "Hypertensive kidney disease"}, {"label": "C", "disease": "Membranous nephropathy"}, {"label": "D", "disease": "Penile cancer"}], "answer_idx": "C", "symptoms": ["A 52-year-old homosexual man presents to the emergency department because of sudden right loin pain accompanied by nausea and vomiting.", "He has essential hypertension for 3 years for which he takes bisoprolol.", "He has a remote history of intravenous drug use.", "His temperature is 36.9°C (98.4°F), the blood pressure is 137/92 mm Hg, and the pulse is 95/min.", "Physical examination reveals right flank tenderness and 2+ edema of both legs up to the knees.", "Doppler ultrasonography shows an enlarged, echogenic right kidney with the absent venous signal.", "Laboratory results are shown: | Serum creatinine 2.2 mg/dL | Urine dipstick 4+ protein | Urine analysis 20–25 red cells/high power field | ."], "s1": [0, 4, 5, 6], "s2": [1, 2, 3]} {"key": 381, "questions": "Which of the following best describes this cause of the patients symptoms after treatment?", "options": [{"label": "A", "disease": "Lateral medullary syndrome"}, {"label": "B", "disease": "Medial medullary syndrome"}, {"label": "C", "disease": "Osmotic demyelination syndrome"}, {"label": "D", "disease": "Hypercalcemia"}], "answer_idx": "C", "symptoms": ["A 65-year-old man with squamous cell carcinoma of the lung presents with a sodium of 118 and an altered mental status.", "He is started on 3% normal saline over the next several hours.", "The patient’s mental status improves and and a repeat sodium six hours later is 129.", "Three days after treatment he demonstrates quadriparesis and dysarthria."], "s1": [0, 3], "s2": [1, 2]} {"key": 382, "questions": "Which of the following is the most likely cause of this patient's anemia?\"", "options": [{"label": "A", "disease": "Extravascular hemolysis"}, {"label": "B", "disease": "Vitamin B12 deficiency"}, {"label": "C", "disease": "Intravascular hemolysis"}, {"label": "D", "disease": "Impaired heme synthesis"}], "answer_idx": "C", "symptoms": ["A 25-year-old woman comes to the physician for a routine examination.", "She is a recreational runner and for the past 3 months she has been training for a marathon.", "Yesterday she ran 17 km (10.5 mile).", "Menses occur at regular 28-day intervals and last 6 days.", "She drinks three to five beers when going out with friends.", "She follows a vegan diet.", "She appears well.", "Her temperature is 36.6°C (98.0°F), blood pressure is 110/72 mm Hg, pulse is 70/min, and respirations are 14/min.", "Physical examination shows no abnormalities.", "Laboratory studies show: | Hemoglobin 9.8 g/dL | Mean corpuscular volume 91 fL | Lactate dehydrogenase 400 U/L | Platelet count 250,000/mm3 | Reticulocyte count 3% | Haptoglobin 23 mg/dL (N=41–165 mg/dL) | Serum | Iron 90 μg/dL | Ferritin 170 ng/mL | Total iron binding capacity (TIBC) 220 μg/dL (N=251–406 μg/dL) | Urinalysis shows trace blood but no RBCs."], "s1": [0, 1, 2, 3, 4, 5, 6, 7, 8], "s2": [9]} {"key": 383, "questions": "Which personality disorder is most likely?", "options": [{"label": "A", "disease": "Narcissistic personality disorder"}, {"label": "B", "disease": "Avoidant personality disorder"}, {"label": "C", "disease": "Histrionic personality disorder"}, {"label": "D", "disease": "Obsessive-compulsive personality disorder"}], "answer_idx": "A", "symptoms": ["A 45-year-old man comes into the ED after a car accident.", "After waiting an hour in the waiting room, he begins to berate the receptionist and demands to be seen by a doctor.", "During the exam, the patient brags that he recently had to quit his job because they did not appreciate his talent and drive.", "After a medical student declines his invitation for a date, he becomes very angry and throws supplies across the room.", "The physician manages the patient's physical injuries and begins to suspect the patient has a personality disorder."], "s1": [0, 4], "s2": [1, 2, 3]} {"key": 384, "questions": "Which of the following is the most likely underlying cause of this patient's symptoms?", "options": [{"label": "A", "disease": "Adverse effect of medication"}, {"label": "B", "disease": "Degeneration of ganglion cells within the myenteric plexuses"}, {"label": "C", "disease": "Infection with herpes simplex virus"}, {"label": "D", "disease": "Infection with Candida species"}], "answer_idx": "C", "symptoms": ["A 52-year-old woman comes to the physician because of a 4-day history of painful swallowing and retrosternal pain.", "She was diagnosed with HIV infection 2 months ago; her medications include tenofovir, emtricitabine, and raltegravir.", "Vital signs are within normal limits.", "Physical examination of the oral cavity shows no abnormalities.", "The patient's CD4+ T-lymphocyte count is 70/mm3 (N ≥ 500).", "Empiric treatment is started.", "Two weeks later, she reports no improvement in symptoms.", "Esophagogastroduodenoscopy is performed and shows multiple round superficial ulcers in the distal esophagus."], "s1": [0, 6, 7], "s2": [1, 2, 3, 4, 5]} {"key": 385, "questions": "Which of the following is the most likely diagnosis?", "options": [{"label": "A", "disease": "Appendicitis"}, {"label": "B", "disease": "Endometriosis"}, {"label": "C", "disease": "Midcycle pain"}, {"label": "D", "disease": "Ruptured ectopic pregnancy"}], "answer_idx": "C", "symptoms": ["An 18-year-old girl comes to the emergency room with abdominal pain.", "She states that the pain started 6 hours ago, is 8/10, and localizes to the right lower abdomen.", "She recalls a similar episode last month.", "Her older brother was hospitalized 2 years ago for perforated appendicitis.", "Her mother has diverticulitis.", "The patient got her first menstrual period at age 14.", "Her periods occur regularly every 28 days.", "Her last menstrual period was 2 weeks ago.", "She is sexually active with multiple partners and uses condoms inconsistently.", "The patient’s temperature is 99°F (37.2°C), blood pressure is 110/74 mmHg, pulse is 89/min, and respirations are 14/min with an oxygen saturation of 98% on room air.", "On physical exam, the patient is in moderate distress.", "There is lower abdominal tenderness with guarding.", "A pelvic examination reveals clear mucous in the vaginal introitus, a mobile uterus, and tenderness of the right adnexa.", "Labs are obtained, as shown below: | | Leukocyte count: 10,000/mm^3 with normal differential | Hemoglobin: 13.5 g/dL | Platelet count: 200,000/mm^3 | Beta-human chorionic gonadotropin: Pending | | A pelvic ultrasound demonstrates a small fluid collection in the cul-de-sac posterior to the uterus."], "s1": [0, 1, 2, 3, 4, 10, 11, 13], "s2": [5, 6, 7, 8, 9, 12]} {"key": 386, "questions": "What is the most likely diagnosis?", "options": [{"label": "A", "disease": "Ehlers-Danlos syndrome"}, {"label": "B", "disease": "Osteosarcoma"}, {"label": "C", "disease": "Ankylosing spondylitis"}, {"label": "D", "disease": "Osteopetrosis"}], "answer_idx": "C", "symptoms": ["A 26-year-old man presents to the office complaining of persistent back pain for the past 3 months.", "You saw the patient previously and prescribed NSAIDs and rest, but he has not improved.", "The patient states that his maternal grandfather had ‘back problems his whole life’, and he worries that he might have the same issues.", "An X-ray is performed and shows squaring of the vertebrae with longitudinal fibrous bands."], "s1": [0, 1, 2], "s2": [3]} {"key": 387, "questions": "What is the most likely explanation for this patient’s symptoms?", "options": [{"label": "A", "disease": "Accumulation of N-acetyl-p-benzoquinone imine in the liver"}, {"label": "B", "disease": "Impaction of a gallstone in the ileus"}, {"label": "C", "disease": "Malignant overgrowth of glial cells in the brain"}, {"label": "D", "disease": "Obstruction of the cystic duct by gallstone"}], "answer_idx": "A", "symptoms": ["A 57-year-old man comes to the clinic complaining of nausea and 1 episode of vomiting during the past day.", "He denies any precipitating event except for a headache for which he took some acetaminophen yesterday.", "The headache is described as bilateral, dull, and 8/10 in severity.", "He has never had such symptoms before.", "His past medical history is significant for hypertension, biliary colic, and a past hospitalization for delirium tremens when he “tried to quit alcohol cold turkey.” When asked about his current alcohol intake, the patient looks away and mutters “just a couple of drinks here and there.” The patient reports mild abdominal pain that began this morning but denies vision changes, bowel changes, fever, or weight loss.", "Physical examination demonstrates tenderness at the right upper quadrant (RUQ) that does not worsens with inhalation."], "s1": [2, 3, 0, 1], "s2": [4, 5]} {"key": 388, "questions": "Which of the following pathologic findings is she most likely to have?", "options": [{"label": "A", "disease": "Follicular thyroid neoplasia "}, {"label": "B", "disease": "Silent thyroiditis"}, {"label": "C", "disease": "Chronic lymphocytic thyroiditis"}, {"label": "D", "disease": "Fibrous thyroiditis"}], "answer_idx": "B", "symptoms": ["A 47-year-old G3P2 woman presents to her endocrinologist for a follow-up appointment.", "Her last menstrual period was at the age of 35 years.", "She now feels fatigued and cannot concentrate on her work or type properly on the keyboard because of an onset of tremor in her hands.", "Her symptoms are taking a toll on her quality of life.", "The patient’s blood pressure is 124/83 mm Hg, pulse is 91/min, respirations are 17/min, and temperature is 36.7°C (98.1°F).", "On physical examination, there is a mild enlargement of the thyroid; the gland is minimally firm in texture and is nontender."], "s1": [0, 1, 3, 5], "s2": [2, 4]} {"key": 389, "questions": "What is the most likely cause of her presentation?", "options": [{"label": "A", "disease": "Echinococcosis"}, {"label": "B", "disease": "Tuberculosis"}, {"label": "C", "disease": "Pulmonary thromboembolism"}, {"label": "D", "disease": "Heart failure"}], "answer_idx": "B", "symptoms": ["A 45-year-old woman presents to the office with a chief complaint of a cough that has persisted for a month and a half.", "She also has night fevers and has lost 5 kg (11 lb) in the last 2 months.", "She recently returned from a 6-month trip to Peru.", "Her vitals include: heart rate 82/min, respiratory rate 17/min, temperature 38.0°C (100.4°F), and blood pressure 107/80 mm Hg.", "On auscultation, she has diminished respiratory sounds at the base of the right lung.", "Chest radiography shows an opacity in the right apex."], "s1": [2, 3], "s2": [0, 1, 4, 5]} {"key": 390, "questions": "What is the most likely diagnosis?", "options": [{"label": "A", "disease": "Alcohol-related dementia"}, {"label": "B", "disease": "Hepatic encephalopathy"}, {"label": "C", "disease": "Delirium"}, {"label": "D", "disease": "Wernicke's encephalopathy"}], "answer_idx": "D", "symptoms": ["A 52-year-old homeless man is brought to the urgent care clinic with altered mental status and disorientation for the last day.", "Past medical records are unavailable.", "His temperature is 37°C (98.6°F), the respiratory rate is 15/min, the pulse is 107/min, and the blood pressure is 92/67 mm Hg.", "He has a flapping tremor of the wrists during attempted extension.", "He is started on intravenous D5 1/2NS and blood is drawn for further testing.", "His condition deteriorates overnight.", "On re-examination the next morning, he has developed visual disturbances with a new horizontal nystagmus and a staggering gait.", "His blood tests show: | Serum | Albumin 3.2 gm/dL | Alkaline phosphatase 150 U/L | Alanine aminotransferase 76 U/L | Aspartate aminotransferase 155 U/L | Gamma-glutamyl transpeptidase 202 U/L | ."], "s1": [0, 1, 4, 5], "s2": [2, 3, 6, 7]} {"key": 391, "questions": "Which of the following is the most likely diagnosis?", "options": [{"label": "A", "disease": "Patent ductus arteriosus"}, {"label": "B", "disease": "Venous hum"}, {"label": "C", "disease": "Coronary artery fistula"}, {"label": "D", "disease": "Aortic stenosis"}], "answer_idx": "B", "symptoms": ["A 6-year-old girl is brought to the physician for a well-child examination.", "She has no history of serious illness.", "Her temperature is 37°C (98.6°F), pulse is 95/min, and blood pressure is 96/52 mm Hg.", "Examination shows a 2/6 continuous, low-pitched murmur over the left upper sternal border that radiates towards the neck.", "The murmur disappears when she flexes her neck.", "The remainder of the examination shows no abnormalities."], "s1": [0, 1, 2, 5], "s2": [3, 4]} {"key": 392, "questions": "Which of the following is the most likely cause of the patient's symptoms?", "options": [{"label": "A", "disease": "Cavernous sinus thrombosis"}, {"label": "B", "disease": "Suppurative spread of dacrocystitis"}, {"label": "C", "disease": "Bacterial infection from sphenoid sinus"}, {"label": "D", "disease": "Bacterial infection from ethmoidal sinus"}], "answer_idx": "D", "symptoms": ["A 12-year-old girl is brought to the emergency department because of severe pain and swelling of her right eye for 3 days.", "Over the past two weeks, she had nasal congestion and a constant, mild headache.", "She did not visit a doctor and instead took over-the-counter naproxen and phenylephrine.", "Her nasal congestion has since improved, but the headache persists.", "She underwent a dental procedure for caries about ten days ago.", "She does not recollect any recent insect bites or trauma.", "Her immunizations are up-to-date.", "She appears ill and reports occasional chills.", "Her temperature is 38.3°C (101°F), pulse is 89/min, and blood pressure is 110/70 mm Hg.", "Examination shows proptosis of the right eye.", "Vision is impaired in the affected eye.", "Her right upper and lower eyelid are erythematous, swollen, and tender to touch.", "Ocular movements cause severe pain.", "Her headache is worsened by leaning forward and improves when the patient is lying on her back."], "s1": [0, 9, 10, 11, 12], "s2": [1, 2, 3, 4, 5, 6, 7, 8, 13]} {"key": 393, "questions": "Which of the following is the most likely diagnosis?", "options": [{"label": "A", "disease": "Major depressive disorder"}, {"label": "B", "disease": "Bulimia nervosa"}, {"label": "C", "disease": "Body dysmorphic disorder"}, {"label": "D", "disease": "Anorexia nervosa\n\""}], "answer_idx": "B", "symptoms": ["A 17-year-old girl is brought to the physician for a well-child examination.", "She is worried about gaining weight.", "She frequents buffet restaurants but feels guilty soon after.", "She has a history of burning her extremities with cigarettes.", "Her last menstrual period was 3 weeks ago.", "She attends high school and plays field hockey on the school team.", "She is at the 25th percentile for height, 12th percentile for weight, and 17th percentile for BMI.", "Examination shows bilateral parotid gland enlargement.", "Oropharyngeal examination shows perimolysis.", "The remainder of the examination shows no abnormalities."], "s1": [0, 5, 4, 9], "s2": [1, 2, 3, 6, 7, 8]} {"key": 394, "questions": "What is the most likely diagnosis?", "options": [{"label": "A", "disease": "Dubin-Johnson syndrome"}, {"label": "B", "disease": "Gilbert syndrome"}, {"label": "C", "disease": "Breast feeding jaundice"}, {"label": "D", "disease": "Erythroblastosis fetalis"}], "answer_idx": "D", "symptoms": ["A 1-day-old male is seen in the neonatal intensive care unit for respiratory distress.", "He was born at 37 weeks to a 24-year-old G3P11011 Rh- mother who had no prenatal care.", "On physical examination, temperature is 99°F (37.2°C), blood pressure is 120/85 mmHg, pulse is 120/min, respirations are 26/min, and pulse oximetry is 92% on room air.", "There is abdominal distention with a positive fluid wave.", "Laboratory results are as follows: | | Serum: | Alkaline phosphatase: 80 U/L | ALT: 33 U/L | AST: 32 U/L | Bilirubin (total): 10 mg/dL | Bilirubin (conjugated): 0.2 mg/dL | Amylase: 76 U/L | | Leukocyte count: 5,000/mm^3 with normal differential | Hemoglobin: 8 g/dL | Platelet count: 200,000/mm^3 | Mean corpuscular volume: 80 µm^3 | Reticulocyte count: 3% | | ."], "s1": [0, 1, 2], "s2": [3, 4]} {"key": 395, "questions": "Which of the following is the most likely diagnosis?", "options": [{"label": "A", "disease": "Endemic typhus"}, {"label": "B", "disease": "Lyme disease"}, {"label": "C", "disease": "Rocky Mountain spotted fever"}, {"label": "D", "disease": "Q fever"}], "answer_idx": "C", "symptoms": ["A 29-year-old woman comes to the physician with complaints of fever, headache, and rash, which have gradually worsened over the past 6 days.", "The patient informs the physician that the rash appeared after 2 days of fever.", "The rash started at the wrists and ankles and then gradually spread to the trunk, palms, and soles.", "On examination, the physician notes a maculopapular rash as shown in the image below.", "Vital signs show a temperature of 39.4°C (103.0°F), a blood pressure of 110/70 mm Hg, and a pulse rate of 86/min."], "s1": [0, 1, 2, 3], "s2": [4]} {"key": 396, "questions": "Which of the following is the most likely cause of these findings?", "options": [{"label": "A", "disease": "Obstructed orifice of the Bartholin duct"}, {"label": "B", "disease": "Reactivation of the genital herpes"}, {"label": "C", "disease": "Squamous cell carcinoma of the vulva"}, {"label": "D", "disease": "Allergic reaction to the intrauterine device"}], "answer_idx": "A", "symptoms": ["An otherwise healthy 23-year-old newly-married woman comes to the physician because of a 6-day history of discomfort in her vaginal area during and after sexual intercourse.", "Her last menstrual period was 3 weeks ago.", "Two years ago, she was diagnosed with genital herpes which was treated with acyclovir.", "She has been in a monogamous relationship for the past year and has been using an intrauterine device for contraception for the past month.", "Examination shows a 4-cm, mildly tender mass in the inferior aspect of the left labium minus with no signs of inflammation.", "Speculum examination causes her discomfort but shows no abnormalities."], "s1": [0, 1, 2, 3], "s2": [4, 5]} {"key": 397, "questions": "Which of the following is the most likely cause of this infant’s presentation?", "options": [{"label": "A", "disease": "ß-cell hyperplasia"}, {"label": "B", "disease": "Neonatal sepsis"}, {"label": "C", "disease": "Inborn error of metabolism"}, {"label": "D", "disease": "Neonatal encephalopathy"}], "answer_idx": "A", "symptoms": ["A 37-year-old G1P1001 delivers a male infant at 9 pounds 6 ounces after a C-section for preeclampsia with severe features.", "The mother has a history of type II diabetes with a hemoglobin A1c of 12.8% at her first obstetric visit.", "Before this pregnancy, she was taking metformin, and during this pregnancy, she was started on insulin.", "At her routine visits, her glucose logs frequently showed fasting fingerstick glucoses above 120 mg/dL and postprandial values above 180 mg/dL.", "In addition, her routine third trimester culture for group B Streptococcus was positive.", "At 38 weeks and 4 days gestation, she was found to have a blood pressure of 176/103 mmHg and reported a severe headache during a routine obstetric visit.", "She denied rupture of membranes or vaginal bleeding.", "Her physician sent her to the obstetric triage unit, and after failure of several intravenous doses of labetalol to lower her blood pressure and relieve her headache, a C-section was performed without complication.", "Fetal heart rate tracing had been reassuring throughout her admission.", "Apgar scores at 1 and 5 minutes were 7 and 10.", "After one hour, the infant is found to be jittery; the infant's temperature is 96.1°F (35.6°C), blood pressure is 80/50 mmHg, pulse is 110/min, and respirations are 60/min.", "When the first feeding is attempted, he does not latch and begins to shake his arms and legs.", "After 20 seconds, the episode ends and the infant becomes lethargic."], "s1": [0, 1, 2, 3, 4, 5, 6, 7, 8, 9], "s2": [10, 11, 12]} {"key": 398, "questions": "What is the most likely diagnosis?", "options": [{"label": "A", "disease": "Mitral valve insufficiency"}, {"label": "B", "disease": "Aortic regurgitation"}, {"label": "C", "disease": "Mitral stenosis"}, {"label": "D", "disease": "Aortic stenosis"}], "answer_idx": "B", "symptoms": ["A 65-year-old male presents to his cardiologist to discuss increasing episodes of dyspnea after climbing stairs.", "He also now needs three pillows at night to sleep.", "Physical examination reveals an early diastolic murmur best appreciated at the left sternal border with bounding peripheral pulses.", "The cardiologist is very concerned and immediately refers the patient for a surgical workup."], "s1": [0, 1], "s2": [2, 3]} {"key": 399, "questions": "Which of the following is the most likely cause of this patient's symptoms?", "options": [{"label": "A", "disease": "Deficiency of β-glucocerebrosidase"}, {"label": "B", "disease": "Dysfunction of ATP-binding cassette transporter"}, {"label": "C", "disease": "Deficiency of arylsulfatase A"}, {"label": "D", "disease": "Deficiency of lysosomal galactocerebrosidase"}], "answer_idx": "B", "symptoms": ["A 7-year-old boy is brought to the physician because of decreased vision, hearing, and speaking over the past 3 months.", "During this time, he has also had difficulty walking, concentrating, drawing, and feeding himself.", "His maternal male cousin had similar complaints and died at the age of 5 years.", "Examination shows hyperpigmented skin and nails.", "His speech is dysarthric.", "Neurologic examination shows an ataxic gait, spasticity, and decreased muscle strength in all extremities.", "Fundoscopy shows optic atrophy."], "s1": [3], "s2": [0, 1, 2, 4, 5, 6]} {"key": 400, "questions": "Which of the following is the most likely pathology for this presentation?", "options": [{"label": "A", "disease": "Prion infection"}, {"label": "B", "disease": "α-synuclein defect"}, {"label": "C", "disease": "Depigmented substantia nigra and locus cerulus"}, {"label": "D", "disease": "Abnormal cleavage of amyloid precursor protein"}], "answer_idx": "D", "symptoms": ["A 75-year-old man was brought in by his daughter since he was having increased incidences of forgetting things.", "His daughter said that he becomes increasingly frustrated searching for his glasses and keys most of the time.", "He was helped out a couple of times in the supermarket for forgetting the way out.", "He recently lost his driving license when he was spotted by the cops driving in the wrong direction on the interstate."], "s1": [0, 1, 2], "s2": [3]} {"key": 401, "questions": "What is the most likely primary metabolic disturbance?", "options": [{"label": "A", "disease": "Anion gap metabolic acidosis"}, {"label": "B", "disease": "Non-anion gap metabolic acidosis"}, {"label": "C", "disease": "Metabolic alkalosis"}, {"label": "D", "disease": "Respiratory alkalosis"}], "answer_idx": "A", "symptoms": ["A 12-year-old female presents to the emergency room difficult to arouse and occasionally vomiting.", "On physical exam, her oral mucosa looks dry, her breath has a fruity odor, and her breathing is slow, deep and labored."], "s1": [0], "s2": [1]} {"key": 402, "questions": "Which of the following is the most likely diagnosis?", "options": [{"label": "A", "disease": "Bacterial endocarditis"}, {"label": "B", "disease": "Bicuspid aortic valve"}, {"label": "C", "disease": "Cardiac myxoma"}, {"label": "D", "disease": "Senile calcific changes"}], "answer_idx": "B", "symptoms": ["A 36-year-old man presents to the emergency room with subacute worsening of chronic chest pain and shortness of breath with exertion.", "The patient is generally healthy, lifts weights regularly, and does not smoke.", "His temperature is 97.8°F (36.6°C), blood pressure is 122/83 mm Hg, pulse is 80/min, respirations are 13/min, and oxygen saturation is 98% on room air.", "Cardiac auscultation reveals a crescendo-decrescendo murmur heard right of the upper sternal border with radiation into the carotids.", "An ECG shows left axis deviation and meets criteria for left ventricular hypertrophy.", "An initial troponin is < 0.01 ng/mL."], "s1": [0, 3, 4], "s2": [1, 2, 5]} {"key": 403, "questions": "Which of the following is the most likely diagnosis?", "options": [{"label": "A", "disease": "Thrombotic thrombocytopenic purpura"}, {"label": "B", "disease": "Genetic form of hemolytic uremic syndrome"}, {"label": "C", "disease": "Hemolytic uremic syndrome associated with systemic disease"}, {"label": "D", "disease": "Escherichia-induced hemolytic uremic syndrome"}], "answer_idx": "D", "symptoms": ["A 10-year-old boy is brought to the physician because of fever and bloody diarrhea for the past few days.", "His parents report that he has become increasingly lethargic and irritable.", "His temperature is 38.6°C (101.4°F), pulse is 102/min, and respirations are 22/min.", "He has no significant past medical history.", "His parents say that he mostly only eats a diet of chicken, hamburgers, fries, cheese, and milk.", "On physical examination, pallor and edema in both legs are present.", "His laboratory studies show: | Hemoglobin 8.9 gm/dL | Leukocyte count 9,300/mm3 | Platelet count 67,000/mm3 | Blood urea nitrogen 43 mg/dL | Serum creatinine 2.46 mg/dL | Coombs test Negative | ."], "s1": [0, 4], "s2": [1, 2, 3, 5, 6]} {"key": 404, "questions": "Which of the following is the most likely cause of these findings?\"", "options": [{"label": "A", "disease": "Hemochromatosis"}, {"label": "B", "disease": "Pituitary adenoma"}, {"label": "C", "disease": "Graves disease"}, {"label": "D", "disease": "Hashimoto thyroiditis"}], "answer_idx": "B", "symptoms": ["A 40-year-old man comes to the physician because of decreased sexual arousal and erectile dysfunction that has put strain on his marriage for the last year.", "He has also had fatigue and generalized weakness for the past 9 months.", "He has noticed his bowel movements have not been as frequent as usual.", "He has occasional dry coughing and back pain.", "He has not had fever, headache, or changes in vision.", "One year ago, he traveled to South Africa with his wife.", "He looks fatigued.", "He is 168 cm (5 ft 6 in) tall and weighs 89 kg (196 lb); BMI is 31.6 kg/m2.", "His temperature is 36.5°C (97.7°F), pulse is 50/min, and blood pressure is 125/90 mm Hg.", "Physical examination shows dry skin and a distended abdomen.", "Neurological examination reveals delayed deep tendon reflexes.", "Laboratory studies show: | Hemoglobin 11.0 g/dL | Platelet count 380,000/mm3 | Serum | Na+ 130 mEq/L | Cl- 97 mEq/L | K+ 4.5 mEq/L | HCO3- 25 mEq/L | Glucose 95 mg/dL | TSH 0.2 μU/mL | ."], "s1": [0, 1, 5, 6, 7, 8], "s2": [2, 3, 4, 9, 10, 11]} {"key": 405, "questions": "Which of the following is most likely to confirm the most likely diagnosis in this patient?", "options": [{"label": "A", "disease": "Radiograph of the spine"}, {"label": "B", "disease": "The straight leg test and the clinical presentation"}, {"label": "C", "disease": "MRI of the sacroiliac joint"}, {"label": "D", "disease": "HLA typing"}], "answer_idx": "C", "symptoms": ["A 25-year-old man presents to his primary care physician with lower back pain.", "He states that he has had the pain for the past two years.", "The patient works as a butcher, and recently was moving heavy meat carcasses.", "The patient states that his pain is worse in the morning and that nothing improves it aside from swimming.", "The patient has a past medical history of anabolic steroid abuse, acne, hypertension and obesity.", "His current medications are hydrochlorothiazide, ibuprofen, topical benzoyl peroxide, and acetaminophen.", "On physical exam there is no tenderness upon palpation of the spine.", "There is limited range of motion of the spine in all 4 directions."], "s1": [0, 2, 4, 5], "s2": [1, 3, 6, 7]} {"key": 406, "questions": "Which of the following is the most likely diagnosis?", "options": [{"label": "A", "disease": "HIV infection"}, {"label": "B", "disease": "Type 1 diabetes mellitus"}, {"label": "C", "disease": "Hyperthyroidism"}, {"label": "D", "disease": "Anorexia nervosa"}], "answer_idx": "D", "symptoms": ["A 16-year-old girl is brought to the physician because her mother is concerned about her lack of appetite and poor weight gain.", "She has had a 7-kg (15-lb) weight loss over the past 3 months.", "The patient states that she should try to lose more weight because she does not want to be overweight anymore.", "She maintains a diary of her daily calorie intake.", "Menarche was at the age of 13 years, and her last menstrual period was 3 months ago.", "She is on the high school track team.", "She is sexually active with 2 male partners and uses condoms inconsistently.", "She is at 50th percentile for height and below the 5th percentile for weight and BMI.", "Her temperature is 37°C (98.6°F), pulse is 58/min and blood pressure is 96/60 mm Hg.", "Examination shows fine hair over the trunk and extremities."], "s1": [0, 1, 2, 3, 4, 7, 8, 9], "s2": [5, 6]} {"key": 407, "questions": "What is the most likely diagnosis in this patient?", "options": [{"label": "A", "disease": "Multiple sclerosis"}, {"label": "B", "disease": "Pituitary mass"}, {"label": "C", "disease": "Pineal tumor"}, {"label": "D", "disease": "Craniopharyngioma"}], "answer_idx": "C", "symptoms": ["A 25-year-old male presents with progressively worsening headaches over the past two months.", "He also feels that he has been losing his balance more often over the past week, but he denies any motor weakness or sensory impairment.", "His neurological exam reveals impaired upward gaze, pupils that constrict poorly to light but react to accommodation, and bilateral upper eyelid retraction.", "On tandem walking, he tends to fall on both sides.", "The remainder of the physical examination is unremarkable."], "s1": [0, 1, 3], "s2": [2, 4]} {"key": 408, "questions": "Which of the following is the most likely cause of the patient's symptoms?", "options": [{"label": "A", "disease": "Infection with Aspergillus species"}, {"label": "B", "disease": "Abnormal squamous epithelial growth"}, {"label": "C", "disease": "Infection with Pseudomonas aeruginosa"}, {"label": "D", "disease": "Pleomorphic replacement of normal bone\n\""}], "answer_idx": "C", "symptoms": ["A 12-year-old girl is brought to the primary physician because of severe ear pain and yellow discharge from her left ear for the past 2 days.", "It is also mildly pruritic.", "The pain started during her last day of summer camp, where she spent a lot of time outdoors hiking, horseback riding, and swimming.", "Her temperature is 37°C (98.6°F), pulse is 76/min, and blood pressure is 110/75 mm Hg.", "Examination shows tragal tenderness and a red and edematous external auditory canal.", "A diagnostic tuning fork is placed in the middle of the patient's forehead.", "The patient reports hearing the sound more loudly in the left ear.", "To complete the workup, the tuning fork is placed on the mastoid process of the left ear.", "Once she can no longer hear the sound, the tuning fork is placed in front of the auricle, and she reports no longer hearing the sound.", "On the right side, once the sound from the mastoid process subsides and the tuning fork is placed in front of the right ear, she reports hearing the sound again."], "s1": [0, 1, 2, 3, 4], "s2": [5, 6, 7, 8, 9]} {"key": 409, "questions": "Which of the following is the most likely diagnosis?\"", "options": [{"label": "A", "disease": "Acute mesenteric ischemia"}, {"label": "B", "disease": "Peptic ulcer disease"}, {"label": "C", "disease": "Myocardial infarction"}, {"label": "D", "disease": "Acute pancreatitis"}], "answer_idx": "D", "symptoms": ["A 50-year-old man comes to the emergency department because of severe lower chest pain for the past hour.", "The pain radiates to the back and is associated with nausea.", "He has had two episodes of non-bloody vomiting since the pain started.", "He has a history of hypertension and type 2 diabetes mellitus.", "He has smoked one pack of cigarettes daily for 30 years.", "He drinks five to six beers per day.", "His medications include enalapril and metformin.", "His temperature is 38.5°C (101.3°F), pulse is 110/min, and blood pressure is 90/60 mm Hg.", "The lungs are clear to auscultation.", "Examination shows a distended abdomen with epigastric tenderness and guarding but no rebound; bowel sounds are decreased.", "Laboratory studies show: | Hemoglobin 14.5 g/dL | Leukocyte count 5,100/mm3 | Platelet count 280,000/mm3 | Serum | Na+ 133 mEq/L | K+ 3.5 mEq/L | Cl- 98 mEq/L | Total bilirubin 1.0 mg/dL | Amylase 160 U/L | Lipase 880 U/L (N = 14–280) | An ECG shows sinus tachycardia."], "s1": [0, 1, 2, 7, 8, 10], "s2": [3, 4, 5, 6, 9]} {"key": 410, "questions": "Based on these findings, which of the following is the most likely diagnosis?", "options": [{"label": "A", "disease": "Uterine leiomyoma"}, {"label": "B", "disease": "Endometrial hyperplasia"}, {"label": "C", "disease": "Endometrial carcinoma"}, {"label": "D", "disease": "Endometrial polyp"}], "answer_idx": "A", "symptoms": ["A 30-year-old woman seeks evaluation at a local walk-in clinic with a week-long history of lightheadedness and palpitations.", "She also complains of fatigability and shortness of breath of the same duration.", "The past medical history is significant for menarche at 9 years of age, heavy menstrual bleeding for the past several years and abdominal pain that worsens during menses.", "She stopped trying to conceive a child after 2 spontaneous abortions in the past 4 years and has been on iron oral supplementation for the last 2 years.", "She adds that she feels a dull pressure-like discomfort in her pelvis and constipation.", "The physical examination is significant for pale mucous membranes and a grade 2/6 ejection systolic murmur in the area of the pulmonic valve.", "Pelvic examination reveals an enlarged, mobile uterus with an irregular contour.", "The hemoglobin level is 10 g/dL and the hematocrit is 27%."], "s1": [0, 1, 5, 7], "s2": [2, 3, 4, 6]} {"key": 411, "questions": "What is the most likely diagnosis?", "options": [{"label": "A", "disease": "Disseminated intravascular coagulation"}, {"label": "B", "disease": "Henoch-Schonlein purpura"}, {"label": "C", "disease": "Systemic vasculitis"}, {"label": "D", "disease": "Hemolytic-uremic syndrome"}], "answer_idx": "D", "symptoms": ["A 3-year-old girl is brought by her parents to the office for bloody diarrhea and a seizure.", "The parents say she started having fever, abdominal pain, and diarrhea about 3 days ago, but the bloody diarrhea started 12 hours ago.", "The seizure was the last symptom to appear 3 hours ago, and it consisted of repetitive movement of arms and legs with loss of consciousness.", "Her 4-year-old brother had a similar case with bloody diarrhea 2 weeks ago.", "Her vital signs include: blood pressure is 130/85 mm Hg, her respiratory rate is 25/min, her heart rate is 120/min, and her temperature is 39.2°C (102.6°F).", "On physical examination, she looks pale and sleepy, the cardiopulmonary auscultation is normal, the abdomen is painful on palpation, and the skin assessment reveals the presence of disseminated pinpoint petechiae.", "The urinalysis show hematuria.", "The Coombs test is negative.", "The complete blood count results are as follows: | Hemoglobin 7 g/dL | Hematocrit 25 % | Leukocyte count 17,000/mm3 | Neutrophils 70% | Bands 2% | Eosinophils 1% | Basophils 0% | Lymphocytes 22% | Monocytes 5% | Platelet count 7,000/mm3 | Her coagulation tests are as follows: | Partial thromboplastin time (activated) 30 seconds | Prothrombin time 12 seconds | Reticulocyte count 1 % | Thrombin time < 2 seconds deviation from control | ."], "s1": [0, 1, 4, 5], "s2": [2, 3, 6, 7, 8]} {"key": 412, "questions": "What is the most likely diagnosis?", "options": [{"label": "A", "disease": "Febrile hemolytic reaction"}, {"label": "B", "disease": "Febrile nonhemolytic reaction"}, {"label": "C", "disease": "Transfusion related bacterial contamination"}, {"label": "D", "disease": "Transfusion related acute lung injury"}], "answer_idx": "B", "symptoms": ["A 45-year-old woman presents to the emergency department with hematemesis.", "Her son said that the patient had complained of gnawing abdominal pain that worsened before and improved after meals for the past 3 weeks.", "She frequently travels for work and is often stressed.", "She drinks alcohol socially and does not smoke tobacco or marijuana.", "She has not had anything to eat or drink for the past 24 hours due to the pain, but coughed up \"several cup of\" frank red blood before arrival.", "Her temperature is 98°F (37°C), blood pressure is 80/50 mmHg, pulse is 140/min, respirations are 23/min, and oxygen saturation is 96% on room air.", "Gastric ultrasound reveals high gastric residual volume.", "The patient is empirically started on 3 units of uncrossmatched O-negative blood transfusion and pantoprazole.", "The patient's initial laboratory data return in the meantime: | | Hemoglobin: 5 g/dL | Hematocrit: 18 % | Leukocyte count: 8,000/mm^3 with normal differential | Platelet count: 400,000/mm^3 | | Serum: | Na+: 140 mEq/L | Cl-: 115 mEq/L | K+: 3.2 mEq/L | HCO3-: 26 mEq/L | BUN: 60 mg/dL | Glucose: 104 mg/dL | Creatinine: 1.1 mg/dL | | Blood type: AB | Rhesus status: negative | | After bedside esophageogastroduodenoscopy with stapling of several peptic ulcers, repeat labs are obtained in 2 hours.", "The patient now complains of feeling hot with intermittent shivering.", "She denies any other symptoms.", "Her temperature is 101°F (38.3°C), blood pressure is 100/55 mmHg, pulse is 100/min, respirations are 20/min, and oxygen saturation is 99% on room air.", "Her lungs are clear to auscultation bilaterally, and no accessory respiratory muscle use is observed.", "No rash is seen."], "s1": [0, 1, 4, 5, 6, 7, 8], "s2": [2, 3, 9, 10, 11, 12, 13]} {"key": 413, "questions": "Which of the following genetic conditions is most likely related to these clinical features?", "options": [{"label": "A", "disease": "Morquio syndrome"}, {"label": "B", "disease": "Tay-Sachs disease"}, {"label": "C", "disease": "Hurler syndrome"}, {"label": "D", "disease": "Fabry disease"}], "answer_idx": "C", "symptoms": ["A 1-year-old boy is brought to his pediatrician’s office due to regression of development.", "She has observed that he is not progressing as he did during the 1st year of his life.", "There seems to be a regression in some behavioral and other milestones.", "She is also concerned about his facial development.", "On examination, the vital signs are normal.", "He has coarse facial features with a bulging frontal bone.", "Additionally, his lips are enlarged and corneal clouding is observed.", "Urine studies show an increase in dermatan sulfate."], "s1": [0, 1, 2, 3], "s2": [4, 5, 6, 7]} {"key": 414, "questions": "Which of the following is most likely the diagnosis?", "options": [{"label": "A", "disease": "Adjustment disorder"}, {"label": "B", "disease": "Illness anxiety disorder"}, {"label": "C", "disease": "Panic disorder"}, {"label": "D", "disease": "Specific phobia"}], "answer_idx": "C", "symptoms": ["A 22-year-old man presents to the emergency department for chest pain.", "He says that the chest pain began a few hours prior to presentation and is associated with nausea, abdominal discomfort, and sweating.", "He has experienced similar symptoms multiple times in the past, which brings him great concern because he believes he may be having a heart attack.", "This concern has persisted for 2 months.", "This has caused him to avoid taking long train rides or driving on divided highways.", "Approximately 7 months ago his parents divorced.", "Medical history is significant for obesity.", "He denies any medications or illicit drug use.", "Physical examination is unremarkable.", "An electrocardiogram demonstrates sinus tachycardia with a pulse of 110/min."], "s1": [0, 1, 2, 8, 9], "s2": [3, 4, 5, 6, 7]} {"key": 415, "questions": "Which of the following is the most likely diagnosis?", "options": [{"label": "A", "disease": "Pulmonary valve stenosis"}, {"label": "B", "disease": "Aortic valve regurgitation"}, {"label": "C", "disease": "Tricuspid valve regurgitation"}, {"label": "D", "disease": "Mitral valve prolapse"}], "answer_idx": "B", "symptoms": ["An otherwise healthy 52-year-old man comes to the physician because of a 2-month history of palpitations.", "He says that he can feel his heart beat in his chest all the time.", "For the past 3 weeks, he has shortness of breath on exertion.", "He has not had chest pain, orthopnea, or nocturnal dyspnea.", "He emigrated from Nepal at the age of 25 years.", "He does not take any medications.", "His temperature is 36.5°C (97.7°F), pulse is 103/min, and blood pressure is 138/56 mm Hg.", "The lungs are clear to auscultation.", "Cardiac examination is shown."], "s1": [0, 1, 2, 3, 8], "s2": [4, 5, 6, 7]} {"key": 416, "questions": "Which of the following is the most likely cause of this patient's seizure?", "options": [{"label": "A", "disease": "Hypoglycemia"}, {"label": "B", "disease": "Benzodiazepine withdrawal"}, {"label": "C", "disease": "Alcohol withdrawal"}, {"label": "D", "disease": "Subdural hemorrhage"}], "answer_idx": "B", "symptoms": ["A 46-year-old woman is brought to the emergency department for respiratory depression.", "Routine urine toxicology screening is positive for alcohol, fentanyl, and benzodiazepines.", "Blood glucose is 80 mg/dL.", "Naloxone, flumazenil, and lactated Ringer's solution are administered intravenously.", "Two hours later, the woman has a tonic-clonic seizure lasting for one minute.", "Her pulse is 100/min, blood pressure is 145/90 mm Hg, and respiratory rate is 24/min.", "Pulse oximetry on room air shows an oxygen saturation of 98%.", "The patient appears drowsy and diaphoretic.", "She is not oriented to place or time but responds to her name.", "Examination shows a hematoma on her left temple.", "Neurological examination shows stiff extremities and a tremor in both hands.", "The remainder of the examination shows no abnormalities."], "s1": [0, 1, 3, 4, 7, 8], "s2": [2, 5, 6, 9, 10, 11]} {"key": 417, "questions": "Which of the following is the most likely diagnosis in this patient?", "options": [{"label": "A", "disease": "Celiac disease"}, {"label": "B", "disease": "Crohn's disease"}, {"label": "C", "disease": "Bacterial overgrowth"}, {"label": "D", "disease": "Hypothyroidism"}], "answer_idx": "B", "symptoms": ["A 45-year-old woman presents with a 2-year history of fatigue, paresthesia, diarrhea, and abdominal pain.", "She says she experiences 2–3 episodes of non-bloody diarrhea per month associated with crampy abdominal pain.", "She also has the bilateral tingling sensation of her upper and lower limbs in a glove and stocking pattern which is sometimes associated with the burning sensation of her hands and legs.", "Past medical history is unremarkable.", "The patient denies smoking, alcohol consumption, illicit drug use.", "Her vital signs include: blood pressure 100/60 mm Hg without postural changes, heart rate 92/min, respiratory rate 16/min, temperature 37.0℃ (98.6℉).", "Laboratory findings are significant for the following: | Hgb 7.2 gm/dL | MCV 110 fL | RBC 3.6 X109/L | Platelets 10,000/mm3 | Total count 4,200/mm3 | Reticulocyte 0.8% | A peripheral blood smear demonstrates hypersegmented neutrophils.", "Anti-intrinsic factor and anti-tissue transglutaminase antibodies were negative.", "An upper GI series with small bowel follow-through is performed which shows a narrowing of the barium contrast in the lumen of the terminal ileum to the width of a string."], "s1": [0, 1, 2, 8], "s2": [3, 4, 5, 6, 7]} {"key": 418, "questions": "What is the most likely cause of this patient’s condition?", "options": [{"label": "A", "disease": "Diffusion defect"}, {"label": "B", "disease": "Anemic hypoxia"}, {"label": "C", "disease": "Ischemic hypoxia"}, {"label": "D", "disease": "Hypoxemic hypoxia"}], "answer_idx": "D", "symptoms": ["A 35-year-old man attempted to climb Mount Everest.", "At an altitude of 7,000 meters, he became short of breath and developed a dusky blue tinge around his face and lips.", "A physician in the climbing party performs a rudimentary physical exam which reveals a respiratory rate of 30/min, blood pressure of 130/90 mm Hg, heart rate of 108/min, and temperature of 36.4°C (97.5°F), with few basal crackles on the right side of chest.", "No jugular venous distension or dependent edema is noted.", "With this information, the mountain climber is sent down the mountain for further evaluation.", "Investigation reveals a normal chest X-ray with a normal heart size."], "s1": [0, 4], "s2": [1, 2, 3, 5]} {"key": 419, "questions": "Which of the following is the most likely precipitating factor for this patient's symptoms?", "options": [{"label": "A", "disease": "Elevated systemic vascular resistance"}, {"label": "B", "disease": "Destruction of gut anaerobes"}, {"label": "C", "disease": "Presence of hemoglobin in the intestine"}, {"label": "D", "disease": "Low protein consumption"}], "answer_idx": "C", "symptoms": ["A 56-year-old man is brought to the emergency department by his wife because of increasing confusion and lethargy for the past 12 hours.", "He is oriented only to person.", "His temperature is 37.3°C (99.1°F), pulse is 109/min, respirations are 18/min, and blood pressure is 108/67 mm Hg.", "Examination shows abdominal distention and several erythematous, lacy lesions on the chest that blanch with pressure.", "His hands make a flapping motion when they are dorsiflexed."], "s1": [0, 1, 2], "s2": [3, 4]} {"key": 420, "questions": "What is the most likely diagnosis?", "options": [{"label": "A", "disease": "Roseola"}, {"label": "B", "disease": "Erythema infectiosum"}, {"label": "C", "disease": "Rubella"}, {"label": "D", "disease": "Kawasaki disease"}], "answer_idx": "A", "symptoms": ["Parents bring an 11-month-old baby to the clinic because the baby has a fever of 39.0°C (102.2°F).", "The baby is irritated and crying constantly.", "She is up to date on immunizations.", "A complete physical examination reveals no significant findings, and all laboratory tests are negative.", "Five days after resolution of her fever, she develops a transient maculopapular rash."], "s1": [0, 1, 4], "s2": [2, 3]} {"key": 421, "questions": "Which of the following is the most likely diagnosis?\"", "options": [{"label": "A", "disease": "Chronic pancreatitis with pseudocyst rupture"}, {"label": "B", "disease": "Emphysematous cholecystitis"}, {"label": "C", "disease": "Gallstone ileus"}, {"label": "D", "disease": "Primary sclerosing cholangitis"}], "answer_idx": "B", "symptoms": ["A 73-year-old man comes in to the emergency department because of nausea and severe pain in his upper abdomen and right shoulder blade for the past day.", "He was able to eat a little for breakfast but vomited up the meal a couple of hours later.", "He has type 2 diabetes mellitus treated with metformin.", "He has smoked half a pack of cigarettes daily for 40 years and drinks 4 beers every week.", "He appears acutely distressed.", "His temperature is 38.8°C (102°F), pulse is 124/min, and blood pressure is 92/68 mm Hg.", "Pulse oximetry on room air shows an oxygen saturation of 95%.", "The abdomen is soft and there is tenderness to palpation of the right upper quadrant with soft crepitus.", "The remainder of the examination shows no abnormalities.", "Laboratory studies show: | Hemoglobin 14.3 g/dL | Leukocyte count 18,100/mm3 | Platelet count 216,000/mm3 | Serum | Aspartate aminotransferase (AST, SGOT) 41 U/L | Alanine aminotransferase (ALT, SGPT) 19 U/L | Alkaline phosphatase 110 U/L | Total bilirubin 0.9 mg/dL | Direct bilirubin 0.2 mg/dL | Albumin 4.1 g/dL | Ultrasonography of the right upper quadrant shows a gallbladder with an air-filled, thickened wall and no stones in the lumen."], "s1": [2, 3, 6, 8], "s2": [0, 1, 4, 5, 7, 9]} {"key": 422, "questions": "Which of the following is the most likely diagnosis?", "options": [{"label": "A", "disease": "Language disorder"}, {"label": "B", "disease": "Rett syndrome"}, {"label": "C", "disease": "Fragile X syndrome"}, {"label": "D", "disease": "Trisomy 21"}], "answer_idx": "B", "symptoms": ["A 12-month-old girl is brought in by her mother to the pediatrician for the first time since her 6-month checkup.", "The mother states that her daughter had been doing fine, but the parents are now concerned that their daughter is still not able to stand up or speak.", "On exam, the patient has a temperature of 98.5°F (36.9°C), pulse is 96/min, respirations are 20/min, and blood pressure is 100/80 mmHg.", "The child appears to have difficulty supporting herself while sitting.", "The patient has no other abnormal physical findings.", "She plays by herself and is making babbling noises but does not respond to her own name.", "She appears to have some purposeless motions.", "A previous clinic note documents typical development at her 6-month visit and mentioned that the patient was sitting unsupported at that time."], "s1": [0, 1, 3, 5, 6], "s2": [2, 4, 7]} {"key": 423, "questions": "Which of the following is the most likely diagnosis?", "options": [{"label": "A", "disease": "Idiopathic thrombocytopenic purpura"}, {"label": "B", "disease": "Acute myeloid leukemia"}, {"label": "C", "disease": "Acute lymphoblastic leukemia"}, {"label": "D", "disease": "Hodgkin lymphoma"}], "answer_idx": "C", "symptoms": ["A 4-year-old boy is brought to the physician because of a 1-month history of generalized fatigue.", "During the past week, he has also had fever and severe leg pain that keeps him awake at night.", "Examination shows cervical and axillary lymphadenopathy.", "His liver is palpated 4 cm below the right costal margin and the spleen is palpated 3 cm below the left costal margin.", "His hemoglobin concentration is 10.2 g/dL, leukocyte count is 64,500/mm3, and platelet count is 29,000/mm3.", "A bone marrow aspirate predominantly shows immature cells that stain positive for CD10, CD19, and TdT."], "s1": [0, 1, 3], "s2": [2, 4, 5]} {"key": 424, "questions": "Which of the following is the most likely diagnosis?", "options": [{"label": "A", "disease": "Conduction aphasia"}, {"label": "B", "disease": "Global aphasia"}, {"label": "C", "disease": "Wernicke aphasia"}, {"label": "D", "disease": "Transcortical sensory aphasia"}], "answer_idx": "C", "symptoms": ["A 73-year-old man is brought to the emergency department 30 minutes after the sudden onset of right-sided body weakness.", "His wife reports that he does not seem to understand simple questions.", "He has type 2 diabetes mellitus and has smoked 1 pack of cigarettes daily for 45 years.", "The patient speaks fluently, but he answers questions with nonsensical phrases and cannot repeat single words."], "s1": [0, 3], "s2": [1, 2]} {"key": 425, "questions": "This patient's skin finding is most likely associated with which of the following conditions?", "options": [{"label": "A", "disease": "Niacin deficiency"}, {"label": "B", "disease": "Primary adrenal insufficiency"}, {"label": "C", "disease": "Polycystic ovarian syndrome"}, {"label": "D", "disease": "Hashimoto thyroiditis"}], "answer_idx": "C", "symptoms": ["A 23-year-old woman comes to physician for an annual health maintenance examination.", "She feels well.", "She is 155 cm (5 ft 1 in) tall and weighs 79 kg (174 lb); BMI is 33 kg/m2.", "Examination shows a skin rash over both axillae.", "A photograph of her left axilla is shown."], "s1": [0, 1, 2], "s2": [3, 4]} {"key": 426, "questions": "Which of the following is the most likely underlying cause of this patient's symptoms?", "options": [{"label": "A", "disease": "Pancoast tumor"}, {"label": "B", "disease": "Constrictive pericarditis"}, {"label": "C", "disease": "Descending aortic aneurysm"}, {"label": "D", "disease": "Mediastinal lymphoma"}], "answer_idx": "D", "symptoms": ["A 78-year-old man comes to the physician because of a 3-week history of dyspnea, chest pain, dry cough, painful swallowing, and a feeling of fullness in his head.", "The symptoms were initially mild but have progressively worsened.", "He has a history of gastroesophageal reflux disease treated with esomeprazole.", "He does not smoke cigarettes.", "His pulse is 99/min and blood pressure is 95/66 mm Hg.", "Examination shows swelling and redness of the face and neck.", "There is distention of the superficial veins of his neck and upper thorax.", "Plain radiographs of the chest show a widened mediastinum and unremarkable lung fields."], "s1": [0, 1, 4, 5, 6, 7], "s2": [2, 3]} {"key": 427, "questions": "What is the most likely explanation for this patient’s symptoms?", "options": [{"label": "A", "disease": "Implantation of the embryo outside of the uterus"}, {"label": "B", "disease": "Inflammation of the appendix"}, {"label": "C", "disease": "Mutation of the porphobilinogen deaminase enzyme"}, {"label": "D", "disease": "Obstruction of the cystic duct and subsequent inflammation of the gallbladder"}], "answer_idx": "C", "symptoms": ["A 37-year-old woman is brought to the emergency department for acute abdominal pain for the past 2 hours.", "She reports that the pain is constant, 10/10, and is diffusely spread across her stomach.", "She reports nausea and 1 episode of vomiting 1 hour ago, painless bloody urine, and paresthesias in her hands bilaterally.", "The patient claims similar episodes in the past following the use of alcohol, though with milder pain.", "Her past medical history is significant for pelvic inflammatory disease that was adequately treated 5 years ago.", "She is currently sexually active with her husband without contraceptive use.", "Her temperature is 98.6°F (37°C), blood pressure is 148/98 mmHg, pulse is 103/min, respirations are 18/min, and oxygen saturation is 99% on room air.", "A physical examination demonstrates a patient in acute distress with diffuse abdominal tenderness and decreased sensation of the hands bilaterally."], "s1": [0, 1, 3, 4, 5], "s2": [2, 6, 7]} {"key": 428, "questions": "Which of the following is the most likely cause of this patient's jaundice?\"", "options": [{"label": "A", "disease": "Biliary obstruction"}, {"label": "B", "disease": "Hepatocellular injury"}, {"label": "C", "disease": "Increased formation of bilirubin"}, {"label": "D", "disease": "Decreased hepatic uptake of unconjugated bilirubin\n\""}], "answer_idx": "C", "symptoms": ["Ten days after a motor vehicle collision, a 28-year-old man develops jaundiced skin.", "Upon initial presentation for his injuries, the patient was taken for an emergency laparotomy, which revealed significant internal hemorrhage from blunt abdominal trauma to the spleen.", "He required rapid transfusion with a total of 7 units of packed red blood cells.", "He has recovered well from the procedure until this morning, when he began to develop jaundiced skin and sclerae.", "He does not have pruritus.", "He has had no prior surgeries and takes no other medications.", "He is sexually active with one female partner.", "Prior to the accident, he drank 4 beers per day.", "His vital signs are within normal limits.", "Abdominal examination is limited due to pain.", "There are no palpable abdominal masses.", "There is a midline surgical scar with no erythema, purulence, or drainage.", "He has healing abrasions on the upper left side of his face and bruises over the anterior abdomen.", "CT scans show a resolving hematoma in the peritoneal cavity.", "Laboratory studies show: | Hemoglobin 9.7 g/dL | Hematocrit 30% | Leukocyte count 7,000/mm3 | Platelet count 135,000/mm3 | Serum | Total bilirubin 3.9 mg/dL | Indirect bilirubin 3.7 mg/dL | Direct bilirubin 0.2 mg/dL | Aspartate aminotransferase (AST) 60 U/L | Alanine aminotransferase (ALT) 92 U/L | ."], "s1": [0, 3, 4, 6, 7, 14], "s2": [1, 2, 5, 8, 9, 10, 11, 12, 13]} {"key": 429, "questions": "Which of the following is the most likely diagnosis?", "options": [{"label": "A", "disease": "Tuberculosis"}, {"label": "B", "disease": "Lobar pneumonia"}, {"label": "C", "disease": "Small cell lung carcinoma"}, {"label": "D", "disease": "Squamous cell lung carcinoma\n\""}], "answer_idx": "D", "symptoms": ["A 69-year-old man comes to the physician because of a cough for the past 3 months.", "The cough is mostly dry, but the patient recalls a recent episode in which he coughed up mucus with dark red streaks.", "He has felt tired for the past few months.", "Over the past month, he has had poor appetite and a 5-kg (11-lb) weight loss.", "He also has occasional back pain, which has worsened in recent weeks.", "He has no recent travel history.", "He has smoked a pack of cigarettes daily for 30 years.", "He does not drink alcohol.", "His vitals are within normal limits.", "Auscultation of the lungs reveals wheezing in the lower right lung field.", "There is no peripheral lymphadenopathy.", "The liver is of normal size, and the spleen is not palpable.", "Laboratory studies show a hemoglobin concentration of 13.5 g/dL, serum calcium concentration of 12.3 mg/dL, and a total serum protein of 7.0 g/dL.", "X-rays of the chest are shown."], "s1": [0, 1, 3, 4, 6, 9, 13], "s2": [2, 5, 7, 8, 10, 11, 12]} {"key": 430, "questions": "What is the most likely cause of this patient’s condition?", "options": [{"label": "A", "disease": "Cyanide poisoning"}, {"label": "B", "disease": "Methemoglobinemia"}, {"label": "C", "disease": "Sulfhemoglobinemia"}, {"label": "D", "disease": "Carbon monoxide poisoning"}], "answer_idx": "B", "symptoms": ["A 16-year-old boy is brought to the emergency department by his parents after collapsing at home.", "He was resting at home after an uneventful dental procedure that involved the extraction of several teeth.", "He became drowsy and then unconscious and was unrousable.", "At the hospital, his temperature is 37.0° C (98.6° F), respiratory rate is 15/min, pulse rate is 67/min, and blood pressure is 122/98 mm Hg.", "Oxygen saturation is 85% on room air.", "The patient is deeply cyanosed despite a good respiratory effort and a clear airway.", "His lungs are clear to auscultation, bilaterally.", "Even though an endotracheal tube is introduced and assisted ventilation is induced, his condition does not improve.", "A review of dental records reveals the details of the procedure where the local anesthetic pilocarpine was administered."], "s1": [0, 1, 2, 8], "s2": [3, 4, 5, 6, 7]} {"key": 431, "questions": "Which of the following is the most likely underlying cause of this patient's upper extremity symptoms?", "options": [{"label": "A", "disease": "Injury to the median nerve"}, {"label": "B", "disease": "Damage to the upper trunk of the brachial plexus"}, {"label": "C", "disease": "Damage to the lower trunk of the brachial plexus"}, {"label": "D", "disease": "Aspiration of meconium"}], "answer_idx": "B", "symptoms": ["Patient Information.", "Age: 1 day.", "Sex: F.", "Ethnicity: Hispanic.", "Site of Care: office.", "History.", "Reason for Visit/Chief Concern: brought in by her parents because “her arm looks funny”.", "History of Present Illness: mother had no prenatal care.", "labor was spontaneous with rupture of membranes yielding fluid with dark green streaks.", "the infant was delivered vaginally 1 day ago at home at approximately 39 weeks' gestation.", "the delivery was complicated by shoulder dystocia, which was managed with suprapubic pressure and the McRoberts maneuver.", "father reports that the infant's right arm “just hangs by the side” and that she never bends her right elbow.", "the infant is breastfeeding, stooling, and voiding without complication.", "Past Medical History: none.", "Family History: mother has type 2 diabetes mellitus.", "Medications: none.", "Immunizations: has not received any routine vaccinations.", "Allergies: no known drug allergies.", "Social History: the infant lives with her mother, father, and paternal grandmother.", "no one in the residence smokes.", "Physical Examination.", "Temp Pulse Resp BP O2 Sat Ht Wt Head circumference.", "37.1°C.", "(98.8°F).", "154/min 45/min 87/49 mm Hg.", "99%.", "50 cm.", "(20 in; 69th percentile).", "4,400 g.", "(9 lb 11 oz; 99th percentile).", "35 cm.", "(13.8 in; 82nd percentile).", "Appearance: well-appearing; crying during the examination.", "HEENT: red reflex is seen bilaterally; there is a fluctuant area over the left parietal bone that crosses suture lines.", "Pulmonary: clear to auscultation.", "Cardiac: regular rate and rhythm; normal S1 and S2; no murmurs, rubs, or gallops.", "Abdominal: no tenderness, masses, or hepatosplenomegaly; bowel sounds normal; umbilical stump is intact and clamped.", "Extremities: hips are stable bilaterally.", "Musculoskeletal: clavicles are intact bilaterally; the right upper extremity hangs limply from the shoulder in full extension, adduction, and fixed internal rotation; the hand is pronated, and the wrist and fingers are flexed.", "Skin: dry, warm; no jaundice.", "Neurologic: normal suck and grasp reflexes; the Moro reflex is normal in the left upper extremity and absent in the right upper extremity; deep tendon reflexes are 2+ bilaterally. Please refer to the summary above to answer this question."], "s1": [0, 1, 2, 3, 4, 5, 6, 13, 14, 15, 16, 17, 18, 19, 21, 22, 23, 24, 25, 26, 27, 28, 29, 30, 31, 32, 33, 34, 35, 36, 37, 39], "s2": [7, 8, 9, 10, 11, 12, 20, 38, 40]} {"key": 432, "questions": "Which of the following is the most likely diagnosis?", "options": [{"label": "A", "disease": "Osteoporosis"}, {"label": "B", "disease": "Osteomalacia"}, {"label": "C", "disease": "Osteitis deformans"}, {"label": "D", "disease": "Osteonecrosis"}], "answer_idx": "C", "symptoms": ["A 70-year-old man is brought to the emergency department because of severe back pain that began when he was lifting a box 1 hour ago.", "He also has a 2-year history of increasingly severe right hip pain.", "Physical examination shows tenderness to palpation of the lower spine as well as erythema of the skin over the right hip.", "Neurologic examination shows decreased hearing in the left ear; the Weber test localizes to the left side.", "Serum studies show an alkaline phosphatase concentration of 410 U/L, calcium concentration of 9.5 mg/dL, and parathyroid hormone level of 322 pg/mL.", "An x-ray of the spine shows a fracture of the L4 vertebra."], "s1": [0, 1, 2, 5], "s2": [3, 4]} {"key": 433, "questions": "Which of the following is the most likely diagnosis?", "options": [{"label": "A", "disease": "Ventricular septal defect"}, {"label": "B", "disease": "Patent ductus arteriosus"}, {"label": "C", "disease": "Atrial septal defect"}, {"label": "D", "disease": "Tricuspid atresia"}], "answer_idx": "A", "symptoms": ["An 8-year-old African-American male is found to have a holosystolic, harsh-sounding murmur upon physical examination.", "The murmur is best appreciated at the left sternal border, and is found to be louder when the patient squats."], "s1": [0], "s2": [1]} {"key": 434, "questions": "Which of the following is the most likely diagnosis?", "options": [{"label": "A", "disease": "Rheumatoid arthritis"}, {"label": "B", "disease": "Systemic lupus erythematosus"}, {"label": "C", "disease": "Primary Sjogren’s syndrome"}, {"label": "D", "disease": "Polymyositis"}], "answer_idx": "C", "symptoms": ["A 55-year-old woman comes to the office complaining of a dry mouth for the past few months.", "She has to drink water more frequently, as she finds it difficult to chew and swallow solid foods.", "She has to wake up 3–5 times each night to drink due to intense thirst.", "She also complains of a foreign body sensation in both the eyes for the past month.", "She has had no joint pain, fever, weight loss, or urinary or bowel changes.", "She does not smoke cigarettes but drinks alcohol socially.", "Her mother has rheumatoid arthritis for which she takes methotrexate, and her father died of prostatic carcinoma 7 years ago.", "Her temperature is 36.7°C (98°F), blood pressure is 130/75 mm Hg, pulse is 80/min, respirations are 14/min, and BMI is 28 kg/m2.", "On examination, the eyes and oral cavity appear dry, and dental caries are present.", "No lymphadenopathy is noted.", "Cardiopulmonary and abdominal examinations are negative.", "Laboratory results are shown below: | Complete blood count: | Hemoglobin 10 g/dL | Leukocytes 13,500/mm3 | Platelets 170,000/mm3 | ESR 65 mm/hr | Antinuclear antibody Positive | Rheumatoid factor Positive | Anti dsDNA Negative | Anti Ro Positive | Anti-CCP Negative | Anti Jo 1 Negative | ."], "s1": [0, 1, 2, 3, 8], "s2": [4, 5, 6, 7, 9, 10, 11]} {"key": 435, "questions": "Which of the following is the most likely diagnosis in this patient?", "options": [{"label": "A", "disease": "Amebiasis"}, {"label": "B", "disease": "Crohn’s disease"}, {"label": "C", "disease": "Salmonellosis"}, {"label": "D", "disease": "Diverticulosis"}], "answer_idx": "A", "symptoms": ["A 35-year-old man presents with loose stools and left lower quadrant abdominal pain.", "He says he passes 8–10 loose stools per day.", "The volume of each bowel movement is small and appears mucoid with occasional blood.", "The patient reports a 20-pack-year smoking history.", "He also says he recently traveled abroad about 3 weeks ago to Egypt.", "The vital signs include: blood pressure 120/76 mm Hg, pulse 74/min, and temperature 36.5°C (97.8°F).", "On physical examination, mild to moderate tenderness to palpation in the left lower quadrant with no rebound or guarding is present.", "Rectal examination shows the presence of perianal skin ulcers."], "s1": [0, 1, 2, 6], "s2": [3, 4, 5, 7]} {"key": 436, "questions": "Which of the following is the most likely etiology of this patient’s condition?", "options": [{"label": "A", "disease": "Hepatocellular carcinoma"}, {"label": "B", "disease": "Portal hypertension"}, {"label": "C", "disease": "Spontaneous bacterial peritonitis"}, {"label": "D", "disease": "Hepatorenal syndrome"}], "answer_idx": "B", "symptoms": ["A 50-year-old man presents for a routine examination.", "Past medical history is significant for cirrhosis secondary to hepatitis C virus (HCV) infection diagnosed 4 years ago and complicated by ascites.", "Current medications include furosemide 40 mg orally daily.", "Physical examination is unremarkable.", "Laboratory findings are significant for the following: | Laboratory test | Aspartate Aminotransferase (AST) 80 U/L | Alanine Aminotransferase (ALT) 50 U/L | Total bilirubin 2.5 mg/dL | Direct bilirubin 1.8 mg/dL | Alkaline phosphatase (ALP) 140 U/L | International normalized ratido (INR) 1.9 | Serum creatinine 1 mg/dL | Urinalysis | Sodium 200 mmol/24h | Potassium 60 mmol/24h | Protein Nil | RBCs Nil | RBC casts Nil | WBCs Nil | Urea 13 g/24h | Creatinine 6 mmol/24h | Abdominal and renal ultrasound reveals no interval change over the past 6 months.", "Moderate ascites is present.", "Upper GI endoscopy reveals esophageal varices with a hepatic venous pressure gradient measuring 14 mm Hg.", "Diagnostic paracentesis is performed and yields a clear liquid with an absolute polymorphonuclear neutrophil (PMN) count of 75 cells/mm3."], "s1": [0, 2, 3], "s2": [1, 4, 5, 6, 7]} {"key": 437, "questions": "What is the most likely cause of this patient's menorrhagia?", "options": [{"label": "A", "disease": "Factor V Leiden"}, {"label": "B", "disease": "Von Willebrand's disease"}, {"label": "C", "disease": "Hemophilia B"}, {"label": "D", "disease": "Antiphospholipid antibody syndrome"}], "answer_idx": "B", "symptoms": ["A 25-year-old woman comes into your office with complaints of heavy bleeding.", "She states that her mother also has heavy bleeding during her menstrual cycle.", "She has had a heavy flow as long as she can remember and had her first menstrual cycle at age 12.", "She states during her cycle, she has to change pads every 2 hours for at least 3 days.", "She also states that she gets bruised easily just like her mother.", "She denies any past medical history other than her heavy menstrual flow and denies taking any medications.", "She also denies any medical history in her father and says he is \"perfectly healthy.\" Her vitals are HR 85, T 98.8 F, RR 13, BP 125/75.", "Her CBC is significant for Hgb 10.5, WBC 5.8, Plts 250, Hct 33.", "On coagulation studies, her PT is 14 seconds and her PTT is 43 seconds.", "Her INR is 1.1."], "s1": [0, 1, 2, 3, 5, 6], "s2": [4, 7, 8, 9]} {"key": 438, "questions": "Which of the following is the most likely underlying cause of this patient’s current condition?", "options": [{"label": "A", "disease": "Acoustic neuroma"}, {"label": "B", "disease": "Cholesteatoma"}, {"label": "C", "disease": "Chronic serous otitis media"}, {"label": "D", "disease": "Chronic suppurative otitis media"}], "answer_idx": "B", "symptoms": ["A 14-year-old girl is brought to the emergency department because of occipital headache, nausea, and vomiting for the last 2 hours.", "She has a multi-year history of frequent left ear infections and discharge, with poor response to antimicrobial therapy.", "She has muffled hearing in the left ear.", "Her blood pressure is 134/78 mm Hg, the pulse is 83/min, the respiratory rate is 16/min, and the temperature is 36.5°C (97.7°F).", "She is alert and oriented.", "Physical examination of the left ear shows perforation of the tympanic membrane, granulation tissue, and white keratinaceous debris in the posterosuperior quadrant of the tympanic membrane.", "An MRI shows evidence of sigmoid sinus thrombosis on the left side and a hyperintense area in the middle ear on."], "s1": [0, 3, 4], "s2": [1, 2, 5, 6]} {"key": 439, "questions": "Which of the following is the most likely diagnosis?", "options": [{"label": "A", "disease": "Rhabdomyosarcoma"}, {"label": "B", "disease": "Neurofibroma"}, {"label": "C", "disease": "Sarcoma"}, {"label": "D", "disease": "Aortic aneurysm"}], "answer_idx": "B", "symptoms": ["A 23-year-old woman comes to the physician for a 6-month history of dry cough, hoarseness, and chest pain.", "She does not smoke and has not lost weight.", "Laboratory studies show no abnormalities.", "An x-ray of the chest shows a mass that projects across the right hilum.", "A CT scan of the chest is shown."], "s1": [0, 3, 4], "s2": [1, 2]} {"key": 440, "questions": "Which of the following is the most likely diagnosis in this patient?", "options": [{"label": "A", "disease": "Arnold-Chiari malformation type 1"}, {"label": "B", "disease": "Arnold-Chiari malformation type 2"}, {"label": "C", "disease": "Arnold-Chiari malformation type 3"}, {"label": "D", "disease": "Arnold-Chiari malformation type 4"}], "answer_idx": "A", "symptoms": ["A 32-year-old male presents to his primary care physician with complaints of chronic headaches that have developed and increased in frequency and severity over the last several months.", "Additionally, the patient has noted he has been less coordinated over the last few weeks, stumbling and tripping often when he is walking.", "Physical examination is significant for notably reduced hand grip strength bilaterally as well as decreased pain and temperature sensation along the upper back and down both arms to the hands.", "A referral to the appropriate specialist is made, and an MRI of the brain and neck is obtained.", "Results of the MRI are show in Figures A and B."], "s1": [0, 1], "s2": [2, 3, 4]} {"key": 441, "questions": "Which of the following is the most likely diagnosis?", "options": [{"label": "A", "disease": "Osgood-Schlatter disease"}, {"label": "B", "disease": "Patellar stress fracture"}, {"label": "C", "disease": "Patellar tendonitis"}, {"label": "D", "disease": "Patellofemoral syndrome"}], "answer_idx": "C", "symptoms": ["A 17-year-old girl comes to the primary care clinic with her father complaining of right knee pain.", "She reports that the pain started about a month ago, and since then it has gotten progressively worse.", "The knee pain is not constant but becomes most noticeable when going up or down the stairs.", "She also endorses that her knee becomes uncomfortable towards the end of class.", "Her father is worried because the pain is affecting her ability to play basketball, and she has college scouts coming to watch her play.", "The patient has no chronic medical conditions.", "She had a tonsillectomy as a child.", "She takes a multivitamin and uses ibuprofen as needed for the pain.", "On physical examination, there is tenderness at the inferior pole of the patella, without swelling or overlying skin changes."], "s1": [0, 1, 2, 3, 4, 8], "s2": [5, 6, 7]} {"key": 442, "questions": "Which of the following is the most likely diagnosis?\"", "options": [{"label": "A", "disease": "Chronic granulomatous disease"}, {"label": "B", "disease": "DiGeorge syndrome"}, {"label": "C", "disease": "Chediak-Higashi syndrome"}, {"label": "D", "disease": "Wiskott-Aldrich syndrome"}], "answer_idx": "D", "symptoms": ["A 13-month-old boy is brought to the physician for the evaluation of rectal bleeding that occurred earlier that morning.", "The patient has also had several itchy and red skin lesions that started on his scalp and spread downwards.", "The parents report that their son has had six episodes of bilateral otitis media since birth.", "His immunizations are up-to-date.", "He is at the 3rd percentile for height and weight.", "His vital signs are within normal limits.", "Examination shows several eczematous lesions over the scalp, neck, and upper and lower extremities, as well as multiple red spots that do not blanch on pressure.", "The remainder of the physical examination shows no abnormalities.", "Laboratory studies show: | Hemoglobin 9.4 g/dL | Leukocyte count 11,500/mm3 | Platelet count 30,000/mm3 | Prothrombin time 14 sec | Partial thromboplastin time 33 sec | ."], "s1": [0, 1, 6], "s2": [2, 3, 4, 5, 7, 8]} {"key": 443, "questions": "Which of the following is the most likely diagnosis?", "options": [{"label": "A", "disease": "Pulmonary embolism"}, {"label": "B", "disease": "Panic attack"}, {"label": "C", "disease": "Bacterial pneumonia"}, {"label": "D", "disease": "Acute pericarditis"}], "answer_idx": "A", "symptoms": ["A 28-year-old woman, gravida 3, para 2, at 34 weeks' gestation comes to the physician because of a 1-day history of dyspnea, dry cough, and chest pain.", "Her pulse is 112/min, respirations are 24/min, and blood pressure is 108/78 mm Hg.", "Pulse oximetry on room air shows an oxygen saturation of 90%.", "Examination shows jugular venous distention and bilateral pitting edema below the knees that is worse on the right side.", "There is dullness to percussion over the right lung base."], "s1": [0, 1], "s2": [2, 3, 4]} {"key": 444, "questions": "Which of the following is the most likely cause of this patient's symptoms?", "options": [{"label": "A", "disease": "Pneumocystis pneumonia"}, {"label": "B", "disease": "Aspiration pneumonia"}, {"label": "C", "disease": "Mitral valve regurgitation"}, {"label": "D", "disease": "Pharyngoesophageal diverticulum\n\""}], "answer_idx": "B", "symptoms": ["A 55-year-old man is brought to the emergency department because of cough and poor appetite for the past week.", "Since it began, he has been coughing up small amounts of malodorous phlegm.", "During the past two nights, he has also had night sweats.", "He was diagnosed with HIV infection 5 years ago.", "He has hypertension, type 2 diabetes mellitus, and severe heartburn.", "The patient is homeless and does not take any medication.", "He has smoked a pack of cigarettes daily for 30 years.", "He drinks 8–10 beers daily.", "His temperature is 38.9°C (102.0°F), pulse is 101/min, respirations are 25/min and blood pressure is 145/92 mm Hg.", "The patient appears intoxicated.", "Physical examination shows crackles and dullness to percussion at the right lung base.", "Scattered expiratory wheezing is heard throughout both lung fields.", "A grade 2/6 mid-systolic ejection murmur is heard along the upper right sternal border.", "His CD4+T-lymphocyte count is 280/mm3 (Normal ≥ 500).", "An x-ray of the chest shows a hazy infiltrate in the right lower lung field."], "s1": [0, 1, 2, 10, 11, 14], "s2": [3, 4, 5, 6, 7, 8, 9, 12, 13]} {"key": 445, "questions": "What is the most likely diagnosis?", "options": [{"label": "A", "disease": "Renal osteodystrophy"}, {"label": "B", "disease": "Osteopenia"}, {"label": "C", "disease": "Osteomalacia"}, {"label": "D", "disease": "Transplantation-related osteoporosis"}], "answer_idx": "D", "symptoms": ["A 45-year-old man who underwent liver transplantation 3 months ago for chronic liver failure presents to the physician because of a backache following a fall from sitting.", "He is currently on immunosuppressive therapy with glucocorticoids and cyclosporine.", "He has no comorbidities.", "On physical examination, his vitals are within normal limits.", "He has tenderness over his lumbar spine.", "An X-ray of the lumbar spine shows a wedge compression fracture of the L1 vertebra.", "His serum testosterone and serum creatinine levels are normal.", "Bone mineral densitometry shows a T-score of –3.0."], "s1": [0, 1, 2, 3], "s2": [4, 5, 6, 7]} {"key": 446, "questions": "Which of the following is the most likely underlying cause of this patient's symptoms?", "options": [{"label": "A", "disease": "Immune thrombocytopenic purpura"}, {"label": "B", "disease": "Sickle cell disease"}, {"label": "C", "disease": "Systemic lupus erythematosus"}, {"label": "D", "disease": "Adverse effect of medication"}], "answer_idx": "D", "symptoms": ["A 14-year-old girl is brought to the physician by her mother for evaluation of several bruises on her lower extremities.", "She has had these bruises for about 6 weeks, and the mother is concerned that she might be bullied at school.", "The patient has had increasing fatigue and paleness over the past several days.", "She has a history of recurrent generalized tonic-clonic seizures treated with carbamazepine.", "She appears pale and ill.", "Her temperature is 37.8°C (100.1°F), pulse is 115/min, and blood pressure is 100/60 mm Hg.", "The lungs are clear to auscultation.", "Examination shows a soft, nontender abdomen with no organomegaly.", "There are several subcutaneous purple spots on her legs bilaterally.", "Her hemoglobin concentration is 8.4 g/dL, leukocyte count is 2,600/mm3, platelet count is 18,000/mm3, and reticulocyte count is 0.3%.", "Serum electrolyte concentrations are within normal limits."], "s1": [3, 6, 7, 10], "s2": [0, 1, 2, 4, 5, 8, 9]} {"key": 447, "questions": "Which is the most likely diagnosis of this condition?", "options": [{"label": "A", "disease": "Vitiligo"}, {"label": "B", "disease": "Tinea versicolor"}, {"label": "C", "disease": "Pityriasis alba"}, {"label": "D", "disease": "Halo nevus"}], "answer_idx": "A", "symptoms": ["A 33-year-old man presents to his primary care practitioner, complaining about the presence of white spots in both of his hands.", "He states that the white spots have expanded in the last few months; they are not tender nor ulcerated.", "His past medical history is relevant for hypothyroidism.", "Upon physical examination, the patient shows hypopigmented macules on both hands and on the back and shoulders adjacent to a patch of skin, with signs of excoriation and scratching.", "Under the Wood’s lamp, the skin lesions on the hands, back, and shoulders show fluorescence.", "There are no signs of inflammation in any of the skin lesions.", "The vital signs of the patient are within normal limits."], "s1": [2, 6], "s2": [0, 1, 3, 4, 5]} {"key": 448, "questions": "Which of the following congenital heart diseases does this infant most likely present with?", "options": [{"label": "A", "disease": "Supravalvar aortic stenosis"}, {"label": "B", "disease": "Tetralogy of Fallot"}, {"label": "C", "disease": "Atrial septal defect"}, {"label": "D", "disease": "Atrioventricular septal defect"}], "answer_idx": "D", "symptoms": ["A 6-month-old infant is brought to a pediatrician for his scheduled immunizations.", "The parents deny any specific current complaints, but his facial features differ from those of other children in the family.", "During the physical examination, the pediatrician notes that the infant’s vital signs are stable.", "His facial features include a medial epicanthic fold, a face that appears flat, and a flat occiput with low-set ears.", "The pediatrician also notes a single transverse palmar crease on both hands.", "An echocardiogram is performed which suggests that the infant has a congenital heart disease which is the most common form of congenital heart disease seen in children with this particular genetic disorder."], "s1": [0, 1, 2], "s2": [3, 4, 5]} {"key": 449, "questions": "Which of the following is the most likely diagnosis?", "options": [{"label": "A", "disease": "NSTEMI"}, {"label": "B", "disease": "Stable angina"}, {"label": "C", "disease": "Unstable angina"}, {"label": "D", "disease": "Variant angina"}], "answer_idx": "C", "symptoms": ["A 72-year-old man with a history of chronic kidney disease presents to his primary care physician complaining of recurrent chest pain with activity.", "The patient used to have chest pain when he mowed his lawn.", "Now he gets chest pain whenever he walks short distances such as to get his mail.", "The pain resolves on its own when the patient sits and rests.", "His temperature is 98.2°F (36.8°C), blood pressure is 157/98 mm Hg, pulse is 80/min, respirations are 15/min, and oxygen saturation is 98% on room air.", "Physical exam is notable for an obese man who is in no distress.", "An initial ECG is unchanged from a previous ECG.", "The patient's first troponin is 0.06 ng/mL which is unchanged from previous troponins."], "s1": [0, 1, 2, 3], "s2": [4, 5, 6, 7]} {"key": 450, "questions": "What is the most likely diagnosis?", "options": [{"label": "A", "disease": "Acute labyrinthitis"}, {"label": "B", "disease": "Benign paroxysmal positional vertigo"}, {"label": "C", "disease": "Cerebellar tumor"}, {"label": "D", "disease": "Meniere's disease"}], "answer_idx": "C", "symptoms": ["A 73-year-old male is brought to his family practitioner by his daughter with the complaints of a spinning sensation for the past 4 weeks.", "He says that the room appears to be continuously spinning.", "This has progressively worsened over the last 4 weeks to the point that he has become bed bound and cannot walk without support.", "These spinning sensations are present throughout the day and do not change with position.", "They are associated with nausea and vomiting.", "He denies ear pain, ear discharge, ringing in the ear, hearing disturbances, ear fullness, head trauma, fever, or recent flu-like illness.", "He has a blood pressure of 133/80 mm Hg, heart rate of 80/min, respiratory rate of 12/min, and temperature of 36.7°C (98.2°F).", "His extraocular eye movements are normal in all directions, but a vertical nystagmus is present that does not disappear despite repetitive testing.", "Hearing tests are within normal limits."], "s1": [0, 1, 2, 3, 4, 7], "s2": [5, 6, 8]} {"key": 451, "questions": "What is the most likely cause of the following findings?", "options": [{"label": "A", "disease": "Membranoproliferative glomerulonephritis"}, {"label": "B", "disease": "Rapidly progressive glomerlonephritis"}, {"label": "C", "disease": "Postinfectious glomerulonephritis"}, {"label": "D", "disease": "Membranous glomerulonephritis"}], "answer_idx": "D", "symptoms": ["A 45-year-old man presents to the outpatient unit with a complaint of lower extremity edema for the past few weeks.", "He has also been observing puffiness of the face lately.", "The urinalysis shows 4+ proteinuria.", "The serum creatinine is 3.5 mg/dL, and antinuclear antibodies are absent.", "The biopsy findings are given in the picture."], "s1": [0, 1], "s2": [2, 3, 4]} {"key": 452, "questions": "What is the most likely explanation for this patient’s symptoms?", "options": [{"label": "A", "disease": "Prolapse of the bladder"}, {"label": "B", "disease": "Detrusor muscle overactivity"}, {"label": "C", "disease": "Genitourinary syndrome of menopause"}, {"label": "D", "disease": "Rectovaginal fistula"}], "answer_idx": "A", "symptoms": ["A 49-year-old G4P4 woman comes to the clinic complaining of repeated leakage of fluid from her vagina for the past 5 months.", "She noticed an increase in episodes following her cold last week when she was coughing and sneezing a lot.", "Her past medical history is significant for Crohn disease, which is well controlled with sulfasalazine.", "Her last menstrual period was 1 year ago.", "She is currently sexually active with multiple partners with inconsistent condom use.", "She denies any vaginal itching, abnormal discharge, pain, subpubic pressure, urinary urges, or odors.", "Physical examination is significant for a bulge at the anterior vaginal wall."], "s1": [0, 1, 5, 6], "s2": [2, 3, 4]} {"key": 453, "questions": "What is the most likely etiology of this patient’s diagnosis?", "options": [{"label": "A", "disease": "Behçet's syndrome"}, {"label": "B", "disease": "Protein C deficiency"}, {"label": "C", "disease": "Factor V Leiden mutation"}, {"label": "D", "disease": "Polycythemia vera"}], "answer_idx": "D", "symptoms": ["A 14-year-old boy is brought to the office by his parents because he states that for the past 2 months he has been feeling constantly tired, and also noticed a dull pain in the pit of his stomach.", "The patient has no relevant family history.", "The vital signs include a heart rate of 105/min, a respiratory rate of 16/min, a temperature of 37.0°C (98.6°F), and a blood pressure of 111/66 mm Hg.", "On physical exam, the abdomen is distended with hepatomegaly 5 cm underneath the xiphoid process.", "The complete blood count results are as follows: | Hemoglobin 17.6 g/dL | Hematocrit 64% | RBC 6.02 x 1012/L | Leukocyte count | 26,300/mm3 | Neutrophils 55% | Bands 2% | Eosinophils 1% | Basophils 0% | Lymphocytes 29% | Monocytes 2% | Platelet count 480,000/mm³ | Erythropoietin < 1.0 mU/mL | The coagulation test results are as follows: | Partial thromboplastin time (activated) 30.9 s | Prothrombin time 14.0 s | The abdominal Doppler ultrasound imaging is shown in the picture."], "s1": [0, 3], "s2": [1, 2, 4]} {"key": 454, "questions": "Which of the following is the most likely underlying cause of this patient's symptoms?", "options": [{"label": "A", "disease": "Bilateral renal cysts"}, {"label": "B", "disease": "Glomerular IgA deposits"}, {"label": "C", "disease": "Ureteropelvic junction obstruction"}, {"label": "D", "disease": "Ascending urinary tract infection\n\""}], "answer_idx": "C", "symptoms": ["A 25-year-old man comes to the emergency department because of left flank pain for 2 hours.", "The pain is colicky in nature and he describes it as 8 out of 10 in intensity.", "He has nausea and has vomited once.", "He had a similar episode 6 months ago for which he took naproxen.", "There is no personal or family history of serious illness.", "He is a second-year medical student and has been consuming more coffee and energy drinks than normal to stay awake and study for the past 2 days.", "He does not smoke or drink alcohol.", "He takes no medications.", "His temperature is 37.3°C (99.1°F), pulse is 98/min, and blood pressure is 124/78 mm Hg.", "The abdomen is soft and nontender.", "Examination of the back shows no costovertebral angle tenderness.", "The remainder of the examination shows no abnormalities.", "Urinalysis is unremarkable."], "s1": [0, 1, 2, 3, 5], "s2": [4, 6, 7, 8, 9, 10, 11, 12]} {"key": 455, "questions": "What is the most likely diagnosis?", "options": [{"label": "A", "disease": "Measles"}, {"label": "B", "disease": "Chickenpox"}, {"label": "C", "disease": "Erythema infectiosum"}, {"label": "D", "disease": "Roseola infantum"}], "answer_idx": "C", "symptoms": ["A 16-month-old boy is brought to the pediatrician after his parents noticed the appearance of a rash on his face, torso, and limbs.", "The boy has been ill for almost 2 weeks, initiating with fever, malaise, coryza, headache, nausea, diarrhea, and a rash on both of his cheeks.", "Physical examination is unremarkable except for an erythematous maculopapular rash on the face, trunk, and extremities with a reticular pattern (as shown in the photograph)."], "s1": [0], "s2": [1, 2]} {"key": 456, "questions": "Which of the following congenital defects is the most likely cause of these findings?", "options": [{"label": "A", "disease": "Atrial septal defect"}, {"label": "B", "disease": "Coarctation of the aorta"}, {"label": "C", "disease": "Patent ductus arteriosus"}, {"label": "D", "disease": "Tetralogy of Fallot"}], "answer_idx": "A", "symptoms": ["A 4-year-old girl presents to the pediatrician’s office for a physical examination prior to starting preschool.", "She was born at 39 weeks gestation via spontaneous vaginal delivery.", "She is up to date on all vaccines and is meeting all developmental milestones.", "Past medical history is noncontributory.", "She takes no medications but does take a chewable vitamin daily.", "She lives with her parents and older brother in a house.", "Today, her blood pressure is 110/65 mm Hg, heart rate is 90/min, respiratory rate is 22/min, and temperature of 37.0°C (98.6°F).", "On physical exam, she appears well developed and pleasant.", "She sits listening to the conversation and follows directions.", "Palpation of the heart reveals a mild parasternal heave.", "Auscultation reveals a normal S1 but the S2 is split and remains split during inhalation and exhalation.", "Additionally, there is a medium pitched midsystolic murmur that is loudest between ribs 2 and 3 on the left side and a very soft diastolic rumble."], "s1": [0, 1, 2, 3, 4, 5, 6, 7, 8], "s2": [9, 10, 11]} {"key": 457, "questions": "What is the most likely cause of this patient’s visual loss?", "options": [{"label": "A", "disease": "Age-related macular degeneration"}, {"label": "B", "disease": "Cataracts"}, {"label": "C", "disease": "Open-angle glaucoma"}, {"label": "D", "disease": "Refractive error"}], "answer_idx": "B", "symptoms": ["A 67-year-old woman presents to the ophthalmologist with complaints of worsening visual loss.", "She states that her vision is blurry.", "Driving has become difficult, particularly at night, as she experiences substantial glare and sees halos around lights.", "On physical examination, there is absence of a red reflex."], "s1": [0, 1], "s2": [2, 3]} {"key": 458, "questions": "What Is Your Diagnosis?", "options": [{"label": "A", "disease": "Parapharyngeal abscess"}, {"label": "B", "disease": "Retropharyngeal abscess"}, {"label": "C", "disease": "Emphysematous thyroiditis"}, {"label": "D", "disease": "Mediastinitis"}], "answer_idx": "C", "symptoms": ["A healthy patient in their teens presented to the emergency department with high-grade fever (38.7 °C), neck pain, and a foreign body sensation in the throat after eating chicken 3 days prior", "Laboratory investigations demonstrated an elevated leukocyte count (22 500/µL", "to convert to ×109/L, multiply by 0.001)", "Lateral soft tissue radiography showed a radio-opaque foreign body at the level of C6. Computed tomography (CT) scan confirmed a 2-cm radio-opaque osseous foreign body at the level of the cricopharyngeus muscle with signs of esophageal perforation, including air leak tracking toward the left lobe of the thyroid gland (Figure, A and B)", "The patient was brought to the operating room, and the esophageal foreign body was successfully removed", "Postoperatively, the patient continued to become febrile despite intravenous antibiotic therapy", "A repeated CT scan of the neck and chest was done (Figure, C)", "A, Axial cut of computed tomography (CT) scan of the neck demonstrating free air in the left lobe of the thyroid gland (arrowheads)", "B, Sagittal cut of neck CT scan showing 2-cm radiodense osseous foreign body in the upper cervical esophagus at C6 level (arrowhead)", "C, Coronal cut of CT scan showing gas in the thyroid gland (arrowheads)"], "s1": [0, 1, 5], "s2": [2, 3, 4, 6, 7, 8, 9]} {"key": 459, "questions": "What Is Your Diagnosis?", "options": [{"label": "A", "disease": "Herpes simplex virus"}, {"label": "B", "disease": "Histoplasmosis"}, {"label": "C", "disease": "Molluscum contagiosum"}, {"label": "D", "disease": "Mpox"}], "answer_idx": "D", "symptoms": ["A man in his 30s with AIDS presented with acute-onset painful scattered umbilicated papulopustules and ovoid ulcerated plaques with elevated, pink borders on the face, trunk, and extremities (Figure, A)", "The patient also had a new-onset cough but was afebrile and denied other systemic symptoms", "Due to his significant immunocompromise, the clinical presentation was highly suspicious for infection", "For rapid bedside differentiation of multiple infectious etiologies, a Tzanck smear was performed by scraping the base of an ulcerated lesion and inner aspect of a pseudopustule and scraping its base with a #15 blade", "These contents were placed on a glass slide, fixed, and stained with Wright-Giemsa and subsequently Papanicolaou staining to further characterize the changes seen", "A, Clinical image demonstrating papulopustules and ovoid ulcerated plaques with elevated, pink borders on the elbows", "B, Tzanck smear using Wright-Giemsa staining of specimen demonstrating ballooning of keratinocytes and peripheralization of nuclear material (original magnification ×20)"], "s1": [0, 1, 2, 5], "s2": [3, 4, 6]} {"key": 460, "questions": "What Is Your Diagnosis?", "options": [{"label": "A", "disease": "Primary leptomeningeal lymphoma"}, {"label": "B", "disease": "Tolosa-Hunt syndrome"}, {"label": "C", "disease": "Perineural spread of cutaneous malignancy"}, {"label": "D", "disease": "Sphenoid wing meningioma"}], "answer_idx": "C", "symptoms": ["A 68-year-old man presented with progressive right-sided facial numbness and weakness, vision changes, and headaches", "His symptoms began 5 years earlier with right temple numbness, which expanded to include the entire right face and mouth", "He then developed progressive right facial weakness with a droop, slurred speech, and eyelid ptosis", "Finally, binocular vertical and horizontal diplopia and periorbital headaches began", "Trials of gabapentin and steroids provided no relief", "His history was significant for a left cheek melanoma completely excised years prior, numerous cutaneous squamous and basal cell carcinomas of the bilateral head and neck removed without issue, type 2 diabetes, and hyperlipidemia", "His medications included metformin and simvastatin", "he had no history of smoking or drinking", "On examination, he had right-sided ptosis and severe right facial weakness in a lower motor neuron pattern with temporal wasting", "The right pupil was fixed and dilated", "There was no afferent pupillary defect", "The left pupil was appropriately sized and reactive", "His right-sided vision was impaired (20/250) with blurriness", "Vertical and horizontal binocular diplopia was present", "Right extraocular movements were significantly limited in all directions and painful, and right-sided facial sensation was diminished to pinprick in all cranial nerve (CN) V distributions", "His speech was slightly slurred", "Left-sided extraocular movements were intact", "His left facial sensation was intact to pinprick, and his strength was without deficit", "There were no other neurological abnormalities", "Initial magnetic resonance images (MRIs) were unremarkable", "A biopsy of CN VII was nondiagnostic", "Hemoglobin A1c was 6.8%", "Tests for infection and autoimmune disease had negative results", "Cerebrospinal fluid (CSF) studies showed mildly elevated protein values", "Figure 1 shows the MRI at presentation", "T1 turbo spin-echo fat-saturated magnetic resonance images of the brain through the skull base at presentation"], "s1": [0, 1, 2, 3, 8, 12, 13, 14, 15], "s2": [4, 5, 6, 7, 9, 10, 11, 16, 17, 18, 19, 20, 21, 22, 23, 24, 25]} {"key": 461, "questions": "What Is Your Diagnosis?", "options": [{"label": "A", "disease": "Kimura disease"}, {"label": "B", "disease": "Classic Hodgkin lymphoma"}, {"label": "C", "disease": "T-cell acute lymphoblastic lymphoma/leukemia"}, {"label": "D", "disease": "Myeloid/lymphoid neoplasms with eosinophilia and tyrosine kinase gene fusions"}], "answer_idx": "D", "symptoms": ["A 31-year-old man presented with left cervical and left inguinal masses", "He reported intermittent itching and night sweats for 2 years", "He denied fever, weight loss, shortness of breath, rashes, diarrhea, and neurological symptoms", "On a preemployment evaluation, the patient was told he had a high white blood cell count 2 years ago", "On examination, there was left cervical and inguinal lymphadenopathy and no other organomegaly", "Complete blood cell count and peripheral blood smear showed marked leukocytosis, with a white blood cell count of 22 340/μL, an absolute neutrophil count of 5360/μL, and 55% eosinophils with an absolute eosinophil count of 12 290/μL (to convert all to cells ×109/L, multiply by 0.001)", "Vitamin B12 was markedly elevated at more than 4000 pg/mL (to convert to pmol/L, multiply by 0.7378)", "The erythrocyte sedimentation rate was 5 mm/h", "Lactate dehydrogenase was 180 U/L, and alkaline phosphatase was 81 U/L (to convert both to μkat/L, multiply by 0.0167)", "Evaluations for HIV and hepatitis B and C were all negative", "Serum creatinine was 0.76 mg/dL (to convert to μmol/L, multiply by 88.4)", "alanine aminotransferase and aspartate aminotransferase were 11 U/L and 9.9 U/L, respectively (to convert to μkat/L, multiply by 0.0167)", "and total bilirubin was 0.35 mg/dL (to convert to μmol/L, multiply by 17.104)", "Bone marrow biopsy showed hypercellular marrow (cellularity of 100%), myeloid hyperplasia, increased eosinophils with some dysplasia, and a blast count of 2%", "Positron emission tomographic–computed tomographic scan showed a left upper cervical lymph node of 2.6 cm and a left inguinal lymph node of 3.1 × 2.3 cm with an standardized uptake value max of 5.7 (Figure, A)", "Left inguinal lymph node biopsy showed partial involvement by atypical cells with high proliferation index (Ki-67 >95%) that were positive for CD3, CD4, CD8, BCL2, and TDT, suggestive of T-cell lymphoblastic lymphoma/leukemia (Figure, B)", "A, Positron emission tomographic–computed tomographic (PET/CT) scan of the head and neck at presentation showing a left upper cervical lymph node of 2.6 cm (arrowhead)", "B, Lymph node biopsy immunohistochemical stain with terminal deoxynucleotidyl transferase", "The inset shows interphase fluorescence in situ hybridization for FIP1L1::PDGFRA rearrangement (positive)", "C, PET/CT 12 weeks after treatment initiation", "Myeloid/lymphoid neoplasms with eosinophilia and tyrosine kinase gene fusions"], "s1": [0, 1, 2, 4, 14, 15, 16, 17], "s2": [3, 5, 6, 7, 8, 9, 10, 11, 12, 13, 18, 19, 20]} {"key": 462, "questions": "What Is Your Diagnosis?", "options": [{"label": "A", "disease": "Lymphoma"}, {"label": "B", "disease": "Kikuchi-Fujimoto disease"}, {"label": "C", "disease": "Systemic lupus erythematosus"}, {"label": "D", "disease": "Rosai-Dorfman disease"}], "answer_idx": "B", "symptoms": ["A 28-year-old woman presented with a 5-day history of painful cervical lymphadenopathy along with fever, nausea, and vomiting", "Treatment with antibiotics and corticosteroids was started, without improvement of symptoms", "She denied recent travel, sick contacts, or autoimmune disease", "Her medical history was significant for α-thalassemia but otherwise noncontributory", "On examination, she had tender lymphadenopathy involving right level V", "A computed tomography scan of the neck revealed a collection of enlarged, matted nodes without evidence of an abscess", "Fine-needle aspiration (FNA) biopsy showed nondiagnostic reactive lymph nodes", "She was subsequently admitted to the hospital, where the lymphadenopathy progressed to the left side of the neck and a skin eruption developed over her face, back, and trunk", "Results of an infectious disease and rheumatological workup, including bone marrow biopsy, were negative", "Given her persistent symptoms, the ear, nose, and throat service consulted for excisional biopsy", "On pathologic examination, the lymph nodes showed extensive coagulative necrosis, apoptosis, and karyorrhectic debris (Figure 1)", "Cellular components included histiocytes and dendritic cells, with absent neutrophils and hematoxylin bodies", "Well-defined areas of fibrinoid necrosis in paracortical areas (hematoxylin-eosin, original magnification ×20)"], "s1": [0, 1, 2, 4, 7], "s2": [3, 5, 6, 8, 9, 10, 11, 12]} {"key": 463, "questions": "What Is Your Diagnosis?", "options": [{"label": "A", "disease": "Arteriovenous malformation of the scalp"}, {"label": "B", "disease": "Solitary fibrous tumor"}, {"label": "C", "disease": "Diffuse-type neurofibroma"}, {"label": "D", "disease": "Dermatofibrosarcoma protuberans"}], "answer_idx": "C", "symptoms": ["A 27-year-old woman presented to her primary care physician with primary complaints of headaches and visual-spatial difficulties when driving at night", "In addition, the patient described a progressively enlarging lesion on her left scalp, which she had initially noted in high school", "She denied any overlying loss of hair, sharp pain, or bleeding from the scalp lesion, and she only reported pain with prolonged pressure", "On her scalp examination, a 7-cm ill-defined spongy and pulsatile mass was noted with an overlying gray patch of hair", "Magnetic resonance imaging (MRI) of the brain/orbit was initially performed (Figure, A and B), which demonstrated an ill-defined, diffuse, intensely enhancing scalp lesion in the left frontoparietal region with very prominent vascularity and underlying calvarial thickening", "No osseous destruction or intracranial extension was present", "Subsequent catheter angiography (Figure, C) confirmed the hypervascular mass, supplied via hypertrophied superficial temporal and occipital arteries with marked internal vascularity without direct involvement of the calvarium or intracranial structures", "The patient underwent biopsy under general anesthesia", "the specimen showed an ill-defined deep dermal/subcutaneous proliferation of spindle cells without significant nuclear pleomorphism or mitotic activity", "The lesions stained positive for S100 and SOX10 immunostains", "Coronal T2-weighted (A) and postcontrast T1-weighted (B) fat-saturated images show the large left T2 hyperintense scalp lesion (double arrowheads", "A and B), with underlying calvarial thickening (asterisk", "A and B) without invasion, prominent vascularity (single arrowhead", "A), and enhancement (B)", "Note the preservation of the overlying skin", "Catheter angiography (C) shows the hypervascular scalp lesion being supplied by hypertrophied branches arising from the left superficial temporal and occipital arteries, with marked internal vascularity (arrowheads)", "R indicates right"], "s1": [0, 1, 2, 3, 7, 8, 9], "s2": [4, 5, 6, 10, 11, 12, 13, 14, 15, 16]} {"key": 464, "questions": "What Is Your Diagnosis?", "options": [{"label": "A", "disease": "Chromoblastomycosis"}, {"label": "B", "disease": "Hyalohyphomycosis"}, {"label": "C", "disease": "Blastomycosis"}, {"label": "D", "disease": "Phaeohyphomycosis"}], "answer_idx": "D", "symptoms": ["A woman in her 30s presented for evaluation of asymptomatic erythematous scaly plaques over the face and proximal extremities", "The lesions started as an erythematous papule on the face, which had progressed to larger plaques within 10 years", "There was involvement of the upper respiratory tract, causing palatal perforation and stridor, necessitating a tracheostomy a year ago", "In addition, there was history of wheezing and breathlessness", "There was no history of preceding trauma, fever, malaise, joint pain, cough, epistaxis, hemoptysis, or hematuria", "Examination revealed multiple well-defined brown erythematous scaly plaques with central scarring measuring 2 × 1 to 8 × 7 cm on the face, arm, and thighs", "The patient also had saddle nose deformity, perforation of the hard palate, and necrotic plaques on the ear, causing destruction of ear cartilage (Figure, A and B)", "There was cervical lymphadenopathy: multiple, nontender, discrete nodes, ranging from 1.5 to 2 cm without any surface changes", "The results of sensory examination and peripheral nerve examination were within normal limits", "Respiratory examination and chest radiography results were normal", "Hemogram showed anemia (hemoglobin, 10.1 g/dL [to convert to g/L, multiply by 10.0])", "the results of a kidney function test, a liver function test, a fasting blood glucose test, and routine urinalysis were within normal limits", "Serologic analysis results for HIV-1/2, antinuclear antibody, and antineutrophil cytoplasmic antibodies were negative", "Punch skin biopsy was obtained for histopathologic analysis and culture (Figure, C)", "A, Multiple well-defined brown erythematous scaly plaques with central scarring, saddle nose deformity, and necrotic lesions on the ear with loss of ear cartilage", "B, Perforation of the hard palate", "C, Infiltrate of foamy histiocytes and pigmented yeasts without multiaxial septation (Fontana-Masson)"], "s1": [0, 1, 5, 6, 7, 14, 15, 16], "s2": [2, 3, 4, 8, 9, 10, 11, 12, 13]} {"key": 465, "questions": "What Is Your Diagnosis?", "options": [{"label": "A", "disease": "Langerhans cell histiocytosis"}, {"label": "B", "disease": "Meningioma"}, {"label": "C", "disease": "Acute coalescent mastoiditis with sigmoid sinus thrombosis"}, {"label": "D", "disease": "Cholesteatoma"}], "answer_idx": "B", "symptoms": ["A 64-year-old man with a 2-year history of seizure disorder presented to the emergency department (ED) at an outside hospital with breakthrough seizures", "One month prior to his ED visit, he had been treated with oral antibiotics when he presented to an urgent care center with left-sided otorrhea, otalgia, and hearing loss", "In the ED, he denied headaches, fever, otalgia, otorrhea, or hearing loss", "Magnetic resonance imaging of the brain with contrast showed diffuse enhancement and possible coalescence in the left mastoid process with adjacent dural enhancement and sigmoid sinus occlusion", "He received intravenous vancomycin and cefepime and was started on a heparin drip before being transferred to our facility", "On presentation at our facility, the patient appeared well with normal vital signs", "There was no periauricular swelling or erythema", "Otoscopic examination showed no inflammation of the external auditory canals, and the tympanic membranes were translucent without evidence of retraction or effusion", "Computed tomographic venogram confirmed occlusion of the left transverse and sigmoid sinuses, and computed tomography of the temporal bone showed coalescence of mastoid air cells with erosion into the sigmoid sinus", "Mastoidectomy was performed for biopsy and culture specimens", "Histologic findings from the specimens demonstrated cells with oval nuclei and abundant cytoplasm arranged in syncytial clusters with occasional whorls, pseudoinclusions, and psammomatous calcifications (Figure, A)", "Immunohistochemical stains were positive for epithelial membrane antigen (EMA) and vimentin (Figure, B)", "Hematoxylin-eosin stain (A) and immunohistochemical stain (B) after mastoidectomy for biopsy and culture specimens"], "s1": [0, 1, 2, 4, 5, 6, 7], "s2": [3, 8, 9, 10, 11, 12]} {"key": 466, "questions": "What Is Your Diagnosis?", "options": [{"label": "A", "disease": "Leishmaniasis"}, {"label": "B", "disease": "Tuberculosis"}, {"label": "C", "disease": "Leprosy"}, {"label": "D", "disease": "Lethal midline granuloma"}], "answer_idx": "A", "symptoms": ["A 62-year-old healthy man with no recent travel history or sick contacts presented with 2 weeks of midline sore throat without systemic signs, such as fever, myalgia, or rash", "A physical examination, including flexible laryngoscopy, revealed cobbling, adherent, thick yellow mucus and ulceration of the uvula (Figure 1A)", "A, Nasopharyngoscopic imaging revealed infected nasal surface of uvula", "B, Physical examination showed recurrent skin lesions at incision site and in the surrounding skin", "Empirical treatment with augmentin and nystatin oral suspension for possible bacterial or fungal pharyngitis was unsuccessful, with symptoms worsening during the subsequent 2 weeks", "Results of laboratory testing for group A Streptococcus, COVID-19, and mononucleosis were negative", "A tissue biopsy was performed, and results of pathology revealed epithelial hyperplasia with suppurative granulomatous inflammation without evidence of dysplasia or malignancy", "There was no evidence of fungal or acid-fast organisms", "Tissue culture revealed only normal respiratory flora", "Additional treatment with fluconazole, augmentin, doxycycline, omeprazole, prednisone, and bactrim failed to improve symptoms", "Further exploration of the patient’s history revealed that 5 years earlier, he had traveled to Guyana in South America", "After that trip, he sought medical care for multiple neck subcutaneous nodules as well as for a 4-cm focus of neck cellulitis", "After failed treatment with empirical antibiotics, the largest lesion was excised", "Findings of pathologic testing revealed dense granulomatous inflammation", "However, results of stains for bacteria, fungus, and acid-fast bacillus were negative (Figure 1B)", "The remaining cutaneous and subcutaneous nodules had resolved spontaneously"], "s1": [0, 1, 2, 4, 5, 6, 7, 8, 9], "s2": [3, 10, 11, 12, 13, 14, 15]} {"key": 467, "questions": "What Is Your Diagnosis?", "options": [{"label": "A", "disease": "Laugier-Hunziker syndrome"}, {"label": "B", "disease": "Melanoma"}, {"label": "C", "disease": "Medication adverse effect"}, {"label": "D", "disease": "Oral involvement of mycosis fungoides"}], "answer_idx": "C", "symptoms": ["A 53-year-old woman with a history of stage IVA1 (T4N1M0B2) mycosis fungoides presented with a new 1-month history of hyperpigmentation of the oral mucosa (Figure 1)", "Examination of the mouth revealed multiple coalescing painless nonpruritic black macules and patches on the tongue, roof of the mouth, and buccal mucosa", "Examination of the skin was notable for erythematous and hyperpigmented patches covering 90% of the body surface area, consistent with her known mycosis fungoides", "Other notable findings on examination were 1- to 2-cm lymphadenopathy in the bilateral inguinal folds and axillae", "Review of systems was notable for fatigue", "The patient had previously received 5 cycles of romidepsin with progression of disease, followed by 4 doses of pegylated liposomal doxorubicin hydrochloride, which was followed by partial response", "Black macules and patches on the tongue, roof of the mouth, and buccal mucosa"], "s1": [0, 1, 2, 3], "s2": [4, 5, 6]} {"key": 468, "questions": "What Is Your Diagnosis?", "options": [{"label": "A", "disease": "Pretibial myxedema"}, {"label": "B", "disease": "Elephantiasis nostras verrucosa"}, {"label": "C", "disease": "Lobomycosis"}, {"label": "D", "disease": "Euthyroid pretibial mucinosis"}], "answer_idx": "D", "symptoms": ["A woman in her 60s with long-standing thickening and induration of the legs presented with 1 week of left leg pain", "The patient had previously been diagnosed with elephantiasis nostras verrucosa, and she was advised to elevate and compress the leg but was unavailable for follow-up", "Her medical history was notable for morbid obesity, hypertension, diabetes, and stroke", "She reported no fever, fatigue, weight changes, gastrointestinal symptoms, difficulty concentrating, anxiety, or hyperhidrosis", "Physical examination results revealed multiple firm skin-colored papules and nodules coalescing to form a large plaque on the anterior aspect of the left lower leg, whereas the right lower leg had diffuse induration and hyperpigmentation (Figure 1)", "There was no palpable lymphadenopathy", "Laboratory analysis revealed normal complete blood cell count, thyroid function, serum protein electrophoresis, and serum immunofixation", "A biopsy of a left leg nodule was performed", "Multiple firm skin-colored papules and nodules coalesce to form a large plaque on the anterior aspect of the left lower leg"], "s1": [0, 1, 2, 3], "s2": [4, 5, 6, 7, 8]} {"key": 469, "questions": "What Is Your Diagnosis?", "options": [{"label": "A", "disease": "Blue rubber bleb nevus syndrome"}, {"label": "B", "disease": "Familial cerebral cavernous malformations"}, {"label": "C", "disease": "Glomuvenous malformations"}, {"label": "D", "disease": "Verrucous venous malformations"}], "answer_idx": "B", "symptoms": ["A female patient in her early 50s presented with multiple (approximately 10) bluish, partially keratotic papules and nodules on the lower legs (Figure, A)", "Since the age of 24 years, the lesions had been growing very slowly in number and size and had been bleeding occasionally after a trauma", "The patient used a wheelchair due to a right-sided hemiparesis and hemianopsia caused by an intracranial hemorrhage", "At the age of 4 years, recurrent seizures had led to the diagnosis of cerebral venous malformations", "Clinical examination also demonstrated a lipolymphedema of the lower legs that had been attributed to a functional venous insufficiency with insufficient venous pump activity", "The patient had been wearing compression stockings for years", "The family history was positive for cerebral venous malformations involving the patient’s father and aunt, the father having died of an intracranial hemorrhage in his early 70s", "A, Clinical picture", "B and C, Hematoxylin-eosin stain", "Sonography of the skin lesions demonstrated intradermal slow-flow vascular malformations with a maximum diameter of 1.2 cm", "A biopsy of a lower-extremity papule was performed (Figure, B and C)", "Mutational analysis resulted in the detection of a heterozygous pathogenic KRIT1 variant"], "s1": [0, 1, 9, 10], "s2": [2, 3, 4, 5, 6, 7, 8, 11]} {"key": 470, "questions": "What Is Your Diagnosis?", "options": [{"label": "A", "disease": "Pretibial pruritic papular dermatitis (PPPD)"}, {"label": "B", "disease": "Hypertrophic lichen planus (HLP)"}, {"label": "C", "disease": "Epidermolysis bullosa pruriginosa (EBP)"}, {"label": "D", "disease": "Lichen amyloidosis (LA)"}], "answer_idx": "C", "symptoms": ["A woman in her 30s presented with progressive, multiple itchy erythematous to violaceous papules and plaques localized to the left shin for more than 10 years (Figure, A and B)", "On close inspection, milia and a few erosions with partially detached epidermis were identified", "There were no abnormalities of the mucous membranes, nails, hair, or teeth", "No extracutaneous involvement was observed", "A, Multiple pruritic erythematous to violaceous lichenified papules and plaques on the left shin", "B, Close-up of the skin lesions showing an erosion with partially detached epidermis and multiple milia", "C, Histopathologic examination revealed dermoepidermal separation (hematoxylin-eosin stain)", "D, Histopathologic examination revealed several milia (hematoxylin-eosin stain)"], "s1": [0, 4, 1, 5], "s2": [2, 3, 6, 7]} {"key": 471, "questions": "What Is Your Diagnosis?", "options": [{"label": "A", "disease": "Hemangioma"}, {"label": "B", "disease": "Rhabdomyoma"}, {"label": "C", "disease": "Rhabdomyosarcoma"}, {"label": "D", "disease": "Teratoma"}], "answer_idx": "B", "symptoms": ["An otherwise healthy 13-month-old male was scheduled to undergo excisional biopsy of a left anterior tongue submucosal intramuscular mass (Figure 1)", "This firm-to-palpation mass was noted at birth and remained proportional in size with the child’s somatic growth, measuring approximately 1 cm in diameter on presentation", "Preoperative magnetic resonance imaging (MRI) documented the mass to be uniformly solid and well circumscribed without cystic components or flow voids, hypointense on T2, and hyperintense with homogeneous contrast enhancement on T1. His head, neck, and general examination findings were otherwise normal with no cervical lymphadenopathy or additional masses"], "s1": [0], "s2": [1, 2]} {"key": 472, "questions": "What Is Your Diagnosis?", "options": [{"label": "A", "disease": "Disseminated fusariosis"}, {"label": "B", "disease": "Invasive aspergillosis"}, {"label": "C", "disease": "Pseudomonal ecthyma gangrenosum"}, {"label": "D", "disease": "Sweet syndrome"}], "answer_idx": "A", "symptoms": ["A 56-year-old woman with a history of multiple myeloma complicated by therapy-related acute myeloid leukemia was admitted for myeloablative conditioning in preparation for allogeneic hematopoietic cell transplant", "Her course was complicated by severe mucositis, acute kidney injury, and neutropenic fever", "During her admission, the patient also developed painful necrotic skin lesions, a nonproductive cough, and altered mental status", "Physical examination findings demonstrated dusky, purpuric, and pink papulonodules, some with overlying eschar, scattered on the scalp, neck, trunk, and extremities (Figure, A)", "Laboratory study results revealed a white blood cell count of 150/μL (reference range, 3500/μL to 10 500/μL), an absolute neutrophil count of 130/μL (reference range, 1500/μL to 7400/μL), and a platelet count of 19 ×103/μL (reference range, 150 ×103/μL to 400 ×103/μL)", "(To convert the white blood cell and neutrophil counts to cells ×109/L, multiply by 0.001", "conversion of the platelet count to cells ×109/L is 1:1.) Blood culture results were negative", "The results of a computed tomography (CT) scan of the brain showed no acute intracranial hemorrhage, territorial infarction, or mass effect", "The results of a CT scan of the chest demonstrated multifocal bilateral ground-glass patchy opacities, scattered solid bilateral pulmonary nodules, and a moderately sized pericardial effusion", "A biopsy specimen from a skin nodule on the left lateral thigh was obtained for histopathologic examination and tissue culture, and wet mount preparation of a tissue culture colony was performed (Figure, B)", "A, Physical examination findings demonstrate dusky and pink papulonodules, some with overlying eschar, scattered on the scalp, neck, and extremities", "B, Lactophenol cotton blue staining of a tissue culture colony reveals septated hyphae and oval microconidia"], "s1": [0, 1, 2, 4, 5, 6, 7, 8], "s2": [3, 9, 10, 11]} {"key": 473, "questions": "What Is Your Diagnosis?", "options": [{"label": "A", "disease": "Radiotherapy-induced epithelioid angiosarcoma"}, {"label": "B", "disease": "Kaposi sarcoma"}, {"label": "C", "disease": "Acquired angiokeratomas"}, {"label": "D", "disease": "Carcinoma hemorrhagiectoides"}], "answer_idx": "D", "symptoms": ["A man in his 80s presented to the dermatology clinic with a 3-month history of a violaceous plaque with blackish papules and nodules on his left cheek, neck, and chest", "He reported that the lesion had been asymptomatic but was increasing in size, with progression from his face to anterior chest", "He received a diagnosis of primary salivary duct carcinoma of the hard palate 1 year prior and had been receiving treatment with oral bicalutamide and radiotherapy", "A partially regressive change of the palatal tumor was noted radiographically during regular follow-up 3 months previously", "On clinical examination, a hemorrhagic and erythematous plaque with multiple purpuric-to-blackish infiltrative papules and nodules extending from cheek, lateral neck, to the interclavicular area was found (Figure 1)", "An incisional biopsy from the infiltrative nodule on cheek was performed and submitted for histopathologic analysis", "Hemorrhagic and erythematous plaque with multiple infiltrative purpuric-to-blackish papules and nodules with varying sizes extending from left cheek, neck, to chest"], "s1": [2, 3], "s2": [0, 1, 4, 5, 6]} {"key": 474, "questions": "What Is Your Diagnosis?", "options": [{"label": "A", "disease": "Matrical carcinoma"}, {"label": "B", "disease": "Panfolliculoma"}, {"label": "C", "disease": "Bullous pilomatricoma"}, {"label": "D", "disease": "Trichodiscoma"}], "answer_idx": "C", "symptoms": ["A previously healthy 16-year-old girl presented to our department with a red tumor on her right upper arm that enlarged over 6 months", "The lesion initially started as a small cutaneous nodule without obvious triggers that gradually developed and enlarged to form a red-colored tumor with a hard nodule inside", "There was no history of local trauma or insect bite", "Her personal, past, and family histories were unremarkable", "On physical examination, there was a 6 × 6-cm protuberant, thick-walled, and well-defined red bullalike tumor on the right upper arm, which extended 1 to 3 cm from the epidermal surface (Figure, A)", "Inside the tumor, a nontender, firm-to-hard nodule was palpated", "There was no regional lymphadenopathy, and results of the rest of the physical and systemic examinations were normal", "Laboratory examinations of hematologic, biochemical, and urinalysis tests were normal", "The tumor was excised completely under local anesthesia, and part of the tissue was sent for pathological examination", "A, A tumor on the patient’s right shoulder", "B, The excisional tumor showed multilocular spaces below the epidermis and a solid tumor present in the deep dermis with a chalky-white cut surface and lobulated appearance", "C and D, Histological examination revealed basophilic cells and shadow cells as well as dilated lymphatic vessels (hematoxylin-eosin", "original magnification ×100)"], "s1": [0, 1, 2, 3, 4, 5, 6, 7], "s2": [8, 9, 10, 11, 12]} {"key": 475, "questions": "What Is Your Diagnosis?", "options": [{"label": "A", "disease": "Perineurioma"}, {"label": "B", "disease": "Fibroblastic lipoblastoma"}, {"label": "C", "disease": "Pediatric fibromyxoid soft tissue tumor"}, {"label": "D", "disease": "Schwannoma"}], "answer_idx": "C", "symptoms": ["A 15-year-old male presented to the pediatric otolaryngology clinic with dysphagia to solid foods, and dyspnea when lying on the right side that was progressive during 8 months", "He denied pain, aspiration, noisy breathing, obstructive episodes, dysphonia, and weight loss", "A physical examination revealed fullness in the left oropharynx and left level 3 of the neck", "Findings of flexible laryngoscopy showed a submucosal mass in the left oropharynx extending inferiorly into the hypopharynx, obliterating the left pyriform sinus, abutting the base of tongue with hooding over the epiglottis", "The supraglottis and glottis were unremarkable", "Computed tomography imaging with contrast of the neck revealed a 4.6 × 4.0 × 8.0-cm heterogeneously enhancing mass in the left oropharynx and hypopharynx with cystic components and clear planes separating the mass from the spinal musculature and carotid sheath (Figure 1)", "Computed tomography imaging with contrast of the neck with the coronal cut demonstrating the well-circumscribed heterogeneous mass involving the retropharyngeal space, abutting the great vessels, and extending from superiorly in the oropharynx to inferiorly in the hypopharynx"], "s1": [0, 1], "s2": [2, 3, 4, 5, 6]} {"key": 476, "questions": "What Is Your Diagnosis?", "options": [{"label": "A", "disease": "Squamous cell carcinoma"}, {"label": "B", "disease": "Basal cell carcinoma"}, {"label": "C", "disease": "Melanoma"}, {"label": "D", "disease": "Merkel cell carcinoma"}], "answer_idx": "C", "symptoms": ["A 46-year-old man presented with a left shoulder mass", "It began with a quarter-size maculopapular lesion and continued to progress during the next 3 years", "He reported limited shoulder movements and denied other symptoms", "There was no medical or surgical history, and he had a negative personal or familial oncologic history", "The patient was a construction worker and used tobacco daily", "On physical examination, there was a 24 × 15-cm fungating, necrotic, and ulcerated left shoulder mass (Figure 1A)", "A punch biopsy showed sheets of highly pleomorphic atypical spindle and epithelial cells within the superficial and deep dermis, extending into the subcutaneous tissue, and with 18 mitoses/mm2. Immunohistochemistry demonstrated SOX10-positive and pankeratin-negative tumor cells", "A whole-body positron emission tomographic and computed tomographic scan revealed an intensely hypermetabolic left shoulder mass and multiple hypermetabolic enlarged left axillary lymph nodes, with no distant metastases (Figure 1B)", "Brain magnetic resonance imaging was negative for intracranial metastasis", "A, Left shoulder giant fungating mass in a 46-year-old man", "B, Positron emission tomographic and computed tomographic (PET-CT) scan shows the hypermetabolic left shoulder mass and no distant metastases"], "s1": [0, 1, 2, 3, 4], "s2": [5, 6, 7, 8, 9, 10]} {"key": 477, "questions": "What Is Your Diagnosis?", "options": [{"label": "A", "disease": "Drug-induced SCLE"}, {"label": "B", "disease": "VEXAS syndrome"}, {"label": "C", "disease": "Histiocytoid Sweet syndrome"}, {"label": "D", "disease": "Halogenoderma"}], "answer_idx": "B", "symptoms": ["A man in his 60s presented to our clinic with a 5-year history of diffuse erythematous, edematous annular plaques (Figure 1)", "low-grade fevers", "and mild leukopenia", "The result of a skin biopsy performed 4 years prior was interpreted as subacute cutaneous lupus erythematosus (SCLE)", "Treatments included hydroxychloroquine, 400 mg/d, for 4 years", "mycophenolate mofetil, 1000 mg/d, for 6 months", "and oral prednisone tapers, starting at 40 mg/d", "Of these, only prednisone appeared to elicit a response", "Review of symptoms was negative for fatigue, weight loss, cough, shortness of breath, joint pain, ocular symptoms, mucosal ulcerations, history of blood clots, photosensitivity, pleurisy, urine changes, and diarrhea", "Medical history included hypertension, for which he took losartan, and seasonal allergies, for which he used fexofenadine and fluticasone", "Laboratory evaluation revealed mild neutropenia and lymphopenia with normal hemoglobin and platelet counts", "Antinuclear antibody screening results were negative", "The results of a comprehensive metabolic panel, C3, C4, serum protein electrophoresis, serum free light chains, methylmalonic acid, homocysteine, ferritin, copper, zinc, haptoglobin, HIV, hepatitis B and C serologic tests, and rapid plasma reagin were negative", "Lactate dehydrogenase was mildly elevated", "Erythrocyte sedimentation rate was elevated at 41 mm/h", "Routine cancer screening was up to date", "Therapeutic trials of dapsone and dapsone in combination with colchicine were not effective"], "s1": [0, 1, 2, 3, 8, 10, 13, 14], "s2": [4, 5, 6, 7, 9, 11, 12, 15, 16]} {"key": 478, "questions": "What Is Your Diagnosis?", "options": [{"label": "A", "disease": "Pseudoxanthoma elasticum"}, {"label": "B", "disease": "Inherited cutis laxa syndrome"}, {"label": "C", "disease": "Pseudoxanthoma elasticum–like disorder with coagulation deficiency"}, {"label": "D", "disease": "Pseudoxanthoma elasticum/pseudoxanthoma elasticum–like overlap syndrome"}], "answer_idx": "C", "symptoms": ["A Chinese woman in her late 20s presented with a 20-year history of progressive skin laxity", "In early childhood, she had developed asymptomatic yellowish coalesced papules confined to flexural areas", "Subsequently, severe skin sagging occurred on her abdomen with loss of elasticity, then spread extensively with developmental growth", "The results of a physical examination showed thick and leathery skinfolds on the neck, axillae, inguinal regions, abdomen, and limbs (Figure, A), with yellowish papules scattered on the dorsal neck", "The patient had no extracutaneous involvement and was born to healthy nonconsanguineous parents", "However, her younger sister had developed the same skin symptoms", "Coagulation-related tests indicated a low clotting activity of factor X (52.2%", "reference range, 77%-131%) and a prolonged prothrombin time (12.8 seconds", "reference range, 10.4-12.6 seconds)", "The results of other laboratory investigations were unremarkable", "Echocardiographic and fundoscopic examination results were normal", "A skin biopsy specimen was obtained from her neck for histopathologic examination (Figure, B)", "A, Clinical image of the neck, axillae, and abdomen", "Severe skin laxity with deep cutaneous folds on the cervical, axillary, and abdominal regions", "B, Skin biopsy specimen taken from the neck shows excessive calcification of elastic fiber fragments scattered in the reticular dermis (von Kossa)"], "s1": [0, 1, 2, 3, 4, 5, 11, 12, 13, 14], "s2": [6, 7, 8, 9, 10]} {"key": 479, "questions": "What Is Your Diagnosis?", "options": [{"label": "A", "disease": "Schwannoma"}, {"label": "B", "disease": "Venous malformation"}, {"label": "C", "disease": "Pleomorphic adenoma"}, {"label": "D", "disease": "Paraganglioma"}], "answer_idx": "B", "symptoms": ["A 71-year-old woman presented to the otolaryngology clinic for evaluation of a parapharyngeal space (PPS) mass that was incidentally found on magnetic resonance imaging (MRI) of the face", "The patient experienced left jaw tightness, aural fullness, and facial discomfort", "The MRI results demonstrated a well-circumscribed, ovoid mass in the left prestyloid PPS that measured 12 × 15 × 22 mm", "The lesion was isointense on T1-weighted imaging, was hyperintense on T2-weighted imaging, and demonstrated heterogeneous enhancement following gadolinium administration (Figure 1)", "A computed tomography–guided fine-needle aspiration was performed, which demonstrated blood, fibrin, and rare mesenchymal cells", "Given her ongoing symptoms, the patient requested surgical resection", "The patient underwent transoral robotic surgery for resection of her PPS mass to achieve definitive diagnosis and treatment", "Histopathologic evaluation demonstrated a well-circumscribed, 12-mm mass", "Magnetic resonance imaging of the face", "A, Coronal section from a T2-weighted image demonstrates a hyperintense mass in the left parapharyngeal space (PPS)", "B, Axial section from a postcontrast T1-weighted image shows a heterogeneously enhancing mass in the prestyloid PPS"], "s1": [0, 1, 4, 5, 6, 7], "s2": [2, 3, 8, 9, 10]} {"key": 480, "questions": "What Is Your Diagnosis?", "options": [{"label": "A", "disease": "Maxillary carcinoma with perineural spread"}, {"label": "B", "disease": "Schwannoma"}, {"label": "C", "disease": "Invasive fungal sinusitis (mucormycosis) with perineural spread"}, {"label": "D", "disease": "Metastases from ovarian primary"}], "answer_idx": "C", "symptoms": ["A 47-year-old woman was seen with left-sided facial weakness and swelling for 2 months", "She was diagnosed as having recurrent high-grade serous cystadenocarcinoma of the ovary and developed these symptoms after cycle 4 of chemotherapy", "She was receiving oral antidiabetic agents, and her blood glucose level was 110 mg/dL (to convert to millimoles per liter, multiply by 0.0555)", "Physical examination findings were consistent with left lower-motor-neuron type of facial nerve palsy", "She was conscious and well oriented, with no other relevant clinical signs or focal neurological deficits", "There was no history of prior COVID-19 infection", "Magnetic resonance imaging (MRI) of the brain was requested, with clinical suspicion of metastasis in view of mildly elevated CA125 levels", "The results of MRI of the brain did not reveal any neuroparenchymal or meningeal lesions", "However, both maxillary sinuses showed mucosal thickening, which was intermediate to hypointense on T2-weighted imaging and hypointense on T1-weighted imaging with postcontrast enhancement", "Computed tomography sections confirmed bony destruction with foci of remodeling involving the walls of the maxillary sinuses (Figure 1)", "A, Axial T2-weighted magnetic resonance imaging (MRI) shows T2 heterogeneous soft tissue in the maxillary sinus with perimaxillary inflammatory changes (yellow arrowhead)", "B, In this axial contrast-enhanced, T1-weighted (T1 + C) image, linear enhancement corresponds to expected course of buccal branch of facial nerve (VII) (white arrow)", "The enhancement extends medially and involves the mandibular branch of the trigeminal nerve (V3)", "P indicates parotid gland, and the yellow dashed line outlines the edge of the parotid gland", "B and C, The white arrow shows the intercommunication between the auriculotemporal branch of V3 and the intraparotid facial nerve (green marker)", "C, Additionally, involvement of enhanced thickening is seen up to the left foramen ovale, without involvement of the trigeminal ganglion", "D, In oblique sagittal images, the thick enhancing branches of the facial nerves within the left parotid gland are seen (orange arrows)", "The enhancement of facial nerve extends up to the stylomastoid foramen (SMF) (blue arrow), and there is subtle enhancement along its mastoid segments (MSs)", "Iodinated contrast medium was used", "Fat stranding and enhancing soft tissue was seen in the retroantral spaces bilaterally as well as the pterygopalatine fossae", "Anteriorly, enhancing soft tissue was present in premaxillary space, from which contiguous linear enhancement was seen extending to left parotid gland—along the expected course of buccal branch of left facial nerve", "The entire extracranial left facial nerve proximal to this showed diffuse enhancement and thickening", "Associated stylomastoid foramen widening was present with subtle enhancement along its mastoid and tympanic segments", "These features were compatible with perineural extension of disease (Figure 1A and B)", "Additionally, there was involvement of the mandibular branch of the trigeminal nerve via intercommunication between the auriculotemporal branch of trigeminal nerve and the intraparotid facial nerve", "Diffuse enhancement and thickening was seen up to the left foramen ovale, without involvement of the trigeminal ganglion (Figure 1C and D)"], "s1": [2, 5, 6, 7, 18], "s2": [0, 1, 3, 4, 8, 9, 10, 11, 12, 13, 14, 15, 16, 17, 19, 20, 21, 22, 23, 24, 25]} {"key": 481, "questions": "What Is Your Diagnosis?", "options": [{"label": "A", "disease": "Fibroma"}, {"label": "B", "disease": "Granular cell tumor"}, {"label": "C", "disease": "Median rhomboid glossitis (nodular variant)"}, {"label": "D", "disease": "Localized amyloidosis (amyloidoma)"}], "answer_idx": "D", "symptoms": ["A 46-year-old man presented for an evaluation of a lesion on the dorsal tongue", "The patient was asymptomatic and unaware of the lesion prior to it being discovered by his dentist", "He underwent incisional biopsy at another institution and presented for a second opinion on the diagnosis", "Overall, he was in good health with no underlying systemic diseases", "Results of the most recent serologic analysis and urinalysis were also reportedly within normal limits", "The extraoral examination was unremarkable, with no skin lesions, asymmetry, redness, swelling, or lymphadenopathy observed", "The intraoral examination revealed an erythematous denuded area of the midline posterior dorsal tongue, with a central nodular component (Figure 1A)", "The nodule was firm to palpation and nontender", "The remaining oral soft tissue was unremarkable", "Because a biopsy specimen was already obtained, blank slides were requested from the aforementioned institution, and staining was performed in house", "A, Clinical examination revealed a pink nodule in the midline dorsal tongue with surrounding atrophic papillae", "B, Hematoxylin-eosin stain of the biopsy specimen showed acellular, amorphous, and eosinophilic material arranged in lobules", "Histologic evaluation revealed a piece of soft tissue with normal overlying epithelium", "In the connective tissue, acellular, amorphous, and eosinophilic material arranged in lobules was noted (Figure 1B)"], "s1": [0, 1, 3, 4, 5, 6, 7, 8, 10], "s2": [2, 9, 11, 12, 13]} {"key": 482, "questions": "What Is Your Diagnosis?", "options": [{"label": "A", "disease": "Metastatic renal cell carcinoma"}, {"label": "B", "disease": "Blastic plasmacytoid dendritic cell neoplasm"}, {"label": "C", "disease": "Epithelioid angiosarcoma"}, {"label": "D", "disease": "Myeloid leukemia cutis"}], "answer_idx": "B", "symptoms": ["A man in his 80s presented to the emergency department with a 4-week history of progressive weakness and fatigue, with associated development of purple bruiselike lesions on his head", "He denied any history of trauma, falls, or occlusive headgear use", "His medical history was significant for atrial fibrillation, receiving anticoagulation medication, and heart failure", "On physical examination, numerous nontender violaceous plaques were seen on the forehead and scalp of the patient (Figure, A)", "On full skin examination, nonspecific skin-colored plaques were noted on the chest, and a 1.8 × 2.5-cm erythematous plaque was noted on the left lower back", "There was no appreciable lymphadenopathy or hepatosplenomegaly", "Peripheral blood test results revealed a hemoglobin level of 9.1 g/dL, a platelet count of 68 × 103/μL, a white blood cell count of 2800/μL, and an absolute neutrophil count of 1.0/μL", "Lactate dehydrogenase level was 202 U/L", "(To convert hemoglobin to g/L, multiply by 10.0", "platelets to ×109/L, by 1", "white blood cell and neutrophil counts to ×109/L, by 0.001", "and lactate dehydrogenase to μkat/L, by 0.0167.) A 4-mm punch biopsy was performed on the erythematous plaque on the left lower back (Figure, B and C)", "A, Violaceous plaques on the scalp", "B, Punch biopsy showing a grenz zone and infiltrate extending into deeper dermis (hematoxylin-eosin)", "C, Punch biopsy showing cellular infiltrate of basophilic blastoid cells (hematoxylin-eosin)", "D, Punch biopsy showing cellular infiltrate with CD123 staining"], "s1": [0, 1, 2, 3, 4, 5, 6, 7], "s2": [8, 9, 10, 11, 12, 13, 14, 15]} {"key": 483, "questions": "What Is Your Diagnosis?", "options": [{"label": "A", "disease": "Cutaneous involvement of multiple myeloma"}, {"label": "B", "disease": "Erythema multiforme"}, {"label": "C", "disease": "Histiocytoid Sweet syndrome"}, {"label": "D", "disease": "Cutaneous cryptococcosis"}], "answer_idx": "C", "symptoms": ["A man in his late 60s was referred to the dermatologic clinic with a 10-day history of fever up to 38.5 °C and tender lesions on the neck and trunk", "In the previous month, he had received the first cycle of a chemotherapeutic regimen of ixazomib, lenalidomide, and dexamethasone to treat a recurrence of IgG-κ multiple myeloma with a 10-year history", "Physical examination showed annular erythematous plaques of up to 2 cm in diameter, with violaceous and crusted centers on the neck and trunk", "On the right chest, an annular erythematous lesion with 2 zones of color—a central area of purpura and an outer erythematous and edematous ring—was present (Figure, A)", "Laboratory investigations found pancytopenia: a white blood cell count of 2.3 × 103/μL (reference range, 3.5-9.8 × 103/μL), hemoglobin levels of 12.1 g/dL (reference range, 13.5-17.6 g/dL), platelet count of 29 × 103/μL (reference range, 130-400 × 103μL), and elevated serum C-reactive protein levels of 3.31 mg/dL (reference range, <0.30 mg/dL)", "Blood culture results were negative", "Granulocyte colony-stimulating factor analogs had not been prescribed to the patient", "Skin biopsy from a lesion on the left chest was performed for histopathological and immunohistochemical analyses (Figure, B-D)", "A, Clinical image of annular erythematous plaques on the chest", "B, A histopathologic image revealing inflammatory infiltrate of predominantly mononuclear cells in the superficial and mid dermis (hematoxylin-eosin stain)", "C, The predominantly infiltrated cells showing large, elongated, vesicular nuclei, single inconspicuous nucleoli, and scant, slightly eosinophilic cytoplasm (hematoxylin-eosin stain)", "D, An immunohistochemical image showing myeloperoxidase-positive cells"], "s1": [0, 1, 4, 5, 6], "s2": [2, 3, 7, 8, 9, 10, 11]} {"key": 484, "questions": "What Is Your Diagnosis?", "options": [{"label": "A", "disease": "Moyamoya disease with renovascular hypertension"}, {"label": "B", "disease": "Atherosclerotic disease"}, {"label": "C", "disease": "Fibromuscular dysplasia"}, {"label": "D", "disease": "Takayasu arteritis"}], "answer_idx": "D", "symptoms": ["A 6-year child presented with headache for 1 month and seizures followed by altered consciousness for 1 day", "He had new-onset daily occipital headache for 1 month of mild to moderate severity and pulsating character", "Occasionally, the headache became severe accompanied by vomiting, photophobia, and phonophobia", "The child had 4 to 5 episodes of generalized tonic-clonic seizures 1 day prior to admission", "He remained unconscious for several hours and spontaneously regained consciousness over 24 hours", "There was no history of fever, weight loss, limb weakness, vision impairment, or any strokelike illness", "His birth and developmental history were unremarkable", "On examination, his pulse rate was 80 beats per minute (regular) with all peripheral pulses palpable without any radio-femoral delay", "His blood pressure was 170/110 mm Hg with less than 10 mm Hg difference of systolic and diastolic blood pressure in other limbs", "Abdominal examination revealed abdominal bruit", "Fundus examination revealed papilledema with changes of hypertensive retinopathy", "He was not oriented to place and time", "The cranial nerves, motor, sensory, and autonomic system examination were unremarkable", "Meningeal signs were absent", "There was no evidence of neurocutaneous markers", "His cranial magnetic resonance imaging revealed bilateral white matter hyperintensity that was more marked in the bilateral parieto-occipital region", "Ultrasound of the abdomen revealed a small right kidney and renal Doppler was suggestive of right renal-artery stenosis", "His computed tomography angiography revealed short segment (3 cm) and short circumferential thickening of the descending aorta with right renal artery narrowing (Figure 1)", "His repeat cranial magnetic resonance imaging showed significant reduction in the white matter hyperintensities", "Maximum-intensity projection computed tomography (CT) angiography abdomen axial section (thickness 7 mm) (A) showing narrowing of whole length of right renal artery and thickened abdominal aortal wall (black arrowhead) and coronal section (B) showing small asymmetrical right kidney and compensatory enlarged left kidney"], "s1": [0, 1, 2, 3, 4, 11, 15, 18], "s2": [5, 6, 7, 8, 9, 10, 12, 13, 14, 16, 17, 19]} {"key": 485, "questions": "What Is Your Diagnosis?", "options": [{"label": "A", "disease": "Teratoma"}, {"label": "B", "disease": "Dermoid cyst"}, {"label": "C", "disease": "Hairy pharyngeal polyp"}, {"label": "D", "disease": "Squamous papilloma"}], "answer_idx": "C", "symptoms": ["A 7-year-old girl was referred to the otolaryngology clinic by her pediatrician for a left pharyngeal lesion", "Her parents noted the lesion 18 months prior", "It remained stable in size and was asymptomatic", "She denied sore throat, dysphagia, and dyspnea", "She was otherwise healthy with no history of recurrent pharyngitis", "On examination, her vital signs were within normal limits", "She was comfortable with no noted stridor and tolerated her oral secretions", "Her oropharyngeal examination was notable for size 2+ palatine tonsils without exudate and a white mass on the posterior tonsillar pillar", "She did not tolerate palpation of the lesion", "There was no palpable cervical adenopathy, and her lungs were clear to auscultation", "No laboratory work or imaging were performed", "The patient was taken to the operating room for an excisional biopsy", "There, a more thorough gross examination of the lesion was notable for a pedunculated, white rubbery mass arising from the left posterior tonsillar pillar with a narrow mucosal stalk (Figure, A)", "A, Pedunculated, white rubbery mass arising from the left posterior tonsillar pillar with a narrow mucosal stalk", "B and C, A keratinizing squamous epithelium with Actinomyces colonization was found overlying pilosebaceous units including hair follicles, arrector pili muscles, and sebaceous glands (hematoxylin-eosin stains)", "Gross examination revealed a 2.2 × 1.1 × 0.7-cm, pink-tan to purple, rubbery lesion", "Histologic examination demonstrated both ectodermal and mesodermal components", "A keratinizing squamous epithelium with Actinomyces colonization was found overlying pilosebaceous units including hair follicles, arrector pili muscles, and sebaceous glands (Figure, B and C)", "The core of the lesion comprised adipose and fibrous tissue", "The stalk was covered with acanthotic squamous epithelium, and the base contained embedded salivary glands"], "s1": [0, 1, 2, 3, 4, 5, 6, 7, 8, 9, 10, 11], "s2": [12, 13, 14, 15, 16, 17, 18, 19]} {"key": 486, "questions": "What Is Your Diagnosis?", "options": [{"label": "A", "disease": "Leukemia cutis"}, {"label": "B", "disease": "Pyoderma gangrenosum"}, {"label": "C", "disease": "Leukocytoclastic vasculitis"}, {"label": "D", "disease": "Lymphomatoid papulosis"}], "answer_idx": "B", "symptoms": ["A 75-year-old man with myelodysplastic syndrome (MDS) with isolated deletion 5q (del5q) presented with multiple nonhealing skin ulcers on the chest and upper extremities", "Four years earlier, he started taking lenalidomide, 10 mg/d (28-day cycle), to treat worsening anemia", "after 2 cycles of treatment, he became transfusion independent", "Since then, he has remained stable with the same dose and schedule of lenalidomide without significant adverse effects", "The presenting skin lesions developed 1 month before presentation and began as small pustules that rapidly transformed into painful ulcers", "He was treated with valacyclovir for a presumed diagnosis of shingles, but the lesions did not improve", "Because of suspected necrotizing fasciitis, he underwent incision and drainage of the lesions", "Cultures from the surgical specimen grew methicillin-resistant Staphylococcus hemolyticus", "However, the lesions continued to worsen despite vancomycin and additional debridement", "Thus, he was transferred to our facility", "Physical examination showed multiple cutaneous ulcers with gray undermined borders and red papules of varying depths involving the chest, back, and upper extremity", "lesions were as large as 9 × 15 cm", "A complete blood cell count showed a white blood cell count of 16.2 × 103/μL (to convert to 109/L, multiply by 1.0) with an absolute neutrophil count of 12.7 × 103/μL, hemoglobin level of 9.4 g/dL (to convert to g/L, multiply by 10.0), and platelet count of 262 × 103/μL with no circulating blasts", "Results of additional laboratory studies, including blood culture and tests for herpes zoster virus (via polymerase chain reaction), antinuclear antibody, rheumatoid factor, and antineutrophil cytoplasmic antibody were all negative", "Additionally, the patient also had hypertension and hypercholesterolemia for which he was taking lisinopril and simvastatin, respectively", "Pathologic sampling from skin lesions demonstrated a dense dermal infiltrate with predominantly neutrophils along with reactive histiocytes, fibroblasts, and angioplasia (Figure, A)", "Immunohistochemical staining showed myeloperoxidase-positive neutrophils (Figure, B), histiocytic markers (CD43, CD63, CD168), and CD117-positive mast cells, but no immature myeloid precursors", "Results of Grocott-Gomori methenamine silver staining were unremarkable", "A, Hematoxylin-eosin staining shows polymorphonuclear neutrophils admixed with scattered histiocytes and vasculatures with a prominent endothelial lining", "B, Immunohistochemical staining shows myeloperoxidase-positive neutrophils peripherally and in aggregates"], "s1": [0, 1, 2, 3, 9, 14], "s2": [4, 5, 6, 7, 8, 10, 11, 12, 13, 15, 16, 17, 18, 19]} {"key": 487, "questions": "What Is Your Diagnosis?", "options": [{"label": "A", "disease": "Symplastic glomus tumor"}, {"label": "B", "disease": "Angioleiomyoma"}, {"label": "C", "disease": "Solitary fibrous tumor"}, {"label": "D", "disease": "Malignant myopericytoma"}], "answer_idx": "D", "symptoms": ["A 74-year-old man with a history of multiple squamous cell carcinomas that were treated with Mohs micrographic surgery presented with a tender, enlarging scalp lesion of 6 months’ duration", "Skin examination revealed a 5-mm bright pink, dome-shaped papule with overlying vessels located at the crown of the scalp (Figure, A)", "There was no purulent discharge or overlying skin changes", "A shave skin biopsy was performed and submitted for histopathologic analysis (Figure, B-D)", "A, Bright pink, dome-shaped papule of the scalp crown with overlying vessels measuring 5 mm", "B, Hematoxylin-eosin staining of the lesion from biopsy", "Scanning magnification demonstrates dermal-based tumor cells arranged in a vague nodular configuration", "C, Proliferation of tumor cells is present around small vessels with scattered mitoses (black arrowhead) and necrosis (yellow arrowheads) (hematoxylin-eosin)", "D, Smooth muscle actin stain"], "s1": [0, 1, 2, 4], "s2": [3, 5, 6, 7, 8]} {"key": 488, "questions": "What Is Your Diagnosis?", "options": [{"label": "A", "disease": "Patch-stage Kaposi sarcoma"}, {"label": "B", "disease": "Interstitial granulomatous dermatitis"}, {"label": "C", "disease": "Acquired progressive lymphangioma"}, {"label": "D", "disease": "Microvenular hemangioma"}], "answer_idx": "D", "symptoms": ["An otherwise healthy female patient in her late 20s was seen with a 2-year history of multiple, slowly growing, infiltrated erythematous-violaceous patches and plaques, with occasional slight pain extending from her left upper arm to the left side of her chest", "No triggers, such as local infection, insect bite, or trauma, occurred before the appearance of the lesions", "She denied fever, weight loss, or fatigue", "Physical examination showed multiple firm, nonscaly, purple erythematous patches and plaques with irregular borders on her left axilla, medial upper arm, and lateral thoracic region (Figure, A)", "No extracutaneous involvement was observed", "A skin biopsy specimen was obtained from the left side of her chest for histopathological examination (Figure, B and C)", "A, Multiple nonscaly, erythematous-violaceous patches and plaques with irregular borders on the patient’s left axilla, medial upper arm, and lateral thoracic region", "B, Irregularly branched, thin-walled blood vessels dissecting through collagen bundles within the superficial to mid dermis (hematoxylin-eosin)", "C, Irregularly branched, thin-walled blood vessels with plump endothelial cells dispersed between thickened collagen bundles without signs of atypia (hematoxylin-eosin)", "D, Endothelial cells positive for cluster of differentiation (CD) 31 (CD31)"], "s1": [0, 1, 2, 3, 4, 6], "s2": [5, 7, 8, 9]} {"key": 489, "questions": "What Is Your Diagnosis?", "options": [{"label": "A", "disease": "Papillary carcinoma of the thyroid"}, {"label": "B", "disease": "High-grade serous carcinoma of gynecological origin"}, {"label": "C", "disease": "High-grade carcinoma of the breast with occult primary"}, {"label": "D", "disease": "High-grade papillary carcinoma of unknown primary"}], "answer_idx": "B", "symptoms": ["A 76-year-old woman presented with a palpable left axillary mass", "When she was 36 years old, she underwent a hysterectomy due to abnormal uterine bleeding and was taking estradiol to control postmenopausal vasomotor symptoms for the past 20 years", "Bilateral diagnostic mammography revealed a high-density irregular mass in the left axilla with no nodules, architectural distortions, or microcalcifications in the breasts", "Ultrasonography of the left breast showed a 4.6-cm lobulated mixed cystic and solid left axillary mass", "Contrast-enhanced magnetic resonance imaging showed an enhancing 5.2-cm left axillary mass with no breast lesions (Figure 1A)", "The biopsy examination indicated high-grade adenocarcinoma with papillary features (estrogen receptor– and progesterone receptor–positive, ERBB2 [formerly HER2]–negative, and a high Ki67 index)", "Positron emission tomography/computed tomography revealed only high uptake in the left axilla region", "The patient was treated at an outside hospital with neoadjuvant dose-dense doxorubicin and cyclophosphamide, followed by weekly paclitaxel for 10 weeks", "She then transferred her care to our center for surgical management", "We requested the pathology slides for review at our laboratory and additional immunostaining was performed, which revealed high-grade adenocarcinoma with tumor cells positive for PAX8 (Figure 1B), p53, and WT1 and negative for GATA3.A, Magnetic resonance image of the breasts shows an enhancing 5.2-cm left axillary mass (arrowhead) but no breast lesions", "B, Immunohistochemical staining revealed tumor cells positive for PAX8 (original magnification ×100)", "High-grade carcinoma of the breast with occult primary"], "s1": [0, 1, 8, 11], "s2": [2, 3, 4, 5, 6, 7, 9, 10]} {"key": 490, "questions": "What Is Your Diagnosis?", "options": [{"label": "A", "disease": "Branchial cleft cyst"}, {"label": "B", "disease": "Epidermoid cyst"}, {"label": "C", "disease": "Cervical chondrocutaneous branchial remnant"}, {"label": "D", "disease": "Hair follicle nevus"}], "answer_idx": "C", "symptoms": ["A 7-year-old boy presented with a slowly growing, asymptomatic lump on his left lower neck since birth", "His parents denied any medical history of trauma, infection, or surgery", "Physical examination showed a 1.3 × 0.7 × 0.6 cm, yellowish, hump-like mass with hairy surface and cartilage-like consistency on the anterior border of the lower third of the left sternocleidomastoid muscle (SCM) (Figure, A)", "No other anomalies were observed", "Ultrasonography revealed a 1.2 × 1.0 × 0.9 cm, hypoechoic, avascular, bulging nodule with an anechoic tubular structure extending to the subcutaneous fat and reaching the surface of the muscular layer", "Magnetic resonance imaging (MRI) demonstrated a protuberant nodule with diffuse, slight hyperintensity on T1WI", "and 2 mild hyperintense foci surrounded by hypointense tissue on fat-suppressed T2WI (Figure, B)", "The neck mass was completely excised under general anesthesia in the department of pediatric surgery", "Intraoperatively, the mass extended into the anteromedial fascia of the left SCM and did not involve the deeper neck structures", "Postoperative histopathologic analysis showed normal-appearing epidermis, multiple pilosebaceous follicles, abundant adipose tissue, and 2 centrally circumscribed segments of cartilage in the subcutis (Figure, C)", "Van Gieson stain highlighted elastic fibers surrounding individual chondrocytes", "A, A yellowish hump-like mass with hairy surface on the anterior-inferior third border of the left sternocleidomastoid muscle (SCM)", "B, Transverse T2-weighted, fat-suppressed magnetic resonance image (MRI) revealed a nodular bulge with 2 mild hyperintense foci (arrowheads) surrounded by hypointense tissue", "C, Whole-slide hematoxylin-eosin stain showed normal-appearing epidermis, multiple pilosebaceous follicles, abundant adipose tissue, and 2 centrally circumscribed segments of cartilage (arrowheads) in the subcutis (original magnification ×10)"], "s1": [0, 1, 3, 7, 8], "s2": [2, 4, 5, 6, 9, 10, 11, 12, 13]} {"key": 491, "questions": "What Is Your Diagnosis?", "options": [{"label": "A", "disease": "Soft tissue giant cell tumor"}, {"label": "B", "disease": "Ossifying fibromyxoid tumor"}, {"label": "C", "disease": "Soft tissue aneurysmal bone cyst"}, {"label": "D", "disease": "Soft tissue osteosarcoma"}], "answer_idx": "C", "symptoms": ["An 11-year-old girl with no significant medical history was referred to the pediatric otolaryngology department for a painless, left-sided neck mass that had been slowly enlarging for 8 months", "On examination, there was an easily palpable 3-cm firm, fixed, well-circumscribed left supraclavicular mass without overlying skin changes or cutaneous involvement", "She had no other masses, cervical lymphadenopathy, or systemic symptoms", "In workup for this mass, she initially underwent ultrasound imaging, which demonstrated a hypoechoic mass", "Findings from fine-needle aspiration and core-needle biopsy were inconclusive, demonstrating histiocytes", "Magnetic resonance imaging demonstrated a well-circumscribed mass of mainly soft tissue density with contrast enhancement (Figure 1)", "Magnetic resonance imaging of the left-sided supraclavicular mass", "T2 sequencing with contrast demonstrates strong enhancement of the mass", "Given persistence of the lesion, and inconclusive biopsies, the decision was made to pursue surgical excision", "In the operating room, a well-defined 5.5-cm left level V mass was excised", "The mass was found to attach deep to the middle scalene and was fed by several unnamed vessels", "however, there was no involvement of the clavicle", "Pathology confirmed complete excision of a well-circumscribed lesion with peripheral casing of bone, cyst-like spaces filled with serous fluid, walls of spindle-shaped fibroblasts, osteoclast-type giant cells, thin trabeculae of woven bone surfaced with osteoblasts, and scattered macrophages", "There was local infiltration into surrounding skeletal muscle, but no atypical mitoses or pleomorphic cells were identified", "The lesion was positive for USP6 locus rearrangement (17p13.2) on florescence in situ hybridization"], "s1": [0, 1, 2, 8, 9, 10, 11], "s2": [3, 4, 5, 6, 7, 12, 13, 14]} {"key": 492, "questions": "What Is Your Diagnosis?", "options": [{"label": "A", "disease": "Pilonidal sinus"}, {"label": "B", "disease": "Cutaneous larva migrans"}, {"label": "C", "disease": "Median raphe cyst"}, {"label": "D", "disease": "Vellus hair cyst"}], "answer_idx": "A", "symptoms": ["A healthy, uncircumcised man in his 30s presented with a 14-year history of dyspareunia and recurrent, purulent discharge from his penis", "He had undergone multiple evaluations by dermatologists and urologists and had not responded to systemic antibiotics and topical corticosteroids", "Physical examination revealed an erythematous, pinpoint orifice on his glans penis, with peripheral telangiectasias and a linear bluish structure on dermoscopy (Figure, A and B)", "Ultrasonography revealed a hypoechoic focal zone on the right side of the glans, with multiple hyperechoic bilaminar structures (Figure, C)", "A biopsy was performed (Figure, D)", "A, Clinical image of glans penis with a pinpoint orifice", "B, Dermoscopy reveals an orifice surrounded by peripheral telangiectasias and an extending bluish linear structure", "C, Ultrasonography of the glans penis demonstrates a hypoechoic focal zone in the parenchyma on the right containing multiple hyperechoic bilaminar tracts (arrowheads) (18 MHz", "longitudinal view)", "D, Hematoxlyin-eosin stain with original magnification ×100."], "s1": [0, 1, 2, 3], "s2": [4, 5, 6, 7, 8, 9]} {"key": 493, "questions": "What Is Your Diagnosis?", "options": [{"label": "A", "disease": "Cutaneous deposits of metastatic breast cancer"}, {"label": "B", "disease": "Inflammation of actinic keratoses"}, {"label": "C", "disease": "Medication hypersensitivity reaction"}, {"label": "D", "disease": "Discoid lupus erythematosus"}], "answer_idx": "B", "symptoms": ["A 56-year-old woman presented at the clinic for follow-up of metastatic hormone receptor–positive, ERBB2-negative breast cancer", "She had locally advanced disease that was invading the skin overlying her right breast and metastatic disease to the thoracic and lumbar spine, liver, and mediastinal, axillary, and retroperitoneal lymph nodes", "Her disease had progressed after receiving first-line therapy with palbociclib and anastrozole", "Subsequent treatments with fulvestrant and paclitaxel were also followed by progressive disease", "She received treatment with capecitabine", "after 6 weeks of treatment, she developed some redness on her hands and feet that resolved on its own", "She then presented to the clinic with pruritic, erythematous, scaly macules and papules on her forearms, faces, chest, and upper back (Figure)", "The rash had been present for 3 weeks and had recently worsened", "Her palms and soles were spared", "The rash did not respond to treatment with topical clindamycin, oral solumedrol, or doxycycline", "She had a history of substantial sun exposure but no history of skin cancer"], "s1": [0, 1, 2, 3, 4, 10], "s2": [5, 6, 7, 8, 9]} {"key": 494, "questions": "What Is Your Diagnosis?", "options": [{"label": "A", "disease": "Chondroma"}, {"label": "B", "disease": "Glomus tumor"}, {"label": "C", "disease": "Schwannoma"}, {"label": "D", "disease": "Granular cell tumor"}], "answer_idx": "C", "symptoms": ["A 12-year-old girl presented to the clinic with a 3-month history of intermittent stridor", "Her symptoms were initially most prominent while playing sports and were suspected to be due to asthma or seasonal allergies", "However, medical management with albuterol, intranasal fluticasone, and cetirizine failed to provide relief", "More recently, the patient’s parents noted that she developed stridor while sleeping and while at rest, prompting the family to present for medical attention", "Previous workup, including pulmonary function testing, was concerning for an upper airway obstructive process, for which she was referred to otolaryngology", "In-office flexible videostroboscopy revealed a mass in the distal cervical trachea that appeared nearly obstructive", "Chest radiography confirmed the presence of an approximately 1.5-cm, well-circumscribed soft tissue mass within the cervical trachea", "Direct laryngoscopy and bronchoscopy were performed in the operating room under general anesthesia", "This demonstrated an exophytic, pedunculated mass that emanated from the anterior wall of the cervical trachea (Figure 1A and B)", "The patient was intubated via Seldinger technique to bypass the mass, with a 4.0-mm cuffed endotracheal tube loaded over a 0-degree Hopkins endoscope (Figure 1C)", "With the airway secured, attention was turned to excision of the mass", "Lidocaine, 1%, with epinephrine 1:100 000 was injected in a submucosal plane, and laryngeal scissors were used to excise the attachment of the mass from the tracheal wall", "The mass was then resected en bloc and removed with laryngeal cupped forceps", "The attachment site was then ablated with the Coblator (Figure 1D)", "Intraoperative photographs demonstrating supraglottic view of intratracheal mass (A)", "exophytic, pedunculated mass emanating from the anterior wall of the cervical trachea (B)", "endotracheal intubation via Seldinger technique bypassing the mass (C)", "and cervical trachea following ablation of attachment site (D)"], "s1": [0, 1, 2, 3, 4], "s2": [5, 6, 7, 8, 9, 10, 11, 12, 13, 14, 15, 16, 17]} {"key": 495, "questions": "What Is Your Diagnosis?", "options": [{"label": "A", "disease": "Gorham-Stout disease"}, {"label": "B", "disease": "Langerhans cell histiocytosis"}, {"label": "C", "disease": "Cholesteatoma"}, {"label": "D", "disease": "Sarcoma"}], "answer_idx": "A", "symptoms": ["A 25-month-old child with unilateral hearing loss was referred to a pediatric ear, nose, and throat surgeon in a secondary care hospital", "Clinical findings suggested unilateral right middle ear effusion associated with ipsilateral conductive hearing loss", "After an initial observation period, a right tympanostomy tube was inserted", "During the procedure, a clear liquid pulsating from the middle ear was noted, and samples were sent for β-2-transferrine analysis", "This confirmed the presence of a cerebrospinal fluid (CSF) leak", "The patient was referred to our tertiary referral center hospital", "The leak settled with conservative management, and imaging of the brain and skull base was performed", "A high-resolution computed tomography scan of the temporal bone showed a lytic aspect of the right temporal bone extending to the petrous apex and the petrous bone, sparing the otic capsule (Figure 1A)", "There was soft tissue opacification of the right mastoid air cells", "Magnetic resonance imaging of the skull base with contrast was performed (Figure 1B)", "It showed a hyperintense T2 signal in the petrous apex and mastoid air cells", "There was a suspected communication into the soft tissues of the right neck in the posterior triangle", "Magnetic resonance imaging also showed signs of Chiari malformation type 1. After a skull base multidisciplinary discussion, the patient underwent a bone biopsy through a transmastoid approach", "A, Axial high-resolution computed tomography scan of the temporal bones at the level of the basal turn of the cochlea showed osteolysis involving the petrous part of the right temporal bone", "There was associated soft tissue partial opacification of the right mastoid air cells", "The destructive process did not involve the otic capsule", "B, Coronal magnetic resonance imaging of the brain and skull base, T2-weighted images showed a hypersignal in the right mastoid and middle ear with suspected communication through the tegmen", "There was also a hypersignal in the soft tissues of the right neck in keeping with extravasation of cerebrospinal fluid in cervical lymphatics"], "s1": [0, 1, 2, 3, 5, 6, 8, 12], "s2": [4, 7, 9, 10, 11, 13, 14, 15, 16, 17]} {"key": 496, "questions": "What Is Your Diagnosis?", "options": [{"label": "A", "disease": "Hydroxyurea-induced ulcerations"}, {"label": "B", "disease": "Hydroxyurea-induced squamous dysplasia"}, {"label": "C", "disease": "Hydroxyurea-induced phototoxic eruption"}, {"label": "D", "disease": "Iatrogenic epidermodysplasia verruciformis"}], "answer_idx": "B", "symptoms": ["A woman in her 80s initially presented with numerous, primarily photodistributed, crusted, and ulcerated plaques of the trunk and extremities", "She had a medical history of essential thrombocytosis (ET), which was treated with hydroxyurea for 14 years, with a total cumulative dose of approximately 2500 g", "She was otherwise reasonably well and denied any other constitutional symptoms", "The ulcerations had slowly increased in number and size over several years and were quite painful (Figure, A-C)", "She did have a medical history of nonmelanoma skin cancer (NMSCa), which had previously been treated surgically", "Several tangential shave biopsies were performed to help clarify the patient’s diagnosis", "A-C, Clinical findings including widespread photodistributed ulcerated pink papules and plaques on the extremities", "D, Histopathologic findings (hematoxylin-eosin stain)"], "s1": [0, 2, 3, 6], "s2": [1, 4, 5, 7]} {"key": 497, "questions": "What Is Your Diagnosis?", "options": [{"label": "A", "disease": "Infantile myofibromatosis"}, {"label": "B", "disease": "Mafucci syndrome"}, {"label": "C", "disease": "Lipoid proteinosis"}, {"label": "D", "disease": "Juvenile hyaline fibromatosis"}], "answer_idx": "D", "symptoms": ["A woman in her late 40s was admitted to the hospital with a 40-year history of multiple hard nodules on the scalp", "At 5 years of age, the patient had developed multiple subcutaneous masses in the occipital and bilateral temporal regions", "At age 10, she was diagnosed with neurofibromatosis and underwent surgical resection of the tumors at a local hospital", "Tumors recurred 5 years after surgery, with their size and number gradually increasing with age", "Physical examination revealed scalp masses from 4 to 6 cm in diameter, with hard textures and smooth surfaces (Figure, A), and subcutaneous nodules of 1 × 1 cm to 2 × 2 cm with hard textures on the hands (Figure, B), feet, trunk, and lower extremities", "The patient had no tenderness, swelling, or other skin manifestations", "The patient had consanguineous parents, but no other family member, including her children, had the disease", "A, Multiple subcutaneous masses in the occipital and bilateral temporal regions", "The diameter of the scalp masses varied from 4 to 6 cm, with hard textures and smooth surfaces", "B, Subcutaneous hard nodules on the hands, ranging from 1 × 1 cm to 2 × 2 cm", "C and D, Large amounts of homogeneous eosinophilic hyaline matrix with scattered fibroblasts"], "s1": [0, 1, 2, 3, 6, 10], "s2": [4, 5, 7, 8, 9]} {"key": 498, "questions": "What Is Your Diagnosis?", "options": [{"label": "A", "disease": "Atypical infection, nocardia"}, {"label": "B", "disease": "Opportunistic fungal infection"}, {"label": "C", "disease": "Angiosarcoma"}, {"label": "D", "disease": "Granulation tissue"}], "answer_idx": "C", "symptoms": ["A man in his 50s presented with bleeding wounds and nodules on the bilateral hips and forearms", "He previously received failed systemic and radiation therapy for tumor stage mycosis fungoides (MF)", "He had received 11 courses of extended beam radiation therapy, including 3 courses of total skin irradiation of 24 to 30 Gy, and focal irradiation to the left posterior thigh and hip/buttock with 2.5/3 Gy to 12.5/24 Gy, respectively, and the right posterior thigh and hip with 2 Gy to 20 and 26 Gy, respectively", "Seven years before this visit, he underwent a reduced-intensity, matched, unrelated donor, allogeneic hematopoietic stem cell transplant (HSCT)", "His post-transplant course was complicated by severe chronic graft vs host disease of the skin, eyes, and gut, for which he received systemic corticosteroid therapy, tacrolimus, methotrexate, and long-term extracorporeal photopheresis", "Three years after the HSCT, he developed chronic ulcerations on the hips, back, and arms at the site of previously irradiated tumors, as well as multiple friable, hemorrhagic, and bleeding nodules on bilateral hips and his forearms while receiving treatment with corticosteroids and extracorporeal photopheresis", "He underwent several debulking procedures and excision biopsies of the nodules, which revealed granulation tissue on histology", "In this latest visit, physical examination revealed recurrent hemorrhagic and friable nodules on his left lateral hip and left lateral buttock (Figure 1)", "Similar nodules were noted on his right hip and forearm", "The patient underwent repeated debulking and biopsy", "He was also treated with micafungin, amphotericin B, levofloxacin, dapsone, acyclovir, pentamidine, penicillin VK, and posaconazole", "Positron emission tomography (PET) computed tomography (CT) results did not show evidence of visceral or extracutaneous disease", "Recurrent hemorrhagic and friable nodules on the left lateral hip"], "s1": [0, 7, 8, 12], "s2": [1, 2, 3, 4, 5, 6, 9, 10, 11]} {"key": 499, "questions": "What Is Your Diagnosis?", "options": [{"label": "A", "disease": "Graves disease"}, {"label": "B", "disease": "Invasive fungal sinusitis with orbital extension"}, {"label": "C", "disease": "IgG4-related disease"}, {"label": "D", "disease": "Sarcoidosis"}], "answer_idx": "C", "symptoms": ["A 48-year-old woman presented with 3 days of worsening right orbital swelling, eye pain, blurry vision, and 3 months of bilateral photophobia", "She also reported bilateral rhinorrhea, congestion, and right frontal and temporal headaches", "Active medical issues included hypertension, very poorly controlled type 2 diabetes, hyperlipidemia, tobacco use, and asthma", "Her brother and father both had a history of chronic orbital swelling", "Vital signs were unremarkable, and examination of the right eye showed moderate chemosis, injected conjunctiva, and periorbital edema", "Both eyes were proptotic, right worse than left", "She had normal bilateral extraocular movements, pupillary light reflexes, and gross vision", "Computed tomography with contrast revealed bilateral ethmoid and frontal sinus opacification and right worse than left orbital fat stranding (Figure 1A)", "Magnetic resonance imaging with contrast demonstrated both enlargement and enhancement of bilateral extraocular muscles, especially the right superior rectus (Figure 1B)", "Initially she was started on antibiotic therapy", "Right nasal biopsy, right endoscopic sinus surgery, and right anterior superior orbital tissue biopsy were performed", "Intraoperatively, the sinus mucosa demonstrated only edema and mild secretions, but there was no necrosis or purulence", "Both orbital and nasal cultures grew methicillin-resistant Staphylococcus aureus", "Orbital biopsy showed nodular lymphoplasmacytic infiltrates without evidence of fungal infection or necrosis (Figure 1C)", "Further workup revealed elevated erythrocyte sedimentation rate, C-reactive protein, total IgG, and IgG4 levels and normal white blood cell count, thyroid-stimulating hormone, angiotensin-converting enzyme, lysozyme, antinuclear antibody, and antineutrophil cytoplasmic antibody (ANCA) levels", "A, Computed tomography with contrast revealed bilateral ethmoid and frontal sinus opacification and right worse than left orbital fat stranding", "B, Magnetic resonance imaging with contrast demonstrated both enlargement and enhancement of bilateral extra ocular muscles, with the right superior rectus muscle being the most severely affected", "C, Orbital biopsy showed nodular lymphoplasmacytic infiltrates separated by thick collagen bundles, without evidence of fungal infection or necrosis (hematoxylin-eosin)"], "s1": [0, 1, 4, 5, 6, 7, 8, 15, 16], "s2": [2, 3, 9, 10, 11, 12, 13, 14, 17]} {"key": 500, "questions": "What Is Your Diagnosis?", "options": [{"label": "A", "disease": "Infection and suppuration"}, {"label": "B", "disease": "Immunotherapy-related inflammation"}, {"label": "C", "disease": "Tumor progression"}, {"label": "D", "disease": "Allergic reaction"}], "answer_idx": "B", "symptoms": ["A 69-year-old woman with a newly diagnosed T3N2cM0 poorly differentiated squamous cell carcinoma unrelated to human papillomavirus (HPV) of the lower lip mucosa presented to the medical oncology clinic 3 days after initiating neoadjuvant immune checkpoint blockade immunotherapy with redness and swelling of the tumor site", "Four days after treatment, a white head formed overlying the tumor that erupted through the skin surface (Figure 1)", "Thin, cloudy discharge that was not foul-smelling drained from the tumor from posttreatment days 5 to 10, at which point the drainage stopped spontaneously", "Throughout, the patient reported no pain and remained afebrile", "Laboratory results on days 5, 9, and 14 demonstrated the absence of leukocytosis, and serial wound cultures failed to demonstrate pathogenic growth", "No antibiotics were administered", "The patient completed the neoadjuvant immunotherapy and underwent a margin-negative tumor resection without intraoperative or postoperative complications", "This photo documents clinical tumor behavior after neoadjuvant immunotherapy treatment"], "s1": [0, 6, 7], "s2": [1, 2, 3, 4, 5]} {"key": 501, "questions": "What Is Your Diagnosis?", "options": [{"label": "A", "disease": "Pigmented epithelioid melanocytoma"}, {"label": "B", "disease": "Hemosiderotic fibrolipomatous tumor"}, {"label": "C", "disease": "Pigmented dermatofibrosarcoma protuberans (Bednar tumor)"}, {"label": "D", "disease": "Angiomatoid fibrous histiocytoma"}], "answer_idx": "B", "symptoms": ["An otherwise healthy woman in her 30s presented with a painful bruise on her hand that initially appeared a few days after she sustained a twisting injury to the affected hand while participating in acroyoga 2 years before", "Since the initial onset, the bruise and pain had persisted despite icing and resting the area", "She denied any personal or family history of bleeding disorders", "On examination, an ill-defined indurated blue-green plaque was noted on the dorsum of the left hand (Figure 1)", "Radiography results showed soft tissue swelling but were otherwise unremarkable", "Subsequent magnetic resonance imaging showed a heterogeneously fluid hyperintense and T1 heterogeneously hyperintense lesion measuring approximately 1.6 × 0.5 × 2.9 cm that was centered within the dorsal subcutaneous soft tissue of the hand", "An excision was performed"], "s1": [0, 1, 2, 3], "s2": [4, 5, 6]} {"key": 502, "questions": "What Is Your Diagnosis?", "options": [{"label": "A", "disease": "Radiation-induced bullous pemphigoid"}, {"label": "B", "disease": "Urticarial vasculitis"}, {"label": "C", "disease": "Eosinophilic, polymorphic, and pruritic eruption associated with radiotherapy (EPPER)"}, {"label": "D", "disease": "Urticaria multiforme"}], "answer_idx": "C", "symptoms": ["A woman in her 60s presented for evaluation of a new pruritic rash", "The rash initially developed on the patient’s legs 13 days before presentation and subsequently progressed to her feet, arms, hands, and trunk", "Associated symptoms included a burning sensation and occasional pain", "Her medical history was notable for endometrial adenocarcinoma status post hysterectomy and recent postoperative radiotherapy (total of 50.4 Gy in 28 fractioned doses), which was completed 4 days after the onset of the rash", "She denied any new medications or exposure to chemotherapy, imaging with intravenous contrast, or recent travel", "The remainder of her review of systems was unremarkable", "On physical examination, there were numerous erythematous edematous coalescing papules and plaques on the upper and lower extremities and scattered erythematous papules and plaques on the back and buttocks (Figure, A)", "Some plaques were annular and polycyclic in configuration, with central hyperpigmentation (Figure, B)", "Her chest, abdomen, face, and oral mucosa were clear", "A complete blood cell count was notable for an eosinophilia of 11%, with an absolute cell count of 0.80 ×103/uL", "Results from a comprehensive metabolic panel were normal", "A biopsy was performed on her right thigh for histopathological examination", "A, Clinical photographs show numerous erythematous edematous coalescing papules and plaques on the bilateral upper and lower extremities", "B, Some plaques were annular and polycyclic in configuration with central hyperpigmentation", "C and D, Punch biopsy results demonstrated a moderately dense perivascular and interstitial infiltrate of mononuclear cells with innumerable eosinophils (hematoxylin-eosin stain)", "Eosinophilic, polymorphic, and pruritic eruption associated with radiotherapy (EPPER)"], "s1": [0, 1, 2, 6, 7, 12, 13], "s2": [3, 4, 5, 8, 9, 10, 11, 14, 15]} {"key": 503, "questions": "What Is Your Diagnosis?", "options": [{"label": "A", "disease": "Neurosyphilis"}, {"label": "B", "disease": "Metastatic tumor"}, {"label": "C", "disease": "Rickettsial infection"}, {"label": "D", "disease": "Synovitis, acne, pustulosis, hyperostosis, and osteitis syndrome"}], "answer_idx": "D", "symptoms": ["A 51-year-old previously healthy man was admitted with severe neck pain for 6 days", "In the past month, he felt neck stiffness, accompanied by swelling and persistent dull pain of the upper anterior chest wall, which could be transiently relieved by taking oral ibuprofen", "Six days before admission, the patient’s neck stiffness transited to an endurable dull ache", "When turning his head, there was severe burning pain in the posterior neck and occiput, followed immediately by an electric shocklike numbness on ipsilateral side tongue, which led to a forced head position", "Magnetic resonance imaging of the cervical spine with gadolinium on admission revealed signal abnormality in C1/C2 vertebrae (Figure 1A)", "In the past 3 days, with the intensification of neck pain, the patient found pustules on both hands, which gradually increased", "He had no history of tobacco, alcohol, or drug misuse", "On examination, he was afebrile", "There was tenderness in the right sternoclavicular articulation, upper neck, post aurem, and occiput", "Multiple pustules were seen in the palm and back of both hands (Figure 1B)", "Complete blood cell count and liver and kidney function yielded normal results, except for elevated erythrocyte sedimentation rate (ESR) of 93 mm/h and C-reactive protein (CRP) level of 5.5 mg/dL (to convert to milligrams per liter, multiply by 10)", "Results of workup for rheumatic and infectious disease, including antinuclear, antineutrophil cytoplasmic and anticardiolipin antibodies, human leukocyte antigen B27, repeated blood cultures, rapid plasma reagin test, Aspergillus galactomannan antigen, brucella, Mycobacterium tuberculosis, and rickettsial antibodies, were all negative", "Cell count, protein, and glucose in cerebrospinal fluid were normal", "Cultures of cerebrospinal fluid revealed no organisms", "Whole-body fluorodeoxyglucose–positron emission tomography/computed tomography revealed no malignancy", "Abnormal enhancement of C1/C2 vertebrae with prevertebral and retrodental tissue as well as of the anterior vertebrae at C4/C5 with hyperostotic anterior osteophytes on magnetic resonance imaging (MRI) (A) and multiple pustules on the palm (B)"], "s1": [0, 1, 2, 3, 4, 8, 15], "s2": [5, 6, 7, 9, 10, 11, 12, 13, 14]} {"key": 504, "questions": "What Is Your Diagnosis?", "options": [{"label": "A", "disease": "Cutaneous angiosarcoma"}, {"label": "B", "disease": "Leukocytoclastic vasculitis from COVID-19 vaccine"}, {"label": "C", "disease": "Inflammatory breast cancer"}, {"label": "D", "disease": "Primary adnexal gland carcinoma"}], "answer_idx": "C", "symptoms": ["A 71-year-old man presented to the dermatologist with a 6-week history of a rapidly progressing rash and swelling of the left side of the chest wall", "He had noticed the rash a few days after receiving the second dose of an mRNA COVID-19 vaccine (Moderna)", "He denied itching, weeping, bleeding, or discharge from the rash or the nipple", "He had a history of metabolic syndrome treated with antihypertensives and oral hypoglycemics", "The physical examination showed a morbidly obese White man", "The skin examination showed a 19 × 10-cm pink, fallacious purple color deep-infiltrated papule in the left breast and a similar 15 × 7-cm papule under the left clavicle (Figure, A)", "The breast examination was notable for bilateral gynecomastia and inverted left nipple without any palpable lumps", "No lymphadenopathy was noted either", "His laboratory results showed stage 2 chronic kidney disease", "Results of a viral panel including hepatitis A, B, and C and HIV were negative", "The punch biopsy of the skin showed a diffuse island of atypical cells in the dermis (Figure, B)", "On immunohistochemical staining, the atypical cells stained positive for CK-7, GATA-3, and PAX-8 (weak focal staining) and negative for estrogen receptor (ER)/progesterone receptor (<1%), TTF-1, SATB-2, CDX-2, and NKX-3.1. The positron emission tomography–computed tomography scan showed a diffuse, infiltrating fluorodeoxyglucose (FDG)-avid cutaneous lesion on the left chest wall (standardized uptake value, 6.5) and multiple FDG-avid nodes in the axilla (bilaterally), left internal mammary, and left cervical chain", "The core biopsy from the lymph node is shown in the Figure, C", "A, Rash on the left chest wall", "B and C, Hematoxylin-eosin stains of the punch biopsy of the skin and core biopsy of the lymph node, respectively (both original magnification ×10)"], "s1": [0, 1, 2, 3, 4, 6, 7, 8, 9], "s2": [5, 10, 11, 12, 13, 14]} {"key": 505, "questions": "What Is Your Diagnosis?", "options": [{"label": "A", "disease": "Madelung disease"}, {"label": "B", "disease": "Liposarcoma"}, {"label": "C", "disease": "Familial lipomatosis"}, {"label": "D", "disease": "Drug-induced lipomatosis"}], "answer_idx": "A", "symptoms": ["A 55-year-old male jazz musician with a medical history that was significant for hypertension, coronary artery disease, and obstructive sleep apnea presented with an enlarging right more than left-sided neck mass of more than 2 years’ duration", "The patient was seen and evaluated by a local surgical oncologist with a working diagnosis of possible lymphoma", "The growing neck mass was causing solid food dysphagia and difficulty breathing at night", "He did not have any neurological deficits", "On further questioning, he had a similar mass that was treated with liposuction 10 years prior", "His family history was negative for any head and neck benign or malignant disease", "His social history was negative for tobacco or alcohol use", "This patient was not taking any steroid medications", "Physical examination showed an enlarged right more than left-sided neck mass", "It was tense but compressible to palpation", "Flexible fiber-optic laryngoscopy showed a crowded oropharynx with lingual tonsil hypertrophy and narrow oropharynx and hypopharynx", "An outside computed tomography scan showed diffuse symmetric fatty infiltration of the pharynx, larynx, trachea, and esophagus", "A magnetic resonance imaging scan with and without contrast (Figure, A) showed extensive areas of fat deposition within the neck that bilaterally extended from the level of C1 in the retropharyngeal space to the level of the sternal notch", "Given that the patient was symptomatic, surgical intervention was recommended", "Preoperative magnetic resonance imaging (A) and gross specimen (B)", "Surgery was approached like a neck dissection", "The entire specimen was removed, including its capsule, which measured 17.0 × 14.0 × 5.0 cm (Figure, B)", "Six months later, a postsurgical magnetic resonance imaging scan showed that the fat prominence had largely been resected"], "s1": [0, 1, 2, 3, 5, 6, 7], "s2": [4, 8, 9, 10, 11, 12, 13, 14, 15, 16, 17]} {"key": 506, "questions": "What Is Your Diagnosis?", "options": [{"label": "A", "disease": "Inflammatory linear verrucous epidermal nevus"}, {"label": "B", "disease": "Trichodysplasia spinulosa"}, {"label": "C", "disease": "Porokeratotic eccrine ostial and dermal duct nevus"}, {"label": "D", "disease": "Multiple minute digitate hyperkeratosis"}], "answer_idx": "C", "symptoms": ["A school-aged boy presented with occasionally pruritic, generalized hyperkeratotic papules with central white spicules and comedo-like plugs that were in a linear distribution on his face, trunk, and extremities since birth", "Some of the eruptions on his trunk had spontaneously resolved with developmental growth", "There was nonconsanguineous marriage of his parents, and no family members had similar lesions", "The patient was otherwise in good general health and without developmental delay", "Physical examination showed generalized, linear hyperkeratotic papules with numerous central small filiform keratotic white spicules and brown plugs on the head, face, neck, trunk, and bilateral upper and lower extremities (Figure, A and B), including his palms and soles along Blaschko lines, accompanied with alopecia of the scalp and eyebrows", "The examination of nails, mucosae, and teeth was unremarkable", "No extracutaneous involvement was observed", "Dermoscopic examination showed many central white spines on the face (Figure, C) and brown cores surrounded by yellow crater-like rings on the right palm", "A skin biopsy specimen was taken from the hyperkeratotic papules with white spicules on his right thigh (Figure, D)", "A, Generalized, linear hyperkeratotic papules and plaques with numerous white spicules and plugs on the head, face, neck, and both arms are accompanied by alopecia of the scalp and eyebrows", "B, Generalized hyperkeratotic papules with numerous central small filiform keratotic white spicules on the face (close-up view)", "C, Dermoscopic examination on the face showing many central whitish spines", "D, Skin biopsy specimen taken from the right thigh showing hyperkeratosis, acanthosis, and parakeratotic columns overlying the dilated epidermal invaginations, as well as thinner granular layers and dilated eccrine ostia and acrosyringia under the parakeratotic columns (hematoxylin-eosin)"], "s1": [0, 4, 7, 8, 9, 10, 11, 12], "s2": [1, 2, 3, 5, 6]} {"key": 507, "questions": "What Is Your Diagnosis?", "options": [{"label": "A", "disease": "Behcet disease"}, {"label": "B", "disease": "Metastatic Crohn disease"}, {"label": "C", "disease": "Langerhans cell histiocytosis"}, {"label": "D", "disease": "Cutaneous T-cell lymphoma"}], "answer_idx": "C", "symptoms": ["A female patient in her late 40s presented with a 3-year history of nonhealing ulcers in her groin and axillae", "These lesions started as itchy, weepy, reddish papules and plaques, which gradually progressed to form ulcers and were associated with pain and pus discharge", "They did not heal completely after multiple courses of topical as well as systemic antibiotics, antifungals, and steroids", "She frequently experienced scaling and greasiness on her scalp, which was treated with shampoos and topical steroids", "She also complained of polyuria and polydipsia for the past 4 years", "General examination revealed multiple mobile, firm, nonmatted, and nontender lymph nodes in cervical and inguinal region", "On systemic examination, hepatomegaly (2 cm below the right costal margin) was seen", "There was also bilateral enlargement of the parotid area, which was tender on palpation", "Cutaneous examination revealed yellowish greasy scales on the scalp, suggestive of seborrheic dermatitis", "Erythematous edematous papules were present over the retroauricular areas and the inframammary folds (Figure, A)", "On examination of external genitalia, deep ulcers were observed in the bilateral inguinal folds, resembling knife-cut ulcers (Figure, B)", "Similar lesions were present over the junction of labia majora and labia minora and were associated with yellowish pus discharge", "A 3.5-mm punch biopsy specimen was obtained for histopathological examination from the edge of an ulcer on one of the groin folds (Figure, C and D)", "A, Erythematous and edematous papules in the inframammary area", "B, Deep ulcers in the groin folds resembling knife-cut ulcers with yellowish pus discharge", "C, Histopathological examination of the area with intact epidermis shows dense infiltration of the ulcer bed by atypical cells", "These cells were large and had characteristic grooved vesicular nuclei and moderate amphophilic cytoplasm", "D, Immunohistochemistry shows diffuse positivity for CD1a in the lower epidermis and dermis"], "s1": [0, 1, 2, 4, 5, 6, 7, 12, 15, 16, 17], "s2": [3, 8, 9, 10, 11, 13, 14]} {"key": 508, "questions": "What Is Your Diagnosis?", "options": [{"label": "A", "disease": "Late-onset Pompe disease (glycogen-storage disease type 2)"}, {"label": "B", "disease": "Myasthenia gravis"}, {"label": "C", "disease": "Paraneoplastic myositis"}, {"label": "D", "disease": "Sporadic late-onset nemaline rod myopathy"}], "answer_idx": "D", "symptoms": ["A 72-year-old man presented with mild proximal weakness developing into progressive camptocormia, head drop, and prominent dysphagia over a 2-year period", "He reported a 20-kg weight loss and numbness to the ankles", "His medical history was significant for thyroid cancer treated with total thyroidectomy resulting in a chronic left accessory nerve injury, type 2 diabetes, hypertension, and gout", "His medications were metformin, allopurinol, perindopril, thyroxine, and rabeprazole", "He had no history of smoking and drank 3 glasses of wine per week", "His family history was unremarkable", "Examination revealed significant wasting of proximal and paraspinal muscles with a flexed spine posture in the upright position (Figure 1A)", "He had symmetric Medical Research Council grade 4/5 weakness of neck flexion, shoulder abduction, elbow flexion and extension, and hip flexion and extension bilaterally", "He had weakness of left shoulder elevation and winging of the left scapula secondary to the accessory nerve injury", "His deep tendon reflexes were normal and symmetric", "Pinprick sensation was reduced in both feet", "Lateral profile of the patient (A) revealing a flexed spine posture (camptocormia) with marked generalized muscle atrophy", "B, A Gömöri trichrome stain of frozen left vastus muscle (original magnification ×400) reveals 3 atrophic muscle fibers with a patchy granular appearance (red arrowheads)", "Metabolic, endocrine, autoimmune, and infectious screens had unremarkable findings", "Creatine kinase levels were normal", "A myositis panel demonstrated low-positive Mi2 antibodies", "Serum electrophoresis and immunofixation identified a small immunoglobulin G (IgG) κ paraprotein with normal findings on bone marrow biopsy", "A whole-body positron emission tomography scan was normal", "Nerve conduction studies revealed a mild axonal sensorimotor polyneuropathy", "Concentric needle electromyography showed no spontaneous activity with widespread small, brief, polyphasic units, particularly in proximal and axial muscles, consistent with a myopathic process", "An initial deltoid muscle biopsy did not show evidence of myopathy", "Because the patient’s symptoms were worsening, a second muscle biopsy of vastus lateralis was performed", "This revealed atrophic myofibers with a patchy granular appearance (Figure 1B)"], "s1": [0, 6, 7, 8, 10, 11, 18, 19], "s2": [1, 2, 3, 4, 5, 9, 12, 13, 14, 15, 16, 17, 20, 21, 22]} {"key": 509, "questions": "What Is Your Diagnosis?", "options": [{"label": "A", "disease": "Rosai-Dorfman disease"}, {"label": "B", "disease": "Castleman disease"}, {"label": "C", "disease": "Inflammatory pseudotumor"}, {"label": "D", "disease": "Sarcoidosis"}], "answer_idx": "A", "symptoms": ["A previously healthy 12-year-old boy presented with a 3-month history of nasal obstruction, progressive dysphonia, recurrent deep neck abscesses, and tender, bulky cervical lymphadenopathy", "Computed tomography (CT) of the neck with contrast at his initial hospital admission was concerning for suppurative lymphadenitis", "Despite incision and drainage, and intravenous ampicillin and sulbactam, his cervical lymphadenopathy persisted", "Results of subsequent workup—including Haemophilus, tetanus, and pneumococcal antibody titers", "QuantiFERON-TB Gold test", "serum IgG, IgM, IgA, and IgE levels", "absolute lymphocyte count", "neutrophil phenotype and function", "angiotensin-converting enzyme level", "HLA-B27 test", "and antinuclear antibody level—were within normal limits", "Magnetic resonance imaging of the neck with gadolinium contrast demonstrated bilateral, bulky, enlarged cervical lymph nodes, as well as enhancing nodules in the left laryngeal ventricle (2.3 × 1.6 × 1.2 cm), right false vocal cord, and right tracheal wall (0.6 × 0.4 cm)", "Findings from nasal endoscopy, awake flexible laryngoscopy, and operative microlaryngoscopy revealed yellow submucosal masses in the left nasal cavity, left laryngeal ventricle, and right trachea, as well as left vocal cord paresis (Figure, A)", "Biopsies were taken of the tracheal, laryngeal, and nasal cavity masses, and an excisional biopsy was taken of a left cervical lymph node (Figure, B)", "Histopathologic examination revealed a prominent infiltrate of benign-appearing histiocytes with numerous forms showing emperipolesis and some nuclei showing distinct central nucleoli", "Immunohistochemical staining in these cells was positive for S100 and negative for CD1a", "A, Awake flexible laryngoscopy showing a yellow mucosal mass in the left laryngeal ventricle", "B, Hematoxylin-eosin–stained excisional lymph-node specimen at ×1000 magnification revealing histiocytes with central nucleoli and cytoplasm containing intact lymphocytes (emperipolesis)"], "s1": [0, 1, 2, 11, 12, 13, 16], "s2": [3, 4, 5, 6, 7, 8, 9, 10, 14, 15, 17]} {"key": 510, "questions": "What Is Your Diagnosis?", "options": [{"label": "A", "disease": "Oral hairy leukoplakia"}, {"label": "B", "disease": "Multifocal squamous cell carcinoma in situ"}, {"label": "C", "disease": "Secondary syphilis"}, {"label": "D", "disease": "Herpetic infection"}], "answer_idx": "C", "symptoms": ["A 19-year-old male presented for evaluation of painful lesions on the tongue that had been present for at least 3 months", "He stated that the pain was exacerbated by spicy foods", "He had initially presented to a primary care practitioner, was prescribed clotrimazole lozenges, and reported mild initial improvement followed by symptom progression", "Subsequently, the practitioner referred the patient to an otolaryngologist", "The patient denied any enlarged lymph nodes, fevers, fatigue, rash, weight loss, or abnormal gastrointestinal symptoms and had no noteworthy medical history", "He reported mild alcohol consumption, denied any cigarette or illicit substance use, and endorsed engaging in oral and anal intercourse", "A physical examination was notable for bilateral raised leukoplakia on the tongue", "it was larger on the right side than the left and had a slightly hairy appearance", "The lesion was tender to touch and could not be scraped off (Figure 1A)", "The lips and oral mucosa were without lesions, masses, or ulcers", "The patient had no cervical lymphadenopathy", "The findings of the physical examination were otherwise unremarkable", "A, Clinical image of the right anterolateral tongue showing a 1 × 3 cm plaque over tongue mucosa", "Left lateral tongue (not shown) had multiple gray papular lesions with similar appearance", "B, Hematoxylin-eosin stain of right tongue biopsy specimen revealing prominent lymphoplasmacytic infiltrate involving squamous mucosa", "no cellular atypia is seen among lymphocytes", "A biopsy procedure of the right oral tongue lesion was performed, and hematoxylin-eosin staining of the specimen revealed dense perivascular plasmacytic infiltration and neutrophilic inflammation (Figure 1A)"], "s1": [0, 1, 3, 4, 5, 9, 10, 11], "s2": [2, 6, 7, 8, 12, 13, 14, 15, 16]} {"key": 511, "questions": "What Is Your Diagnosis?", "options": [{"label": "A", "disease": "Short anagen syndrome"}, {"label": "B", "disease": "Loose anagen syndrome"}, {"label": "C", "disease": "Trichodental dysplasia"}, {"label": "D", "disease": "Hypotrichosis simplex of the scalp"}], "answer_idx": "D", "symptoms": ["A school-aged girl presented to the dermatology clinic with a 6-year history of hair loss and short hair", "Her scalp hair was relatively normal in terms of density and length at birth", "Since the age of 1 year, the girl had begun to have frequent hair loss without pulling", "Her mother also found that her hair grew to approximately the same maximum length down to the upper neck without trimming", "On clinical examination, diffuse, short, sparse, lanugolike, black hair over the scalp was found (Figure, A and B)", "The surface of the scalp was smooth without inflammatory changes or focal alopecic patches", "No involvement of eyebrows, eyelashes, nails, teeth, or body hair was found", "Her intellectual performance was normal at presentation and no developmental delay had ever been noted", "She denied experiencing a stressful event recently, and no habitual pulling of hair was observed by her family", "Family history regarding hair loss was unremarkable", "A and B, Diffuse, sparse, and abnormally short hair over the entire scalp is shown", "C, Punch biopsy showed a mild lymphocytic perifollicular infiltrate and increased telogen hair with decreased anagen-to-telogen ratio (hematoxylin-eosin staining, magnification 2x)", "The hair pull test was positive with predominately anagen hair extracted", "Light microscopy of scalp hair shafts revealed no structural abnormalities, such as cuticular damage, twisting, bending, or miniaturization", "Blood levels of ferritin, zinc, and hemoglobin were normal", "A punch biopsy from the alopecic area of the scalp was performed (Figure, C)"], "s1": [0, 2, 3, 4, 5, 6, 8, 9, 10], "s2": [1, 7, 11, 12, 13, 14, 15]} {"key": 512, "questions": "What Is Your Diagnosis?", "options": [{"label": "A", "disease": "Annular lichen planus"}, {"label": "B", "disease": "Perforating granuloma annulare"}, {"label": "C", "disease": "Eruptive pruritic papular porokeratosis"}, {"label": "D", "disease": "Atypical pityriasis rosea"}], "answer_idx": "C", "symptoms": ["A woman in her 50s presented with a 3-month history of rapidly spreading intensively pruritic papules on her extremities", "The lesions consisted of scattered, erythematous, annular papules measuring up to 5 mm wide (Figure 1)", "Further examination revealed a few asymptomatic brown flat papules scattered on her face", "the palms, soles, and oral mucosa were not involved", "The patient had no remarkable medical or family history and was not concurrently being treated with any medications", "She had been previously diagnosed with eczema and treated with topical steroid ointments and oral compound glycyrrhizin tablets, which had no obvious effect", "Clinical images of keratotic papules diffusely distributed on an extremity, original (A) and magnified (B)", "Laboratory investigations, including routine blood, liver and kidney function, antistreptolysin O, C-reactive protein, antinuclear antibody, and rheumatoid factor tests, disclosed no abnormal findings", "A biopsy specimen was obtained for histopathologic examination"], "s1": [0, 1, 2, 3, 5, 6], "s2": [4, 7, 8]} {"key": 513, "questions": "What Is Your Diagnosis?", "options": [{"label": "A", "disease": "Friedreich ataxia"}, {"label": "B", "disease": "Fragile X–associated tremor/ataxia syndrome"}, {"label": "C", "disease": "Niemann-Pick disease type C"}, {"label": "D", "disease": "Mitochondrial ataxia"}], "answer_idx": "D", "symptoms": ["A 55-year-old female patient with a history of hypercholesterolemia and anxiety was referred for progressive imbalance while walking and intense fear of falling for the past 14 years", "The patient had become progressively disabled and had been using a wheelchair for 4 years prior to presentation", "Neurological examination showed gaze-evoked horizontal nystagmus in both directions along with slow saccades and gaze restriction in both vertical and horizontal planes", "There was ataxic dysarthria", "Tone was normal in all 4 limbs but stretch reflexes were absent and plantar fascia were mute", "Limb power was normal", "Ataxia was seen on finger-to-nose and heel-knee-shin tests", "Chorea was present in both legs", "Ankle inversion with bilateral hammertoes was noted", "Electromyography revealed sensory neuronopathy", "On brain magnetic resonance imaging (MRI), diffuse cerebellar atrophy was noted along with T2 hyperintensities in bilateral middle cerebellar peduncles, cerebellar white matter, pons, and bilateral thalamus (Figure)", "Psychiatric treatment was commenced for her severe anxiety but did not relieve the fear of falling", "Magnetic resonance brain images showing T2 fluid-attenuated inversion recovery hyperintensities"], "s1": [0, 1, 11], "s2": [2, 3, 4, 5, 6, 7, 8, 9, 10, 12]} {"key": 514, "questions": "What Is Your Diagnosis?", "options": [{"label": "A", "disease": "Arnold-Chiari malformation"}, {"label": "B", "disease": "Spinal abscess causing laryngeal edema"}, {"label": "C", "disease": "Idiopathic bilateral vocal fold paralysis"}, {"label": "D", "disease": "Delayed iatrogenic bilateral vocal fold paralysis"}], "answer_idx": "D", "symptoms": ["A 68-year-old man with a medical history of coronary artery disease, atrial fibrillation, and obesity had undergone C5 anterior corpectomy and fusion (C4-6) with expandable cage, autograft, and anterior instrumentation and posterior spinal decompression and fusion (C4-7) for cervical spondylotic myelopathy", "Five months after surgery, he presented to the otolaryngology clinic with 3 weeks of mild odynophagia and dysphagia", "It was noted that the patient had a prominent posterior pharyngeal wall, subtle asymmetry of vocal fold motion, and pooling of secretions in the left pyriform (Video 1)", "During the next 6 months, he gradually developed shortness of breath and stridor, and eventually presented to the emergency department in respiratory distress requiring intubation for 2 days", "Shortly after extubation, laryngeal examination showed fullness of the posterior pharyngeal wall and severely limited vocal fold abduction bilaterally (Figure 1A", "Video 1)", "A modified barium swallow study showed intermittent laryngeal penetration without aspiration (Video 2)", "Although the patient was unable to tolerate magnetic resonance imaging, a soft tissue computed tomography scan of the neck with contrast revealed a well-positioned plate with no evidence of fluid collection in the setting of a normal leukocyte count", "One month later, a neuromuscular specialist investigated potential underlying neurological causes", "however, results of nerve conduction, antiacetylcholine receptor, antimuscle-specific kinase, methylmalonic acid, HIV, rapid plasma reagin, vitamin B-12, thyroid stimulating hormone, thyroxine, copper, creatine kinase, and immunofixation studies were all unremarkable", "Maximum abduction of vocal folds on laryngoscopic examination 11 months after placement of anterior cervical spinal plate (A) and 2 months after removal (B)"], "s1": [0, 8, 9], "s2": [1, 2, 3, 4, 5, 6, 7, 10]} {"key": 515, "questions": "What Is Your Diagnosis?", "options": [{"label": "A", "disease": "Cutaneous sarcoidosis"}, {"label": "B", "disease": "Lymphomatoid granulomatosis"}, {"label": "C", "disease": "Blastic plasmacytoid dendritic cell neoplasm"}, {"label": "D", "disease": "Cutaneous angioimmunoblastic T-cell lymphoma"}], "answer_idx": "D", "symptoms": ["A 61-year-old man presented to our clinic with a 3-month history of rapidly growing and itchy cutaneous nodules all over the body", "He denied systemic complaints such as weakness, fever, night sweats, or weight loss", "Oral glucocorticoids did not show any therapeutic effect", "His medical history was remarkable for mild anemia because of untreated hemorrhoids", "Physical examination revealed numerous symmetrically distributed, well-defined, erythematous skin nodules on his back, buttocks, and limb extensors (Figure, A)", "Prominent lymphadenopathies on bilateral cervical, axillary, and inguinal lymph nodes were noted, with the largest lymph node measuring approximately 5 × 2 cm", "Laboratory tests were significant for atypical lymphocytes in the peripheral blood (count up to 6%), an elevated CD4+/CD8+ lymphocyte ratio (9.21", "normal range, 0.7-3.1), moderate anemia (hemoglobin level, 8.7 g/dL", "normal range, 11.5-15.0 g/dL [to convert to g/L, multiply by 10.0]), a high erythrocyte sedimentation rate (35 mm/h", "normal range, 0-25 mm/h), and cryoglobulins composed of mixed polyclonal immunoglobulins", "The platelet count (161 × 103/μL", "normal range, 125-350 × 103/μL [to convert to ×109/L, multiply by 1.0]) and lactate dehydrogenase (217 U/L", "normal range, 100-240 U/L [to convert to μkat/L, multiply by 0.0167]) were normal", "Quantification of Epstein-Barr virus (EBV) DNA showed high loads of virus in the peripheral blood (846 copies/mL in serum, 27 700 copies/mL in lymphocytes", "normal range, <500 copies/mL)", "A biopsy specimen from a skin nodule was sent for histopathologic examination (Figure, B)", "A, Physical examination showed widespread, symmetrically distributed, well-defined nodules on the trunk and extremities", "B, Skin biopsy sample showed infiltration of atypical lymphocytes and histiocytes with proliferation of high endothelial venules (hematoxylin-eosin)", "C, In situ hybridization was positive for Epstein-Barr virus in the skin"], "s1": [0, 4, 16, 1, 6, 7, 8, 9, 10, 11, 12], "s2": [5, 13, 3, 17, 18, 15, 14, 2]} {"key": 516, "questions": "What Is Your Diagnosis?", "options": [{"label": "A", "disease": "Verrucous xanthoma"}, {"label": "B", "disease": "Colloid milium–classic adult type"}, {"label": "C", "disease": "Nodular amyloidosis"}, {"label": "D", "disease": "Nodular colloid degeneration"}], "answer_idx": "D", "symptoms": ["A man in his 80s with a medical history of gastric adenocarcinoma treated with total gastrectomy presented with asymptomatic lesions on his tongue that gradually developed during the previous 3 months", "Intraoral examination revealed multiple yellowish to erythematous papules and verrucous cobblestone-like plaques on the mucosal surface on both sides of his tongue (Figure, A and B)", "The skin of the face, neck, arms, and other sun-exposed areas did not reveal any associated lesions", "He reported occasional alcohol intake, and there was no history of tobacco-related habits or oral trauma", "The results of laboratory investigations, including a complete blood cell count, serum chemistry panel, liver and kidney function test, C-reactive protein levels, lipid profile, and serum and urine electrophoresis disclosed no abnormalities", "With the clinical suspicion of squamous cell carcinoma, an incisional biopsy was performed (Figure, C and D)", "Yellowish plaques in a cobblestone pattern on the right (A) and left (B) side of the tongue", "C and D, Circumscribed eosinophilic, amorphous fissured masses in the subepithelial connective tissue (hematoxylin-eosin)"], "s1": [0, 1, 5, 6, 7], "s2": [2, 3, 4]} {"key": 517, "questions": "What Is Your Diagnosis?", "options": [{"label": "A", "disease": "Chromoblastomycosis"}, {"label": "B", "disease": "Cutaneous cryptococcosis"}, {"label": "C", "disease": "Cutaneous protothecosis"}, {"label": "D", "disease": "Cutaneous leishmaniasis"}], "answer_idx": "C", "symptoms": ["A man in his 70s presented with a 6-month history of a rash on the left forearm, which gradually increased in severity", "The patient developed a rash on the dorsal aspect of his hand half a year ago without any pain or itching discomfort and denied any history of injury or exposure to suspected pathogens prior to onset", "After self-medication with topical herbal medicine, the lesions gradually spread to the forearm and fused into plaques, and ulcers often recurred on the surface", "He denied any systemic symptoms and concerns", "Physical examination revealed large, firm, mildly swollen erythematous plaques on the back of the left forearm with purulent discharge and crusting (Figure, A)", "Results of routine laboratory investigations, including complete blood cell count, biochemical profile, and chest radiography, were normal", "A biopsy specimen was taken for pathological examination (Figure, B-D)", "A, A large, firm, mildly swollen erythematous plaque with purulent discharge and crusting on the back of the left forearm", "B, Hyperkeratosis, pseudoepitheliomatous hyperplasia, and lymphohistiocytic infiltrate with neutrophils, plasma cells, and a small amount of multinucleated giant cells in the dermis (hematoxylin-eosin)", "C, Periodic acid–Schiff staining", "D, Transmission electron microscopy"], "s1": [0, 1, 2, 3, 4], "s2": [5, 6, 7, 8, 9, 10]} {"key": 518, "questions": "What Is Your Diagnosis?", "options": [{"label": "A", "disease": "Portal hypertensive gastropathy"}, {"label": "B", "disease": "Immunotherapy-related gastritis"}, {"label": "C", "disease": "Infectious gastritis"}, {"label": "D", "disease": "Liver cancer progression"}], "answer_idx": "B", "symptoms": ["A 66-year-old woman with chronic hepatitis B infection and hepatocellular carcinoma (HCC) presented with subacute epigastric pain", "Three years prior, she was diagnosed with a solitary HCC and underwent surgical resection with curative intent", "Pathology revealed poorly differentiated HCC, 16 cm in largest dimension, with large vein involvement (T3bNXM0, R0 resection)", "The patient underwent expectant observation", "One and a half years prior to the current presentation, laboratory results revealed an elevated α-fetoprotein (AFP), and cross-sectional imaging revealed multiple lung nodules consistent with metastases", "She was treated with nivolumab every other week and attained a biochemical (baseline, 212.2 ng/dL", "best response, 3.3 ng/dL) and radiographic complete response", "In the current presentation, the patient reported nonradiating epigastric pain of moderate intensity for 1 month", "Associated symptoms included nausea, anorexia, and water brash without emesis", "Review of systems was positive for unchanged chronic abdominal bloating and negative for other constitutional or gastrointestinal symptoms", "Laboratory results, including liver function tests, amylase, lipase, and AFP, were within normal limits, and hepatitis B virus polymerase chain reaction was undetectable", "Computed tomography with multiphasic liver protocol showed new enlarged mesenteric nodes and diffuse thickening of the gastric body (Figure, A)", "Esophagogastroduodenoscopy (EGD) identified diffuse inflammation and ulcerations in the gastric body and antrum (Figure, B)", "Biopsy specimens showed chronic and active mixed inflammatory infiltrate, including intraepithelial lymphocytosis (Figure, C)", "No overt viral cytopathic effects were observed, and immunostaining results for Helicobacter pylori, cytomegalovirus, and adenovirus were negative", "A, Coronal view of restaging computed tomography of chest, abdomen, and pelvis with contrast", "The yellow circle notes borderline enlarged mesenteric nodes inferior to the stomach, and the yellow arrowheads show thickened gastric body wall", "B, Gross appearance of the gastric mucosa visualized via upper endoscopy, which showed diffuse inflammation and ulceration in the gastric body and antrum", "C, Biopsy of the gastric mucosa showed marked chronic and active intraepithelial infiltration of lymphocytes (blue arrowheads) and neutrophils (yellow arrowhead) and associated glandular epithelial injury with withered glands and gland dropout (red circle) (original magnification ×100)"], "s1": [0, 1, 2, 3, 4, 5, 6, 7, 8, 9, 10], "s2": [11, 12, 13, 14, 15, 16, 17, 18]} {"key": 519, "questions": "What Is Your Diagnosis?", "options": [{"label": "A", "disease": "Cholesteatoma"}, {"label": "B", "disease": "Osteoma"}, {"label": "C", "disease": "Tophaceous gout"}, {"label": "D", "disease": "Tympanosclerosis"}], "answer_idx": "C", "symptoms": ["A 65-year-old patient was referred to the ear, nose, and throat department with left-sided hearing loss during the past 6 months without evident cause", "He had a medical history of myocardial infarction, sleep apnea, Ménière disease, and congenital cataract", "Furthermore, he did not report otalgia, otorrhea, a recent episode of Ménière disease, or other ear, nose, and throat or systemic symptoms", "The patient used atorvastatin and acetylsalicylic acid, did not smoke, and reported drinking 2 glasses of wine daily", "Otoscopy of the left ear revealed a yellow-whitish calcareous lobular mass located in the mesotympanum anterior to the malleus with an intact tympanic membrane", "The lesion was hard and painful during palpation (Figure 1A)", "Otoscopy results for the right side were normal", "Conductive hearing loss of the left ear because of a middle ear mass, seen on otoscopy (A) and on computed tomography (CT) (B)", "Pure-tone audiometry results showed a left-sided mixed hearing loss with a pure tone average of 110 dB HL and bone conduction levels at approximately 60 dB HL", "The tympanogram showed a normal A-type curve with a normal to low middle ear compliance of 0.5 mm on the left side", "Computed tomography (CT) imaging of the middle ear showed a heterogeneous hyperdense round mass of 5.5 mm in the left middle ear with a close relation to the malleus, eardrum, and tegmen (Figure 1B)", "There were no other abnormalities in the mastoid, ossicular chain, facial nerve canal, or surrounding blood vessels"], "s1": [0, 1, 2, 3], "s2": [4, 5, 6, 7, 8, 9, 10, 11]} {"key": 520, "questions": "What Is Your Diagnosis?", "options": [{"label": "A", "disease": "Thymic cyst"}, {"label": "B", "disease": "Foregut duplication cyst"}, {"label": "C", "disease": "Hemangioma"}, {"label": "D", "disease": "Lymphatic malformation"}], "answer_idx": "B", "symptoms": ["A 6-month-old girl presented to the emergency department for evaluation of fever and was noted to have mild inspiratory stridor", "Her parents stated that her stridor began acutely at 4 months of age without any inciting illness or event", "The stridor was worse with agitation but was without positional exacerbation", "The family reported that the stridor had remained stable since onset", "Her parents noted mild dysphagia but denied shortness of breath, cyanosis, or apnea", "Laboratory test results revealed acute COVID-19 infection", "Physical examination was unremarkable other than mild inspiratory stridor", "Findings from bedside flexible laryngoscopy revealed intact vocal fold motion bilaterally with glottic competence", "No supraglottic or glottic masses were identified", "Anterior-posterior neck and chest radiography demonstrated narrowing and tortuosity of the trachea with displacement to the right proximal to the thoracic inlet and diffuse patchy heterogenous airspace opacities (Figure, A)", "Subsequent neck ultrasonography and computed tomography scan demonstrated a 3.2 × 2.0-cm low-density lesion abutting the trachea and hypopharynx at the level of thoracic inlet, with rightward displacement of the trachea and larynx (Figure, B-D)", "Operative laryngoscopy, bronchoscopy, and esophagoscopy revealed extrinsic compression of the trachea, right mainstem bronchus, and esophagus with rightward deviation of the trachea and esophagus", "No mucosal tract was identified", "A, Narrowing and tortuosity of the trachea with rightward deviation on anterior-posterior neck radiograph (arrowhead delineates tracheal deviation)", "B, Echogenic cystic lesion on ultrasonogram", "C and D, Low-density lesion abutting the trachea and pharynx with rightward deviation of the trachea and larynx on coronal and axial neck computed tomography (black arrowheads delineate tracheal deviation", "yellow arrowheads depict the lesion)"], "s1": [0, 1, 2, 3, 4, 5, 6], "s2": [7, 8, 9, 10, 11, 12, 13, 14, 15, 16]} {"key": 521, "questions": "What Is Your Diagnosis?", "options": [{"label": "A", "disease": "Pilomatricoma"}, {"label": "B", "disease": "Epidermoid cyst"}, {"label": "C", "disease": "Type 1 first branchial cleft cyst"}, {"label": "D", "disease": "Hemangioma"}], "answer_idx": "C", "symptoms": ["A 15-year-old girl presented to the otorhinolaryngologic department for a right-side postauricular mass", "The mass had been present since birth but had slowly grown larger in the previous 2 years", "On physical examination, a 2.5 × 1.5-cm, nontender, blue, soft, cystic mass was observed in the right postauricular area", "There were no fistular openings around the auricle or in the external auditory canal (EAC), and there were no additional cysts, sinuses, or fistulas at the right neck from the hyoid bone and chin to the EAC", "On temporal bone computed tomography, a 2.5 × 1.5-cm, hypodense, oval-shaped cystic mass was revealed at the subcutaneous fat layer superoposterior to the right auricle", "On temporal magnetic resonance imaging (MRI), an oval cystic mass was revealed at the same location, with high signal intensity on both T1-weighted and T2-weighted images and low signal intensity on enhanced T1-weighted fat suppression and T2-weighted short T1 inversion recovery images (Figure, A-D)", "In surgery, the well-demarcated cystic mass was completely excised", "Yellowish fluid leaked out of the cyst during excision", "On histopathologic examination, the cyst was lined by keratinizing stratified squamous epithelium with hair follicles and sebaceous glands", "The cyst contained keratinous debris (Figure, E)", "There was no recurrence 6 months after surgery", "A-D, Magnetic resonance imaging showed a 2.5 × 1.5-cm, oval cystic mass at the subcutaneous fat layer superoposterior to the right auricle with high signal intensity on both T1-weighted and T2-weighted images and low signal intensity on enhanced T1-weighted fat suppression and T2-weighted short T1 inversion recovery images (in order from the left)", "E, Hematoxylin-eosin staining showed a cyst composed of keratinizing stratified squamous epithelium, hair follicles, and sebaceous glands"], "s1": [0, 1, 2, 3], "s2": [4, 5, 6, 7, 8, 9, 10, 11, 12]} {"key": 522, "questions": "What Is Your Diagnosis?", "options": [{"label": "A", "disease": "Laryngopharyngeal reflux"}, {"label": "B", "disease": "Laryngeal sarcoidosis"}, {"label": "C", "disease": "Granulomatosis with polyangiitis"}, {"label": "D", "disease": "Laryngeal tuberculosis"}], "answer_idx": "B", "symptoms": ["A 28-year-old White woman with no remarkable medical history presented with difficulty phonating and singing for the past 15 months", "She also reported odynophonia, globus sensation, and mild dysphagia to solids", "The patient had been treated previously with steroids and amoxicillin for the presumptive diagnosis of chronic laryngitis but reported no improvement in symptoms", "Laboratory workup was positive for elevated antinuclear antibodies and erythrocyte sedimentation rate and was consequently referred to our institution", "On presentation, the patient had only minimal dysphonia", "She underwent a transnasal flexible laryngoscopy that demonstrated a severely edematous epiglottis, posterior pharyngeal wall, aryepiglottic folds, and arytenoids (Figure, A)", "The vocal folds were difficult to visualize but seemed normal in appearance and mobility", "Stroboscopy could not be performed because of obstruction from the prolapsing arytenoids", "Interestingly, the patient denied any shortness of breath", "Results of laboratory testing revealed normal levels of all components of a comprehensive metabolic panel, thyroid-stimulating hormone, and angiotensin-converting enzyme", "The patient’s erythrocyte sedimentation rate was elevated to 75 mm/h (reference range, 0-15 mm/h)", "Findings of chest radiography were normal", "An esophagram showed no evidence of reflux esophagitis and a computed tomography of the neck showed edema of the supraglottis with some edema in the posterior hypopharyngeal wall", "A, Awake flexible laryngoscopy image of the edematous epiglottis, arytenoids, and posterior pharyngeal wall", "B, High-magnification of hematoxylin-eosin staining reveals well-formed noncaseating granulomas (arrowheads) with lymphoid reaction visible around the granulomas", "Subsequently, the patient underwent a direct laryngoscopy with laryngeal biopsy", "Pathology results demonstrated hyperplastic squamous mucosa with submucosal chronic inflammation and fibrosis and were negative for high-grade dysplasia and carcinoma", "scattered nonnecrotizing granulomas in the subepithelium were also observed (Figure, B)"], "s1": [0, 1, 2, 4, 5, 6, 8, 11], "s2": [3, 7, 9, 10, 12, 13, 14, 15, 16, 17]} {"key": 523, "questions": "What Is Your Diagnosis?", "options": [{"label": "A", "disease": "Prurigo nodularis"}, {"label": "B", "disease": "Acquired perforating dermatosis"}, {"label": "C", "disease": "Epidermolysis bullosa pruriginosa"}, {"label": "D", "disease": "Pemphigoid nodularis"}], "answer_idx": "D", "symptoms": ["An otherwise healthy man in his 60s presented to our department with an 8-month history of widespread pruritic papulonodular skin lesions on his extremities and trunk", "The patient had been initially diagnosed with nodular prurigo at a local hospital and treated with various topical steroids and oral antihistamines for 2 months, without significant improvement", "The skin lesions continued to progress and worsened after scratching", "Physical examinations revealed disseminated erythematous papules, hypertrophic plaques approximately 1 to 2 cm in diameter, and dome-shaped nodules with excoriated whitish or pink centers on the scalp, chest, abdomen, back, waist, and extremities (Figure, A and B)", "Subsequent examinations of the mucosal membranes appeared normal", "Other physical examinations as well as his medical and family histories were unremarkable", "Peripheral eosinophils, serum total immunoglobulin E level, enzyme-linked immunosorbent assay (ELISA) test for HIV, and results of other routine laboratory tests were within normal ranges or negative", "A biopsy specimen was taken from one of the skin lesions on his right forearm for histopathological examination and direct immunofluorescence (DIF) testing (Figure, C and D)", "A and B, Clinical images showed disseminated erythematous papules, hypertrophic plaques approximately 1 to 2 cm in diameter, and dome-shaped nodules with excoriated whitish or pink centers on the back, waist, and extremities", "C, A biopsy specimen taken from the right forearm showed marked hyperkeratosis, acanthosis with irregular elongation of the rete ridges, fibrotic dermis, and perivascular lymphocytic infiltrate (hematoxylin-eosin)", "D, Direct immunofluorescence revealed linear deposition of immunoglobulin G along the basement membrane zone"], "s1": [0, 1, 2, 3, 4, 5], "s2": [6, 7, 8, 9, 10]} {"key": 524, "questions": "What Is Your Diagnosis?", "options": [{"label": "A", "disease": "Multifocal infantile hemangioma"}, {"label": "B", "disease": "Bacillary angiomatosis"}, {"label": "C", "disease": "Multifocal lymphangioendotheliomatosis with thrombocytopenia"}, {"label": "D", "disease": "Congenital disseminated pyogenic granuloma"}], "answer_idx": "D", "symptoms": ["A 3-month-old boy was referred for evaluation of multiple cutaneous red papules", "The guardian reported first noticing the papules at birth", "During the following months, the papules grew in number and size, involving the head and neck, trunk, extremities, right palm, mons pubis, and buttocks", "At the height of severity, the patient had 39 lesions, some of which were associated with ulceration and bleeding", "The patient was born at full term via spontaneous vaginal delivery", "the birth mother had not received prenatal care", "Neither the birth mother nor the infant had any known history of environmental or occupational exposure to hazardous substances", "The paternal family medical history was unremarkable", "maternal family medical history was unknown", "Physical examination revealed numerous bright red macules, papules, and plaques ranging in size from 1.0 × 1.0 mm to 2.2 × 2.3 cm, many of which had hemorrhagic crust and dark scale (Figure, A and B)", "Additional examination findings included retrognathia, thickened maxillary frenulum, and anterior ankyloglossia", "A, Irregularly shaped red vascular plaque (2.2 × 2.3 mm) with overlying scale, dark crusting, and a dark blue component with surrounding (1.0-3.0 mm) red papules", "B, Numerous bright-red macules and papules (1 × 1 mm-3 × 3 mm) and 1 vascular red nodule (1.6 × 1.1 cm) on the left lateral neck with dark overlying crusting", "C, Low-power view of histopathology findings demonstrating epidermal hyperplasia and an upper dermal vascular proliferation (hematoxylin-eosin section)"], "s1": [0, 1, 2, 3, 9, 11, 12, 13], "s2": [4, 5, 6, 7, 8, 10]} {"key": 525, "questions": "What Is Your Diagnosis?", "options": [{"label": "A", "disease": "Myasthenia gravis"}, {"label": "B", "disease": "Holmes Adie pupil"}, {"label": "C", "disease": "Posterior communicating artery aneurysm that is not picked up on CT angiography"}, {"label": "D", "disease": "Neurovascular conflict of the oculomotor nerve"}], "answer_idx": "D", "symptoms": ["A 37-year-old Asian nonsmoking man with no known medical or drug history presented with an insidious onset of left-eye ptosis (only brought to attention by his friends on review of recent photographs) and subsequent painless vertical diplopia for 1 year", "He did not complain of headaches, facial numbness, dysarthria, facial droop, dysphagia, or limb weakness or numbness", "There was no history of joint pain or constitutional symptoms", "Neurological examination revealed a nonfatigable partial left-eyelid ptosis, left-eye hypotropia at primary position, and impaired left eye supraduction (Figure 1) with a nonreactive left pupil on direct and consensual light reflexes", "There were no other cranial nerve deficits", "Limb reflexes, motor testing, and sensory testing results were normal, and the plantar responses were flexor", "Top left (looking right and up), middle left (looking right), bottom left (looking right and down), center (primary position), top right (looking left and up), middle right (looking left), and bottom right (looking left and down)", "The images demonstrate left-eye hypotropia and left-eyelid ptosis at the primary position and impaired left-eye supraduction worse at lateral gaze on extraocular movements", "Laboratory evaluation showed a normal fasting serum glucose level, thyroid function test result, and erythrocyte sedimentation rate", "Anti–double-stranded DNA, antinuclear antibodies, and HIV test results were negative", "Repetitive nerve stimulation did not show any decremental response and anti-acetylcholine receptor antibodies were negative", "A computed tomography (CT) angiography of the circle of Willis did not show any aneurysms and a routine magnetic resonance imaging (MRI) of the brain did not show any infarcts or masses", "Lumbar puncture was acellular with normal glucose and protein levels, and multiple microbiological test results were negative", "Posterior communicating artery aneurysm that is not picked up on CT angiography"], "s1": [0, 3, 7, 13], "s2": [1, 2, 4, 5, 6, 8, 9, 10, 11, 12]} {"key": 526, "questions": "What Is Your Diagnosis?", "options": [{"label": "A", "disease": "Myopericytoma"}, {"label": "B", "disease": "Synovial sarcoma"}, {"label": "C", "disease": "Neurofibroma"}, {"label": "D", "disease": "Solitary fibrous tumor"}], "answer_idx": "D", "symptoms": ["An 81-year-old woman presented with a 3-month history of a slow-growing mass in her left cheek", "She denied having any symptoms, tenderness, numbness, or facial nerve weakness", "Physical examination revealed a mass in the buccinator space that was smooth, was nontender to palpation, and had no overlying cutaneous changes", "A contrast-enhanced maxillofacial computed tomography scan demonstrated a 1.8 × 1.8 × 1.9-cm rounded, well-circumscribed lesion at the anterior aspect of the left masticator space (Figure 1)", "The mass was abutting the parotid duct, which did not appear dilated, and appeared to have a peripheral rim of hyperenhancing soft tissue with a central cystic/necrotic component", "Results of ultrasound-guided fine-needle aspiration revealed rare epithelial cells with degenerative changes and inflammatory cells present", "Findings from a core biopsy demonstrated a spindle cell neoplasm of uncertain origin", "On immunostaining, spindle cells were CK8/18, S100, and SOX10 negative", "myosin and smooth muscle actin (SMA) negative", "and CD31 negative", "However, STAT6 staining was diffusely positive", "Axial view of the contrast-enhanced maxillofacial computed tomography scan demonstrating the left cheek mass (arrowhead)"], "s1": [0, 1, 2, 4, 11], "s2": [3, 5, 6, 7, 8, 9, 10]} {"key": 527, "questions": "What Is Your Diagnosis?", "options": [{"label": "A", "disease": "Facial nerve schwannoma"}, {"label": "B", "disease": "Vestibular nerve schwannoma"}, {"label": "C", "disease": "Neuromuscular choristoma"}, {"label": "D", "disease": "Hemangioma"}], "answer_idx": "C", "symptoms": ["A 21-year-old man was referred to our tertiary care hospital with a congenital left hemifacial weakness and a chronic unilateral left-sided hearing loss that evolved since birth", "On audiometry, normal hearing sensitivity in the right ear and a severe to profound sensorineural hearing loss with poor word recognition score in the left ear were noted", "On physical examination, the patient presented a grade VI peripheral left facial paralysis", "Findings from the rest of the clinical examination were within normal limits", "Results of cerebral magnetic resonance imaging (MRI) and temporal bone computed tomography (CT) revealed a left internal auditory canal mass, measuring 13×12 mm, developed at the expense of nerve VII, with a dilation of the various portions of the facial nerve with extension to its distal branches", "It presented a hypointensity on T2, isointensity on T1, and was strongly enhanced after the injection of gadolinium", "This mass was responsible for the enlargement of the internal auditory canal and was in close contact with the left middle cerebral fossa, without intraparenchymal extension (Figure 1A and B)", "A, Cerebral MRI T1-weighted contrast-enhanced image showing the mass", "B, Temporal bone CT scan showing the mass", "Arrowheads indicate the abnormal facial nerve", "CT denotes computed tomography, and MRI, magnetic resonance imaging"], "s1": [0, 1, 2, 3], "s2": [4, 5, 6, 7, 8, 9, 10]} {"key": 528, "questions": "What Is Your Diagnosis?", "options": [{"label": "A", "disease": "Pyoderma gangrenosum"}, {"label": "B", "disease": "Lupus vulgaris"}, {"label": "C", "disease": "Coccidioidomycosis"}, {"label": "D", "disease": "Granulomatosis with polyangiitis"}], "answer_idx": "A", "symptoms": ["A woman in her late 70s with a history of immunoglobulin A monoclonal gammopathy of unknown significance presented with a tender, draining lesion of the central face despite several courses of treatment with antibiotics (Figure, A)", "The patient lived in Southern California but had visited Kenya for an extended trip a few years prior", "Review of systems was negative for fevers, night sweats, weight loss, headache, vision changes, respiratory distress, or arthralgias", "On the left nasal dorsum and medial cheek, there was a 3 × 4 cm violaceous plaque with focal purulent drainage", "Within several weeks, the lesion rapidly expanded into a vegetative, freely draining plaque covering the cheeks and glabella and approaching the medial canthi (Figure, B)", "Computed tomography revealed findings consistent with abscess that did not involve the orbits, sinuses, or bone", "Chest radiography results were unremarkable", "Punch biopsy specimens were obtained for histopathological analysis and tissue cultures (Figure, C)", "Tissue culture results at 2 weeks remained negative, and a QuantiFERON-TB Gold test result was negative", "A, Violaceous plaque on the nose and left cheek", "B, Rapid progression of the lesion involved the cheeks and glabella, with substantially worsening drainage at 6-week follow-up", "C, Punch biopsy of lesion showing suppurative and granulomatous dermatitis (hematoxylin-eosin)", "D, Posttreatment, there was improvement of the lesion from initial presentation, but substantial residual facial scarring remained"], "s1": [0, 1, 3, 4, 9, 10, 12], "s2": [2, 5, 6, 7, 8, 11]} {"key": 529, "questions": "What Is Your Diagnosis?", "options": [{"label": "A", "disease": "Myotonic dystrophy"}, {"label": "B", "disease": "Systemic sclerosis"}, {"label": "C", "disease": "Deep morphea (morphea profunda)"}, {"label": "D", "disease": "Eosinophilic fasciitis"}], "answer_idx": "D", "symptoms": ["A 31-year-old man with insidious onset of limited mobility of joints visited our clinic for evaluation", "Six months prior, he found his palms could not make full contact with each other (the “prayer sign”)", "He had not been able to fully clench his fists for 2 months or squat for the past 10 days", "No erythema, edema, or sclerotic skin plaque had been found on his limbs or trunk", "No Raynaud phenomenon had occurred", "His medical and family histories were unremarkable", "On examination, he showed bilateral limited mobility of interphalangeal (hands), metacarpophalangeal, wrist, interphalangeal (feet), metatarsophalangeal, and ankle joints (Video)", "No muscular atrophy or sclerodactyly was found", "The prayer sign (Figure 1A) and the “groove sign” (Figure 1B) were observed", "Blood test results indicated eosinophilia (absolute eosinophil count, 1300 μL", "reference, <500μL [to convert to ×109/L, multiply by 0.001]), elevated erythrocyte sedimentation rate (ESR, 31 mm/h", "reference, <15 mm/h) and C-reactive protein (CRP, 3.01 mg/dL", "reference, <0.8 mg/dL [to convert to mg/L, multiply by 10])", "Liver and kidney function tests, creatine kinase level, electrolyte level, thyroid function, globulin concentration, and antinuclear antibody (ANA) level were unremarkable", "Electrocardiogram, cervical spine magnetic resonance imaging, and plain radiographs of hands, wrists, feet, and ankles were normal", "Nerve conduction studies performed in all 4 limbs were normal", "Electromyography (EMG) showed spontaneous activities in muscles of limbs, including fibrillation potentials and positive sharp waves (1+ or 2+)", "Myotonic discharges were seen in bilateral extensor digitorum", "A, The patient’s bilateral palms and fingers could not make full contact due to the limited mobility of interphalangeal and metacarpophalangeal joints", "B, Linear depression (arrowheads) was seen along the course of superficial vein when limb was elevated"], "s1": [0, 1, 2, 3, 4, 5, 6, 7, 8, 18, 19], "s2": [9, 10, 11, 12, 13, 14, 15, 16, 17]} {"key": 530, "questions": "What Is Your Diagnosis?", "options": [{"label": "A", "disease": "Intravascular anaplastic large cell lymphoma"}, {"label": "B", "disease": "Large B-cell lymphoma"}, {"label": "C", "disease": "Metastatic carcinoma"}, {"label": "D", "disease": "Natural killer/T-cell lymphoma"}], "answer_idx": "A", "symptoms": ["A 79-year-old female patient presented with a 2-year history of asymptomatic erythematous-infiltrated patches and plaques on the right side of her scalp and face", "The lesions gradually enlarged and coalesced into large plaques and nodules", "Mild erosions occurred occasionally on the surface of the lesions", "She denied having fever, fatigue, chills, night sweats, or weight loss", "Physical examination demonstrated ill-defined, indurated erythematous-violaceous patches, nodules, and plaques involving the surface of her head and face with sporadic erosion and brownish crusts (Figure, A)", "The findings of systemic reviews were unremarkable, and there was no lymphadenopathy", "The complete blood cell count revealed normal white blood cell count", "The biopsy from an erythematous nodule displayed sheets of medium to large mononuclear cells infiltrated in the dermis, some of which filled and expanded apparent dermal blood vessels and had large, irregular, pleomorphic nuclei (Figure, B)", "In addition, some neoplastic cells formed a balloon appearance", "No epidermotrophism was found", "The initial immunohistochemical stain revealed the tumor cells were positive for CD3 and negative for CD20, AE1/AE3, CK20, chromogranin A, and melan-A", "The Ki-67 labeling index was 70%", "A, Ill-defined, indurated erythematous-violaceous patches, nodules, and plaques on the head and face with sporadic erosion and brownish crusts", "B, The biopsy showed dilated vessels expanded by cohesive clusters of large atypical lymphoid cells (hematoxylin-eosin staining)", "C, Immunohistochemical staining of D2-40 highlighted the exclusively lymphovascular location of the lymphoma"], "s1": [0, 1, 2, 4, 12], "s2": [3, 5, 6, 7, 8, 9, 10, 11, 13, 14]} {"key": 531, "questions": "What Is Your Diagnosis?", "options": [{"label": "A", "disease": "Sarcoidosis"}, {"label": "B", "disease": "Granulomatosis with polyangiitis"}, {"label": "C", "disease": "Lepromatous leprosy"}, {"label": "D", "disease": "Progression of mycosis fungoides"}], "answer_idx": "C", "symptoms": ["A 66-year-old African American man presented to the otolaryngology clinic with a several-month history of severe nasal congestion", "Five years before this otolaryngology evaluation, the patient developed diffuse skin plaques with erythema and was subsequently diagnosed with hypopigmented cutaneous T-cell lymphoma (CTCL, mycosis fungoides type)", "The patient was treated with a regimen of narrowband UV-B phototherapy and triamcinolone cream, which he has continued for 4 years", "Owing to significant worsening of his symptoms 4 months before his evaluation, a biopsy of his cutaneous lesions was performed to rule out progression of CTCL (Figure 1)", "Physical examination demonstrated large annular plaques with peripheral hyperpigmentation of the body", "There were also confluent, indurated pink plaques with slightly raised edges along the forehead, malar eminence, and scalp", "The nasal septum demonstrated an inverted V deformity with compromise of the keystone junction", "Nasal passages were obstructed with thick mucus and synechiae formation, nearly obstructing the entire nasal passage bilaterally", "A, Fite acid-fast stain of cutaneous lesion identifying pink bacilli (original magnification ×100)", "B, Hematoxylin-eosin stain of the same cutaneous lesion showing poorly formed granulomas (original magnification ×40)"], "s1": [0, 6, 7], "s2": [1, 2, 3, 4, 5, 8, 9]} {"key": 532, "questions": "What Is Your Diagnosis?", "options": [{"label": "A", "disease": "Squamous cell carcinoma"}, {"label": "B", "disease": "Coccidioidomycosis"}, {"label": "C", "disease": "Tuberculosis"}, {"label": "D", "disease": "Recurrent respiratory papillomatosis"}], "answer_idx": "B", "symptoms": ["A 29-year-old Pakistani woman was referred to the otolaryngology clinic for increased postnasal drip, pharyngitis, odynophagia, cervicalgia, cervical lymphadenopathy, and right-sided otalgia that had persisted for more than a week", "She was breastfeeding her 10-month-old, had no history of alcohol use, and had a remote history of hookah use", "She was a Southern California native with travel history to Pakistan, Turkey, Jordan, Israel, Qatar, and Dubai 5 years prior to symptom onset", "One month prior to onset, she drove from Los Angeles, California, to Houston, Texas", "She denied recent unintentional weight loss, cough, fever, night sweats, chills, stridor, dyspnea, dysphonia, or rash", "On examination, her vital signs were within normal limits, and physical examination revealed a small right tonsillolith, mild tonsillar erythema that was worse on the right side, and a mobile 2-cm, right level 2 lymph node that was slightly tender to palpation", "Laboratory evaluation was notable for an elevated eosinophil count and positive Epstein-Barr immunoglobulin G", "Her white blood cell count and mononucleosis spot test result were within normal parameters", "Her symptoms persisted despite treatment with penicillin, azithromycin, and steroids from her primary physician and amoxicillin-clavulanate, sinus rinses, fluticasone propionate, and azelastine spray from an otolaryngologist to treat any contributing postnasal drainage and tonsillitis", "Flexible transnasal videolaryngoscopy revealed a lesion on the laryngeal surface of the right epiglottis passing midline extending to the right aryepiglottic fold, but not involving the true vocal cords (Figure 1A)", "Given inconclusive office biopsy results, direct laryngoscopy with biopsy was performed, showing extensively inflamed squamous mucosa with ulceration, granulation tissue, pseudoepitheliomatous hyperplasia, and atypia (Figure 1B)", "A, In-office laryngoscopy showing an ulcerative, papillomatous lesion on the laryngeal surface of the right epiglottis that passed midline and extended to the right aryepiglottic fold but did not involve the true vocal cords", "B, Laryngeal biopsy specimen demonstrating extensively inflamed squamous mucosa with ulceration, granulation tissue, pseudoepitheliomatous hyperplasia, and atypia, with background chronic granulomatous inflammation"], "s1": [0, 5, 8, 9, 10, 11, 12], "s2": [1, 2, 3, 4, 6, 7]} {"key": 533, "questions": "What Is Your Diagnosis?", "options": [{"label": "A", "disease": "Sarcoidosis"}, {"label": "B", "disease": "Blastomycosis"}, {"label": "C", "disease": "Squamous cell cancer"}, {"label": "D", "disease": "Tuberculosis"}], "answer_idx": "B", "symptoms": ["An otherwise healthy man in his 60s presented to a tertiary otolaryngology clinic reporting a 1-year medical history of progressively worsening hoarseness and dyspnea", "He denied any history of dysphagia, odynophagia, fevers, unintentional weight loss, cough, or reflux symptoms", "There was no prior history of smoking, head and neck surgeries, or radiation exposure", "He was treated for asthma with inhaled steroids by his primary care physician without improvement in symptoms", "On physical examination, the patient was stridulous and considerably hoarse", "His neck examination was unremarkable", "Subsequent videostroboscopy revealed severe laryngeal mucosal edema, most significant in the false vocal folds, interarytenoid, and postcricoid regions (Figure 1A)", "There was right true vocal fold immobility with compensatory squeeze of the false vocal folds and an absent mucosal wave", "Computed tomographic (CT) scan of the neck with contrast showed irregularity along the free edges of the true vocal folds but no evidence of a neoplasm or cervical lymphadenopathy", "The patient underwent urgent direct laryngoscopy with biopsy, which showed edematous, irregular, friable laryngeal mucosa with significant airway narrowing", "Laryngeal biopsy findings are shown in Figure 1, B and C", "A, Laryngeal videostroboscopic image", "B, Hematoxylin-eosin stain", "C, Grocott methenamine silver stain"], "s1": [0, 3, 4, 6, 7, 9], "s2": [1, 2, 5, 8, 10, 11, 12, 13]} {"key": 534, "questions": "What Is Your Diagnosis?", "options": [{"label": "A", "disease": "Multicentric reticulohistiocytosis"}, {"label": "B", "disease": "Juvenile hyaline fibromatosis"}, {"label": "C", "disease": "Farber disease (disseminated lipogranuloma)"}, {"label": "D", "disease": "Rheumatoid nodules"}], "answer_idx": "C", "symptoms": ["A 2-year-old female toddler born to nonconsanguineous parents presented with multiple skin-colored subcutaneous nodules that developed 40 days after birth", "The lesions started on the left foot and subsequently progressed to involve the face, trunk, and extremities, with distal interphalangeal joint deformity of both hands (Figure, A)", "The child had an older sibling with no skin ailments, and no other family members were affected", "Parents reported decreased oral intake, decreased activity, and delayed motor development", "A, Nodules on the right hand and joint deformity", "B and C, Histopathological images show a granulomatous pattern, numerous foamy cells, marked collagen degeneration, and mucin deposition (hematoxylin-eosin stain)", "D, Comma-shaped curvilinear tubular structures (yellow arrowhead)", "On examination, dozens of tough to hard subcutaneous skin-colored nodules ranging from 0.5 to 5 cm in diameter were observed on both sides of the nose and parts of both wrists, the knees, elbows, ankles and hips, and the trunk", "Symmetrical beaded subcutaneous nodules were observed in the bilateral auricles, while the mucous membrane was not involved", "The child weighed 8 kg (normal, 13-15 kg) and was 87 cm (normal, 91-95 cm) in length", "In addition, a physical examination revealed horizontal nystagmus and voice hoarseness", "The distal interphalangeal joints of both hands were stiff with limited extension", "The interphalangeal joints of the third through fifth distal segments of both feet were slightly bent", "A complete hematological and biochemical survey, including liver and renal function, C-reactive protein levels, thyroid function, rheumatoid factor, and antinuclear antibodies, identified no abnormalities", "A magnetic resonance imaging study of the brain showed volume loss in the parietooccipital region", "Ultrasonographic evaluation of metacarpophalangeal joints showed multiple hypoechoic nodules on the skin of the hands", "Radiography showed deformities of the interphalangeal joints with soft tissue swelling", "A punch biopsy was obtained from 1 of the nodules (Figure, B-D)"], "s1": [0, 1, 7, 8, 4, 11, 12, 15, 16], "s2": [2, 3, 9, 10, 13, 14, 5, 6, 17]} {"key": 535, "questions": "What Is Your Diagnosis?", "options": [{"label": "A", "disease": "Sarcoidosis"}, {"label": "B", "disease": "Lobomycosis"}, {"label": "C", "disease": "Keloidal scleroderma"}, {"label": "D", "disease": "Fibroblastic rheumatism"}], "answer_idx": "C", "symptoms": ["A man in his 60s with anti-RNA polymerase III antibody–positive systemic sclerosis complicated by scleroderma kidney crisis, heart failure, interstitial lung disease, sclerodactyly, and Raynaud phenomenon while receiving mycophenolate mofetil, 1 g twice daily, presented with a pruritic rash that had been present for 3 months", "The patient denied any known preceding triggers or trauma to the area of the rash", "He had tried treating with topical aloe, without improvement", "Review of systems was negative for any new or worsening constitutional, respiratory, or gastrointestinal symptoms", "The patient’s medical history was otherwise unremarkable", "Examination findings were notable for firm, red-brown plaques without overlying scale on the proximal upper extremities and anterior trunk (Figure, A and B)", "There was no appreciable skin tightening in the affected areas", "Complete blood cell count with differential was unremarkable", "Lesional punch biopsy specimens were obtained (Figure, C and D)", "A, Clinical image of firm, red-brown plaques without overlying scale on the proximal upper extremities and anterior trunk", "B, Clinical image of nodular plaques on the right shoulder and chest", "C and D, Lesional punch biopsy specimens (hematoxylin-eosin)"], "s1": [0, 1, 3, 4, 5, 6], "s2": [2, 7, 8, 9, 10, 11]} {"key": 536, "questions": "What Is Your Diagnosis?", "options": [{"label": "A", "disease": "Giant cell arteritis"}, {"label": "B", "disease": "Anti–transfer RNA synthetase myositis"}, {"label": "C", "disease": "Immunoglobulin G4–related disease"}, {"label": "D", "disease": "Myasthenia gravis"}], "answer_idx": "A", "symptoms": ["A 75-year-old woman presented with horizontal binocular diplopia, right-sided ptosis, and a new headache that was progressive over 3 days", "She reported difficulty opening her jaw, pain when chewing, and a 2.3-kg weight loss", "The week prior, she experienced left-sided ptosis that persisted for 2 days and subsequently resolved", "She denied vision changes, eye pain, scalp tenderness, and myalgias", "Past medical history included schizophrenia, hypothyroidism, and supraventricular tachycardia", "Her examination demonstrated normal visual acuity in both eyes", "Pupils were equal, round, and reactive", "There was right upper-eyelid ptosis", "Extraocular motility demonstrated limitation of elevation and abduction bilaterally (Figure 1)", "Results of dilated fundus examination were normal, and the remainder of her neurologic examination results were normal", "Her eye movements and ptosis did not change with fatiguability, rest, or an ice-pack test", "Investigations revealed a C-reactive protein level of 292 mg/L (normal, <10 mg/L", "to convert to milligrams per deciliter, divide by 10) and erythrocyte sedimentation rate of 93 mm/h (normal, <30 mm/h)", "A lumbar puncture demonstrated normal opening pressure, cell count, and glucose and protein levels", "Magnetic resonance imaging with contrast of the brain demonstrated diffuse edema and enhancement of the scalp, skull base, neck, and paraspinal soft tissues, as well as T2 hyperintensity and enhancement of the extraocular muscles bilaterally", "An additional test was obtained, and a diagnosis was made", "Extraocular movements on presentation", "Examination reveals deficits in elevation and abduction of both eyes not conforming to a cranial nerve distribution"], "s1": [0, 1, 2, 3, 7, 11, 12, 14, 15], "s2": [4, 5, 6, 8, 9, 10, 13, 16, 17]} {"key": 537, "questions": "What Is Your Diagnosis?", "options": [{"label": "A", "disease": "Eosinophilic sialodochitis"}, {"label": "B", "disease": "Sjogren syndrome"}, {"label": "C", "disease": "Sialolithiasis"}, {"label": "D", "disease": "Mikulicz syndrome"}], "answer_idx": "A", "symptoms": ["A 32-year-old Black woman presented with a 1-year history of facial swelling, predominantly in the right preauricular and left submandibular regions", "Her symptoms began after relocation to California and occurred more frequently at her home", "However, she could not discern any environmental or dietary triggers", "She reported pruritus and swelling, which were usually moderate but occasionally severe, prompting several emergency department visits", "During an emergency department visit, she was diagnosed with acute parotitis based on clinical and radiographic findings of diffuse gland edema, without salivary stones or masses", "Her symptoms responded to oral prednisone bursts and diphenhydramine, but not to second-generation antihistamines or montelukast", "Her workup included immune serology and total immunoglobulin (Ig) E measurements, all of which were within normal limits", "A differential blood cell count revealed 10% eosinophils (normal, <6%) and an absolute eosinophil count 500/µL (to convert to ×109/L, multiply by 0.001)", "Titers for antinuclear antibodies, rheumatoid factor, anti-Ro (SS-A), and anti-La (SS-B) were negative", "Sialendoscopy of her right parotid and left submandibular glands found no obstructing stones or strictures", "Concurrent steroid irrigation did not provide any substantial benefit", "An open biopsy of the right parotid tail revealed nonspecific inflammatory changes and otherwise normal parenchyma", "Given the frequency and severity of her symptoms along with multiple treatment failures, she opted to undergo a right superficial parotidectomy, which proceeded without complications", "Further histologic examination of the surgical specimen revealed substantial periductal inflammatory infiltrate with eosinophils, as seen in the Figure", "Medium-powered (A) and high-powered (B) field from the superficial parotid gland (hematoxylin-eosin)"], "s1": [0, 1, 2, 3, 4, 5, 12], "s2": [6, 7, 8, 9, 10, 11, 13, 14]} {"key": 538, "questions": "What Is Your Diagnosis?", "options": [{"label": "A", "disease": "Foregut duplication cyst"}, {"label": "B", "disease": "Branchial cleft cyst"}, {"label": "C", "disease": "Plunging ranula"}, {"label": "D", "disease": "Macrocystic lymphatic malformation"}], "answer_idx": "A", "symptoms": ["A 2-month-old, full-term female presented with a left upper neck mass that was first noted at birth", "Her parents reported no fevers, constitutional symptoms, or enlargement of the mass", "On physical examination, there was a nontender 4-cm mass in the left submandibular region with no overlying skin changes", "Ultrasonography results showed an avascular cystic structure with internal debris and a single thin septation", "Contrast-enhanced computed tomography (CT) of the neck soft tissue revealed a 3-cm low attenuation, nonenhancing, cystic-appearing mass that elevated the submandibular gland to the level of the parapharyngeal fat space with medial extension of the lesion into the tongue musculature", "The walls were thin except for mildly thickened capsule along the posterior aspect of the lesion (Figure, A and B)", "Fluid that aspirated from the cyst was serous, and cytology showed mature and anucleate squamous cells with acute inflammation", "The mass returned after 3 weeks", "Further analysis 8 months after the initial CT via contrast-enhanced magnetic resonance imaging (MRI) showed a large, multiseptated cystic structure (Figure, C)", "the capsule of the wall was mildly thickened and was contrast enhancing", "At age 12 months, the mass was excised via neck dissection", "A lateral component at the level of the sternocleidomastoid muscle was completely excised", "An intraoperative rupture of the thin capsule caused some spillage of cystic contents that were milky white in some areas and mucinous in others", "Contrast-enhanced computed tomography images", "A, Axial: the blue arrowhead indicates extension into the tongue musculature, the yellow arrowhead indicates focally thickened capsule posteriorly, and the white arrowhead indicates a displaced sternocleidomastoid muscle", "B, Coronal: the black arrowhead indicates an elevated left submandibular gland, and the white arrowhead indicates a normal right submandibular gland", "C, Axial T2-weighted magnetic resonance image showing marked interval enlargement of the multiseptated cystic mass", "Arrowheads indicate involvement of the tongue musculature"], "s1": [0, 1, 2, 6, 7, 10, 11, 12], "s2": [3, 4, 5, 8, 9, 13, 14, 15, 16, 17]} {"key": 539, "questions": "What Is Your Diagnosis?", "options": [{"label": "A", "disease": "Poikilodermatous plaque–like hemangioma"}, {"label": "B", "disease": "Acquired elastotic hemangioma"}, {"label": "C", "disease": "Microvenular hemangioma"}, {"label": "D", "disease": "Acroangiodermatitis"}], "answer_idx": "A", "symptoms": ["An otherwise healthy man in his early 70s presented to the Department of Dermatology with a 2-year history of a mildly pruritic, erythematous, and gradually enlarging lesion on the proximal portion of his left calf", "The lesion had appeared without an identifiable cause, and for the previous 2 months, the patient had applied topical corticosteroids and antifungals with no improvement", "Physical examination revealed an ill-defined, nonindurated, erythematous-violaceous plaque that was slightly raised but with an atrophic appearance (Figure, A)", "A skin biopsy was obtained for histopathology examination (Figure, B-D)", "A, Clinical image of the left calf showing an ill-defined, erythematous-violaceous plaque with an atrophic appearance", "B, Band-like vascular proliferation in the upper dermis with no grenz zone (hematoxylin-eosin)", "C, Round vascular channels lined by a single layer of endothelial cells without atypia (hematoxylin-eosin)", "D, CD34 immunostaining labels endothelial cells"], "s1": [0, 1, 2, 4], "s2": [3, 5, 6, 7]} {"key": 540, "questions": "What Is Your Diagnosis?", "options": [{"label": "A", "disease": "Extrapulmonary tuberculosis"}, {"label": "B", "disease": "Multicentric Castleman disease"}, {"label": "C", "disease": "Non-Hodgkin lymphoma"}, {"label": "D", "disease": "Sarcoidosis"}], "answer_idx": "B", "symptoms": ["A White man in his 70s with a medical history notable only for mitral valve repair was incidentally found to have cervical lymphadenopathy on magnetic resonance imaging while undergoing workup for shoulder pain", "A fine-needle biopsy of an enlarged left-sided cervical lymph node yielded benign results", "One month later, he presented to a local emergency department with diffuse abdominal pain", "Symptoms began 6 weeks earlier and had been intermittent in nature", "He had also experienced fatigue, night sweats, decreased appetite, and weight loss for 6 weeks", "He otherwise denied having fevers, myalgias, nausea, vomiting, diarrhea, or easy bruising", "Physical examination revealed bilateral cervical, axillary, and inguinal lymphadenopathy in addition to mild diffuse abdominal pain", "Laboratory results revealed a hemoglobin level of 13.1 g/dL (to convert to g/L, multiply by 10), a white blood cell count of 6.4 × 103/μL, and a platelet count of 404 × 103/μL (to convert to × 109/L, multiply by 1) in addition to negative results for HIV, hepatitis B virus, and hepatitis C assays", "Additional workup results were notable for elevated levels of interleukin (IL) 10 (1091 pg/mL), IL-6 (3.1 pg/mL), C-reactive protein (4.9 mg/dL [to convert to mg/L, multiply by 10]), and erythrocyte sedimentation rate (79 mm/h)", "A computed tomography scan of the neck, chest, abdomen, and pelvis further revealed extensive cervical, supraclavicular, retroperitoneal, pelvic, and inguinal lymphadenopathy, with extensive inflammation and fat stranding adjacent to these enlarged nodes", "Positron emission tomography–computed tomography further demonstrated fluorodeoxyglucose-avid lymph nodes above and below the diaphragm", "The patient then underwent excisional biopsy of an enlarged right cervical lymph node (Figure)", "A, Proliferation of slitlike blood vessels (hematoxylin-eosin)", "B, The lymphoid follicle shows regression, with onion-skinning of mantle zone lymphocytes and hypervascularity (hematoxylin-eosin)", "Scattered atypical plasma cells are present"], "s1": [0, 2, 3, 4, 5, 6, 7, 8], "s2": [1, 9, 10, 11, 12, 13, 14]} {"key": 541, "questions": "What Is Your Diagnosis?", "options": [{"label": "A", "disease": "Superficial lymphatic malformation"}, {"label": "B", "disease": "Primary superficial Ewing sarcoma"}, {"label": "C", "disease": "Lymphoblastic lymphoma"}, {"label": "D", "disease": "Primary cutaneous embryonal rhabdomyosarcoma"}], "answer_idx": "D", "symptoms": ["An adolescent girl presented with an asymptomatic slightly reddish verrucous plaque on the right temple", "The skin lesion was nail sized when first noticed 1½ years earlier, then gradually developed over the 8 months prior to presentation", "The patient denied any associated fevers, weight loss, or night sweats", "Physical examination revealed a slightly reddish verrucous mass with translucent papules and plaques on the right temple, about 5 cm in diameter, with surface hemorrhagic crust in the side adjacent to right eyebrow arch (Figure, A)", "The lesion was nontender with no purulent discharge and no severe necrotic changes", "There was no hepatosplenomegaly or superficial lymphadenopathy", "Laboratory tests, including blood cell counts, urinalysis, and kidney and hepatic panels, revealed no abnormalities", "A skin biopsy was performed and submitted for histopathologic analysis (Figure, B-D)", "A, A slightly reddish verrucous mass with translucent papules and plaques on the right temple", "B, The histological examination showed papillomatous epidermal hyperplasia, edema of dermal papillae, and neoplastic infiltration consisting of small round blue cells with alternating areas of dense and loose cellularity (hematoxylin-eosin)", "C, Small, round blue cells were observed in the dermis with round, fusiform, or oval hyperchromatic nuclei, conspicuous nucleoli, and scant cytoplasm (hematoxylin-eosin)", "D, Tumor cells were positive for MyoD1 (original magnification ×400)"], "s1": [0, 1, 2, 3, 4, 5, 6], "s2": [7, 8, 9, 10, 11]} {"key": 542, "questions": "What Is Your Diagnosis?", "options": [{"label": "A", "disease": "Chronic ulcerative herpes simplex virus"}, {"label": "B", "disease": "Epstein-Barr virus–positive mucocutaneous ulcers"}, {"label": "C", "disease": "Crohn disease associated with pyostomatitis vegetans"}, {"label": "D", "disease": "Ulcers secondary to fungal infection"}], "answer_idx": "B", "symptoms": ["A man in his 40s with a long-standing history of poorly controlled HIV (multiple treatment lapses, viral load of 22 871 copies/mL 8 months prior to presentation but now undetectable with use of highly active antiretroviral therapy, with a CD4 count of 40 cells/mm3) presented to the emergency department with nausea, abdominal pain, bloody stools, odynophagia, and recurrent oral and perianal ulcers and was found to have pancytopenia and septic shock", "During admission in the intensive care unit, treatment with broad-spectrum oral antibiotics for presumed infection and oral fluconazole for thrush led to improvement in the patient’s medical condition, but the ulcers persisted", "Dermatology was consulted for input on the cause of the ulcers, which had been ongoing for approximately 6 months prior", "individual lesions lasted 1 to 3 weeks before self-resolving", "Biopsy of an oral mucosal lesion performed 1 month earlier had shown nonspecific inflammation", "Examination revealed exquisitely tender, ovoid, punched-out ulcerations of the inferior lip and scrapeable white plaques on the tongue (Figure, A), right buccal mucosa, and external anal mucosa (Figure, B)", "A punch biopsy from 1 of the perianal ulcers was performed (Figure, C and D)", "A, Healing oval punched-out ulceration of inferior lip and white plaques on tongue consistent with thrush", "B, Two ovoid, punched-out ulcerations of the external anal mucosa", "C, Hematoxylin-eosin staining of perianal lesion showed mixed infiltrate of histiocytes, neutrophils, lymphocytes, and eosinophils, with prominent admixed large and atypical cells at high power", "D, Epstein-Barr virus–encoded RNA staining showed scattered positivity in some cells"], "s1": [0, 1, 2, 3], "s2": [4, 5, 6, 7, 8, 9, 10]} {"key": 543, "questions": "What Is Your Diagnosis?", "options": [{"label": "A", "disease": "Orbital compartment syndrome"}, {"label": "B", "disease": "Ocular motor apraxia"}, {"label": "C", "disease": "Pituitary apoplexy"}, {"label": "D", "disease": "Cavernous sinus thrombosis"}], "answer_idx": "C", "symptoms": ["A 50-year-old man was referred for ophthalmoplegia that developed immediately after an 8.5-hour robotic cystectomy for small cell cancer of the bladder", "After recovering from anesthesia in the postanesthesia care unit, he experienced headache, facial numbness, inability to open both his eyes, and inability to move both eyes in any direction", "He had a known enhancing central skull base lesion in sella turcica with compression of the optic chiasm and mild mass effect on the pons that was felt to be a pituitary macroadenoma (Figure)", "His baseline examination before the cystectomy showed visual acuity of 20/20 OU with intact extraocular movements and unremarkable confrontational fields, although a mild bitemporal hemianopia was seen on automated visual fields", "T1-weighted coronal magnetic resonance imaging (MRI) with contrast shows a 4.9 × 4.6 × 5.2-cm sellar/parasellar lobulated mass with homogenous enhancement before symptom onset (A), which became 5.3 × 5.1 × 5.3 cm and had loss of enhancement after symptom onset (B)", "The mass caused superior displacement of the optic chiasm and had extension into the cavernous sinus bilaterally", "The mass also extended posteriorly and deformed the anterior aspect of the pons (not shown)", "After symptom onset, the mass was hyperintense on T2-weighted images (not shown)", "During the bedside examination in the postanesthesia care unit, he was noted to have complete bilateral ptosis", "His visual acuity was 20/20 OU with full visual fields on confrontational visual field testing", "He had decreased sensation to light touch and pinprick in the V1/V2 distribution of the trigeminal nerve bilaterally", "The right and left pupils were 5 mm and 6 mm in size, respectively, in both light and dark", "There was no pupillary response to light or accommodation", "No exophthalmos was noted", "Intraocular pressure was normal in both eyes", "The patient was unable to move either eye in any direction on extraocular motility testing", "Vestibulo-ocular reflex did not overcome the ophthalmoplegia", "Magnetic resonance imaging (MRI) was obtained 12 hours after symptom onset redemonstrating the large sellar mass, which no longer had central enhancement (Figure)"], "s1": [0, 1, 8, 10, 11, 12, 13, 14, 15, 16], "s2": [2, 3, 4, 5, 6, 7, 9, 17]} {"key": 544, "questions": "What Is Your Diagnosis?", "options": [{"label": "A", "disease": "Superior semicircular canal dehiscence syndrome"}, {"label": "B", "disease": "Pneumolabyrinth"}, {"label": "C", "disease": "Labyrinthitis"}, {"label": "D", "disease": "Meniere disease"}], "answer_idx": "B", "symptoms": ["A 51-year-old woman presented with a 24-hour history of severe dizziness that she described as “room spinning,” right aural fullness, and a subjective right-sided hearing loss", "She denied any medical history of vertigo, recent upper respiratory infection, head trauma, or prior otologic surgery", "A physical examination revealed no abnormalities on otoscopic evaluation", "however, a strongly positive right-sided Dix−Hallpike test was elicited with rotary nystagmus", "a subsequent Epley maneuver was performed", "The patient’s symptoms subjectively worsened during the following several days", "Therefore, an audiogram test (Figure 1) and computed tomography (CT) of the temporal bone were performed", "Preoperative audiogram reveals a right-sided low- to mid-frequency moderate conductive hearing loss and a bilateral high-frequency sensorineural hearing loss", "dB denotes decibels", "HTL, hearing threshold level", "SL, sensation level", "SRT, speech reception threshold", "and TDH, telephonic dynamic headphones"], "s1": [0, 1, 3, 4, 5], "s2": [2, 6, 7, 8, 9, 10, 11, 12]} {"key": 545, "questions": "What Is Your Diagnosis?", "options": [{"label": "A", "disease": "Benign thyroid nodule"}, {"label": "B", "disease": "Parathyroid cyst"}, {"label": "C", "disease": "Reactive viral lymphadenopathy"}, {"label": "D", "disease": "Thyroglossal duct cyst"}], "answer_idx": "B", "symptoms": ["An 18-year-old male presented to the primary care physician with a 4-month history of a slowly progressive painless lateral neck mass on the right side", "The patient denied any history of dysphagia, dysphonia, drainage, previous neck surgery, trauma, or recent fever", "There was no significant medical or family history of thyroid disease", "On examination, the mass moved with swallowing", "The patient had normal thyroid stimulating hormone and calcium levels and a parathyroid hormone (PTH) level of 63 pg/mL", "Neck ultrasonography revealed a well-circumscribed anechoic lesion with a diameter of 5.9 cm in the right thyroid lobe without any internal solid nodularity or septation", "A neck computed tomography (CT) scan revealed an intrathyroidal lesion extending inferiorly to 3 cm above the angle of Louis causing tracheal compression and displacement (Figure, A)", "The patient underwent right hemithyroidectomy to relieve tracheal compression", "A near-infrared autofluorescence (NIRAF) camera yielded a homogenously dim lesion (Figure, B)", "Pathologic examination confirmed a normal right thyroid lobe, a normal right parathyroid gland, and a cystic lesion at the inferior pole with no evidence of malignant disease in the specimen", "The tissue inside the lesion stained positive for PTH", "A, A neck CT scan showing an intrathyroidal mass causing tracheal compression and is in close proximity to the right common carotid artery and right internal jugular vein", "B, A NIRAF image of the surgical field showing right thyroid lobe, a right parathyroid gland, and a homogenously dim capsulated lesion of interest"], "s1": [0, 1, 2, 3, 4], "s2": [5, 6, 7, 8, 9, 10, 11, 12]} {"key": 546, "questions": "What Is Your Diagnosis?", "options": [{"label": "A", "disease": "Primary parotid gland cancer"}, {"label": "B", "disease": "Parotid gland lymphoma"}, {"label": "C", "disease": "Parotid gland hemorrhage"}, {"label": "D", "disease": "Benign parotid gland tumor"}], "answer_idx": "C", "symptoms": ["A 53-year-old woman presented to the emergency department after awakening with right preauricular and right upper neck swelling", "She had a history of coronary artery disease and prior myocardial infarction that was treated with warfarin", "She denied any personal or family history of cancer", "She was previously briefly a smoker", "A physical examination revealed no erythema or tenderness of the parotid", "Her cranial nerves were intact, specifically the right facial nerve and its branches", "The oral mucosa was intact with no epithelial lesions", "Urinalysis results revealed slight hematuria", "Her international normalized ratio (INR) was 4.4. Contrast-enhanced computed tomography (CT) revealed an ill-defined infiltrative mass centered in the deep lobe of the right parotid with extension into surrounding soft tissue spaces (Figure, A)", "A follow-up CT was performed 6 weeks later (Figure, B)", "A, Axial contrast-enhanced computed tomography (CT) reveals an ill-defined, infiltrative lesion centered in the deep lobe of the right parotid gland", "B, Six weeks later"], "s1": [0, 4, 5, 6, 8, 10], "s2": [1, 2, 3, 7, 9, 11]} {"key": 547, "questions": "What Is Your Diagnosis?", "options": [{"label": "A", "disease": "Morphea"}, {"label": "B", "disease": "Lupus panniculitis"}, {"label": "C", "disease": "Lipodystrophia centrifugalis abdominalis infantilis"}, {"label": "D", "disease": "Subcutaneous fat necrosis of the newborn"}], "answer_idx": "C", "symptoms": ["A 2-year-old girl who was otherwise healthy presented with a 10-month history of asymptomatic skin depressions in her neck, nape, and submandibular regions", "The lesion initially occurred as an erythema in the neck and progressed centrifugally to the nape and submandibular areas", "Atrophy was observed in the center of the lesion, surrounded by an erythematous border, with underlying blood vessels being visible deep to the atrophy (Figure, A)", "A, Atrophy in the center of the lesion, surrounded by an erythematous border, with underlying blood vessels deep to the atrophy", "B, A biopsy specimen taken from the edge of the depressed lesion showing normal epidermis and dermis, no inflammatory cell infiltration within the dermis, and somewhat diminished subcutaneous fatty tissue (hematoxylin-eosin)", "C, The same specimen at a greater magnification (hematoxylin-eosin)", "D, Ultrasonographic findings showing markedly thinner subcutaneous fat of the skin (thickness of thinner region, 0.08 cm)", "Cervical lymph nodes were palpable bilaterally", "Tactile responses within the depressed as well as surrounding areas were normal", "Other physical examinations were without abnormalities", "Her medical and family histories were unremarkable", "Her parents denied drug injection or trauma history", "Results of routine laboratory tests, HIV test, serological test of antibodies for Borrelia, and antinuclear antibody test were within normal limits or negative", "After obtaining informed consent from her parents, a biopsy specimen was taken from the edge of the depressed lesion for histological examination"], "s1": [0, 1, 2, 3, 6, 7, 8], "s2": [4, 5, 9, 10, 11, 12, 13]} {"key": 548, "questions": "What Is Your Diagnosis?", "options": [{"label": "A", "disease": "Interstitial granulomatous dermatitis"}, {"label": "B", "disease": "Wolf isotopic response"}, {"label": "C", "disease": "Metastatic carcinoma"}, {"label": "D", "disease": "Cutaneous endometriosis"}], "answer_idx": "C", "symptoms": ["A woman in her 40s presented with painful erythematous plaques on her trunk (Figure, A)", "Skin lesions developed 2 months earlier and progressively spread", "She reported abdominal discomfort, weight loss, and vaginal bleeding", "A solid mass on her lower abdomen that had been palpable about 10 years prior had rapidly increased in size within the past few months", "Physical examination revealed erythematous plaques on the right chest and flank without warmth", "Punch biopsy of a lesion from the chest was sent for histopathological examination (Figure, B and C)", "A, Erythematous plaques involving the right chest and flank at the initial presentation", "B and C, Large atypical cells with clear or eosinophilic cytoplasm forming papillary and glandular architectures are present in the deep dermis", "lymphovascular invasion was observed in the superficial and deep dermis (hematoxylin-eosin)", "D, Contrast-enhanced computed tomographic scan of the abdomen and pelvis showing a large heterogeneous mass in the right rectus abdominis muscle (blue arrowhead) and cystic lesion in the left ovary (pink arrowhead)"], "s1": [0, 1, 4, 5, 6, 7, 8], "s2": [2, 3, 9]} {"key": 549, "questions": "What Is Your Diagnosis?", "options": [{"label": "A", "disease": "Progressive supranuclear palsy"}, {"label": "B", "disease": "Mitochondrial encephalopathy, lactic acidosis, and strokelike episodes"}, {"label": "C", "disease": "Autosomal recessive parkinsonism"}, {"label": "D", "disease": "Paraneoplastic syndrome"}], "answer_idx": "C", "symptoms": ["A man aged 44 years was referred for progressive parkinsonism with poor levodopa response", "His medical history comprised unexplained sensory hearing loss, hyperlipidemia, and osteoarthritis", "At age 23 years, he had bilateral cataract surgery, and over the last 2 years, he developed worsening stiffness and walking difficulties with frequent falls", "He reported features of rapid eye movement (REM) sleep behavior disorder, restless legs syndrome (RLS), visual daytime hallucinations, and rapid cognitive decline", "He was given levodopa/benserazide (up to 200 mg/50 mg 4 times daily) without improvement on his Unified Parkinson Disease Rating Scale scores", "Therefore, his dopaminergic medication was ceased", "Family history was negative for movement or developmental disorders", "His neurologic examination showed hypomimia, symmetrical bradykinesia, and rigidity of the extremities without tremors (Video)", "There was full range of eye movements with large-amplitude saccadic intrusions on smooth pursuit in all directions", "Neuropsychological assessment revealed significant abnormalities in cognitive functioning, affecting verbal and nonverbal cognitive skills (ie, speed of information processing, visuospatial skills, verbal and visual memory loss, executive skills)", "Mini-Mental Status Examination score was 26 of 30 points", "Systemic examination was unremarkable", "Laboratory and cerebral spinal fluid testing (ie, paraneoplastic antibodies Hu, Yo, Ri, CV2, Ma2, amphiphysin, glutamic acid decarboxylase, N-methyl-d-aspartic acid, LGI1, and Caspr2, 14-3-3 protein) were normal", "Cranial magnetic resonance imaging (MRI) showed mild frontal and parietal atrophy", "An electroencephalogram demonstrated slowing in keeping with moderate generalized encephalopathy", "Total-body positron emission tomography (PET) using fluorine 18 (18F) fluorodeoxyglucose (FDG) as a tracer showed no FDG-avid lesions", "however, a subsequent brain-specific FDG-PET/MRI revealed hypometabolism throughout the left cerebral hemisphere and crossed cerebellar diaschisis (Figure 1)", "As the patient’s cognitive function and parkinsonism had deteriorated rapidly, a brain biopsy of the left frontal cortex was done", "Immunohistochemistry showed scattered globular inclusions consistent with Lewy bodies", "A, Axial fluorine 18 (18F) fluorodeoxyglucose (FDG) positron emission tomography (PET) imaging shows reduced FDG uptake in the left frontal, temporal, and occipital lobes, as well as subtle hypometabolism in the left basal ganglia and left thalamus", "B, Crossed cerebellar diaschisis as a result of contralateral supratentorial asymmetry is seen in the coronal plane"], "s1": [0, 1, 2, 3, 6, 7, 8, 9, 10], "s2": [4, 5, 11, 12, 13, 14, 15, 16, 17, 18, 19, 20]} {"key": 550, "questions": "What Is Your Diagnosis?", "options": [{"label": "A", "disease": "Reactive lymphoid hyperplasia"}, {"label": "B", "disease": "Greater auricular nerve neuroma"}, {"label": "C", "disease": "Recurrent parotid tumor"}, {"label": "D", "disease": "Metastatic tumor deposit"}], "answer_idx": "B", "symptoms": ["A 63-year-old man presented with a history of previous left superficial parotidectomy 10 years before for a Warthin tumor", "His histology results showed a 5-cm Warthin tumor, with a focus of nodular oncocytic hyperplasia in the adjacent parenchyma and reactive intraparotid lymph nodes", "He presented to the ear, nose, and throat clinic and complained of pain in the operative site of 6 months duration", "This occurred intermittently once a week and lasted for a few minutes each time", "It was associated with a small lump over the left neck, and the pain was worse on touch and head turning", "On examination, there was a 3-mm nodularity palpated deep to the neck wound along the mid sternocleidomastoid muscle", "It was associated with focal tenderness", "His facial nerve function was otherwise normal, and sensation in the ear lobe was intact", "A magnetic resonance imaging (MRI) scan was performed, and the results showed no focal lesion in either parotid glands or at the area of interest over the left sternocleidomastoid muscle", "The patient was counseled for an excisional biopsy of the left neck nodule", "The intraoperative photograph of the neck and the excised specimen is shown in Figure 1.Intraoperative photograph of the left lateral neck and the excised specimen", "The black arrowhead indicates left neck lesion, the white arrowhead the external jugular vein"], "s1": [0, 1, 8, 9, 10, 11], "s2": [2, 3, 4, 5, 6, 7]} {"key": 551, "questions": "What Is Your Diagnosis?", "options": [{"label": "A", "disease": "Metastatic progression: proceed with irradiation"}, {"label": "B", "disease": "Radiation recall myelopathy: continue steroids, discontinue doxorubicin"}, {"label": "C", "disease": "Radiation necrosis: continue steroids, add bevacizumab"}, {"label": "D", "disease": "Spinal cord infarct: underlying cause workup, supportive care"}], "answer_idx": "B", "symptoms": ["A woman in her 60s was diagnosed with high-grade leiomyosarcoma of the uterus in 2017 for which she underwent total abdominal hysterectomy and bilateral salpingo-oophorectomy", "In 2020, she relapsed in the liver and spine", "In November 2020, she underwent posterior lateral stabilization of C7-T4 for a T2 vertebral body metastasis causing cord compression, followed by postoperative stereotactic body radiation therapy to T2 to T3 (35 Gy/5 fractions) in January 2021. In February 2021, she received palliative radiation therapy (20 Gy/5 fractions) to T9 to T12 for a T11 vertebral body metastasis with epidural extension", "From January to August 2021, she was treated with gemcitabine, and in September 2021, she received her first dose of liposomal doxorubicin", "Two weeks later, she developed progressive bilateral lower extremity numbness, pain, weakness, and imbalance, prompting emergent evaluation", "Magnetic resonance imaging (MRI) findings demonstrated extensive spinal cord edema extending from T2 to the conus and a posterior ring-enhancing intramedullary lesion at T9 to T10 without any mass effect (Figure 1)", "Because the intramedullary lesion appeared atypical for malignant disease given the lack of mass effect, she was prescribed high-dose steroids while further workup was pursued", "Lumbar puncture showed no malignant cells", "Positron emission tomographic-computed tomographic (PET/CT) scan demonstrated multifocal intensely F-fluorodeoxyglucose (FDG)-avid disease, but only mild uptake from T9 to T12 and no definitive FDG-avidity at the intramedullary lesion", "Her symptoms remained stable with steroid treatment and a plan was made to repeat an MRI of the spine in 6 to 8 weeks", "A, T1-weighted postgadolinium sagittal image demonstrating an enhancing spinal intramedullary lesion (blue arrowhead)", "Of note, there was enhancement of the previously irradiated T11 vertebral body metastasis, and adjacent vertebrae demonstrated marrow hyperintensity on T1-weighed noncontrast imaging", "B, T2-weighted short τ inversion recovery imaging demonstrating extensive spinal cord edema (blue bracket)", "Spinal cord infarct: underlying cause workup, supportive care"], "s1": [0, 1, 2, 3, 4], "s2": [5, 6, 7, 8, 9, 10, 11, 12, 13]} {"key": 552, "questions": "What Is Your Diagnosis?", "options": [{"label": "A", "disease": "Encephalocele"}, {"label": "B", "disease": "Deep lobe parotid tumor"}, {"label": "C", "disease": "Trigeminal schwannoma"}, {"label": "D", "disease": "Juvenile nasopharyngeal angiofibroma"}], "answer_idx": "A", "symptoms": ["A 51-year-old woman was referred for evaluation of headache, right-side jaw pain, paresthesia, and trismus", "The patient reported that the facial and jaw symptoms had evolved gradually over the past 5 years and that multiple medical therapies had provided only minimal relief of pain", "The patient’s medical history included anxiety, depression, emphysema, hyperlipidemia, migraines, and pseudoseizures", "She denied prior head trauma and had no pertinent surgical history", "On physical examination, the patient had normal body habitus and reported bilateral tenderness of the temporomandibular joints and paresthesia of the right trigeminal nerve in the V2 and V3 distribution with intact masseteric nerve and symmetric facial nerve", "Audiometric evaluation showed normal hearing bilaterally", "Magnetic resonance imaging (MRI) revealed a cystic mass in the right side of infratemporal fossa (ITF) that was nonenhancing on T1 sequencing with contrast and hyperintense on T2 (Figure 1A)"], "s1": [2, 3, 5], "s2": [0, 1, 4, 6]} {"key": 553, "questions": "What Is Your Diagnosis?", "options": [{"label": "A", "disease": "Eczema"}, {"label": "B", "disease": "Aquagenic syringeal acrokeratoderma"}, {"label": "C", "disease": "Hereditary papulotranslucent acrokeratoderma"}, {"label": "D", "disease": "Progressive symmetric erythrokeratoderma"}], "answer_idx": "B", "symptoms": ["An otherwise healthy man in his 20s presented to the dermatology department with thickening and swelling of hands after immersion in water, accompanied by burning pruritus for 3 years", "The lesions occurred as symmetrical white and transparent papules, white keratosis, excessive wrinkling on the dorsum of hands after water immersion and worsened in summer, and remitted in winter", "He was treated with topical tretinoin ointment intermittently per a diagnosis of chronic eczema at the local clinic with little improvement", "The lesions gradually aggravated, and the affected area dispersed to wrists and elbows during past the 1.5 years", "He attributed these changes to the need for washing hands frequently in the period of the COVID-19 epidemic", "He had no family history of similar skin problems and did not have a history of hyperhidrosis, cystic fibrosis, atopy, or trauma to the hands", "Physical examination showed some changes emerged on both hands after water immersion contrasting with any other condition (Figure, A and B)", "We took a skin biopsy specimen from white papules on the right hand, and representative histopathologic features are shown in the Figure, C and D", "A and B, After immersing both hands in water for 10 mins, there were symmetrical white papules, white hyperkeratosis spots, and excessive wrinkling on the thenar eminence, wrist flexure of both hands, and dorsum of fingers and hands", "C and D, A skin biopsy taken from white papules after water immersion for 10 mins showed hyperkeratosis, dilated eccrine ducts in the epidermis and dermis, and hyperplasia of the eccrine sweat glanda (hematoxylin-eosin)"], "s1": [0, 1, 3, 4, 5, 6, 8], "s2": [2, 7, 9]} {"key": 554, "questions": "What Is Your Diagnosis?", "options": [{"label": "A", "disease": "Pressure necrosis"}, {"label": "B", "disease": "Levamisole-induced vasculitis"}, {"label": "C", "disease": "COVID-19−associated acroischemia (thrombotic vasculopathy)"}, {"label": "D", "disease": "Nonpseudomonal ecthyma gangrenosum"}], "answer_idx": "A", "symptoms": ["A man in his 40s with a medical history of factor VII deficiency hemophilia presented with acute hypoxic respiratory failure secondary to COVID-19. Following admission, he was intubated with prone positioning, and treatment with tocilizumab and dexamethasone was started", "He was also found to have Staphylococcus hemolyticus bacteremia, and treatment with cefepime was initiated", "Three days after admission, the patient developed erythema and edematous necrotic plaques of the bilateral ears (Figure, A and B)", "No devices had been used on the ears", "The necrosis progressed during the following days, primarily involving the earlobe of the right ear and a large portion of the helix of the left ear", "A punch biopsy specimen was performed (Figure, C and D)"], "s1": [0, 1], "s2": [2, 3, 4, 5]} {"key": 555, "questions": "What Is Your Diagnosis?", "options": [{"label": "A", "disease": "Mumps (epidemic parotitis)"}, {"label": "B", "disease": "Alcoholic sialosis"}, {"label": "C", "disease": "Sjogren disease"}, {"label": "D", "disease": "Crohn disease"}], "answer_idx": "D", "symptoms": ["A 32-year-old woman was referred by her dentist to an oral surgeon for evaluation of persistent bilateral cheek swelling and mouth pain", "Two months prior to presentation, the patient had awoken with bilateral linear “gashes” in her mandibular vestibule that were accompanied by bilateral buccal facial swellings", "Since that day, the facial swelling remained unchanged, and although the mucosal lesions healed, the oral pain persisted", "The patient’s medical history was noteworthy for depression, for which she was taking bupropion and fluoxetine, and prior alcohol use disorder, for which she was 3 years sober", "She denied any smokeless tobacco use but endorsed daily electronic cigarette use", "The extraoral examination detected bilateral buccal facial swelling of the lower facial third without associated trismus or lymphadenopathy (Figure, A)", "Intraorally, bilateral scarring of the mandibular buccal vestibule was noted with focal areas of ulceration", "Small aphthous ulcerations were also present on the right buccal mucosa (Figure, B)", "An incisional biopsy of the right vestibular lesion was performed, and the specimen showed diffuse plasmalymphocytic infiltrates within the lamina propria", "On higher magnification, noncaseating granulomatous inflammation was observed (Figure, C)", "Staining for acid-fast bacilli was negative", "Clinical and histologic features of a patient with oral ulceration and facial swelling", "A, Clinical examination shows bilateral buccal swelling", "B, Intraoral examination shows linear vestibular ulcerations with buccolabial aphthous ulcers", "C, Hematoxylin-eosin stain of biopsy specimen of the vestibular ulcerated tissue shows noncaseating granulomatous inflammation (original magnification, ×100)", "Acid-fast bacilli staining was negative"], "s1": [3, 4, 2, 0], "s2": [15, 8, 7, 14, 6, 1, 10, 11, 5, 9, 12, 13]} {"key": 556, "questions": "What Is Your Diagnosis?", "options": [{"label": "A", "disease": "Laryngeal cancer"}, {"label": "B", "disease": "Metastatic colon cancer"}, {"label": "C", "disease": "Lymphoma"}, {"label": "D", "disease": "Esophageal cancer"}], "answer_idx": "B", "symptoms": ["A 64-year-old man presented to the otolaryngology clinic with hoarseness of voice for 3 weeks associated with intermittent dry cough", "He had no medical history or family history of cancer and was not a smoker", "In the absence of other suggestive symptoms, as well as a normal physical examination that did not reveal any enlarged cervical lymph nodes or oropharyngeal or neck mass and an unremarkable nasoendoscopic examination, a diagnosis of laryngopharyngeal reflux was made", "The patient was advised to try conservative management with dietary advice and proton pump inhibitors, and a clinical review was scheduled in 6 weeks", "The patient only returned 3 months later when his hoarseness became worse", "Repeat nasoendoscopy now showed left vocal cord palsy in the paramedian position", "In the absence of a history of trauma and associated neurological or systemic symptoms, the main concern was an underlying tumor that impinged on the vagus nerve at various points or its recurrent branch", "Results of a computed tomographic (CT) scan showed an ill-defined soft tissue mass, which was likely nodal disease, at the left supraclavicular (Figure 1), as well as the prevertebral and paravertebral region, which partially encased the left common carotid, left subclavian, and left vertebral arteries", "This was along the expected course of the left recurrent laryngeal nerve and likely accounted for the left vocal cord palsy", "However, the primary site of disease remained unclear", "A fused positron emission tomography with CT (PET/CT) scan with fluorodeoxyglucose (FDG) tracer was then performed", "Coronal view of the computed tomographic scan of the patient’s neck, including the left supraclavicular node (yellow arrowhead) and the left common carotid artery (black arrowhead)"], "s1": [0, 1, 2, 3, 4], "s2": [5, 6, 7, 8, 9, 10, 11]} {"key": 557, "questions": "What Is Your Diagnosis?", "options": [{"label": "A", "disease": "Lichen spinulosus"}, {"label": "B", "disease": "Multiple minute digitate hyperkeratosis"}, {"label": "C", "disease": "Porokeratosis"}, {"label": "D", "disease": "Viral warts"}], "answer_idx": "B", "symptoms": ["A patient in their 50s presented with asymptomatic keratotic papules, first noticed on the left middle and ring fingers and left toe web spaces more than 10 years earlier", "The papules progressed proximally over the subsequent decade to involve the forearm, arm, scapular area, ankle, shin, and thigh, but only affected the left side of the body", "The patient’s face, palms, soles, and entire right side of the body were spared", "The patient has a history of rosacea being treated with topical metronidazole gel for flares but was otherwise asymptomatic", "The patient did not have any alopecia, night blindness, or dry eyes or skin", "There was no history of any exposure to radiation or arsenic, nutritional deficiency, inflammatory skin disease, or malignant neoplasm", "There was no family history of a similar condition", "Examination revealed multiple, submillimeter, skin-colored to yellowish keratotic spiny projections over the aforementioned areas, only on the left side of the body (Figure, A and B)", "Punch biopsy from a lesion showed orthokeratosis with parakeratotic columns emerging from an epidermal invagination", "The epidermis was mildly acanthotic and with a normal granular layer", "There was a sparse superficial perivascular lymphocytic infiltrate", "Adjacent hair follicles were unaffected (Figure, C)", "A, Numerous, submillimeter, skin-colored spiny projections over left foot web spaces were present", "B, Close-up photograph over the left scapular area showed lesions of similar morphologic characteristics", "C, Histologic results showed a parakeratotic column overlying a mildly acanthotic epidermis with a normal granular layer and keratinocyte maturation", "A sparse superficial perivascular lymphocytic infiltrate was present (hematoxylin-eosin)"], "s1": [0, 1, 2, 3, 4, 5, 6, 7, 12, 13], "s2": [8, 9, 10, 11, 14, 15]} {"key": 558, "questions": "What Is Your Diagnosis?", "options": [{"label": "A", "disease": "Nonbullous neutrophilic lupus erythematosus"}, {"label": "B", "disease": "Histiocytoid Sweet syndrome"}, {"label": "C", "disease": "Palisaded neutrophilic and granulomatous dermatitis"}, {"label": "D", "disease": "Hypocomplementemic urticaria vasculitis"}], "answer_idx": "A", "symptoms": ["A teenaged boy presented to the outpatient clinic with a 1-year history of asymptomatic cutaneous lesions on the face that had progressively spread to the trunk and extremities", "He also had asthenia and mild intermittent arthralgias", "Physical examination revealed multiple millimetric erythematous to hyperpigmented monomorphic keratotic papules involving the head, trunk, and extremities", "Symmetric annular brown erythematous plaques on the cheeks and temples were also seen", "There were no vesicles or pustules (Figure, A and B)", "Laboratory investigations revealed a normal white blood cell count, mild anemia (hemoglobin level, 11.3 g/dL", "normal level 12.50-15.50 g/dL", "to convert to g/L, multiply by 10.0), hypocomplementemia, and moderate proteinuria", "Autoimmune serology results showed raised levels of antinuclear antibodies (>1:160), anti-Smith antibodies, SSA/Ro antibodies, SSB/La antibodies, and anti-U1RNP antibodies", "Anti-dsDNA antibodies, lupus anticoagulant, rheumatoid factor, IgG and IgM anticardiolipin antibody, and anti-β2 glycoprotein 1 antibody results were negative", "A biopsy specimen obtained from an erythematous papule on the trunk and stained with hematoxylin-eosin is shown in the Figure, C and D", "A, Millimetric erythematous to hyperpigmented monomorphic papules involving the trunk", "B, Erythematous papules and symmetric annular brown erythematous plaques on the cheeks and temples", "C and D, Punch biopsy results (hematoxylin-eosin)"], "s1": [0, 1, 2, 3, 4, 10, 11, 12, 13], "s2": [5, 6, 7, 8, 9]} {"key": 559, "questions": "What Is Your Diagnosis?", "options": [{"label": "A", "disease": "Branchial cleft cyst"}, {"label": "B", "disease": "Atypical mycobacterium infection"}, {"label": "C", "disease": "Chronic sialadenitis with abscess"}, {"label": "D", "disease": "Venolymphatic malformation"}], "answer_idx": "B", "symptoms": ["A previously healthy 8-year-old boy presented to an outpatient clinic for further evaluation of a fluctuating right parotid mass that had been present for 3 years", "The patient was not immunocompromised and denied any trauma to the area", "In the 2 months prior, he had received multiple rounds of antibiotics from other health care professionals", "Outside computed tomography of the face showed a 3-cm cystic structure in the superficial aspect of the right parotid gland with associated lymphadenopathy", "An outside health care professional had attempted incision and drainage of the area with negative cultures", "The patient presented to our clinic with an isolated right 2-cm fluctuant and tender mass overlying the parotid gland", "There was no accompanying cervical lymphadenopathy, and facial nerve function remained fully intact", "The patient denied fevers or systemic symptoms", "Results of laboratory evaluation revealed normal complete blood cell count and erythrocyte sedimentation rate levels with negative infectious workup results for Epstein-Barr virus, cytomegalovirus, HIV, and tuberculosis (TB)", "Magnetic resonance imaging showed a 1.9-cm peripherally enhancing, centrally necrotic lesion overlying the superficial right parotid gland with multiple subcentimeter necrotic lymph nodes within the right parotid gland (Figure 1)", "The T1-weighted, contrast-enhanced, and fat-suppressed axial (A) and coronal (B) magnetic resonance imaging scans demonstrate the enhancing right-sided lesions overlying the distal aspect of the parotid and extending deep into the parotid parenchyma"], "s1": [0, 1, 2, 4, 5, 7, 8], "s2": [3, 6, 9, 10]} {"key": 560, "questions": "What Is Your Diagnosis?", "options": [{"label": "A", "disease": "Intravascular papillary endothelial hyperplasia"}, {"label": "B", "disease": "Pseudo-aneurysmal metastasis of melanoma"}, {"label": "C", "disease": "Axillary web syndrome"}, {"label": "D", "disease": "Traumatic humeral pseudo-aneurysm"}], "answer_idx": "C", "symptoms": ["A woman in her 40s with a medical history of multiple sclerosis consulted for the sudden development of 2 adjacent nodules on the left arm", "An excision with 1-cm margins of the left arm demonstrated desmoplastic melanoma with a Breslow thickness of 1 mm, and a left axillary sentinel lymph node biopsy (SLNB) had been performed 3 weeks before under general anesthesia", "The patient was also being treated with subcutaneous interferon beta 1a for the multiple sclerosis", "Physical examination revealed 2 indurated and mobile subcutaneous nodules on the anterior surface of the arm 5 cm distal to the surgical scar, the larger of them measuring 15 mm in diameter (Figure, A) and the smaller of them measuring 4 mm", "When the arm was abducted to 90°, the patient denied pain but described a subtle tightness sensation on the arm", "An excisional biopsy from the larger lesion was performed (Figure, B-D)", "A, Clinical image showing indurated and mobile subcutaneous nodule on the anterior surface of the arm", "Arrowheads highlight a subtle linear plaque adjacent to the nodule", "B, Histopathologic image showing dilated and thrombosed lymphatic vessels in subcutaneous tissue with recanalization (hematoxylin-eosin)", "C, Histopathologic image of partially thrombosed vessels", "D, Immunohistochemistry finding was positive for D2-40 (podoplanin)"], "s1": [0, 2, 4], "s2": [1, 3, 5, 6, 7, 8, 9, 10]} {"key": 561, "questions": "What Is Your Diagnosis?", "options": [{"label": "A", "disease": "Actinomycosis"}, {"label": "B", "disease": "Atypical vasculitis"}, {"label": "C", "disease": "Synovitis, acne, pustulosis, hyperostosis, and osteitis syndrome"}, {"label": "D", "disease": "Chronic recurrent multifocal osteomyelitis"}], "answer_idx": "B", "symptoms": ["An otherwise healthy White man in his 60s presented with right jaw pain, trismus, and facial swelling", "Symptoms started a year before, after receiving dental implants for new lower dentures", "The patient noted right upper and lower eyelid swelling most notable in the morning but denied associated fevers, chills, vision changes, dysphagia, headaches, urinary symptoms, or rashes", "Physical examination revealed a 3-cm round fluctuant area lateral to the right lateral canthus, mildly tender to palpation", "No overlying warmth or erythema and no lymphadenopathy", "The patient’s dentures appeared to be well fitting, but mouth opening was at approximately 50% of expected", "Laboratory results included an erythrocyte sedimentation rate (ESR) of 20 mm/h, and normal complete blood cell count, comprehensive metabolic panel results, and lactate levels", "Findings on magnetic resonance imaging (MRI) (Figure, A) were most consistent with osteomyelitis of the mandible with inflammation of surrounding soft tissues", "Right temporalis muscle biopsy results for Gram stain and culture, fungal stain and culture, and acid-fast bacillus stain were unremarkable", "The patient’s symptoms did not improve after 7 weeks of empirical intravenous ertapenem followed by 8 weeks of oral cefuroxime and minocycline", "Additional laboratory test results, including serum IgG4, SPEP, brucella AB, Q fever AB, RF, urinalysis, and microalbumin/creatinine levels, were all unremarkable", "Results for p-ANCA and c-ANCA were negative", "Repeated MRI findings showed progression of the inflammation and new involvement of the temporal lobe dura", "Findings on positron emission tomographic computed tomography (PET-CT) scan showed nonspecific inflammatory changes to the bone, muscle, and brain meninges", "A biopsy of the temporalis muscle, sphenoid wing, and dura was performed (Figure, B)", "A, Magnetic resonance image (MRI) showing inflammation involving the right mandibular ramus and greater wing of sphenoid with overlying myositis involving masticator space", "B, Histopathologic image of biopsy specimen (original magnification ×100) shows aggregates of multinucleated giant cells (arrowhead) associated with intima fibroplasia (hematoxylin-eosin)"], "s1": [0, 1, 2, 3, 4, 5, 7, 12, 13, 15], "s2": [6, 8, 9, 10, 11, 14, 16]} {"key": 562, "questions": "What Is Your Diagnosis?", "options": [{"label": "A", "disease": "Papulopustular rosacea"}, {"label": "B", "disease": "Pinpoint papular light eruption"}, {"label": "C", "disease": "Actinic folliculitis"}, {"label": "D", "disease": "Acute localized exanthematous pustulosis"}], "answer_idx": "C", "symptoms": ["A woman in her 20s was referred for evaluation of a 4-year history of recurrent pustular eruptions on her face, which initially presented while she was vacationing in Florida", "She noted mild associated pruritus but denied burning sensation", "The eruptions occurred intermittently, appearing as erythematous papules with prominent pustules, and were exacerbated by sunlight", "On average, her lesions would resolve in 3 to 4 days with only mild background erythema by 5 to 7 days", "Even mild sun exposure would induce the eruption, leading to frequent recurrences in summer months but long periods of remission in the fall through spring", "Trials with various broad-spectrum sunscreens in combination with topical and oral antibiotics and topical ivermectin did not prevent flares or mitigate severity of eruptions when they occurred", "She denied joint pain, fever, fatigue, changes in urine color, use of other topical or systemic medications, or presence of rashes elsewhere", "She was otherwise healthy without other medical history", "On physical examination, she was well appearing with numerous 1- to 2-mm monomorphous pustules over background erythema on the medial cheeks, chin, and nose (Figure, A)", "No nail fold capillary changes or mucosal lesions were observed", "Serum analysis results for antinuclear, anti-SSA/Ro, and anti-SSB/La antibodies were negative", "A 4-mm punch biopsy was taken from a representative lesion along the right jawline (Figure, B-D)", "A, Pustular eruption of the face characterized by monomorphous 1- to 2-mm pustules over background erythema on the cheeks, chin, and nose appearing less than 24 hours after sun exposure", "Punch biopsy of skin demonstrated intrafollicular (B and C) and subcorneal (B and D) pustule formation (hematoxylin-eosin)"], "s1": [0, 1, 2, 3, 4, 5, 12], "s2": [6, 7, 8, 9, 10, 11, 13]} {"key": 563, "questions": "What Is Your Diagnosis?", "options": [{"label": "A", "disease": "Hair follicle nevus"}, {"label": "B", "disease": "Sebaceous nevus"}, {"label": "C", "disease": "Connective tissue nevus"}, {"label": "D", "disease": "Trichofolliculoma"}], "answer_idx": "A", "symptoms": ["A male in his late teens presented with asymptomatic congenital papules, plaques, and nodules in a linear bilateral distribution following Blaschko lines on the scalp and face region", "The papules and nodules appeared to grow as the boy grew, and some pedunculated lesions tended to fall off spontaneously", "He had not received any prior treatment", "The patient had no other significant medical history and denied having a family history of similar lesions", "Clinical examination findings revealed multiple skin-colored, nontender soft papules of varying sizes protruding from the skin surface on the right cheek, forehead, and jaw, as well as on the anterior surface of the scalp, distributed along Blaschko lines, with sparse vellus hairs and multiple hair follicles on the lesions (Figure, A and B)", "However, no hair was observed on the lesion because the patient had shaved the hair on his face", "An incisional biopsy of a pedunculated lesion above the left eyebrow was performed (Figure, B)", "A and B, Papules, plaques, and nodules on the skin surface of the right cheek (A, B), forehead (B), intercilium (B), and jaw (B)", "White arrowhead indicates the biopsy site (B)", "C and D, Composition of hair follicles", "Presence of sebaceous glands and the perifollicular fibrous sheath within a fibrotic stroma can be clearly seen (hematoxylin-eosin)"], "s1": [0, 1, 2, 3, 4, 5], "s2": [6, 7, 8, 9, 10]} {"key": 564, "questions": "What Is Your Diagnosis?", "options": [{"label": "A", "disease": "Listeriosis"}, {"label": "B", "disease": "Neuroschistosomiasis"}, {"label": "C", "disease": "Eosinophilic granulomatosis with polyangiitis"}, {"label": "D", "disease": "Neuromyelitis optica spectrum disorder"}], "answer_idx": "B", "symptoms": ["A previously healthy 25-year-old woman presented with headache, dysarthria, double vision, and dizziness for 1 week", "She denied any history of fever, head trauma, vomiting, diarrhea, abdominal pain, or cough", "She had traveled to northeastern Brazil 2 months before the onset of the neurological symptoms", "On examination, she had a mild right-side hemiparesis with hyperreflexia and hemiataxia", "She could walk unassisted but presented with a broad base", "There were no meningeal signs", "She did not present with hepatosplenomegaly of lymphadenopathy", "Her cranial magnetic resonance imaging showed a T2-weighted fluid-attenuated inversion recovery hyperintense lesion with heterogeneous gadolinium enhancement in the pons (Figure)", "There were no signs of diffusion restriction on diffusion-weighted imaging or hemorrhages on susceptibility-weighted imaging", "Sagittal gadolinium-enhanced T1-weighted magnetic resonance imaging (MRI) (A) and sagittal T2-weighted fluid-attenuated inversion recovery (FLAIR) (B) showing the extent of the lesion", "The laboratory test results showed a hemoglobin level of 15.3 g/dL (to convert to grams per liter, multiply by 10.0), total leukocyte count of 10 390/uL (to convert to ×109/L, multiply by 0.001), of which 1703 (16.4%) were eosinophils, and a platelet count of 284 000/μL (to convert to ×109/L, multiply by 0.001)", "The patient’s liver and kidney function test results were normal", "Computed tomography of the chest revealed bilateral patchy pulmonary infiltrates", "Cerebrospinal fluid (CSF) analysis showed 640 cells, of which 57% were lymphocytes, 26% were eosinophils, and 13% were monocytes", "protein levels were 70 mg/dL, and glucose levels were 46 mg/dL", "Her blood cultures were found to be sterile"], "s1": [0, 1, 2, 3, 4, 5, 6], "s2": [7, 8, 9, 10, 11, 12, 13, 14, 15]} {"key": 565, "questions": "What Is Your Diagnosis?", "options": [{"label": "A", "disease": "Linear atrophoderma of Moulin"}, {"label": "B", "disease": "Linear discoid lupus erythematosus"}, {"label": "C", "disease": "Linear or annular lupus erythematosus panniculitis of the scalp"}, {"label": "D", "disease": "Localized lipodystrophy"}], "answer_idx": "C", "symptoms": ["A man in his 50s presented to our dermatology department with an asymptomatic, arc-shaped, depressed skin groove on the left side of his forehead, frontal scalp, and vertex area", "Localized linear hair loss and hair thinning on the left scalp area were observed", "The patient reported that when the lesion had begun to appear, approximately 10 years earlier, he had received 2 doses of intralesional steroid via injection on a small bald patch on the vertex", "Thereafter, the lesion gradually progressed to the frontal scalp and eventually made a linear furrow on the left side of his forehead", "Physical examination revealed 2 long, curved-line, depressed, alopecic patches on the left side of the vertex and frontal scalp area extending to the left eyebrow (Figure, A and B)", "The lesions measured (length × width) 7 × 2 cm and 6 × 1 cm, respectively", "Dermoscopic examination of the alopecic area revealed several short vellus hairs, yellow dots, and honeycomb hyperpigmentation", "however, broken hairs were not seen", "The result of a hair pull test was negative", "The patient denied any trauma history and had no history of connective tissue disease nor any evidence of systemic involvement, such as arthralgia, photosensitivity, or skin rashes", "Skin biopsies were performed on the left side of the vertex and the forehead", "Histopathologic findings of the 2 biopsy specimens were identical (Figure, C and D)", "A, Clinical image of annular arc from a longitudinally furrowed skin lesion on the left eyebrow and forehead to an alopecic lesion", "B, Clinical image of the bandlike grooved alopecic lesion on the left side of the frontal scalp and vertex", "C, Scanning view shows predominant lymphocytic infiltrates in perifollicular and periappendageal regions of the dermis and lobular panniculitis (hematoxylin-eosin staining)", "D, High-power view shows inflammatory infiltrates between adipocytes with hyaline fat necrosis in the subcutis (hematoxylin-eosin staining)", "Linear or annular lupus erythematosus panniculitis of the scalp"], "s1": [0, 1, 3, 4, 5, 6, 7, 8, 12, 13], "s2": [2, 9, 10, 11, 14, 15, 16]} {"key": 566, "questions": "What Is Your Diagnosis?", "options": [{"label": "A", "disease": "Paraneoplastic autoimmune multiorgan syndrome"}, {"label": "B", "disease": "Thymoma-associated multiorgan autoimmunity"}, {"label": "C", "disease": "Good syndrome"}, {"label": "D", "disease": "Paraneoplastic lichen planus"}], "answer_idx": "C", "symptoms": ["A White man in his 50s with a clinical history of recurrent respiratory infections and diarrhea presented with painful oral erosions that began 2 years before", "Results of a previous intestinal biopsy had shown unspecific findings and absence of apoptosis", "He reported a 5-kg weight loss that he attributed to impaired intake owing to the oral lesions", "He was also being evaluated for a mediastinal mass detected in radiography of the chest performed for pneumonia", "Physical examination revealed whitish edematous lacy patches with erosions on the dorsal and lateral aspects of the tongue and buccal mucosa (Figure, A)", "skin and nails were not involved", "A lingual biopsy specimen was obtained for pathologic evaluation and direct immunofluorescence (DIF", "Figure, B and C)", "The result of indirect immunofluorescence (IIF) testing, using monkey esophagus, salt-skin split, and rat bladder as substrates, was negative", "Enzyme-linked immunosorbent assay (ELISA) and immunoblot analysis did not detect any desmoglein-1 or -3, bullous pemphigoid-180 or -230, collagen VII, desmocollin-1 to -3, laminin-332, envoplakin, or periplakin antibodies", "Results were normal for liver function", "negative for hepatitis B and C", "indicative of hypogammaglobulinemia, with a decreased immunoglobulin (Ig) G serum level of 465 mg/dL (normal, 750-1600 g/L", "for g/L, multiply by 0.01)", "and normal for IgA and IgM levels", "Positron emission tomography revealed a 10-cm hypercapturing extrapulmonary mass (Figure, D)", "Surgical excision of the mediastinal mass was performed", "its histopathologic features were consistent with a type AB thymoma, Masaoka-Koga stage 2A", "A, Lacy, white erosive lesions on the tongue", "B, Epidermis with acanthosis, parakeratosis, necrotic keratinocytes, and dermal band lymphocytic infiltrate with effacement of the dermoepidermal junction (hematoxylin-eosin)", "C, Fibrogen deposition along the basement membrane", "D, Hypercapturing, heterogeneous mass in the right anterior hemithorax"], "s1": [0, 2, 4, 5, 8, 9, 12, 13, 14, 18, 19, 20], "s2": [1, 3, 6, 7, 10, 11, 15, 16, 17, 21]} {"key": 567, "questions": "What Is Your Diagnosis?", "options": [{"label": "A", "disease": "Malignant atrophic papulosis"}, {"label": "B", "disease": "Disseminated discoid lupus"}, {"label": "C", "disease": "Scleromyxoedema"}, {"label": "D", "disease": "Secondary syphilis"}], "answer_idx": "A", "symptoms": ["A 46-year-old man presented with acute-onset weakness of his bilateral lower limbs for 3 days, accompanied by ascending numbness to just below his nipples", "Three weeks prior, he had developed acute urinary retention", "Neurological examination revealed weakness in bilateral lower limbs (Medical Research Council grade 4 on the right and grade 3 on the left), extensor plantar responses, and a sensory level at T6. Magnetic resonance imaging of his thoracic spine was performed 10 days after the onset of weakness and did not show any signal abnormalities or restricted diffusion", "Cerebrospinal fluid analysis findings were remarkable for elevated protein (0.14 g/dL [to convert to grams per liter, multiply by 10]) but white cell count (5 cells/mm3) and glucose levels (54.05 mg/dL [to convert to millimoles per liter, multiply by 0.0555]) were normal", "Cytology and flow cytometry were negative for malignancy", "Antibodies to aquaporin 4 and myelin oligodendrocyte glycoprotein were not detected", "The patient developed a sudden worsening of weakness in his legs 2 weeks after initial presentation", "Repeated thoracic spine imaging showed an area of T2 hyperintensity in the left posterolateral cord at T7, which did not enhance with contrast or show restricted diffusion (Figure 1A)", "Notably, he had a 4-year history of an asymptomatic papular skin eruption with more lesions developing in the last 4 months", "These were distributed on the neck, trunk, and limbs, sparing his face, scalp, and genitalia (Figure 1B)", "Findings of extensive workup for malignancy, including full-body computed tomography scan, paraneoplastic antibody panel, and systemic autoimmune diseases (antinuclear antibody, antidouble stranded DNA, and myositis antibody panel), were negative", "The skin papules were biopsied", "T2-weighted magnetic resonance imaging (MRI) of the thoracic spine showing an area of hyperintensity in the left posterolateral cord at T7 (A) (arrowhead)", "Multiple papules were seen on the patient’s abdomen", "These consisted of hypopigmented papules with central hyperpigmented umbilication and larger papules with typical central atrophic porcelain white scarring (B)"], "s1": [0, 1, 2, 3, 4, 5, 6, 7, 10, 12], "s2": [8, 9, 11, 13, 14]} {"key": 568, "questions": "What Is Your Diagnosis?", "options": [{"label": "A", "disease": "Type I cryoglobulinemia"}, {"label": "B", "disease": "Calciphylaxis"}, {"label": "C", "disease": "Cholesterol emboli"}, {"label": "D", "disease": "Oxalate vasculopathy"}], "answer_idx": "B", "symptoms": ["A 65-year-old woman with locally advanced, unresectable intrahepatic cholangiocarcinoma being treated with pemigatinib, a fibroblast growth factor receptor inhibitor (FGFRi), presented with a painful eruption on the bilateral lower extremities", "Pemigatinib treatment was initiated 2 months prior to the skin findings", "Laboratory evaluation demonstrated a mildly elevated D-dimer level at 578 ng/mL fibrinogen-equivalent units, prolonged prothrombin time of 12.7 seconds, elevated phosphorus level at 6.8 mg/dL (of note, phosphorus was 3.1 mg/dL 1 month prior), and normal calcium level and kidney function", "Other laboratory tests to note include cryoglobulin and cryofibrinogen, which both had negative results", "Within weeks, there was rapid progression with skin necrosis despite treatment initiation with rivaroxaban", "Physical examination revealed retiform purpura with areas of necrosis involving the bilateral lower extremities (Figure, A and B)", "A punch biopsy specimen from the right calf was obtained for histopathologic analysis (Figure, C and D)", "Early reticulated erythematous patches (A) that rapidly progressed to prominent retiform purpura with cutaneous necrosis (B)", "Calcium deposits within wall of a blood vessel in the subcutaneous tissue (C, white arrowhead) and pseudoxanthoma elasticum–like changes with fragmented calcified elastic fibers (D, white arrowheads) in the dermis and subcutaneous tissue (hematoxylin-eosin–stained sections", "original magnification ×200)"], "s1": [0, 1, 4, 5, 7], "s2": [2, 3, 6, 8, 9]} {"key": 569, "questions": "What Is Your Diagnosis?", "options": [{"label": "A", "disease": "Kaposi sarcoma"}, {"label": "B", "disease": "Stewart-Bluefarb syndrome"}, {"label": "C", "disease": "Acroangiodermatitis of Mali"}, {"label": "D", "disease": "Lichen planus"}], "answer_idx": "B", "symptoms": ["A young man presented to the clinic with large, purple plaques on the left lower extremity", "The plaques had been present for years, but the patient noted enlargement and darkening for the past 5 years", "The lesions were asymptomatic with no associated pruritus, bleeding, or pain", "No treatments or diagnostic workup had been attempted prior to presentation", "The patient was otherwise healthy and was a never-smoker with no history of hypertension, atherosclerotic disease, or HIV", "Physical examination revealed dark violaceous plaques, scattered subcutaneous nodules, and swelling to the mid-shin of the left lower extremity (Figure, A)", "There was mild warmth noted, but no palpable bruits", "Complete metabolic panel, complete blood cell count, prothrombin, and serology were negative or within normal limits", "A punch biopsy was performed and submitted for histopathologic analysis (Figure, B)", "A, Violaceous papules, plaques, and nodules on the left lower shin", "B, Proliferation of superficial and deep vessels, with thickening of the endothelium (hematoxylin-eosin)", "C, Magnetic resonance angiogram demonstrating hypervascular soft tissue mass in the left lower extremity"], "s1": [0, 1, 2, 4, 6, 9], "s2": [3, 5, 7, 8, 10, 11]} {"key": 570, "questions": "What Is Your Diagnosis?", "options": [{"label": "A", "disease": "Nontuberculous mycobacterial infection"}, {"label": "B", "disease": "Cutaneous aspergillosis"}, {"label": "C", "disease": "Phaeohyphomycosis"}, {"label": "D", "disease": "Entomophthoromycosis"}], "answer_idx": "B", "symptoms": ["A man in his 30s, employed as a painter, presented with multiple painless nodules on his right cheek of 4 years’ duration", "He reported episodic breathlessness, cough, and wheezing with expectoration of brownish sputum, exacerbated with exposure to hay dust, in the past 5 years", "He also reported recent-onset headache while reading", "Physical examination revealed a depressed plaque measuring 6 × 4 cm in the right infraorbital region, surrounded by multiple nodules of sizes ranging from 1 × 1 cm to 5 × 1 cm (Figure, A)", "The nodules were firm, mobile, and nontender on palpation", "Local temperature was normal", "There was proptosis of the right eye with mild restriction of extraocular movements on lateral gaze", "Examination of the oral cavity revealed dental caries, while gingivolabial sulcus was free on bidigital examination", "Nasal mucosa was normal", "Respiratory system examination revealed occasional polyphonic wheeze", "Chest radiography imaging showed nodular opacities in right upper zone and bronchiectasis", "Punch biopsy from 1 nodule was sent for histopathological and microbiological examination (Figure, B and C)", "A, Destructive depressed plaque involving the medial half of the right side of the face surrounded by multiple nodules of varying sizes", "B, Hematoxylin-eosin stain of skin biopsy specimen showing inflammatory infiltrate composed of giant cells in the deeper dermis extending into the subcutaneous fat", "C, High-power view showing epithelioid cell granulomas and fungal profiles within giant cells (arrowheads)", "D, Periodic acid–Schiff stain highlighting septate fungal hyphae with acute-angled branching within giant cells"], "s1": [0, 3, 4, 5, 6, 7, 8, 11, 12, 13, 14, 15], "s2": [1, 2, 9, 10]} {"key": 571, "questions": "What Is Your Diagnosis?", "options": [{"label": "A", "disease": "Leiomyosarcoma"}, {"label": "B", "disease": "Liposarcoma"}, {"label": "C", "disease": "Angiosarcoma"}, {"label": "D", "disease": "Hemangioendothelioma"}], "answer_idx": "A", "symptoms": ["A 33-year-old woman with a history of hereditary bilateral retinoblastoma as an infant presented to a primary care clinic with 1 week of right lower extremity swelling", "At age 8 months, she was treated with right enucleation and left eye radiotherapy (50.4 Gy)", "At age 4 years, she developed a recurrence of the retinoblastoma in the nasal cavity and was treated with 4 cycles of vincristine, doxorubicin, and cyclophosphamide followed by 1 cycle of cisplatin, etoposide, and intrathecal cytarabine", "Primary care evaluation was prompted by her mother, as the patient had not noticed her lower extremity edema", "She had no fevers, weight loss, shortness of breath, chest pain, abdominal pain, or history of deep vein thrombosis", "Her physical examination was notable for firm, nonpitting edema to the hip", "She did not have any associated tenderness, inguinal lymphadenopathy, or palpable abdominal masses", "Her vital signs were within normal limits and she ambulated comfortably", "Laboratory work results, including a complete blood cell count with a differential, comprehensive metabolic panel, and D-dimer levels, were within normal limits", "Doppler ultrasonography of her right lower extremity showed no deep vein thrombosis", "A computed tomography venogram demonstrated a 4.0 × 3.5 cm mass of likely vascular origin that was extending into the right common iliac vein (Figure)", "Computed tomography venogram demonstrating a 4.0 × 3.5 cm heterogeneously enhancing lobulated mass (arrowhead) extending into the right common iliac vein"], "s1": [0, 1, 2], "s2": [3, 4, 5, 6, 7, 8, 9, 10, 11]} {"key": 572, "questions": "What Is Your Diagnosis?", "options": [{"label": "A", "disease": "Branchial cleft cyst"}, {"label": "B", "disease": "Thyroglossal duct cyst"}, {"label": "C", "disease": "Parathyroid cyst"}, {"label": "D", "disease": "Cystic thyroid nodule"}], "answer_idx": "C", "symptoms": ["A 44-year-old woman presented to the otolaryngology clinic with a 6-month history of a right neck mass with compressive symptoms", "She first noticed swelling that progressed to pressure and dyspnea while supine", "She reported fatigue but denied pain, voice change, weight loss, hemoptysis, and dysphagia", "A physical examination revealed a visible and palpable fullness to the right neck without overlying cutaneous or sinus tract changes", "Results of a complete blood cell count, a thyroid stimulating hormone test, triiodothyronine and levorotatory thyroxine testing, and a complete metabolic panel were normal", "Ultrasonography showed a 7.9 × 6.3-cm septate cystic mass lateral to the right thyroid gland", "A contrast-enhanced computed tomography scan of the neck demonstrated a large, nonenhancing cystic-appearing lesion abutting the lateral and posterior margins of the right thyroid lobe and extending from the level of the piriform sinus to the thoracic inlet (Figure)", "Ultrasound-guided fine-needle aspiration revealed clear fluid with a parathyroid hormone (PTH) level of 67 pg/mL (reference range, <100 pg/mL", "to convert to ng/L multiply by 1.00)", "A barium swallow study showed no communication between the cyst and the piriform sinus", "Excision in the operating room was performed, and right thyroid lobectomy was necessary because of the intimate association of the lesion to the thyroid lobe", "Contrast-enhanced computed tomography scan of the neck with coronal (A) and axial (B) views demonstrating the right cystic neck mass"], "s1": [2, 4, 8], "s2": [0, 1, 3, 5, 6, 7, 9, 10, 11]} {"key": 573, "questions": "What Is Your Diagnosis?", "options": [{"label": "A", "disease": "Latent metastasis from the NSGCT"}, {"label": "B", "disease": "Supraclavicular lymphocele"}, {"label": "C", "disease": "Growing teratoma syndrome"}, {"label": "D", "disease": "Benign teratoma"}], "answer_idx": "C", "symptoms": ["A 40-year-old man presented with a large left neck mass in the setting of recently treated mixed nonseminomatous germ cell tumor (NSGCT) of the right testicle with metastasis to the retroperitoneal and mediastinal lymph nodes", "He was treated with a right radical orchiectomy and 4 cycles of etoposide, ifosfamide, and cisplatin (VIP) chemotherapy with resulting reduction in metastatic tumor burden and normalization of α-fetoprotein and β-human chorionic gonadotropin", "The patient’s neck swelling, which initially decreased in size while undergoing chemotherapy, progressively enlarged shortly after concluding treatment", "He underwent magnetic resonance imaging and computed tomography of the neck that showed a clearly defined, bulky supraclavicular mass confined to left neck levels 3, 4, and 5 (Figure)", "A, Postcontrast coronal magnetic resonance imaging (MRI) demonstrated a bulky, cystic left neck mass at levels 3, 4, and 5. B, Axial computed tomography (CT) scan with contrast highlighted extensive septations and modularity within the mass"], "s1": [0, 1], "s2": [2, 3, 4]} {"key": 574, "questions": "What Is Your Diagnosis?", "options": [{"label": "A", "disease": "Squamous cell carcinoma of the larynx"}, {"label": "B", "disease": "Graft-vs-host disease of the larynx"}, {"label": "C", "disease": "Extramedullary myeloid sarcoma of the larynx"}, {"label": "D", "disease": "Marginal zone lymphoma of the larynx"}], "answer_idx": "C", "symptoms": ["A 59-year-old man presented with progressive hoarseness and dyspnea over a 2-month period", "Fifteen months previously, he had undergone allogeneic stem cell transplant (allo-SCT) with a matched related donor for treatment of fms-like tyrosine kinase (FLT3) internal tandem duplication–mutated acute myeloid leukemia (AML)", "The patient had been diagnosed with chronic gastrointestinal graft-vs-host disease after a workup for diarrhea", "Results of a restaging bone marrow biopsy 2 weeks prior to presentation showed no evidence of leukemia", "Physical examination findings revealed no neck masses or abnormalities, but the patient was noted to be severely dysphonic and stridulous", "Flexible laryngoscopy findings demonstrated a right vocal fold neoplasm crossing the posterior commissure with associated bilateral vocal fold motion impairment and obstruction of the glottic airway (Figure, A)", "Awake tracheostomy was performed for airway protection, and biopsy of the neoplasm was obtained via operative microlaryngoscopy (Figure, B)", "Histologic findings demonstrated diffuse infiltrate of large cells with irregular nuclear contours, vesicular chromatin, and prominent nucleoli (Figure, C)", "Immunohistochemical stains showed CD33, CD43, and CD117 positivity, with a subset positive for CD7 and myeloperoxidase", "Stains were negative for CD3, CD20, and cytokeratin expression", "Evaluation of the larynx from awake flexible laryngoscopy (A) including posterior commissure (A, inset) and operative microlaryngoscopy (B) with biopsy (C", "original magnification ×40)"], "s1": [0, 2, 3, 4, 6, 10], "s2": [1, 5, 7, 8, 9, 11]} {"key": 575, "questions": "What Is Your Diagnosis?", "options": [{"label": "A", "disease": "Malignant neoplasm–associated microscopic polyangiitis"}, {"label": "B", "disease": "Intravascular involvement by NK/T-cell lymphoma"}, {"label": "C", "disease": "Cutaneous coccidiomycosis"}, {"label": "D", "disease": "Erythema nodosum"}], "answer_idx": "B", "symptoms": ["A woman in her 50s with a history of nasal-type natural killer (NK)/T-cell lymphoma (diagnosed 2 years prior", "treated with pembrolizumab and methotrexate, ifosfamide, etoposide, dexamethasone, and pegaspargase) and chronic pulmonary coccidiomycosis being treated with fluconazole was admitted for fever, hypotension, and rash", "Three months prior, pembrolizumab treatment was stopped because of a widespread morbilliform reaction, which had resolved completely 6 to 8 weeks before admission", "The new widespread, asymptomatic skin eruption had been ongoing for 1 to 2 weeks and appeared different from her pembrolizumab reaction", "A complete blood cell count was notable for a normal white blood cell count of 4100/μL, normal absolute neutrophil count of 2940/μL, chronic normocytic anemia (hemoglobin level of 8.2 g/dL), and chronic thrombocytopenia (platelet count of 62 ×103/μL)", "Blood culture results showed no growth at 1 week", "Serum β-D-glucan, galactomannan, Coccidioides antibody by complement fixation, and immunodiffusion results were negative", "Respiratory viral panel and SARS-CoV-2 test results were negative", "Total body skin examination findings were notable for widespread, nontender, erythematous patches and thin plaques, some with prominent overlying telangiectases and hyperpigmentation (Figure, A)", "A punch biopsy of a representative lesion was performed (Figure, B-D)", "A, Physical examination findings were notable for widespread erythematous patches and thin plaques, some with prominent overlying telangiectases", "B and C, Hematoxylin-eosin stained section", "D, CD56 immunostaining labels intravascular atypical cells"], "s1": [0, 1, 2, 4, 5, 6, 7], "s2": [3, 8, 9, 10, 11, 12]} {"key": 576, "questions": "What Is Your Diagnosis?", "options": [{"label": "A", "disease": "Giant cell arteritis"}, {"label": "B", "disease": "Trigeminal herpes zoster"}, {"label": "C", "disease": "Trigeminal trophic syndrome"}, {"label": "D", "disease": "Pyoderma gangrenosum"}], "answer_idx": "C", "symptoms": ["A 66-year-old man presented with multiple progressive ulcers on the right side of his face that had developed over the past 3 months (Figure, A)", "He had also experienced a burning sensation on the right side of his face, but denied headache, shoulder, or jaw pain and visual disturbance", "Neurological examination revealed hypoesthesia of the right side of the face, while cold-warm sensation and motor function were intact", "The erythrocyte sedimentation rate was 82 mm per hour (normal value, less than 15 mm per hour) and the serum C–reactive protein concentration was 107 mg/L (normal value, less than 5 mg/L)", "Chest radiography showed bilateral perihilar infiltrates consistent with pneumonia", "About 2 weeks before the ulcerations developed, the patient had been admitted to the hospital for unstable angina, and coronary angiography was performed", "Immediately after the angiography, the patient developed left-sided hemiplegia and hemianopsia", "Magnetic resonance imaging of the brain revealed an extensive ischemic posterior circulation stroke involving the right occipital lobe, lower cerebellar peduncle, and pontomedullary boundary (Figure, B)", "Clinical and radiological findings in a man aged 66 years", "A, Multiple ulcers limited to the right half of the face", "B, Magnetic resonance image showing brainstem involvement by the stroke (diffusion-weighted sequence)"], "s1": [0, 1, 2, 8, 9], "s2": [3, 4, 5, 6, 7, 10]} {"key": 577, "questions": "What Is Your Diagnosis?", "options": [{"label": "A", "disease": "Immune checkpoint inhibitor–induced acute interstitial nephritis"}, {"label": "B", "disease": "Postinfectious glomerulonephritis"}, {"label": "C", "disease": "Acute tubular necrosis secondary to kidney melanoma metastasis"}, {"label": "D", "disease": "Drug-induced acute tubular necrosis"}], "answer_idx": "C", "symptoms": ["A 53-year-old man received a diagnosis of stage IV melanoma with metastatic disease to the spine and brain 3 months before presentation", "Four weeks following receipt of immunotherapy with ipilimumab and nivolumab, he developed acute abdominal pain in the setting of a rapidly rising lactic acid dehydrogenase levels, prompting hospital admission", "Imaging results showed mild disease progression (new ascites and periportal metabolic lymph node)", "Splenic infarct and portal/splenic vein thrombi were identified as etiologies of his acute pain and were presumed to be secondary to hypercoagulable state of malignancy", "Over the next 3 days, he had rising serum creatinine (sCr) levels (baseline, approximately 1 mg/dL [to convert to μmol/L, multiply by 88.4])", "The initial workup included ultrasonography and urinalysis", "Ultrasonography results showed abnormal intraparenchymal flow to both kidneys without evidence of kidney artery stenosis", "Urine studies were consistent with prerenal etiology, while the presence of casts suggested acute tubular necrosis", "Nephrotoxic medications (omeprazole/pantoprazole) were discontinued", "The top differential diagnoses at this juncture included contrast-induced nephropathy or immune checkpoint inhibitor (ICI)–induced acute interstitial nephritis, and empirical high dose steroids were prescribed", "Despite observation and steroid treatment, his sCr levels continued to increase over the next 5 days, peaking at 5.8 mg/dL, and the patient progressed into anuric kidney failure, requiring hemodialysis", "This prompted a kidney biopsy for definitive diagnosis (Figure)", "One week later, he received encorafenib, a BRAF inhibitor.1 Two weeks later, the patient’s kidney function began to improve, and hemodialysis was discontinued", "He continued to show gradual improvement, with a return to near baseline kidney function (sCr level, 1.3 mg/dL) approximately 6 weeks after treatment initiation", "Histopathology (original magnification x40) of kidney capillaries with hematoxylin and eosin stain showing atypical cells (arrowheads) (A), tubules with hematoxylin and eosin stain (B), and capillaries with melanin-A (C)", "Acute tubular necrosis secondary to kidney melanoma metastasis"], "s1": [0, 1, 2, 3, 12, 13], "s2": [4, 5, 6, 7, 8, 9, 10, 11, 14, 15]} {"key": 578, "questions": "What Is Your Diagnosis?", "options": [{"label": "A", "disease": "Nasopharyngeal carcinoma"}, {"label": "B", "disease": "Autoimmune vasculitis"}, {"label": "C", "disease": "Squamous cell carcinoma"}, {"label": "D", "disease": "EBV-positive mucocutaneous ulcer"}], "answer_idx": "D", "symptoms": ["A 71-year-old man with a history of rheumatoid arthritis and type 2 diabetes presented to the otolaryngology team with 6 weeks’ history of pain and ulcers in the pharynx", "He also complained of nasal regurgitation", "He denied having any systemic symptoms", "He took linagliptin, valsartan, and 15 mg of methotrexate weekly", "He was a nonsmoker and consumed very little alcohol", "Clinical examination including flexible nasendoscopy revealed multiple shallow ulcers on the posterior wall of the nasopharynx", "There was a large defect involving the entire soft palate on the left", "There was no obvious mass or any exophytic lesion", "The edges of the soft palate defect were smooth and clean (Figure 1)", "There were no cranial nerve palsies", "There was no cervical, axillary, or inguinal lymphadenopathy", "Findings of the remaining head and neck examinations were normal", "Results of laboratory investigations, including lactate dehydrogenase level and autoimmune antibody tests, such as antinuclear antibody and antineutrophil cytoplasmic antibody, were negative", "Screening test results for HIV and Treponema pallidum were also negative", "The only positive result was immunoglobulin (Ig) G for Epstein-Barr virus (EBV)", "The IgM for EBV was not detected", "Magnetic resonance imaging of the head and neck on short tau inversion recovery sequence confirmed the soft palate defect but also showed some thickening and hyperenhancement around the edges of the defect", "There was no demonstrable nasopharyngeal mass", "The hard palate was intact, and there were no significant cervical lymph nodes", "The patient had biopsies taken from the ulcers and edges of the soft palate defect"], "s1": [0, 1, 5, 6, 7, 11, 16, 17, 18, 19], "s2": [2, 3, 4, 8, 9, 10, 12, 13, 14, 15]} {"key": 579, "questions": "What Is Your Diagnosis?", "options": [{"label": "A", "disease": "Medullary thyroid carcinoma"}, {"label": "B", "disease": "Papillary thyroid carcinoma"}, {"label": "C", "disease": "Hyalinizing trabecular tumor"}, {"label": "D", "disease": "Thyroid paraganglioma"}], "answer_idx": "D", "symptoms": ["A 50-year-old woman presented with a 2-month history of a neck mass that was associated with hoarseness of voice and globus sensation", "Physical examination showed thyromegaly (right greater than left) with an approximately 5-cm palpable nodule in the right lobe", "There was no tenderness or cervical lymphadenopathy", "There was no prior radiation exposure to the head and neck area", "Thyroid function test results were normal, and she did not take thyroid medications", "A relative had received a diagnosis of papillary thyroid cancer at age 50 years", "There was no other pertinent family history, including a history of multiple endocrine neoplasia", "Ultrasonography results showed a multinodular goiter with a dominant 4.6-cm right thyroid nodule, 1.6-cm left-sided thyroid nodule, and suspect left level IV lymph node", "Ultrasonography-guided fine-needle aspiration biopsy results showed papillary thyroid carcinoma (PTC) (Bethesda VI) with a BRAFV600E variation in the left nodule and lymph node, and atypia of undetermined significance (Bethesda III) with a suspicious Afirma genomic sequence classifier in the right nodule", "She underwent total thyroidectomy with modified left lateral neck dissection", "Gross examination of the specimen showed a 2.9-cm pale-tan, partially circumscribed, soft nodule in the right thyroid lobe", "a 1.4-cm pale-tan, fibrotic, centrally brown nodule in the left lobe", "and a 1.8-cm pale-tan, rubbery, partially circumscribed nodule in the isthmus", "Histologic examination of the right thyroid lesion was performed (Figure), with ancillary testing", "A, The right-sided nodule showed an organoid zellballen pattern, with sharp demarcation from thyroid tissue (hematoxylin-eosin)", "B, At higher magnification, the tumor cells exhibited amphophilic granular cytoplasm, round to oval nuclei, and salt-and-pepper chromatin (hematoxylin-eosin)"], "s1": [0, 1, 3, 4, 5, 6, 7], "s2": [2, 8, 9, 10, 11, 12, 13, 14, 15]} {"key": 580, "questions": "What Is Your Diagnosis?", "options": [{"label": "A", "disease": "Intralymphatic histiocytosis"}, {"label": "B", "disease": "Reactive angioendotheliomatosis"}, {"label": "C", "disease": "Intravascular large B-cell lymphoma"}, {"label": "D", "disease": "Intralymphatic proliferation of T-cell lymphoid blasts"}], "answer_idx": "C", "symptoms": ["A woman in her 70s presented with a 3-month history of episodic fever and painless cutaneous lesions on her lower limbs", "The patient otherwise felt well and denied malaise, weight loss, or any other symptoms", "She did not have any known benign inflammatory conditions", "Physical examination showed indurated plaques with multiple telangiectasias (Figure, A and B) on almost all the surface of the legs", "Except for slightly elevated C-reactive protein levels, the results of other routine blood and urine tests were unremarkable", "A skin biopsy specimen was obtained for histopathology examination (Figure, C and D)", "A, Indurated violaceous plaques with multiple telangiectasias on the lower limbs", "B, Prominent telangiectasia on the thigh", "C, Histopathologic analysis shows large round cells proliferating within dermal blood vessels (hematoxylin-eosin)", "D, Round cells with scanty cytoplasm and prominent nucleoli filling a vessel (hematoxylin-eosin)"], "s1": [0, 1, 2, 3, 4], "s2": [5, 6, 7, 8, 9]} {"key": 581, "questions": "What Is Your Diagnosis?", "options": [{"label": "A", "disease": "Ichthyosis vulgaris"}, {"label": "B", "disease": "Netherton syndrome"}, {"label": "C", "disease": "Pityriasis rubra pilaris"}, {"label": "D", "disease": "Blau syndrome"}], "answer_idx": "D", "symptoms": ["A school-aged boy experienced generalized erythematous rashes for 4 years", "Skin examination showed ichthyosiform brownish plaques on his lower extremities and multiple erythematous confluent papules and plaques on his trunk and face (Figure, A and B)", "Topical steroid and pimecrolimus were administered, but their effects were limited", "In addition, painful distal and proximal interphalangeal joints were noted for 2 years, and the patient could not clench his fists", "Uveitis in both eyes was also diagnosed by an ophthalmologist", "Microscopic examination of his hair showed no trichorrhexis invaginata or trichorrhexis nodosa", "A skin biopsy specimen was obtained and submitted for further histopathologic analysis (Figure, C and D)", "A, Diffuse erythematous papules and plaques with mild scaling and some confluence on the face, neck, and trunk", "B, Ichthyosiform rashes predominantly distributed over the lower extremities", "C, Tissue biopsy results revealed multiple circumscribed nests of granulomas that were infiltrating the dermis and subcutis (hematoxylin-eosin)", "D, At higher magnification, histologic findings showed noncaseating granulomas with multinucleated giant cells and neutrophil infiltration in the dermis and subcutis (hematoxylin-eosin)"], "s1": [0, 1, 2, 4, 5, 7, 8], "s2": [3, 6, 9, 10]} {"key": 582, "questions": "What Is Your Diagnosis?", "options": [{"label": "A", "disease": "Aleukemic cutaneous myeloid sarcoma"}, {"label": "B", "disease": "Cutaneous lymphoma"}, {"label": "C", "disease": "Lepromatous leprosy"}, {"label": "D", "disease": "Sarcoidosis"}], "answer_idx": "A", "symptoms": ["A 29-year-old man presented with a 2-month history of rapidly growing, widespread, asymptomatic skin nodules all over his body", "He denied having fever, chills, fatigue, night sweats, or weight loss", "He had no history of malignant neoplasm or chemical exposure", "Physical examination findings demonstrated numerous erythematous, indurated nodules involving the surface of his head, neck, torso, and extremities, without ulceration or scarring (Figure, A and B)", "Findings of systemic reviews were unremarkable, and there was no lymphadenopathy or peripheral nerve enlargement", "Complete blood cell count with differential results were normal (white blood cell count, 6060/μL, normal range, 4500-10 000/μL", "absolute neutrophils, 3070/μL, normal range, 1800-8300/μL [to convert cell counts to ×109/L, multiply by 0.001])", "Serum lactate dehydrogenase level was elevated (279 U/L, normal range, 100-240 U/L [to convert to μkat/L, multiply by 0.0167])", "Biopsies from skin nodules (Figure, C) and bone marrow were obtained", "Skin lesions on the patient with widespread, erythematous, indurated nodules involving his head and neck (A) and trunk (B)", "C, Histologic findings demonstrate diffuse dermal infiltrates, extending to the subcutis, with sparing of the upper papillary dermis (original magnification ×20)", "The infiltrates are characterized by immature myeloid cells (inset, original magnification ×400)"], "s1": [0, 1, 2, 3, 4, 8, 9], "s2": [5, 6, 7, 10, 11]} {"key": 583, "questions": "What Is Your Diagnosis?", "options": [{"label": "A", "disease": "Acinic cell carcinoma"}, {"label": "B", "disease": "Mammary analog secretory carcinoma"}, {"label": "C", "disease": "Sclerosing polycystic adenoma"}, {"label": "D", "disease": "Myoepithelial carcinoma"}], "answer_idx": "C", "symptoms": ["A 9-year-old boy was referred to the pediatric otolaryngology clinic with a 3-month history of a left-sided neck mass", "The mass had been slowly enlarging and he had been experiencing mild intermittent pain, but no additional symptoms were reported", "His medical history was significant for several episodes of upper respiratory infection during the previous winter season, and he previously had an adenotonsillectomy several years prior and bilateral ear tubes placed 1 year before presentation", "Physical examination showed a mildly tender, nonmobile, left parotid mass near the angle of the mandible with no overlying skin changes or drainage around the mass", "There was no lymphadenopathy or other lesions noted in the neck area, and cranial nerve VII was fully intact", "Ultrasonography results of the neck showed a left 1.5-cm preauricular nodular mass with internal and marginal vascularity", "A neck magnetic resonance imaging study without contrast performed at an outside institution revealed the 1.5-cm heterogeneous intraparotid nodule that was predominantly isotense to glandular tissue on T1 images with vague hypointensity of a small irregular central region, as well as vague hyperintensity on T2 images", "He subsequently underwent fine needle aspiration biopsy to assess the lesion, which demonstrated benign-appearing salivary gland tissue and clusters of myoepithelial cells", "The mass was continuing to enlarge, and the patient’s pain was increasing in that area, so the decision was made to perform left superficial parotidectomy with preservation of the facial nerve", "Intraoperatively, the left parotid mass was noted to be just below the lower branches of the facial nerve", "The excised parotid mass was sent to the pathology department for analysis", "Gross examination of the specimen demonstrated a 1.6 × 1.5 × 1.1–cm tan-white, well-circumscribed lesion that was partially surrounded by salivary gland parenchyma (Figure, A)", "Histologic examination of the nodule was performed (Figure, B and C)", "Gross and histologic findings", "A, Serial sections of the intraparotid mass demonstrate a well-demarcated solid nodule with tan-white, glistening cut surfaces", "B, Microscopic examination shows a multilobulated, complex lesion with prominent stromal fibrosis and collections of ductal structures with varying degrees of cystic change (hematoxylin-eosin)", "C, Acinar cells with brightly eosinophilic intracytoplasmic granules (hematoxylin-eosin)"], "s1": [0, 1, 2, 3, 4, 5, 6, 8, 9], "s2": [7, 10, 11, 12, 13, 14, 15, 16]} {"key": 584, "questions": "What Is Your Diagnosis?", "options": [{"label": "A", "disease": "Amebiasis cutis"}, {"label": "B", "disease": "Herpes simplex virus infection"}, {"label": "C", "disease": "Cytomegalovirus infection"}, {"label": "D", "disease": "Cutaneous Crohn disease"}], "answer_idx": "C", "symptoms": ["A woman in her late 30s was evaluated by the dermatology inpatient consult service for a 4-day history of perianal pain", "Her medical history included systemic lupus erythematosus, antiphospholipid antibody syndrome, and external hemorrhoids", "Six months before admission, she underwent a deceased donor kidney transplant", "One month before presentation, she was hospitalized for acute cellular transplant rejection managed with systemic corticosteroids, belatacept, mycophenolate mofetil, and tacrolimus therapies", "Her immunosuppressive regimen at the time of dermatologic evaluation consisted of prednisone, 5 mg daily, and mycophenolate mofetil, 500 mg twice daily", "On admission, she was afebrile and normotensive", "laboratory results were notable for an elevated creatinine level of 1.43 mg/dL (reference range, 0.40-1.30 mg/dL) (to convert to micromoles per liter, multiply by 88.4) and thrombocytopenia (platelet count, 92 × 103/μL [reference range, 140-440 × 103/μL] [conversion to ×109 per liter is 1:1)", "She developed diarrhea 1 day after admission", "Fecal leukocytes were absent", "Stool antigen test results for cryptosporidium, giardia, and rotavirus were negative", "Polymerase chain reaction (PCR) test results for campylobacter, norovirus, salmonella, shigella, Shiga toxin–producing Escherichia coli 1 and 2, Vibrio cholerae, and Yersinia enterocolitica in the stool were negative", "Antigen and toxin test results for Clostridium difficile in the stool were positive", "Total body skin examination was notable for 1- and 3-cm tender, round perianal ulcers with a clean base", "Another 4-mm round ulcer was present at the superior intergluteal cleft (Figure, A)", "There were no ocular or oral ulcers", "A biopsy specimen of a perianal ulcer border was obtained for histopathologic analysis (Figure, B-D)", "Clinical and histologic images of multiple perianal ulcers", "A, Two round ulcers, each with well-demarcated borders, were present perianally", "A smaller, third ulcer was also noted in the intergluteal cleft", "B, Microscopic examination of the punch biopsy on low magnification reveals epidermal ulceration and mixed dermal inflammation (hemoxylin-eosin", "C, On higher magnification, the epidermis lacks viral cytopathic changes but there are enlarged dermal fibroblasts and endothelial cells (hemoxylin-eosin)", "D, Higher magnification reveals large fibroblasts and endothelial cells with glassy cytoplasm and intranuclear inclusions (hemoxylin-eosin)"], "s1": [0, 1, 2, 3, 4, 5, 6, 7, 11], "s2": [8, 9, 10, 12, 13, 14, 15, 16, 17, 18, 19, 20, 21]} {"key": 585, "questions": "What Is Your Diagnosis?", "options": [{"label": "A", "disease": "Lupus pernio"}, {"label": "B", "disease": "Granulomatous mycosis fungoides"}, {"label": "C", "disease": "Lupus vulgaris"}, {"label": "D", "disease": "Leishmania recidivans"}], "answer_idx": "C", "symptoms": ["A woman in her 50s presented with an edematous, massively enlarged right auricle", "The patient had immigrated to Israel 9 years before admission", "She was adamant that the lesion had been present since early childhood but had gradually increased with a recent appearance of foul-smelling otorrhea", "On examination, the auricle was markedly enlarged, edematous, and encircled by an erythematous, indurated plaque partially covered with a fine scale (Figure 1A)", "Diascopy revealed an “apple jelly” appearance", "There was no local or distant lymphadenopathy", "general physical examination findings were unremarkable", "A, Clinical findings at presentation in 2008 showed an enlarged and edematous auricle encircled by an indurated erythematous plaque", "Bloody otorrhea from the external auditory canal was seen", "Image resolution was digitally enhanced", "B, Biopsy specimen showed superficial and deep diffuse granulomatous dermatitis (hematoxylin-eosin)", "C, Higher magnification demonstrated classic tuberculoid granuloma with histiocytes and Langhans giant cells surrounded by lymphoplasmacytic infiltrate (hematoxylin-eosin)", "Results of routine laboratory investigations, including complete blood cell count, biochemical profile, liver function tests, and chest radiography, were normal", "An intradermal Mantoux test induced a 20-mm induration", "Histopathologic evaluation of the cutaneous lesion revealed noncaseating granulomatous dermatitis involving the dermis (Figure 1B)", "High magnification demonstrated tuberculoid dermal granulomas composed of histiocytes and Langhans giant cells surrounded by lymphoplasmacytic infiltrate (Figure 1C)", "Results of polarized light examination and periodic acid–Schiff, Ziehl-Neelsen, Giemsa, Gram, and Grocott methenamine silver stains were negative for microorganisms"], "s1": [0, 1, 2, 3, 5, 6, 7, 8], "s2": [4, 9, 10, 11, 12, 13, 14, 15, 16]} {"key": 586, "questions": "What Is Your Diagnosis?", "options": [{"label": "A", "disease": "Sweet syndrome"}, {"label": "B", "disease": "Urticaria"}, {"label": "C", "disease": "Neutrophilic eccrine hidradenitis"}, {"label": "D", "disease": "Leukemia cutis"}], "answer_idx": "C", "symptoms": ["A 16-year-old boy with acute leukemia of ambiguous lineage developed tender, 1- to 2-cm erythematous plaques and nodules during treatment with chemotherapy", "Previously, he had been well, with morphological remission found on a recent bone marrow biopsy", "The patient had no significant dermatological history, except an allergy to cashew nuts and a prior episode (6 mo) of urticaria during a platelet transfusion that had resolved with 1 dose of oral antihistamine", "The patient’s medications included ondansetron, aprepitant, prednisolone, and oxycodone (supportive care initiated with chemotherapy), and long-term posaconazole and trimethoprim/sulfamethoxazole treatment", "Lesions were noted on day 2 of the fourth cycle of a chemotherapy regimen comprising fludarabine, cytarabine, and filgrastim (G-CSF)", "Initially present over the distal limbs, palms, and soles, the lesions progressed to the proximal limbs and lower trunk during 24 hours (Figure, A and B)", "The patient developed a fever, which we extensively investigated", "Intravenous cefotaxime was commenced, but because no infectious source was identified, the fever was thought to be secondary to cytarabine", "A and B, Clinical images showing erythematous plaques and nodules on patient’s palm and forearm", "C, Hematoxylin-eosin–stained specimen showing leukocytoclastic neutrophilic infiltrate in deeper dermis, around appendage structures and coiled eccrine glands, and into the interstitium", "The results of a complete examination of blood drawn at lesion onset demonstrated a total white cell count of 6330/μL (to convert to 109/L multiply by 0.001) with absolute neutrophil count of 6140/μL, more than the previous day’s counts of 3210/μL and 1720/μL, respectively", "The blood film demonstrated increased neutrophil granulation with a left shift", "A punch biopsy from the upper arm was performed (Figure, C)"], "s1": [0, 1, 2, 3, 4, 6, 7], "s2": [5, 8, 9, 10, 11, 12]} {"key": 587, "questions": "What Is Your Diagnosis?", "options": [{"label": "A", "disease": "Cervical necrotizing fasciitis"}, {"label": "B", "disease": "Spontaneous lymphorrhea"}, {"label": "C", "disease": "Angioneurotic edema"}, {"label": "D", "disease": "Quincke edema"}], "answer_idx": "B", "symptoms": ["A 56-year-old patient with no medical history presented spontaneously to the emergency department of our university hospital (Assistance Publique Hôpitaux de Paris) for bilateral neck swelling that had developed over 24 hours", "The patient had no traumatic or iatrogenic triggers", "He reported an edema of the lower lip that resolved spontaneously the day before", "He had no dyspnea nor dysphagia, but did have a discrete dysphonia", "He had no fever", "Neck swelling was global, bilateral, and painless, and was most severe on the left side", "There was a rash, the border of which was marked with a felt pen", "Nasofibroscopy results showed a normal airway with a posterior pharyngeal wall bulging", "Oropharyngeal and oral cavity examination results were normal, with no trismus", "A contrast-enhanced computed tomography (CT) scan showed a large retropharyngeal fluid accumulation of 2 × 10 × 6 cm that extended laterally to the vascular spaces, especially on the left side (Figure, A and B)", "This was associated with infiltration of the mediastinal fat and a thin left pleural effusion", "There was no vascular thrombosis or dental abnormality", "Hyperleukocytosis levels were 12 000/mL", "We did not have the result of the C reactive protein assay", "Cervical computed tomography scan of the sagittal (A) and axial (B) planes", "In the sagittal plane, there is retropharyngeal fluid accumulation, and in the axial plane, there is fluid accumulation in the retropharyngeal and vascular spaces with fat infiltration"], "s1": [0, 1, 2, 3, 4, 5, 6, 7], "s2": [8, 9, 10, 11, 12, 13, 14, 15]} {"key": 588, "questions": "What Is Your Diagnosis?", "options": [{"label": "A", "disease": "Abscess"}, {"label": "B", "disease": "Head and neck malignant neoplasm(s)"}, {"label": "C", "disease": "Angioedema"}, {"label": "D", "disease": "Spontaneous airway hemorrhage with obstructive hematoma"}], "answer_idx": "D", "symptoms": ["A 71-year-old man presented to the emergency department with dyspnea, dysphonia, dysphagia, and neck swelling", "He denied chills, fever, neck stiffness, new medications, otalgia, sick contacts, trismus, and use of an angiotensin-converting enzyme inhibitor", "His complex medical history included factor V Leiden deficiency requiring therapeutic anticoagulation, heart failure with atrial fibrillation, and chronic obstructive pulmonary disease associated with 110-pack-year smoking history", "Laboratory studies demonstrated an international normalized ratio of 16.4, a prothrombin time of 118.1 seconds, and a partial thromboplastin time of 81.4 seconds", "On examination, the patient had substantial dysphonia, fullness in the left submandibular region, and ecchymosis of the lateral left neck extending toward the midline", "Contrast-enhanced computed tomography performed in the emergency department revealed a large, intermediate-density, modestly enhancing, left-sided submucosal lesion extending from the nasopharynx inferiorly to the glottis with encroachment on the airway (Figure)", "Flexible fiberoptic laryngoscopy demonstrated a large submucosal mass extending from the left posteroinferior nasopharyngeal wall to the left lateral and posterior pharyngeal walls, obstructing the view of the left supraglottis, glottis, and piriform sinus", "The right side of the epiglottis was ecchymotic and edematous", "The mass extended farther along the aryepiglottic fold to involve the arytenoid and interarytenoid space, as well as the false vocal fold on the left", "The vocal cords were only partially visualized", "Given the important laryngoscopy findings, the decision was made to proceed with awake fiberoptic nasotracheal intubation in the operating room to secure the patient’s airway", "Contrast-enhanced computed tomography of the neck in the coronal (A) and axial (B) views demonstrating a large, predominantly left-sided, obstructive, submucosal, upper airway lesion extending from the nasopharynx to the glottis"], "s1": [0, 1, 2, 3, 4, 7], "s2": [5, 6, 8, 9, 10, 11]} {"key": 589, "questions": "What Is Your Diagnosis?", "options": [{"label": "A", "disease": "Lichen striatus"}, {"label": "B", "disease": "Idiopathic follicular mucinosis"}, {"label": "C", "disease": "Follicular mucinous nevus"}, {"label": "D", "disease": "Trichodysplasia spinulosa"}], "answer_idx": "B", "symptoms": ["A girl younger than 2 years presented with a 1-year history of slightly pruritic lesions involving her face", "She was born after an uncomplicated pregnancy and delivery, and her growth and development were appropriate for her age", "No associated symptoms such as fever, anorexia, or weight loss were noted", "Physical examination revealed multiple skin-colored and erythematous follicular papules, 1 to 2 mm in diameter, distributed in a segmental pattern on the right side of the forehead, as well as the dorsum and tip of the nose", "A small, excoriated plaque was also noted on the right malar area (Figure 1A)", "No mucosal lesions were present, and the rest of the physical examination findings were unremarkable", "A complete hematological and biochemical survey, including blood cell count, liver and renal function tests, C-reactive protein levels, thyroid function, rheumatoid factor, and antinuclear antibodies, disclosed no abnormalities", "Two skin punch biopsies were taken from the forehead and nose", "Representative histopathological features are shown in Figure 1B and C", "Multiple skin-colored and erythematous follicular papules are seen coalescing into a linear plaque involving the patient’s forehead, as well as the dorsum and tip of the nose (A)", "Two skin punch biopsies were taken from the forehead and nose (B and C)"], "s1": [0, 3, 4, 9], "s2": [1, 2, 5, 6, 7, 8, 10]} {"key": 590, "questions": "What Is Your Diagnosis?", "options": [{"label": "A", "disease": "Becker nevus"}, {"label": "B", "disease": "Neurocristic hamartoma"}, {"label": "C", "disease": "Smooth muscle hamartoma"}, {"label": "D", "disease": "Plexiform neurofibroma"}], "answer_idx": "B", "symptoms": ["A 17-month-old boy was referred to pediatric dermatology for evaluation of a slow-growing, nontender plaque on the lower back", "The child's mother reported that, on discovery, the plaque was initially 1 × 1 cm and had since grown in size with the child", "She was uncertain if the plaque was present at birth", "The patient’s medical history was significant for dermal melanocytosis of the right hip and mid back, increased head circumference (97th percentile), and expressive language delay", "There was no family history of congenital birthmarks or neurofibromatosis type 1 or 2.Physical examination revealed a well-appearing boy with normal vital signs and Fitzpatrick type III skin type", "On the left lower back, there was a 15 × 5-cm indurated skin-colored to yellow, firm, multinodular plaque with hypertrichosis, extending from the midline to the left lower flank (Figure, A)", "There were additional blue-gray patches on the back and right hip, as well as a small café au lait macule on the right lower back", "Limited neurologic examination was within normal limits", "A 4-mm punch biopsy was performed for further diagnostic evaluation (Figure, B-D)", "A, Large, well-defined, firm yellow to skin-colored multinodular hypertrichotic plaque on the left lower back", "B, Hematoxylin-eosin staining demonstrates diffuse dermal infiltration of spindle cells, predominantly encircling terminal folliculosebaceous units (original magnification ×4)", "C, Higher-power magnification highlights a mix of partially fusiform melanocytes and dendritic cells with glassy, eosinophilic cytoplasm (hematoxylin-eosin, original magnification ×40)", "D, Immunohistochemical staining reveals diffuse and periadnexal S100 positivity in the mid to deep dermis (original magnification ×4)"], "s1": [0, 1, 2, 3, 4, 5, 6, 7], "s2": [8, 9, 10, 11, 12]} {"key": 591, "questions": "What Is Your Diagnosis?", "options": [{"label": "A", "disease": "Folliculitis decalvans"}, {"label": "B", "disease": "Squamous cell carcinoma"}, {"label": "C", "disease": "Erosive pustular dermatosis"}, {"label": "D", "disease": "Pemphigus foliaceus"}], "answer_idx": "C", "symptoms": ["A 73-year-old man with metastatic BRAF-mutant colorectal cancer presented with a 1-month history of crusted lesions with purulent drainage on the scalp, with associated pruritus and pain", "Five months earlier he began treatment with cetuximab, binimetinib, and encorafenib, complicated by a grade 1 acneiform eruption that was managed with doxycycline hyclate and topical hydrocortisone and clindamycin phosphate", "Oncologic therapy was withheld pending further evaluation of the scalp eruption", "He had a distant history of actinic keratoses on the scalp treated with liquid nitrogen but otherwise denied previous skin cancer", "Examination of the scalp revealed male pattern baldness, diffuse actinic damage, and multiple thick, yellow-brown crusts overlying erosions with purulent drainage (Figure 1)", "A wound culture was performed and grew methicillin-sensitive Staphylococcus aureus", "A shave biopsy was obtained"], "s1": [0, 3, 4, 5], "s2": [1, 2, 6]} {"key": 592, "questions": "What Is Your Diagnosis?", "options": [{"label": "A", "disease": "Meningitis"}, {"label": "B", "disease": "VACTERL constellation"}, {"label": "C", "disease": "Kallmann syndrome"}, {"label": "D", "disease": "Mayer-Rokitansky-Kuster-Hauser (MRKH) syndrome"}], "answer_idx": "D", "symptoms": ["A 16-year-old female complained of an olfactory disorder since birth", "She had a history of a laryngeal tracheoesophageal fissure, intestinal hernia, and hearing loss because of an ossicular malformation", "From age 10 years, she was unable to catch up with conversation of curry smell", "Olfactory tests were performed, and T&T olfactometry1 results showed severe hyposmia (5.4/5.4", "detection threshold/odor recognition threshold)", "On the card-type odor identification test, Open Essence,2 she only identified menthol correctly", "Her Self-Administered Odor Questionnaire3 score was 20%", "Her visual analog scale4 score was 10 for odor but 100 for taste", "There were no unusual nasal findings on endoscopy or computed tomography", "Because we suspected a congenital olfactory disorder, we recommended magnetic resonance imaging (MRI), which showed olfactory bulb (OB) atrophy (Figure, A)", "A, Coronal short-tau inversion recovery magnetic resonance imaging (MRI) of the olfactory bulb shows that the olfactory fissure remains aerated, but the olfactory bulb is slightly atrophied on both sides (arrowheads)", "There were no dysplastic signs in the frontal gyri", "B and C, An axial MRI T2-weighted image of the pelvis shows vaginal defects and bilateral 1 layer–differentiated uterine rudiments (arrowheads) that were 35 mm on the left and 37 mm on the right", "Both ovaries are normal (equivalent to Mullerian duct anomaly class I)", "The patient also confirmed that she had not started menstruating", "After consulting the pediatric department, an endocrine test was performed", "The results showed an atypical reaction to the insulin tolerance test", "However, the results for adrenocorticotropic hormone/cortisol corticotropin-releasing hormone load test, diurnal variation, and urine accumulation were normal", "The luteinizing hormone (LH)/follicle-stimulating hormone (FSH) values were normal as determined by LH-releasing hormone load tests", "She was subsequently referred to the gynecology department", "Her MRI results showed a vaginal defect and uterine rudiments", "however, the bilateral ovaries were normal (Figure, B and C)"], "s1": [0, 1, 2, 3, 4, 5, 6, 7, 8, 9, 10, 11], "s2": [12, 13, 14, 15, 16, 17, 18, 19, 20, 21]} {"key": 593, "questions": "What Is Your Diagnosis?", "options": [{"label": "A", "disease": "Erythema annulare centrifugum"}, {"label": "B", "disease": "Eosinophilic annular erythema"}, {"label": "C", "disease": "Granuloma annulare"}, {"label": "D", "disease": "Drug-induced bullous pemphigoid"}], "answer_idx": "B", "symptoms": ["A woman in her 70s with recurrent primary peritoneal carcinoma receiving pembrolizumab and ipilimumab presented to the Division of Dermatology with a pruritic rash that had been ongoing for 2 weeks", "The rash started as an erythematous papule on her right arm 2 months after initiation of a new immunotherapy regimen, then gradually enlarged and spread to involve both legs (Figure, A and B)", "She denied any new systemic symptoms associated with onset of the rash", "A recent trial of oral terbinafine for possible tinea corporis yielded no improvement", "Physical examination revealed erythematous annular plaques without scale on her right arm and bilateral legs", "A 4-mm punch biopsy was obtained from the patient’s right arm (Figure, C and D), and she was prescribed fluocinonide, 0.05%, ointment", "She returned to the clinic 3 weeks later with resolution of pruritus but enlarging lesions", "Her last dose of ipilimumab was 2 months prior, and she was scheduled to receive pembrolizumab with doxorubicin, carboplatin, and prednisone for treatment of her underlying malignant tumor", "For management of her rash, the patient was prescribed betamethasone dipropionate, 0.05%, ointment (augmented)", "A, Erythematous annular plaques without scale on the left lower leg", "B, Erythematous annular plaques without scale on the right lower leg", "C, Hematoxylin and eosin stain demonstrating superficial and deep perivascular and interstitial inflammation", "D, Hematoxylin and eosin stain demonstrating numerous scattered eosinophils"], "s1": [0, 1, 2, 3, 4, 6, 7, 8], "s2": [5, 9, 10, 11, 12]} {"key": 594, "questions": "What Is Your Diagnosis?", "options": [{"label": "A", "disease": "Mantle cell lymphoma"}, {"label": "B", "disease": "Folliculotropic mycosis fungoides"}, {"label": "C", "disease": "Diffuse large B-cell lymphoma"}, {"label": "D", "disease": "Marginal zone lymphoma"}], "answer_idx": "A", "symptoms": ["A man in his 60s presented as a transfer from an outside hospital", "The division of dermatology was consulted for evaluation of lesions on the face, chest, back, and arms that were gradually increasing in size and number over the past year", "Physical examination revealed leonine facies with madarosis and an eschar on the nasal dorsum (Figure, A)", "Red-to-brown dome-shaped papules coalescing into plaques were present on the upper extremities", "The lesions were nontender and nonpruritic", "Cervical and axillary lymph nodes were palpable", "The patient had a muffled voice and reported shortness of breath with exertion", "Results of laboratory investigation demonstrated marked leukocytosis with a white blood cell count of 77.5 × 103/uL with 76.5% lymphocytes", "No blasts were present on peripheral smear", "Computed tomography of the chest, abdomen, and pelvis demonstrated multiple enlarged mediastinal, axillary, subpectoral, abdominopelvic, and inguinal lymph nodes", "Small pulmonary nodules were also detected", "A punch biopsy was obtained from the right arm (Figure, B, C, and D)", "A, Infiltrative plaques and nodules obscuring normal facial architecture", "B and C, Histopathologic images show normal epidermis with dense diffuse dermal lymphoid infiltrate composed of small to medium cells with irregular nuclear contours (hematoxylin-eosin stain)", "D, Immunohistochemistry shows diffusely positive cyclin D1 staining"], "s1": [1, 2, 3, 4, 11, 12, 13, 14], "s2": [0, 5, 6, 7, 8, 9, 10]} {"key": 595, "questions": "What Is Your Diagnosis?", "options": [{"label": "A", "disease": "Turner syndrome"}, {"label": "B", "disease": "Fragile X syndrome"}, {"label": "C", "disease": "Rett syndrome"}, {"label": "D", "disease": "Cerebral palsy"}], "answer_idx": "C", "symptoms": ["A 42-year-old woman presented with clinical features of microcephaly, short stature, intellectual disability, severe degree of scoliosis, gait abnormality, and encephalopathy (Figure, A and B)", "She was born in 1973 at 38th week of gestation followed by a normal delivery (weight, 3.15 kg and head circumference, 42 cm) to nonconsanguineous parents", "The head size was normal at birth, and no other abnormalities were seen during her infancy", "At age 2 years, her parents noticed that she had delayed motor and language milestones", "She experienced a progressive developmental delay such as reduction of head growth, loss of acquired communication, and loss of motor functions between ages 9 and 12 years", "The severity of scoliosis increased with age, and she became a wheelchair user at age 18 years", "She was referred to us at age 40 years with the characteristic features of growth deceleration (height, 142 cm", "weight, 22 kg", "body mass index, 11 [calculated as weight in kilograms divided by height in meters squared]), apraxia, left-sided hemiplegia, spasticity, sleep apnea, constipation, osteoporosis, hypoalgesia, and repetitive hand tapping (Video)", "Blood reports showed anemia (hemoglobin level of 9.1 g/dL", "to convert to grams per liter, multiply by 10) and low levels of high-density lipoprotein cholesterol (47 mg/dL", "to convert to millimoles per liter, multiply by 0.0259)", "There was no history of autistic behavior, vision or hearing impairments, or epilepsy", "The electrocardiogram showed sinus rhythm with T wave inversion (V3-V6), and echocardiography showed mild cardiomegaly", "The brain magnetic resonance imaging studies showed a diffuse reduction in the corpus callosum involving splenium and enlarged right ventricle, resulting in cerebral lateral ventricular asymmetry (Figure, C)", "A, Characteristic features of a 42-year-old woman with microcephaly", "B, Severe scoliosis", "C, Magnetic resonance imaging of the patient with reduced volume of corpus callosum (arrowhead), sagittal view"], "s1": [1, 2, 3, 4, 5, 6, 7, 8, 0], "s2": [9, 10, 11, 12, 13, 14, 15, 16, 17]} {"key": 596, "questions": "What Is Your Diagnosis?", "options": [{"label": "A", "disease": "Ectopic parathyroid lipoadenoma"}, {"label": "B", "disease": "Liposarcoma"}, {"label": "C", "disease": "Ectopic thyroid"}, {"label": "D", "disease": "Retropharyngeal abscess"}], "answer_idx": "A", "symptoms": ["A female smoker in her 50s was referred for hyperparathyroidism and a multinodular goiter", "The patient reported symptoms of joint and musculoskeletal pain and fatigue but denied fever, sore throat, abdominal pain, and kidney stones", "Her medical history was remarkable for congestive heart failure and schizophrenia", "Neck examination revealed no meaningful findings except right thyroid enlargement", "Results of laboratory evaluation demonstrated normal white blood cell count, borderline hypercalcemia (10.6 mg/dL), and elevated parathyroid hormone level (208 pg/mL)", "She underwent ultrasonography at an outside facility, and results demonstrated a 2.2-cm dominant right thyroid nodule with additional smaller thyroid nodules", "Findings of preoperative technetium-99m (Tc-99m) sestamibi planar and single-photon emission computed tomographic/computed tomographic (SPECT/CT) imaging (Figure, A) were nonlocalizing for a parathyroid adenoma and showed increased uptake in the dominant right thyroid nodule (Figure, B)", "Results of an ultrasound-guided biopsy of the thyroid nodule demonstrated a benign colloid nodule with cystic changes", "Computed tomography with intravenous contrast was obtained, and findings were initially reported as negative except for right multinodular goiter", "However, on additional review, a well-circumscribed fat density mass measuring 3.0 × 2.3 × 1.1-cm was identified in the right retropharyngeal space posterior to the hypopharynx (Figure, C)", "A, Results of single-photon emission computed tomography/computed tomography (SPECT/CT) demonstrated relatively low radiopharmaceutical uptake superior to the expected area of the parathyroid glands", "B, Findings of technetium-99m sestamibi SPECT/CT demonstrated increased radiopharmaceutical uptake in the previously identified dominant right inferior thyroid nodule (white arrowhead)", "C, A CT neck scan with contrast showing a mass in the right retropharyngeal space with complex internal areas of increased attenuation (red circle)"], "s1": [0, 1, 2, 3, 4], "s2": [5, 6, 7, 8, 9, 10, 11, 12]} {"key": 597, "questions": "What Is Your Diagnosis?", "options": [{"label": "A", "disease": "Dentigerous cyst"}, {"label": "B", "disease": "Odontogenic keratocyst (keratocystic odontogenic tumor)"}, {"label": "C", "disease": "Ameloblastoma"}, {"label": "D", "disease": "Adenomatoid odontogenic tumor"}], "answer_idx": "D", "symptoms": ["A 19-year-old woman with no noteworthy medical issues or surgical history presented to the oral and maxillofacial surgery clinic with a 3-month history of right-sided maxillary pain and swelling", "The patient denied any drainage from the site but reported worsening swelling and pain over the past several weeks", "On physical examination, there was noted intraoral swelling involving the buccal and palatal aspects of the posterior maxilla", "The lesion was soft and fluctuant with no appreciable drainage", "The patient’s history was notable for an 8-year history of right maxillary first premolar impaction", "Findings from computed tomography of the head demonstrated a relatively spherical 4-cm growth of the right anterior maxilla abutting the maxillary sinus and orbital floors, and enclosing an impacted premolar at the superior margin of the growth (Figure, A)", "There was no proptosis or visual field changes associated with the lesion", "An incisional biopsy was performed via an intraoral approach, and results demonstrated a thickly encapsulated proliferation of odontogenic epithelial cells with pseudoglandular elements (Figure, B and C)", "Following diagnostic confirmation, the patient was taken to the operating room, and a midface (Weber-Ferguson) incision was used to access the maxillary lesion for complete removal", "A, Sagittal computed tomography (CT) of the head and neck demonstrating an expansile, 5.0-cm mass enclosing an impacted first maxillary premolar (white arrowhead)", "B, Neoplastic proliferation of swirls and anastomosing cords of odontogenic epithelium with formation of psuedoglandular structures (black arrowheads", "hematoxylin-eosin)", "C, High-powered view of ductlike structures (black arrowhead", "hematoxylin-eosin)"], "s1": [0, 1, 2, 3, 4, 6, 8], "s2": [5, 7, 9, 10, 11, 12, 13]} {"key": 598, "questions": "What Is Your Diagnosis?", "options": [{"label": "A", "disease": "Dupuytren contracture"}, {"label": "B", "disease": "Fibroblastic rheumatism"}, {"label": "C", "disease": "Palmar fasciitis and polyarthritis syndrome"}, {"label": "D", "disease": "Scleroderma"}], "answer_idx": "C", "symptoms": ["A woman in her late 40s presented with rapidly progressive redness and stiffness of both hands, accompanied by 2 months of multiple finger joint and right shoulder pain", "Results of autoimmune serology, including antinuclear antibody, C3 and C4, rheumatoid factor, and Scl-70 antibody tests, were negative", "Raynaud phenomenon was not seen", "There was an absence of nailfold capillary changes and distal digital pitting", "The patient was treated with nonsteroidal anti-inflammatory drugs, systemic steroid therapy, and disease-modifying antirheumatic drugs (ie, hydroxychloroquine), but symptoms persisted", "Two weeks after treatment initiation, multiple tender nodules developed over the palms and interphalangeal joints, and flexion contractures continued (Figure, A)", "Incisional skin biopsy of the palmar nodules (Figure, B and C) and computed tomography (CT) were performed", "Results of the CT demonstrated bilateral ovarian tumors with highly suspect peritoneal metastases", "A, Clinical image of erythematous and edematous changes of bilateral hands and multiple nodules on palms and interphalangeal joints", "B and C, Hematoxylin-eosin–stained specimen showing increased fibroblasts with fibrosis from dermis to subcutis"], "s1": [0, 1, 2, 3, 4, 5], "s2": [6, 7, 8, 9]} {"key": 599, "questions": "What Is Your Diagnosis?", "options": [{"label": "A", "disease": "Benign lymphangioendothelioma"}, {"label": "B", "disease": "Kaposi sarcoma"}, {"label": "C", "disease": "Hobnail hemangioma"}, {"label": "D", "disease": "Low-grade angiosarcoma"}], "answer_idx": "A", "symptoms": ["An otherwise healthy man in his 50s was referred to the Department of Dermatology with a 5-year history of an asymptomatic, gradually enlarging, reddish hematomalike plaque with a brownish peripheral border in his left axilla (Figure 1)", "The patient described the spontaneous onset of the lesion and reported no previous history of trauma or any other systemic symptoms", "he had previously applied a topical antifungal cream but observed no lesion improvement", "Physical examination revealed a nonindurated and well-defined 10 × 8-cm reddish-brown annular plaque with peripheral ecchymosis", "No local lymphadenopathy was observed", "Two biopsy specimens were obtained from different areas of the lesion for immunohistochemical studies", "vascular channel endothelial cells were positive for CD31, CD34, and D2-40 and negative for human herpesvirus 8 (HHV-8) (latent nuclear antigen) and Wilms tumor 1 (WT1)", "Serologic tests for HIV and hepatitis B and C were negative", "Clinical image of the left axilla showing an asymptomatic, reddish hematomalike plaque with a brownish peripheral border"], "s1": [0, 1, 2, 3, 4, 8], "s2": [5, 6, 7]} {"key": 600, "questions": "What Is Your Diagnosis?", "options": [{"label": "A", "disease": "Langerhans cell histiocytosis"}, {"label": "B", "disease": "Rosai-Dorfman disease"}, {"label": "C", "disease": "Neuroblastoma"}, {"label": "D", "disease": "Lymphoma"}], "answer_idx": "B", "symptoms": ["A 7-year-old African American girl without significant medical history presented with progressive painless right-sided neck swelling for 2 months", "Initially, she experienced fever and sore throat", "Physical examination findings demonstrated minimally tender and nonmobile 4-cm right submandibular space swelling", "Laboratory examination findings were significant for leukocytosis with neutrophilia (white blood cell count, 19.8 K/μL", "neutrophil count, 18.2 K/μL [to convert to ×109/L, multiply by 0.001]), anemia (hemoglobin level, 9.6 g/dL [to convert to g/L, multiply by 10.0]), elevated erythrocyte sedimentation rate (>130 mm/h), and elevated C-reactive protein level (3.1 mg/dL [to convert to mg/L, multiply by 10])", "Ultrasonography revealed multiple enlarged hyperemic lymph nodes within the right side of the neck, the largest measuring 3.4 × 2.4 × 2.1 cm in the submandibular space", "She was treated with oral antibiotics without improvement", "Ultrasonography at 1-month follow-up revealed interval enlargement of the lymph nodes, which appeared rounded in shape, without preservation of internal architecture", "The mother reported that the patient lost 3.2 kg in 2 months, without any chills or night sweats, difficulty swallowing or breathing, change in voice, abdominal pain, recent travel, or exposure to animals or pets", "Magnetic resonance imaging (MRI) demonstrated extensive unilateral right-sided neck adenopathy, the largest nodes measuring approximately 4 cm at level IB", "an approximately 2-cm destructive mass involving the left maxillary malar eminence", "and additional lesions involving the T4 vertebra and upper sternum (Figure, A and B)", "Fluorodeoxyglucose 18F-labeled positron emission tomography with computed tomography (FDG PET/CT) demonstrated extensive FDG-avid right-sided cervical lymphadenopathy and innumerable FDG-avid lytic foci involving the axial and appendicular skeleton (Figure, C)", "A, Postcontrast T1-weighted fat-suppressed magnetic resonance imaging (MRI) of the brain demonstrated an approximately 2-cm destructive mass involving the left malar eminence of maxilla with homogeneous enhancement", "B, Axial T2-weighted fat-suppressed MRI through the neck demonstrated unilateral right-sided neck adenopathy", "C, Fluorodeoxyglucose F18-labeled positron emission tomography whole-body imaging demonstrated avid right cervical lymphadenopathy and innumerable avid foci involving the axial and appendicular skeleton"], "s1": [0, 1, 2, 6, 8], "s2": [3, 4, 5, 7, 9, 10, 11, 12, 13, 14, 15]} {"key": 601, "questions": "What Is Your Diagnosis?", "options": [{"label": "A", "disease": "Treat with chemotherapy for disease recurrence because the agents available for recurrent ROS1 tumors remain limited"}, {"label": "B", "disease": "Administer antibiotics with coverage for pneumonia pathogen for 7 to 10 days and a repeated CT scan in 4 weeks"}, {"label": "C", "disease": "Administer steroid treatment and discontinue crizotinib and other tyrosine kinase inhibitors (TKIs) permanently because TKI-induced interstitial lung disease is highly suspected"}, {"label": "D", "disease": "Perform a CT-guided biopsy for histological confirmation of recurrence or identification of other benign disease, such as granulomatous inflammation"}], "answer_idx": "D", "symptoms": ["A 69-year-old man with a smoking history and complaints of back pain and cough for 2 weeks was found to have a right lung mass on chest radiography results", "A chest computed tomography (CT) scan showed a right upper lobe lung mass with multiple lymphadenopathies (N3), and bone scan and brain magnetic resonance imaging results showed multiple metastatic foci", "A biopsy was performed, and the pathologic report showed a poorly differentiated lung adenocarcinoma with ROS1 rearrangement identified by immunohistochemical staining (clone: D4D6", "H-score: 280) and further confirmative fluorescence in situ hybridization", "The patient received ceritinib, 450 mg/day, and his tumor and brain and bone metastatic lesions subsequently decreased in size", "The ceritinib was then shifted to crizotinib, 500 mg/day, when the reimbursement of crizotinib as a first-line therapy was approved by Taiwan's National Health Insurance system", "Three weeks after the first dose of crizotinib, the patient developed a dry cough without fever or purulent sputum", "There was no significant finding on physical examination except mild crackles over the right lung field, although chest radiography results showed a new consolidation lesion over the right upper lobe", "Meanwhile, the results of mycobacterial and bacteriologic surveys were unremarkable", "The tumor marker carcinoembryonic antigen level decreased from an initial 14.76 to 0.71 ng/mL (reference range, 0-5.00 ng/mL [to convert to micrograms per liter, multiply by 1])", "A CT scan was arranged, and developing lung cancer with lymphangitic carcinomatosis was highly suspected by the radiologist (Figure, A)", "A bronchoscopy with bronchial brushing and biopsy was performed, and no pathogen was identified", "A, The comparison of the chest computed tomographic (CT) scan of the new consolidation lesion (red arrowheads) mimicking recurrence (right panel) with the primary tumor (yellow arrowhead) in the initial CT scan (left panel)", "B, Microscopic findings of a CT-guided biopsy (left panel) and CT scan after resuming ceritinib treatment (right panel)", "Granulomatous inflammation or sarcoid-like reaction with predominant T-cell infiltration (hematoxylin and eosin stain, magnification ×200) and less B cell infiltration", "Inset: double immunohistochemical staining showing abundant T cells (CD3, red staining", "red arrowheads) and rare B cell infiltration (CD20, brown staining", "yellow arrowhead)", "The CT scan after resuming ceritinib treatment for 2 months showed decreased lung consolidation and the primary tumor (yellow arrowhead)", "Treat with chemotherapy for disease recurrence because the agents available for recurrent ROS1 tumors remain limitedAdminister antibiotics with coverage for pneumonia pathogen for 7 to 10 days and a repeated CT scan in 4 weeksAdminister steroid treatment and discontinue crizotinib and other tyrosine kinase inhibitors (TKIs) permanently because TKI-induced interstitial lung disease is highly suspectedPerform a CT-guided biopsy for histological confirmation of recurrence or identification of other benign disease, such as granulomatous inflammation"], "s1": [0, 1, 2, 3, 4, 5, 9, 10, 11, 19], "s2": [6, 7, 8, 12, 13, 14, 15, 16, 17, 18]} {"key": 602, "questions": "What Is Your Diagnosis?", "options": [{"label": "A", "disease": "Retinoblastoma"}, {"label": "B", "disease": "Rhabdomyosarcoma"}, {"label": "C", "disease": "Atypical teratoid/rhabdoid tumor"}, {"label": "D", "disease": "Medulloepithelioma"}], "answer_idx": "C", "symptoms": ["A male newborn was noted to have right eye and right cheek masses at birth", "His prenatal course was complicated by maternal drug use, hepatitis C, and lack of routine prenatal care", "Intraocular pressure of the right eye was elevated to 31 mm Hg (normal pressure, <20 mm Hg)", "Parotid ultrasonography demonstrated a lobulated solid vascular mass measuring 2.1 cm", "The infant was referred to the ophthalmology and otolaryngology service", "At the age of 5 weeks, a swollen right eye with a foggy cornea and conjunctival vascular anomaly were noted", "A soft, mobile, right parotid mass had doubled in size", "Ocular B-scan ultrasonography revealed a large subretinal lesion with multiple intralesional, hyperechoic foci extending temporally from the optic disc to the periphery", "Results of magnetic resonance imaging (MRI) showed an intraocular mass in the right eye, with faint contrast enhancement thought to represent intratumoral calcification or hemorrhage, and a well-circumscribed, 3.1-cm, right superficial parotid mass with faint enhancement (Figure 1)", "18F-fludeoxyglucose–positron emission tomography (FDG-PET) showed hypermetabolic masses at the posterior aspect of the right globe, the lateral aspect of the orbit, and the right parotid gland", "Computed tomography of the chest, abdomen, and pelvis provided no evidence of distant metastasis", "Cerebral spinal fluid analysis and examination of a bone marrow biopsy specimen showed no suspicious findings", "A pathological specimen from the parotid gland featured small blue cells with a fine chromatin granular pattern and complete loss of integrase interactor 1 (INI-1)", "Axial magnetic resonance image showing a right orbital mass (arrowhead) with faint contrast enhancement"], "s1": [0, 1, 4, 10, 11], "s2": [2, 3, 5, 6, 7, 8, 9, 12, 13]} {"key": 603, "questions": "What Is Your Diagnosis?", "options": [{"label": "A", "disease": "Tracheal granuloma"}, {"label": "B", "disease": "Metastatic rectal adenocarcinoma"}, {"label": "C", "disease": "Subglottic squamous cell carcinoma"}, {"label": "D", "disease": "Subglottic papilloma"}], "answer_idx": "B", "symptoms": ["The patient is a 48-year-old man with a history of a rectal adenocarcinoma (cT3N2bM0) diagnosed 9 years prior", "Following neoadjuvant chemoradiotherapy, low anterior resection, and 6 cycles of adjuvant FOLFOX therapy (fluorouracil, leucovorin, and oxaliplatin), he had developed a lung metastasis (KRAS wild type) 1 year later", "He subsequently developed multiple lung metastases treated with palliative local interventions (lobectomy, radiofrequency ablation) and 4 lines of chemotherapy", "Owing to symptomatic lung disease, he was enrolled in a clinical trial (NCT03502733) following informed consent and achieved a confirmed partial response", "After 9 months, contrast computed tomography restaging scans noted a new, 1-cm posterior subglottic lesion of unclear origin (Figure, A)", "He had no shortness of breath or noisy breathing, dysphagia, voice change, or hemoptysis", "A partial response was maintained without evidence of progression of lung disease", "A, An axial computed tomographic image reveals a new posterior subglottic lesion (pink arrowhead) of unclear etiology", "B, In the operating room, the larynx was exposed with a Lindholm laryngoscope, and a Hopkins rod-lens endoscope was used to photograph the lesion under apneic conditions", "C, Photomicrograph (original magnification ×20) of a section of the mass stained for CDX2 via standard immunohistochemical techniques", "Otolaryngology was consulted to assess the lesion", "The differential diagnosis based on imaging included a mass vs inspissated mucus", "Examination using distal-chip flexible laryngoscopy revealed a 1-cm, broad-based, exophytic mass along the posterior wall of the subglottis with prominent vasculature", "No other anatomic abnormalities in the upper aerodigestive tract or cervical lymphadenopathy were noted", "The decision was made to proceed to the operating room for definitive endoscopic biopsy under general anesthesia (Figure, B)", "Examination revealed a subglottic mass that did not extend inferiorly beyond the caudal aspect of the cricoid cartilage", "The mass was debulked and sent to pathology for analysis, including hematoxylin-eosin and immunohistochemical staining (Figure, C)"], "s1": [0, 1, 2, 3, 6], "s2": [4, 5, 7, 8, 9, 10, 11, 12, 13, 14, 15, 16]} {"key": 604, "questions": "What Is Your Diagnosis?", "options": [{"label": "A", "disease": "Carbon monoxide poisoning"}, {"label": "B", "disease": "Methanol poisoning"}, {"label": "C", "disease": "Organophosphate poisoning"}, {"label": "D", "disease": "Fahr disease"}], "answer_idx": "C", "symptoms": ["A 69-year-old woman returning from a vacation in Cuba was brought to the emergency department directly from the airport", "Her medical history was unremarkable, and she had no allergies", "She had been well until 2 hours before the flight home, when at the Havana airport, she developed generalized weakness, increased sweating, severe nausea, and vomiting", "In flight, she had lethargy, vomiting, and urinary incontinence", "On arrival to the emergency department, she was stuporous and required intubation", "Initial laboratory tests showed a hemoglobin level of 10.9 g/dL (reference range, 12.0-15.2 g/dL [to convert to grams per liter, multiply by 10.0]), a creatinine level of 1.31 mg/dL (reference range, 0.51-1.08 [to convert to micromoles per liter, multiply by 88.4]), and a pH of 7.22 with a normal anion gap", "Serum electrolyte levels", "a white blood cell count", "levels of glucose, creatine kinase, lactic acid, and thyrotropin", "coagulation studies", "and liver function tests had normal results", "An electrocardiogram showed sinus bradycardia at 53 bpm and borderline QT interval prolongation (QTc", "465 milliseconds)", "A urine toxicology screen had negative results for acetaminophen, salicylates, opiates, barbiturates, benzodiazepines, cocaine, and ethanol", "Cerebrospinal fluid was clear, with normal levels of glucose, total protein, and white blood cells and negative microbiology and cytology testing results", "A malaria smear and antigen test, respiratory viral assay, and cultures of cerebrospinal fluid, blood, and urine had negative results", "Computed tomography of the head showed bilateral hyperdensity of the globus pallidi (Figure 1)", "Within several hours, her level of consciousness improved spontaneously, and she was extubated", "Her blood pressure was 99/61 mm Hg", "pulse, 80 bpm", "temperature, 37.6 °C", "and oxygen saturation, 99% on room air", "The patient was alert but disoriented", "The pupils were 2 mm and reactive", "Involuntary facial muscle twitching was noted", "Her muscle tone was normal, and she had mild proximal muscle weakness with occasional bilateral upper extremity and lower extremity fasciculations and jerky movements", "Deep tendon reflexes were normal, with flexor plantar responses", "Her sensations to pain and vibration were normal", "Computed tomography of the head showing bilateral hyperdensity of the globus pallidi"], "s1": [0, 2, 3, 4, 17, 18, 19, 20, 21, 22, 24, 25], "s2": [1, 5, 6, 7, 8, 9, 10, 11, 12, 13, 14, 15, 16, 23, 26, 27, 28]} {"key": 605, "questions": "What Is Your Diagnosis?", "options": [{"label": "A", "disease": "Chondroid syringoma"}, {"label": "B", "disease": "Cutaneous meningeal heterotopia"}, {"label": "C", "disease": "Scalp metastasis"}, {"label": "D", "disease": "Proliferative nodule in a congenital melanocytic nevus"}], "answer_idx": "B", "symptoms": ["A woman in her early 70s was referred to the dermatology department for evaluation of a solitary, asymptomatic lump on her scalp", "She was born with the lesion but noticed some growth following a minor local trauma a couple of months prior to seeking medical evaluation", "The patient denied local inflammation, purulent discharge, and systemic symptoms", "Her medical history was unremarkable apart from an allegedly benign breast nodule resection in her teenage years", "In addition, her father died of metastatic cutaneous melanoma at 83 years old", "Clinically, there was an ill-defined, infiltrated plaque with a normochromic, exophytic nodule on her scalp vertex measuring 4.2 × 3.8 cm (Figure, A)", "It had a waxy surface with irregular transverse furrows and overlying alopecia", "Dermoscopic evaluation findings revealed a homogeneous yellow hue with no visible vascular or pigmented structures and absence of follicular ostia", "The lesion was firm and mildly tender to the touch and seemed adherent to the underlying bone", "There were no palpable lymph nodes", "A brain magnetic resonance imaging was performed for further elucidation (Figure, B), as well as a punch biopsy (Figure, C and D)", "A, Clinical image of the lesion on the scalp", "B, Paramedian sagittal gadolinium-enhanced T1-weighted brain magnetic resonance imaging (MRI)", "C, Cellular infiltration in the reticular dermis (hematoxylin-eosin, original magnification ×100)", "D, Monomorphic cells in a syncytial configuration surrounding collagen bundles (hematoxylin-eosin, original magnification ×400)"], "s1": [0, 1, 3, 4, 5, 6, 7, 8, 9], "s2": [2, 10, 11, 12, 13, 14]} {"key": 606, "questions": "What Is Your Diagnosis?", "options": [{"label": "A", "disease": "Drug reaction with eosinophilia and systemic symptoms"}, {"label": "B", "disease": "Eosinophilic dermatosis of hematologic malignancy"}, {"label": "C", "disease": "Urticarial vasculitis"}, {"label": "D", "disease": "Urticarial bullous pemphigoid"}], "answer_idx": "B", "symptoms": ["A woman in her 50s with a recent diagnosis of acute myeloid leukemia (subtype M5a, French-American-British classification) was admitted to the hospital for induction chemotherapy", "At presentation she had a 3-week history of a striking pruritic and generalized cutaneous eruption", "The patient otherwise felt well and denied fever, malaise, insect bites, or starting new drug treatments", "Except for leukopenia and a slightly elevated C-reactive protein level, results of other routine blood and urine tests were unremarkable", "Physical examination revealed an erythematous eruption composed of small indurated and edematous papules (Figure, A and B)", "Confluent papules leading to large urticariform plaques, some with annular configuration, were present on the flanks, lower legs, breasts, and scalp", "Hyperpigmented patches and focal areas of bruiselike discoloration were seen adjacent to acute lesions", "Mucous membranes, palms, and soles were spared", "A biopsy specimen was obtained (Figure, C and D)", "A, Erythematous indurated and edematous papules and plaques on the thighs with some residual discoloration", "B, Large erythematous plaques involving the scalp and ear", "C, Hematoxylin-eosin–stained histologic specimen showing a superficial and deep dermal perivascular and interstitial mononuclear infiltrate with numerous eosinophils (original magnification ×40)", "D, Higher magnification of C showing an abundant number of eosinophils in the infiltrate (original magnification ×400)"], "s1": [0, 3, 8, 11, 12], "s2": [1, 2, 4, 5, 6, 7, 9, 10]} {"key": 607, "questions": "What Is Your Diagnosis?", "options": [{"label": "A", "disease": "Cytomegalovirus ileitis"}, {"label": "B", "disease": "Capecitabine-induced terminal ileitis"}, {"label": "C", "disease": "Ileal metastasis"}, {"label": "D", "disease": "Small bowel carcinoid tumor"}], "answer_idx": "B", "symptoms": ["A 72-year-old man with recently diagnosed metastatic appendiceal adenocarcinoma undergoing chemotherapy with capecitabine plus oxaliplatin presented with lower abdominal pain and watery, large-volume diarrhea up to 15 times per day for the previous 3 weeks", "His diarrhea was nonbloody, woke him up from sleep, was accompanied by crampy abdominal pain, and was not associated with nausea or vomiting", "The diarrhea persisted despite treatment with combinations of loperamide, atropine/diphenoxylate, colestipol, and tincture of opium", "He required repeated intravenous infusions for fluid and electrolyte repletion", "His physical examination results revealed normal vital signs and a mildly tender abdomen with hyperactive bowel sounds, but no peritoneal signs", "His laboratory workup results were notable for severe hypokalemia and hypomagnesemia, as well as positive fecal lactoferrin and elevated fecal calprotectin levels of 646 μg/g", "His infectious workup results were unremarkable", "A computed tomography scan of the abdomen/pelvis revealed diffuse long-segment wall thickening involving nearly the entire terminal ileum (Figure, A)", "An ileocolonoscopy revealed erythematous and congested mucosa with scattered erosions and ulcerations throughout the examined terminal ileum (Figure, B)", "Targeted biopsies were obtained (Figure, C)", "Contrasted computed tomography of the abdomen and pelvis (A), colonoscopy (B), and histopathology (hematoxylin-eosin, original magnification ×100) (C)"], "s1": [0, 1, 2, 3, 5], "s2": [4, 6, 7, 8, 9, 10]} {"key": 608, "questions": "What Is Your Diagnosis?", "options": [{"label": "A", "disease": "Odontogenic keratocyst"}, {"label": "B", "disease": "Ameloblastoma"}, {"label": "C", "disease": "Clear cell odontogenic carcinoma"}, {"label": "D", "disease": "Salivary clear cell carcinoma"}], "answer_idx": "C", "symptoms": ["A man in his 40s presented with mobility of the anterior mandibular dentition", "A consultation with a dentist 1 year prior revealed an impacted mandibular canine (No", "22)", "He denied any pain, swelling, or paresthesia, and his medical history was unremarkable", "Clinical examination revealed fullness on the mentolabial fold but no gross facial asymmetry or mental nerve paresthesia", "Intraorally, there was an expansile mass of the left anterior mandible with ill-defined margins, which crossed the midline and obliterated the gingivolabial sulcus", "Results of dental cone beam computed tomography imaging revealed a 4.3 × 4.0-cm radiolucent mandibular lesion with an unerupted mandibular canine in the center of the lesion, cortical destruction, and root deviation of teeth Nos", "23 through 26 (Figure, A)", "An incisional biopsy was performed, and the patient underwent radical mandibular resection 2 months later", "Histological sections from the resection demonstrated nests and cords of polygonal cells with clear to eosinophilic cytoplasm embedded in a dense collagenous stroma (Figure, B)", "The lesion was centered in the cortical bone (Figure, C) and extended into the surrounding mucosa (Figure, D), fibroadipose tissue, and skeletal muscle", "Perineural invasion was identified (Figure, E), and lymphovascular invasion was absent", "The tumor involved multiple margins", "Immunohistochemistry findings revealed the tumor cells were positive for p40. Fluorescence in situ hybridization studies demonstrated an EWSR1 gene rearrangement in 60% of the tumor cells (Figure, F)", "A, Dental cone beam computed tomography (CT) scan showing 4.3 × 4.0-cm radiolucent lesion in the anterior mandible with impacted mandibular canine at the center of the lesion", "B, Nests and cords of polygonal cells with clear to eosinophilic cytoplasm (hematoxylin-eosin stain)", "C, Lesion centered in the bone (hematoxylin-eosin stain)", "D, Tumor cells extending beyond the cortex of the mandible to involve the gingival mucosa (hematoxylin-eosin stain)", "E, Perineural invasion is present (hematoxylin-eosin stain)", "F, Fluorescence in situ hybridization for EWSR1 gene rearrangement highlighted separation of the red and green signals in 60% of cells"], "s1": [0, 1, 2, 3, 4, 5, 6, 7], "s2": [8, 9, 10, 11, 12, 13, 14, 15, 16, 17, 18, 19]} {"key": 609, "questions": "What Is Your Diagnosis?", "options": [{"label": "A", "disease": "Pagetoid Bowen disease"}, {"label": "B", "disease": "Inverse lichen planus"}, {"label": "C", "disease": "Pigmented extramammary Paget disease"}, {"label": "D", "disease": "Superficial spreading melanoma"}], "answer_idx": "C", "symptoms": ["A woman in her 70s presented with an enlarged and variegated plaque on her left axilla, which she had had for 4 years", "The lesion started as a small eczematous, pink-to-brown plaque", "As the lesion was asymptomatic and slow growing, she did not initially seek treatment", "Recently, the color of the lesion had changed to an alarming dark brown color (Figure, A)", "No breast mass or lymphadenopathy was observed", "Her medical and family history was unremarkable", "A skin biopsy specimen was obtained and submitted for further histopathologic analysis", "A, Clinical presentation of pigmented plaque in the axilla with well-demarcated margins", "B, Histologic findings revealed an intraepidermal neoplasm consisting of pagetoid cells, characterized by round, pale, vacuolated cytoplasm, and large pleomorphic nuclei (hematoxylin-eosin", "original magnification ×200)", "C, Immunohistochemical findings showed positive staining for EMA (original magnification ×200)", "D, The lesion was surrounded by numerous reactive dendritic HMB-45–positive melanocytes scattered among the tumor cells (original magnification ×200)"], "s1": [0, 1, 2, 3, 4, 5, 7], "s2": [6, 8, 9, 10, 11]} {"key": 610, "questions": "What Is Your Diagnosis?", "options": [{"label": "A", "disease": "Onchocercoma"}, {"label": "B", "disease": "Subcutaneous dirofilariasis"}, {"label": "C", "disease": "Cutaneous loiasis"}, {"label": "D", "disease": "Subcutaneous hydatid cyst"}], "answer_idx": "B", "symptoms": ["An otherwise healthy man in his 40s presented with a 3-month history of an asymptomatic nodule on his scalp", "There were no systemic complaints, history of arthropod bite, immunosuppression, travel abroad, or contact with animals in the recent past", "Physical examination revealed a skin-colored, firm, nontender, freely mobile 2 × 1.5-cm nodule with overlying normal skin on the right occipital region of the scalp (Figure, A)", "There was no lymphadenopathy", "No other cutaneous lesions were present", "A, Clinical photograph shows a 2 × 1.5-cm skin-colored nodule on the occiput", "B, Epidermis is unremarkable", "A nodular lesion surrounded by a fibrous capsule is seen in the dermis with cross-sections of many parasites along with pink material in the nodule (hematoxylin-eosin, original magnification ×2)", "C, The parasite morphology depicts a thick cuticle, well-developed inner muscular layer, tubular structures containing microfilariae, and necrotic debris (hematoxylin-eosin, original magnification ×20)", "D, Higher magnification shows spinous processes on the outer surface of the cuticle (hematoxylin-eosin, original magnification ×20)", "The nodule was completely excised and sent for histopathologic examination, which revealed a nodule in the dermis with an unremarkable epidermis", "Cross-sections of parasites were present, surrounded by a fibrous capsule (Figure, B)", "The parasite showed an outer cuticle, muscle layer underneath, and gravid uterus filled with microfilariae (Figure, C)", "The cuticle demonstrated a wavy pattern of longitudinal ridges on its outer surface (Figure, D)", "The fibrous capsule contained mixed inflammatory infiltrate of lymphocytes, eosinophils, histiocytes, and plasma cells", "Findings of slitlamp examination and fundoscopy of the eyes were normal", "Findings of routine hematologic and biochemical investigations were normal", "No microfilaria was seen in peripheral blood", "Findings of chest radiography and abdomen ultrasonography were unremarkable"], "s1": [0, 1, 2, 3, 4, 5, 10, 15, 16, 17, 18], "s2": [6, 7, 8, 9, 11, 12, 13, 14]} {"key": 611, "questions": "What Is Your Diagnosis?", "options": [{"label": "A", "disease": "Relapsing polychondritis"}, {"label": "B", "disease": "Granulomatosis with polyangiitis"}, {"label": "C", "disease": "Eosinophilic granulomatosis with polyangiitis"}, {"label": "D", "disease": "Sarcoidosis"}], "answer_idx": "A", "symptoms": ["A healthy 21-year-old Chinese man with no relevant medical history presented to multiple general practitioners for a recurrent sore throat associated with hoarseness of voice for 3 months’ duration", "Despite having taken multiple courses of oral antibiotics, he saw no improvement in his symptoms", "Over the course of 3 months, the patient went on to notice swelling and redness of his left ear (Figure 1A) that would resolve and recur every few weeks", "Clinical photographs demonstrating auricular chondritis (A) and saddle nose deformity (B)", "Bothered by his hoarse voice and sore throat, he consulted a private otolaryngologist, who performed a nasoendoscopy, the results of which showed severe Reinke edema and chronic laryngitis", "He was treated with a short course of oral prednisolone that alleviated all his symptoms", "However, on cessation of oral steroids, the patient noticed a saddle-shaped deformity on his nose (Figure 1B) that appeared without any antecedent trauma", "In addition, his hoarseness of voice worsened during a 2-week span and was now associated with difficulty breathing when he laid supine", "In our otolaryngology clinic, physical examination results revealed a saddle-nose deformity, left pinna perichondritis, and a soft inspiratory stridor", "He was unable to speak a full sentence without drawing a deep, labored breath", "Laboratory test results showed elevated C-reactive protein levels of 143 mg/L, erythrocyte sedimentation rate of 97 mm/h, and a microcytic hypochromic anemia of hemoglobin level of 12.1 g/dL (to convert to grams per liter, multiply by 10)", "The results for antinuclear antibody, anti–double-stranded DNA, and antineutrophil cytoplasmic antibodies were negative"], "s1": [0, 1, 4, 5, 7, 9], "s2": [2, 3, 6, 8, 10, 11]} {"key": 612, "questions": "What Is Your Diagnosis?", "options": [{"label": "A", "disease": "Tuberculosis"}, {"label": "B", "disease": "Sarcoidosis"}, {"label": "C", "disease": "Epiglottitis"}, {"label": "D", "disease": "Granulomatosis with polyangiitis"}], "answer_idx": "B", "symptoms": ["A 9-year-old, vaccinated boy with glucose-6-phosphate dehydrogenase deficiency and β-thalassemia trait presented with 1 month of progressive dyspnea, snoring, dysphagia, and weight loss", "Both parents had a history of latent tuberculosis infection treated after immigrating to the US from Syria shortly after the patient’s birth", "The patient’s pediatrician diagnosed him with reactive airway disease and adenotonsillar hypertrophy", "He was referred to the otolaryngology clinic, where flexible laryngoscopy revealed diffuse upper airway lymphoid hyperplasia and supraglottic edema", "He was sent to the emergency department, where he was stridulous and afebrile with a normal white blood cell count and an elevated erythrocyte sedimentation rate", "A computed tomography scan of the neck revealed diffuse edema and contrast enhancement of the nasopharynx, oropharynx, hypopharynx, and supraglottis", "A chest radiograph was normal", "The patient was taken urgently to the operating room for direct microlaryngoscopy (Figure 1A and B) and intubated", "Results of biopsies and cultures of the tonsils, adenoids, and supraglottis revealed diffuse non-necrotizing granulomas (Figure 1C) and oropharyngeal flora with negative acid-fast bacilli and Grocott methenamine silver stains", "He remained intubated, and treatment with systemic corticosteroids and antibiotics was initiated", "During this time, he had a normal angiotensin-converting enzyme (ACE) level and normal titers for antinuclear antibody (ANA), cytoplasmic antineutrophilic cytoplasmic antibody, perinuclear antineutrophilic cytoplasmic antibody, antimyeloperoxidase antibody, antiproteinase 3 antibody, anti–double-stranded DNA antibody, Smith antibody, Sjogren antibodies, and histoplasma antibodies", "He also had normal results on tuberculin purified protein derivative and interferon-γ release assay (QuantiFERON", "Qiagen)", "A, View with laryngoscope blade in vallecula, revealing only a profoundly swollen epiglottis", "B, View with laryngoscope blade beneath epiglottis, revealing edema and distortion of supraglottic structures that obscured the glottis (* indicates left arytenoid", "**, right aryepiglottic fold", "***, right arytenoid)", "C, Supraglottic biopsy specimen demonstrating non-necrotizing granulomas in a background of lymphoid tissue", "hematoxylin-eosin stain, original magnification ×200."], "s1": [0, 1, 2, 9, 10, 11], "s2": [3, 4, 5, 6, 7, 8, 12, 13, 14, 15, 16, 17, 18]} {"key": 613, "questions": "What Is Your Diagnosis?", "options": [{"label": "A", "disease": "Cancer-associated systemic lupus erythematosus"}, {"label": "B", "disease": "Cancer-associated pancreatitis, panniculitis, and polyarthritis syndrome (PPP) syndrome"}, {"label": "C", "disease": "Dermatomyositis"}, {"label": "D", "disease": "Leukocytoclastic vasculitis"}], "answer_idx": "B", "symptoms": ["A man in his 30s presented with a 3-month history of fourth finger right hand articular heat, pain, redness, and swelling", "Similar symptoms subsequently spread to the left hallux and ankle, and he developed a fever", "Despite many medical consultations and various attempts at treatment with nonsteroidal anti-inflammatories, the patient developed widespread arthritis within the following month, which was predominant in the knee, shoulder, and hip", "In the month before presentation the articular symptoms worsened, the joints had become tender, and ill-defined skin nodules had appeared predominantly on the legs (Figure, A and B)", "An acute abdominal pain in the left hypochondrium developed, accompanied by nausea and vomiting, which led to a diagnosis of pancreatitis", "The patient was admitted to the hospital and clinical measures relieved the pain", "Laboratory test results revealed strikingly high levels of lipase and amylase, as well as normal levels of cancer antigen 19-9. Bone computed tomography (CT) of the right shoulder and knee demonstrated cortical bone destruction and intraosseous medullary necrosis (Figure, C and D)", "An abdominal CT was performed (Figure, E), and the patient underwent surgery", "Cancer-associated pancreatitis, panniculitis, and polyarthritis syndrome (PPP) syndrome"], "s1": [0, 1, 2, 3], "s2": [4, 5, 6, 7, 8]} {"key": 614, "questions": "What Is Your Diagnosis?", "options": [{"label": "A", "disease": "Juxtafacet cyst"}, {"label": "B", "disease": "Mucus retention cyst"}, {"label": "C", "disease": "Tornwaldt cyst"}, {"label": "D", "disease": "Spinal bronchogenic cyst"}], "answer_idx": "A", "symptoms": ["A 73-year-old woman with a history of chronic cervical neck pain presented with complaints of globus sensation, dysphagia, and localized throat swelling", "On examination, she had an approximately 1.5-cm mucosally covered mass on her left lateral pharyngeal wall that was cystlike in appearance", "The remainder of her examination findings were normal", "The patient reported that this cyst had been drained 2 years before with resolution of her symptoms", "Contrasted computed tomographic (CT) imaging of her head and neck was obtained, which revealed a pharyngeal cyst that extended to the atlantoaxial joint (Figure, A)", "Further magnetic resonance imaging (MRI) of the area was obtained, which demonstrated communication of the cyst with the atlantoaxial joint (Figure, B and C)", "Computed tomographic (CT) image (A) and magnetic resonance images (MRIs) (B and C) of the neck show a pharyngeal cyst (all images) that is communicating with the left atlantoaxial joint (B) at the level of the second cervical vertebra (C)"], "s1": [0, 1, 3], "s2": [2, 4, 5, 6]} {"key": 615, "questions": "What Is Your Diagnosis?", "options": [{"label": "A", "disease": "Eosinophilic granulomatosis with polyangiitis"}, {"label": "B", "disease": "IgG4-related disease"}, {"label": "C", "disease": "Eosinophilic angiocentric fibrosis"}, {"label": "D", "disease": "Chronic lymphoplasmacytic rhinosinusitis"}], "answer_idx": "B", "symptoms": ["A 19-year-old White woman who was previously in good health, with no reported history of drug abuse, presented with progressive bilateral nasal airway obstruction, crusting, and saddle nose deformity over a 6-month period", "The patient denied systemic symptoms, including oral ulcers, sicca, photosensitivity, dermatitis, or arthralgias", "Her medical history had negative results for allergic rhinitis, asthma, chronic rhinosinusitis, epistaxis, nasal trauma, or surgery", "Her family history was noncontributory", "An in-office nasal endoscopy demonstrated synechiae bridging from the septum to the lateral nasal walls, polypoid changes, and yellow crusting bilaterally", "Her immune workup showed elevated complement C3 (200 mg/dL) and total hemolytic complement (CH50", ">60 U/mL) levels, with otherwise unremarkable levels of C4, C-reactive protein, erythrocyte sedimentation rate, rheumatoid factor, Sjögren antibodies, angiotensin-converting enzyme, antinuclear antibody, antineutrophil cytoplasmic antibody, bullous pemphigoid IgG antibodies, and quantitative immunoglobulins and IgG subclasses (IgG4, 53 mg/dL [to convert to grams per liter, multiply by 0.01])", "Nasal cultures had positive results for methicillin-resistant Staphylococcus aureus and Bacteroides", "Preoperative computed tomography showed no evidence of paranasal mucosal disease or bone erosion", "An operative endoscopy with elevation of the mucoperichondrial flaps revealed an absence of the quadrangular cartilage caudal struts and irregular, friable tissue along the cartilage and perichondrium", "Surgical pathology examination demonstrated prominent lymphoplasmacytic infiltrate (Figure, A), angiocentric fibrosis with a relative lack of eosinophils (Figure, B), and increased IgG4 absolute count (up to 50 cells per high-power field), although an IgG4-IgG ratio that was only slightly elevated (0.2) (Figure, C and D)", "Histopathologic findings: A, hematoxylin-eosin (original magnification ×40)", "B, hematoxylin-eosin (original magnification ×100)", "C and D, original magnification ×200."], "s1": [0, 4, 7, 8, 9], "s2": [1, 2, 3, 5, 6, 10, 11, 12, 13]} {"key": 616, "questions": "What Is Your Diagnosis?", "options": [{"label": "A", "disease": "Erythema gyratum repens associated with lung adenocarcinoma"}, {"label": "B", "disease": "Erythema gyratum repens–like eruption associated with mycosis fungoides"}, {"label": "C", "disease": "Erythema gyratum repens associated with pityriasis rubra pilaris"}, {"label": "D", "disease": "Tinea imbricata"}], "answer_idx": "B", "symptoms": ["A man in his 60s presented to the dermatology clinic with a widespread, intermittently pruritic eruption", "Examination showed coalescing annular, arcuate, and serpiginous red scaling plaques in a concentric pattern diffusely involving the chest, back, and abdomen with extension to proximal extremities (Figure, A and B)", "Punch biopsies from the chest (Figure, C and D) and back were obtained for histopathology and tissue culture", "Treatment with topical corticosteroids under wet dressings was associated with partial improvement in the eruption and pruritus", "A and B, Clinical images show concentric annular, arcuate, and serpiginous red scaling plaques diffusely involving the back and right flank", "C, Histopathology of skin biopsy from the chest reveals epidermal hyperplasia with spongiosis and parakeratosis (hematoxylin-eosin, original magnification ×400)", "There is a superficial dermal and epidermotropic infiltrate of atypical lymphocytes focally distributed along the dermal-epidermal junction and aggregated in Pautrier microabscesses", "D, CD30 immunohistochemical stain (original magnification ×400) highlighting atypical lymphocytes", "Erythema gyratum repens–like eruption associated with mycosis fungoidesErythema gyratum repens associated with pityriasis rubra pilaris"], "s1": [0, 1, 3, 4, 8], "s2": [2, 5, 6, 7]} {"key": 617, "questions": "What Is Your Diagnosis?", "options": [{"label": "A", "disease": "Schwannoma"}, {"label": "B", "disease": "Paraganglioma"}, {"label": "C", "disease": "Persistent trigeminal artery"}, {"label": "D", "disease": "Persistent hypoglossal artery"}], "answer_idx": "D", "symptoms": ["A 62-year-old man with a prior 10–pack-year history of smoking presented with a 3-cm right level IIa lymph node", "Fine-needle aspiration results showed metastatic p16-positive squamous cell carcinoma", "No mucosal primary site was identified with positron emission tomography, computed tomography, or a head and neck examination", "The patient underwent direct laryngoscopy with directed biopsies and bilateral tonsillectomy", "Pathology results revealed a 1.8-cm primary cancer in the right tonsil, and the patient received a stage designation of T1 N1 M0. The tumor was intracapsular, and all surgical margins were negative", "Multidisciplinary tumor board discussion concluded that surgical and nonsurgical treatment options were available, and the patient elected to proceed with further surgery, which would entail additional directed biopsies at the primary site with additional resection if necessary and selective neck dissection of levels Ib to IV", "In preparation for the neck dissection, computed tomography of the neck with contrast was obtained", "Imaging results revealed an enhancing structure at the skull base adjacent to the carotid sheath (Figure)", "Computed tomography of the neck with contrast and the axial (A) and coronal (B) views revealing an enhancing structure at the skull base along the carotid sheath (arrowheads)"], "s1": [0, 1, 2, 4, 5], "s2": [3, 6, 7, 8]} {"key": 618, "questions": "What Is Your Diagnosis?", "options": [{"label": "A", "disease": "Granulomatosis with polyangiitis"}, {"label": "B", "disease": "Cryptococcal infection"}, {"label": "C", "disease": "Sweet syndrome"}, {"label": "D", "disease": "Drug-induced vasculitis"}], "answer_idx": "D", "symptoms": ["A woman in her 70s presented with a week of painless, rapidly progressive, edematous, necrotic papules and plaques on the extremities, buttocks, and face with sharp demarcation at the nasolabial folds and extension into the oral mucosa (Figure, A and B)", "Notable symptoms included 4 months of extreme fatigue, 20.4 kg weight loss, and a recent episode of hematochezia", "Her medical history included diabetes, hyperlipidemia, and hypertension that were well controlled with simvastatin, chlorthalidone, hydralazine hydrochloride, lisinopril, and verapamil hydrochloride", "She had been taking these medications for 5 years with no new exposures", "Age-appropriate cancer screening was up to date", "She was afebrile and hemodynamically stable, but laboratory evaluation revealed that she had leukocytosis, normocytic anemia, elevated erythrocyte sedimentation rate, hematuria, and proteinuria", "On hospital day 2, she developed bright red blood per rectum, necessitating transfusion", "A computed tomography of the chest, abdomen, and pelvis revealed parenchymal lung nodules", "On admission, punch biopsies of the skin were performed for both histological examination and bacterial, fungal, and acid-fast bacterial cultures (Figure, C and D)", "A, Edematous, ulcerated pink plaques confluent on forehead and cheeks with sharp demarcation at nasolabial fold are shown", "B, Retiform purpuric plaque is shown on the left leg with overlying desquamation", "C and D, Biopsy specimen findings (hematoxylin-eosin)"], "s1": [0, 8, 9, 10, 11], "s2": [1, 2, 3, 4, 5, 6, 7]} {"key": 619, "questions": "What Is Your Diagnosis?", "options": [{"label": "A", "disease": "Merkel cell carcinoma"}, {"label": "B", "disease": "Chloroma"}, {"label": "C", "disease": "Blastic plasmacytoid dendritic cell neoplasm"}, {"label": "D", "disease": "Histiocytoid Sweet syndrome"}], "answer_idx": "B", "symptoms": ["A man in his 60s presented for evaluation of asymptomatic skin lesions on the chest that had been stable since onset 4 months prior", "His medical history was significant for hyperlipidemia, familial polyposis, and acute myelogenous leukemia (AML) for which he had undergone an allogenic hematopoietic stem cell transplant 13 months earlier", "Cutaneous examination revealed green-gray nodules and plaques on the left and right upper chest (Figure 1)", "Bilateral cervical, axillary, and inguinal lymph node chains were negative for adenopathy", "The patient had not attempted measures to alleviate the nodules and denied experiencing exacerbating factors or pain", "Biopsies of the skin lesions were performed", "A, Representative dusky, green-gray nodule located on the upper chest", "B, A prominent green hue is present at the base of a lesion following shave biopsy"], "s1": [0, 1, 3, 4], "s2": [2, 5, 6, 7]} {"key": 620, "questions": "What Is Your Diagnosis?", "options": [{"label": "A", "disease": "Encephalomyelitis with hyperIgEaemia"}, {"label": "B", "disease": "Eosinophilic granulomatosis with polyangiitis (Churg-Strauss syndrome)"}, {"label": "C", "disease": "Lymphoma"}, {"label": "D", "disease": "Sarcoidosis"}], "answer_idx": "A", "symptoms": ["A white man in his early 40s presented with dizziness and fatigue followed by tingling of both lower extremities as well as gait disturbances", "The patient’s medical history was unremarkable besides mild intermittent whole-body itching starting 9 months prior", "Physical examination revealed mild proximal pyramidal weakness of the left leg (Medical Research Council grade 4/5), with brisk reflexes and extensor plantar response as well as hypoesthesia below the T2 level without sphincter disturbances", "In addition, mild decrement in position sense and marked decrement of vibration sense were observed in both lower extremities", "No skin lesions were detected, while cervical and supraclavicular lymph nodes were enlarged", "Multiple enhancing lesions were observed on brain and cervical spine magnetic resonance imaging (MRI) (Figure)", "Cerebrospinal fluid (CSF) was acellular, and eosinophils and oligoclonal bands were not detected, while proteins, glucose, and angiotensin-converting enzyme (ACE) levels were within normal range", "Test results for B-cell clones, viral, and fungal infections were all negative", "Routine blood test results were unremarkable besides an elevated eosinophilic count (2.76 K/μL", "17.6%)", "Antibodies for AQP4, MOG, and paraneoplastic syndromes were not detected", "Whole-body fluorodeoxyglucose positron emission tomography demonstrated generalized lymphadenopathy without any other pathological findings", "Whole-body gallium scan, chest high-resolution computed tomography, and bone marrow biopsy results were normal, while axillary and inguinal lymph node biopsies revealed nonspecific reactive lymphadenopathy", "Parasitic infections were excluded by plasma and CSF antibodies testing with enzyme-linked immunosorbent assay, stool, and lymph node microscopy", "Extensive diagnostic workup for malignancies and systemic autoimmune diseases was also inconclusive", "Fluid-attenuated inversion recovery (FLAIR) images presenting bilateral subcortical lesions in the brain (A) and T2-weighted images demonstrating multiple lesions and swelling of the cervical cord (C)", "Presence of gadolinium (GD) enhancement of the lesions in corresponding post-GD T1-weighted images of the brain (B) and predominantly in the posterior columns of the spinal cord (D)"], "s1": [0, 1, 3, 5, 8, 13, 15], "s2": [2, 4, 6, 7, 9, 10, 11, 12, 14, 16]} {"key": 621, "questions": "What Is Your Diagnosis?", "options": [{"label": "A", "disease": "Histiocytoma"}, {"label": "B", "disease": "Mastocytoma"}, {"label": "C", "disease": "Malignant pheochromocytoma"}, {"label": "D", "disease": "Myeloid sarcoma"}], "answer_idx": "B", "symptoms": ["A 41-year-old man presented with a 7-month history of episodes of back and abdominal pain associated with nausea, vomiting, and flushing", "Immediately following workup with noncontrast abdominal ultrasonography, he experienced intense flushing and vomiting", "Subsequent contrast-enhanced magnetic resonance imaging identified a 5-cm thoracic paraspinal mass (Figure, A)", "Computed tomography–guided biopsy of the paraspinal mass revealed a dense cellular infiltrate on histopathology composed of cells with round nuclei and abundant pale cytoplasm (Figure, B)", "A, Axial postcontrast T1-weighted magnetic resonance imaging of the spine identifying a paraspinal soft-tissue mass (bracketed by white arrowheads) encasing the descending aorta with neural foramen extension (pink arrowhead)", "B, Computed tomography–guided biopsy sample (original magnification ×400) showing a monomorphic infiltrate of small- to medium-sized round cells with uniform angulated nuclei and abundant amphophilic hypogranular cytoplasm"], "s1": [0, 1], "s2": [2, 3, 4, 5]} {"key": 622, "questions": "What Is Your Diagnosis?", "options": [{"label": "A", "disease": "Herpetic whitlow"}, {"label": "B", "disease": "Bacterial paronychia"}, {"label": "C", "disease": "Cutaneous leishmaniasis"}, {"label": "D", "disease": "Pyogenic granuloma"}], "answer_idx": "C", "symptoms": ["A girl younger than 2 years presented with a 7-month history of an inflammatory plaque on her left index finger (Figure, A)", "This lesion had started as a light erythema a few months before, initially oriented as a bacterial paronychia or herpetic whitlow and treated with an oral antibiotic (amoxicillin) and antiviral (acyclovir) without improvement", "There was no relevant medical history", "The family reported living in a rural area on the Mediterranean coast", "A, Erythematous nodule on the proximal nail fold", "B, Inflammatory erythematous plaque with a central shiny orange crust on the left index", "C, Diffuse dermatitis with epithelioid granulomas (hematoxylin-eosin stain)", "D, Intracellular amastigotes in the dermal macrophages (hematoxylin-eosin stain)", "Physical examination findings revealed an inflammatory erythematous plaque with a central shiny orange crust on the left index finger (Figure, B)", "No other fingers were affected", "There were no systemic symptoms or family history of similar lesions", "Given the chronic course, with no improvement with empiric therapy, a biopsy specimen was obtained for further evaluation (Figure, C and D)"], "s1": [0, 1, 4, 5, 8], "s2": [2, 3, 6, 7, 9, 10, 11]} {"key": 623, "questions": "What Is Your Diagnosis?", "options": [{"label": "A", "disease": "Esthesioneuroblastoma"}, {"label": "B", "disease": "Pott puffy tumor"}, {"label": "C", "disease": "Sinonasal meningioma"}, {"label": "D", "disease": "Sinonasal neuroendocrine carcinoma"}], "answer_idx": "C", "symptoms": ["An 85-year-old woman with a history of arthritis, hypercholesterolemia, hypertension, and glaucoma presented to the clinic for evaluation of chronic sinusitis", "Symptoms included a several-year history of frontal pain, pressure, clear nasal discharge, and progressive nontender facial swelling above her glabella", "She denied nasal disturbance, prior sinus surgery, asthma, or acetylsalicylic acid (aspirin) sensitivity", "Symptoms were refractory to treatment with several rounds of oral antibiotics and oral steroids", "Physical examination findings revealed symmetric facial movement with prominent glabellar swelling and soft tissue thickening", "Nasal endoscopy findings revealed a soft tissue mass anterior to the axilla of the middle turbinates bilaterally, with normal-appearing mucosa overlying", "Computed tomography and magnetic resonance imaging (MRI) scans were obtained for further evaluation (Figure, A)", "Recommendations were made to proceed to the operating room for biopsy of the nasal mass", "Pathology results revealed uniform, epithelioid cells arranged in a whorled, lobulated growth pattern without atypia, necrosis, increased cellularity, or diffuse growth pattern (Figure, B and C)", "A, T1-Weighted postcontrast magnetic resonance imaging scan, sagittal view, showed hyperintense soft tissue mass on anterior septum with extension into frontal sinus", "Adjacent soft tissue thickening noted on skin overlying frontal sinus and nasal bones", "No intracranial extension noted", "B, Hematoxylin-eosin stain (original magnification ×10) results showed lobules of neoplastic cells with whorled architecture (arrowhead)", "C, Hematoxylin-eosin stain (original magnification ×20) results showed intranuclear inclusions (arrowhead)"], "s1": [0, 1, 2, 3, 4, 5], "s2": [6, 7, 8, 9, 10, 11, 12, 13]} {"key": 624, "questions": "What Is Your Diagnosis?", "options": [{"label": "A", "disease": "Papillary thyroid carcinoma"}, {"label": "B", "disease": "Metastatic squamous cell carcinoma"}, {"label": "C", "disease": "Thyroid lymphoma"}, {"label": "D", "disease": "Anaplastic thyroid carcinoma"}], "answer_idx": "B", "symptoms": ["A 71-year-old man with no history of smoking was referred to our tertiary center in August 2018 for an asymptomatic left level 5 neck mass that had been present for 6 months", "Biopsy of the mass demonstrated metastatic p16-positive (human papillomavirus [HPV]-related) squamous cell carcinoma", "Examination revealed a large left base of tongue mass filling the vallecula", "Computed tomography (CT) demonstrated an enhancing mass within the left base of tongue and bilateral cervical adenopathy", "The patient was treated with concurrent radiation therapy and cetuximab, completing treatment without breaks in November 2018. His 3-month posttreatment fluorodeoxyglucose (FDG) positron-emission tomography/CT (FDG-PET/CT) imaging results demonstrated resolution of the activity at the base of tongue mass and the involved cervical lymph nodes, consistent with complete treatment response", "In July 2019, the patient presented with a palpable left supraclavicular lymph node", "Restaging FDG-PET/CT demonstrated avid supraclavicular and anterolateral neck adenopathy", "Additionally, there was moderately intense activity isolated to the left thyroid lobe", "Ultrasonography of the thyroid demonstrated a 4-cm mass within the gland, and subsequent biopsy was performed with immunohistochemistry staining positive for p16, CK5, and p40 and negative for thyroid transcription factor 1 (TTF-1) (Figure, A, B)", "After multidisciplinary review of the case, the patient was offered salvage left modified radical neck dissection and total thyroidectomy", "Surgical pathology confirmed the diagnosis (Figure, C)", "Fine-needle aspiration of the left thyroid lesion showing sheets and clusters of tumor cells with indistinct cell borders, with immunohistochemical staining positive for p16 (A) and negative for TTF-1 (B) (both panels, original magnification ×20)", "C, Lesional specimen showing tumor cells arranged in irregular nests and cords invading the thyroid parenchyma", "normal thyroid tissue shown by the arrowhead (original magnification ×4)"], "s1": [0, 1, 2, 3, 4, 5, 6], "s2": [7, 8, 9, 10, 11, 12, 13]} {"key": 625, "questions": "What Is Your Diagnosis?", "options": [{"label": "A", "disease": "Sporotrichosis"}, {"label": "B", "disease": "Koebnerized psoriasis vulgaris"}, {"label": "C", "disease": "Granulomatosis with polyangiitis"}, {"label": "D", "disease": "Sarcoidosis"}], "answer_idx": "A", "symptoms": ["An older adult presented with a history of a mildly pruritic plaque on the elbow at the site of a prior injury", "The medical history was unremarkable, and findings of a review of systems were negative", "The patient had been treated in the past with topical steroids without improvement, and the size of the lesion had continued to grow slowly", "Physical examination findings revealed a solitary, well-circumscribed pink plaque with overlying yellow-white micaceous scale with interspersed punctate hemorrhagic crusts (Figure, A)", "Lymphadenopathy was not identified", "Punch biopsy for histopathologic examination was performed, with representative sections shown (Figure, B and C)", "A, Well-circumscribed pink, scaly plaque on the right elbow", "B and C, Hematoxylin-eosin–stained sections reveal psoriasiform epidermal hyperplasia overlying suppurative granulomatous infiltrates with multinucleate giant cells and rare eosinophilic spheroid forms"], "s1": [0, 2, 3, 6], "s2": [1, 4, 5, 7]} {"key": 626, "questions": "What Is Your Diagnosis?", "options": [{"label": "A", "disease": "Saccular cyst"}, {"label": "B", "disease": "Chondroid metaplasia"}, {"label": "C", "disease": "Chondroma"}, {"label": "D", "disease": "Chondrosarcoma"}], "answer_idx": "B", "symptoms": ["A 45-year-old male professional voice user with a history of asthma being treated with inhaled steroids presented with a 1-year history of dysphonia", "He reported throat tightness after significant voice use and consistent voice roughness that worsened over several weeks", "On examination, he had moderate to severe dysphonia with predominant roughness and strain", "Videostroboscopy findings were consistent with fungal laryngitis", "a 2-week course of fluconazole was administered", "Follow-up videostroboscopy findings demonstrated resolution of the fungal laryngitis but revealed residual fullness of the left false vocal fold and appearance of prolapsed ventricular mucosa obstructing the lateral extent of the left true vocal fold", "There was a reduction of lateral excursion of the mucosal wave on the left", "The patient continued to report dysphonia and increased vocal effort", "A computed tomography scan with contrast showed no specific abnormalities", "A direct laryngoscopy for evaluation and treatment was planned", "Intraoperatively, there was a firm lesion in the left paraglottic space extending to the lateral aspect of the true vocal fold (Figure, A)", "A subtle but similar finding was seen on the right true vocal fold, not causing any mass effect (Figure, B)", "An incision was made in the mucosa of the ventricle overlying the lesion at its lateral extent, and a mucosal flap was elevated over the lesion", "This revealed a cartilaginous lesion, which was removed using an AcuBlade CO2 laser (Lumenis) and cup forceps until the cartilaginous edge was smooth within the paraglottic space (Figure, C and D)", "The histologic specimen showed fragments of cartilage with no nuclear atypia or mitosis (Figure, E and F)", "At 3-months’ follow-up, the patient reported improvement in his voice with decreased effort and strain, which correlated with improvement in the Voice Handicap Index score and objective assessment by the clinicians", "Direct laryngoscopy intraoperative images demonstrating left lesion (A), right lesion (B), submucosal firm, white mass excised with a CO2 laser (C), and postoperative view (D)", "hematoxylin-eosin stain of the left lateral vocal fold lesion showing fragments of cartilage (arrow indicates a peripheral thin fibroblastic rim with a transition to mature chondrocytes of the fibroelastic cartilage toward the center of the nodule) (E) and cartilage with mild nuclear atypia without mitosis (F)"], "s1": [0, 1, 2, 3, 5, 7, 8, 15], "s2": [4, 6, 9, 10, 11, 12, 13, 14, 16, 17]} {"key": 627, "questions": "What Is Your Diagnosis?", "options": [{"label": "A", "disease": "Focal melanotic macule (focal melanosis)"}, {"label": "B", "disease": "Oral melanoacanthosis (melanoacanthoma)"}, {"label": "C", "disease": "Blue nevus"}, {"label": "D", "disease": "Malignant melanoma"}], "answer_idx": "B", "symptoms": ["A 57-year-old African American man presented to an oral and maxillofacial surgeon for evaluation of a brown pigmented patch on the left side of the hard palate", "The lesion was of unknown duration and was reported by the patient to be asymptomatic, although he did endorse occasional pruritus", "An extraoral examination was performed without any abnormal findings", "On intraoral soft tissue examination, a brown pigmented patch located predominantly on the hard palate was observed", "The lesion had well-defined but irregular borders, measured 2 × 1.2 cm in size, and was nontender to palpation (Figure, A)", "Findings of the remainder of the intraoral soft tissue examination were within normal limits", "An incisional biopsy was performed, and the specimen was sent for microscopic diagnosis", "A, Intraoral photograph demonstrating the brown pigmented palatal patch with irregular, albeit sharply demarcated borders", "B and C, Medium-magnification and high-magnification hematoxylin-eosin–stained images demonstrate slender, elongated dendritic melanocytes (marked by arrows) throughout the epithelium", "Histologic examination findings revealed soft tissue covered by benign-appearing stratified squamous epithelium, focally spongiotic and acanthotic", "Within the epithelium, numerous benign dendritic melanocytes were identified (Figure, B and C)", "The underlying connective tissues were unremarkable"], "s1": [0, 1, 3, 4, 7], "s2": [2, 5, 6, 8, 9, 10, 11]} {"key": 628, "questions": "What Is Your Diagnosis?", "options": [{"label": "A", "disease": "Schwannoma"}, {"label": "B", "disease": "Juvenile angiofibroma"}, {"label": "C", "disease": "Odontogenic keratocyst"}, {"label": "D", "disease": "Chondrosarcoma"}], "answer_idx": "A", "symptoms": ["A 34-year-old man presented with a 3-month history of progressive, unilateral lower lip and chin numbness and ipsilateral facial fullness", "The hypoesthesia was limited to the distribution of the mandibular branch of the left trigeminal nerve (V3)", "On review of systems, the patient denied experiencing visual changes, dizziness, headaches, oral or neck pain, voice changes, weight or appetite changes, or swallowing dysfunction", "however, he did report biting his lip occasionally due to the numbness", "On examination, there were no palpable masses noted in the head, neck, or face", "Aside from the V3 sensory deficit, the patient had no cranial nerve deficits", "The patient had no significant medical history", "prior surgical history was notable for tonsillectomy", "Magnetic resonance imaging of the face revealed a T1 isointense, T2 hyperintense 5.9-cm lesion in the left masticator space, which appeared to be centered in the lateral pterygoid muscle with benign-appearing remodeling of the posterior maxillary wall (Figure)", "A, The lesion (indicated by the arrow) is isointense on T1-weighted magnetic resonance imaging (MRI) without contrast", "B, The lesion enhances with contrast on fat-suppressed T1-weighted imaging", "C, The mass is hyperintense on the T2-weighted sequence", "D, Computed tomography imaging reveals bony erosion and expansion of the lesion into the maxillary sinus"], "s1": [0, 1, 2, 3, 5, 6, 7], "s2": [4, 8, 9, 10, 11, 12]} {"key": 629, "questions": "What Is Your Diagnosis?", "options": [{"label": "A", "disease": "Metastatic glomus vagale tumors"}, {"label": "B", "disease": "Metastatic carotid body tumors"}, {"label": "C", "disease": "Metastatic sarcoma"}, {"label": "D", "disease": "Lymphoma"}], "answer_idx": "B", "symptoms": ["A young adult nonsmoker patient with a history of asthma presented with a palpable mass on the left side of the neck that had been present for several months, reportedly unchanged", "The patient had been treated presumptively for infection with antibiotics, with no improvement", "No fever, odynophagia, dysphagia, otalgia, or other constitutional symptoms were reported", "Physical examination findings demonstrated a palpable, soft, nontender left level II/III neck mass", "Clinical examination findings of the thyroid and upper aerodigestive tract were unremarkable", "Contrast-enhanced neck computed tomography (CT) showed a large avidly enhancing mass centered at the left carotid bifurcation splaying the internal and external carotid arteries and extending cranially along the cervical internal carotid artery", "The left internal jugular vein was displaced posteriorly", "There was also a smaller mass with similar appearance at the right carotid bifurcation (Figure 1A)", "Contrast-enhanced abdominal CT showed several enhancing masses in the liver (not shown)", "Based on CT findings, gallium Ga 68 (68Ga) dotatate positron emission tomography (PET)/magnetic resonance imaging (MRI) was performed", "results demonstrated intense radiotracer avidity in bilateral carotid space masses, hepatic masses, and numerous CT occult osseous lesions (Figure 1B)", "A, Axial contrast-enhanced neck computed tomography (CT) demonstrates large left (black asterisk) and small right (white arrowhead) avidly enhancing bilateral carotid space masses", "The left tumor circumferentially encases the left external carotid artery (black arrowhead) and partially encases the left internal carotid artery (yellow arrowhead)", "B, Coronal gallium 68 dotatate positron emission tomography/magnetic resonance imaging (PET/MRI) demonstrates intense radiotracer uptake of bilateral carotid space masses (left, black asterisk", "right, white arrowhead), hepatic masses (black arrowhead), and CT occult osseous lesions as seen in the right femoral neck (yellow arrowhead)"], "s1": [0, 1, 2, 3, 4], "s2": [5, 6, 7, 8, 9, 10, 11, 12, 13, 14]} {"key": 630, "questions": "What Is Your Diagnosis?", "options": [{"label": "A", "disease": "Plaque and ichthyosiform sarcoidosis"}, {"label": "B", "disease": "Lymphomatoid granulomatosis"}, {"label": "C", "disease": "Interstitial granulomatous dermatitis"}, {"label": "D", "disease": "Cutaneous tuberculosis"}], "answer_idx": "A", "symptoms": ["An African American woman in her 30s presented with a 6-month history of 1-cm to 2-cm erythematous and scaly plaques on her lower legs associated with 30-kg weight loss", "The plaques began around her ankles and spread proximally, reaching the upper thighs by the time of presentation", "She also noted new lesions on her arms and abdomen in the preceding weeks", "The eruption was asymptomatic, and the plaques were preceded by lower extremity edema and weakness, resulting in difficulty ambulating and frequent falls", "The patient also reported allodynia, tingling, and numbness in her lower legs", "She took no medications and her medical history was significant for a first-trimester miscarriage", "She was a schoolteacher and had not traveled outside of the country in the past 10 years", "Physical examination of her lower extremities revealed erythematous and indurated plaques, some with overlying coarse scale (Figure, A and B)", "Her neurological examination showed symmetric 4/5 strength and decreased reflexes in her lower extremities", "Sensation to light touch and proprioception were diminished below the knee", "Two punch biopsies were performed for histologic review (Figure, C and D)", "A, Dark brown, coarse, ichthyosiform scale on the bilateral lower legs", "B, Firm pink plaques with overlying coarse scale", "C, Granulomatous inflammation throughout the dermis with mild hyperparakeratosis around a hair follicle (hematoxylin-eosin)", "D, Well-formed, epithelioid granulomas (hematoxylin-eosin)"], "s1": [0, 1, 2, 3, 4, 8, 9], "s2": [5, 6, 7, 10, 11, 12, 13, 14]} {"key": 631, "questions": "What Is Your Diagnosis?", "options": [{"label": "A", "disease": "Metastatic breast cancer"}, {"label": "B", "disease": "Central nervous system lymphoma"}, {"label": "C", "disease": "Progressive multifocal leukoencephalopathy"}, {"label": "D", "disease": "Glioma"}], "answer_idx": "C", "symptoms": ["A 75-year old woman with a history of follicular lymphoma and breast cancer presented with a 3-month history of gradual vision loss and 1-month history of confusion and difficulty with word finding", "She had been diagnosed with hormone receptor–positive right breast cancer 8 years prior and underwent lumpectomy and treatment with an aromatase inhibitor, with remission since", "Five years later, she subsequently developed left inguinal adenopathy, which biopsy results showed was follicular lymphoma", "At that time, she was treated with a course of bendamustine and rituximab with subsequent maintenance treatment with rituximab for 2 years", "Magnetic resonance imaging (MRI) of the brain was performed, demonstrating T2 fluid-attenuated inversion recovery hyperintensity in the left occipital and temporal lobes, crossing the corpus callosum and extending into the right occipital lobe without enhancement (Figure)", "A, Axial T2 fluid-attenuated inversion recovery precontrast magnetic resonance imaging (MRI) of the brain", "B, Axial T1 magnetization-prepared rapid acquisition gradient-echo postcontrast MRI of the brain"], "s1": [0, 1, 2, 3], "s2": [4, 5, 6]} {"key": 632, "questions": "What Is Your Diagnosis?", "options": [{"label": "A", "disease": "Kaposi sarcoma"}, {"label": "B", "disease": "Syphilitic gumma of the larynx"}, {"label": "C", "disease": "Bacillary angiomatosis"}, {"label": "D", "disease": "Angiosarcoma"}], "answer_idx": "A", "symptoms": ["A man in his 30s with a history of treatment-naive HIV (CD4 count < 200 cells/mm3) and untreated syphilis presented to the emergency department with 3 weeks of progressively worsening dysphonia, dyspnea, and biphasic stridor", "On arrival to the hospital, the patient had a computed tomography scan performed", "results demonstrated an obstructive laryngeal mass but no signs of local invasion or lymphadenopathy", "The patient was taken urgently to the operating room where an awake tracheostomy was performed without complication", "Findings of a direct laryngoscopy demonstrated violaceous nodules of the base of the tongue as well as a mass of the infraglottis and subglottis, which was nearly completely obstructing the airway (Figure, A and B)", "Full physical examination findings also demonstrated violaceous nodules involving the patient’s lower extremities", "Results of an excisional biopsy demonstrated a subepithelial spindle cell lesion with extravasation of erythrocytes and stained strongly for human herpesvirus-8 (Figure, C)", "Left base of tongue mass with a violaceous hue (A), obstructive mass of the subglottic airway (B), and subepithelial spindle cell lesion with extravasation of erythrocytes and stained strongly for human herpesvirus-8 (hematoxylin-eosin", "original magnification ×10) (C)"], "s1": [0, 1, 2, 3], "s2": [4, 5, 6, 7, 8]} {"key": 633, "questions": "What Is Your Diagnosis?", "options": [{"label": "A", "disease": "Acinic cell carcinoma"}, {"label": "B", "disease": "Adenocarcinoma, not otherwise specified"}, {"label": "C", "disease": "Secretory carcinoma"}, {"label": "D", "disease": "Low-grade mucoepidermoid carcinoma"}], "answer_idx": "C", "symptoms": ["A 44-year-old woman presented with an asymptomatic enlarging right parotid mass of about 6 years’ duration", "She had no prior medical or surgical history and took no medications", "Ultrasonography demonstrated a solid and hypoechoic mass, 1.5 × 1.0 × 0.9 cm, in the inferior right parotid gland with minimal vascularity and without extraparotid extension", "No lymph node abnormality was observed", "Fine-needle aspiration was performed, and the smear was extremely cellular and showed sheets and clusters of bland polygonal epithelial cells admixed with bright pink filamentous matrix", "Because of the size and location, extracapsular dissection II was carried out.1 The final pathology of the lesion showed histologic findings compatible with salivary gland carcinoma, with microcystic and papillary architecture (Figure, A and B)", "Abundant bubbly secretion was present within these microcystic spaces, and this material was positive for periodic acid–Schiff (Figure, C)", "Immunohistochemically, there was positivity for epithelial membrane antigen, cytokeratin (CK) 7 and CK19, S100 protein, and gross cystic disease fluid protein 15 (GCDFP-15)", "Stains for gastrointestinal stromal tumors 1 (DOG1) and p63 were negative", "The tumor appeared to be contained within the surgical excision margins", "There was perineural invasion without evidence of neoplastic emboli in the peritumoral lymphovascular spaces", "The patient underwent a right parotidectomy I to IV1 and a level II superselective neck dissection", "A, Hematoxylin-eosin, original magnification ×40. Tumor cells with oval and rounded nuclei with fine chromatin and centrally localized nucleolus, with papillary architecture", "B, Microcystic growth pattern (hematoxylin-eosin, original magnification ×40)", "C, The secretory material stains positive for periodic acid–Schiff", "original magnification ×20."], "s1": [0, 1, 2, 3, 4, 5, 11], "s2": [6, 7, 8, 9, 10, 12, 13, 14, 15]} {"key": 634, "questions": "What Is Your Diagnosis?", "options": [{"label": "A", "disease": "Herpes simplex virus–associated pseudolymphoma"}, {"label": "B", "disease": "Actinic prurigo cheilitis"}, {"label": "C", "disease": "Cutaneous lymphoid hyperplasia"}, {"label": "D", "disease": "Marginal zone lymphoma"}], "answer_idx": "B", "symptoms": ["A white woman in her early 40s presented with generalized pruritus of 10-years’ duration", "She had no other medical history", "The patient provided the report of a skin biopsy performed 2 years before presentation", "It was described as indicating spongiotic dermatitis", "She had been treated with topical corticosteroids with no control of the symptoms", "Physical examination revealed erythematous papules, some of them crusted, and some excoriations located over the trunk and upper extremities (Figure 1A)", "In addition, some scales and fissures were seen on her lower lip (Figure 1B), sparing the surrounding skin, as well as inner mouth and upper lip", "A new biopsy sample was taken to rule out actinic cheilitis", "A, Clinical image of actinic prurigo in the skin of the patient showing unspecific erythematous papules on the arm", "B, Clinical image of actinic prurigo cheilitis in the patient showing scales, crusts, and fissures on the lower lip"], "s1": [0, 1, 2, 3, 4, 8], "s2": [5, 6, 7, 9]} {"key": 635, "questions": "What Is Your Diagnosis?", "options": [{"label": "A", "disease": "Epidermal nevus"}, {"label": "B", "disease": "Linear atrophoderma of Moulin"}, {"label": "C", "disease": "Lichen striatus"}, {"label": "D", "disease": "Segmental incontinentia pigmenti"}], "answer_idx": "B", "symptoms": ["A woman in her late 20s presented with an asymptomatic hyperpigmentation on the right side of the trunk, right upper arm, and right leg, strictly following the lines of Blaschko (Figure, A and B)", "A slight atrophy of the hyperpigmented skin was palpable without any signs of induration", "Clinical images show slightly atrophic hyperpigmented macules following the lines of Blaschko on the right side of the (A) trunk and (B) upper arm", "Histopathologic images of punch biopsy specimen show (C) normal epidermis and dermis with a slight hyperpigmentation of the basal epidermis layer (hematoxylin-eosin stain), and (D) preserved elastic fibers in the reticular dermis (Elastic van Gieson stain)", "The first onset was in her childhood, at the age of 6 years, with a small hyperpigmentation on the right side of her lower chest remaining stable for years", "In her early 20s the area of hyperpigmentation started to spread slowly over the above-mentioned areas and became more and more intense", "Intravenous penicillin G therapy with 10 million units 3 times daily over 1 week combined with a UV-A1 phototherapy in 18 sessions with a maximum dose of 40 J/cm2 and a cumulative overall dose of 670 J/cm2 showed no effects", "A punch biopsy was obtained (Figure, C and D)"], "s1": [0, 1, 2, 4, 5], "s2": [3, 6, 7]} {"key": 636, "questions": "What Is Your Diagnosis?", "options": [{"label": "A", "disease": "Kaposi sarcoma"}, {"label": "B", "disease": "Elder abuse"}, {"label": "C", "disease": "Rosacea-like angiosarcoma"}, {"label": "D", "disease": "Telangiectatic metastatic carcinoma"}], "answer_idx": "C", "symptoms": ["A man in his 80s with a pacemaker", "a history of congestive heart failure, coronary artery disease, atrial fibrillation, transient ischemic attack, and Parkinson disease", "and dependence in all activities of daily living presented to the dermatology department with a 4-month history of new-onset persistent facial eruption", "He denied a history of facial flushing", "The patient was initially treated for rosacea at an outside hospital with topical 1% metronidazole cream for 1 month without improvement and developed acute facial purpura after 1 day of treatment with oral doxycycline, which was discontinued", "Because of the eruption’s rapid onset and violaceous appearance, as well as empirical treatment failure, the patient was referred for further evaluation", "On examination, the patient had asymmetric, centrofacial, erythematous-violaceous indurated telangiectatic and ecchymotic plaques over a phymatous background (Figure, A)", "A series of punch biopsies were performed (Figure, B-D)", "A, Clinical image showing erythemato-violaceous plaques with ecchymotic areas", "B, Histopathologic image showing ectatic vessels with plump endothelia involving the deep dermis and hypodermis (hemotoxylin-eosin)", "C, On higher magnification, ectatic vascular channels were lined by atypical plump endothelial cells (hemotoxylin-eosin)", "D, Immunohistochemistry was negative for Human herpesvirus 8 (original magnification ×100)"], "s1": [0, 1, 2, 3, 4, 5], "s2": [6, 7, 8, 9, 10, 11]} {"key": 637, "questions": "What Is Your Diagnosis?", "options": [{"label": "A", "disease": "Progressive myoclonic epilepsies (PMEs)"}, {"label": "B", "disease": "Faciobrachial dystonic seizures"}, {"label": "C", "disease": "Creutzfeldt-Jakob disease (CJD)"}, {"label": "D", "disease": "Progressive encephalomyelitis with rigidity and myoclonus (PERM)"}], "answer_idx": "D", "symptoms": ["A 39-year-old woman presented to the emergency department with a 3-week history of worsening spasms in her axial and lower limb muscles and new-onset urinary retention", "Sudden episodes of painful involuntary spasms first appeared in her abdominal muscles 2 weeks after a cold", "Soon after, spasms spread to all of her axial muscles and both lower limbs", "These paroxysmal episodes lasted a few seconds, occurred up to 20 times a day, and were exacerbated by external stimuli", "There was no impairment of consciousness during the attacks", "A review of her previous records revealed a diagnosis of Hashimoto thyroiditis 1 month earlier", "She had no history of tobacco, alcohol, or drug use", "She had hyperhidrosis and tachypnea on admission", "Neurological examination results disclosed an abnormal mental status", "Cranial nerve examination results demonstrated slow saccades", "Motor system examination results revealed a symmetric and severe hypertonia and rigidity throughout her trunk and all 4 extremities", "Further notable occurrences were severe spasms (Figure, A", "Video 1) and patella clonus associated with brisk deep tendon reflexes (Video 2)", "She demonstrated marked myoclonus on tactile and acoustic stimulation", "Extensor plantar responses were present bilaterally", "A, Patient experiencing spasms", "B, Electromyography examination of the patient", "Laboratory results were significant for an elevated level of thyroperoxidase antibodies (50.85 IU/mL", "normal, <5.16 IU/mL) and thyroglobulin antibodies (88.35 ng/mL", "normal: <4.11 ng/mL", "to convert to micrograms per liter, multiply by 1)", "Cerebrospinal fluid testing results revealed normal protein level and cell counts, as well as absent oligoclonal bands or evidence of infection", "Her paraneoplastic panel results were negative", "Cranial and spinal magnetic resonance imaging yielded no abnormalities", "Needle electromyography (EMG) results showed a continuous firing of normal motor unit potentials at rest within the axial and lower limb muscles (Figure, B", "Video 3)", "A routine electroencephalogram (EEG) showed mild background slowing without periodic discharges", "Computed tomography scan results of the thorax, abdomen, and pelvis were negative", "An electrocardiogram showed sinus tachycardia"], "s1": [0, 1, 2, 3, 6, 7, 10, 11, 13, 15, 21, 25], "s2": [4, 5, 8, 9, 12, 14, 16, 17, 18, 19, 20, 22, 23, 24, 26, 27, 28]} {"key": 638, "questions": "What Is Your Diagnosis?", "options": [{"label": "A", "disease": "Erysipelas"}, {"label": "B", "disease": "Intravascular large B-cell lymphoma"}, {"label": "C", "disease": "Intravascular natural killer/T-cell lymphoma"}, {"label": "D", "disease": "Cutaneous leukocytoclastic vasculitis"}], "answer_idx": "C", "symptoms": ["A 47-year-old woman presented with a 7-month history of multiple painful erythematous plaques on her lower legs", "The lesions appeared as erythematous nodules on the left leg with tenderness and pruritus and rapidly progressed to both legs", "Marked lower-leg edema was noted 20 days prior to the visit", "She also had an intermittent fever during the past 3 months that did not respond to antibiotic therapy", "The patient’s medical history was unremarkable", "Physical examination revealed ill-defined, indurated erythematous-violaceous plaques on the lower extremities with substantial edema (Figure, A and B)", "Complete blood cell count revealed a normal white blood cell count with moderate anemia (hemoglobin level, 87 g/L", "normal range 115-150 g/L)", "Additional laboratory tests demonstrated an elevated level of serum lactate dehydrogenase (749 U/L", "normal range 100-240 U/L) (to convert to μkat/L, multiply by 0.0167)", "Quantification of Epstein-Barr virus (EBV) DNA in sera showed a high viral load of EBV in peripheral blood (1 800 000 copies/mL, normal range <500 copies/mL)", "A skin biopsy specimen was obtained from a violaceous plaque (Figure, C)", "A and B, Ill-defined, irregular erythematous plaques with substantial edema of lower extremities", "C, Lesional skin biopsy specimen revealed dilated vessels filled with atypical large lymphoid cells throughout the dermis (hematoxylin-eosin)"], "s1": [0, 1, 2, 3, 5, 11, 12, 13], "s2": [4, 6, 7, 8, 9, 10]} {"key": 639, "questions": "What Is Your Diagnosis?", "options": [{"label": "A", "disease": "Squamous cell carcinoma"}, {"label": "B", "disease": "CD30-positive primary cutaneous anaplastic large cell lymphoma"}, {"label": "C", "disease": "Herpes vegetans"}, {"label": "D", "disease": "Giant condyloma acuminatum (Buschke-Löwenstein tumor)"}], "answer_idx": "C", "symptoms": ["A woman in her 80s presented with an 8-month history of a growing, painful tumor on her right buttock", "The patient felt well otherwise and denied experiencing fevers, chills, sweats, weight loss, or malaise", "Her medical history was notable for chronic kidney disease and genital herpes simplex virus (HSV)", "She reported no history of genital warts, HIV, or inflammatory bowel disease", "A biopsy was performed, and results were consistent with a noninfectious, granulomatous ulcer", "Based on the clinical appearance and prior pathologic findings, the patient received treatment of oral antibiotics, intralesional steroids, topical steroids, and immunomodulators, without improvement", "Findings from colorectal surgical evaluation of the internal anal mucosa were negative for malignant neoplasm", "At referral, physical examination findings revealed a 5 × 7-cm verrucous, exophytic tumor with scant yellow drainage on the right medial buttock extending to the perianal skin (Figure 1A)", "Findings from the remaining skin examination were normal", "Superficial bacterial cultures were obtained from the lesion and grew methicillin-resistant Staphylococcus aureus", "An incisional biopsy was performed for diagnostic clarification (Figure 1B and C)", "A, Clinical image of 5 × 7-cm verrucous, exophytic tumor with scant yellow drainage on the right medial buttock extending to the perianal skin", "B, Hematoxylin-eosin stain image from the incisional biopsy featuring a hyperplastic epidermis with ulceration, necrosis, and a very dense inflammatory infiltrate", "C, Hematoxylin-eosin stain demonstrating plasma cells composing the infiltrate while the epithelium shows focal multinucleated keratinocytes, some with gray-blue cytoplasm"], "s1": [0, 1, 2, 3, 5, 6, 8], "s2": [4, 7, 9, 10, 11, 12, 13]} {"key": 640, "questions": "What Is Your Diagnosis?", "options": [{"label": "A", "disease": "CD8+ mycosis fungoides"}, {"label": "B", "disease": "Lymphomatoid papulosis, type D"}, {"label": "C", "disease": "Primary cutaneous aggressive epidermotropic cytotoxic CD8+ T-cell lymphoma"}, {"label": "D", "disease": "Natural killer/T-cell lymphoma"}], "answer_idx": "C", "symptoms": ["A woman in her 50s presented to a tertiary dermatology referral center with scaly erythematous patches and plaques covering 10% of her total body surface area without clinically appreciable lymphadenopathy", "Histopathological testing demonstrated an epidermotropic infiltrate of hyperchromatic, convoluted, atypical CD3+, CD4−, and CD8+ lymphocytes with haloed nuclei with basilar tagging and papillary dermal fibrosis", "Studies of T-cell gene rearrangements identified a clonal population", "Blood-flow cytometry did not reveal aberrant T-cell immunophenotypes", "She was treated initially with topical and intralesional corticosteroids and narrow band UV B phototherapy", "Oral bexarotene was subsequently added for recalcitrant disease", "Within 2 months, she developed isolated, painful tumors", "A second biopsy was performed, which was notable for an epidermotropic infiltrate of CD3+, CD4−, and CD8− weak lymphocytes, which were positive for TIA-1, granzymes, and B factor (Bf) 1 and negative for CD30 and CD56. The results of flow cytometry and imaging were unremarkable", "After an initial favorable response with chlorambucil, she developed expanding ulcerated tumors complicated by a polymicrobial superinfection", "Chlorambucil was discontinued in favor of antimicrobial treatment", "She then received liposomal doxorubicin, although skin involvement continued to progress rapidly, with ulcerating tumors making up as much as 80% of her total body surface area", "She was hospitalized for worsening disease and pain (Figure, A)", "Repeated histopathologic analysis was done (Figure, B, C, and D)", "A, Clinical image", "B, Original magnification ×50. C, Considerable lymphocytic epidermotropism in a pagetoid reticulosis–like pattern without striking Pautrier microabscesses (original magnification ×230)", "D, Original magnification ×50.Primary cutaneous aggressive epidermotropic cytotoxic CD8+ T-cell lymphoma"], "s1": [0, 4, 5, 10, 11], "s2": [1, 2, 3, 6, 7, 8, 9, 12, 13, 14, 15]} {"key": 641, "questions": "What Is Your Diagnosis?", "options": [{"label": "A", "disease": "Cutaneous B-cell lymphoma"}, {"label": "B", "disease": "Angiosarcoma"}, {"label": "C", "disease": "Metastatic carcinoma"}, {"label": "D", "disease": "Merkel cell carcinoma"}], "answer_idx": "B", "symptoms": ["A 55-year old woman presented with a 6-week history of an enlarging, tender, pruritic, 4-cm erythematous patch with central inflammatory papules distributed on the medial forehead (Figure, A)", "No inciting events, such as arthropod bite or trauma, were noted", "She was initially treated for presumed shingles with valacyclovir, cephalexin, gabapentin, and hydrocortisone 2.5% topical ointment with no improvement in appearance or symptoms", "No other cutaneous abnormalities were noted", "A punch biopsy specimen of the left medial forehead was obtained, and histological and immunohistochemical analyses were performed (Figure, B and C)", "A, Erythematous patch with central inflammatory papules on the forehead", "B, Histopathological results revealed dissecting vascular channels (black arrowheads) and spindle cells (white arrowheads) (hematoxylin-eosin)", "C, Immunohistochemical staining revealed CD31 and diffuse expression of FLI-1 and ERG in the vascular and spindle cell components"], "s1": [0, 1, 2, 3, 5], "s2": [4, 6, 7]} {"key": 642, "questions": "What Is Your Diagnosis?", "options": [{"label": "A", "disease": "Rosai-Dorfman disease"}, {"label": "B", "disease": "Kikuchi-Fujimoto disease"}, {"label": "C", "disease": "Dianzani autoimmune lymphoproliferative disease"}, {"label": "D", "disease": "Castleman disease"}], "answer_idx": "D", "symptoms": ["A 17-year-old white male patient was referred to the otolaryngology clinic with a 6-month history of a left-sided neck mass", "His primary care physician initially obtained an ultrasonographic image, which demonstrated a left, level 2 lymph node measuring 4.0 × 1.5 × 2.8 cm", "The patient then underwent several courses of antibiotic treatment, but the mass persisted", "After a period of conservative management, he developed a sore throat and fever", "He subsequently underwent a computed tomographic scan of the neck and chest with contrast, which demonstrated an isolated, enlarged, homogenously enhancing left-neck level 2B lymph node measuring 3.4 × 2.8 × 2.1 cm (Figure 1A)", "When he presented to our facility, he was found to have an asymptomatically enlarged left level 2/5A neck mass that was nontender and fixed", "The patient had an unremarkable medical, surgical, and social history and no known infectious exposures", "A white blood cell count and C-reactive protein level were both elevated, at 11 310 cells/μL (to convert to cells × 109 per liter, multiply by 0.001) and 56.2 mg/L (to convert to nanomoles per liter, multiply by 9.524), respectively", "Studies for influenza A and B, HIV, cytomegalovirus, American foulbrood, Bartonella henselae, infectious mononucleosis, and Epstein-Barr virus had negative results", "An ultrasonography-guided needle biopsy was performed", "Fine-needle aspiration and flow cytometry had negative results for lymphoproliferative disorders or a malignant condition", "A tissue culture had negative results", "Given the inconclusive diagnosis, the patient underwent a left neck dissection of levels 2A, 2B, and 5A", "Intraoperative frozen sections did not show a malignant process", "On final analysis, a pathologic examination of the largest lymph node was performed (Figure 1B)", "A, Axial computed tomographic (CT) scan of neck with contrast, showing homogenously enhancing 3.4-cm left level 2B neck mass", "B, Low-power histology of resected lymph node, demonstrating a regressed germinal center penetrated by a hyalinized vessel and small lymphocytes in mantle zone palisading around the residual germinal centers"], "s1": [0, 3, 5, 7, 8, 9, 10, 11], "s2": [1, 2, 4, 6, 12, 13, 14, 15, 16]} {"key": 643, "questions": "What Is Your Diagnosis?", "options": [{"label": "A", "disease": "Left ventricular systolic dysfunction"}, {"label": "B", "disease": "Sarcoidosis"}, {"label": "C", "disease": "Pulmonary tumor thrombotic microangiopathy"}, {"label": "D", "disease": "Chronic thromboembolic pulmonary hypertension"}], "answer_idx": "C", "symptoms": ["An 82-year-old woman presented with dyspnea on exertion and weight gain of 5 kg over the past month", "Her medical history included atrial fibrillation and lung adenocarcinoma (a lung lobectomy was performed 8 years ago)", "Her blood pressure was 112/60 mm Hg, her heart rate was 100 beats per minute, her respiratory rate was 37 breaths per minute, her body temperature was 36.6°C, and her oxygen saturation was 100% while breathing 10 L of oxygen", "A cardiac examination was clinically significant for an irregular rhythm, an increased pulmonic closure sound, and jugular venous distension", "Bibasilar coarse crackles and pretibial edema were also noted", "A complete blood count had results within normal limits", "The serum lactate dehydrogenase level was 278 U/L (to convert to microkatal per liter, multiply by 0.0167)", "A chest radiographic image showed bibasilar infiltrates", "An electrocardiogram was notable for atrial fibrillation, and echocardiography revealed normal left ventricular function, mild right atrial and ventricular dilatation, and severe tricuspid regurgitation (transtricuspid pressure gradient, 40 mm Hg)", "A contrast-enhanced computed tomography image revealed bilateral ground-glass opacities and right basilar consolidations, with no evidence of thromboembolisms (Figure, A)", "Para-aortic and inguinal lymphadenopathies were also noted", "Presumptive diagnoses of pulmonary hypertension (PH) and right-sided heart failure were made", "The patient began receiving dobutamine and furosemide, but she continuously required 2 to 10 L of oxygen, and further invasive evaluations could not be performed", "An echocardiogram identified aggravating PH (transtricuspid pressure gradient, 57 mm Hg)", "A thorough physical examination identified scaly, crusty genital skin lesions pathologically compatible with extramammary Paget disease", "The patient deteriorated and died despite treatment", "An autopsy was performed (Figure, B)", "A, The chest computed tomography showed ground-glass opacities (white arrowheads) and consolidative lesions (a yellow arrowhead), but no evidence of thromboembolism was obtained", "B, Hematoxylin-eosin stain of the lung showed proliferation of intimal fibromuscular cells with (yellow arrowheads) or without (black arrowheads) the presence of Paget cells", "Yellow arrowheads show embolized Paget cells", "The encircled area (black arrowheads) indicates the vascular lumen, the inside of which is filled with proliferated fibromuscular cells but not Paget cells (original magnification ×400)"], "s1": [0, 1, 2, 3, 4, 5, 6, 7, 8, 9, 11, 12, 13, 15], "s2": [10, 14, 16, 17, 18, 19, 20]} {"key": 644, "questions": "What Is Your Diagnosis?", "options": [{"label": "A", "disease": "Laryngeal tuberculosis"}, {"label": "B", "disease": "Systemic lupus erythematosus"}, {"label": "C", "disease": "IgG4-related related disease"}, {"label": "D", "disease": "Churg-Strauss syndrome"}], "answer_idx": "C", "symptoms": ["A 29-year-old man with a history of reactive airway disease and eczema presented to the clinic with progressive odynophagia, hoarseness, and dysphagia of solids, liquids, and secretions for the past 6 months", "There were no respiratory symptoms, and results of a complete head and neck physical examination were unremarkable", "Flexible nasolaryngoscopy was performed and revealed erythema and swelling to the right arytenoid", "A 2-week course of doxycycline was completed for presumed infectious supraglottitis without a change in symptoms", "A biopsy and culture of the lesion was then collected in the office and revealed inflammatory disease but ultimately was nondiagnostic", "The patient chose observation vs further workup at that time", "The patient returned 6 months later with worsening symptoms", "Repeat flexible nasolaryngoscopy was performed and revealed significant edema of the right arytenoid extending into the aryepiglottic fold, limiting the mobility of the right vocal fold (Figure 1)", "Microdirect laryngoscopy and biopsy under general anesthesia was performed, and a specimen was sent to the pathology department for permanent section analysis and flow cytometry", "Permanent sections showed unremarkable squamous mucosa", "Flow cytometry revealed mixed inflammatory infiltrate and an increased plasma cell component", "Results of stains for infectious organisms were negative", "Further immunohistochemical analysis confirmed the diagnosis", "A full-body nuclear magnetic resonance positron emission tomography/computed tomography scan was performed to rule out systemic disease"], "s1": [0, 1, 3, 6, 7], "s2": [2, 4, 5, 8, 9, 10, 11, 12, 13]} {"key": 645, "questions": "What Is Your Diagnosis?", "options": [{"label": "A", "disease": "Glomus tumor"}, {"label": "B", "disease": "Inverted papilloma"}, {"label": "C", "disease": "Hemangiopericytoma"}, {"label": "D", "disease": "Paraganglioma"}], "answer_idx": "A", "symptoms": ["A 31-year-old female patient presented with right nasal pain and radiating facial pain over the previous 3 years", "She reported feeling a mass in her right nostril that had not changed in size, as well as associated nasal tenderness to the external nose, worsening pain with exposure to cold temperatures, and frequent sneezing", "Prior to presentation, she had been prescribed fluticasone and loratadine for allergic rhinitis without resolution of symptoms", "She denied difficulty breathing, epistaxis, rhinorrhea, fever, chills, weight loss, or night sweats", "She was a nonsmoker and nondrinker", "On physical examination, her external nose had no visible lesions", "A rhinoscopy revealed normal mucosa with a submucosal, round, firm, tender lesion, measuring approximately 1 cm and located on the right anterior nasal septum", "No drainage was visualized", "Her turbinates appeared normal", "The patient underwent surgical excision of the septal lesion under general anesthesia for definitive diagnosis", "The specimen measured 1.1 × 0.7 × 0.2 cm and was erythematous, soft, vascular, and adherent, with a broad base to the anterior superior nasal septal mucosa just inferior to the upper lateral cartilage (Figure 1A and B)", "The lesion was excised down through the perichondrium", "Endoscopic photographs", "A, Intraoperative view and intraoperative photograph of a mucosal lesion on the anterior septum after the initial incision", "B, Elevation with a suction shows attachments to the underlying perichondrium"], "s1": [0, 1, 2, 3, 4], "s2": [5, 6, 7, 8, 9, 10, 11, 12, 13, 14]} {"key": 646, "questions": "What Is Your Diagnosis?", "options": [{"label": "A", "disease": "Annular pustular psoriasis"}, {"label": "B", "disease": "Bullous tinea"}, {"label": "C", "disease": "Linear IgA dermatosis"}, {"label": "D", "disease": "Acute localized exanthematous pustulosis"}], "answer_idx": "C", "symptoms": ["A man in his 60s was referred to our clinic because of a well-demarcated, circular erythematous plaque that had been present at the area of median sternotomy for 3 months", "Physical examination findings revealed a 20-cm–diameter plaque that consisted of multiple concentric annular formations with raised erythematous margins and studded with numerous blisters and pustules, some coalescing into lakes of pus (Figure 1A)", "Nikolsky sign was not elicited, and oral, ocular, and genital mucosa involvement was absent", "The patient did not report other symptoms, and findings of the overall physical examination were normal", "A, A crusted, focally vesiculating, well-demarcated erythematous plaque at the area of median sternotomy", "B, Histopathologic examination of lesional skin (hematoxylin-eosin)", "C, Histopathologic examination of lesional skin (hematoxylin-eosin)", "Over the past 5 years, the patient’s medications consisted of metoprolol and acetylsalicylic acid for treatment of established ischemic heart disease", "Nine months prior to being referred to our department, he was given oral furosemide and amiloride and then underwent median sternotomy and coronary artery bypass surgery 2 months later", "Four days after the operation, he developed a red patch at the area of the incision that was accompanied by mild pruritus", "Over the following months, the lesion showed a marked tendency for centrifugal extension over the chest area", "Results of routine laboratory studies, Mantoux test, chest radiography, and chest computed tomography of the area were within the normal range", "Gram stain, periodic acid–Schiff stain, and tissue culture results were negative (Figure 1B and C)"], "s1": [0, 1, 2, 3, 4, 9, 10], "s2": [5, 6, 7, 8, 11, 12]} {"key": 647, "questions": "What Is Your Diagnosis?", "options": [{"label": "A", "disease": "Lupus panniculitis (subcutaneous lupus erythematosus)"}, {"label": "B", "disease": "Spindle cell lipoma"}, {"label": "C", "disease": "Dermatofibrosarcoma protuberans"}, {"label": "D", "disease": "Diffuse pattern neurofibroma"}], "answer_idx": "B", "symptoms": ["A woman in her 40s presented to the dermatology clinic with a 10-year history of progressive skin thickening and subcutaneous enlargement of the right side of her nose and right cheek", "She had been seen by several dermatologists and otolaryngologists and had undergone nondiagnostic skin biopsies", "She underwent a septorhinoplasty 6 months prior to presentation because of collapse of her right nasal ala", "During the surgical procedure, the surgeon noted a significant amount of submucosal tissue in the right nasal cavity, which was removed", "histopathological analysis was performed, and the results were reported as fibroadipose tissue and vascular proliferation", "A skin biopsy was performed at the time of the surgical procedure and was interpreted as sebaceous hyperplasia", "The patient initially had improvement in her skin, but she again noticed enlargement of the soft tissue with thickening of the overlying skin on her right cheek and right side of her nose several months later", "Clinical examination revealed slightly hyperplastic soft tissue with dilated patulous follicles involving the right medial malar cheek and right nasal ala in addition to deviation of the nasal tip (Figure, A)", "The surgical pathology specimens from her septorhinoplasty were obtained and reviewed", "A, Clinical image shows facial asymmetry with deviation of the nasal tip", "B, Hematoxylin-eosin stain of an interstitial proliferation of spindle cells within mature adipocytes that extends between bundles of skeletal muscle", "C, Hematoxylin-eosin stain of mature adipocytes that are separated by a proliferation of spindle cells associated with fragments of thickened collagen"], "s1": [0, 6, 7, 9], "s2": [1, 2, 3, 4, 5, 8, 10, 11]} {"key": 648, "questions": "What Is Your Diagnosis?", "options": [{"label": "A", "disease": "Infectious bone-marrow disease"}, {"label": "B", "disease": "Bone marrow–liver–spleen type of large B-cell lymphoma"}, {"label": "C", "disease": "Primary bone lymphoma"}, {"label": "D", "disease": "Hemophagocytic lymphohistiocytosis"}], "answer_idx": "B", "symptoms": ["A 71-year-old man had a 3-year history of a recurrent seronegative autoimmune constrictive pericarditis and pleural effusions, which were treated with pericardiectomy and prednisone and azathioprine", "The pleural fluid was transudative and sterile, and the pericardium contained focal lymphoplasmacytic aggregates", "Attempts to taper immunosuppressive drugs failed because of a recrudescence of the inflammatory syndrome and pericardial effusion", "The patient was admitted with fever of unknown origin with a surge of the inflammatory syndrome and pancytopenia", "He reported night sweats beginning 3 weeks prior associated with fever and unintentional weight loss", "No lymphadenopathy was noted", "Laboratory evaluations demonstrated a C-reactive protein level of 133 mg/L (reference range, <10 mg/L)", "ferritin level of 2607 ng/mL (reference, 30-300 ng/mL)", "lactate dehydrogenase (LDH) level of 289 U/L (reference, 135-214 U/L)", "pancytopenia with a hemoglobin level of 9.3 g/dL (reference, 13.3-17.7 g/dL), white blood cell count of 2700/μL (reference, 4000-10 000/μL), and platelet count of 99 ×103/μL (reference, 150-350 ×103/μL)", "and prerenal acute kidney injury (Acute Kidney Injury Network stage 1) with a serum creatinine level of 1.4 mg/dL (reference, 0.7-1.2 mg/dL)", "Results of a comprehensive serologic workup were negative", "Gastroscopy results showed mycotic distal esophagitis", "The patient was readmitted 3 weeks later with fever and hypotension without improvement despite broad-spectrum antibiotics and antifungals", "Considering the increase in ferritin and LDH levels and pancytopenia, a hematologic disease was suspected", "The patient did not report any bone pain", "Computed tomography (CT) results showed a splenomegaly without evidence of metastatic disease or lymph node enlargement", "Positron emission tomography (PET)–CT imaging (Figure, A and B) and a bone-marrow biopsy (Figure, C and D) were performed", "A rapidly evolving sepsislike syndrome led to the patient’s death", "Fluorodeoxyglucose/positron emission tomography–computed tomography imaging results show diffuse hypercaptation in the bone marrow (A) and spleen (B)", "The entire skeleton is highly metabolic, which suggests advanced bone-marrow involvement", "C, Bone-marrow biopsy results show a diffuse infiltrate of large lymphoid cells with scattered residual hematopoietic elements (immunoperoxidase, original magnification ×200)", "D, Immunostaining shows lymphoid infiltrate strongly positive for paired box protein Pax-5 (immunoperoxidase, original magnification ×200)"], "s1": [0, 1, 2, 3, 4, 5, 6, 7, 10, 11, 12, 13, 16, 18], "s2": [8, 9, 14, 15, 17, 19, 20, 21, 22]} {"key": 649, "questions": "What Is Your Diagnosis?", "options": [{"label": "A", "disease": "Malignant fibrous histiocytoma"}, {"label": "B", "disease": "Paraganglioma"}, {"label": "C", "disease": "Squamous cell carcinoma"}, {"label": "D", "disease": "Spindle cell rhabdomyosarcoma"}], "answer_idx": "D", "symptoms": ["A 69-year-old man with a history of metastatic squamous cell carcinoma in the left parotid gland had undergone left parotidectomy and modified neck dissection followed by adjuvant radiation therapy 12 years previously", "Ten years later, he developed a second primary tumor in the hypopharynx requiring total laryngectomy and bilateral neck dissection", "He presented with new-onset headaches, left ear pain, and persistent unsteadiness during the previous month", "There was no palpable lymphadenopathy, neurological deficits, or mastoid tenderness, and otoscopic examination results of the tympanic membrane appeared normal", "Magnetic resonance imaging of the head showed an enhancing lesion in the left infratemporal fossa (3.4 × 3.4 × 2.8 cm) abutting the cerebellum, and subsequent computed tomographic imaging of the head demonstrated osseous destruction of the left temporal bone extending along the left internal jugular vein (Figure, A)", "Fine-needle aspiration of the left mastoid showed atypical spindle cells", "The patient underwent tumor excision with a left infratemporal approach", "A large mass with rubbery texture was noted, extending into the jugular foramen and adherent to the dura mater without evidence of intradural invasion", "The lower cranial nerves were identified, and the mass was resected to the external wall of the jugular bulb", "The patient tolerated the procedure without complications or neurological deficits", "Histopathologic assessment demonstrated myogenin-positive spindle cells with no tumor necrosis (Figure, B and C)", "In addition, the tumor expressed desmin-positive and CD34-positive cells", "The tumor focally infiltrated into the fibrous tissue, skeletal muscle, and bone", "A, Axial computed tomography of the head and neck demonstrating osseous destruction of the left temporal bone", "B and C, Histopathologic test results", "B, High-powered view of hematoxylin-eosin stain (original magnification ×40) demonstrating spindle cell morphologic features with admixed fragments of bone (pink-red areas)", "C, High-powered view of myogenin stain (original magnification ×40) depicting small-cell or spindle cell morphologic features with rare positive results for myogenin (brown cells)"], "s1": [0, 1, 2, 3, 6, 9], "s2": [4, 5, 7, 8, 10, 11, 12, 13, 14, 15, 16]} {"key": 650, "questions": "What Is Your Diagnosis?", "options": [{"label": "A", "disease": "Spindle cell carcinoma"}, {"label": "B", "disease": "Inflammatory myofibroblastic tumor"}, {"label": "C", "disease": "Laryngeal granuloma"}, {"label": "D", "disease": "Plasmacytoma of the larynx"}], "answer_idx": "B", "symptoms": ["A 45-year-old woman was referred to a tertiary care center for hoarseness", "She had a left vocal fold polyp that was removed 2 months earlier at an outside institution, with benign pathology reported", "Subsequent stroboscopy revealed a hemorrhagic-appearing polyp (2-3 mm)", "She was sent for preoperative voice therapy, with plans for excision", "However, her voice worsened with therapy, and she returned to the otolaryngology clinic after a few weeks", "A larger, 6- to 7-mm, irregular, left vocal fold mass was noted", "Preoperative imaging was nonspecific, revealing a small soft-tissue density along the true vocal fold", "The patient was taken to the operating room for excision (Figure, A)", "After the final pathology results were obtained, the patient was taken back to the operating room for cordectomy with negative margins", "A, The left vocal fold lesion has a hemorrhagic exophytic mass that has prolapsed into the subglottis", "B, Lesional spindled, fibroblastic-myofibroblastic, and inflammatory cells (both plasma cells and lymphocytes) appear in a myxoid background with abundant blood vessels", "Even within this field, the denser collagenized stroma interrupts looser edematous areas", "The overlying squamous epithelium demonstrates only minimal reactivity with some acanthosis and parakeratosis", "C, Neoplastic, spindled, fibroblastic-myofibroblastic cells have diffuse, strong, granular cytoplasmic staining with the ALK1 (D5F3 clone)", "The intralesional blood vessels and inflammatory cells and overlying epithelium do not stain", "Pathologic results revealed spindle-shaped cells with variable atypia noted and inflammatory cells, including lymphocytes and plasma cells (Figure, B)", "Cellular proliferation was present in a myxomatous and focally fibrous-appearing stroma", "Immunohistochemical analysis revealed that the lesional cells were negative for desmin, S100, P63, and cytokeratin (Figure, C)", "They were reactive for vimentin and anaplastic lymphoma kinase 1 (ALK1)", "These findings were confirmed at the time of the initial excision", "She has had no recurrent disease for the past 3 years"], "s1": [0, 1, 2, 3, 4, 5, 6, 7, 8, 9], "s2": [10, 11, 12, 13, 14, 15, 16, 17, 18, 19, 20]} {"key": 651, "questions": "What Is Your Diagnosis?", "options": [{"label": "A", "disease": "Lipoma"}, {"label": "B", "disease": "Rhabdomyosarcoma"}, {"label": "C", "disease": "Infantile hemangioma"}, {"label": "D", "disease": "Glial heterotopia"}], "answer_idx": "D", "symptoms": ["A full-term, 7-month-old male infant presented to an outpatient pediatric otolaryngology clinic with a firm, irregularly shaped posterior neck mass", "He had been diagnosed prenatally with cerebellar hypoplasia, so a magnetic resonance imaging study of the brain was performed at 1 month of age", "This study demonstrated an incidental solid, homogenous mass in the subcutaneous fat of the right occiput with irregular borders and infiltration into the paraspinal musculature (Figure, A)", "On examination, the upper posterior neck mass was deep to palpation without overlying skin changes", "The patient had no discomfort, and neck movement was not restricted", "A, Axial T2-weighted magnetic resonance imaging of the neck without contrast", "The arrowhead points to a 1.1-cm × 0.8-cm × 1.7-cm right occipital soft tissue mass with irregular borders and infiltration into the deep paraspinal musculature", "B and C, Hematoxylin-eosin–stained images demonstrating cells with oval nuclei and multiple small nucleoli surrounded by a pale eosinophilic and slightly fibrillary stroma", "At 7 months of age, the patient underwent ultrasonography to assess interval change of the lesion, which confirmed a 3.4-cm × 1.6-cm × 3.7-cm mass that had grown proportionally with the patient and was relatively avascular", "Shortly thereafter, the mass was excised en bloc along with a cuff of adjacent posterior neck muscle and subcutaneous tissue", "Grossly, the lesion was firm, round, and white in color with a smooth intact border but no discrete capsule", "Histologic examination revealed cells with oval nuclei and multiple small nucleoli surrounded by a pale eosinophilic and slightly fibrillary stroma (Figure, B and C)", "Immunohistochemical studies were diffusely positive for glial fibrillary acidic protein and vimentin, and focally positive for S100, NeuN, and synaptophysin"], "s1": [0, 1, 3, 4, 8, 9], "s2": [2, 5, 6, 7, 10, 11, 12]} {"key": 652, "questions": "What Is Your Diagnosis?", "options": [{"label": "A", "disease": "Carotid artery dissection"}, {"label": "B", "disease": "Cerebral vein thrombosis"}, {"label": "C", "disease": "Posttraumatic central motor impairment"}, {"label": "D", "disease": "Postoperative laryngeal paralysis"}], "answer_idx": "B", "symptoms": ["A woman in her 30s with a medical history of epilepsy and alcohol use disorder who was taking lamotrigine presented with severe head trauma", "Her Glasgow Coma Score (GCS) was 11 M5V4E2 on admission, and a computed tomographic (CT) scan showed a right frontal epidural hematoma with a maximum thickness of 19 mm and a volume of 30 cm3 (Figure 1A)", "The hematoma was evacuated by neurosurgery, and her GCS improved to 14 postoperatively", "On postoperative day 2, she developed isolated dysphagia and dysphonia", "Laryngoscopic examination revealed right vocal cord paralysis (Figure 1B)", "A, Computed tomographic (CT) image of the head shows a right frontal epidural hematoma", "B, Clinical laryngoscopic image at day 2 shows right vocal cord palsy", "C and D, Computed tomographic images with contrast show a thrombosis of the right sigmoid-jugular complex (yellow arrowheads), which causes acute compression of the recurrent nerve as it follows the course of the cranial accessory nerve then the vagus nerve"], "s1": [0, 1, 2, 5], "s2": [3, 4, 6, 7]} {"key": 653, "questions": "What Is Your Diagnosis?", "options": [{"label": "A", "disease": "Hodgkin lymphoma"}, {"label": "B", "disease": "Mastoiditis"}, {"label": "C", "disease": "Kimura disease"}, {"label": "D", "disease": "Brachial cleft cyst"}], "answer_idx": "C", "symptoms": ["A 31-year-old Hispanic woman had a 3-year history of a steadily growing, increasingly pruritic, painless mass behind the right ear", "The patient denied bleeding from the mass, ear pain, trauma, fever, fatigue, night sweats, or recent illnesses", "Her medical history included eczema treated with triamcinolone acetonide cream", "Otherwise, the patient had no family history or exposure to tobacco or alcohol", "On physical examination, a 2-cm, nontender, mobile retroauricular mass with superficial excoriations was palpable over the right mastoid cortex", "The patient had no hearing deficits and no fistula into the external auditory canal", "There was no palpable head and neck lymphadenopathy", "Serologic studies revealed an elevated absolute eosinophil count of 790/μL (reference range, <700/μL", "to convert to ×109 per liter, multiply by 0.001) and an eosinophilia percentage of 9.7% (reference range, <6.0%)", "The T1-weighted postcontrast magnetic resonance imaging of the head and neck showed a heterogeneously enhancing mass with irregular borders in the right retroauricular space, with a tract coursing anteriorly toward the parotid gland (Figure, A)", "A, Axial T1-weighted postcontrast magnetic resonance image of the head and neck", "B, Lesional specimen of the excised mass (original magnification ×10)", "The excised mass was an enlarged, 2.2 × 1.5 × 2.2-cm lymph node with a smooth, pale core and no hemorrhage or necrosis", "The capsule was moderately thickened by collagen fibrosis with focal areas of interstitial fibrosis", "Reactive follicular hyperplasia was present with prominent eosinophilia in the interfollicular areas and within follicles, forming eosinophilic microabscesses and follicular cysts (Figure, B)", "A few small and ill-defined granulomas were present", "Results of Gomori methenamine silver and acid-fast stains were negative"], "s1": [0, 1, 2, 3, 4, 5, 6], "s2": [7, 8, 9, 10, 11, 12, 13, 14, 15, 16]} {"key": 654, "questions": "What Is Your Diagnosis?", "options": [{"label": "A", "disease": "Langerhans cell histiocytosis"}, {"label": "B", "disease": "Primary cutaneous large B-cell lymphoma, leg type"}, {"label": "C", "disease": "Merkel cell carcinoma"}, {"label": "D", "disease": "Cutaneous angiosarcoma"}], "answer_idx": "B", "symptoms": ["A woman in her early 80s presented with a 6-month history of an asymptomatic, enlarging, partly infiltrated erythematous cutaneous patch localized to the periareolar area of her left breast (Figure, A)", "She had no history of cancer or chronic diseases", "No cutaneous nodules were present", "Physical examination did not show adenopathies, hepatosplenomegaly, or other remarkable findings", "Peripheral blood test results were within normal limits", "Tests for tumor markers including cancer antigen (CA) 15-3, carcinoembryonic antigen, CA 19-9, and CA 125 were unremarkable", "No palpable breast masses were identifiable", "Breast ultrasonography and mammography did not reveal parenchymal nodules", "A local steroid therapy was administered, with no benefit", "A biopsy of the skin lesion was performed (Figure, B and C)", "A, Clinical image of the cutaneous breast lesion shows a large, erythematous, and slightly infiltrated patch in the periareolar region", "B and C, Hematoxylin-eosin–stained lesional specimens show a dense dermal infiltrate of cells with focal epidermis involvement (B) and large atypical cells with scanty cytoplasm (C)"], "s1": [0, 10, 9, 11], "s2": [1, 2, 3, 4, 5, 6, 7, 8]} {"key": 655, "questions": "What Is Your Diagnosis?", "options": [{"label": "A", "disease": "Pantothenate kinase–associated neurodegeneration"}, {"label": "B", "disease": "Hypobetalipoproteinemia"}, {"label": "C", "disease": "Chorea-acanthocytosis"}, {"label": "D", "disease": "Huntington disease–like 2"}], "answer_idx": "C", "symptoms": ["A 30-year-old man had involuntary orofaciolingual movements for 7 years and unsteady gait for 2 months", "At age 23 years, he began experiencing involuntary orofaciolingual movements and vocalizations", "At age 27 years, he developed dysphagia, dysarthria, and involuntary tongue- and lip-biting", "This oral dyskinesias worsened while eating", "However, a handkerchief in the mouth markedly reduced involuntary tongue-biting and mouth movements", "No neuropsychiatric symptoms or seizures were exhibited", "Neurological examination results revealed mild cognitive decline, with a Montreal Cognitive Assessment score of 18/30. Orofacial dyskinesia (Video 1), involuntary vocalizations, and mild dysarthria were exhibited", "His muscle strength and tone were normal, but his deep reflexes were diminished or absent", "The patient’s sensory and coordination examination results were unremarkable, and bilateral Babinski signs were negative", "The patient’s gait was mildly wide based (Video 2)", "Fundus examination results were normal", "Laboratory examination results, including blood electrolytes and lactate, liver, kidney, and thyroid function, antinuclear antibodies, ceruloplasmin levels, and vitamin A and E levels were normal", "Routine and biochemical examination results, oligonucleotide band, antibodies against antiaquaporin 4, and autoimmune encephalitis-related antibodies (ie, N-methyl-d-aspartate, α-amino-3-hydroxy-5-methyl-4-isoxazoleproprionic, γ-aminobutyric acid-B, leucine-rick glioma inactivated 1, contactin-associated protein 2, and IgLON5) of cerebrospinal fluid were normal", "Creatine kinase (CK) levels were significantly elevated at 2304 U/L (to convert to microkatals per liter, multiply by 0.0167", "normal range, 25-200 U/L)", "Low-density lipoprotein and apolipoprotein B levels were slightly decreased at 61 mg/dL (to convert to micromoles per liter, multiply by 0.0357", "normal range, 80-119 mg/dL) and 58 mg/dL (to convert to grams per liter, multiply by 0.01", "normal range, 66-133 mg/dL), respectively", "Acanthocytes were found in the blood smear via Wright staining (Figure, A)", "Electrophysiological examination results revealed reduced compound muscle action potential and sensory nerve action potential of bilateral median and ulnar nerves", "Electromyogram results showed neurogenic damage in lower limb muscles", "Cerebral magnetic resonance imaging results revealed a marked bilateral caudate head atrophy (Figure, B)", "A, Acanthocytes in the peripheral blood smear shown via Wright staining (original magnification, ×1000)", "B, Axial T2-weighted magnetic resonance image of the patient’s brain"], "s1": [0, 1, 2, 3, 4, 5, 6, 7, 8, 9], "s2": [10, 11, 12, 13, 14, 15, 16, 17, 18, 19, 20, 21, 22, 23]} {"key": 656, "questions": "What Is Your Diagnosis?", "options": [{"label": "A", "disease": "Cutaneous T-cell lymphoma"}, {"label": "B", "disease": "Granuloma annulare"}, {"label": "C", "disease": "Small-vessel vasculitis"}, {"label": "D", "disease": "Sarcoidosis"}], "answer_idx": "A", "symptoms": ["A man in his 50s was referred to the dermatology clinic with a diagnosis of granuloma annulare made on the basis of prior skin biopsy results", "He had been treated by a rheumatologist for polyarticular inflammatory arthritis involving the wrists, metacarpophalangeal joints, and proximal interphalangeal joints, as well as fatigue and dyspnea on exertion", "Examination revealed numerous, pink-purple dermal papules on the trunk and extremities, with preference for the extensor surfaces of the arms (Figure, A) and legs", "No lymphadenopathy was noted", "The patient reported a 5- to 7-kg weight loss without fever or sweats", "His pulmonary and joint symptoms were relieved by daily oral prednisone and flared with taper below 10 mg", "Other medications prescribed included hydroxychloroquine, 400 mg, daily for 18 months and weekly adalimumab injections for 7 months, without improvement", "A, Pink-purple papules on the extensor surface of the arms", "B, Biopsy results show a nodular to diffuse lymphohistiocytic infiltrate (hematoxylin-eosin)", "C, Admixed atypical, irregularly contoured lymphocytes with numerous histiocytes are present", "D, Biopsy results show low CD4 signal and robust CD8 signal (magnification ×20)", "Pulmonary function testing revealed mild restrictive disease with decreased forced vital capacity and total lung capacity", "Results of a complete blood cell count and metabolic panel", "hemoglobin A1c", "viral and rheumatologic panels, including HIV blood tests", "chest radiograph", "high-resolution chest computed tomographic scan", "echocardiography", "endoscopic studies", "hand radiographs", "and serum and urine protein electrophoresis were normal or otherwise unremarkable", "The results of lesional skin biopsies obtained from multiple involved sites initially revealed nonspecific granulomatous dermatitis before a diagnostic biopsy was performed (Figure, B, C, and D)"], "s1": [0, 2, 3, 7, 8, 9, 10, 21], "s2": [1, 4, 5, 6, 11, 12, 13, 14, 15, 16, 17, 18, 19, 20]} {"key": 657, "questions": "What Is Your Diagnosis?", "options": [{"label": "A", "disease": "Porokeratotic eccrine ostial and dermal duct nevus"}, {"label": "B", "disease": "Punctate palmoplantar keratoderma"}, {"label": "C", "disease": "Familial comedonal Darier disease"}, {"label": "D", "disease": "Nevus comedonicus"}], "answer_idx": "D", "symptoms": ["A girl in her teens presented for evaluation of asymptomatic, punctate, keratotic papules localized to the right palm and right third proximal finger that had been present since age 2 years", "Prior treatments included cryotherapy, pulsed dye laser, and 40% salicylic acid, with partial improvement reported but never complete clearance", "Medical history and family history were unremarkable", "Physical examination revealed grouped and circumscribed, firm, skin-colored, 1- to 3-mm papules with a central punctate dark core (Figure, A and B)", "A 4-mm punch biopsy from a papule on the palm was performed (Figure, C and D)", "A and B, Grouped, dome-shaped, skin-colored, punctate papules with central keratotic plugging localized to the right palm and finger", "C and D, Dilated follicular infundibulum with keratinous core and absent hair shaft (hematoxylin-eosin)"], "s1": [2, 3, 5, 6], "s2": [0, 1, 4]} {"key": 658, "questions": "What Is Your Diagnosis?", "options": [{"label": "A", "disease": "Erosive osteoarthritis"}, {"label": "B", "disease": "Metastatic bone disease"}, {"label": "C", "disease": "Nivolumab-induced inflammatory arthritis"}, {"label": "D", "disease": "Seronegative rheumatoid arthritis"}], "answer_idx": "C", "symptoms": ["A 58-year-old white woman with history of metastatic human papillomavirus–positive cervical adenocarcinoma presented with multiple joint deformities", "Her initial symptoms started 1 month after the initiation of nivolumab, which was 1 year prior to the index visit", "Treatment with nonsteroidal anti-inflammatory drugs failed, and the patient had a partial response to intra-articular steroids", "She refused systemic therapies owing to concern about diminishing the effectiveness of nivolumab", "Despite the joint symptoms, nivolumab treatment was continued for a year", "Evaluation showed fixed swan neck deformities in multiple fingers (Figure, A)", "There was evidence of chronic synovial hypertrophy with no active synovitis on examination", "The results of a comprehensive autoantibody blood panel, including antinuclear antibody, rheumatoid factor, and anti-citrullinated peptide antibody, were unremarkable", "Plain radiographs of the hands demonstrated diffuse osteopenia, joint space narrowing, and multiple deformities", "Magnetic resonance imaging of both hands revealed multifocal osseous erosions (Figure, B), synovitis, and tenosynovitis", "Erosive deforming inflammatory arthritis", "A, Fixed swan neck deformities in multiple fingers (arrowheads)", "B, Magnetic resonance images of both hands reveal multifocal osseous erosions (arrowheads)"], "s1": [0, 1, 3, 4, 6, 7], "s2": [2, 5, 8, 9, 10, 11, 12]} {"key": 659, "questions": "What Is Your Diagnosis?", "options": [{"label": "A", "disease": "Postinfectious myelopathy"}, {"label": "B", "disease": "Toxic myelopathy"}, {"label": "C", "disease": "Glioma"}, {"label": "D", "disease": "Spinal dural arteriovenous fistula"}], "answer_idx": "C", "symptoms": ["A 48-year-old woman presented to the emergency department with a 2-month history of progressive lower extremity weakness, sensory loss, and sphincter dysfunction", "In the weeks prior to onset, she traveled to Yosemite National Park but denied any particular infectious exposures or symptoms including rash", "She reported no personal or family medical history", "Although she did consume a variety of nutraceuticals, she denied medication and recreational drug use", "On examination, mental status, cranial nerves, and upper extremities were normal", "Lower extremities were significant for her right side having greater pyramidal weakness than the left, with brisk reflexes and right ankle clonus, as well as a sensory level to pinprick at T6. Magnetic resonance imaging (MRI) of the total spine with and without gadolinium is seen in Figure 1. Initial investigations were notable for normal complete blood cell count, electrolytes, kidney and liver function, B12, and thyrotropin as well as an unremarkable MRI of the brain", "Three lumbar punctures were performed over the course of the patient’s hospitalization, each with a white blood cell count less than 5 /µL (to convert to ×109/L, multiply by 0.001), normal IgG index, absent oligoclonal bands, normal protein and glucose levels, and benign cytology", "A comprehensive infectious and inflammatory workup including HIV, herpes simplex virus, varicella-zoster virus, Lyme disease, antinuclear antibodies, astrocyte aquaporin-4 autoantibody, myelin oligodendrocyte glycoprotein autoantibody, and serum and cerebrospinal fluid autoimmune panels were negative", "Whole-body positron emission tomography demonstrated hypermetabolism of the midthoracic cord", "A, Long-segment, expansile T2 hyperintense lesion spanning C5 through T12 and involving greater than two-thirds of the cord diameter", "The spinal cord is mildly diffusely swollen without an appreciated focal mass", "B, Associated extensive ill-defined enhancement of the spinal cord spanning T3 through T9."], "s1": [1, 2, 3, 4, 6, 7], "s2": [0, 5, 8, 9, 10, 11]} {"key": 660, "questions": "What Is Your Diagnosis?", "options": [{"label": "A", "disease": "Squamous cell carcinoma"}, {"label": "B", "disease": "Lymphoma"}, {"label": "C", "disease": "Papillary adenocarcinoma"}, {"label": "D", "disease": "Chondrosarcoma"}], "answer_idx": "C", "symptoms": ["A healthy Asian man in his 20s presented with a 5-month history of progressive bilateral nasal obstruction", "He also complained of recurrent epistaxis", "He had no notable medical history except stable asthma", "On physical examination, nasopharyngoscopy showed a pedunculated mass with a smooth mucosal surface arising from the roof of the nasopharynx", "Computed tomographic (CT) imaging (Figure, A and B) revealed a midline, well-defined, pedunculated polypoid lesion with areas of calcifications in the nasopharynx, measuring approximately 1.2 × 1.5 × 2.0 cm", "An endoscopic resection via retrovelar approach using a 70-degree endoscope combined with transnasal approach was performed under general anesthesia", "The mass was completely resected with its stalk and with adequate surgical margin", "Histopathological results (Figure, C and D) showed that the tumor cells were arranged in papillary structures with fibrovascular cores and these cells also had nuclear grooves", "Results of other investigations after surgery were negative for cancer, including magnetic resonance imaging (MRI) of the neck and thyroid ultrasonography", "In addition, no residual tumor or systemic metastasis was observed on positron emission tomography (PET)-CT results", "There was no evidence of recurrence at 12-month postoperative follow-up", "A and B, Computed tomography images with contrast enhancement reveal a 1.2 × 1.5 × 2.0-cm soft-tissue–density mass (white arrowheads) with multiple nodular calcifications (red arrowhead) in the nasopharynx", "C and D, Lesional histopathologic images (hematoxylin-eosin) show tumor cells arranged in papillary structures with fibrovascular cores (asterisk)", "Nuclear grooves (black arrowheads) in these cells were also identified"], "s1": [0, 1, 2, 5, 6, 10], "s2": [3, 4, 7, 8, 9, 11, 12, 13]} {"key": 661, "questions": "What Is Your Diagnosis?", "options": [{"label": "A", "disease": "Squamous cell carcinoma"}, {"label": "B", "disease": "Paracoccidioidomycosis"}, {"label": "C", "disease": "Pemphigus vulgaris"}, {"label": "D", "disease": "Lupus vulgaris (cutaneous tuberculosis)"}], "answer_idx": "C", "symptoms": ["A 66-year-old man presented to the emergency department for painful and pruritic ulcerative cutaneous lesions involving his back, nose, lips (Figure, A), and mucosal lesions involving his oral cavity and oropharynx", "The lesions initially only involved his lips and nasal bridge, and appeared 5 months prior while he was living on a ranch in Mexico", "The lesions did not respond to treatment with an unknown oral antibiotic", "His medical history included hypertension treated with captopril and a remote history of psoriasis", "Physical examination demonstrated several dry, crusted skin lesions with blistering and ulceration, and shallow erythematous ulcerations of his oral mucosa", "Internal medicine, otolaryngology, and dermatology departments were consulted", "Results of flexible fiberoptic laryngoscopy demonstrated erythematous and ulcerative lesions with surrounding leukoplakia involving the base of tongue, glossoepiglottic fold, lingual and laryngeal surfaces of the epiglottis, and aryepiglottic folds (Figure, C and D)", "A QuantiFERON-TB Gold test was obtained, and results were found to be positive", "Serum antigen testing for blastomyces, histoplasmosis, syphilis, HIV, hepatitis B virus, hepatitis C virus, human herpesvirus 6, cytomegalovirus, Epstein-Barr virus, varicella zoster virus, human polyomavirus 2, and West Nile virus were performed, and all results were found to be negative", "A biopsy of the skin lesion was obtained from the edge of the lesion found on the patient’s back (Figure, B)", "Enzyme-linked immunosorbent assay serological testing was subsequently performed, and results demonstrated elevated levels of antidesmoglein 1 immunoglobulin G (IgG) and antidesmoglein 3 IgG", "A, Lesions involving the patient’s nose and lower lip", "B, Direct immunofluorescence staining for immunoglobin G of an intralesional biopsy at magnification ×34 demonstrating autoantibody deposition in a lacelike pattern localized to the intracellular matrix of the stratified squamous epithelium", "C, Bullous lesion involving the epiglottis (black arrowhead)", "D, Bullous lesions involving the left posterior true vocal fold, left false vocal fold, and right vestibule (black arrowheads)"], "s1": [0, 1, 3, 4, 8, 10, 12], "s2": [2, 5, 6, 7, 9, 11, 13, 14]} {"key": 662, "questions": "What Is Your Diagnosis?", "options": [{"label": "A", "disease": "IgG4-related sialadenitis"}, {"label": "B", "disease": "Kimura disease"}, {"label": "C", "disease": "Eosinophilic sialodochitis"}, {"label": "D", "disease": "Angiolymphoid hyperplasia with eosinophilia"}], "answer_idx": "C", "symptoms": ["A woman presented with a 2-year history of recurring left submandibular gland swelling and pain", "She reported no associated fevers, chills, dryness of mouth, nausea, vomiting, dysphagia or odynophagia", "Her medical history was unremarkable except for allergic rhinitis", "On physical examination, she had slight enlargement of the left submandibular gland, which was not tender on palpation", "Ultrasonography results revealed normal submandibular and parotid glands bilaterally without any mass or calcifications", "Blood examinations and basic metabolic panel results were all within normal limits", "Sialoendoscopy of the left submandibular duct was performed under general anesthesia", "The patient was found to have a cast at the opening of the Wharton duct, which was removed and sent for pathological evaluation (Figure)", "Using sialoendoscopy with saline irrigation, the entire duct was examined up to the hilum of the gland", "No stones were noted", "There were areas of mild partial webbing without complete obstruction", "The areas of webbing were broken up with irrigation, and at the end of the procedure steroid instillation and irrigation were performed", "There were no complications during or after the procedure", "Hematoxylin-eosin–stained histological slide of the cast showing numerous eosinophils"], "s1": [0, 1, 2, 3, 4, 5], "s2": [6, 7, 8, 9, 10, 11, 12, 13]} {"key": 663, "questions": "What Is Your Diagnosis?", "options": [{"label": "A", "disease": "Guillain-Barré syndrome"}, {"label": "B", "disease": "Lambert-Eaton syndrome"}, {"label": "C", "disease": "Miller Fisher syndrome"}, {"label": "D", "disease": "Myasthenia gravis and myositis syndrome"}], "answer_idx": "D", "symptoms": ["A 59-year-old man with nonalcoholic steatohepatitis was found to have a 12.4-cm liver mass, with imaging characteristics concerning for malignant neoplasm (Figure, A)", "A biopsy was performed, and pathology results were consistent with a well-differentiated hepatocellular carcinoma", "He enrolled in a phase 1 clinical trial (NCT03299946) using neoadjuvant cabozantinib plus nivolumab followed by definitive resection for patients with locally advanced disease", "A, Magnetic resonance image of the abdomen shows a liver mass", "B, On single-fiber electromyography, the mean consecutive difference of the tested pairs was abnormal, and there was no blocking", "Five weeks after the first dose of nivolumab, he developed diplopia and myalgia that mostly affected proximal muscle groups", "Physical examination revealed ptosis, modest restriction of ocular motility in all directions bilaterally, and esotropia with left hypertropia in the primary gaze", "Muscle strength was 5/5 on hip flexion and extension bilaterally", "There was no weakness with repetitive and sustained testing of power", "Workup revealed elevated creatine kinase (1222 U/L", "reference range, 24-195 U/L [to convert to μkat/L, multiply by 0.0167]), aldolase (24.5 U/L", "reference range, <8.1 U/L [to convert to μkat/L, multiply by 0.0167]), and C-reactive protein levels (106 mg/L", "reference range, <5 mg/L [to convert to nmol/L, multiply by 9.524])", "Acetylcholine receptor binding and modulating antibody levels were elevated (0.69 nmol/L [reference range, <0.3 nmol/L] and 48% [reference range, 0%-20%], respectively)", "Results of the striated muscle antibody titer were also abnormal (1:160", "reference range, <1:40)", "Blood test results were negative for acetylcholine receptor blocking and muscle-specific receptor tyrosine kinase antibodies", "Nerve conduction studies showed a normal response to repetitive nerve stimulation", "Single-fiber electromyography was pathologic and revealed an abnormal mean consecutive difference in more than 50% of the tested pairs (Figure, B), and no blocking was seen", "Myopathic action potentials were not present", "A chest computed tomographic scan ruled out the presence of a thymoma"], "s1": [0, 1, 2, 3, 20], "s2": [4, 5, 6, 7, 8, 9, 10, 11, 12, 13, 14, 15, 16, 17, 18, 19]} {"key": 664, "questions": "What Is Your Diagnosis?", "options": [{"label": "A", "disease": "Sclerotic neurofibroma"}, {"label": "B", "disease": "Familial cutaneous collagenoma"}, {"label": "C", "disease": "Shagreen patch"}, {"label": "D", "disease": "Fibrous cephalic plaque"}], "answer_idx": "D", "symptoms": ["A man in his 20s presented with a 6-year history of multiple growing papules superimposed on an asymptomatic firm plaque on the occipital scalp", "The lesions were never ulcerated or inflamed, but hemorrhage occurred occasionally after rubbing", "At 3 years of age, scarlike macules appeared on the occipital scalp and coalesced to form a larger plaque with sparse hair", "Simultaneously, multiple brown-red, dome-shaped papules developed on the nose and gradually evolved to involve the cheeks", "Rubbery plaques also developed on the sacrococcygeal area, along with hypopigmented patches on the right buttock", "The patient has no systemic symptoms or mental impairment and denied a family history of genetic disease", "Physical examination revealed a group of unevenly distributed skin–colored-to-pink, discrete, coalescing papules superimposed on a firm plaque on the occipital scalp (Figure, A)", "A biopsy specimen was obtained from one of these papules (Figure, B, C, and D)", "Another biopsy from a facial papule demonstrated features of angiofibroma", "A, Clinical photograph shows a firm plaque with superimposed papules and decreased hair density", "Biopsy specimens from one of the papules demonstrate diffuse dense fibrosis (B), capillary vessel proliferation with some ectasia (C) (hematoxylin-eosin), and perifollicular fibroblastic proliferation (D)"], "s1": [0, 1, 2, 3, 4, 5], "s2": [6, 7, 8, 9, 10]} {"key": 665, "questions": "What Is Your Diagnosis?", "options": [{"label": "A", "disease": "Neurofibroma"}, {"label": "B", "disease": "Anetoderma"}, {"label": "C", "disease": "Dermatofibrosarcoma protuberans, atrophic variant"}, {"label": "D", "disease": "Medallionlike dermal dendrocyte hamartoma"}], "answer_idx": "C", "symptoms": ["A young man in his late teens presented with numerous atrophic papules and plaques on his trunk and extremities that developed gradually over several years", "Although most were asymptomatic, the patient was concerned about the ongoing appearance of new lesions and the associated pain that some lesions were causing", "His medical history was significant for adenosine deaminase–deficient severe combined immunodeficiency (ADA-SCID), which had been managed since childhood with twice weekly intramuscular injections of pegademase bovine enzyme replacement, thrice weekly trimethoprim-sulfamethoxazole for Pneumocystis prophylaxis, and monthly infusions of intravenous immunoglobulin", "Physical examination revealed 9 smooth-surfaced, skin-colored to hyperpigmented, 0.5- to 1.5-cm atrophic papules and plaques that exhibited the buttonhole sign on palpation (Figure, A)", "The multicentric lesions were located on the left knee, left thigh, bilateral chest, and back", "Punch biopsies of lesions on his trunk and extremities were sent for histopathological examination with hematoxylin-eosin (H&E) staining (Figure, B and C), immunohistochemical (IHC) studies, and molecular evaluation with reverse transcription polymerase chain reaction", "A, The patient presented with 2 distinct, smooth-surfaced, 0.5- to 1.0-cm atrophic plaques with overlying hyperpigmentation on the left thigh", "Both lesions exhibited the buttonhole sign on palpation", "B and C, Histopathology shows poorly circumscribed hypercellular proliferation of spindle cells occupying the lower reticular dermis with lacelike infiltration of the subcutaneous fat (hematoxylin-eosin)", "The somewhat whorled, uniform appearing spindle cells with mildly hyperchromatic, elongated, wavy nuclei surrounded by pale cytoplasm", "Mitotic figures and nuclear pleomorphism are uncommon"], "s1": [0, 1, 2, 3, 4, 6, 7], "s2": [5, 8, 9, 10]} {"key": 666, "questions": "What Is Your Diagnosis?", "options": [{"label": "A", "disease": "Olfactory neuroblastoma"}, {"label": "B", "disease": "Sinonasal lymphoma"}, {"label": "C", "disease": "NUT carcinoma"}, {"label": "D", "disease": "Nasopharyngeal carcinoma"}], "answer_idx": "C", "symptoms": ["A 29-year-old healthy woman presented with a 2-month history of left-sided epiphora, headache, and facial pressure", "A polypoid lesion was identified at the left lateral nasal wall", "Noncontrasted computed tomography showed a hyperdense mass involving the left medial canthus extending through the nasolacrimal duct into the left nasal cavity", "The patient underwent endoscopic left maxillary antrostomy, ethmoidectomy, and biopsy of mass, with results of pathology analysis showing poorly differentiated squamous cell carcinoma", "Immunohistochemical (IHC) staining was positive for cytokeratin 5/6 and p63, and negative for Epstein-Barr virus, p16, CD45 (leukocyte common antigen), and synaptophysin", "The patient was referred to a tertiary care facility for further treatment", "On examination, there was a palpable subcutaneous nodule 1 cm inferior to the left medial canthus with normal overlying skin", "On nasal endoscopy, residual tumor was visible at the anterior aspect of the lateral nasal wall", "Computed tomography showed postoperative changes with enhancement of left medial canthus extending into nasolacrimal duct and along the lateral nasal sidewall (Figure, A)", "A, Coronal view of contrast-enhanced maxillofacial computed tomography obtained after initial biopsy and before resection showing tumor involvement of lateral nasal sidewall and nasolacrimal duct", "B and C, Hematoxylin eosin–stained histologic sections of tumor after resection showing bony invasion by sheets of undifferentiated cells (B) and focal areas of abrupt keratinization (C)", "The patient subsequently underwent left medial maxillectomy via lateral rhinotomy approach, with resection of residual tumor", "The tumor appeared to originate from left lateral nasal wall, with involvement of nasal floor and extension into the orbit and periorbita", "Histologically, the tumor extensively eroded maxillary bone, with abundant necrosis and widespread involvement of surgical margins", "Microscopic examination showed sheets of monomorphic undifferentiated tumor cells with interspersed areas of abrupt squamous differentiation and keratinization (Figure, B and C)"], "s1": [0, 1, 3, 5, 6, 7, 11], "s2": [2, 4, 8, 9, 10, 12, 13, 14]} {"key": 667, "questions": "What Is Your Diagnosis?", "options": [{"label": "A", "disease": "Cutaneous metastasis from small cell lung cancer"}, {"label": "B", "disease": "Loxoscelism"}, {"label": "C", "disease": "Merkel cell carcinoma"}, {"label": "D", "disease": "Primary cutaneous lymphoma (Sézary syndrome)"}], "answer_idx": "C", "symptoms": ["An 81-year-old woman presented with a 30 cm-wide, easily bleeding, violaceous, cutaneous lesion of the left scapular region with a superinfected necrotic central portion surrounded by satellite telangiectatic papules (Figure, A)", "The patient first noticed a rapidly expanding, itchy, painful nodule, which measured 3 cm in diameter, 5 months prior to presentation (Figure, B)", "Lesion appearance at the first visit (A) and 5 months earlier (B)", "C, Hematoxylin-eosin stain, magnification ×20.The patient reported decreased appetite with weight loss of 7 kg over 5 months and denied fever, itching aside from the lesion, vomiting, or bowel alterations", "Owing to intense and continuous pain from the lesion, she began opioid treatment with oxycodone/naloxone association at an initial dose of 5 mg/2.5 mg twice a day and titrated to 10 mg/5 mg twice a day after a week to achieve pain control", "Abdominal ultrasonography revealed only a left adrenal gland mass, consistent with adenoma", "On physical examination, there were no other cutaneous lesions though multiple ipsilateral axillary lymph nodes, and enlarged supraclavicular and cervical lymph nodes were present", "All other physical and laboratory findings were unremarkable", "A skin punch biopsy of the lesion was obtained (Figure, C)"], "s1": [0, 1, 2, 3, 4, 8], "s2": [5, 6, 7]} {"key": 668, "questions": "What Is Your Diagnosis?", "options": [{"label": "A", "disease": "Linear morphea"}, {"label": "B", "disease": "Serpentine supravenous hyperpigmentation"}, {"label": "C", "disease": "Intraoperative cutaneous electrical injury (Lichtenberg figure)"}, {"label": "D", "disease": "Intralymphatic histiocytosis"}], "answer_idx": "D", "symptoms": ["A man in his 80s presented with an 18-month history of an irregularly linear, rusty-brown, atrophic, streaklike plaque extending from the tip of his right shoulder to the right biceps, where it ended in arborizing purple plaque with scattered, slightly indurated, irregular papules within it (Figure, A and B)", "There was no warmth, thrills, or pulsations across the affected area", "The patient complained about a mild tingling sensation within the lesion", "The patient could not think of any triggers or causes for his condition and denied any aggravating or alleviating factors as well as any treatment attempts", "Complete review of systems yielded negative results except for easy bruising", "He was taking finasteride, montelukast, enalapril, and metoprolol", "His medical history was significant for right total reverse shoulder replacement 5 years prior", "Apart from the previously described lesion, his physical examination was unremarkable, and he had full range of motion of his right shoulder", "A punch biopsy of the slightly indurated papule from the involved area on the right biceps was completed (Figure, C and D)", "A and B, An irregularly linear, rusty-brown, atrophic, streaklike plaque extends from the tip of the right shoulder to the right biceps, where it ends in arborizing purple plaque with scattered, slightly indurated, irregular papules within it", "C, Hematoxylin-eosin staining demonstrates a dermal infiltrate arranged in nodules with clefting between nodules", "some inflammatory aggregates are also seen within dilated vessels", "D, Immunohistochemical staining for CD68 antigen reveals that the infiltrate is composed mostly of histiocytes"], "s1": [0, 1, 2, 3, 4, 6, 7], "s2": [5, 8, 9, 10, 11, 12]} {"key": 669, "questions": "What Is Your Diagnosis?", "options": [{"label": "A", "disease": "Hemochromatosis"}, {"label": "B", "disease": "Diffuse melanosis cutis"}, {"label": "C", "disease": "Addison disease"}, {"label": "D", "disease": "Hyperpigmentation due to sun exposure"}], "answer_idx": "B", "symptoms": ["A white man in his early 40s with a medical history of chronic lymphocytic leukemia (Rai stage 0) reported a 6-month history of a growing mass in his right axilla and weight loss", "Physical examination revealed palpable right axillary lymphadenopathy", "The patient was an active smoker (1 pack per day) with an unremarkable family history", "A positron emission tomography scan showed a hypermetabolic right axillary lymph node that measured 5 × 5 cm as well as multiple liver lesions, the largest of which measured 8 cm", "The results of a core needle biopsy specimen from the right axillary lymph node were consistent with malignant neoplasm, with immunohistochemistry staining positive for S100, HMB-45, and MART-1. He began pembrolizumab therapy", "After starting treatment, the patient began to notice slowly worsening painless, nonpruritic, diffuse dark gray skin, nail beds, and eye discoloration (Figure)", "The results of laboratory studies revealed normal electrolytes, normal kidney function, mild elevation of liver function, and mild anemia with a hemoglobin level of 12 g/dL (for SI unit conversion, multiply by 10.0 to convert to liters)", "Computed tomography of the chest, abdomen, and pelvis showed worsening hepatic metastases and right axillary adenopathy"], "s1": [2, 5, 6], "s2": [0, 1, 3, 4, 7]} {"key": 670, "questions": "What Is Your Diagnosis?", "options": [{"label": "A", "disease": "Tuberculosis of the palate"}, {"label": "B", "disease": "Tertiary syphilis"}, {"label": "C", "disease": "Histoplasmosis"}, {"label": "D", "disease": "Sarcoidosis"}], "answer_idx": "C", "symptoms": ["A 45-year-old man was admitted to the hospital following 3 weeks of dysphagia", "1 week of nasal regurgitation, nonproductive cough, and breathy voice", "and a 25-pound weight loss over the preceding 2 months", "He had been diagnosed a year earlier with HIV infection and hepatic tuberculosis", "Baseline CD4 count was 7 cells/μL, and following treatment with HAART (highly active antiretroviral therapy) consisting of dolutegravir, emtricitabine, and tenofovir-disoproxil-fumarate, he achieved full virological suppression", "however, CD4 counts remained low at 31 cells/μL", "He had completed 12 months of directly observed therapy with ethambutol, moxifloxacin, and rifabutin—a liver-sparing regimen—owing to toxic effects from the first-line regimen 1 month prior to admission", "Physical examination revealed evidence of perforation of the hard palate (Figure, A)", "A computed tomographic scan of the chest revealed necrotic lymph nodes, and a palatine biopsy was performed (Figure, B and C)", "A, Clinical view of the perforation of the hard palate", "B, Low-power microscopic view of a biopsy specimen from the hard palate (hematoxylin-eosin)", "C, Oil immersion image of the same tissue shown panel B stained with Grocott methenamine silver"], "s1": [0, 1, 2, 3, 4, 5, 6], "s2": [7, 8, 9, 10, 11]} {"key": 671, "questions": "What Is Your Diagnosis?", "options": [{"label": "A", "disease": "Benign acanthosis nigricans"}, {"label": "B", "disease": "Cutaneous T-cell lymphoma"}, {"label": "C", "disease": "Malignant acanthosis nigricans"}, {"label": "D", "disease": "Malignant Addison disease"}], "answer_idx": "C", "symptoms": ["A 30-year-old woman presented with a 4-month history of widespread thickened, verrucous, hyperpigmented plaques distributed symmetrically on her body, especially on the axillae (Figure 1A), anogenital region, inguinal skin, and both palms, with slight itching", "The verrucous plaques also involved the conjunctiva, lips, and gingiva (Figure 1B)", "The patient had experienced a weight loss of 6 kg during the last 3 months", "Within the last month, the patient’s thyroid became enlarged, and she developed anorexia and abdominal discomfort", "Laboratory test results revealed remarkably elevated levels of carcinoembryonic antigen, carbohydrate antigen 19-9, carbohydrate antigen 125, tissue polypeptide antigen and carbohydrate antigen 242. Biopsy specimens of lesions on the axillae and lips both revealed a papillary structure with hyperkeratosis", "An endoscopic examination and computed tomographic scan of the abdomen were performed", "Axilla with velvety, hyperpigmented skin (A) and verrucous plaques on the lips and gingiva (B)"], "s1": [0, 1, 6], "s2": [2, 3, 4, 5]} {"key": 672, "questions": "What Is Your Diagnosis?", "options": [{"label": "A", "disease": "Acrokeratosis paraneoplastica"}, {"label": "B", "disease": "Cutaneous leukocytoclastic vasculitis"}, {"label": "C", "disease": "Purpura fulminans"}, {"label": "D", "disease": "Henoch-Schönlein purpura"}], "answer_idx": "B", "symptoms": ["A 70-year-old Indian woman presented with a generalized rash over the lower extremity associated with pedal edema extending up to the shins", "The onset was gradual and started on the lower extremities and gradually extended to involve the lower trunk", "The rash was associated with mild discomfort", "The patient denied itching, weeping, or bleeding from the rash", "She denied any constitutional symptoms (weight loss, fever, night sweats), joint pains, history of blood in urine, periorbital edema, and recent intake of medications or herbal medications", "Social history was significant for 25 pack-years of smoking", "Physical examination revealed multiple well-defined, coalescent, erythematous, palpable, purpuralike lesions that were predominantly present over the trunk and lower extremities (Figure, A and B)", "Laboratory test results at presentation included a hemoglobin level of 11.2 g/dL (to convert to g/L, multiply by 10), mean corpuscular volume of 84 fL, erythrocyte sedimentation rate of 72 mm/h, and a lactate dehydrogenase level of 412 IU/L", "Hepatic and renal function levels were within normal limits", "Complement levels were normal", "Hepatitis B and C and HIV profiles were negative", "A chest radiograph showed a 2-cm lesion in the upper lobe of the left lung", "A computed tomographic scan of the chest showed a spiculated mass in the same region along with mediastinal lymphadenopathy", "An ultrasound of the abdomen showed bilateral adrenal masses", "A lesional skin biopsy specimen was obtained (Figure, C)", "Coalescent, erythematous, palpable lesions on the lower extremity (A) and trunk (B)", "C, Lesional skin biopsy sample"], "s1": [0, 1, 2, 3, 4, 6, 15, 16], "s2": [5, 7, 8, 9, 10, 11, 12, 13, 14]} {"key": 673, "questions": "What Is Your Diagnosis?", "options": [{"label": "A", "disease": "Pyogenic granuloma"}, {"label": "B", "disease": "Gingival/periodontal abscess"}, {"label": "C", "disease": "Oral metastasis"}, {"label": "D", "disease": "Squamous cell carcinoma"}], "answer_idx": "C", "symptoms": ["A 59-year-old Hispanic woman presented with a 6-month history of intermittent pain in the right posterior mandibular gingiva", "Her local dentist had prescribed an antibiotic and performed a tooth extraction in the area 6 months prior with no improvement in symptoms", "Ultimately, her pain became persistent and radiated to the right lower lip", "She had a history of stage IV endometrial serous adenocarcinoma (ESA) 4 years prior that was treated with total hysterectomy, bilateral salpingo-oophorectomy, omentectomy, and peritoneal lavage", "She had developed a recurrence for which she had just completed 3 cycles of treatment with carboplatin", "She also had a history of anxiety, hypertension, and pterygium, and she was taking gabapentin, lorazepam, prochlorperazine, metoprolol succinate, bromfenac and ketorolac ophthalmic solution, and aprepitant", "Intraoral examination revealed a 1.5 × 1.0-cm firm, erythematous gingival mass in the area of missing right mandibular second bicuspid and first molar with buccal and lingual expansion (Figure 1A)", "The right mandibular first bicuspid was grade 3 mobile", "A periapical radiograph was obtained, and results showed a diffuse, poorly demarcated radiolucency measuring approximately 1.2 × 0.8 cm (Figure 1B)", "Findings from an incisional biopsy of the mass showed an infiltrative tumor (Figure 1C)", "Tumor cells were positive for CK7, PAX8, and WT-1, and negative for CK20 and TTF-1 (Figure 1C, inset)", "A, A 1.5 × 1-cm gingival mass is distal to the right mandibular first bicuspid", "B, Diffuse, poorly demarcated radiolucency is seen in the area of the missing right mandibular second bicuspid and first molar (arrowheads)", "C, Multiple nests of round to ovoid epithelial cells form ducts (hematoxylin-eosin stain, original magnification ×100)", "Diffuse nuclear positivity for PAX8 (immunohistochemistry, original magnification ×200) (inset)"], "s1": [0, 1, 2, 3, 4, 5], "s2": [6, 7, 8, 9, 10, 11, 12, 13, 14]} {"key": 674, "questions": "What Is Your Diagnosis?", "options": [{"label": "A", "disease": "Diphtheria"}, {"label": "B", "disease": "Mononucleosis"}, {"label": "C", "disease": "Type 2 herpes simplex virus (HSV-2) pharyngitis"}, {"label": "D", "disease": "Pharyngeal gonorrhea"}], "answer_idx": "C", "symptoms": ["A 27-year-old man with a history of pharyngeal gonorrhea presented with a 7-day history of severe sore throat and odynophagia, worse when swallowing solids than liquids", "Owing to the odynophagia, he had presented to the emergency department for dehydration", "At that time, he was febrile to 101°F and denied any dysphonia or dyspnea", "The results of a rapid Streptococcus test, recent HIV screen, and a rapid plasma reagin test were all negative", "He denied any genital lesions or symptoms at that time", "He had been taking prophylactic emtricitabine-tenofovir daily and had received his last diphtheria booster 2 years prior to presentation", "Treatment with amoxicillin clavulanate over several days had not prevented a worsening of his symptoms", "On physical examination, there was no cervical lymphadenopathy, and there were no genital lesions", "Flexible fiber-optic nasopharyngoscopy showed ulcerative lesions with exudate and erythema extending diffusely along the base of tongue, oropharynx, nasopharynx, hypopharynx, and epiglottis (Figure 1)"], "s1": [0, 1, 6, 8], "s2": [2, 3, 4, 5, 7]} {"key": 675, "questions": "What Is Your Diagnosis?", "options": [{"label": "A", "disease": "Posttransplant lymphoproliferative disorder"}, {"label": "B", "disease": "Squamous cell carcinoma"}, {"label": "C", "disease": "Nocardiosis"}, {"label": "D", "disease": "Aspergillosis"}], "answer_idx": "D", "symptoms": ["A woman in her 60s with a history of end-stage renal disease who had a kidney transplant 9 months previously presented with progressive cough, dyspnea, and hoarseness", "Her history also included chronic gastroesophageal reflux disease and a 14-pack-year smoking history", "She initially presented 3 months prior with upper respiratory symptoms, which continued to worsen despite oral antibiotic therapy", "Her cough became productive with white, frothy phlegm that was accompanied by fever, wheezing, hoarseness, and fatigue", "Her dyspnea worsened in the supine position", "Flexible laryngotracheoscopy results revealed diffuse, white-to-yellow pseudomembranous changes involving the glottis and supraglottis (Figure, A), posterior tracheal wall, and left mainstem bronchus with skip lesions in the subglottis and distal 1 cm of the trachea (Figure, B)", "Glottic mobility was limited in both abduction and adduction with substantial airway obstruction", "Computed tomographic findings revealed posterior tracheal wall thickening with the loss of the fat plane between the trachea and esophagus, and a moderate to severe obstruction within the distal left mainstem bronchus", "Biopsy results are shown in Figure, C", "Laryngoscopic images demonstrate pseudomembranous plaques lining supraglottis and glottis (A) and posterior wall of the trachea (B)", "Supraglottic biopsy demonstrates branching organisms superficial to squamous epithelium (original magnification ×400) (C)"], "s1": [0, 1, 2, 3, 4], "s2": [5, 6, 7, 8, 9, 10]} {"key": 676, "questions": "What Is Your Diagnosis?", "options": [{"label": "A", "disease": "Low-grade sinonasal adenocarcinoma"}, {"label": "B", "disease": "Inflammatory nasal polyps"}, {"label": "C", "disease": "Inverted papilloma"}, {"label": "D", "disease": "Nasal cavity hamartoma"}], "answer_idx": "D", "symptoms": ["An 80-year-old man with acromegaly due to a hypoenhancing pituitary microadenoma opted for transsphenoidal resection", "Preoperative magnetic resonance imaging examination of the sella revealed a left-sided pituitary microadenoma (Figure, A) and lobulated polypoid soft tissue masses in the superior nasal cavities arising from the olfactory clefts", "These lesions demonstrated avid post–contrast enhancement with small areas of cystic change and heterogeneously hyperintense T2 signal", "There was mild widening of the olfactory clefts, particularly on the left side (Figure, B)", "The intervening nasal septum was intact, and there was no intracranial extension", "The patient did not have any noteworthy rhinological symptoms on review", "He later underwent nasal endoscopy, and results showed tan-colored polypoid lesions emanating from the olfactory clefts of both nasal cavities (Figure, C)", "Biopsy findings revealed submucosal proliferation of seromucinous and respiratory epithelial glands (Figure, D)", "A, An incidental polypoid-enhancing mass in the nasal cavity demonstrating avid post–contrast enhancement (yellow arrowheads) was observed on the magnetic resonance imaging examination", "The white arrowhead points to the pituitary adenoma", "B, Widening and remodeling of the olfactory clefts (yellow arrowheads) was also observed on the magnetic resonance imaging examination", "C, The incidental polypoid-enhancing mass in the nasal cavity (yellow arrowheads) as seen on nasal endoscopic analysis", "D, Results of the nasal cavity mass biopsy (original magnification ×100) revealed submucosal proliferation of bland-appearing seromucinous and respiratory epithelial glands"], "s1": [0, 9], "s2": [1, 2, 3, 4, 5, 6, 7, 8, 10, 11, 12]} {"key": 677, "questions": "What Is Your Diagnosis?", "options": [{"label": "A", "disease": "Cutaneous T-cell lymphoma (mycosis fungoides [MF])"}, {"label": "B", "disease": "Scleromyxedema"}, {"label": "C", "disease": "Lepromatous leprosy"}, {"label": "D", "disease": "Scleroderma"}], "answer_idx": "B", "symptoms": ["A man in his 50s presented with dysphagia, arthritis, joint contractures, carpal tunnel syndrome, and infiltrated and redundant skin with sheets of 1- to 2-mm-wide waxy, firm, skin-colored papules", "The patient first noticed papules on his forehead 1 year prior to presentation that spread to his face, trunk, and extremities", "He had mild dysphagia and severe joint pain in his hands and wrists, with carpal tunnel of his right wrist", "Additionally, his daily activities were impaired owing to decreased mobility", "During clinical evaluation, he denied cough, shortness of breath, chest pain, Raynaud phenomenon, fevers, night sweats, weight loss, seizures, or confusion", "Physical examination revealed diffuse skin-colored to pink papules coalescing into plaques on the entire face, with exaggerated skin folds of the forehead (Figure, A), glabella, periocular regions, and superior cutaneous lip", "This was accompanied by swelling and erythema of both ears", "On his extremities and trunk, he had diffuse infiltration of the skin, with exaggerated skin folds (Figure, B), decreased finger flexion, and four-fifths grip strength", "His nailfold capillaries were normal", "Skin sensation was intact", "A, Clinical image of the exaggerated skin folds of the forehead and glabella", "B, Clinical image of the wrist and hand depicting the exaggerated skin folds and decreased finger flexion", "C, Hematoxylin-eosin stain image from the punch biopsy featuring fibroblastic proliferation from the papillary to the reticular dermis (original magnification ×40)", "D, Hematoxylin-eosin and Alcian blue stain image featuring mucin deposition in the reticular dermis", "Notable laboratory results included a normal level of thyroid-stimulating hormone, macrocytic anemia, normal creatinine and calcium levels, and a monoclonal spike on a serum protein electrophoresis test", "Serum free light chains revealed normal κ free light chains, λ free light chains, and a normal κ/λ ratio", "Immunofixation results revealed IgG λ monoclonal gammopathy", "A punch biopsy was obtained for diagnostic clarification (Figure, C and D)"], "s1": [0, 1, 2, 3, 5, 6, 7, 8, 9], "s2": [4, 10, 11, 12, 13, 14, 15, 16, 17]} {"key": 678, "questions": "What Is Your Diagnosis?", "options": [{"label": "A", "disease": "Stucco keratoses"}, {"label": "B", "disease": "Hyperkeratosis lenticularis perstans"}, {"label": "C", "disease": "Disseminated superficial actinic porokeratosis"}, {"label": "D", "disease": "Kyrle disease"}], "answer_idx": "B", "symptoms": ["A man in his 60s presented with a 10-year history of a slowly progressive, asymptomatic cutaneous eruption on his left leg (Figure 1A)", "He had a 16-year history of diabetes mellitus that was being treated with metformin", "Hematological and biochemical test results were unremarkable", "There was no history of another endocrine disorder or malignant neoplasm", "There was no family history of similar cutaneous findings", "Physical examination revealed reddish brown hyperkeratotic papules 1 to 10 mm wide on the front and back of the left leg from knee to ankle", "The rest of the physical examination findings were unremarkable", "Removal of the scales caused slight bleeding", "A lesional skin biopsy was performed (Figure 1B and C)", "A, Clinical image of the reddish brown hyperkeratotic papules on the front of the left leg", "B and C, Hematoxylin-eosin–stained lesional skin biopsy specimens revealing compact hyperkeratosis, epidermal atrophy, and bandlike lymphocyte infiltrate in the superficial dermis"], "s1": [0, 5, 7, 9, 10], "s2": [1, 2, 3, 4, 6, 8]} {"key": 679, "questions": "What Is Your Diagnosis?", "options": [{"label": "A", "disease": "Myoclonus"}, {"label": "B", "disease": "Paroxysmal dyskinesia"}, {"label": "C", "disease": "Faciobrachial dystonic seizures"}, {"label": "D", "disease": "Tic disorder"}], "answer_idx": "C", "symptoms": ["A man in his 60s was evaluated in the epilepsy monitoring unit for various spells he had been having up to 5 times a day over the previous 8 months", "He described transient episodes of expressive speech difficulties, paroxysmal dizziness, and involuntary nonsuppressible jerks of the right arm, which occurred without warning and were associated with mild disorientation", "Video electroencephalography (EEG) captured the movements (Figure, A)", "They consisted of involuntary, synchronous contractions of the right face, arm, and leg that were sustained for a few seconds before muscle relaxation occurred (Figure, B and C", "Video)", "The movements were not elicited by action or exertion", "There was no epileptic abnormality that correlated with the movements on EEG, and head magnetic resonance imaging was unremarkable", "A, Electroencephalogram (EEG) during the event (bipolar montage, 60-Hz filter, 7-uV/mm sensitivity)", "B, Patient position prior to the event", "C, Patient position while experiencing an involuntary, sustained contraction of the right lower face, arm, and leg lasting almost 2 seconds before spontaneous resolution", "The event took place during the time span marked with red vertical lines on the EEG and did not show a clear change from baseline rhythm", "Muscle artifacts are seen prior to and during the event"], "s1": [0, 1, 3, 5, 9], "s2": [2, 4, 6, 7, 8, 10, 11]} {"key": 680, "questions": "What Is Your Diagnosis?", "options": [{"label": "A", "disease": "Bleomycin toxic effects"}, {"label": "B", "disease": "Paraneoplastic skin disease"}, {"label": "C", "disease": "Polymorphic eruption of pregnancy"}, {"label": "D", "disease": "Vinblastine toxic effects"}], "answer_idx": "A", "symptoms": ["A 31-year-old gravida 1, para 1 woman with a history of Wolff-Parkinson-White disease and recently diagnosed stage II Hodgkin lymphoma presented with a pruritic rash", "She was diagnosed with Hodgkin lymphoma at 35 weeks’ gestation, and 1 week later she underwent cesarean delivery", "At 3 weeks’ postpartum, she began chemotherapy (cycle 1, day 1) with adriamycin, bleomycin, vinblastine, and dacarbazine (ABVD)", "On day 15 of cycle 1, she presented with linear, hyperpigmented streaks and welts on her arms, back, and legs", "Intense, diffuse pruritus had developed within 24 hours of the patient’s first day of treatment, and her symptoms were poorly responsive to antihistamines and topical steroids", "Pruritus was followed by the appearance of dark scratch marks as demonstrated in the Figure", "Laboratory test results revealed a white blood cell count of 1.6 ×103 cells/mm3, with an absolute neutrophil count of 500 cells/mm3 and a total bilirubin of 0.3 mg/dL (to convert to μmol/L, multiply by 17.104), which was unchanged from the baseline total bilirubin", "Clinical photograph of the patient’s back shows scratch marks that appeared after diffuse pruritus", "Prior to the diagnosis of Hodgkin lymphoma, she was receiving routine prenatal care, and the only complication was pruritus without a rash that developed during the first trimester", "She was evaluated for right supraclavicular adenopathy that was unresponsive to antibiotics during the third trimester", "Findings from a lymph node biopsy were consistent with classic Hodgkin lymphoma, nodular sclerosis subtype", "Staging imaging revealed bilateral supraclavicular jugular chain, paratracheal and mediastinal lymphadenopathy, and the erythrocyte sedimentation rate was 67 mm/h", "She was diagnosed with stage II, unfavorable-risk Hodgkin lymphoma"], "s1": [0, 3, 4, 5, 7], "s2": [1, 2, 6, 8, 9, 10, 11, 12]} {"key": 681, "questions": "What Is Your Diagnosis?", "options": [{"label": "A", "disease": "Kaposi sarcoma"}, {"label": "B", "disease": "Diffuse large B-cell lymphoma"}, {"label": "C", "disease": "Burkitt lymphoma"}, {"label": "D", "disease": "Leukemia cutis"}], "answer_idx": "C", "symptoms": ["A man in his 30s presented with painless swelling of the left upper limb and nodules on the left side of the chest and abdomen", "The lesions had started a month previously as red nodules on the left axilla, ipsilateral chest, and back", "Twenty days later, he developed edema in the left upper limb", "While being evaluated for the cutaneous lesions, the patient was diagnosed with AIDS and began antiretroviral therapy", "On examination, 6 erythematous, firm, subcutaneous nodules were present on the left side of the chest, abdomen, and left upper limb, varying in size from 2 × 2 cm to 4 × 4 cm (Figure, A)", "There was diffuse erythema and edema on the left upper limb, leading to difficulty in limb movement (Figure, A)", "Multiple dilated superficial vessels were present on the posterior aspect of the left arm and ipsilateral chest wall", "Left axillary lymph nodes were grossly enlarged, firm, nontender, and nonmatted, with the largest lymph node measuring 6 × 4 cm", "The patient’s CD4 cell count was 141/μL (to convert to ×109/L, multiply by 0.001), and his HIV load was 504 473 copies/mL", "A skin biopsy sample was obtained from the plaque and sent for histopathologic examination (Figure, B and C)", "A, Multiple erythematous indurated plaques and nodules confined to the left side of the chest and abdomen with axillary lymphadenopathy and upper limb edema", "B, Skin biopsy sample showing infiltration with lymphoid cells and few histiocytes in the subcutaneous tissue (hematoxylin-eosin, original magnification ×400)", "C, On immunohistochemical analysis, these cells showed nuclear positivity for c-Myc (c-Myc, original magnification ×400)"], "s1": [3, 8], "s2": [0, 1, 2, 4, 5, 6, 7, 9, 10, 11, 12]} {"key": 682, "questions": "What Is Your Diagnosis?", "options": [{"label": "A", "disease": "Simple ranula"}, {"label": "B", "disease": "Epidermoid cyst"}, {"label": "C", "disease": "Vascular malformation"}, {"label": "D", "disease": "Odontogenic abscess"}], "answer_idx": "B", "symptoms": ["A woman in her 30s presented with a 1-year medical history of a painless, slowly enlarging, midline neck mass without associated dysphagia, odynophagia, weight loss, or history of infection", "The mass was soft, mobile, and compressible in midline level IA, measuring 7 cm and posterior-superiorly displacing the tongue", "The mass was visible as a clear blue lesion in the floor of the mouth and was nontender", "Computed tomographic (CT) scan of the neck with IV contrast was performed (Figure)", "The patient was taken to the operating room for a successful combined intraoral and transcervical approach to excision", "Computed tomographic images", "A, Coronal view of midline neck mass", "B, Sagittal view of midline neck mass"], "s1": [0, 1, 2], "s2": [3, 4, 5, 6, 7]} {"key": 683, "questions": "What Is Your Diagnosis?", "options": [{"label": "A", "disease": "Granulomatosis with polyangiitis"}, {"label": "B", "disease": "Cocaine-induced midline destructive lesion"}, {"label": "C", "disease": "Extranodal natural killer/T-cell lymphoma, nasal type"}, {"label": "D", "disease": "Chronic invasive fungal sinusitis"}], "answer_idx": "B", "symptoms": ["A 36-year-old otherwise healthy white woman with a history of chronic sinusitis presented with progressive left facial pain and swelling of 5 months’ duration", "Associated symptoms included epistaxis and left-sided epiphora", "Prior to presentation, she had been treated as an outpatient with multiple rounds of oral antibiotics and steroids for presumed sinusitis without improvement", "Social history included routine cocaine use 10 years ago", "Physical examination demonstrated substantial erythema and edema of the left infraorbital region and left nasal sidewall and purulent nasal secretions (Figure 1A)", "Endoscopic nasal examination revealed substantial swelling of the left nasal vestibule completely obstructing the left naris, swelling of the left nasolacrimal duct orifice, nearly total septal perforation with areas of necrotic bone, and no identifiable intranasal landmarks except for a remnant of the left middle turbinate (Figure 1B)", "There was no cervical lymphadenopathy", "Laboratory evaluation revealed a white blood cell count of 12 300/μL with normal neutrophil count", "erythrocyte sedimentation rate of 83 mm/h", "and a C-reactive protein level of 134 mg/L", "(To convert white blood cells to ×109/L, multiply by 0.001", "C-reactive protein to nanomoles per liter, multiply by 9.524.) A maxillofacial computed tomographic (CT) scan demonstrated cartilaginous and anterior osseous nasal septal perforation and left nasal and preseptal soft-tissue thickening with associated periosteal reaction of the nasal process of the maxilla", "A, Periorbital and nasal cellulitis", "B, Nasal endoscopic examination demonstrating significant soft-tissue necrosis"], "s1": [0, 1, 2, 4, 5, 12, 13], "s2": [3, 6, 7, 8, 9, 10, 11]} {"key": 684, "questions": "What Is Your Diagnosis?", "options": [{"label": "A", "disease": "Famciclovir-resistant disseminated herpes zoster"}, {"label": "B", "disease": "Bullous Sweet syndrome"}, {"label": "C", "disease": "Bullous Wells syndrome"}, {"label": "D", "disease": "Bullous pemphigoid"}], "answer_idx": "C", "symptoms": ["A man in his 40s was admitted to the hospital with a 10-day history of edema and presented with vesicles and bullae on his right cheek, neck, back, both eyelids, and the dorsal surface of his hands (Figure 1A)", "Prior to presentation, he had been treated for herpes zoster with famciclovir for 1 week, during which the lesions kept developing and were accompanied with irregular fever (maximum temperature, 39°C)", "Physical examination revealed edema on both eyelids", "multiple papules, blisters, and crusted erosions with thin exudation on his right cheek and neck", "and erythematous plaques and tense blisters and bullae on his back and on the dorsal surface of both hands (Figure 1B)", "A full laboratory workup was performed for autoantibodies for systemic lupus erythematosus (SLE), pemphigus, and bullous pemphigoid", "skin and bone-marrow biopsies and direct immunofluorescence were performed", "and immunohistochemichal analysis, a swab of exudation, and bacterial, fungal, and atypical mycobacterial cultures from blood and tissue were also examined", "A, Edema on both eyelids", "multiple papules, blisters, and crusted erosions with thin exudation on right cheek and neck", "B, Erythematous plaques, tense blisters, and bullae on both hands (right hand depicted)"], "s1": [0, 1, 2, 3, 4, 8, 9, 10], "s2": [5, 6, 7]} {"key": 685, "questions": "What Is Your Diagnosis?", "options": [{"label": "A", "disease": "Cutaneous angiosarcoma"}, {"label": "B", "disease": "Radiation dermatitis"}, {"label": "C", "disease": "Wolf isotopic response"}, {"label": "D", "disease": "Telangiectatic metastatic breast carcinoma"}], "answer_idx": "D", "symptoms": ["A woman in her 60s presented to the clinic with a 3-month history of pain, swelling, and erythema on her face", "Eleven years earlier, she had been diagnosed with left breast carcinoma and had undergone a mastectomy with subsequent radiotherapy and chemotherapy", "Seven years later, she had been found to have left supraclavicular lymph nodes metastasis and had undergone a surgical excision", "On presentation for the current problem, the patient experienced pain on palpation, but denied fever, chills, or other symptoms", "A physical examination revealed violaceous swelling with telangiectasia and necrosis on the forehead, eyelids, nose, and cheek, predominantly involving the left side of the face (Figure 1)", "A biopsy of the lesion was performed for histopathological analysis (Figure 2)", "A, A full image and B, a three-quarters image of the face, showing telangiectatic, swelling erythema mainly involving the left side of the forehead, eyelids, nose, and cheek"], "s1": [1, 2], "s2": [0, 3, 4, 5, 6]} {"key": 686, "questions": "What Is Your Diagnosis?", "options": [{"label": "A", "disease": "Reactive granulomatous disorder"}, {"label": "B", "disease": "Posttransplantation lymphoproliferative disorder"}, {"label": "C", "disease": "Deep fungal infection"}, {"label": "D", "disease": "Cutaneous T-cell lymphoma"}], "answer_idx": "B", "symptoms": ["A man in his 50s presented with an abdominal skin eruption of 2 days’ duration", "His medical history included hepatitis C, cirrhosis, and an orthotopic liver transplant (OLT) 3 years prior that was subsequently treated with tacrolimus, 5 mg, and mycophenolate, 750 mg, twice daily", "Approximately 2½ years after the transplant, the patient developed gastric outlet obstruction secondary to an infiltrative gastric wall mass", "The gastric wall mass was found to be associated with plasmablastic posttransplantation lymphoproliferative disorder (pPTLD)", "He was subsequently instructed to stop use of mycophenolate and decrease use of tacrolimus to 0.25 mg daily, then treated with 1 cycle of CHOP chemotherapy (cyclophosphamide, doxorubicin hydrochloride, vincristine sulfate, and prednisone) and lenalidomide, resulting in symptomatic and radiologic improvement", "Five weeks after treatment with CHOP chemotherapy, the patient presented with an acute-onset abdominal skin eruption without fever or pain", "A computed tomographic scan of the abdomen and pelvis showed possible abdominal wall cellulitis, so treatment with piperacillin-tazobactam and vancomycin was empirically started", "Physical examination revealed confluent indurated violaceous papules and plaques coalescing into an annular pattern encircling the healed OLT incisional scar with central sparing (Figure 1A)", "A 4-mm punch biopsy specimen was obtained for histopathological evaluation (Figure 1B and C) and bacterial and fungal cultures", "A, Confluent, indurated, violaceous papules and plaques coalescing into an annular pattern encircling the well-healed orthotopic liver transplant incisional scar", "B, Hematoxylin-eosin staining of the punch biopsy shows an atypical, hyperchromatic infiltrate present throughout the superficial and deep dermis", "C, High magnification reveals crushed but markedly atypical-appearing lymphoid cells with nuclear hyperchromasia and scant amounts of cytoplasm", "Multiple mitotic and apoptotic figures are easily identified"], "s1": [0, 5, 7, 8, 9, 10, 11, 12], "s2": [1, 2, 3, 4, 6]} {"key": 687, "questions": "What Is Your Diagnosis?", "options": [{"label": "A", "disease": "Mitochondrial encephalopathy, lactic acidosis, and strokelike episodes"}, {"label": "B", "disease": "Metachromatic leukodystrophy"}, {"label": "C", "disease": "X-linked Charcot-Marie-Tooth disease"}, {"label": "D", "disease": "Pediatric-onset multiple sclerosis"}], "answer_idx": "C", "symptoms": ["A 13-year-old boy with a longstanding history of gait imbalance presented with 2 episodes of acute-onset left hemibody weakness and dysarthria without changes in sensorium in a 24-hour period", "Symptoms lasted approximately 60 minutes each before completely resolving", "prior to the day of the events, he had never experienced similar phenomena", "Initial examination following the second event revealed mild-to-moderate dysarthria, pes planovalgus and tight achilles tendons bilaterally, and decreased vibratory and fine touch sensation from the great toe up to the mid shin bilaterally", "Reflexes were absent at the patella and achilles bilaterally", "Bilateral dorsiflexion weakness was present (4 of 5)", "There were otherwise no motor deficits, muscle atrophy, or lateralizing neurologic abnormalities", "On arrival, the patient underwent a stroke/transient ischemic attack workup", "Results of a comprehensive metabolic panel, complete blood cell count, erythrocyte sedimentation rate, C-reactive protein, and a coagulation panel were within normal limits", "Computed tomography angiogram demonstrated no evidence of extra or intracranial dissection", "Magnetic resonance imaging of the brain with and without contrast (Figure) demonstrated symmetric, bilateral, restricted diffusion, with T2 hyperintensities in the supratentorial white matter and corpus callosum", "Lumbar puncture was obtained and revealed normal cell counts and no oligoclonal bands or evidence of infection", "A, Diffusion-weighted imaging sequence demonstrating bilateral, symmetric, restricted diffusion of the supratentorial white matter (corresponding hypointensity noted on apparent diffusion coefficient sequences)", "B, T2 sequence demonstrating bilateral and symmetric increased signal in the supratenorial white matter–sparing U-fibers", "The patient had reportedly always walked on the outside of his feet and had been considered clumsy when compared with other children but had no history of motor delays", "Family history revealed a mother with diminished reflexes, hand tremor, and balance issues with a prior inconclusive electromyogram and nerve conduction study", "A younger brother with attention-deficit/hyperactivity disorder also had similar gait abnormalities"], "s1": [0, 1, 2, 7, 8, 9, 10, 11, 12, 13], "s2": [3, 4, 5, 6, 14, 15, 16]} {"key": 688, "questions": "What Is Your Diagnosis?", "options": [{"label": "A", "disease": "Angiosarcoma"}, {"label": "B", "disease": "Leukemia cutis"}, {"label": "C", "disease": "Squamous cell carcinoma"}, {"label": "D", "disease": "Basal cell carcinoma"}], "answer_idx": "D", "symptoms": ["A white man in his 20s presented with numerous pink and brown papules and plaques, some with ulceration, on the head and along the midline of the neck and spine (Figure)", "Medical history included medulloblastoma treated with irradiation of the brain and spine at age 2 years", "He also received radiation therapy for a meningioma of the posterior fossa in adulthood", "He was also treated for several seemingly similar lesions on the face and scalp and had a family history of similar cutaneous lesions reported in 2 brothers and 1 niece", "Figures show crusted erythematous papules and plaques adjacent to the surgical excision scar within a prior irradiation field on the head and neck (A) and erythematous and hyperpigmented papules and plaques overlying diffuse postsurgical scarring within a prior irradiation field on the back (B)"], "s1": [0, 4], "s2": [1, 2, 3]} {"key": 689, "questions": "What Is Your Diagnosis?", "options": [{"label": "A", "disease": "Disease progression"}, {"label": "B", "disease": "Nivolumab-induced extensive panniculitis"}, {"label": "C", "disease": "Cold panniculitis"}, {"label": "D", "disease": "Erythema nodosum under nivolumab"}], "answer_idx": "B", "symptoms": ["A 41-year-old woman with a history of Hodgkin disease in her adolescence was diagnosed with melanoma of the left upper thigh that was classified as American Joint Committee on Cancer (AJCC) stage IIA with (pT3aN0 [sn0/1] M0) BRAF mutation", "One year later she underwent inguinal lymphadenectomy for macrometastasis followed by adjuvant immunotherapy with ipilimumab, a monoclonal antibody that targets cytotoxic T-lymphocyte antigen 4. Owing to local disease progression, treatment escalation with combined immunotherapy (ipilimumab and nivolumab, an anti–PD-1 [programmed cell death 1]) antibody was initiated", "Apart from immunotherapy-induced thyroiditis, the treatment was well tolerated", "However, after 11 cycles of nivolumab, the patient presented with progressive painful reddening and swelling of the left thigh", "There was no deterioration of the general condition, no fever, no change in current medication, and no recent cold exposure", "Clinical examination revealed diffuse erythematous induration of the upper thigh expanding to the lower left abdominal area, as well as multiple small palpable subcutaneous nonindurated nodules on the ipsilateral gluteal region and leg (Figure, A)", "Fluorine 18 (18F) fluorodeoxyglucose (FDG) positron emission tomography–computed tomography (PET-CT) detected multiple FDG-active lesions in both lower extremities and the lumbar region", "a previously described left iliac lymphadenopathy was partially regressed (Figure, B)", "A diagnostic biopsy specimen of the left thigh revealed a lobular lymphocytic panniculitis with dense infiltrate of mainly lymphocytes and plasma cells with no signs of vasculitis (Figure, C and D)", "Direct immunofluorescence and blood analysis showed an absence of specific autoimmune antigens (test results for α-1 antitrypsin, antinuclear antibody, anti-Sjögren syndrome–related antigen, and anti-Sjögren syndrome–related antigen B were all unremarkable)", "A, Localized erythematous brownish induration seen on the upper thigh", "Arrowheads correlate with the most prominent clinical findings", "B, Radioactive fluorine 18 (18F) fluorodeoxyglucose (FDG) positron emission tomography–computed tomography (PET-CT) reveals multiple subcutaneous lesions in both lower extremities, the lumbar region, and on the ipsilateral lower leg with high FDG uptake highly suggestive of (in transit) metastasis", "C and D, Lesional skin biopsy samples obtained from the proximal left thigh show normal epidermis and dermis with patchy infiltrate of the deep subcutaneous fat (hematoxylin-eosin stain)", "Green arrows indicate the plasma cells, and blue arrows indicate the lymphocytes"], "s1": [0, 1, 2, 3, 5, 6, 8, 9, 12], "s2": [4, 7, 10, 11, 13, 14]} {"key": 690, "questions": "What Is Your Diagnosis?", "options": [{"label": "A", "disease": "Pott puffy tumor"}, {"label": "B", "disease": "Mucocele"}, {"label": "C", "disease": "Basal cell carcinoma"}, {"label": "D", "disease": "Sebaceous carcinoma"}], "answer_idx": "D", "symptoms": ["A white man in his 70s presented with a gradually enlarging right supraorbital mass that had been neglected for 4 years", "His comorbidities included congestive heart failure, atrial fibrillation, coronary artery disease, chronic obstructive pulmonary disease, hypertension, and a 50-pack-year history of tobacco abuse", "Physical examination revealed a firm, nonmobile, nontender, bulging mass situated over the right frontal sinus, which expanded to the upper right eyelid", "The mass covered the globe completely, resulting in total vision obstruction (Figure, A)", "However, the patient’s vision remained intact when the mass was elevated off the eye with great effort", "His right pupil was round and reactive to light without afferent pupillary defect", "No palpable lymphadenopathy was noted", "A non–contrast-enhanced computed tomography (CT) scan demonstrated a large, right supraorbital mass with bony destruction of the anterior table of the right frontal sinus (Figure, B)", "It extended into the superior aspect of the orbit, displacing the right globe inferiorly", "Air was seen within the mass (Figure, C)", "A fine-needle aspiration biopsy yielded inconclusive results", "After medical clearance, the patient was taken to the operating room for a wedge excisional biopsy with decompression of the right supraorbital mass under monitored anesthesia care", "Histopathological investigation revealed large nests of neoplastic epithelioid cells surrounded by fibrotic tissue and necroinflammatory debris", "The neoplastic cells contained round nuclei with single prominent nucleoli and abundant clear to vacuolated cytoplasm (Figure, D)", "A, Enlarged right supraorbital mass", "B, Computed tomography (CT) scan of destruction of the anterior table of right frontal sinus", "C, Air pocket within the mass", "D, Neoplastic epithelioid cells of the mass (original magnification ×40)"], "s1": [0, 1, 2, 3, 4, 5, 6], "s2": [7, 8, 9, 10, 11, 12, 13, 14, 15, 16, 17]} {"key": 691, "questions": "What Is Your Diagnosis?", "options": [{"label": "A", "disease": "Prostate cancer with brain metastasis"}, {"label": "B", "disease": "Tuberculosis"}, {"label": "C", "disease": "Toxoplasmosis"}, {"label": "D", "disease": "Schistosomiasis"}], "answer_idx": "D", "symptoms": ["A 46-year-old African swimmer presented with a 1-month history of progressive left hearing loss and vertigo", "He also reported asthenia, sleep disorders, and urinary frequency", "The urinary frequency has been treated for 2 years with many drugs (tamsulosin, desmopressin, and solifenacin) without improvement", "The patient had spent his childhood in eastern Africa, where he swam in Lake Tanganyika", "His history did not reveal trauma, infection, or any other associated conditions at the onset of the complaints", "The ear, nose, and throat examination revealed a right horizontal nystagmus and a poorly delineated hypoesthesia of the left pinna", "He could not walk properly owing to his vertigo", "The rest of the clinical examination findings were unremarkable", "Additional examinations (ie, pure-tone audiometry, video nystagmography, tympanometry) revealed left profound hearing loss with an air-bone gap of 40 dB and central vertigo", "Otoacoustic emissions were absent on the left side", "There was no left acoustic reflex", "Auditory brainstem responses showed asymmetric interpeak latencies of waves I through V and III through V, suggesting slower conduction on the left side", "Computed tomography and cerebral magnetic resonance imaging (MRI) revealed a 24 × 20-mm hemorrhagic lesion in the left middle cerebellar peduncle associated with a thickening of the cochleovestibular nerve (Figure)", "The patient was referred to urology, where a 13 × 8-mm mass was discovered in the lateral wall of the prostate gland", "Biopsies of prostate and brain lesions, blood tests, and urinalysis were performed", "Computed tomography (CT) scan and magnetic resonance images (MRIs) of the left middle cerebellopontine angle and prostate gland", "A, Axial CT scan showed a hyperdense lesion suggesting hemorrhage in the left middle cerebellar peduncle", "B, The lesion extended to the base of the vestibulocochlear nerve that was thickened (arrowheads) on the Fast Imaging Employing Steady-State Acquisition (FIESTA) MRI (axial plane)", "C, Coronal postcontrast MRI demonstrated a T1 hyperintense lesion that was consistent with hemorrhagic lesion with no true enhancement", "D, Axial pelvic MRI showed a prostatic lesion (13 × 4 mm) located in the left posterior and lateral wall (arrowhead)", "the prostatic volume and size were substantially increased (132 mL, 59 × 60 × 70 mm)"], "s1": [0, 3, 5, 6, 8, 9, 10, 11, 12, 16, 17, 18], "s2": [1, 2, 4, 7, 13, 14, 15, 19, 20]} {"key": 692, "questions": "What Is Your Diagnosis?", "options": [{"label": "A", "disease": "Fixed cutaneous sporotrichosis"}, {"label": "B", "disease": "Lupus vulgaris"}, {"label": "C", "disease": "Atypical mycobacterial infection"}, {"label": "D", "disease": "Cutaneous leishmaniasis"}], "answer_idx": "B", "symptoms": ["A preschool boy presented with asymptomatic, reddish, raised lesions over his right ear, noticed 3 months earlier, associated with multiple painless swellings on the ipsilateral side of the neck for the past 3 weeks", "On examination, there were multiple, erythematous papules coalescing to form a plaque with areas of scarring over the posterior aspect of right ear, along with firm, nontender posterior cervical lymphadenopathy (Figure, A)", "There was no history of trauma or intervention prior to the onset of lesions", "His medical history was unremarkable, as were the rest of the mucocutaneous and systemic examination findings", "A skin biopsy from the plaque was submitted for histopathological examination (Figure, C and D)", "A, Erythematous papules coalescing to form a plaque with areas of scarring, along with ipsilateral posterior cervical lymphadenopathy", "B, The lesions have healed with atrophic scarring after 6 weeks of treatment", "C, Microphotograph showing unremarkable epidermis with evidence of granulomatous inflammation and central caseous necrosis in the dermis (original magnification ×100)", "D, Ziehl-Neelsen stain shows acid-fast bacilli (oil immersion", "original magnification ×1000)"], "s1": [0, 1, 2, 3, 5, 6], "s2": [4, 7, 8, 9]} {"key": 693, "questions": "What Is Your Diagnosis?", "options": [{"label": "A", "disease": "Granulomatous cheilitis"}, {"label": "B", "disease": "Leukemia labialis"}, {"label": "C", "disease": "Lip licker’s dermatitis"}, {"label": "D", "disease": "Pyostomatitis vegetans"}], "answer_idx": "B", "symptoms": ["A man in his 60s presented with an approximately 4-month medical history of lip enlargement with nodular growths", "Skin examination was significant for multiple erythematous and flesh-colored papules and nodules symmetrically affecting the upper and lower lips with extension onto the vermilion border (Figure 1A)", "There were no intraoral lesions, and all other findings of his examination were within normal limits", "The patient denied fevers, chills, weight loss, night sweats, cough, lymphadenopathy, abdominal pain, change in bowel habits, or other systemic symptoms", "A biopsy was obtained of the vermilion lip (Figure 1B)", "A, Symmetric lip enlargement with infiltrative erythematous and skin-colored papules and nodules extending onto the vermilion border", "B, Hematoxylin-eosin stain (original magnification ×40)"], "s1": [0, 1, 5], "s2": [2, 3, 4, 6]} {"key": 694, "questions": "What Is Your Diagnosis?", "options": [{"label": "A", "disease": "Necrobiotic xanthogranuloma"}, {"label": "B", "disease": "Xanthelasma palpebrum"}, {"label": "C", "disease": "Erdheim-Chester disease"}, {"label": "D", "disease": "Langerhans cell histiocytosis"}], "answer_idx": "C", "symptoms": ["A woman in her 50s with a right lower-eyelid yellow plaque presented with unremitting abdominal and back pain", "Computed tomographic (CT) scan of the chest, abdomen, and pelvis demonstrated extensive infiltrative stranding from the thorax through the retroperitoneal space", "Sclerotic changes in multiple osseous structures were noted", "Idiopathic retroperitoneal fibrosis was diagnosed following biopsies", "Prednisone and azathioprine were started", "Biopsy of the lower-eyelid yellow plaque returned xanthelasma (Figure, A)", "Ureteral obstruction, chronic kidney disease, and lymphedema complicated her course", "Azathioprine was stopped after 1 year", "Prednisone was slowly tapered but she relapsed 3 years later", "A second retroperitoneal biopsy exhibited fibroadipose tissue, foamy macrophages, and chronic inflammation with fibrosis", "Azathioprine and prednisone were restarted", "She required multiple surgeries for spinal stenosis and ureteral obstruction", "Her kidney function continued declining", "Then, owing to knee pain, a plain radiograph was obtained which demonstrated osteosclerosis of the distal femur", "Consequently, the previous eyelid and retroperitoneal biopsies were reviewed", "The clinicopathologic presentation was confirmed with a molecular test", "A, Clinical image of a soft, nonscaly yellow-tan plaque on the medial canthus", "B, Hematoxylin-eosin stain image demonstrating foamy histiocytes with Touton giant cells (original magnification ×100)", "C, CD68 immunohistochemical stain diffusely positive in the histiocytic infiltrate (original magnification ×100)", "D, Computed tomographic image with contrast demonstrating retroperitoneal stranding and fibrosis with hydronephrosis (arrowheads indicate hairy kidney appearance of fibrosis)"], "s1": [0, 5, 6, 8, 9, 13, 14, 16, 17, 18], "s2": [1, 2, 3, 4, 7, 10, 11, 12, 15, 19]} {"key": 695, "questions": "What Is Your Diagnosis?", "options": [{"label": "A", "disease": "Folliculotropic mycosis fungoides"}, {"label": "B", "disease": "Papulopustular rosacea"}, {"label": "C", "disease": "Eosinophilic pustular folliculitis"}, {"label": "D", "disease": "Majocchi granuloma"}], "answer_idx": "C", "symptoms": ["A Japanese man in his 50s presented with a 6-month history of worsening pruritic hand dermatitis that progressed to involve his face, trunk, upper extremities, and feet", "He had emigrated from Japan more than 10 years earlier but had no history of recent travel", "He took no medications, had no significant medical or surgical history, and denied personal and family history of atopy", "While he worked as a biomedical researcher, he denied any caustic exposures", "On physical examination, his face, trunk, upper extremities, palms, and soles were marked by 1- to 2-mm erythematous papules and pustules coalescing into large, partially indurated and polycyclic plaques with centrifugal extension and occasional central clearing (Figure, A and B)", "Treatment with potent topical corticosteroids was ineffective", "Punch biopsies of lesions on the shoulder (Figure, C) and foot (Figure, D) were performed", "A, Annular erythematous plaque studded with papules and pustules on the cheek and temple", "B, Erythematous plaques studded with papules and pustules on the shoulder", "C and D, Lesional specimens under hematoxylin-eosin stain taken from the shoulder (C, original magnification ×400) and plantar foot (D, original magnification ×40)"], "s1": [0, 1, 2, 3, 5], "s2": [4, 6, 7, 8, 9]} {"key": 696, "questions": "What Is Your Diagnosis?", "options": [{"label": "A", "disease": "Multiple sclerosis"}, {"label": "B", "disease": "Behçet disease"}, {"label": "C", "disease": "Susac syndrome"}, {"label": "D", "disease": "Cerebral autosomal dominant arteriopathy with subcortical infarcts and leukoencephalopathy (CADASIL)"}], "answer_idx": "C", "symptoms": ["A 42-year-old white woman with a history of episodic migraine with visual aura presented for evaluation of transient right upper extremity weakness and word-finding difficulty associated with headache", "She admitted to having developed new intermittent vertigo and more frequent and severe throbbing migraine with transient blurry vision and nonpulsatile tinnitus several months prior", "She had recently developed hearing loss with a robotic quality of auditory perception", "She denied mouth or genital ulcers", "A neurologic examination revealed diminished hearing to a finger rub on the right side", "Fundoscopy showed several segmental retinal arterial plaques, with the right side worse than the left", "A brain magnetic resonance image (MRI) with gadolinium revealed multiple white-matter lesions that were hyperintense on diffusion-weighted imaging (DWI), including some with enhancement, as well as a small DWI-negative lesion in the left thalamus and corpus callosum that was hyperintense on T2/fluid-attenuated inversion recovery magnetic resonance imaging (FLAIR", "Figure 1)", "An MRI of the spine with gadolinium did not reveal a signal abnormality in the spinal cord", "Infectious, rheumatologic, and hypercoaguable test results were negative", "A lumbar puncture revealed a white blood cell count of 8 cells/μL (normal range, 0-5 cells/μL", "to convert to cells × 109/L, multiply by 0.001) with 78% lymphocytes and 22% monocytes (to calculate these as proportions of 1.0, multiply by 0.01), and a protein level of 0.067 g/dL (normal range, 0.015-0.045 g/dL", "to convert to grams per liter, multiply by 10)", "Cerebrospinal fluid (CSF) glucose and IgG index test results were normal, with culture and oligoclonal bands negative", "An audiogram revealed bilateral sensorineural hearing loss, with the right side worse than the left", "Retinal fluorescein angiography was obtained", "A, Pink arrowheads indicate hyperintensities in the right centrum semiovale", "B, The yellow arrowhead indicates a hyperintensity in the corpus callosum", "Cerebral autosomal dominant arteriopathy with subcortical infarcts and leukoencephalopathy (CADASIL)"], "s1": [0, 1, 2, 3, 4, 14, 18], "s2": [5, 6, 7, 8, 9, 10, 11, 12, 13, 15, 16, 17]} {"key": 697, "questions": "What Is Your Diagnosis?", "options": [{"label": "A", "disease": "T-cell prolymphocytic leukemia"}, {"label": "B", "disease": "Advanced cutaneous T-cell lymphoma/Sézary syndrome"}, {"label": "C", "disease": "Polycythemia rubra vera"}, {"label": "D", "disease": "Cutaneous lupus erythematosus"}], "answer_idx": "A", "symptoms": ["A white woman in her 70s with advanced Alzheimer disease was referred to the hematology clinic for evaluation of a high hemoglobin level (169 g/L", "normal range, 120-160 g/L) and red blood cell count (5.67 × 1012/L", "normal range, 3.8-4.8 × 1012/L) as well as a generalized itch that was worse after a bath", "On examination, she had a florid, erythematous macular eruption over the trunk and limbs (Figure, A) but no hepatosplenomegaly or lymphadenopathy", "In addition to the high hemoglobin level, mild lymphocytosis (absolute lymphocyte count, 6.2 × 109/L, range 1.5-4.0 × 109/L) was noted with the lymphocyte morphology, suggesting reactive changes", "Skin biopsy specimen (Figure, B) showed a normal epidermis with a pericapillary infiltrate of small lymphocytes restricted to the dermis and no leukocytoclastic vasculitis, fungal organisms, or dermal mucin", "The absence of cellular atypia and epidermal involvement suggested a diagnosis of lupus, or gyrate or annular erythema, and the need for clinicopathological correlation", "A, Photograph shows erythroderma", "B, Skin biopsy specimen shows a pericapillary infiltrate of small lymphoid cells restricted to the dermis (hematoxylin-eosin, original magnification ×200)", "C, Blood smear shows a typical lymphocyte (May-Grünwald Giemsa stain, original magnification ×400)", "Treatment began with a topical emollient, steroid creams, and oral antihistamines", "Skin biopsy was repeated 2 months after the original procedure owing to a suboptimal clinical response, but the histological appearances were unchanged", "The hemoglobin level remained high, and the lymphocyte count had increased to 9.0 × 109/L (Figure, C)"], "s1": [0, 1, 2, 4, 12], "s2": [3, 5, 6, 7, 8, 9, 10, 11]} {"key": 698, "questions": "What Is Your Diagnosis?", "options": [{"label": "A", "disease": "Squamous cell carcinoma"}, {"label": "B", "disease": "Oral eosinophilic ulcer"}, {"label": "C", "disease": "Abscess"}, {"label": "D", "disease": "Pleomorphic rhabdomyosarcoma"}], "answer_idx": "D", "symptoms": ["A 53-year-old African American man with a history of bipolar disorder presented to the emergency department with an enlarging right-sided tongue mass that had caused mild discomfort and halitosis over the course of 1 week", "He denied recent injury, fever, chills, odynophagia, or difficulty breathing", "Laboratory examination was significant for leukocytosis (white blood cell count, 15 700/μL) and hypercalcemia (total calcium level, 13.1 mg/dL) (to convert white blood cells to ×109 per liter, multiply by 0.001", "to convert calcium to millimoles per liter, multiply by 0.25)", "Physical examination was notable for a 4- to 5-cm pedunculated, ulcerative mass on the dorsal oral aspect of the tongue near midline, anterior to the foramen cecum, without palpable cervical lymphadenopathy", "Contrast-enhanced computed tomography (CT) demonstrated a rim-enhancing, centrally hypodense lesion in the right dorsal tongue abutting the lingual septum with a somewhat ill-defined border (Figure 1)", "there was no associated osseous erosion of the mandible or hard palate, no tongue prolapse to suggest hypoglossal nerve palsy, no involvement of the soft palate, and no cervical lymphadenopathy by radiological size criteria", "An excisional biopsy of the anterior aspect of the mass was performed at bedside, and the patient was discharged home with instructions to follow up in the otolaryngology clinic", "A, Sagittal contrast-enhanced computed tomography (CT) demonstrates an exophytic mass on the dorsal aspect of the tongue", "B, Axial contrast-enhanced CT demonstrates a peripherally enhancing mass of the oral tongue, to the right of midline"], "s1": [0, 1, 2, 3], "s2": [4, 5, 6, 7, 8, 9]} {"key": 699, "questions": "What Is Your Diagnosis?", "options": [{"label": "A", "disease": "Albers-Schönberg disease"}, {"label": "B", "disease": "Gigantiform cementoma"}, {"label": "C", "disease": "Fibrous dysplasia"}, {"label": "D", "disease": "Polyostotic ossifying fibroma"}], "answer_idx": "B", "symptoms": ["A 17-year-old girl with a history of chronic sinusitis presented with left-sided epiphora, purulent drainage, and pain and right-sided jaw pain", "She had previously been treated with antibiotics and mupirocin nasal irrigation, with temporary improvement", "The patient and her identical twin sister had required multiple surgical procedures in the past for osseous lesions in the upper and lower jaws, including an aggressive resection via bilateral Caldwell-Luc approach 2 years previously", "a computed tomographic scan from that initial presentation is shown in the Figure, A", "At the current presentation, examination using flexible nasal endoscopy revealed bulging of the left lateral nasal wall", "imaging revealed a 14-mm, dense, expansile, sclerotic mass with moderate calcifications obstructing the left nasolacrimal duct", "a similar 2.3 × 1.8–cm lesion of the right mandibular body", "and evidence of previous sinus surgery (Figure, B)", "Because of these findings, left-sided endoscopic dacryocystorhinostomy and sinus surgery was performed", "Histologic findings are shown in the Figure, C and D", "A, Contrast-enhanced computed tomographic image at initial presentation", "B, Non–contrast-enhanced computed tomographic image at 2 years after bilateral Caldwell-Luc resection showing mass in left nasolacrimal duct", "C, Photomicrograph depicting a proliferation of highly cellular, fibrous connective tissue (arrow) and cementum-like hard tissue (arrowhead) (hematoxylin-eosin, original magnification ×100)", "D, Photomicrograph showing whorls of spindle cells (arrow) with relatively acellular calcified masses of cemento-osseous material (arrowhead) (hematoxylin-eosin, original magnification ×200)"], "s1": [0, 1, 3, 4, 5, 6, 7, 8, 10, 11], "s2": [2, 9, 12, 13]} {"key": 700, "questions": "What Is Your Diagnosis?", "options": [{"label": "A", "disease": "Multifocal basal cell adenoma"}, {"label": "B", "disease": "Basal cell adenocarcinoma"}, {"label": "C", "disease": "Multifocal canalicular adenoma"}, {"label": "D", "disease": "Adenoid cystic carcinoma"}], "answer_idx": "C", "symptoms": ["A woman in her 60s reported right upper lip submucosal masses that had been present for several weeks", "She denied associated pain, bleeding, or recent trauma to the lip", "She also denied any other constitutional symptoms", "Her social history was significant for a 45–pack-year smoking history and social drinking", "On physical examination, the patient had right upper lip submucosal masses without any tenderness or mucosal abnormality", "The remainder of the otolaryngic examination was unremarkable", "A computed tomographic scan of the neck with contrast demonstrated multiple hyperenhancing nodules", "the largest was 3 × 7 × 6 mm, with no local bony erosion or cervical lymphadenopathy (Figure 1)", "Computed tomographic (CT) scans of hyperenhancing nodules of the upper lip (red arrows)"], "s1": [0, 1, 2, 3, 4, 5], "s2": [6, 7, 8]} {"key": 701, "questions": "What Is Your Diagnosis?", "options": [{"label": "A", "disease": "Linear IgA bullous dermatosis"}, {"label": "B", "disease": "Cryothermic dermatitis artefacta"}, {"label": "C", "disease": "Herpes virus infection"}, {"label": "D", "disease": "Bullous pemphigoid"}], "answer_idx": "B", "symptoms": ["A woman in her 20s presented with almost equally sized, round erythematous plaques with central pallor and confluent blisters at the periphery on both forearms", "Older lesions showed central crusts", "According to the patient, the lesions recurrently appeared during the last 9 years, with a symptom-free period of 2 years (Figure, A)", "She could not identify or recall a specific cause for these skin lesions and denied self-inflicting them", "She had been treated with antibiotics (cefuroxime) as well as with topical steroids (dexamethasone) but denied any lasting effect", "Her family history was without abnormalities", "She reported being vaccinated against varicella zoster virus and was not immunocompromised", "A, A clinical photograph shows an annular erythematous plaque with crusts, bullae, and central pallor", "B, Histopathologic specimen shows normal stratum corneum, confluent necrosis of the upper epidermal layers, some multinucleated keratinocytes in the adjacent epidermis, and papillary edema with a subepidermal blister", "A moderately dense perivascular lymphocytic infiltrate with eosinophilic and neutrophilic granulocytes is present in the upper dermis (hematoxylin-eosin, original magnification x200)", "A skin biopsy of the forearm (Figure, B), direct immunofluorescence, and blood samples were obtained"], "s1": [0, 1, 3, 7, 8], "s2": [2, 4, 5, 6, 9, 10]} {"key": 702, "questions": "What Is Your Diagnosis?", "options": [{"label": "A", "disease": "Confluent and reticulated papillomatosis"}, {"label": "B", "disease": "Prurigo pigmentosa"}, {"label": "C", "disease": "Dowling-Degos disease"}, {"label": "D", "disease": "Lichen planus pigmentosus"}], "answer_idx": "A", "symptoms": ["An adolescent girl with polycystic ovarian syndrome (PCOS) presented for evaluation of hyperpigmented skin lesions affecting her neck, upper limbs, and trunk", "The lesions had started about 6 months previously as recurrent crops of pruritic erythematous papules that subsided leaving behind brownish-bluish black hyperpigmentation after each episode", "On examination, there were multiple, brown, hyperpigmented patches symmetrically affecting her neck, axillae, antecubital fossa, and intermammary and inframammary areas (Figure, A)", "The patches showed a reticulate pattern at the peripheries", "Intermammary patches had central erythema and a few subsiding papules", "Additionally, there were dark brown to bluish-gray patches with reticulation on the lower back and dorsum of the neck (Figure, B)", "These patches also had a few resolving erythematous papules in the periphery", "There was no history suggestive of allergic rhinitis or bronchial asthma", "Her IgE levels and thyroid status were normal", "Her fasting blood glucose and basal insulin levels were not raised", "There was no family history of similar lesions", "The rest of the skin, nails, and mucosae were all normal", "Potassium hydroxide mount analysis from skin scrapings did not reveal fungal hyphae or spores", "A skin biopsy was performed from lesions on the lower back, and the specimen was submitted for histopathologic analysis (Figure, C and D)", "A, Illustrated are multiple brown hyperpigmented patches symmetrically affecting the neck, axillae, antecubital fossa, and intermammary and inframammary areas", "The patches show a reticulate pattern at the peripheries", "The intermammary patches have central erythema and a few overlying edematous papules", "B, Bluish-gray pigmented patches with prominent reticulation on lower back", "C, Histopathologic lesional specimen shows mild epidermal hyperkeratosis, acanthosis, multiple foci of spongiosis, and an enhanced basal layer pigmentation", "There is scant lichenoid and moderate perivascular lymphomononuclear and polymorphonuclear infiltrate, basal cell vacuolization, and melanin incontinence (hematoxylin-eosin)", "D, Histopathologic lesional specimen at higher power reveals spongiosis, occasional neutrophilic exocytosis (yellow arrowheads), basal cell vacuolar degeneration, and necrotic keratinocytes (black arrowheads) (hematoxylin-eosin)"], "s1": [0, 1, 2, 3, 4, 5, 6, 14, 15, 16, 17], "s2": [7, 8, 9, 10, 11, 12, 13, 18, 19, 20]} {"key": 703, "questions": "What Is Your Diagnosis?", "options": [{"label": "A", "disease": "Systemic-onset juvenile idiopathic arthritis"}, {"label": "B", "disease": "Cystic fibrosis-associated episodic arthritis"}, {"label": "C", "disease": "Hypertrophic pulmonary osteoarthropathy"}, {"label": "D", "disease": "Serum sickness-like reaction"}], "answer_idx": "B", "symptoms": ["An elementary school-aged girl with cystic fibrosis (CF) presented with a 2-year history of a relapsing-remitting, evanescent, salmon-pink maculopapular cutaneous eruption associated with joint pain", "Each episode of joint pain and cutaneous eruption lasted 24 to 48 hours with episodes occurring anywhere from every other day to once weekly", "The patient’s mother was unable to identify any possible triggers because the episodes occurred independent of pulmonary CF exacerbations or antibiotic use", "The patient’s joint pain and cutaneous symptoms resolved completely between each episode and were not associated with fever", "Review of photos taken during the patient’s flares revealed a variable eruption consisting of oval-shaped, salmon-pink macules on the trunk, urticarial-appearing papules on the dorsal hands, and flagellate arrays of petechial macules on the upper medial arms (Figure 1)", "This eruption was intermittently pruritic but otherwise was asymptomatic", "Her joint pain recurred with each episode of cutaneous eruption and primarily affected the wrist, knee, and ankle joints bilaterally", "She had been treated with naproxen during these flares, which improved her joint pain but not her cutaneous symptoms", "A complete blood cell count and comprehensive metabolic panel as well as C-reactive protein and erythrocyte sedimentation rate obtained during a flare were normal", "Titers for parvovirus, Epstein–Barr virus, and antistreptolysin O were also negative", "A, Approximately 1-cm, oval-shaped, salmon-pink macules on the right lower abdomen and right lateral thigh", "B, 3- to 4-mm pink urticarial-appearing papules on the dorsal hand", "C, Flagellate arrays of nonblanching 1- to 2-mm petechial macules on the upper medial arm"], "s1": [0, 1, 2, 3, 6, 7], "s2": [4, 5, 8, 9, 10, 11, 12]} {"key": 704, "questions": "What Is Your Diagnosis?", "options": [{"label": "A", "disease": "Postinfectious/autoimmune chorea"}, {"label": "B", "disease": "Paraneoplastic chorea"}, {"label": "C", "disease": "Diabetic striatopathy"}, {"label": "D", "disease": "Acute stroke"}], "answer_idx": "C", "symptoms": ["A man in his late 60s with a diagnosis of type 2 diabetes for more than 10 years that was complicated by neuropathy and retinopathy presented with persistent chorea in the left upper extremity for about 1 month despite aggressive correction with insulin", "He was alert and oriented with healthy vital signs", "No evidence of cognitive impairments was present on bedside testing results", "The patient denied any recent infections and risk for human immunodefiency virus (HIV) infection", "Neurological examination results were significant for choreiform and hemiballistic movements involving the left upper extremity", "His home medications were carefully reviewed to rule out any medications that could cause chorea", "A review of his previous records indicated an admission to an outside facility 3 weeks beforehand with severe hyperglycemia and a hemoglobin A1c level of 14.1% (to convert to the proportion of total hemoglobin, multiply by 0.01)", "Despite an aggressive correction of blood glucose levels and metabolic correction, his choreiform movements had remained unresolved", "On admission, a complete electrolyte and toxicology panel, including calcium, magnesium, and phosphorus, yielded normal results", "The patient’s serum glucose level was 159 mg/dL (to convert to millimoles per liter, multiply by 0.0555) and his hemoglobin A1c was 10.9% without ketoacidosis", "Test results for antinuclear antibodies, lupus anticoagulants, and antiphospholipid antibodies were negative", "Serum thyrotropin levels, liver function test results, and parathyroid hormone levels were healthy", "Copper studies, a peripheral smear, and a routine electroencephalogram yielded normal results", "A general screening for cancers yielded negative results", "The patient underwent brain magnetic resonance imaging (MRI) (Figure), which showed a T1 hyperintensity in the right caudate head and a lentiform nucleus with no restricted diffusion", "Oral administration of olanzapine, 2.5 mg daily, in divided doses with careful monitoring of blood glucose levels resulted in the complete resolution of chorea", "Magnetic resonance imaging shows T1 hyperintensity in the right caudate head (A) and a lentiform nucleus with no restricted diffusion (B)"], "s1": [0, 4, 7, 14, 15, 16], "s2": [1, 2, 3, 5, 6, 8, 9, 10, 11, 12, 13]} {"key": 705, "questions": "what is your diagnosis?", "options": [{"label": "A", "disease": "C7 cervical spinal cord compression"}, {"label": "B", "disease": "Impingement of the left sympathetic trunk"}, {"label": "C", "disease": "Right cervical ganglion invasion"}, {"label": "D", "disease": "Transection of the left sympathetic trunk"}], "answer_idx": "A", "symptoms": ["A middle-aged man with a 25-pack-year history of cigarette smoking was in his usual state of health when he developed excessive coughing with hemoptysis, which spontaneously resolved", "Two months later, he presented to a hospital with complaint of shortness of breath, cough, and chest pain", "A computed tomographic image of the chest (Figure 1) identified a large, 12-cm left upper lobe mass abutting the anterior chest wall pleura and extending into the mediastinum, with mediastinal lymphadenopathy as well as several liver lesions", "A biopsy revealed poorly differentiated carcinoma, consistent with a primary tumor in the lung", "On presentation to our multidisciplinary clinic, he reported constant, bothersome sweating of the left forehead", "On physical examination, a well-demarcated area of erythematous, perspiring skin on the left forehead was noted", "A, Coronal computed tomographic image of the thorax with intravenous contrast at initial presentation, demonstrating a left upper lobe mass", "asterisk depicts possible area of sympathetic nerve compression or invasion", "B, Radiation planning computed tomographic image taken 2 weeks after initial presentation, demonstrating growth of the left upper lobe tumor", "The red contour identifies the tumor", "the orange contour, the area of sympathetic trunk compression", "the yellow contour, the trachea", "the teal contour, the left lung", "the purple contour, the esophagus", "the peach contour, the first rib"], "s1": [0, 1, 3, 4, 5], "s2": [2, 6, 7, 8, 9, 10, 11, 12, 13, 14]} {"key": 706, "questions": "What Is Your Diagnosis?", "options": [{"label": "A", "disease": "Squamous cell carcinoma"}, {"label": "B", "disease": "Hemangioma"}, {"label": "C", "disease": "Mixed lymphatic and venous malformation"}, {"label": "D", "disease": "Non-Hodgkin lymphoma"}], "answer_idx": "C", "symptoms": ["A previously healthy 18-year-old woman presented with a growth in the right oropharynx of 4 months’ duration", "At the time of presentation, it had begun to cause changes in her voice, globus sensation, and dysphagia", "The patient reported a 20-pound weight loss over the prior 3 months and decreased appetite", "She denied any associated difficulty breathing, pain, fever, chills, rash, or night sweats", "She was a nonsmoker and nondrinker", "On examination, she appeared well", "Despite a muffled voice, she was not in respiratory distress", "Fiberoptic laryngoscopy revealed a large, partially obstructing mass in the right oropharynx protruding from the right tonsil", "It descended inferiorly and was partially compressing the epiglottis", "The true and false vocal cords, although partially obstructed, appeared normal and mobile", "The lesion was without ulceration, exudate, or erythema", "The contralateral tonsil was small and normal in appearance", "The remainder of her physical examination findings were unremarkable, including no neck masses", "The patient agreed to surgical excision of the mass, including tonsillectomy", "Her airway was secured via an awake transoral fiberoptic intubation without difficulty", "The planned procedure occurred without difficulty or complication", "The specimen excised was a 6 × 3 × 2-cm mass continuous with the right tonsil (Figure 1), without involvement of any surrounding structures", "A, Intraoperative photograph of a large, irregularly shaped lesion protruding from the patient’s right tonsil", "B, The excised specimen measured 6 × 3 × 2 cm and was continuous with the right tonsil"], "s1": [0, 1, 2, 3, 4, 5, 6], "s2": [7, 8, 9, 10, 11, 12, 13, 14, 15, 16, 17, 18]} {"key": 707, "questions": "What Is Your Diagnosis?", "options": [{"label": "A", "disease": "Vestibular schwannoma"}, {"label": "B", "disease": "Idiopathic sudden sensorineural hearing loss"}, {"label": "C", "disease": "Neuroborreliosis cranial polyneuritis"}, {"label": "D", "disease": "Facial nerve schwannoma"}], "answer_idx": "C", "symptoms": ["A healthy man in his mid 30s residing in the northeastern United States was seen by his primary care physician in mid-June with a 2-week history of flulike symptoms", "He was empirically started on doxycycline because he reported frequenting wooded areas", "One week later, he developed a complete right facial nerve paralysis", "He was started on a course of prednisone, with subsequent full recovery of facial nerve function", "About 2 weeks later, he reported new-onset right-sided hearing loss, tinnitus, and intermittent otalgia", "He was evaluated in the otology clinic approximately 3 weeks after the onset of these otologic symptoms", "He had no prior otologic history and no history of noise exposure", "His otologic and head and neck examination findings were unremarkable", "Comprehensive audiometry was performed, which showed a mild sensorineural hearing loss from 3000 to 8000 Hz in the right ear and a mild sensorineural hearing loss at 8000 Hz only in the left ear", "Given the asymmetry in the right ear, magnetic resonance imaging (MRI) of the brain and internal auditory canals (IACs) with contrast was obtained", "There was abnormal linear and smooth enhancement of the right facial nerve in the superior distal IAC, labyrinthine segment, and geniculate ganglion", "Discrete enhancement was also seen within the inferior aspect of the right IAC in the region of the vestibulocochlear nerve (Figure 1)", "Initial magnetic resonance imaging", "T1-weighted postcontrast axial (A) and coronal (B) magnetic resonance imaging of the brain and internal auditory canals", "A, Increased enhancement of the labyrinthine segment of the facial nerve (yellow arrowhead) and geniculate ganglion (pink arrowhead)", "B, Discrete linear and smooth areas of enhancement in the superior distal portion of the right internal auditory canal and the inferior aspect of the right internal auditory canal, corresponding to the facial nerve (pink arrowhead) and the vestibulocochlear nerve (yellow arrowhead), respectively"], "s1": [0, 1, 2, 3, 4, 5, 6, 7], "s2": [8, 9, 10, 11, 12, 13, 14, 15]} {"key": 708, "questions": "What Is Your Diagnosis?", "options": [{"label": "A", "disease": "Macrocystic lymphatic malformation"}, {"label": "B", "disease": "Cystic teratoma (mature)"}, {"label": "C", "disease": "Foregut duplication cyst"}, {"label": "D", "disease": "Dermoid cyst"}], "answer_idx": "B", "symptoms": ["A 1-week-old, full-term male newborn presented with increasingly labored breathing, xerostomia, and poor bowel movements", "He had been born after an uncomplicated pregnancy, during which routine prenatal ultrasonography had not detected any anatomical abnormalities or lesions", "On physical examination, there were no abnormalities other than symmetrical submucosal swelling of the floor of the mouth and a midline external punctum present in the submental area (Figure, A)", "The tongue was displaced posteriorly and superiorly", "Contrast-enhanced magnetic resonance imaging of the neck and face performed shortly after presentation showed a midline sublingual lesion extending inferiorly through the mylohyoid muscle (Figure, B-D)", "The intrinsic muscles of the tongue appeared minimally displaced without obvious infiltration", "however, the genioglossus muscle was difficult to locate relative to the mass", "A thin-walled, multiloculated lesion was identified, measuring approximately 2.0 × 2.5 × 3.2 cm", "The mass was bright on T2-weighted imaging and dark on T1-weighted imaging and did not enhance with intravenous gadolinium", "There were no obvious fluid-fluid levels or calcifications", "When in-phase and out-of-phase images were compared, there was no significant signal intensity drop, suggesting an absence of fat within the lesion", "A, Appearance of the submental neck", "B, Coronal short tau inversion recovery (STIR) T2-weighted magnetic resonance image without contrast enhancement", "C, Coronal T1-weighted, fat-saturated magnetic resonance image after contrast enhancement", "D, Sagittal T1-weighted non–fat-saturated magnetic resonance image after contrast enhancement"], "s1": [0, 1, 2, 3], "s2": [4, 5, 6, 7, 8, 9, 10, 11, 12, 13, 14]} {"key": 709, "questions": "What Is Your Diagnosis?", "options": [{"label": "A", "disease": "Pyoderma gangrenosum"}, {"label": "B", "disease": "Blastomycosis"}, {"label": "C", "disease": "Pyoderma vegetans"}, {"label": "D", "disease": "Halogenoderma"}], "answer_idx": "C", "symptoms": ["A teenaged male presented with a 2-week history of vegetating, bleeding plaques and pustules on his face, scalp, trunk, and extremities", "He reported that lesions began as pustules and developed into painful, itchy plaques", "Before evaluation at a referral hospital in Kenya, the patient was treated with prednisolone tablet 5 mg twice daily, clindamycin hydrochloride 300 mg daily, and fluconazole 200 mg daily, but he continued to develop new lesions", "A complete review of systems was notable for the patient’s difficulty with walking because of skin lesions", "On examination, pustules and hemorrhagic-crusted plaques were seen on the scalp, legs, arms, and trunk along with keloidal plaques on the face (Figure, A and B)", "Enzyme-linked immunosorbent assay testing for HIV was negative", "A punch biopsy specimen from the arm was obtained for histopathologic study (Figure, C and D)", "Extensive vegetative plaques with bleeding and ulceration on extremities (A), and pustules and ulcerated plaques on extremities and trunk (B)", "Histopathologic examination of a biopsy specimen taken from the arm demonstrates pseudoepitheliomatous hyperplasia (hematoxylin-eosin stain, original magnification ×100) (C) and dermal neutrophilia and microabscesses with eosinophilia (hematoxylin-eosin stain, original magnification ×400) (D)"], "s1": [0, 1, 3, 4, 7], "s2": [2, 5, 6, 8]} {"key": 710, "questions": "What Is Your Diagnosis?", "options": [{"label": "A", "disease": "Primary anetoderma"}, {"label": "B", "disease": "Atrophoderma vermiculata"}, {"label": "C", "disease": "Atrophia maculosa varioliformis cutis"}, {"label": "D", "disease": "Facial morphea"}], "answer_idx": "C", "symptoms": ["A 26-year-old man presented with a several-year history of persistent round lesions with a scar-like appearance on his face", "Before evaluation, his condition had been diagnosed as lupus and lipoatrophy and he tried several treatments with no success", "The results of his physical examination showed several round atrophic lesions on his face, symmetric and bilateral, that were located especially on the malar and preauricular areas, with a diameter of 5 to 10 mm", "His skin lesions were not indurated and displayed no epidermal changes (Figure 1)", "The patient denied experiencing previous inflammation", "He had no history of chicken pox or acne or traumatic lesions in those areas", "New punch biopsy specimens were obtained for further evaluation", "Atrophic lesions on the patient’s left cheek (A) and with a cicatricial appearance on the preauricular right area (B)"], "s1": [0, 1, 2, 3, 4, 5], "s2": [6, 7]} {"key": 711, "questions": "What Is Your Diagnosis?", "options": [{"label": "A", "disease": "Primary penile B-cell non-Hodgkin lymphoma"}, {"label": "B", "disease": "Kaposi sarcoma of the penis"}, {"label": "C", "disease": "Plasmacytoma"}, {"label": "D", "disease": "Primary nodular amyloidosis of the glans penis"}], "answer_idx": "C", "symptoms": ["A 60-year-old man with a history of IgA kappa multiple myeloma was referred to our department", "He had been diagnosed with multiple myeloma 2 years earlier after discovery of osteolytic bone lesions of the skull and the findings of a bone marrow biopsy", "The patient had been treated with bortezomib and dexamethasone followed by autologous blood progenitor cell transplantation, which resulted in complete response", "At the time of the consultation, he was free of disease", "The patient was referred for evaluation of an indolent, erythematous papule on his glans penis of 2 weeks’ duration", "He denied sexual risk behavior and reported no recent changes in medication", "Careful physical examination revealed a shiny, tense, erythematous, hemispherical papule measuring 5 mm in diameter located on the lateral aspect of the glans (Figure 1A)", "There were no other skin lesions", "The lymph nodes were not palpable, and there was no organomegaly", "A punch biopsy was performed for histopathological evaluation (Figure 1B and C)", "A, Shiny, erythematous, hemispherical papule on the lateral aspect of the glans penis", "B and C, Hematoxylin-eosin stain was performed for evaluation"], "s1": [0, 1, 2, 3], "s2": [4, 5, 6, 7, 8, 9, 10, 11]} {"key": 712, "questions": "What Is Your Diagnosis?", "options": [{"label": "A", "disease": "Sliding hernia"}, {"label": "B", "disease": "Saphena varix"}, {"label": "C", "disease": "Amyand hernia (appendix included within the inguinal hernia sac)"}, {"label": "D", "disease": "Communicating hydrocele"}], "answer_idx": "A", "symptoms": ["A 55-year-old man with a history of Childs B cirrhosis presented with a 2.5-year history of right groin pain and increased urinary frequency with difficulty voiding", "On examination, he had a 5 cm × 3 cm palpable and partially reducible right groin mass that extended into the scrotum", "He had a leukocytosis with a leukocyte count of 16 400/μL (to convert to ×109/L, multiply by 0.001), his creatinine level was 1.14 mg/dL (to convert to micromoles per liter, multiply by 88.4), and urinalysis showed leukocyturia with cultures that had positive results for Streptococcus viridians", "Axial and sagittal (Figure) computed tomography scans with contrast are shown", "A renography using mercaptoacetyltriglycine showed a 34% reduction of right renal function", "Computed tomography cystogram without intravenous contrast, axial (A) and sagittal (B) view", "Amyand hernia (appendix included within the inguinal hernia sac)"], "s1": [0, 1, 6], "s2": [2, 3, 4, 5]} {"key": 713, "questions": "What Is Your Diagnosis?", "options": [{"label": "A", "disease": "Crohn ileitis"}, {"label": "B", "disease": "Foreign body ingestion"}, {"label": "C", "disease": "Ileo-colic intussusception"}, {"label": "D", "disease": "Adenocarcinoma of small intestine"}], "answer_idx": "C", "symptoms": ["A healthy 29-year-old pregnant woman, with 34 weeks’ gestational age, presented to the emergency department with diffused acute abdominal pain and vomiting", "The pain was located in the periumbilical region and intensified within a few hours", "The patient felt normal fetal movements during this time", "An initial surgical evaluation was done, and the cause for the abdominal pain was assumed to be not of surgical condition", "Next, the patient was evaluated by an obstetrician-gynecologist, and although the gynecologic examination, ultrasonography, and monitoring were all normal, the patient was admitted for further supervision in the obstetrics department", "The next day, when the abdominal pain and vomiting persisted and an elevated leukocyte blood count of 14 200/mL was noted (to convert to ×109 per liter, multiply by 0.001), a second surgical consultation was done", "The physical examination was significant for diffuse abdominal tenderness", "Although the patient was pregnant, and owing to nonavailability of magnetic resonance imaging at that time, a contrast-enhanced abdominopelvic computed tomography was performed (Figure 1)", "Coronal computed tomographic image showing an edematous small intestinal loop on the left abdomen, with uterus and head of fetus in the pelvis"], "s1": [0, 1, 2, 3, 4], "s2": [5, 6, 7, 8]} {"key": 714, "questions": "What Is Your Diagnosis?", "options": [{"label": "A", "disease": "Secondary syphilis"}, {"label": "B", "disease": "Lichen planus"}, {"label": "C", "disease": "Lichenoid drug reaction"}, {"label": "D", "disease": "Janeway lesions secondary to endocarditis"}], "answer_idx": "C", "symptoms": ["A black man in his 30s presented for evaluation of papules and plaques on his bilateral lower extremities", "Nine months before the current presentation, he was treated with intramuscular penicillin for syphilis", "His medical history was also significant for HIV, for which he was receiving highly active antiretroviral therapy, and metastatic programmed cell death ligand 1–positive squamous non–small cell lung cancer", "Four months before the current presentation, the patient had started pembrolizumab monotherapy as a first-line treatment for his cancer", "One month before this presentation, multiple ulcers developed on his buccal mucosa, as well as a nonpainful shallow penile ulcer and pruritic, mildly painful, hyperpigmented papules and plaques on the bilateral soles of his feet in a pattern identical to previous syphilitic eruption", "The patient denied fever, chills, and weight loss", "Physical examination revealed multiple hyperpigmented papules and plaques with violaceous borders on his bilateral soles and lateral feet (Figure, A) and scattered, violaceous papules and nodules with overlying scale on his bilateral lower extremities", "He was referred to the dermatology service, and a punch biopsy specimen was obtained from the right medial sole (Figure, B)", "A, Right foot with multiple hyperpigmented, lichenified papules, and plaques with violaceous borders", "B, A punch biopsy specimen from the right medial sole", "Lymphocytes, eosinophils, and plasma cells can be seen within the infiltrate (hematoxylin-eosin, original magnification ×40)"], "s1": [0, 4, 5, 6, 8], "s2": [1, 2, 3, 7, 9, 10]} {"key": 715, "questions": "What Is Your Diagnosis?", "options": [{"label": "A", "disease": "Squamous cell carcinoma"}, {"label": "B", "disease": "Lymphoma"}, {"label": "C", "disease": "Ectopic pituitary adenoma"}, {"label": "D", "disease": "Granulomatous disease"}], "answer_idx": "C", "symptoms": ["A man in his 50s with a history of chronic hepatitis C, hypothyroidism, and alcoholism presented to the outpatient clinic complaining of nasal congestion and epistaxis", "Nonenhanced computed tomography of the paranasal sinuses revealed a soft-tissue mass extending from the nasopharynx into the left sphenoid sinus with erosion of the sphenoid sinus floor and the clivus (Figure 1A)", "The sella was apparently normal with no disruption of its floor", "The intracranial and orbital soft tissues were grossly normal, as were the remaining soft tissues under the skull base", "A, Sagittal nonenhanced computed tomography images of the paranasal sinus depicting a soft-tissue mass involving the sphenoid sinus, the nasopharynx, and the clivus", "B, Postcontrast sagittal T1-weighted magnetic resonance shows avid enhancement of the mass", "involvement of the clivus is also depicted", "A magnetic resonance imaging study was also performed (Figure 1B)", "There was a lobulated mass centered to the nasopharynx, measuring about 2.8 × 2.6 × 4.5 cm (anteroposterior × transverse × craniocaudal planes, respectively) in maximal diameters", "The mass demonstrated intermediate T2 hyperintense signal and avid postcontrast enhancement", "It extended inferiorly down to the oropharynx and superiorly along the floor of the left sphenoid sinus with extension into the sinus itself", "posteriorly, there were erosions involving the sphenoid bone and the clivus", "The floor of sella was intact, and a normal enhancing pituitary gland and infundibulum were seen within the sella", "No sellar or suprasellar mass was evident", "Both Meckel caves were normal, and the cavernous sinuses were also normal and without evidence of asymmetric mass or abnormal enhancement", "The patient underwent a total mass resection via endoscopic anterior cranial base approach, with extradural tumor resection, reconstruction of the skull base, and a nasoseptal flap"], "s1": [0, 15], "s2": [1, 2, 3, 4, 5, 6, 7, 8, 9, 10, 11, 12, 13, 14]} {"key": 716, "questions": "What Is Your Diagnosis?", "options": [{"label": "A", "disease": "Facial nerve venous malformation"}, {"label": "B", "disease": "Jugulotympanic paraganglioma"}, {"label": "C", "disease": "Primary middle ear meningioma"}, {"label": "D", "disease": "Middle ear adenoma"}], "answer_idx": "C", "symptoms": ["A woman in her 30s presented with a 1-year history of left ear fullness", "She reported diminished left-sided hearing that would temporarily improve with autoinsufflation", "She denied facial numbness or weakness, and reported no dizziness or vertigo", "Physical examination revealed a retrotympanic, nonpulsatile, tan-gray mass anterior to the malleus handle", "A tuning fork examination revealed lateralization to the left ear and bone conduction louder than air conduction on the left side with a 512-Hz tuning fork", "The remainder of the physical examination, including facial nerve function, was normal", "An audiogram confirmed mild conductive hearing loss in the left ear", "Computed tomography (CT) revealed a middle ear lesion lateral to the geniculate ganglion with possible extension into the peritubal space", "Magnetic resonance imaging (MRI) demonstrated an enhancing lesion lateral to the geniculate ganglion (Figure, A-C)", "The patient was initially observed but then experienced progressive hearing loss that no longer improved with the Valsalva maneuver, persistent ear fullness, and rare facial twitching", "The middle ear mass appeared enlarged on repeated MRI and began to obstruct the eustachian tube", "Transcanal endoscopy demonstrated a gray middle ear mass occupying the anterior mesotympanum and epitympanum and extending into the eustachian tube (Figure, D)", "The tegmen was intact", "The specimen revealed predominantly spindled stroma with a focus of tumor that was positive for antiepithelial membrane antigen and somatostatin receptor 2A", "A, Noncontrast axial computed tomogram (CT) of the left temporal bone demonstrates an anterior mesotympanic mass lateral to the cochlea (yellow arrowhead) with blockage and extension into the supratubal recess (white arrowhead)", "A partial mastoid effusion (asterisk) is also visible", "B, The coronal CT demonstrates an intact tegmen tympani with hyperostosis (arrowhead)", "C, Axial, fat-suppressed, postgadolinium, T1-weighted magnetic resonance image (MRI) reveals an enhancing mass of the left perigeniculate region (yellow arrowhead)", "The geniculate ganglion is marked (white arrowhead)", "D, Intraoperative transcanal photograph of a lobulated, gray, middle ear mass involving the left protympanum and epitympanum"], "s1": [0, 1, 2, 9, 10, 15], "s2": [3, 4, 5, 6, 7, 8, 11, 12, 13, 14, 16, 17, 18, 19]} {"key": 717, "questions": "What Is Your Diagnosis?", "options": [{"label": "A", "disease": "Dermoid cyst (mature teratoma)"}, {"label": "B", "disease": "Hairy polyp"}, {"label": "C", "disease": "Glioma"}, {"label": "D", "disease": "Encephalocele"}], "answer_idx": "B", "symptoms": ["A newborn twin girl was referred to the pediatric otolaryngology clinic at a tertiary pediatric hospital for evaluation of a left-sided nasal mass causing nasal obstruction and difficulty breathing, especially with feeding (Figure 1A)", "She was born at 36 weeks’ gestation and spent 10 days in the neonatal intensive care unit", "Her parents noted that the lesion was present at birth and she always seemed congested on the left side", "There was no report of clear drainage", "Physical examination showed an approximately 1-cm, firm, pedunculated polypoid mass in the anterior nasal cavity at the vestibule", "The base was located just anterior to the septum and encompassed nearly the entire left nasal cavity", "There was no fluid in the mass, and there was a negative Furstenberg sign", "Nasal endoscopy was performed in the clinic, and there were no additional masses or mucosal abnormalities", "The choana was patent", "The patient was also noted to have left eye ptosis and left preauricular branchial remnants but no other significant abnormalities on examination", "Magnetic resonance imaging was performed (Figure 1B) and showed a 0.8 × 0.5–cm hyperintense mass on T1-weighted images, suggesting a fatty component and no intracranial tract", "Photograph (A) and magnetic resonance imaging (MRI) (B) of the left anterior nasal cavity mass"], "s1": [0, 2, 4, 5, 6, 7], "s2": [1, 3, 8, 9, 10, 11]} {"key": 718, "questions": "What Is Your Diagnosis?", "options": [{"label": "A", "disease": "Trichothiodystrophy"}, {"label": "B", "disease": "Cockayne syndrome"}, {"label": "C", "disease": "Xeroderma pigmentosum"}, {"label": "D", "disease": "Actinic keratosis"}], "answer_idx": "C", "symptoms": ["A 6-year-old boy presented with several tumoral lesions on sun-exposed skin", "These lesions developed from local freckles beginning at 6 months of age and gradually spread with protruding growth", "Over the past 2 years, tumors started to form and develop rapidly, becoming ulcerative and bleeding", "The patient was intellectually standard and denied blurred vision, impaired hearing, or any other systemic symptoms", "He lived in a remote area in China", "His parents were first cousins, but no similar cutaneous presentation was reported by other relatives", "The physical examination revealed scaly skin with diffuse warty lesions on sun-exposed areas", "The patient’s face was severely disfigured, with numerous ulcerative, malodorous tumors (Figure 1A)", "The general examination findings, including neurological and ophthalmologic tests, were unremarkable", "The lesional discharge culture revealed infection with Proteus vulgaris", "Punch biopsy of a paranasal mass was performed (Figure 1B)", "Craniofacial computed tomography showed heterogenous soft tissue masses located superficially without bony invasion", "No signs of cerebral atrophy were noticed", "Genetic testing was advised but was declined", "A, Preoperative clinical photography revealed a face severely disfigured by diffuse warty lesions and numerous tumors", "B, Infiltrating basal cell carcinoma (original magnification ×40)"], "s1": [0, 1, 2, 6, 7, 9, 11, 14, 15], "s2": [3, 4, 5, 8, 10, 12, 13]} {"key": 719, "questions": "What Is Your Diagnosis?", "options": [{"label": "A", "disease": "Squamous cell carcinoma"}, {"label": "B", "disease": "Mucormycosis"}, {"label": "C", "disease": "Aspergillosis"}, {"label": "D", "disease": "Myeloid sarcoma"}], "answer_idx": "B", "symptoms": ["A man in his 20s presented with rapidly growing, severely painful oral lesions on the left hard palate of 2 months’ duration", "His medical history was significant for Philadelphia chromosome-negative B-cell acute lymphocytic leukemia refractory to multiple treatments, including chimeric antigen receptor T-cell therapy, with transformation to acute myeloid leukemia (AML) status posttreatment", "At the time of evaluation, he had been hospitalized for 5 months with severe protein-calorie malnutrition, pancytopenia, and recurrent neutropenic fevers", "The patient was febrile (38.3°C), with a white blood cell count of 0.5/μL (absolute neutrophil count 0.00", "to convert to ×109/L, multiply by 0.001), hemoglobin levels of 7.6g/dL (to convert to g/L, multiply by 10.0), hematocrit 21%, and platelet count 9/μL (to convert to ×109/L, multiply by 1.0)", "He was not receiving active cancer treatment", "He appeared cachectic and pale", "Intraoral examination revealed poor oral hygiene", "The left hard palate exhibited a large swelling with a 5×3-cm indurated area of ulceration, central necrosis, and desquamation (Figure, A)", "The patient was treated palliatively with dexamethasone solution and nystatin rinse", "A biopsy was not recommended owing to medical instability", "However, the patient retained a sloughed fragment of the affected tissue, which was submitted for histologic evaluation", "A maxillofacial computed tomography (CT) scan completed the etiological investigation", "A, The left hard palate shows enlargement with a large, focal, necrotic lesion", "B, Hematoxylin-eosin stain of palatal tissue (original magnification ×1000)", "C, Grocott methenamine silver stain of palatal tissue (original magnification ×1000)", "D, Computed tomographic image of the maxillofacial complex shows tissue enlargement in the left hard palate and maxillary sinus"], "s1": [0, 1, 2, 3, 4, 5, 6], "s2": [7, 8, 9, 10, 11, 12, 13, 14, 15, 16]} {"key": 720, "questions": "What Is Your Diagnosis?", "options": [{"label": "A", "disease": "Tinea versicolor"}, {"label": "B", "disease": "Vitiligo"}, {"label": "C", "disease": "Clear cell papulosis"}, {"label": "D", "disease": "Idiopathic guttate hypomelanosis"}], "answer_idx": "C", "symptoms": ["A child presented with multiple, asymptomatic, scattered white macules on the lower abdomen and pubic area, which were present at birth and had gradually increased in size and number", "Physical examination revealed multiple, well-demarcated, hypopigmented 1- to 5-mm macules on the lower abdomen and pubic area (Figure, A and B)", "There was no history of any systemic disorder and no family history of similar lesions", "Direct microscopic examination of scales soaked in potassium hydroxide did not reveal hyphae", "Wood light examination revealed no accentuation", "A punch biopsy specimen was obtained for further evaluation (Figure, C and D)", "A, Multiple discrete hypopigmented macules on the lower abdomen and pubic area", "B, Well-demarcated hypopigmented macules on the lower abdomen", "C, Histopathological examination revealed mild hyperkeratosis, acanthosis, and scattered clear cells in the epidermis (hematoxylin-eosin)", "D, The clear cells are larger than the neighboring keratinocytes and contain abundant pale cytoplasm (hematoxylin-eosin)"], "s1": [0, 1, 2, 6, 7], "s2": [3, 4, 5, 8, 9]} {"key": 721, "questions": "What Is Your Diagnosis?", "options": [{"label": "A", "disease": "Extranodal natural killer/T-cell lymphoma, nasal type"}, {"label": "B", "disease": "Lymphomatoid granulomatosis"}, {"label": "C", "disease": "Granulomatosis with polyangiitis (Wegener granulomatosis)"}, {"label": "D", "disease": "Histoplasmosis"}], "answer_idx": "B", "symptoms": ["A 33-year-old white man presented with a 5-month history of a progressively necrotic plaque on the nose and scattered nodules on the extremities", "The plaque first appeared as a small inflammatory papule on the left nostril", "Shortly thereafter, the patient was hospitalized at an outside facility for multifocal pneumonia", "He was also treated for a methicillin-resistant Staphylococcus aureus infection of his nose", "Despite antibiotic therapy, the papule continued to grow and ulcerate and eventually became necrotic (Figure, A)", "One month before presentation at our facility, the patient began developing erythematous papules on his arms and legs", "The papules enlarged and ulcerated", "Some of them healed spontaneously with poxlike scars", "He had no recent history of travel", "At admission, the patient was tachycardic and febrile to a temperature of 38.5°C", "Routine laboratory testing revealed normocytic anemia (hemoglobin, 7.7 g/dL [to convert to grams per liter, multiply by 10])", "Shave biopsies of necrotic nodules on the left arm and left cheek (Figure, B-D) and, later, an excisional biopsy of a cervical lymph node were performed", "Head computed tomography demonstrated a soft-tissue mass that encased the nasal bone and extended into the right maxillary sinus and left nasal cavity", "Chest computed tomography showed bilateral pulmonary nodules", "Physical examination revealed a necrotic plaque with surrounding erythema on the distal nose (Figure, A)", "A 1 × 1-cm crateriform papule with central necrosis was present on the left malar cheek", "Multiple nodules with central, dry necrosis and areas of poxlike scarring were scattered across the extremities", "Slight contractures of his fingers were also noted", "A, Clinical photograph", "B-D, Analyses of a shave biopsy specimen of the necrotic papule on the patient’s left cheek (B, hematoxylin-eosin, original magnification ×400", "C, original magnification ×400", "D, original magnification ×200)"], "s1": [0, 1, 3, 4, 5, 6, 7, 8, 9, 10, 14, 15, 16, 17], "s2": [2, 11, 12, 13, 18, 19, 20, 21]} {"key": 722, "questions": "What Is Your Diagnosis?", "options": [{"label": "A", "disease": "Abdominal tuberculosis"}, {"label": "B", "disease": "Sclerosing encapsulating peritonitis"}, {"label": "C", "disease": "Adhesive small bowel obstruction"}, {"label": "D", "disease": "Peritoneal carcinomatosis"}], "answer_idx": "B", "symptoms": ["A man in his 50s of West African descent with a history of prostate cancer treated with robotic prostatectomy 2 years ago presented with 1 year of progressively worsening postprandial nausea or vomiting, epigastric abdominal pain, anorexia, and weight loss of 27 kg", "Four months earlier, he had presented to another hospital and received a diagnosis of adhesive small bowel obstruction, for which he underwent laparotomy and lysis of small adhesive bands that provided only temporary symptomatic improvement", "At his current presentation, he was unable to tolerate any oral intake and entirely dependent on total parenteral nutrition", "On examination, he was cachectic with a moderately distended, nontender abdomen", "The result of a blood test (QuantiFERON-TB Gold", "Qiagen) was positive for tuberculosis exposure", "An abdominal computed tomography scan showed markedly dilated stomach and duodenum with decompressed, thickened distal small bowel without a clear transition point", "Nonoperative management with persistently high nasogastric tube output failed, and the patient was subsequently taken to the operating room for exploratory laparotomy", "Intraoperative findings are shown in Figure 1.Exploratory laparotomy showing the appearance of the small bowel"], "s1": [0, 2, 3, 6, 7, 8], "s2": [1, 4, 5]} {"key": 723, "questions": "What Is Your Diagnosis?", "options": [{"label": "A", "disease": "Carcinoma in the body of the pancreas"}, {"label": "B", "disease": "Advanced carcinoma of the duodenum"}, {"label": "C", "disease": "Posterior perforation of a duodenal ulcer"}, {"label": "D", "disease": "Infected aortic graft with aortoduodenal fistula"}], "answer_idx": "D", "symptoms": ["A 74-year-old man presented to the primary care physician with a 3-month history of weight loss, anorexia, and back pain", "His medical history was significant for hypertension, hyperlipidemia, and kidney stones", "His surgical history was significant for a bilateral staged carotid endarterectomy in 1999 and 2000 and an open repair of a 7.4-cm transverse diameter juxtarenal abdominal aortic aneurysm with a knitted Dacron tube graft (18 mm) in 2014. Ligation of the left renal vein adjacent to the inferior vena cava was performed to gain proximal control of the aorta", "The patient underwent extraction of a tooth secondary to an abscess 4 months earlier", "Physical examination revealed poor dental hygiene and findings in the lower extremity suggestive of postthrombotic syndrome", "The patient was afebrile with a heart rate of 108 beats per minute and a leukocyte level of 11 600/cm (normal range, 4000-11 000/cm), with 84% polymorphic neutrophils (normal, <75%)", "erythrocyte sedimentation rate was 57 mm/h (normal range, 0-10 mm/h), and C-reactive protein level was 15.9 mg/dL (normal, <0.6 mg/dL [to convert to nmol/L, multiply by 9.524])", "Computed tomography (CT) of the abdomen and pelvis was performed (Figure 1A and B)", "CT indicates computed tomography", "WBC, white blood cell"], "s1": [0, 3, 4, 5, 6], "s2": [1, 2, 7, 8, 9]} {"key": 724, "questions": "What Is Your Diagnosis?", "options": [{"label": "A", "disease": "Mycosis fungoides"}, {"label": "B", "disease": "Follicular lymphoma"}, {"label": "C", "disease": "Lichen planus"}, {"label": "D", "disease": "Marginal zone lymphoma"}], "answer_idx": "B", "symptoms": ["A 67-year-old man presented with a rapidly enlarging scalp lesion (Figure 1A) first noticed 3 months earlier as a pimple", "The lesion was itchy but without generalized pruritus", "The patient reported a weight loss of 10 pounds over the preceding 3 months", "In addition, he reported a longer history of vague, nonspecific, intermittent abdominal pain without nausea, vomiting, or fever starting about 3 years prior to presentation", "Previous work-up had revealed minimal abdominal adenopathy, but the patient declined further workup", "On examination, he had no other cutaneous lesions, but he did have cervical lymphadenopathy", "All other physical findings were within normal limits", "Laboratory findings were unremarkable, save for a mild normochromic normocytic anemia and a mildly elevated levels of lactate dehydrogenase", "A biopsy specimen from the skin lesion is shown in Figure 1B", "A, Clinical image of the patient at presentation", "B, Biopsy specimen from the skin lesion is shown"], "s1": [0, 1, 5, 8, 9, 10], "s2": [2, 3, 4, 6, 7]} {"key": 725, "questions": "What Is Your Diagnosis?", "options": [{"label": "A", "disease": "Aggressive invasive thyroid cancer"}, {"label": "B", "disease": "Amyloidosis"}, {"label": "C", "disease": "Benign thyroid rest"}, {"label": "D", "disease": "Mucoid pseudotumor"}], "answer_idx": "C", "symptoms": ["A woman in her early 70s presented to her local emergency department with the recent onset of wheezing and dyspnea", "Noncontrast computed tomography of her chest showed a 1.1-cm nodular lesion in the upper trachea that appeared to be originating from the right lobe of the thyroid (Figure 1A)", "She was referred to our center for evaluation and management of what appeared to be an aggressive thyroid malignant neoplasm", "She presented with obvious biphasic stridor and a report of dyspnea on mild exertion", "A CT of the chest showed a soft-tissue mass protruding into the trachea in the posterolateral aspect of the right side of the tracheal region that appeared to be contiguous with the thyroid gland", "Ultrasound-guided fine-needle aspiration of the thyroid was performed", "however, there was no nodule in the right thyroid lobe that correlated with the CT findings", "The left thyroid lobe contained an 8-mm nodule", "Examination of a biopsy specimen from this nodule showed medullary thyroid cancer (MTC)", "The patient’s serum calcitonin level was 23 pg/mL", "Magnetic resonance imaging (MRI) of the neck revealed a hyperintense tracheal mass on T2-weighted imaging (Figure 1B) that showed enhancement on administration of contrast medium (not shown)", "In addition, a 0.3-cm nonenhancing nodule at the lower pole of the right thyroid lobe, separate from the tracheal mass, was identified", "Vocal cord mobility was normal on fiberoptic laryngoscopy, and the patient verbally denied having dysphagia", "Both ultrasonography and CT/magnetic resonance imaging failed to identify any suspicious adenopathy", "A, Axial computed tomogram (CT) of the chest without contrast medium shows a 1.1-cm hypodense, exophytic right posterolateral intratracheal mass with apparent continuity with the right thyroid lobe", "B, Axial, T2-weighted magnetic resonance image (MRI) with contrast medium revealed a hyperintense soft-tissue mass within the tracheal lumen", "A T1-weighted image before the addition of contrast medium (not shown) showed a slightly hyperintense mass to the thyroid gland", "That lesion showed enhancement on administration of contrast medium"], "s1": [0, 3, 12], "s2": [1, 2, 4, 5, 6, 7, 8, 9, 10, 11, 13, 14, 15, 16, 17]} {"key": 726, "questions": "What Is Your Diagnosis?", "options": [{"label": "A", "disease": "Lymphoma"}, {"label": "B", "disease": "Acute invasive fungal sinusitis"}, {"label": "C", "disease": "Granulomatosis with polyangiitis"}, {"label": "D", "disease": "Sinonasal undifferentiated carcinoma"}], "answer_idx": "C", "symptoms": ["A woman in her 70s with a history of pulmonary embolism presented to the emergency department with acute right eye vision loss", "She described a 2-week medical history of diplopia and blurry vision as well as 4 months of nasal congestion and progressive headaches despite multiple antibiotic courses", "She denied a medical history of sinonasal issues, immunodeficiency, autoimmune disease, malignant disease, or diabetes", "On examination, she was afebrile with stable vital signs", "Ophthalmologic evaluation revealed a right afferent pupillary defect and cranial sixth-nerve palsy", "Computed tomographic and magnetic resonance imaging of the sinuses revealed abnormal soft tissue throughout her sinuses with destruction of the sinus walls and skull base as well as extension of abnormal appearing tissue into the right orbital apex and posterior nasal septum", "Laboratory evaluation demonstrated a normal white blood cell count and an elevated erythrocyte sedimentation rate and C-reactive protein level", "Nasal endoscopy showed bloody mucus and crusting around the right middle turbinate and sphenoethmoidal recess", "The acute nature of her vision loss amid concern for a destructive sinonasal process warranted operative intervention", "Abnormal soft tissue with associated destruction throughout her sinonasal cavities and right orbital apex was identified", "Frozen sections revealed negative results for fungal elements and cellular dysplasia but demonstrated widespread inflammation with giant cells", "A representative final pathology slide is shown in the Figure", "Sinonasal mucosal biopsy results demonstrate numerous multinucleated giant cells (black arrowheads), necrosis (white arrowhead), and inflammatory cells invading vessel walls (blue arrowheads)", "Hematoxylin-eosin stain (original magnification ×10)"], "s1": [0, 1, 2, 3, 4, 8], "s2": [5, 6, 7, 9, 10, 11, 12, 13]} {"key": 727, "questions": "What Is Your Diagnosis?", "options": [{"label": "A", "disease": "Extranodal T-cell lymphoma"}, {"label": "B", "disease": "Granulomatosis with polyangiitis (Wegner granulomatosis) and relapsing polychondritis"}, {"label": "C", "disease": "Squamous cell carcinoma"}, {"label": "D", "disease": "Necrotizing fasciitis"}], "answer_idx": "A", "symptoms": ["A 32-year-old Hispanic man without significant medical history presented with nasal pain and swelling, progressive for 6 weeks", "He had been hospitalized for this problem at an outside hospital and treated with antibiotics, without improvement", "The swelling worsened and extended to his upper lip", "He developed difficulty swallowing, headache, and nasal drainage", "On presentation, he was afebrile without leukocytosis", "Physical examination demonstrated diffuse swelling and tenderness of the nose with honey-colored crusting that extended to above his upper lip", "Axial and sagittal contrast-enhanced computed tomography (CT) (Figure, A and B) demonstrated marked soft tissue swelling and fullness with abnormal enhancement along the nasal cavity, including bilateral nasal ala, with areas of necrosis and emphysema", "No significant bony erosion or associated calcification on bone windows was noted", "After bedside debridement of the nasal soft tissue region, axial postcontrast T1-weighted fat-suppressed and sagittal T2-weighted fat-suppressed magnetic resonance imaging through the nasal cavity (Figure, C and D) demonstrated soft tissue fullness isointense to muscle on T1-weighted images, hyperintense on T2-weighted images, and with homogenous enhancement of the solid component on postcontrast T1-weighted images", "No intracranial or intraorbital disease extension was noted", "Contrast-enhanced computed tomography (CT) demonstrates marked soft tissue swelling and fullness with abnormal enhancement along the nasal cavity, including bilateral nasal ala, with areas of necrosis and emphysema", "Fat-suppressed magnetic resonance imaging (MRI) demonstrates soft tissue fullness isointense to muscle on T1-weighted images, hyperintense on T2-weighted images, and with homogenous enhancement of the solid component on postcontrast T1-weighted images", "Granulomatosis with polyangiitis (Wegner granulomatosis) and relapsing polychondritis"], "s1": [0, 1, 2, 3, 4, 5], "s2": [6, 7, 8, 9, 10, 11, 12]} {"key": 728, "questions": "What Is Your Diagnosis?", "options": [{"label": "A", "disease": "Osteochondroma"}, {"label": "B", "disease": "Osteoid osteoma"}, {"label": "C", "disease": "Osteosarcoma"}, {"label": "D", "disease": "Myositis ossificans"}], "answer_idx": "D", "symptoms": ["A girl presented with a left posterior triangle neck mass that had progressed for 3 years following a motor vehicle accident", "On physical examination, the mass was firm, mobile, tender to palpation, and caused restricted range of motion", "Ultrasound imaging of the soft tissue of the neck was obtained, which demonstrated a neck mass with dense acoustic shadowing (Figure 1A)", "Further evaluation with contrasted computed tomography (CT) imaging showed a 3.3×2.9×2.6-cm ossified lesion in the left posterior-lateral cervical soft tissue near the atlanto-axial junction", "The lesion was well circumscribed and separate from the underlying bone (Figure 1B)", "The patient underwent excision of the mass without complication", "The gross pathology specimen showed a firm, irregular, encapsulated mass", "A, Ultrasonography of the neck soft tissue demonstrating a neck mass with dense acoustic shadowing in the left posterior triangle", "B, Axial computed tomographic imaging of the neck with contrast depicting a 3.3-cm well-circumscribed ossified mass that is separated from the C2 vertebral body", "No aggressive-appearing periosteal reaction is present (arrowhead indicates calcified neck mass)"], "s1": [0, 1, 5, 6], "s2": [2, 3, 4, 7, 8, 9]} {"key": 729, "questions": "What Is Your Diagnosis?", "options": [{"label": "A", "disease": "Familial benign pemphigus (Hailey-Hailey disease)"}, {"label": "B", "disease": "Inverse lichen planus"}, {"label": "C", "disease": "Fixed drug eruption"}, {"label": "D", "disease": "Erythrasma"}], "answer_idx": "B", "symptoms": ["A man in his 40s presented with a 3-month history of discrete, dark red, circular lesions in his right axilla", "He endorsed severe pruritus at these sites that would wake him up at night", "He denied any recent trauma or history of similar lesions", "He denied associated pain, blisters, or systemic symptoms (eg, fever, chills)", "His medical history was unremarkable, and he noted no environmental triggers or new medications", "Treatment with over-the-counter hydrocortisone cream, ketoconazole cream, and discontinuation of underarm deodorant yielded no improvement", "Physical examination revealed 3 well-circumscribed, violaceous, annular patches without scale or erosion that are limited to his right axilla (Figure, A)", "No other lesions were noted on his skin or mucous membranes", "A Wood lamp examination was negative", "A punch biopsy specimen from a lesion was obtained (Figure, B and C)", "A, Clinical photograph shows oval and annular thin violaceous patches in the right axilla", "B, Hematoxylin-eosin stain (original magnification ×40) demonstrates lymphocytes obscuring the dermal-epidermal junction", "C, Hematoxylin-eosin stain (original magnification ×200)"], "s1": [0, 1, 2, 3, 4, 5, 6, 7, 8], "s2": [9, 10, 11, 12]} {"key": 730, "questions": "What Is Your Diagnosis?", "options": [{"label": "A", "disease": "Oral frictional keratosis"}, {"label": "B", "disease": "Oral lichen planus"}, {"label": "C", "disease": "Oral hairy leukoplakia"}, {"label": "D", "disease": "Oral squamous cell carcinoma"}], "answer_idx": "C", "symptoms": ["An 80-year-old white woman presented with an asymptomatic white patch affecting the tongue of 1 month’s duration", "She had a 15-year history of oral lichen planus (OLP) managed with betamethasone dipropionate, 0.05%, gel twice daily and clotrimazole troches, 10 mg, 3 times daily as needed for symptomatic OLP flares", "Medical history revealed stage 1A mycosis fungoides/cutaneous T-cell lymphoma affecting the right calf and left thigh managed primarily with halobetasol, 0.05%, cream owing to intolerance of narrowband UV-B therapy and mechlorethamine, 0.016%, gel", "Immunosuppression or history of infectious diseases, including human immunodeficiency virus (HIV), was not reported", "A 1.5 × 1.0-cm nonremovable white, plaquelike lesion was observed on the left lateral tongue (Figure 1A)", "Biopsy specimens were obtained with a 3-mm punch instrument at 3 different sites, which demonstrated similar microscopic findings (Figure 1B and C)", "A, A corrugated, nonremovable white lesion on left lateral tongue", "B, Histologic image showing hyperkeratotic, acanthotic stratified squamous epithelium exhibiting cells with lightly stained cytoplasm (balloon cells) throughout the stratum spinosum and superficial layer (hematoxylin-eosin)", "C, Histologic image showing several epithelial nuclei with peripheral chromatin margination also described as chromatin beading (arrowheads) (hematoxylin-eosin)"], "s1": [0, 1, 2, 3], "s2": [4, 5, 6, 7, 8]} {"key": 731, "questions": "What Is Your Diagnosis?", "options": [{"label": "A", "disease": "Pigmented onychomatricoma"}, {"label": "B", "disease": "Subungual melanoma"}, {"label": "C", "disease": "Pigmented squamous cell carcinoma in situ"}, {"label": "D", "disease": "Pigmented onychopapilloma"}], "answer_idx": "A", "symptoms": ["A man in his 40s presented to the dermatology clinic for evaluation of a new pigmented lesion on his left second toenail", "The lesion first appeared 12 months before presentation and was progressively growing", "This was his first time presenting to a physician for this problem, and no prior treatments had been attempted", "The lesion was asymptomatic", "He was otherwise well and denied any fevers or weight loss", "Clinical examination revealed a dark brown longitudinal pigment band covering the full length of the toenail of the left second digit with corresponding nail thickening over the pigmented band", "The pigment was the same width throughout the band and extended to the proximal nail fold (Figure, A)", "No other nail or skin lesions were found at the time of examination", "A biopsy specimen was obtained from the nail plate and nail matrix and stained with hematoxylin-eosin (Figure, B-D)", "A dark brown longitudinal pigment band on the second toenail with pigment extends to the proximal nail fold (A)", "Histopathologic examination of the specimen with hematoxylin-eosin stain is seen at original magnification of ×40 (B), ×100 (C), and ×200 (D)"], "s1": [0, 1, 2, 3, 4, 7], "s2": [5, 6, 8, 9, 10]} {"key": 732, "questions": "What Is Your Diagnosis?", "options": [{"label": "A", "disease": "Leber hereditary optic neuropathy"}, {"label": "B", "disease": "Friedreich ataxia"}, {"label": "C", "disease": "Autosomal recessive spastic ataxia of Charlevoix-Saguenay"}, {"label": "D", "disease": "Complicated hereditary spastic paraplegia"}], "answer_idx": "C", "symptoms": ["A 16-year-old girl presented to our ataxia unit with a report of a peculiar finding in fundoscopy", "Her parents were nonconsanguineous, and her family history was unremarkable", "She had unsteadiness and frequent falls at gait initiation since age 16 months", "There was a progressive worsening of her gait difficulties, and she complained of loss of balance", "On examination, she had saccadic pursuit, subtle appendicular ataxia, and a spastic gait with a wide base", "She could not stand or walk with her feet in the tandem position", "Her muscle strength was normal", "Achilles tendon reflexes and plantar responses were absent, and vibration sense was decreased in her toes and ankles", "Visual acuity, color vision, and visual fields were normal", "Fundoscopy revealed increased visibility of retinal nerve fibers, hiding the contours of retinal vessels bilaterally", "Optical coherence tomography (OCT) showed increased thickness in the retinal nerve fiber layer (RNFL) (Figure 1)", "Ophthalmological findings include color fundus photograph of the right eye (A) with increased visibility of the retinal nerve fiber layer, which hides retinal vessel contour", "an optical coherence tomographic vertical B-scan revealing a markedly thickened retinal nerve fiber layer (asterisks) (B), captured along the green line seen on a red-free image of fundus (C)"], "s1": [0, 1, 2, 3, 4, 5, 6, 7], "s2": [8, 9, 10, 11, 12]} {"key": 733, "questions": "What Is Your Diagnosis?", "options": [{"label": "A", "disease": "Left lateral lumbar lipoma"}, {"label": "B", "disease": "Posterior lateral muscular atrophy with pseudohernia"}, {"label": "C", "disease": "Superior lumbar hernia (Grynfeltt-Lesshaft hernia)"}, {"label": "D", "disease": "Postoperative infected seroma"}], "answer_idx": "C", "symptoms": ["A 65-year-old woman was seen at the surgical clinic with reports of chronic back pain and a bulge in her back", "History revealed multilevel cervical and lumbar fusion surgical procedures using anterior, left lateral, and posterior approaches over a 10-year period", "The patient subsequently noted a new, slowly enlarging bulge in her left midback that was bothersome with activities of daily living (Figure, A)", "She denied pain directly overlying the area and was otherwise tolerating a regular diet and having usual bowel function", "Physical examination revealed a soft bulge in the lateral left midback", "There were no apparent associated skin changes, and the palpable mass appeared fixed and nonreducible", "The mass was most apparent with upright posture and was accentuated with a cough impulse, but it was less distinct when examined in the prone position", "Computed tomography of the abdomen and pelvis was obtained (Figure, B)", "Patient presentation (A) and computed tomography of the abdomen and pelvis (B)"], "s1": [0, 1, 2, 3, 4], "s2": [5, 6, 7, 8]} {"key": 734, "questions": "What Is Your Diagnosis?", "options": [{"label": "A", "disease": "Metastatic lung cancer"}, {"label": "B", "disease": "Traumatic vertebral body fracture"}, {"label": "C", "disease": "Thyroid goiter with compressive symptoms"}, {"label": "D", "disease": "A pathological vertebral body fracture"}], "answer_idx": "D", "symptoms": ["A 56-year-old man presented to the emergency department with a 3-week history of worsening neck and shoulder pain", "He had been seen in his primary care physician’s office about 2 weeks prior to emergency department presentation and had been treated for musculoskeletal symptoms", "He had no history of recent trauma", "He admitted to paresthesia and pain radiating down his right arm but denied any weakness", "He had no difficulty with speech or swallowing", "He had no medical comorbidities or prior surgery", "On examination, he was noted to have some cervical midline spine tenderness", "No weakness or sensory deficits were identified", "No palpable lymphadenopathy, swelling, or mass was noted", "A plain radiograph of the neck demonstrated tracheal deviation, and he then underwent computed tomography of the chest (Figure 1A) and magnetic resonance imaging of the cervical and thoracic spine (Figure 1B)", "Computed tomography scan and magnetic resonance image", "A, Coronal computed tomography image of the chest", "B, sagittal magnetic resonance image of the cervical and thoracic spine"], "s1": [0, 1, 2, 3, 4, 5, 6, 7, 8], "s2": [9, 10, 11, 12]} {"key": 735, "questions": "What Is Your Diagnosis?", "options": [{"label": "A", "disease": "Squamous cell carcinoma of the lung"}, {"label": "B", "disease": "Carcinoid tumor"}, {"label": "C", "disease": "Primary malignant mediastinal germ cell tumor"}, {"label": "D", "disease": "Nuclear protein in testis midline carcinoma"}], "answer_idx": "D", "symptoms": ["A young man in his early 20s presented with a 1-day history of pleuritic chest pain and shortness of breath, with no fever or cough", "He smoked approximately 10 cigarettes per day for 4 years", "Results of the physical examination were unremarkable", "Radiography of the chest revealed an ill-defined consolidation in the right lower lung field", "Laboratory investigations were significant for elevated levels of lactate dehydrogenase (273 U/L", "reference range, 135-225 U/L [to convert to microkatals per liter, multiply by 0.0167]), C-reactive protein (125.3 mg/L", "reference range, 0.0-4.9 mg/L [to convert to nanomoles per liter, multiply by 9.524]), and alkaline phosphatase (217 U/L", "reference range, 40-129 U/L [to convert to microkatals per liter, multiply by 0.0167])", "Computed tomography of the chest confirmed the presence of a dense consolidation involving most of the right middle lobe with right hilar and subcarinal lymphadenopathy (Figure, A)", "He underwent endobronchial ultrasonography-guided transbronchial needle aspiration of a fleshy, whitish endobronchial lesion obstructing both uptakes to the medial and lateral segments", "Histologic analysis revealed a poorly differentiated malignant neoplasm (Figure, B)", "Most of the cells were relatively monotonous, with enlarged vesicular nuclei, prominent nucleoli, and a high ratio of nucleus to cytoplasm", "Foci of abrupt squamous differentiation, including keratinization and intercellular bridges, were identified", "Immunohistochemical staining showed tumor cells to be strongly and diffusely positive for keratin AE1/AE3, cytokeratin 5/6, and p63. However, findings were negative for S-100, melan A, placental alkaline phosphatase, CD56, thyroid transcription factor-1, terminal deoxynucleotidyl transferase, CD45, and cytokeratin 7. A positron emission tomography–computed tomography scan showed a fludeoxyglucose-enhanced avid right middle lobe mass, a large right-sided pleural effusion, and fludeoxyglucose-enhanced avid mediastinal and right hilar lymph nodes, with extensive skeletal metastases to both proximal humeri and scapulae, the sacrum, spine, iliac bones, and both hips", "A, Computed tomography of the chest revealed a dense consolidation in the middle lobe of the right lung with right hilar and subcarinal lymphadenopathy", "B, Histologic analysis of the mass revealed a poorly differentiated malignant neoplasm", "Most cells showed enlarged vesicular nuclei, prominent nucleoli, and a high ratio of nucleus to cytoplasm (hematoxylin-eosin", "original magnification ×200)"], "s1": [0, 1, 2, 3, 4, 5, 6, 7, 8], "s2": [9, 10, 11, 12, 13, 14, 15, 16, 17]} {"key": 736, "questions": "What Is Your Diagnosis?", "options": [{"label": "A", "disease": "Laryngeal tuberculosis"}, {"label": "B", "disease": "Laryngeal gout"}, {"label": "C", "disease": "Vocal fold granuloma"}, {"label": "D", "disease": "Laryngeal amyloidosis"}], "answer_idx": "B", "symptoms": ["A cigarette smoker in his 50s with a history of hypertension, chronic kidney disease, colon adenocarcinoma, and hyperuricemia presented with a right vocal fold mass that had been an incidental finding during esophagogastroduodenoscopy", "He denied any symptoms related to the lesion, including dysphonia or dysphagia, and reported no fevers or weight loss", "Rigid stroboscopy revealed normal vocal fold mobility and a nodular cystic-appearing lesion on the posterior inferior border of the right vocal process (Figure, A)", "There was no overlying ulceration, and vocal fold mucosal wave was intact in the adjacent membranous vocal fold", "Suspension microlaryngoscopy incisional biopsy of the right infraglottic lesion was performed in the operating room", "The endotracheal tube was displaced anteriorly with the laryngoscope", "Intraoperatively, there was submucosal white caseous debris in the substance of the lesion", "The surgical specimen was sent for histopathologic analysis (Figure, B)", "Cultures were analyzed for the presence of fungus, bacteria, and mycobacteria", "Postoperatively, the patient was doing well and continued to have no symptoms of dysphagia or dysphonia"], "s1": [0, 1, 9], "s2": [2, 3, 4, 5, 6, 7, 8]} {"key": 737, "questions": "What Is Your Diagnosis?", "options": [{"label": "A", "disease": "Hodgkin lymphoma"}, {"label": "B", "disease": "Tubercular lymphadenitis"}, {"label": "C", "disease": "Kimura disease"}, {"label": "D", "disease": "Angiolymphoid hyperplasia with eosinophilia"}], "answer_idx": "C", "symptoms": ["A farmer in his 50s presented with bilateral periauricular swelling that began 6 months prior (Figure 1)", "The swelling was mild at onset and gradually progressive", "There was no history of associated pain, but he reported occasional pruritus over the swelling", "There was no history of fever, weight loss, or loss of appetite", "He denied any history of similar swelling elsewhere in his body", "General physical and systemic examination had normal results", "On examination of the swelling on the right side, there was a diffuse swelling around the pinna measuring approximately 6 cm over the preauricular, postauricular, and infra-auricular region", "It was a nontender, soft swelling without fluctuation and abnormal pulsations", "There were similar findings in the left side", "Mouth opening was adequate, secretions from both parotid ducts were normal, and facial nerve function was intact on both sides", "He had undergone fine-needle aspiration cytology and incisional biopsy", "The results of both were consistent with reactive lymphadenitis", "He was prescribed oral antibiotics for 1 week but showed no improvement", "A contrast-enhanced computed tomographic scan of the head and neck showed bilateral multiple enhancing lymph nodes in the postauricular region, the largest measuring 16 × 11 mm2 on the right side and 15.4 × 10.6 mm2 on the left, along with bilateral intraparotid nodes, the largest measuring 16.4 × 10.2 mm2 on the right and 15.3 × 10.7 mm2 on the left side", "His serum eosinophil count was slightly elevated, and his erythrocyte sedimentation rate was 15 mm/h", "However, the IgE level was not measured", "He was then scheduled for excisional biopsy at our center", "The biopsy specimen included skin and subcutaneous tissue along with 2 underlying lymph nodes", "Clinical photographs showing diffuse swelling of the preauricular, infra-auricular, and postauricular region"], "s1": [0, 1, 2, 3, 4, 6, 7, 8, 9, 11, 13, 18], "s2": [5, 10, 12, 14, 15, 16, 17]} {"key": 738, "questions": "What Is Your Diagnosis?", "options": [{"label": "A", "disease": "Poorly undifferentiated carcinoma"}, {"label": "B", "disease": "Ectopic thymoma"}, {"label": "C", "disease": "Follicular dendritic cell sarcoma"}, {"label": "D", "disease": "Peripheral nerve sheath tumor"}], "answer_idx": "C", "symptoms": ["A man in his 40s with Poland syndrome presented with a 6-month history of left throat pain and enlarging oropharyngeal mass", "The patient had been seen repeatedly for odynophagia that persisted despite multiple courses of antibiotics, reflux medications, and steroids prescribed at clinics and emergency departments", "Over the course of 5 months, he underwent a tonsillectomy and 3 separate biopsies at multiple institutions", "All showed inconclusive results, and the mass progressively enlarged", "At presentation for the present report, physical examination showed a 2-cm, well-mucosalized, left oropharyngeal mass not extending to the hard palate (Figure, A)", "Computed tomography (CT) revealed a 58-mm parapharyngeal mass lateralizing and narrowing the left carotid artery against the styloid process (Figure, B)", "A CT-guided fine-needle aspiration biopsy was performed but again was nondiagnostic", "Further confounding the clinical picture, a pacemaker was required for multiple episodes of sinus asystole due to mass effect on the carotid body", "To obtain the diagnosis of the parapharyngeal mass, the patient had a transcervical resection with transoral partial glossectomy, palatectomy, and pharyngectomy with reconstruction", "Results of immunohistochemical analysis of the removed tissue were positive for CD4 (weak), CD21, CD23, and vimentin with a low proliferation rate (Figure, C and D)", "A, A 2-cm well-mucosalized left oropharyngeal soft-palate mass anterior to the tonsillar bed with surrounding erythema", "B, Left parapharyngeal mass measuring 4.3 × 2.7 × 5.8 cm and lateralizing the left carotid artery", "C, Fascicular, storiformed architecture with spindly, polygonal-shaped cells with eosinophilic cytoplasm and indistinct borders (original magnification ×10)", "D, Positive findings for CD21 (left", "original magnification ×10) and CD23 (right", "original magnification ×20)"], "s1": [0, 1, 2, 3, 4, 5, 6, 7, 8], "s2": [9, 10, 11, 12, 13, 14, 15]} {"key": 739, "questions": "What Is Your Diagnosis?", "options": [{"label": "A", "disease": "Lichen sclerosus et atrophicus"}, {"label": "B", "disease": "Acrodermatitis chronica atrophicans"}, {"label": "C", "disease": "Acquired palmoplantar keratoderma"}, {"label": "D", "disease": "Sclerotylosis (Huriez syndrome)"}], "answer_idx": "A", "symptoms": ["A woman in her 60s presented for initial dermatologic evaluation of a 2-year history of a pruritic and sometimes burning hyperpigmented eruption on the bilateral palms", "The eruption had gradually spread from the left palm to the right palm and then focally onto the bilateral volar aspect of the wrists in recent months (Figure, A and B)", "She had not experienced involvement of the plantar surfaces", "No additional concerns were reported, and there was no family history of similar lesions", "There was no known history of travel in relation to onset of the lesions", "Limited clinical examination revealed well-marginated brown atrophic patches on the bilateral palms extending focally onto the volar aspect of the wrist and centrally surmounted by spiny keratotic pits in the palmar creases", "Two punch biopsy specimens were obtained for diagnostic clarification (Figure, C and D)", "A, An atrophic hyperpigmented plaque with spiny papules in the palmar creases", "B, Similar features extending onto the volar wrist", "C and D, Histopathologic images (hematoxylin-eosin, original magnification ×100)"], "s1": [0, 1, 2, 3, 4], "s2": [5, 6, 7, 8, 9]} {"key": 740, "questions": "What Is Your Diagnosis?", "options": [{"label": "A", "disease": "Trichodysplasia spinulosa"}, {"label": "B", "disease": "Lichen spinulosus"}, {"label": "C", "disease": "Demodecosis"}, {"label": "D", "disease": "Disseminated keratosis pilaris"}], "answer_idx": "A", "symptoms": ["A woman in her 70s presented with a 1-year history of a facial cutaneous eruption initially affecting malar cheeks and eyebrows", "Subsequently it spread to involve nose, chin, upper trunk, and extremities", "The lesions were mildly pruritic", "Her medical history included a cadaveric renal transplant 2 years earlier for end-stage renal failure", "Immunosuppressant medications included mycophenolate mofetil, 200 mg, twice daily and tacrolimus, 8 mg, twice daily", "Tacrolimus levels were therapeutic", "Physical examination revealed multiple 1-mm flesh-colored follicular papules and keratin spines against a diffuse erythematous background affecting the face (Figure, A and B) and upper trunk", "Her scalp and eyebrow hairs were unremarkable", "Serology results were unremarkable", "Results of skin biopsy from the right ear demonstrated dilatation and keratotic plugging of the hair infundibula with marked dystrophy and expansion of the inner root sheath", "The inner root sheath cells were enlarged with irregular trichohyaline granules and apoptotic cells with abrupt cornification without formation of a granular layer (Figure, C)", "Immunohistochemical analysis for SV40 polyomavirus was positive (Figure, D)", "Photographs of (A) forehead and (B) nose depicting multiple follicular papules and keratin spines against a diffuse erythematous background", "C, Hematoxylin-eosin staining (original magnification ×40)", "D, SV40 polyomavirus stain (original magnification ×40)"], "s1": [0, 1, 2, 6, 7, 12], "s2": [3, 4, 5, 8, 9, 10, 11, 13, 14]} {"key": 741, "questions": "What Is Your Diagnosis?", "options": [{"label": "A", "disease": "Granuloma annulare"}, {"label": "B", "disease": "Erythema papulosa semicircularis recidivans"}, {"label": "C", "disease": "Deep erythema annulare centrifugum"}, {"label": "D", "disease": "Erythema chronicum migrans"}], "answer_idx": "B", "symptoms": ["A healthy woman in her 50s presented with a large annular erythematous plaque on her lower back that had appeared in June (1 month previously) and had centrifugally expanded", "She complained about mild pruritus that had not responded to conventional antihistamine therapy or topical steroids", "She could not recall an arthropod bite prior to the skin eruption and denied having traveled outside Spain in the past few years", "She reported a 20-year history of the same annular plaques recurring on the trunk every single year by the beginning of summer and spontaneously resolving by the beginning of autumn", "Such lesions had never appeared on the face, hands, or feet", "She did not relate these lesions to any particular hobby or application of any skin care product", "Physical examination revealed a large, 35-cm semicircular erythematous plaque on her lower back with central clearing and a distinctive, well-demarcated papular border (Figure, A and B)", "After obtaining informed consent, 2 biopsy specimens were taken from the papular border of the plaque for both hematoxylin-eosin stain and direct immunofluorescence (Figure, C)", "A, Large erythematous semicircular plaque with central postinflammatory hyperpigmentation on the lower back", "B, Absence of epidermal changes, mild edema of the papillary dermis, and a superficial sparse perivascular inflammatory infiltrate", "C, Inflammatory infiltrate composed mainly by lymphocytes and few neutrophils", "D, Follow-up visit showing clinical resolution in October"], "s1": [0, 1, 2, 3, 4, 5], "s2": [6, 7, 8, 9, 10, 11]} {"key": 742, "questions": "What Is Your Diagnosis?", "options": [{"label": "A", "disease": "Neuromyelitis optica (Devic disease)"}, {"label": "B", "disease": "Amyloidosis"}, {"label": "C", "disease": "Susac syndrome"}, {"label": "D", "disease": "Cobalt toxicity"}], "answer_idx": "D", "symptoms": ["A 63-year-old man with a history of well-controlled hypertension, lumbar stenosis, and hip arthroplasty with revisions presented with bilateral loss of vision and hearing for several weeks", "He also reported a history of painful paresthesias at his feet beginning about 1 year prior", "On examination, his visual acuity was initially 20/80 OD and 20/200 OS", "it deteriorated over the next 2 months to finger counting bilaterally", "The Weber test gave a nonlateralizing result, and bone conduction hearing was absent bilaterally", "There was length-dependent loss of vibratory sensation and absent reflexes at both lower extremities", "Magnetic resonance imaging demonstrated abnormal enhancement of bilateral optic nerves (Figure, A)", "Optical coherence tomography and fluorescein angiogram were normal", "Nerve conduction studies showed mild sensory axonal polyneuropathy", "Laboratory tests were notable for elevated thyrotropin (63 mlU/ml), low levels of free thyroxine (T4) (0.2 ng/dL), and elevated levels of cerebrospinal fluid protein (94 mg/dL)", "Cerebrospinal fluid IgG index, oligoclonal bands, aquaporin 4 antibody, paraneoplastic panel, and protein electrophoresis test results were all normal", "He was treated empirically with high-dose steroids and plasmapheresis, with no improvement in symptoms", "A, Axial postgadolinium, fat-suppressed, T-1–weighted magnetic resonance image of the orbits, demonstrating left greater than right enhancement of the optic nerves (arrowheads)", "The enhancement is centrally located within the optic nerves on coronal sections (not shown)", "No other lesions were seen in the brain", "B, Axial T-2–weighted magnetic resonance image of the right hip, demonstrating susceptibility artifact of orthopedic hardware, as well as soft tissue heterotopic ossification (corroborated by plain-film radiographs), consistent with pseudotumor of metallosis with decompression into the right hip joint space (blue asterisk), the subiliac bursa (red asterisk), greater trochanter bursa (yellow asterisk) extending into the gluteus maximus muscle (arrowheads), and into a potential space between the tensor fascia lata and the rectus femoris (black asterisk)", "Four months after initial presentation, the patient reported new symptoms of chest tightness and dyspnea on exertion", "An echocardiogram showed a left ventricle ejection fraction of 20% to 25%, which had been 55% to 60% at the last test 2 years prior", "At 5 months after initial presentation, the patient developed acute worsening of dyspnea and was admitted for respiratory distress", "He was found to have nonischemic cardiomyopathy and died on hospital day 13."], "s1": [0, 1, 2, 3, 4, 5, 6, 7, 8, 9, 10, 11, 12, 13, 14], "s2": [15, 16, 17, 18, 19]} {"key": 743, "questions": "What Is Your Diagnosis?", "options": [{"label": "A", "disease": "Acute appendicitis"}, {"label": "B", "disease": "Colonic ischemia"}, {"label": "C", "disease": "Diverticulitis with perforation"}, {"label": "D", "disease": "Infectious colitis"}], "answer_idx": "B", "symptoms": ["A woman in her early 50s with a history of polycystic ovarian syndrome, diabetes, stroke, and coronary artery disease that required coronary artery bypass graft 6 years earlier presented to the emergency department with 12 hours of sudden-onset, constant, right lower quadrant abdominal pain associated with vomiting and melena", "She denied any hematochezia or history of peptic ulcer disease", "She had no other surgical history and denied alcohol use or smoking", "On examination, she was hypertensive (blood pressure, 185/85 mm Hg) although not tachycardic and was afebrile", "She was locally tender to palpation in the right lower quadrant with rebound and voluntary guarding", "Her abdomen was not tympanitic", "The patient had no abdominal scars, hernias, or palpable masses", "Laboratory results were significant for the following: leukocyte count, 23 600/µL (to convert to ×109/L, multiply by 0.001) with neutrophilia (94%)", "blood urea nitrogen level, 28 mg/dL (to convert to millimoles per liter, multiply by 0.357)", "creatinine concentration, 0.01 mg/dL (to convert to micromoles per liter, multiply by 88.4)", "lipase concentration, 195 U/L (to convert to microkatals per liter, multiply by 0.0167)", "and lactate concentration, 2.02 mg/dL (to convert to millimoles per liter, multiply by 0.111)", "Electrolyte levels and liver function test results were within normal limits", "An electrocardiogram showed normal sinus rhythm", "Computed tomography of the abdomen and pelvis with oral and intravenous contrast enhancement was performed (Figure)", "Axial cross-sectional computed tomograms of the patient’s abdomen and pelvis with oral and intravenous contrast enhancement show segmental mural thickening of the anterior cecum with wall hypoenhancement and pneumatosis, adjacent fat stranding, and a lucency medial to the cecum (A) and bilateral renal hypodensities (B)"], "s1": [0, 1, 2, 3, 4, 5, 6], "s2": [7, 8, 9, 10, 11, 12, 13, 14, 15]} {"key": 744, "questions": "What Is Your Diagnosis?", "options": [{"label": "A", "disease": "Rupture of an infected endometriotic implant"}, {"label": "B", "disease": "Inflamed diverticulum"}, {"label": "C", "disease": "Perforation by foreign bodies"}, {"label": "D", "disease": "Malignant growth with unknown primary tumor"}], "answer_idx": "C", "symptoms": ["A postmenopausal woman in her mid-50s (gravida 2, aborta 2) with a known history of endometriosis and adenomyosis presented to the emergency department with severe abdominal pain that she rated 10 of 10 in intensity and which had developed suddenly 5 days before presentation", "The abdominal pain was associated with diarrhea, nausea, and abdominal bloating", "On examination, the patient was afebrile and her abdomen was soft and nondistended without guarding or rebound but tender in the left lower quadrant", "The patient had a palpable nodule in the posterior vaginal fornix and fullness in the left lower quadrant of the abdomen on bimanual examination", "Laboratory investigations had normal findings with the exception of an elevated cancer antigen 125 level of 158 U/mL (to convert to kilounits per liter, multiply by 1.0)", "Contrast-enhanced computed tomography (CT) revealed multiple abnormal findings that included a bulky uterus with a thickened myometrium, endometriomas, right pleural effusion, pelvic peritonitis and ascites, enlarged lymph nodes, and solid extracolonic implants in the omentum caudal to the transverse colon (Figure, A) and in the cul-de-sac adjacent to the upper rectum (Figure, B)", "Subsequent magnetic resonance imaging (MRI) showed 2 areas of focus corresponding with the extracolonic implants"], "s1": [0, 1, 2], "s2": [3, 4, 5, 6]} {"key": 745, "questions": "What Is Your Diagnosis?", "options": [{"label": "A", "disease": "Brown recluse envenomation"}, {"label": "B", "disease": "Pyoderma gangrenosum"}, {"label": "C", "disease": "Leukemia cutis"}, {"label": "D", "disease": "Mucormycosis"}], "answer_idx": "D", "symptoms": ["On day 15 of induction therapy with intravenous cytarabine, vincristine sulfate, dexamethasone, pegaspargase, and intrathecal methotrexate, a 3-year-old girl with B-cell acute lymphoblastic leukemia presented to the oncology department with an asymptomatic skin lesion", "No known skin trauma occurred", "The patient was afebrile, and vital signs were stable", "On examination, a 4.0 × 2.0-cm, oval-shaped violaceous to black plaque with a central hemorrhagic bulla and surrounding hyperemia was noted on the left lower back (Figure 1)", "A complete blood cell count revealed a white blood cell count of 1800/μL (to convert to ×109/L, multiply by 0.001), with 2% neutrophils (absolute neutrophil count, 36/μL [to convert to ×109/L, multiply by 0.001]), hemoglobin level of 8.2 g/dL (to convert to grams per liter, multiply by 10), and platelet count of 10 × 103/μL (to convert to ×109/L, multiply by 1)", "Her blood glucose level was 326 mg/dL (to convert to millimoles per liter, multiply by 0.0555)", "The patient was administered broad-spectrum antibiotics empirically, and a dermatologist was consulted", "Two punch biopsy specimens from the lower back plaque were obtained and sent for histologic analysis and tissue culture (bacteria, atypical mycobacteria, and fungus)", "Violaceous to black plaque with central hemorrhagic bulla on the left lower back of the patient"], "s1": [0, 6, 7], "s2": [1, 2, 3, 4, 5, 8]} {"key": 746, "questions": "What Is Your Diagnosis?", "options": [{"label": "A", "disease": "Fixed drug eruption"}, {"label": "B", "disease": "Cutaneous candidiasis"}, {"label": "C", "disease": "Cytomegalovirus ulcers"}, {"label": "D", "disease": "Genital herpes simplex virus infection"}], "answer_idx": "C", "symptoms": ["A woman in her 80s presented to hospital with somnolence and fatigue of 2 days’ duration", "Medical history included lower urinary tract symptoms of 1-week duration for which she was prescribed cotrimoxazole with no improvement", "She also reported progressively worsening vulvar and buttock pain of 5 days’ duration", "The patient was admitted to the intensive care unit 3 weeks prior to presentation for pneumonia and remained hospitalized for 10 days", "She was not sexually active and had no medical history of sexually transmitted infections", "Anogenital examination revealed vulvar (Figure 1A) and buttock ulcers (Figure 1B) tender to palpation", "Initial blood workup showed leukopenia with a white blood cell count of 2400 cells/μL (to convert to cells/L, multiply by 106), a differential of 72% neutrophils, and a hemoglobin level of 8.5 g/dL with a hematocrit differential of 27% (to convert hemoglobin to g/L, multiply by 10.0)", "Cotrimoxazole was discontinued on admission", "A biopsy specimen was obtained from the left vulvar ulcer (Figure 1C)", "A and B, Clinical photographs showing vulvar and buttock ulcers at the time of presentation", "C, Hematoxylin-eosin stain (original magnification ×40)"], "s1": [0, 1, 3, 4, 6], "s2": [2, 5, 7, 8, 9, 10]} {"key": 747, "questions": "What Is Your Diagnosis?", "options": [{"label": "A", "disease": "Chronic cutaneous lupus erythematosus"}, {"label": "B", "disease": "Granuloma faciale"}, {"label": "C", "disease": "Neutrophilic sebaceous adenitis"}, {"label": "D", "disease": "Rosacea-like dermatitis"}], "answer_idx": "C", "symptoms": ["A 21-year-old woman with no significant medical history presented with a 2-year history of asymptomatic, facial redness that flared in sunlight", "She had been previously treated with doxycycline, 100 mg/d, and topical sulfacetamide with no effect", "On examination, on the cheeks and nasal bridge there were multiple erythematous, annular plaques with focal areas of atrophy", "Her medications included only an oral birth control pill", "Serum chemical analyses and complete blood cell count showed no abnormalities, and anti-Ro, anti-La, and qualitative ANA antibodies were negative", "A clinical diagnosis was made, and over the next 9 months, treatments with minocycline, desonide, topical metronidazole, 0.75%, cream and azelaic acid, 15%, were unsuccessful, and the plaques developed more scarring and atrophy (Figure, A)", "In addition, the patient began to develop a burning sensation in these areas", "A punch biopsy specimen was obtained and submitted for histopathologic review (Figure, B and C)"], "s1": [0, 1, 3, 4], "s2": [2, 5, 6, 7]} {"key": 748, "questions": "What Is Your Diagnosis?", "options": [{"label": "A", "disease": "Tinea corporis"}, {"label": "B", "disease": "Erythema marginatum"}, {"label": "C", "disease": "Annular erythema of infancy"}, {"label": "D", "disease": "Erythema migrans"}], "answer_idx": "C", "symptoms": ["A young child presented to clinic with a 6-month history of an asymptomatic expanding erythematous eruption on the lower legs, abdomen, and buttocks (Figure, A)", "The eruption would wax and wane, with each episode lasting for approximately 1 week, without residual pigmentation", "Clinical examination was significant for faint, nonscaling annular serpiginous, erythematous plaques with central clearing, and barely elevated borders", "Test results for erythrocyte sedimentation rate, complete blood cell count, antinuclear antibodies, anti-Ro/SSA and anti-La/SSB antibodies, and Borrelia burgdorferi antibodies were within normal limits", "There was no family history of autoimmune disease, and the child had not experienced any fevers or exposures to ticks or other arthropods", "A punch biopsy specimen from the left thigh was obtained, and histopathological analysis was subsequently performed (Figure, B and C)", "A, Annular serpiginous plaques with areas of central clearing on the patient’s right thigh and buttocks", "B and C, Histopathologic images (hematoxylin-eosin)", "B, Perivascular and inflammatory infiltrate", "C, Perivascular and inflammatory infiltrate that includes eosinophils"], "s1": [0, 1, 2, 6], "s2": [3, 4, 5, 7, 8, 9]} {"key": 749, "questions": "What Is Your Diagnosis?", "options": [{"label": "A", "disease": "Myasthenia gravis"}, {"label": "B", "disease": "Wernicke encephalopathy"}, {"label": "C", "disease": "Miller Fisher variant of Guillain-Barre syndrome"}, {"label": "D", "disease": "Anti-Ma2 encephalitis"}], "answer_idx": "D", "symptoms": ["A 70-year-old woman with long-term tobacco use presented with 2 weeks of bilateral ptosis and binocular horizontal diplopia", "She denied weakness, difficulty swallowing, or breathing", "On initial examination, she had a normal mental status, no pupillary abnormalities, normal visual acuity, and full visual fields", "She had moderate exotropia, severe bilateral ptosis, impaired bilateral adduction and vertical ductions, and impaired convergence", "Her abduction and vertical oculocephalic reflexes were intact", "She had full strength and normal sensation and reflexes but had a slightly wide-based and unsteady gait", "Over the subsequent weeks, she experienced worsening gait, falls, inattention, and short-term memory loss", "Her brain magnetic resonance imaging (MRI) with contrast results were unremarkable", "The test results for acetylcholine receptor antibodies and the muscle-specific kinase antibody were negative", "Her electrodiagnostic test results with repetitive stimulation and single fiber electromyography were unremarkable and there was no evidence of neuropathy", "The test results for thyrotropin, thiamine level, Lyme antibodies, rapid plasma reagin, and the serum ganglioside antibody panel were unremarkable", "A cerebrospinal fluid (CSF) analysis revealed a nucleated cell count of 6/μL (to convert to ×109 liters, multiply by 0.001) with 88% lymphocytes (to convert to the proportion of 1.0, multiply by 0.01), a red blood cell count of 10/μL, a protein level of 0.087 g/dL (to convert to grams per liter, multiply by 10), and glucose level of 63 mg/dL (to convert to millimoles per liter, multiply by 0.0555)", "Serum paraneoplastic antibody testing results were negative for anti-Hu, Ri, Yo, and voltage-gated potassium channel complex (VGKCC) antibodies, and the CSF had negative results for anti-VGKCC, voltage-gated calcium channel, Hu, Ri, Yo, and amphiphysin antibodies", "A repeated brain MRI 3 weeks later revealed an interval development of T2-weighted fluid-attenuated inversion recovery hyperintensities in the right thalamus, inferior temporal gyrus, and hippocampus (Figure)", "Repeated brain magnetic resonance imaging 3 weeks after the patient’s initial presentation: T2-weighted fluid-attenuated inversion recovery hyperintensities (arrowheads) in the right inferior temporal gyrus (A) and right thalamus (B)"], "s1": [0, 3, 5, 6, 13, 14], "s2": [1, 2, 4, 7, 8, 9, 10, 11, 12]} {"key": 750, "questions": "What Is Your Diagnosis?", "options": [{"label": "A", "disease": "Teratoma"}, {"label": "B", "disease": "Bronchopulmonary sequestration"}, {"label": "C", "disease": "Congenital pulmonary airway malformation"}, {"label": "D", "disease": "Carcinoma"}], "answer_idx": "B", "symptoms": ["A pregnant woman in her late 30s had normal findings on anatomic ultrasonography at 19 weeks’ gestation", "Follow-up ultrasonography at 32 weeks’ gestation identified fetal hydrops", "The fetal ultrasonogram showed a large left-sided hydrothorax, mediastinal shift, and ascites", "After ruling out infectious causes, an ultrasonography-guided thoracoamniotic shunt was placed successfully at 32 weeks’ gestation", "Results of follow-up ultrasonography on postoperative day 3 showed improvement in the fetal hydrothorax and resolution of hydrops and identified a feeding vessel originating from the descending aorta supplying a lung mass (Figure 1A)", "A, Feeding vessel (arrowhead) from descending aorta identified on ultrasonography at 32 weeks’ gestation", "B, Feeding vessel (arrowhead) to lung mass (outlined) over left hemidiaphragm on computed tomography at age 9 months", "Owing to premature rupture of membranes, the mother delivered at 33 weeks", "The neonate showed no respiratory distress after birth, and the shunt was removed", "On the day of birth, radiographic findings showed a left-sided pleural effusion", "Ongoing pleural effusion caused respiratory distress, and on day 3 of life, a temporary thoracostomy tube was placed", "During the following weeks, the thoracostomy tube output decreased, the tube was removed, and the patient was discharged home", "At 9 months of age, follow-up computed tomography confirmed a lung mass with a feeding vessel (Figure 1B), and the patient returned for elective thoracoscopic resection"], "s1": [0, 1, 3, 7, 9, 10, 11], "s2": [2, 4, 5, 6, 8, 12]} {"key": 751, "questions": "What Is Your Diagnosis?", "options": [{"label": "A", "disease": "Ganglion cyst of the ankle joint"}, {"label": "B", "disease": "Tenosynovial giant cell tumor"}, {"label": "C", "disease": "Ewing sarcoma (soft-tissue metastasis)"}, {"label": "D", "disease": "Synovial sarcoma"}], "answer_idx": "B", "symptoms": ["An adolescent girl presented with a 3-month history of a growing, painless mass in the right thigh", "A magnetic resonance imaging (MRI) scan showed an inhomogeneous nodule in the subcutaneous layer with a largest diameter of approximately 5 cm", "No enlarged lymph nodes were detectable in the groin", "Results of a complete blood cell count and levels of alkaline phosphatase, lactic dehydrogenase, and C-reactive protein were unremarkable", "An ultrasonography-guided core needle biopsy was performed, and histopathologic evaluation showed the aspects of a small round-cell tumor, which was confirmed to be a Ewing sarcoma by immunohistochemical and molecular analysis (with positivity for the EWS-FLI1 fusion gene [OMIM 133450])", "The patient then underwent disease staging with computed tomography (CT) of the chest and whole-body fluorodeoxyglucose positron emission tomography (FDG-PET)", "The CT findings were negative for secondary lesions", "the FDG-PET scan demonstrated, as expected, high uptake in the right thigh (maximum standard uptake variable [SUVmax], 8.4) at the site of the subcutaneous tumor and at a second lesion in the left ankle joint (SUVmax, 12.0)", "The patient reported vague pain around the left ankle but no functional limitations and no history of trauma at that site", "A radiograph of the ankle did not show calcifications within the soft-tissue lesion", "An MRI scan of the ankle showed a 1.5-cm soft tissue mass in the posterior recess of the ankle joint with a low to intermediate signal in T1- and T2-weighted sequences and moderate contrast enhancement throughout the lesion (Figure 1)", "Magnetic resonance imaging fast spin echo of the left ankle showing the soft-tissue mass in the posterior recess", "The sagittal T1-weighted image shows an isointense, bean-shaped mass (arrowhead) (1.5 × 0.7 cm) (A), whereas the sagittal T2-weighted image shows the isointense signal of the mass (arrowhead) (B)"], "s1": [0, 1, 2, 3, 4, 5, 6], "s2": [7, 8, 9, 10, 11, 12]} {"key": 752, "questions": "What Is Your Diagnosis?", "options": [{"label": "A", "disease": "Desmoid-type fibromatosis"}, {"label": "B", "disease": "Neurofibroma"}, {"label": "C", "disease": "Collagenous fibroma"}, {"label": "D", "disease": "Solitary fibrous tumor"}], "answer_idx": "C", "symptoms": ["An otherwise healthy woman in her 60s presented with a right neck and facial lesion that had been growing slowly for more than 20 years", "She was a former smoker and had unilateral conductive hearing loss associated with the mass", "On examination, a large, nonmobile, and firm tumor extended from the postauricular region to the parotid gland, scalp, and a portion of the neck", "There was no palpable lymphadenopathy", "On otoscopy, the growth occluded view of the tympanic membrane", "Computed tomographic imaging (Figure, A and B) demonstrated a mass emanating from the parapharyngeal space, measuring approximately 10 × 7.3 × 1.5 cm", "There was no radiographic evidence of osseous extension or destruction", "The patient underwent a staged procedure, including a radical resection of the scalp and neck mass, followed by a transcervical and transparotid approach to the parapharyngeal space", "The tumor was noted to stretch the pes anserinus to 4 times its normal length and had to be resected off the internal carotid artery, as well as the fascia overlying the medial pterygoid muscle and the tympanomastoid suture", "The patient recovered well with only temporary marginal mandibular weakness", "Microscopy (Figure, C) demonstrated a well-circumscribed, paucicellular spindle cell neoplasm with abundant myxocollagenous matrix", "The scattered tumor cells were spindled to stellate shaped and had the morphology of myofibroblasts", "Rare vessels were identified, and the tumors focally infiltrated subcutaneous fat and skeletal muscle", "A and B, Contrasted computed tomographic imaging demonstrating a mass approximately 10 × 7.3 × 1.5 cm that included the retroauricular, superficial, and deep parotid space, pterygopalatine fossa, and masticator spaces, with punctate and amorphous calcifications throughout the lesion", "C, Histopathological findings with high-powered view of hematoxylin-eosin staining, revealing sparse spindle-shaped tumor cells in abundant myxocollagenous matrix (original magnification ×40)"], "s1": [0, 1, 2, 3, 4, 5, 6, 7, 8, 9], "s2": [10, 11, 12, 13, 14]} {"key": 753, "questions": "What Is Your Diagnosis?", "options": [{"label": "A", "disease": "Fournier gangrene"}, {"label": "B", "disease": "Pyoderma gangrenosum"}, {"label": "C", "disease": "Severe irritant contact dermatitis (chemical burn)"}, {"label": "D", "disease": "Acquired acrodermatitis enteropathica"}], "answer_idx": "B", "symptoms": ["A healthy 8-month-old male infant was admitted for management of a rapidly progressive, painful anogenital ulcer", "The lesion had developed acutely over 48 hours, starting as symmetric pink patches leading to pustules, erosions, and ulceration", "Associated symptoms included generalized fatigue, but no fevers, chills, or other systemic symptoms", "The child’s parents denied any new topical exposures or medications", "The child was born via vaginal delivery at full term", "Pregnancy was complicated by gestational diabetes", "The child had no major medical issues and met all developmental milestones", "There was no relevant family medical history", "Broad-spectrum intravenous antibiotics were initiated on admission", "Dermatology was consulted prior to scheduled debridement and skin grafting", "Physical examination revealed a well-appearing infant with normal vital signs", "Extending from the gluteal cleft to just medial to the scrotum, there was a symmetric, well-defined ulcer with a cribriform base, central necrosis, and rolled, pink-to-violaceous, undermined border (Figure, A)", "Initial laboratory workup was significant for leukocytosis (22 000/mm3) and thrombocytosis (764 000/mm3)", "Serum zinc and alkaline phosphatase were within normal limits", "A punch biopsy of the edge of the ulcer was obtained and stained with hematoxylin-eosin (Figure, B and C)", "Microbial stains were negative", "Tissue cultures for anaerobic and/or aerobic, mycobacterial, and fungal organisms were all negative, though wound cultures grew Escherichia coli, Klebsiella oxytoca, and Enterococcus faecalis", "Viral cultures were negative", "A, Clinical photograph shows a symmetric, well-defined ulcer with a cribriform base, central necrosis, and rolled, pink-to-violaceous, undermined border extending from the gluteal cleft to just inferior and medial to the scrotum", "B, Histopathology demonstrating an ulcer with surrounding mild epidermal hyperplasia and a neutrophil predominant perivascular and interstitial infiltrate", "C, Histopathology demonstrating an ulcer with surrounding mild epidermal hyperplasia and a neutrophil predominant perivascular and interstitial infiltrate"], "s1": [0, 1, 2, 8, 9, 11, 16, 18], "s2": [3, 4, 5, 6, 7, 10, 12, 13, 14, 15, 17, 19, 20]} {"key": 754, "questions": "What Is Your Diagnosis?", "options": [{"label": "A", "disease": "Borderline-borderline leprosy"}, {"label": "B", "disease": "Mycosis fungoides"}, {"label": "C", "disease": "Disseminated annular granuloma"}, {"label": "D", "disease": "Sarcoidosis"}], "answer_idx": "A", "symptoms": ["A woman in her 50s with a 2-year history of disseminated dermatosis involving the trunk and limbs was seen in our dermatology department", "The dermatosis was characterized by polymorphic erythematous macules and plaques with elevated and indurated borders without evanescence to pressure (Figure, A and B)", "She was referred from another clinic with the presumptive diagnosis of urticarial vasculitis", "Previous treatment included antihistamines and oral corticosteroids for several months without improvement", "Dysesthesia was recorded on her lesions", "Results from complete blood cell count, erythrocyte sedimentation rate, and blood chemistry analyses were normal, as was a thyroid profile", "A punch biopsy specimen from a trunk plaque was obtained, and histopathological analysis was performed (Figure, C and D)", "A, Polymorphic erythematous macules and plaques with elevated and indurated borders without in vitro pressure evanescence were noticed during consultation on patient’s back", "B, Polymorphic erythematous macules and plaques with elevated and indurated borders without in vitro pressure evanescence on the trunk and abdomen were present", "C, Histopathological analysis revealed a chronic dermic granulomatous process, as well as (D) a chronic dermic nonencapsulated granulomatous infiltrate with a perineural distribution formed by mature epitheloid cells and encapsulated by lymphocytes"], "s1": [0, 1, 2, 3, 7, 8], "s2": [4, 5, 6, 9]} {"key": 755, "questions": "What Is Your Diagnosis?", "options": [{"label": "A", "disease": "Leukemia cutis"}, {"label": "B", "disease": "Primary cutaneous anaplastic large cell lymphoma"}, {"label": "C", "disease": "Cutaneous blastic plasmacytoid dendritic cell neoplasm"}, {"label": "D", "disease": "Transformed mycosis fungoides and Sézary syndrome"}], "answer_idx": "C", "symptoms": ["A white man in his 50s with a medical history of myelodysplastic syndrome (MDS) refractory anemia with excess blasts, type 2 (RAEB-2) or MDS with excess blasts, type 2 (MDS-EB-2) RAEB-2 reported a 2-month history of nonpruritic infiltrative skin lesions on the left upper chest, which disseminated to the entire chest, back, and right facial areas (Figure, A)", "The patient was initially diagnosed with MDS 3 years ago and treated with azacitidine followed by CLAG (cladribine, cytarabine, granulocyte-colony stimulating factor)", "Posttreatment bone marrow biopsy findings at that time showed no evidence of disease", "His physical examination on this presentation showed multiple dark brown cutaneous papules measuring at 3.0 to 5.0 cm (Figure, A), but the rest of the examination results were negative for hepatosplenomegaly or lymphadenopathy", "Complete blood cell count showed a white blood cell count of 3.3 × 109/L with absolute neutrophil count of 1.0 × 109/L, Hb 12.7g/dL, platelet counts 118 × 109/L and differential counts showed no evidence of circulating blasts", "In addition, results of laboratory workup including peripheral blood culture, methicillin-resistant Staphylococcus aureus (MRSA) swab, human immunodeficiency virus test, and flow cytometry performed on his peripheral blood were all negative", "Pathologic sampling from the skin lesion showed the presence of monomorphous small to medium cells extending through the dermis and into the subcutaneous fat (Figure, B)", "Immunohistochemical staining demonstrated CD4-, CD33-, CD43-, CD56-, CD123-, and TCL1-positive neoplastic cells with Ki-67 of 85%, but staining results for TdT, MUM1, CD3, CD20, Bcl-6, CyclinD1, and CD8 were negative (Figure, C)", "Results of bone marrow biopsy showed normocellular marrow maturing trilineage hematopoiesis with no increase of blasts or other abnormal populations", "Cytogenetics results were abnormal, showing del(20)(q11.2q13.3)[8]/46,XY[12]", "A, Skin image showing a single, brownish nodule and adjacent scar from prior biopsy", "B, A section of cutaneous blastic plasmacytoid dendritic cell neoplasm with subepidermic confluent atypical cellular infiltrate composed of medium sized, round to oval, or elongated immature precursors with finely granular to open chromatin, inconspicuous to prominent nucleoli and a small amount of cytoplasm (hematoxylin-eosin stain, original magnification ×600)", "C, CD56 (immunoperoxidase stain, original magnification ×600)"], "s1": [0, 1, 2, 3, 4, 5, 8, 9], "s2": [6, 7, 10, 11, 12]} {"key": 756, "questions": "What Is Your Diagnosis?", "options": [{"label": "A", "disease": "Infectious mononucleosis"}, {"label": "B", "disease": "Langerhans cell histiocytosis"}, {"label": "C", "disease": "Reactive sinus histiocytosis"}, {"label": "D", "disease": "Rosai-Dorfman disease"}], "answer_idx": "D", "symptoms": ["A child younger than 10 years presented with an 8-week history of bilateral cervical lymphadenopathy unresponsive to a 10-day course of Augmentin (GlaxoSmithKline)", "The patient’s parents denied any associated fevers, chills, night sweats, sore throat, abdominal pain, or weight changes", "Medical history was noncontributory", "Physical examination showed bilateral, mobile, nontender cervical lymphadenopathy without erythema or swelling", "Tonsils were symmetric and not enlarged", "There was no noticeable thyromegaly or nodules", "Laboratory testing was significant for an elevated erythrocyte sedimentation rate (37 mmol/h)", "Ultrasonography demonstrated cervical lymphadenopathy bilaterally with the largest node measuring 3.8 cm in maximum dimension", "Lymph node needle core biopsy demonstrated increased histiocytes and numerous plasma cells on tissue section (Figure, A)", "Scattered neutrophils and rare giant cells were also present", "Findings of Grocott methenamine silver, acid-fast, and Warthin-Starry stains were negative", "The CD-68 immunostain showed numerous histiocytes", "The specimen stained positive for CD-30 and S-100 immunostains showing emperipolesis (Figure, B and C)", "Findings of a CD1a stain were negative", "Lymphoid markers by flow cytometry showed a polyclonal mixed B-cell and T-cell population", "Lymph node needle core biopsy specimens from the patient", "A, Evident are increased histiocytes and numerous plasma cells (original magnification × 10)", "B, Numerous histiocytes with emperipolesis (original magnification × 40)", "C, Several histiocytes with emperipolesis under oil immersion (original magnification × 100)"], "s1": [0, 1, 2, 3, 4, 5, 6, 7], "s2": [8, 9, 10, 11, 12, 13, 14, 15, 16, 17, 18]} {"key": 757, "questions": "What Is Your Diagnosis?", "options": [{"label": "A", "disease": "Mucocele"}, {"label": "B", "disease": "Lipoma"}, {"label": "C", "disease": "Pleomorphic adenoma"}, {"label": "D", "disease": "Mucoepidermoid carcinoma"}], "answer_idx": "C", "symptoms": ["A man in his 30s presented with a slow-growing mass in the upper lip that had developed over several years", "He had no history of underlying medical diseases, such as hypertension, diabetes, hepatitis, and tuberculosis", "On physical examination, a whitish, firm, painless, nodular mass on the superficial layer of the left upper lip was observed (Figure, A)", "Given the possibility of a tumorous lesion, complete excision was planned under local anesthesia", "In surgery, the 1.5 × 1.2 cm, whitish, firm, nodular, and well-encapsulated mass was excised completely without rupture of the capsule from the surrounding mucosa", "On histopathologic examination, an encapsulated salivary gland tumor with epithelial and myoepithelial cells, ductlike structures, and mixed stroma was observed (Figure, B)", "One month after surgery, the surgical wound on the left upper lip was clean and well healed", "One year later, there had been no recurrence, and the patient had no symptoms at the left upper lip", "A, Clinical image of the left upper lip", "B, Histopathologic image (hematoxylin-eosin, original magnification ×10) showing areas of ductal epithelial components with myxoid stroma"], "s1": [0, 1, 2, 3, 4, 6, 7], "s2": [5, 8, 9]} {"key": 758, "questions": "What Is Your Diagnosis?", "options": [{"label": "A", "disease": "Sarcoidosis"}, {"label": "B", "disease": "Discoid lupus erythematosus"}, {"label": "C", "disease": "Basal cell carcinoma"}, {"label": "D", "disease": "Bowen disease"}], "answer_idx": "B", "symptoms": ["A 40-year-old healthy woman presented with an 8-year history of a slowly growing erythematous plaque on the left lower eyelid, with occasional discomfort", "Physical examination revealed a single erythematous scaling plaque involving the left lower eyelid, including its border", "Eyelashes were totally absent (Figure 1A)", "No other lesions were found after complete physical examination", "Dermoscopy showed an erythematous background, telangiectatic vessels, whitish structureless areas, and whitish scales (Figure 1B)", "A 4-mm punch biopsy specimen was obtained (Figure 1C)", "A, Erythematous scaling plaque involving the left lower eyelid", "B, Dermoscopy shows erythematous background, telangiectatic vessels, structureless whitish areas, and white scales", "C, Histopathological analysis (hematoxylin-eosin)"], "s1": [0, 1, 2, 3], "s2": [4, 5, 6, 7, 8]} {"key": 759, "questions": "What Is Your Diagnosis?", "options": [{"label": "A", "disease": "Granulomatous mycosis fungoides"}, {"label": "B", "disease": "Pretibial myxedema"}, {"label": "C", "disease": "Cutaneous sarcoidosis"}, {"label": "D", "disease": "Necrobiosis lipoidica"}], "answer_idx": "C", "symptoms": ["A man in his late 20s presented with mildly itchy, erythematous plaques of 3 months’ duration on both legs", "The lesions started to appear on the left pretibial region as small papules, which gradually increased in size and coalesced to form bigger plaques", "No history of local trauma, prolonged standing, and occupational exposure to mineral dust or recent drug intake was recollected", "He was not diabetic", "On examination, large, ill-defined, erythematous plaques were present on the pretibial areas extending onto the lateral aspect of both legs", "The plaques were surmounted by multiple, discrete atrophic scars", "The periphery of the plaques showed increased vascularity in the form of telangiectasias and venous prominences, along with atrophy of the overlying skin (Figure, A)", "There were no lesions elsewhere on the body", "A skin biopsy sample was obtained from the plaque and sent for histopathological examination (Figure, B and C)", "A, Clinical photograph on presentation showing ill-defined, erythematous plaques surmounted by multiple atrophic scars", "The periphery of the plaques shows telangiectasia", "Some areas show venous prominence", "B, Skin biopsy showing unremarkable epidermis and multiple epithelioid cell granulomas in the entire dermis", "C, Compact, noncaseating, epithelioid cell granulomas with multiple Langhans giant cells", "D, Photograph after 6 weeks of treatment showing considerable decrease in erythema and induration"], "s1": [0, 1, 2, 4, 5, 6, 7, 9, 10, 11], "s2": [3, 8, 12, 13, 14]} {"key": 760, "questions": "What Is Your Diagnosis?", "options": [{"label": "A", "disease": "Sarcoidosis"}, {"label": "B", "disease": "Cutaneous silica granuloma"}, {"label": "C", "disease": "Infectious granuloma"}, {"label": "D", "disease": "Keloid"}], "answer_idx": "B", "symptoms": ["A miner in his early 40s presented with a 6-month history of asymptomatic, erythematous nodules on his face, his chin, the anterior portion of his neck, and the anterior thorax", "Approximately 15 years earlier, he had sustained injuries from an accidental land mine explosion that had lacerated his face, neck, and thorax in several places", "The wounds apparently healed without difficulty, leaving only scars of expected severity", "Approximately 6 months before presentation, the patient began to notice an increase in the size of these lesions", "Physical examination showed multiple firm, erythematous, nontender papules, nodules, and small plaques on his face, his chin, the anterior portion of his neck, and the anterior thorax (Figure 1)", "Multiple firm, erythematous, nontender nodules and small plaques on the patient’s face"], "s1": [1, 2], "s2": [0, 3, 4, 5]} {"key": 761, "questions": "What Is Your Diagnosis?", "options": [{"label": "A", "disease": "Hepatic artery injury"}, {"label": "B", "disease": "Jejunal injury with intraluminal contrast extravasation"}, {"label": "C", "disease": "Contrast-enhancing gallbladder"}, {"label": "D", "disease": "Rupture of a peripancreatic pseudoaneurysm"}], "answer_idx": "C", "symptoms": ["A man in his 60s with a known history of a large cyst on the right kidney sustained blunt trauma in a skiing crash", "Computed tomography (CT) showed a right-sided grade IV renal injury involving his known renal cyst with active arterial extravasation (Figure, A)", "The patient underwent selective angioembolization of the renal laceration", "A, Abdominal computed tomography (CT) with contrast demonstrating grade IV laceration of the right kidney and active extravasation", "B, Abdominal CT obtained on day 2 of hospitalization showing large right-sided retroperitoneal hematoma and grade IV renal laceration displacing the patient’s abdominal viscera to the left, with contrast-filled structure medial to the liver", "In the 24 hours after embolization, the patient required 2 U of red blood cells via transfusion for anemia", "On examination, the patient reported moderate pain in the abdomen and right side and demonstrated a distended abdomen in the absence of any peritoneal signs", "Contrast-enhanced CT of the abdomen and pelvis was performed to assess for a source of continued bleeding (Figure, B)"], "s1": [0, 1, 3, 4, 7], "s2": [2, 5, 6]} {"key": 762, "questions": "What is Your Diagnosis?", "options": [{"label": "A", "disease": "Lichen planopilaris"}, {"label": "B", "disease": "Trichoepitheliomas"}, {"label": "C", "disease": "Miliary osteoma cutis"}, {"label": "D", "disease": "Angiofibromas"}], "answer_idx": "C", "symptoms": ["A woman in her 50s with biopsy-proven scalp lichen planopilaris (LPP) presented with an increasing number of slightly pruritic facial papules that coincided with the start of her hair loss condition (Figure, A)", "She denied a history of severe acne or other inflammatory skin disorders", "The patient had no health problems and only had a distant medical history of an appendectomy and an oophorectomy", "The only medication she was taking was hydroxychloroquine 200 mg twice daily that was initiated as treatment of her LPP and did not improve the appearance or symptoms of her facial papules", "On physical examination, she had numerous 1-mm skin-colored grouped monomorphic noninflammatory papules on the bilateral temples, the bilateral cheeks, and the chin", "A 3-mm punch biopsy specimen was obtained for histopathologic examination of a representative lesion on the right chin (Figure, B and C)", "A, Clinical photograph shows numerous grouped 1-mm skin-colored monomorphic noninflammatory papules on the left cheek and chin", "B and C, Histology of a 3-mm punch biopsy specimen of a lesion on the right chin using a hematoxylin-eosin stain"], "s1": [0, 3, 4, 6], "s2": [1, 2, 5, 7]} {"key": 763, "questions": "What Is Your Diagnosis?", "options": [{"label": "A", "disease": "Hidradenitis suppurativa"}, {"label": "B", "disease": "Metastatic Crohn disease"}, {"label": "C", "disease": "Pemphigus vegetans"}, {"label": "D", "disease": "Linear hypertrophic herpes simplex virus infection"}], "answer_idx": "B", "symptoms": ["A woman in her 50s with a medical history of hypertension and depression was evaluated by the inpatient dermatology consult service for a 1.5-year history of painful intertriginous wounds", "She was previously diagnosed with hidradenitis suppurativa and treated with surgical excision and split-thickness skin grafting of the genitocrural folds with graft failure", "Over the past 1.5 years, she had been admitted to the hospital on multiple occasions for cellulitis around the wounds, which responded to intravenous antibiotics, and she had received a diverting sigmoid colostomy and indwelling urinary catheter to decrease wound contamination", "Her wound care required a nursing home", "Just prior to presentation, she received adalimumab for 3 months without clinical improvement", "On physical examination there were linear moist, eroded, exophytic, condyloma-like plaques of the bilateral submammary folds (Figure, A), intergluteal cleft, and under the abdominal apron", "There were ulcers of the bilateral genitocrural folds, as well as vulvar edema (Figure, B)", "The lips, oropharynx, and ostomy were unremarkable", "A wedge biopsy specimen of a submammary plaque was obtained, and histopathology was performed (Figure, C and D)", "A, Clinical photograph shows a submammary representative linear condyloma-like ulcerated plaque, as well as secondary irritant contact dermatitis from application of topical medications", "Similar plaques were present in the bilateral submammary fold, under the abdominal apron and in the intergluteal cleft", "B, Clinical photograph of linear ulcers of the genitocrural folds, vulvar edema, and an indwelling urinary catheter", "C, Hematoxylin-eosin stain of a biopsy specimen from the submammary plaque", "D, High-power image from the same specimen"], "s1": [0, 1, 2, 3, 4, 5, 6, 10, 11], "s2": [7, 8, 9, 12, 13]} {"key": 764, "questions": "What Is Your Diagnosis", "options": [{"label": "A", "disease": "Seborrheic dermatitis"}, {"label": "B", "disease": "Multisystem Langerhans cell histiocytosis"}, {"label": "C", "disease": "Crusted scabies"}, {"label": "D", "disease": "Psoriasis vulgaris"}], "answer_idx": "B", "symptoms": ["A 4-year-old boy presented to our department of dermatology with a 3-year history of nonhealing crusted scalp lesions", "He had previously been treated with various topical glucocorticosteroids and antibiotics without improvement", "The physical examination revealed multiple disseminated erythematous papules, petechial hemorrhages, and yellowish crusts on the scalp (Figure, A)", "The remaining skin and mucosal surfaces, as well as neck, axillary, and inguinal lymph nodes, were unremarkable", "The patient did not show any other obvious abnormalities, but his mother reported occasional otitis media", "The family history was negative for skin disease", "Routine blood examinations revealed no pathological findings", "A whole-body magnetic resonance imaging (MRI) scan showed 2 osteolytic lesions on the right frontoparietal skull (Figure, B)", "A punch biopsy specimen of an erythematous papule on the scalp was obtained and stained with hematoxylin-eosin for histopathological analysis (Figure, C)", "In addition, immunohistochemical stainings were performed (Figure, D)", "A, Clinical photograph shows multiple disseminated erythematous papules, petechial hemorrhages and yellowish crusts on the scalp", "B, Contrast-enhanced T1-weighted axial magnetic resonance image shows osteolytic lesions in the frontotemporal skull", "C, Histopathological analysis of a punch biopsy of the scalp demonstrates a subepidermal infiltrate consisting of large cells with kidney-shaped nuclei, solitary lymphocytes, and eosinophilic granulocytes", "D, The infiltrate stained positively for CD1A"], "s1": [0, 1, 2, 3, 4, 5], "s2": [6, 7, 8, 9, 10, 11, 12, 13]} {"key": 765, "questions": "What Is Your Diagnosis?", "options": [{"label": "A", "disease": "Tinea pedis"}, {"label": "B", "disease": "Psoriasis"}, {"label": "C", "disease": "Pityriasis rubra pilaris"}, {"label": "D", "disease": "Juvenile plantar dermatosis"}], "answer_idx": "C", "symptoms": ["A healthy adolescent girl presented with a prolonged dermatitis of both feet characterized by pruritus and accompanying desquamation (Figure 1)", "Physical examination revealed well-demarcated desquamative patches with scales on both distal feet, showing a sandal appearance", "No local heat, swelling, or discharge from the skin lesions was noted", "The patient’s medical history was otherwise unremarkable and she had no known history of atopic dermatitis", "A fungal smear skin test with potassium hydroxide preparation was performed on the lesion, which showed negative findings", "Well-defined hyperkeratotic desquamation on both feet, representing a sandal appearance"], "s1": [0, 3, 4], "s2": [1, 2, 5]} {"key": 766, "questions": "What Is Your Diagnosis?", "options": [{"label": "A", "disease": "Colorectal cancer"}, {"label": "B", "disease": "Testicular cancer"}, {"label": "C", "disease": "Bladder cancer"}, {"label": "D", "disease": "Lymphoma"}], "answer_idx": "B", "symptoms": ["A man in his 30s presented with abdominal pain associated with unintentional weight loss for 6 months", "The patient had a history of hyperlipidemia", "Results of examination revealed a palpable mass of about 15 cm in the left paraumbilical region", "The patient had a normal right testis but a missing left testis", "Results of a computed tomography scan of the thorax, abdomen, and pelvis revealed large intraperitoneal and retroperitoneal masses", "The intraperitoneal heterogenous mass was 20 × 15 × 11 cm, with linear calcifications (Figure 1)", "There was a right moderate hydronephrosis", "The retroperitoneal mass was homogeneous, with associated lymphadenopathy encircling the aorta, inferior mesenteric artery, and both renal arteries", "A large left supraclavicular lymph node and left inguinal lymph node were also detected", "There was no metastasis identified elsewhere", "Serum lactate dehydrogenase was elevated at 1358 U/L (to convert to microkatals per liter, multiply by 0.0167), β human chorionic gonadotrophin was elevated at 1485 mIU/mL (to convert to international units per liter, multiply by 1.0), and α fetoprotein levels were normal", "A computed tomography scan of the thorax, abdomen, and pelvis showing large intraperitoneal and retroperitoneal masses", "The intraperitoneal heterogenous mass was 20 × 15 × 11 cm, with linear calcifications"], "s1": [0, 1, 3, 6, 8, 9], "s2": [2, 4, 5, 7, 10, 11, 12]} {"key": 767, "questions": "What Is Your Diagnosis?", "options": [{"label": "A", "disease": "Pancreas injury"}, {"label": "B", "disease": "Flaring ulcerative colitis"}, {"label": "C", "disease": "Rectosigmoid junction rupture"}, {"label": "D", "disease": "Duodenal injury"}], "answer_idx": "C", "symptoms": ["A 48-year-old man was admitted to our emergency department 12 hours after a bicycle accident with abdominal pain, nausea, and vomiting", "He described abdominal trauma caused by the bicycle handlebar", "The patient reported undergoing an appendectomy at age 23 years and having ulcerative colitis since he was aged 30 years", "He had received treatment (mesalazine and corticosteroids) for inflammatory bowel disease that had been discontinued", "Physical examination revealed paraumbilical ecchymosis (Figure 1A), general abdominal tenderness, and distension", "Body temperature, blood pressure, and heart and respiratory rates were normal", "Laboratory test findings did not show abnormalities except for the leukocyte count (12 300 cells/μL [to convert to ×109/L, multiply by 0.001])", "hemoglobin and hematocrit levels were within the reference ranges", "An abdominal computed tomographic (CT) scan, performed immediately, revealed a pneumoperitoneum and free fluid in the lower quadrants of the abdomen (Figure 1B)", "Preoperative workup", "A, Paraumbilical bruising", "B, Abdominal computed tomography (CT) scan: coronal cut showing free fluid (white arrowhead) and pneumoperitoneum (red arrowheads)"], "s1": [0, 1, 2, 3, 4], "s2": [5, 6, 7, 8, 9, 10, 11]} {"key": 768, "questions": "What Is Your Diagnosis?", "options": [{"label": "A", "disease": "Metastatic colorectal cancer"}, {"label": "B", "disease": "Pulmonary enteric adenocarcinoma (PEA)"}, {"label": "C", "disease": "Adenocarcinoma of unknown primary"}, {"label": "D", "disease": "Lymphoma"}], "answer_idx": "B", "symptoms": ["A man in his 60s presented with a 1-year history of a mild, dry cough", "He was otherwise asymptomatic and had no physical limitations", "He had never smoked, and results of his physical examination were normal", "Results of complete blood cell counts, basic metabolic panel, and liver function tests were all within normal limits", "A chest radiograph revealed a 10 × 9-cm lobulated left lower lobe mass", "Examination of a biopsy specimen showed a primary adenocarcinoma with an immunohistochemical profile of CDX2 positive, CK20 positive, TTF-1 negative, and napsin A negative", "His EGFR (OMIM 131550), ALK (OMIM 105590), and ROS1 (OMIM 165020) genes were wild type, and results of testing for programmed cell death protein 1 (PD-1) and programmed death ligand 1 (PD-L1) were 0%", "Results of upper gastrointestinal tract endoscopy and colonoscopy did not reveal any evidence of a primary malignant neoplasm", "Positron emission tomography showed no evidence of mediastinal lymphadenopathy or extrathoracic disease", "Given the patient’s excellent performance status, he underwent a mediastinoscopy with left lower lobe lobectomy", "Results of postoperative pathologic testing showed a 13-cm adenocarcinoma with similar immunohistochemical markers as in the prior biopsy specimen", "Surgical margins were negative for involvement, with no large vessel or perineural invasion seen", "Five sampled lymph nodes were negative for involvement", "Repeat pathologic interpretation was performed (Figure)", "Initial pathologic testing of a left lower lobe biopsy specimen (hematoxylin-eosin stain, original magnification ×100)"], "s1": [0, 1, 2, 3, 8, 9, 12], "s2": [4, 5, 6, 7, 10, 11, 13, 14]} {"key": 769, "questions": "What Is Your Diagnosis?", "options": [{"label": "A", "disease": "Esthesioneuroblastoma"}, {"label": "B", "disease": "Ewing sarcoma"}, {"label": "C", "disease": "Alveolar rhabdomyosarcoma"}, {"label": "D", "disease": "Non-Hodgkin lymphoma"}], "answer_idx": "C", "symptoms": ["A woman in her 60s presented with a 3-month history of right-sided sinus pressure progressing to dental pain", "She was prescribed antibiotics and steroids by her family physician, which initially provided intermittent, symptomatic improvement", "however, she ultimately experienced a progressive decrease in tactile sensation on her face, anosmia, and changes in her right peripheral vision", "She had a remote 2 pack-year smoking history", "Sinus computed tomography (CT) and neck magnetic resonance imaging (MRI) revealed a large expansile mass arising from the right side of the skull base and nasopharynx with extension into the pterygopalatine fossa, masticator, parapharyngeal, retropharyngeal, and carotid spaces", "In addition, imaging showed involvement of the right posterior orbit with erosion of the posterior maxillary sinus wall, anterior and middle cranial fossa with erosion through the cribriform plate, and cavernous sinus", "Physical examination revealed slight right-sided facial fullness, grossly intact extraocular motion, decreased vision in the right lateral field, and palpable bilateral level 2 neck lymph nodes", "Flexible laryngoscopy revealed a right-sided lobulated mass involving the medial middle turbinate, posterior septum, and nasal floor, as well as the posterior aspect of the maxillary sinus extending superiorly to the nasopharynx and skull base posteriorly", "The mass extended into the left superior nasopharynx and superior septum", "In-office biopsy specimens were obtained and revealed a primitive small round cell tumor with the following immunohistochemical (IHC) staining profile: diffusely positive for desmin, myogenin, and CD56", "focally positive for low-molecular-weight kininogen (LMWCK), p63, and S-100", "and negative for AE1/AE3, CK 5/6, CK7, CK20, p40, chromogranin, SOX10, HMB45, CD99, and CD20 (Figure 1)", "Histopathologic images", "A, Nests of primitive small round blue cells surrounded by fibrovascular septa", "Hematoxylin-eosin", "B, Diffuse, nuclear positivity", "Myogenin stain"], "s1": [0, 1, 2, 3, 6, 7], "s2": [4, 5, 8, 9, 10, 11, 12, 13, 14, 15, 16]} {"key": 770, "questions": "What Is Your Diagnosis?", "options": [{"label": "A", "disease": "Squamous cell carcinoma"}, {"label": "B", "disease": "Lymphoma"}, {"label": "C", "disease": "Amyloidosis"}, {"label": "D", "disease": "Rhabdomyosarcoma"}], "answer_idx": "C", "symptoms": ["A man in his 60s with a history of chronic obstructive pulmonary disease (COPD) and dyspnea presented with right lower quadrant abdominal pain", "A noncontrast computed tomographic (CT) scan of the abdomen demonstrated splenomegaly, ascites, and a retroperitoneal mass posterior to the pancreatic head", "Fine-needle aspiration from the retroperitoneal mass and ascitic fluid revealed no malignant cells", "The patient subsequently developed a left-sided lateral chest wall hematoma, and, in retrospect, it was thought that the retroperitoneal mass was actually a hematoma because multiple follow-up studies demonstrated the mass steadily decreasing in size", "Echocardiography showed right atrial and ventricular dilatation and increased thickness of the interventricular septum", "Laboratory tests showed elevated results for serum-free λ light chain and proteinuria levels and altered liver function tests but a normal coagulation profile", "Routine surveillance noncontrast CT imaging of the neck demonstrated a submucosal soft-tissue mass containing few punctate calcifications centered in the epiglottis with clinically significant narrowing of the oropharyngeal and supraglottic laryngeal airways by the enlarged epiglottis (Figure)", "Computed tomographic images show a soft-tissue mass containing punctate calcifications centered on the epiglottis"], "s1": [0, 1, 2, 3, 4, 5], "s2": [6, 7]} {"key": 771, "questions": "What Is Your Diagnosis?", "options": [{"label": "A", "disease": "Carotid cochlear dehiscence"}, {"label": "B", "disease": "Semicircular canal dehiscence"}, {"label": "C", "disease": "Sigmoid sinus diverticulum"}, {"label": "D", "disease": "Aberrant internal carotid artery"}], "answer_idx": "C", "symptoms": ["A woman in her 40s presented with right pulsatile tinnitus", "She reported constant pulsatile tinnitus with a whooshing quality for 4 years", "Factors exacerbating her symptoms included holding her breath and turning her head to the left", "She reported that compression of her right neck and turning her head to the right decreased the loudness of the tinnitus", "She had not experienced hypertension, vertigo, sleep apnea, hyperthyroidism, hearing loss, head trauma, history of ear infections, otorrhea, rhinorrhea, ear surgery, or family history of ear problems", "The external auditory canals were clear bilaterally", "Tympanic membranes were intact and mobile bilaterally", "There was no evidence of any vascular masses in the middle ear space", "Results from testing with tuning forks were normal", "Toynbee stethoscope revealed a bruit in the right ear that could be stopped with gentle compression of the right side of the neck", "An audiogram showed hearing within normal limits with slight air-bone gaps present at 250, 1000, and 3000 Hz on the right and within normal limits on the left", "Speech reception threshold on the right was 10 dB with a word recognition score of 100% bilaterally", "The tympanogram result revealed type A bilaterally, and acoustic reflexes were all present", "A computed tomographic (CT) temporal bone scan was completed (Figure 1)", "Computed tomographic images of the right temporal bone", "The arrowheads indicate the findings"], "s1": [0, 1, 2, 3, 9, 10], "s2": [4, 5, 6, 7, 8, 11, 12, 13, 14, 15]} {"key": 772, "questions": "What Is Your Diagnosis?", "options": [{"label": "A", "disease": "Primary nerve sheath tumor"}, {"label": "B", "disease": "Thrombophlebitis of external jugular vein"}, {"label": "C", "disease": "Traumatic neuroma from prior surgery"}, {"label": "D", "disease": "Perineural spread of cutaneous squamous cell carcinoma"}], "answer_idx": "D", "symptoms": ["A man in his 60s presented to a tertiary care center after several months of having right-sided lateral neck pain", "He described the pain as “electric” and “stabbing", "” He reported no numbness of the region and had not experienced any other symptoms, including new neck masses", "His medical history was significant for obstructive sleep apnea, gastroesophageal reflux disease, a right ear skin cancer excised by a dermatologist, and coronary artery disease with angina on stress test", "His physical examination revealed a well-healed excision site of the right ear without new skin lesions", "His facial nerve was fully intact", "He had full strength of the shoulder and no numbness of the right ear or cheek", "Palpation of the lateral neck demonstrated a mass running along the lateral border of the sternocleidomastoid (SCM) muscle", "Magnetic resonance imaging (MRI) showed an elongated enhancing tubular structure along the lateral surface of the right SCM muscle (Figure 1)", "A and B, Anterior to posterior coronal fat-saturated T1-weighted postcontrast magnetic resonance imaging (MRI) with an enhancing tubular mass of the right side of the neck (yellow arrowhead) that extends posterior and deep to sternocleidomastoid (SCM) (blue arrowheads)", "C, Axial T1-weighted postcontrast MRI with an enhancing tubular mass (blue arrowheads) that extends posterior and deep to the SCM (blue arrowheads) (white arrowhead indicates external jugular vein)"], "s1": [0, 1, 2, 3, 4, 5, 6], "s2": [7, 8, 9, 10]} {"key": 773, "questions": "What Is Your Diagnosis?", "options": [{"label": "A", "disease": "Aplasia cutis congenita"}, {"label": "B", "disease": "Cellulitis and abscess associated with neonatal group B streptococcal infection"}, {"label": "C", "disease": "Epidermolysis bullosa"}, {"label": "D", "disease": "Halo scalp ring (pressure necrosis)"}], "answer_idx": "D", "symptoms": ["A neonate in her second week of life was brought to our hospital for evaluation of a scalp lesion initially thought to be an open abscess", "She was born full term to a primigravid mother, who had an uncomplicated pregnancy", "At parturition, the baby presented in occiput posterior position and remained partly crowned for more than 24 hours of maternal labor", "Finally, due to failure to progress, the baby was delivered by urgent cesarean delivery", "Birth weight was 3.13 kg (15th percentile), head circumference was 31 cm (<1 percentile), and Apgar score was 9 of 9. The baby’s head had a markedly elongate vertex, which the neonatal intensive care unit team diagnosed as cephalohematoma", "On day 5 of life, her serum total bilirubin and conjugated bilirubin were 20.5 mg/dL (>95 percentile) and 0.3 mg/dL (to convert to micromoles per liter, multiply by 17.104), respectively", "The girl was hospitalized for 1 day of phototherapy before being discharged home weighing 2.95 kg (−5.75%)", "She was exclusively breastfed", "She was admitted to our hospital for evaluation of the scalp", "Wound culture samples were sent for analysis, and she was given empirical intravenous clindamycin hydrochloride", "Dermatology was consulted", "On examination, the baby was alert and healthy appearing, although her sclerae were icteric", "The vertex of her scalp was uniformly boggy, soft, and edematous, extending beyond suture lines, consistent with a diagnosis of caput succedaneum", "On the right occiput, there was a mat of hair and thick serosanguineous crust adhering to the scalp (Figure 1A)", "Beneath the mat and extending laterally in both directions was a deep, 8 × 1-cm, trenchlike ulcer with no drainage (Figure 1B)", "It was nontender to palpation", "A, Serosanguineous crust adhering to occipital scalp of a neonate", "Prior to removing this crust it was being treated as an open ulcer", "B, Trenchlike linear ulcer revealed after removal of the crust", "It was 8 × 1 cm in size, was nontender to palpation, and did not appear to be infected", "Cellulitis and abscess associated with neonatal group B streptococcal infection"], "s1": [0, 8, 9, 10, 13, 14, 16, 17, 18, 19, 20], "s2": [1, 2, 3, 4, 5, 6, 7, 11, 12, 15]} {"key": 774, "questions": "What Is Your Diagnosis?", "options": [{"label": "A", "disease": "Erythema nodosum"}, {"label": "B", "disease": "Pancreatic panniculitis"}, {"label": "C", "disease": "Alpha-1-antitrypsin deficiency panniculitis"}, {"label": "D", "disease": "Polyarteritis nodosa"}], "answer_idx": "B", "symptoms": ["A man in his 20s with panhypopituitarism, septo-optic dysplasia, developmental delay, duodenal stricture, and a history of pancreatitis presented with a painful eruption on his bilateral lower legs", "The patient was well until 2 days prior to hospitalization when he developed bilateral lower extremity edema", "The day prior to admission, he developed painful pink lesions on both legs", "He had not applied any topical medications to his legs or had recent exposure to hot tubs or fish tanks at the home", "He had a history of keratosis pilaris on the lower legs and was otherwise well with no fevers, recent upper respiratory infections, abdominal pain, vomiting, or diarrhea", "Physical examination showed a man in no apparent distress", "On the bilateral anterior legs were scattered tender, pink to purple nodules, and plaques (Figure, A)", "There was also a background of pinpoint folliculocentric papules on leg and thighs consistent with keratosis pilaris", "There was 2+ pitting edema of the lower extremities", "A punch biopsy specimen was obtained for histopathology and microbiology cultures (Figure, B-D)", "Routine complete blood cell count and complete metabolic panel had results within normal limits", "A, Clinical photograph shows scattered pink to purple nodules and plaques on the bilateral lower extremities", "B, C, and D, Histopathologic images of a biopsy specimen (hematoxylin-eosin)", "B, Subcutaneous tissue with superficial and deep, dermal and perivascular inflammation, fibrosis, and fat necrosis", "C, Lobular panniculitis", "D, Dense neutrophilic infiltration of subcutaneous fat associated with fat necrosis and deposition of granular basophilic material (arrowhead)"], "s1": [0, 1, 2, 3, 4, 5, 8, 10], "s2": [6, 7, 9, 11, 12, 13, 14, 15]} {"key": 775, "questions": "What Is Your Diagnosis?", "options": [{"label": "A", "disease": "Lichen amyloidosis"}, {"label": "B", "disease": "Folliculocentric lichen sclerosus et atrophicus"}, {"label": "C", "disease": "Guttate vitiligo"}, {"label": "D", "disease": "Eruptive tumors of the follicular infundibulum"}], "answer_idx": "B", "symptoms": ["A woman in her 60s presented with numerous whitish papules on the back of more than 10 years’ duration", "The lesions previously resolved with topical corticosteroids but flared soon after treatment discontinuation", "Some papules became pruritic and extended to the waist and anterior trunk in the past month", "No family history of similar lesions was recorded", "Physical examination revealed multiple hypopigmented, flat-topped papules on the back, waist, and inframammary area without genital involvement (Figure, A)", "Most lesions were folliculocentric under close inspection", "Dermoscopy revealed central keratin plugs and some foci of structureless, whitish, and homogenous areas with surrounding erythema (Figure, B)", "The results of laboratory tests for antinuclear antibody and hyperglobulinemia were negative", "A biopsy specimen was obtained from the back for histologic analysis (Figure, C)", "A, Multiple hypopigmented, atrophic, keratotic, and folliculocentric papules are seen on the trunk", "B, Dermascopic findings included 2 foci of central keratin plugs and another ill-defined, whitish, and homogenous area with surrounding erythema", "Scale bar corresponds to 1 mm", "C, Histopathologic examination of a biopsy specimen (hematoxylin-eosin, original magnification ×100)"], "s1": [0, 1, 2, 3, 4, 5, 7], "s2": [6, 8, 9, 10, 11, 12]} {"key": 776, "questions": "What Is Your Diagnosis?", "options": [{"label": "A", "disease": "Alport syndrome"}, {"label": "B", "disease": "Branchio-oto-renal syndrome"}, {"label": "C", "disease": "Goldenhar syndrome (oculoauriculovertebral spectrum)"}, {"label": "D", "disease": "Papillorenal syndrome"}], "answer_idx": "B", "symptoms": ["An 8-month-old boy born full term presented to dermatology for evaluation of a bump on his neck that was present from birth (Figure)", "The parent noted that clear fluid drained from this bump when the patient ate", "In addition, the parent reported that, at birth, the patient failed multiple hearing tests and that he had small kidneys", "The patient was asymptomatic", "Upon physical examination, the patient was noted to have a preauricular sinus and ear pit as well as a skin-colored papule on the antitragus of the right ear", "He also had a pedunculated, skin-colored papule on the right lateral neck", "Photograph of infant’s lateral neck depicting skin-colored papule with clear drainage"], "s1": [0, 5, 6], "s2": [1, 2, 3, 4]} {"key": 777, "questions": "What Is Your Diagnosis?", "options": [{"label": "A", "disease": "Anaphylactic shock secondary to ruptured echinococcal cyst"}, {"label": "B", "disease": "Hemorrhagic shock secondary to perforated hemorrhagic calculous cholecystitis"}, {"label": "C", "disease": "Hemorrhagic shock secondary to spontaneous intra-abdominal hemorrhage"}, {"label": "D", "disease": "Septic shock secondary to perforated cholecystitis"}], "answer_idx": "B", "symptoms": ["A 78-year-old man with a history of deep vein thrombosis and pulmonary embolism who had been taking warfarin for the last 20 years presented with 1 day of severe abdominal pain, which began after eating dinner", "The pain had no associated nausea, vomiting, or obstipation", "On examination, he was afebrile and had a normal heart rate but was notably hypotensive", "He was given 2 L of crystalloid, and his blood pressure responded appropriately", "His physical examination showed an abdomen that was soft but diffusely tender without rebound or guarding", "He had a well-healed appendectomy scar", "Blood testing revealed an elevated creatinine level of 1.7 mg/dL (to convert to micromoles per liter, multiply by 88.4), an elevated lactate level of 29.7 mg/dL (to convert to micromoles per liter, multiply by 0.111), an international normalized ratio of 4.6, a lowered hemoglobin level of 10.7 g/dL (to convert to grams per liter, multiply by 10), and a lowered hematocrit level of 27.2%", "all other results were unremarkable", "Given his coagulopathy, low hematocrit level, and abdominal pain, a computed tomography angiogram was performed to rule out bleeding (Figure 1)", "A, Axial cross-section computed tomography without contrast of the abdomen and pelvis revealed gallstones in the neck of the gallbladder, high-density material within the lumen, and free intraperitoneal fluid", "B, Axial cross-section computed tomography with intravenous contrast was significant for a blush of contrast within the lumen of the gallbladder and surrounding hyperemia of the liver at the gallbladder fossa", "Hemorrhagic shock secondary to perforated hemorrhagic calculous cholecystitis"], "s1": [0, 1, 2, 3, 4, 5, 6, 7], "s2": [8, 9, 10, 11]} {"key": 778, "questions": "What Is Your Diagnosis?", "options": [{"label": "A", "disease": "Normal coronary angiography"}, {"label": "B", "disease": "Coronary steal syndrome"}, {"label": "C", "disease": "Subclavian steal syndrome"}, {"label": "D", "disease": "Subclavian stenosis"}], "answer_idx": "B", "symptoms": ["A 71-year-old man with a history of coronary artery bypass graft using the left internal mammary artery (LIMA) to the left anterior descending (LAD) in 1997 presented with frequent ventricular tachycardia on routine pacemaker interrogation", "He reported dyspnea on exertion and left leg claudication at 23 m", "At rest, his blood pressure was 146/65 mm Hg measured on the right arm and 85/50 mm Hg on the left arm", "Physical examination was notable for a diminished left radial pulse", "In 2014, he experienced angina that resolved following placement of a drug-eluting stent in his LAD", "Coronary angiography via the left femoral artery is displayed in the Figure"], "s1": [0, 4, 5], "s2": [1, 2, 3]} {"key": 779, "questions": "What Is Your Diagnosis?", "options": [{"label": "A", "disease": "Lymphomatoid papulosis"}, {"label": "B", "disease": "Reactive lymphomatoid reaction secondary to drugs"}, {"label": "C", "disease": "Mycosis fungoides"}, {"label": "D", "disease": "Adult T-cell leukemia lymphoma"}], "answer_idx": "C", "symptoms": ["A woman in her 20s previously prescribed and currently taking valproic acid and phenytoin for epilepsy presented to the clinic with a 2-month history of gradually worsening pruritic plaques that initially involved the scalp, face, and neck, and later affected the anterior and posterior chest wall, abdomen, upper arms, and thighs", "she also had nodules over the nose and cheeks (Figure 1A)", "Clinical photographs show (A) nodules on the head and neck on presentation and (B) nodules of of increasing thickness after 6 months", "It was thought that she developed exfoliative dermatitis secondary to valproic acid and phenytoin", "Medications were switched to levetiracetam, and a tapering dose of steroid was prescribed", "Her skin lesions slightly improved", "Two months later, she developed diffuse urticarial lesions of varying sizes over the old plaques", "These lesions were well-defined with demarcated borders and were minimally pruritic at the time", "Given that her condition worsened despite changing her medication, the lesions were biopsied and analyzed, and the results were inconclusive, demonstrating atypical lymphoid infiltrate", "The T cell receptor γ gene (TRG) rearrangement analysis by polymerase chain reaction (PCR) was negative", "The plaques and nodules worsened over a 6-month period, becoming more numerous and increasing in thickness, especially on the face, and early leonine facies developed (Figure 1B)", "The results from analysis of a second biopsy specimen showed dense band-like lymphoid infiltrate in the superficial dermis with mild epidermotropism", "As her condition continued to worsen, a third biopsy specimen was obtained, and the result was consistent with the suspected diagnosis"], "s1": [0, 3, 4, 5, 6, 7], "s2": [1, 2, 8, 9, 10, 11, 12]} {"key": 780, "questions": "What Is Your Diagnosis?", "options": [{"label": "A", "disease": "Dermoid"}, {"label": "B", "disease": "Salivary gland anlage tumor"}, {"label": "C", "disease": "Nasopharyngeal carcinoma"}, {"label": "D", "disease": "Hemangioma"}], "answer_idx": "B", "symptoms": ["An otherwise healthy 3-year-old boy presented with symptoms of sleep-disordered breathing", "He had no reported respiratory issues at birth and no feeding difficulties", "His symptoms included gasping and apneic episodes at night, as well as nighttime nasal congestion", "He had been evaluated by an outside otolaryngologist, who had scheduled an adenoidectomy", "Intraoperatively, he was found to have a nasopharyngeal mass, and thus surgery was aborted", "Nasal endoscopy in clinic revealed a firm, mobile mass in the nasopharynx", "Magnetic resonance imaging of the brain revealed a 21 × 14 × 18-mm irregular, but well-defined, heterogeneously enhancing mass in the left nasopharynx with increased perfusion (Figure, A)", "Given the vascularity of the tumor, the boy underwent preoperative embolization with interventional radiology", "He was subsequently taken to the operating room, where endoscopic resection of the mass and division of the pedicle were performed without complication (Figure, B)", "Final pathologic results demonstrated keratinizing squamous cysts and collections of ductal structures set within a myxoid stroma, with small nests of primitive mesenchyme (Figure, C and D)"], "s1": [0, 1, 2, 3], "s2": [4, 5, 6, 7, 8, 9]} {"key": 781, "questions": "What Is Your Diagnosis?", "options": [{"label": "A", "disease": "Mucoepidermoid carcinoma"}, {"label": "B", "disease": "Sialolith"}, {"label": "C", "disease": "Intraoral lipoma"}, {"label": "D", "disease": "Unusual mucocele"}], "answer_idx": "B", "symptoms": ["A man in his 50s presented for an emergency examination with a hard swelling of the left inner cheek", "He stated that he had recently bitten his cheek and had since experienced increasing spontaneous pain and swelling", "There was no obvious swelling extraorally", "Oral examination revealed an erythematous, 15 × 10-mm lesion of the buccal mucosa, adjacent to the maxillary first molar", "The lesion was hard and produced a small amount of bleeding with palpation (Figure 1)"], "s1": [0, 1], "s2": [2, 3, 4]} {"key": 782, "questions": "What Is Your Diagnosis?", "options": [{"label": "A", "disease": "Submandibular gland pleomorphic adenoma"}, {"label": "B", "disease": "Chronic lymphadenitis"}, {"label": "C", "disease": "Hypoglossal nerve schwannoma"}, {"label": "D", "disease": "Chronic submandibular sialadenitis"}], "answer_idx": "C", "symptoms": ["A woman in her 30s with a history of bruxism and long-term use of a bite guard presented with pain centering on left submandibular area for 2 years", "She was initially prescribed a muscle relaxant without notable improvement", "Physical examination revealed a mild firmness in the left submandibular area", "The mucosa of the oral cavity and floor of mouth was normal", "Cranial nerve function, including tongue mobility and sensation, was also normal", "Contrast-enhanced computed tomography (CT) demonstrated a 1.7 × 1.5 × 1.4-cm mass, adjacent to the left submandibular gland", "Scattered, nonenlarged lymph nodes were observed (Figure, A)", "An ultrasonography-guided fine needle aspiration biopsy was nondiagnostic owing to an inability to obtain sufficient tissue", "The patient underwent surgical exploration with left submandibular gland excision and resection of the left submandibular mass (Figure, B)", "Histopathologic findings showed spindle cell neoplasm with morphologic features and no malignant neoplasm identified (Figure, C)", "Results from immunohistochemical analysis were positive for S100 (Figure, D) and negative for smooth muscle actin, CD34, and pankeratin", "A, Contrast-enhanced computed tomographic (CT) image", "B, Submandibular mass after removing the left submandibular gland", "C, Hematoxylin-eosin–stained frozen section of the mass", "D, Positive S-100 immunohistochemical stain"], "s1": [0, 1, 2, 3, 4, 5, 6, 7, 8, 9], "s2": [10, 11, 12, 13, 14]} {"key": 783, "questions": "What Is Your Diagnosis?", "options": [{"label": "A", "disease": "Metastatic nasopharyngeal carcinoma"}, {"label": "B", "disease": "Melanoma"}, {"label": "C", "disease": "Lymphoepithelioma-like carcinoma"}, {"label": "D", "disease": "Cutaneous lymphadenoma"}], "answer_idx": "C", "symptoms": ["A male smoker in his 60s presented to the dermatology department with a right supraclavicular cutaneous mass that had persisted for 2 years (Figure, A)", "This mass underwent progressive growth during the first year but subsequently stabilized", "Despite causing occasional pruritus, the lesion was otherwise asymptomatic", "A shave biopsy specimen revealed islands of large epithelioid cells with increased mitotic index surrounded by a dense lymphocytic infiltrate (Figure, B and C)", "The tumor cells expressed p16 (Figure, D) and epithelial membrane antigen (EMA), p63, and CK903, and were negative for prostate-specific antigen, CDX2, thyroid transcription factor 1, S-100, chromogranin, synaptophysin, and CD56. In situ hybridization for Epstein-Barr virus (EBV) was negative", "Otolaryngology referral was sought to rule out a head or neck primary neoplasm", "With the exception of the cutaneous neoplasm, the findings from the otolaryngologic examination were unremarkable", "A screening chest computed tomographic (CT) image showed multiple right lung nodules, the largest of which measured 2.4 cm in diameter", "A biopsy of this pulmonary nodule showed a well-differentiated adenocarcinoma with lepidic pattern, and further workup revealed an intestinal tumor as the primary source", "Positron emission tomography and CT imaging demonstrated a mildly fludeoxyglucose F 18–avid superficial supraclavicular soft-tissue mass but no primary head or neck malignant neoplasm or local nodal metastases", "A wide local excision of the cutaneous mass and a right level Vb neck dissection were performed", "The final pathology report showed residual neoplasm with negative margins and 5 negative lymph nodes", "Clinical observation was recommended", "A, 6.5 × 2.5-cm erythematous, raised, nontender lesion localized to skin without underlying tethering", "B, Carcinoma and rich lymphoplasmacytic infiltrate (original magnification ×200)", "C, Carcinoma showing anaplasia and atypical mitoses (arrowhead) (original magnification ×400)", "D, Immunohistochemical staining for p16 with strong and diffuse expression (original magnification ×200)"], "s1": [0, 1, 2, 7, 8, 9, 12, 13], "s2": [3, 4, 5, 6, 10, 11, 14, 15, 16]} {"key": 784, "questions": "What Is Your Diagnosis?", "options": [{"label": "A", "disease": "Pigmented purpuric dermatosis"}, {"label": "B", "disease": "Mycosis fungoides"}, {"label": "C", "disease": "Annular lichenoid dermatitis of youth"}, {"label": "D", "disease": "Secondary syphilis"}], "answer_idx": "C", "symptoms": ["A man in his 20s presented with a 2-month history of itchy skin lesions on his trunk, arms, and groin", "He was otherwise healthy, with no other underlying conditions, and he was taking no medications", "He had no history of atopic dermatitis or other eczematous dermatoses", "There were no associated extracutaneous symptoms", "His family medical history was relevant only in that his mother had systemic lupus erythematosus", "On physical examination, there were several erythematous annular patches on the abdomen (flanks and periumbilical region), lower back, groin, wrists, and legs (Figure, A)", "The lesions showed slightly raised red-brown borders and a clear center in some patches with no visible scaling, hypopigmentation, induration, or atrophy", "Results of laboratory studies including complete blood cell count, biochemical analysis, complement levels, antinuclear antibodies, anti-Ro, anti-La, anti-ribonucleoprotein, anti-Smith, and anti-dsDNA antibodies, as well as serologic testing for syphilis and hepatitis B and C virus, were all normal or negative", "A punch biopsy specimen was obtained for histopathologic evaluation (Figure, B and C)", "A, Clinical image shows several erythematous annular patches on the flanks and periumbilical region", "B, Histopathologic analysis of a punch biopsy sample from the abdomen showing superficial lymphocytic infiltrates in the papillary and superficial reticular dermis", "C, There are also prominent vacuolar changes at the dermoepidermal junction, and in some areas mild lymphocytic exocytosis"], "s1": [0, 1, 2, 3, 4], "s2": [5, 6, 7, 8, 9, 10, 11]} {"key": 785, "questions": "What Is Your Diagnosis?", "options": [{"label": "A", "disease": "Infective panniculitides"}, {"label": "B", "disease": "Erythema nodosum"}, {"label": "C", "disease": "Erythema induratum"}, {"label": "D", "disease": "Pyoderma gangrenosum"}], "answer_idx": "C", "symptoms": ["An 82-year-old Chinese woman presented with a 6-month history of a nonhealing ulcer and multiple indurated plaques on the legs", "The first lesion appeared as an erythematous papule, which grew and subsequently ulcerated", "Within a few months, she developed other indurated plaques on her legs", "The ulcer was painful on palpation, but the plaques were mostly asymptomatic", "The patient did not have fevers, chills, weight loss, dyspnea, chest pain, cough, or hemoptysis", "She had moved to Canada from China 30 years ago and was unsure about tuberculosis (TB) exposure and Bacillus Calmette–Guérin vaccine status", "Her medical history included atrial fibrillation, type 2 diabetes mellitus, dyslipidemia, cerebrovascular disease, hypertension, and Parkinson disease", "Physical examination showed a 5 × 4-cm, well-demarcated, oval ulcer with a violaceous, dusky border and central yellow fibrin mixed with granulation tissue on the left lateral aspect of the calf (Figure, A)", "There were numerous violaceous to dull red, indurated plaques on the posterior aspect of the legs (Figure, B)", "Wound cultures were negative for bacteria, mycobacteria, and fungi (Figure, C and D)"], "s1": [0, 1, 2, 3, 7, 8], "s2": [4, 5, 6, 9]} {"key": 786, "questions": "What Is Your Diagnosis?", "options": [{"label": "A", "disease": "Erythema ab igne"}, {"label": "B", "disease": "Prurigo pigmentosa"}, {"label": "C", "disease": "Dowling-Degos disease"}, {"label": "D", "disease": "Confluent and reticulated papillomatosis"}], "answer_idx": "B", "symptoms": ["A woman in her 20s was seen with a 6-year history of recurrent, pruritic skin eruptions involving the middle and lower back", "Before evaluation in our department, the patient had undergone a skin biopsy that showed spongiotic dermatitis, had patch testing that was reportedly negative, and received a diagnosis of atopic dermatitis", "She was treated with potent topical corticosteroids and oral prednisone, with no improvement", "She denied use of heating pads, space heaters, or other forms of external radiation", "Physical examination showed several discrete, erythematous, scaly papules admixed with light brown reticulated macules and patches (Figure, A and B)", "Punch biopsy specimens were obtained for further evaluation (Figure, C and D)", "A, Light brown macules and patches on the back show a strikingly reticular pattern", "B, Discrete erythematous and scaly papules and overlying hyperpigmented patches are shown", "C, Histopathologic examination shows eosinophilic spongiosis with interface dermatitis, dyskeratosis, and neutrophilic exocytosis (hematoxylin-eosin, original magnification ×20)", "D, Prominent dermal melanophages are present (hematoxylin-eosin, original magnification ×20)"], "s1": [0, 1, 2, 3], "s2": [4, 5, 6, 7, 8, 9]} {"key": 787, "questions": "What Is Your Diagnosis?", "options": [{"label": "A", "disease": "Glioma"}, {"label": "B", "disease": "Infarction"}, {"label": "C", "disease": "Germinoma"}, {"label": "D", "disease": "Demyelination"}], "answer_idx": "C", "symptoms": ["An adolescent boy presented with cognitive and behavioral changes, headaches, and progressively worsening weakness of the left arm and leg", "The patient was subsequently admitted to our hospital for further evaluation", "Sixteen months earlier, the patient was admitted to a hospital for weakness of the left fingers and mild spasticity of the left lower extremity", "Magnetic resonance imaging (MRI) at the time revealed a suspicious lesion in the right basal ganglia (Figure, A)", "In addition, carotid and intracranial angiography revealed no abnormalities (not shown)", "Slight weakness in the left lower extremity was detected 3 months later", "Magnetic resonance imaging revealed nonhomogeneous hyperintensity on T2-weighted images in the right basal ganglia without mass effect (Figure, B)", "Atrophy of the right cerebral hemisphere and right cerebral peduncle was observed (not shown)", "A, T2-weighted axial brain magnetic resonance imaging (MRI) at the basal ganglia level performed 16 months before the current hospitalization showing slight hyperintensity at the right putamen and posterior limb of the internal capsule (arrowhead)", "B, T2-weighted axial MRI performed 13 months before the current hospitalization showing moderate hyperintensity at the right basal ganglia without mass effect (arrowhead)", "C, T2-weighted axial MRI at the basal ganglia level showing remarkable progression of the lesions in the right basal ganglia and thalamus with heterogeneous high signal intensity and multicystic change (arrowhead)", "There was no history of tumors elsewhere, and his family history was unremarkable", "Neurological examination revealed distal left hemiparesis, left facial palsy, and decreased pinprick sensation in his left face and extremities", "On MRI, the lesion appeared larger than before (Figure, C), with remarkable heterogeneous enhancement (not shown)", "Human chorionic gonadotropin (HCG) levels in both serum and cerebrospinal fluid (CSF) were normal", "A hormone assay disclosed normal pituitary function"], "s1": [0, 1, 2, 5, 11, 12], "s2": [3, 4, 6, 7, 8, 9, 10, 13, 14, 15]} {"key": 788, "questions": "What Is Your Diagnosis?", "options": [{"label": "A", "disease": "Stickler syndrome"}, {"label": "B", "disease": "Waardenburg syndrome type 1"}, {"label": "C", "disease": "X-linked Alport syndrome"}, {"label": "D", "disease": "Autosomal dominant GJB2 gene mutation"}], "answer_idx": "B", "symptoms": ["A young boy and his parents presented for consultation about the child’s risk for development of hearing loss, given a family history of deafness in both parents", "Both parents were diagnosed with profound, nonprogressive sensorineural hearing loss within the first year of life", "A homozygous mutation in the connexin 26 gene (GJB2 [OMIM 121011]) was found in the mother, whereas the father remained without a causative diagnosis", "No other relatives had hearing impairment", "The child was born full term without complications", "Growth and language skills were appropriate for age", "Hearing screening at birth and the repeated auditory evaluations at follow-up had normal findings", "The physical examination was unremarkable except for some midfacial features consisting of a lateral displacement of the inner canthi of the eyes with a broad nasal root, small nostrils, and medial eyebrow flare (Figure, A)", "The same midfacial anomalies were observed in his father (Figure, B), in whom a hypopigmented patch on the left arm and bright blue eyes were also noticed", "Visual acuity and kidney function were normal, and no skeletal deformities were appreciated in the child or his father", "The child and his father demonstrate a lateral displacement of the inner canthi"], "s1": [0, 1, 2, 3, 4, 5, 6], "s2": [7, 8, 9, 10]} {"key": 789, "questions": "What Is Your Diagnosis?", "options": [{"label": "A", "disease": "Ulcerative colitis flare"}, {"label": "B", "disease": "Conversion to Crohn disease"}, {"label": "C", "disease": "Adenocarcinoma"}, {"label": "D", "disease": "Lymphoma"}], "answer_idx": "C", "symptoms": ["A 25-year-old woman with a history of ulcerative colitis (UC) presented to the emergency department with lower abdominal pain", "She received a diagnosis of UC 6 years earlier and has taken mesalamine to control UC", "She was discharged a week before this admission for a UC flare that was managed with prednisone", "Review of systems was notable for diarrhea", "however, she denied hematochezia, melena, emesis, fevers, or changes in urination", "Her last colonoscopy was 3 years ago, and UC without evidence of dysplasia was confirmed", "Otherwise, her medical history was unremarkable", "Family history was negative for inflammatory bowel diseases and gastrointestinal malignancies", "On examination, she was afebrile with normal vital signs", "Her abdomen was tender to palpation in the left lower quadrant, which was soft and nondistended", "The patient’s complete blood cell count and comprehensive metabolic panel results were unremarkable", "Computed tomographic scan of the abdomen/pelvis demonstrated a thickened colon from the middescending colon to the rectosigmoid", "There was also a 2.7-cm intramural abscess in the descending colon", "She started receiving piperacillin/tazobactam but began to have increased abdominal pain, emesis, and inability to tolerate oral intake", "A repeated computed tomographic scan performed 4 days after the original scan demonstrated that the intramural mass had increased to 3.5 cm and was partially obstructing the descending colon", "She underwent a colonoscopy, which demonstrated multiple areas of inflammation in the rectum through the descending colon (Figure 1)", "Nothing proximal to the midtransverse colon was visualized because of a large fecal burden"], "s1": [0, 1, 2, 3, 4, 5, 6, 7], "s2": [8, 9, 10, 11, 12, 13, 14, 15, 16]} {"key": 790, "questions": "What Is Your Diagnosis?", "options": [{"label": "A", "disease": "Metastatic neuroendocrine carcinoma"}, {"label": "B", "disease": "Metastatic gastrointestinal stromal tumor"}, {"label": "C", "disease": "Metastatic testicular carcinoma"}, {"label": "D", "disease": "Advanced desmoplastic small round cell tumor"}], "answer_idx": "D", "symptoms": ["A man in his 20s presented with a 2-month history of vague, right upper-quadrant abdominal pain, with associated 5-kg weight loss, loss of appetite, and increasing abdominal distension", "He reported a small mass in the right scrotum that was evaluated 5 years previously and was reassured to be a nonmalignant lesion", "Physical examination was notable for a distended abdomen with a palpable left abdominal mass", "Testicular examination revealed an approximately 1-cm firm right spermatic cord nodule", "Laboratory tests showed lactate dehydrogenase levels of 818 U/L (to convert to μkat/L, multiply by 0.0167), with no significant β-human chorionic gonadotropin and α-fetoprotein levels", "Computed tomography (CT) of the abdomen (Figure, A and B) revealed innumerable masses throughout the abdomen and pelvis, with the largest measuring 17.5 × 14 × 14 cm and a 3.9-cm enhancing lesion in the left hepatic lobe, and a small amount of ascites", "Testicular sonogram disclosed a 1.5-cm right epididymal cyst", "Percutaneous biopsy and histologic examination of the dominant left abdominal mass revealed neoplastic cells (Figure, C) that were positive for cytokeratin, desmin but negative for S100. Fluorescence in situ hybridization (FISH) was positive for EWS-WT1 rearrangement", "A, Coronal view and B, sagittal view contrast-enhanced computed tomographic imaging revealed innumerable abdominopelvic masses", "Blue arrowheads indicate the dominant mass and the red arrowhead indicates the liver mass", "C, Hematoxylin-eosin stain (original magnification ×40)"], "s1": [0, 1, 3, 4, 6], "s2": [2, 5, 7, 8, 9, 10]} {"key": 791, "questions": "What Is Your Diagnosis?", "options": [{"label": "A", "disease": "Branchial cleft cyst"}, {"label": "B", "disease": "Glomus tumor"}, {"label": "C", "disease": "Liposarcoma"}, {"label": "D", "disease": "Cervical lymphatic malformation"}], "answer_idx": "D", "symptoms": ["An otherwise healthy man in his 40s was referred to our ear, nose, and throat department for the evaluation of a giant, left-sided neck mass", "He reported no antecedent neck trauma or neck infection, nor any history of swelling during childhood", "He experienced dyspnea, limitation of neck movements, and neck tightness owing to the bulkiness of the lesion", "The swelling had been present for approximately 12 months, and the growth was gradual and continuous", "Contrast-enhanced T1- and T2-weighted magnetic resonance imaging (MRI) revealed a 15 × 7-cm homogenous ovoid bilobulated cystic mass in the left lateral region of the neck with displacement of the surrounding musculature and vascular structures but no sign of invasion", "The imaging of the lesion revealed fibrous septae in the central and inferior parts, with extension to the supraclavicular region (Figure, A and B)", "After written informed consent was obtained, the patient underwent total excision of the cystic mass through an apron flap incision combined with T-incision for preservation of cranial nerve XI and sternocleoidomastoid muscle (Figure, C)", "A, T1-weighted magnetic resonance image (MRI) shows a homogenous ovoid bilobulated hypointense cystic mass in left parapharyngeal area and posterior cervical region, extending to the supraclavicular triangle", "B, T2-weighted MRI demonstrates thin-walled, markedly hyperintense cystic mass indenting neighboring structures", "No fluid-fluid level is observed", "C, Cervical mass with preservation of the vital structures of the neck"], "s1": [0, 1, 2, 3], "s2": [4, 5, 6, 7, 8, 9, 10]} {"key": 792, "questions": "What Is Your Diagnosis?", "options": [{"label": "A", "disease": "Mycobacterial abscess"}, {"label": "B", "disease": "Posttransplant lymphoproliferative disorder"}, {"label": "C", "disease": "Acute invasive fungal sinusitis"}, {"label": "D", "disease": "Nasopharyngeal carcinoma"}], "answer_idx": "B", "symptoms": ["A woman in her 20s with a history of cystic fibrosis underwent bilateral endoscopic sinus surgery and medial maxillectomy 4 months after bilateral lung transplantation", "She presented to her rhinologist for follow-up 6 weeks after surgery reporting improved nasal obstruction and olfaction, but complained of 2 weeks of throat pain and swelling", "Physical examination revealed bilateral tonsillar erythema, exudate, tenderness, and peritonsillar edema", "She was prescribed amoxicillin–clavulanic acid for presumed tonsillitis and began to improve", "Two weeks later her symptoms returned, now accompanied by new-onset snoring and nasal obstruction", "Nasal endoscopy revealed nasopharyngeal soft-tissue swelling with mucopurulent secretions on the left side", "She was referred for a computed tomographic scan with contrast, which showed a rim-enhancing, multilocular hypodense collection extending from the left nasopharynx inferiorly to the level of the right palatine tonsil (Figure 1)", "On the basis of the radiographic findings, she was taken to the operating room urgently", "Intraoperative endoscopy revealed ulceration and necrotic debris in the nasopharynx and oropharynx", "Culture and biopsy specimens were obtained"], "s1": [0, 3, 4, 7, 8, 9], "s2": [1, 2, 5, 6]} {"key": 793, "questions": "What Is Your Diagnosis?", "options": [{"label": "A", "disease": "Florid osseous dysplasia"}, {"label": "B", "disease": "Paget disease of bone"}, {"label": "C", "disease": "Fibrous dysplasia"}, {"label": "D", "disease": "Gardner syndrome"}], "answer_idx": "C", "symptoms": ["An adolescent girl presented with a history of multiple, slowly growing masses of the face and head", "The lesions were first reported when she was 5 years of age and gradually increased in size", "However, her medical history was otherwise unremarkable", "On clinical examination, the lesions were hard in consistency with normal overlying skin", "There was significant bilateral exophthalmos (the left eye was more pronounced), with visual changes, and dental malocclusion", "However, there were no clinical signs of endocrinopathy", "A skeletal survey showed no involvement of other bones", "The patient’s serum calcium, phosphate, alkaline phosphatase, thyroid, and parathyroid hormone levels were within normal limits", "A computed tomographic (CT) examination of the craniofacial skeleton was performed (Figure)", "Noncontrast axial bone algorithm computed tomographic images of the facial bones"], "s1": [0, 1, 4, 5], "s2": [2, 3, 6, 7, 8, 9]} {"key": 794, "questions": "What Is Your Diagnosis?", "options": [{"label": "A", "disease": "Lipoblastoma"}, {"label": "B", "disease": "Liposarcoma, conventional myxoid variant"}, {"label": "C", "disease": "Liposarcoma, pleomorphic myxoid variant"}, {"label": "D", "disease": "Teratoma"}], "answer_idx": "C", "symptoms": ["A young boy presented to the emergency department with a 2-week history of left facial and eye pain with intermittent diplopia on upward and lateral gaze", "He was otherwise healthy and exhibited no constitutional complaints", "His remaining medical, surgical, family, and social history was noncontributory", "Examination showed a healthy-appearing child with left abducens nerve palsy and an afferent pupillary defect", "There was no chemosis, exophthalmos, or enophthalmos", "Findings from nasal endoscopy were unremarkable", "A computed tomographic scan of the sinuses without contrast showed a well-circumscribed posterior-medial extraconal mass extending into the orbital apex with smooth, nonaggressive bony remodeling and expansion of the left superior orbital fissure with further dorsal extension into the left cavernous sinus (Figure, A)", "Magnetic resonance imaging of the orbits showed that orbital component was inseparable from medial rectus muscle", "Tumor also resulted in lateral displacement and compression of the left optic nerve (Figure, B)", "The patient underwent an image-guided left endoscopic orbital decompression with biopsy of the orbital mass", "Representative hematoxylin-eosin–stained sections are shown in the Figure, C and D", "The tumor was negative for FUS (FUsed in Sarcoma", "also termed TLS, Translocated in LipoSarcoma)-CHOP (C/EBP HOmologous Protein", "also termed DDIT3, DNA Damage-Inducible Transcript 3) fusion oncogene on fluorescent in situ hybridization (FISH)", "A, Axial cut of noncontrast computed tomographic (CT) scan of facial bones", "B, Axial cut of T1-weighted magnetic resonance image (MRI)", "C and D, Representative hematoxylin-eosin–stained histologic sections of the lesion"], "s1": [0, 1, 3, 4, 5, 7, 8, 9], "s2": [2, 6, 10, 11, 12, 13, 14, 15, 16]} {"key": 795, "questions": "What Is Your Diagnosis?", "options": [{"label": "A", "disease": "Borderline lepromatous leprosy with trophic ulcer"}, {"label": "B", "disease": "Tropical phagedenic ulcer"}, {"label": "C", "disease": "Post–kala-azar dermal leishmaniasis"}, {"label": "D", "disease": "Cutaneous diphtheria"}], "answer_idx": "C", "symptoms": ["A middle-aged man sought treatment for a painful nonhealing foot ulcer of 1-year duration", "The lesion had started on the left foot as a painful blister that eventually ruptured to form an ulcer, which subsequently progressed", "During this time, multiple hypopigmented asymptomatic lesions appeared on his face and scalp", "A diagnosis of multibacillary leprosy was considered previously in another health care facility for which multidrug therapy with rifampicin, 600 mg/mo, dapsone, 100 mg/d, and clofazimine, 50 mg/d, was administered for 6 months", "There was no appreciable clinical improvement", "On physical examination, an 8 × 6-cm ulcer was noted on the lateral aspect of the dorsum of the left foot (Figure, A)", "The ulcer was well defined with regular margins, and the floor was covered by a yellowish slough", "Perilesional tenderness and pitting edema were observed", "Multiple ill-defined, hypopigmented patches of various sizes were present on the scalp and face (Figure, B)", "Lesional appendages and sensations were preserved", "Inguinal lymph nodes on both sides were enlarged, nontender, and movable", "Peripheral nerve trunks were not thickened, and peripheral arteries could be easily palpated", "Skin punch biopsy specimens were obtained from the ulcer (Figure, C and D) and a hypopigmented facial lesion", "A, Nonhealing ulcer", "B, Ill-defined hypopigmented macules on the left side of the face", "Lesional skin appendages are preserved", "C, Histologic analysis showing acute ulceration of the epidermis with granulation tissue and infiltrating neutrophils and macrophages (hematoxylin-eosin, original magnification ×100)", "D, Histologic analysis showing intracellular and extracellular inclusion bodies (hematoxylin-eosin, original magnification ×1000)"], "s1": [0, 1, 2, 3, 4, 8, 10, 11, 14], "s2": [5, 6, 7, 9, 12, 13, 15, 16, 17]} {"key": 796, "questions": "What Is Your Diagnosis?", "options": [{"label": "A", "disease": "Essential telangiectasia"}, {"label": "B", "disease": "Leukocytoclastic vasculitis"}, {"label": "C", "disease": "Cutaneous collagenous vasculopathy"}, {"label": "D", "disease": "Pigmented purpuric dermatosis, Schamberg type"}], "answer_idx": "C", "symptoms": ["A woman in her 80s presented with a 1-year history of blanchable, salmon-colored, monomorphic macules and fine telangiectasias localized to her right lower leg (Figure, A)", "The asymptomatic patch had slowly enlarged since initial onset, but remained confined to the single extremity", "Physical examination revealed agminated and discrete erythematous macules as well as interspersed fine, reticulated telangiectases over the right pretibial leg (Figure, B)", "No edema or warmth of the affected area was observed, and there was no evidence of mucosal, conjunctival, or nail involvement", "Findings on review of systems were negative for bleeding tendencies or hypercoagulability", "A punch biopsy was performed (Figure, C and D)", "A, Reticulated, blanchable, salmon-colored patch on the pretibial leg", "B, Individual fine, telangiectatic erythematous macules identified on closer inspection", "C, Concentric deposits of hyaline material surround dilated and markedly thickened capillaries within the papillary dermis (original magnification ×200)", "D, Perivascular hyaline deposits are highlighted strongly by periodic acid–Schiff stain (original magnification ×200)"], "s1": [0, 1, 2, 3, 4, 6, 7], "s2": [5, 8, 9]} {"key": 797, "questions": "What Is Your Diagnosis?", "options": [{"label": "A", "disease": "Giant cell tumor"}, {"label": "B", "disease": "Brown tumor"}, {"label": "C", "disease": "Giant cell granuloma"}, {"label": "D", "disease": "Chondromyxoid fibroma"}], "answer_idx": "B", "symptoms": ["A man in his 60s presented with severe fatigue", "His medical history was significant for primary hyperparathyroidism 6 years prior to presentation, for which the patient refused therapeutic management", "The patient was cachectic and exhibited significant muscular weakness", "Clinical examination revealed a large tumor on the distal part of the right fourth finger (Figure, A) that had gradually enlarged in recent months", "Radiography revealed complete lysis of the underlying phalanx and periosteal bone resorption in the phalangeal tuft band", "A, The patient initially presented with a large tumor on the distal part of the right fourth finger", "B, Histopathology shows numerous giant cells with multiple osteoclastlike nuclei, and (C) giant multinucleated cells dispersed in contact with fibers associated with the presence of a few small vessels", "Laboratory evaluation revealed hypercalcemia at 3.6 mmol/L (reference range for total calcium, 2.05-2.55 mmol/L [to convert to mg/dL, divide by 0.25]), hypophosphatemia at 0.65 mmol/L (reference range for phosphate, 0.87-1.45 mmol/L), and elevated serum parathyroid hormone at 380 pmol/L (reference range, 1.6-6.9 pmol/L [to convert to ng/L, multiply by 1.355])", "A chest radiograph revealed an important and unexplained deformation of the ribs", "We performed a 3-mm punch skin biopsy on the tumor and performed histopathological analysis (Figure, B and C)"], "s1": [0, 1, 2, 7, 8], "s2": [3, 4, 5, 6, 9]} {"key": 798, "questions": "What Is Your Diagnosis?", "options": [{"label": "A", "disease": "Tinea capitis"}, {"label": "B", "disease": "Psoriasis"}, {"label": "C", "disease": "Monilethrix"}, {"label": "D", "disease": "Menkes disease"}], "answer_idx": "C", "symptoms": ["A young girl presented for evaluation of hair loss", "The mother stated that the patient was born with normal-appearing dark hair that fell out days after birth", "This was replaced with “peach fuzz,” with no subsequent regrowth of normal hair", "She denied pruritus", "Family history was significant for similar hair loss in the patient’s maternal grandmother and maternal cousin", "The patient had otherwise normal development", "On physical examination, short, brittle terminal hairs of varying lengths covered the entire scalp", "Follicular hyperkeratosis and perifollicular erythema were present, most prominently on the parietal and occipital scalp (Figure, A)", "Mild perifollicular keratotic papules were noted on the posterior upper arms", "Teeth and nails showed no abnormalities", "Her eyebrows and lashes were not affected", "Results of trichography are shown (Figure, B)", "Photograph of a young patient with hair loss depicts shortened, thin hairs on the scalp of a child with follicular hyperkeratosis", "Light microscopy of hair shaft at 4 × magnification shows a beaded appearance"], "s1": [0, 1, 2, 5, 9, 10, 11, 12, 13], "s2": [3, 4, 6, 7, 8]} {"key": 799, "questions": "What Is Your Diagnosis?", "options": [{"label": "A", "disease": "Abdominal wall hematoma"}, {"label": "B", "disease": "Small-bowel perforation"}, {"label": "C", "disease": "Intramural small bowel hematoma"}, {"label": "D", "disease": "Traumatic pancreatic injury"}], "answer_idx": "C", "symptoms": ["A healthy 6-year-old boy presented to the emergency department after experiencing a bicycle handlebar injury", "He complained about abdominal pain and had vomited twice", "At physical examination, according to the Advanced Pediatric Life Support protocol, he had normal vitals results with pain during palpation of the left abdominal region without guarding", "His hemoglobin level was 12.3 g/dL (to convert to grams per liter, multiply by 10) (normal value, 10.5-16.1 g/dL), amylase 68 U/L (to convert to microkatals per liter, multiply by 0.0167) (normal value, <85 U/L), and lipase 25 U/L (normal value, <60 U/L)", "A plain chest radiography result showed no signs of traumatic injury", "Abdominal ultrasonography results showed a bowel loop with a thickened wall in the left abdominal region with no free abdominal fluid", "He was admitted to the pediatric ward for close observation", "The next day, his vomiting persisted, especially after oral intake", "Computed tomography with oral and intravenous contrast was performed (Figure 1)", "Computed tomography, axial reconstruction (A) and coronal reconstruction (B)"], "s1": [0, 3, 4, 8, 9], "s2": [1, 2, 5, 6, 7]} {"key": 800, "questions": "What Is Your Diagnosis?", "options": [{"label": "A", "disease": "Acute appendicitis"}, {"label": "B", "disease": "Salpingitis or sactosalpinx (eg, pyosalpinx or hydrosalpinx)"}, {"label": "C", "disease": "Heterotopic pregnancy"}, {"label": "D", "disease": "Nonspecific abdominal pain or pelvic inflammatory disease"}], "answer_idx": "C", "symptoms": ["A woman in her mid-20s who was 8 weeks pregnant presented to the emergency department for abdominal pain localized in the right iliac fossa (RIF), numbness in the right lower limb, and nausea and vomiting", "On examination she had tenderness in the RIF with a positive Blumberg sign", "blood testing showed leukocyte count 10 450/μL (to convert to ×109/L, multiply by 0.001) without any shift, C-reactive protein 1.29 mg/dL (to convert to nanomoles per liter, multiply by 0.331), and hemoglobin 11.4 g/dL (to convert to grams per liter, multiply by 10)", "The Alvarado score was 7. A transabdominal ultrasonographic scan showed a tubular structure in the RIF consistent with a mildly inflamed appendix and stratification of the appendiceal wall (7 mm in the proximal and 11 mm in the appendiceal distal tract)", "A small amount of free fluid was present in the Douglas pouch", "A transvaginal ultrasonographic scan confirmed normal intrauterine gestation with a vital embryo", "Working diagnosis was uncomplicated appendicitis and initial conservative management was amoxicillin clavulanate because of the potential risks of miscarriage associated with performing an avoidable appendectomy and general anesthesia during early gestation", "The patient initially improved and was discharged home 2 days later with an Alvarado score of 4", "oral antibiotic therapy was continued", "Three days later she returned to the emergency department with recurrent pain", "the clinical and ultrasonographic findings were unchanged, without evidence of complicated appendicitis, intra-abdominal fluid collection, or abscess", "The Alvarado score was 5. The patient was admitted to the surgical ward and given intravenous antibiotics", "In the next 48 hours the symptoms and pain did not resolve, although inflammatory markers were low and the patient was not septic", "The surgeon on call requested an urgent magnetic resonance imaging (MRI) investigation (Figure 1)", "Magnetic resonance imaging showed the gestational chamber and a normal intrauterine pregnancy as well as the presence of a tubular structure in the right iliac fossa (RIF) (A) and what appeared to be an inflamed, thickened appendix (9.83 mm in the proximal appendix) (B)"], "s1": [0, 1, 3, 4, 6, 7, 9, 11, 12, 14], "s2": [2, 5, 8, 10, 13]} {"key": 801, "questions": "What Is Your Diagnosis?", "options": [{"label": "A", "disease": "Metastatic squamous cell carcinoma"}, {"label": "B", "disease": "Primary laryngeal carcinoma in the setting of extensive, recurrent head and neck SCC"}, {"label": "C", "disease": "Inflammatory reaction"}, {"label": "D", "disease": "Postradiation change"}], "answer_idx": "C", "symptoms": ["A woman in her 80s with a history of recurrent oral cancer presented with a new, biopsy-confirmed, squamous cell carcinoma (SCC)", "Her oncologic history began 16 years ago with diagnosis and local resection of a lesion on the palate", "Cancer recurrence at the left buccal mucosa was identified and excised 10 years later", "She then was diagnosed with 3 recurrences of SCC in rapid succession", "Each malignant neoplasm was locally resected and the patient was also treated with adjuvant radiotherapy for the last episode", "She remained asymptomatic, and a routine surveillance clinical examination uncovered a sixth lesion at the margin of a palatal graft", "The patient’s medical history was also noted for a total thyroidectomy in the 1950s, complicated by recurrent laryngeal nerve injury", "She had a history of tobacco use and stopped smoking cigarettes in the 1970s", "Physical examination and laryngoscopy findings confirmed a painless lesion on the left side of the palate and immobility of the left vocal cord", "No other mucosal lesion was identified", "Positron emission tomography (PET) was performed to assess extent of disease and to plan treatment", "The PET scan results showed focal fludeoxyglucose F 18 localization within the known oral cavity SCC (Figure 1A) and in a small, level IIA lymph node, located lateral to the upper right internal jugular vein", "There was unexpected, prominent, metabolic activity in the medial and asymmetrically enlarged left vocal cord (Figure 1B)", "A, Positron emission tomography reveals focal fludeoxyglucose F 18 uptake in biopsy-confirmed squamous cell carcinoma of the left palatal graft margin (arrowhead)", "B, Increased metabolic activity in the left vocal cord (arrowhead)", "Primary laryngeal carcinoma in the setting of extensive, recurrent head and neck SCC"], "s1": [0, 1, 2, 3, 4, 5, 8, 9, 10, 11, 13], "s2": [6, 7, 12, 14, 15]} {"key": 802, "questions": "What Is Your Diagnosis?", "options": [{"label": "A", "disease": "Glomus tumor"}, {"label": "B", "disease": "Myofibroma"}, {"label": "C", "disease": "Solitary fibrous tumor"}, {"label": "D", "disease": "Myopericytoma"}], "answer_idx": "D", "symptoms": ["A woman in her 40s presented with a 2-month history of a slowly enlarging, asymptomatic anterior septal mass", "She denied a history of localized trauma, substance abuse, or prior endonasal surgery", "Examination revealed a nontender, 8-mm, pink-red, soft, fleshy growth just posterior to the membranous septum near the nostril apex", "Histopathological analysis demonstrated a well-circumscribed submucosal lesion comprised of bland, round, syncytial cells, arranged in a concentric fashion around variably sized slit-like vascular spaces, a minority of which were staghorn in appearance (Figure, A-C)", "No significant perivascular hyalinization was identified", "The overlying nasal mucosa and surrounding submucosal glands were unremarkable", "Immunohistochemical stains demonstrate the cellular population to be strongly positive for smooth-muscle actin (SMA), muscle-specific actin, and CD34 (Figure, D), but negative for B-catenin and epithelial membrane antigen", "Histopathology of asymptomatic septal mass shows (A) a well-circumscribed, unencapsulated lesion based in the submucosa with an intact overlaying squamous and respiratory mucosa (hematoxylin-eosin)", "(B) bland, vaguely spindled cells arranged around prominent thin-walled vessels with a “staghorn” branching configuration (hematoxylin-eosin)", "(C) the lesional cells are oval to spindle-shaped with abundant cytoplasm and ill-defined cell borders (hematoxylin-eosin)", "(D) the lesional cells are stained diffusely with myoid markers, including smooth-muscle actin and muscle-specific actin (muscle-specific actin stain)"], "s1": [0, 1, 2], "s2": [3, 4, 5, 6, 7, 8, 9, 10]} {"key": 803, "questions": "What Is Your Diagnosis?", "options": [{"label": "A", "disease": "Merkel cell carcinoma"}, {"label": "B", "disease": "Squamous cell carcinoma"}, {"label": "C", "disease": "Small-cell carcinoma"}, {"label": "D", "disease": "Primitive neuroectodermal tumor"}], "answer_idx": "C", "symptoms": ["A man in his 60s presented with a 6-month history of dysphagia, a 3-month history of a 2.268-kg weight loss (to convert to pounds, divide by 0.454), and a right neck mass", "Tobacco history was minimal and distant, and alcohol use was social only", "Physical examination, including flexible nasolaryngoscopy, was notable for a large smooth nonulcerated right base of tongue mass with almost complete pharyngeal airway obstruction, and a 4-cm right level II lymph node", "Computed tomography (CT) imaging revealed a 3-cm base of tongue–lingual tonsil mass with multiple right Ib, IIa, and II/III nodes, the largest of which was 2.0 cm by 3.0 cm", "Awake tracheotomy and direct laryngoscopy with biopsy of right base of tongue mass lesion was performed", "Histological analysis showed sheets and clusters of small round-oval uniform cells with minimal cytoplasm, granular chromatin pattern, and prominent nuclear molding (Figure, A) with positive immunoreactivity for cytokeratin (CK) 7 (Figure, B) and synaptophysin (Figure, C)", "a high mitotic rate was also exhibited (Figure, D)", "The cells were negative for CK 5 and 6, CK 20, NUT, p63, and chromogranin", "Polymerase chain reaction (PCR) detected human papillomavirus (HPV) 16 DNA and did not detect HPV 6, 11, and 18 DNA", "Histopathological analysis of the tongue lesion shows (A) sheet-like growth pattern with Azzopardi-type crush artifactual distortion of the neoplastic cells (original magnification ×100) (the insert shows round-oval compact molded nuceli, a high nuclear:cytoplasmic ratio, and granular chromatin pattern without conspicuous nucleoli [original magnification ×400])", "B, cytokeratin 7–positive immunoreactivity (original magnification ×200)", "C, synaptophysin-positive tumor cells (original magnification ×200)", "and D, a high proliferation rate by Ki-67 staining (original magnification ×200)"], "s1": [0, 1, 2, 3, 4], "s2": [5, 6, 7, 8, 9, 10, 11, 12]} {"key": 804, "questions": "What Is Your Diagnosis?", "options": [{"label": "A", "disease": "Multiple myeloma"}, {"label": "B", "disease": "Paget disease"}, {"label": "C", "disease": "Intradiploic meningoencephalocele"}, {"label": "D", "disease": "Fibrous dysplasia"}], "answer_idx": "C", "symptoms": ["A man in his 60s presented to the otolaryngology clinic with a 1-month history of clear drainage from his left naris during activity", "Approximately 1 month prior he had been admitted to the hospital and treated for meningitis", "During his hospital course, a computed tomographic (CT) scan of his head revealed a lytic lesion of the left occipital bone", "After treatment with intravenous antibiotics and steroids, his neurologic examination results returned to baseline, and he was discharged home", "Magnetic resonance imaging (MRI) was deferred to the outpatient setting", "At the otolaryngologist’s office, the patient stated that he had not experienced otalgia, otorrhea, tinnitus, or vertigo", "His examination revealed clear nasal discharge and a normal otoscopic finding", "His audiogram demonstrated normal sloping to moderate sensorineural hearing loss with type A tympanometry result on the right and type A on the left", "His word recognition score was 100% bilaterally", "The collection of nasal discharge for β-2 transferrin analysis was attempted, but insufficient quantity was obtained", "To evaluate for a skull base defect and cerebrospinal fluid (CSF) leak, a noncontrast CT scan of the temporal bones was obtained (Figure, A and B)", "This demonstrated a large area of bony destruction and osteolysis involving the left occipital bone", "A T2-weighted MRI sequence showed a lesion, isointense to CSF, with thinning of the bony cortex (Figure, C and D)", "Based on the imaging, the differential diagnosis included metabolic, infectious, or metastatic lesions to the bone", "A and B, Noncontrast temporal bone computed tomographic images", "C, T2-weighted fast imaging using steady state acquisition magnetic resonance imaging sequence demonstrating a signal isointense to cerebrospinal fluid in the left occipital bone", "D, Reformatted postcontrast T1 image shows the herniation of nonenhancing meninges through the defect"], "s1": [0, 1, 3, 5, 6, 7, 8, 9], "s2": [2, 4, 10, 11, 12, 13, 14, 15, 16]} {"key": 805, "questions": "What Is Your Diagnosis?", "options": [{"label": "A", "disease": "Erythema annulare centrifugum"}, {"label": "B", "disease": "Interstitial granulomatous dermatitis"}, {"label": "C", "disease": "Granuloma annulare"}, {"label": "D", "disease": "Leukemia cutis"}], "answer_idx": "D", "symptoms": ["A toddler with a history of neurofibromatosis type 1 (NF1) and juvenile xanthogranulomas presented to the clinic for an eruption on his trunk that began 2 months previously with multiple round red patches that had been slowly increasing in size", "Treatment with griseofulvin and clotrimazole had not improved the rash", "One day prior to presentation he had developed a fever and was diagnosed with otitis media in the emergency department", "Acetaminophen, ibuprofen, and amoxicillin were used for treatment, but the child was receiving no other medications", "Physical examination revealed multiple annular, indurated, dusky red plaques with central clearing and trailing scale ranging from 1 to 6 cm in diameter on his trunk (Figure, A and B)", "A 4-mm punch biopsy of the largest lesion on his back was performed and sent for histopathologic examination (Figure, C)", "A and B, Annular dusky red indurated plaques on the right flank with central clearing and trailing scale", "Scattered café au lait macules and annular dusky red indurated plaques with central clearing and trailing scale on the back", "C, Hematoxylin-eosin stain (original magnification ×40) demonstrates perivascular and periadnexal infiltrate", "D, Hematoxylin-eosin stain (original magnification ×400)"], "s1": [0, 2, 3], "s2": [1, 4, 5, 6, 7, 8, 9]} {"key": 806, "questions": "What Is Your Diagnosis?", "options": [{"label": "A", "disease": "Cutaneous lupus erythematosus"}, {"label": "B", "disease": "Cutaneous microvascular occlusion secondary to essential thrombocytosis"}, {"label": "C", "disease": "Hydroxyurea-induced dermatomyositis-like eruption"}, {"label": "D", "disease": "Polyarteritis nodosa"}], "answer_idx": "C", "symptoms": ["A 69-year-old woman presented for the evaluation of a slowly progressive, asymptomatic cutaneous eruption on her hands and forearms, as well as an enlarging, painful ulcer on her left leg that developed 6 months earlier", "Her medical history was notable for essential thrombocytosis, which had been complicated by portal vein thrombosis, and she had been receiving treatment with hydroxyurea and rivaroxaban for the past 2 years", "Despite worsening cutaneous disease, she otherwise felt well and denied associated fevers, weakness, weight loss, myalgias, arthralgias, or other constitutional symptoms", "Physical examination revealed atrophic and reticulated violaceous plaques with a striking photoaccentuated distribution on the patient’s dorsal hands and forearms (Figure, A)", "Scaly, violaceous plaques were also present on the thenar eminences and radial aspects of her first 2 fingers bilaterally (Figure, B)", "Prominent poikiloderma was seen on her sun-exposed chest and neck, and a large, stellate ulcer was present on her left distal lower leg (Figure, C)", "A lesional skin biopsy was performed on the patient’s left forearm (Figure, D)", "A, Clinical photograph of violaceous reticulated plaques on the dorsal aspect of the hands", "B, Clinical photograph of violaceous, scaly plaques on the thenar and radial aspects of the thumb and second digit (mechanic hands)", "C, Clinical photograph of a shallow ulcer with stellate margins on the left distal lower leg", "D, Histopathology (hematoxylin-eosin stain, original magnification x100)"], "s1": [0, 1, 2, 6, 10], "s2": [3, 4, 5, 7, 8, 9]} {"key": 807, "questions": "What Is Your Diagnosis?", "options": [{"label": "A", "disease": "Cutis marmorata telangiectatica congenita (van Lohuizen syndrome)"}, {"label": "B", "disease": "Systemic lupus erythematosus"}, {"label": "C", "disease": "Ataxia-telangiectasia (Louis-Bar syndrome)"}, {"label": "D", "disease": "Sneddon syndrome"}], "answer_idx": "D", "symptoms": ["A woman in her 40s presented with a 4-hour history of ataxia and left-sided hemidysesthesia", "Annular patches of violaceous erythema forming an irregular net-like pattern were noted on her trunk and extremities (Figure, A and B)", "The patient stated that these skin changes had appeared 2 years prior", "Her medical history was significant for 5 strokes over the past 6 years and Raynaud syndrome for the past 2 years", "In the past, despite extensive workup, no definitive diagnosis had been established", "Complete blood cell count, liver function tests, creatinine, C-reactive protein, erythrocyte sedimentation rate, and urinalysis results were normal", "Antinuclear antibodies and complement C3 and C4 levels were unremarkable", "Antiphospholipid antibody and lupus anticoagulant screening results were repeatedly negative and no evidence of thrombophilia was found", "Magnetic resonance imaging showed acute dot-like diffusion anomalies in the medial and posterior cerebral artery-supplied areas, as well as old stroke residues (Figure, C)", "Transesophageal echocardiography showed no abnormalities", "A skin biopsy sample was taken from the patient’s left thigh (Figure, D)", "A, Patient’s lower back and buttocks and, B, thighs and upper calves show generalized, irregular, violaceous, net-like erythema", "C, Magnetic resonance imaging of the brain showing old stroke residues", "D, Hematoxylin-eosin stain (original magnification ×10)"], "s1": [0, 3, 5, 6, 7, 8, 9, 12], "s2": [1, 2, 4, 10, 11, 13]} {"key": 808, "questions": "What Is Your Diagnosis?", "options": [{"label": "A", "disease": "Multiple sclerosis"}, {"label": "B", "disease": "Central nervous system lymphoma"}, {"label": "C", "disease": "Autoimmune encephalitis"}, {"label": "D", "disease": "Neurosarcoidosis"}], "answer_idx": "C", "symptoms": ["A young girl with an unremarkable medical history presented with 1 week of drowsiness, double vision, and left cranial nerve VI and VII palsies", "Brain magnetic resonance imaging (MRI) revealed T2-weighted and fluid-attenuated inversion recovery changes with associated patchy enhancement in the brainstem, left frontal lobe, left cingulate gyrus, and left parietal lobe without restricted diffusion (Figure, A and B)", "Cervical spine imaging results were negative", "Cerebral spinal fluid (CSF) study results were unremarkable (red blood cell count, 0 × 106/µL [to convert to ×1012/L, multiply by 12]", "white blood cell count, 0/µL [to convert to ×109/L, multiply by 0.001]", "glucose, 61 mg/dL [to convert to millimoles per liter, multiply by 0.0555]", "and total protein, 19 g/dL [to convert to grams per liter, multiply by 10]), and CSF cytologic test results were negative for malignancy", "Oligoclonal bands and IgG index were not measured", "The results of serum cell-based aquaporin 4 antibody tests were negative", "She was diagnosed with acute demyelinating encephalomyelitis (ADEM) and received 5 days of high-dose intravenous corticosteroids followed by a 1-month course of oral prednisone with improvement back to baseline in 3 weeks", "Brain magnetic resonance imaging during initial presentation: T2-weighted and fluid-attenuated inversion recovery (FLAIR) hyperintensity in the left frontal lobe (A) and associated enhancement on T1-weighted postcontrast imaging (B)", "Brain magnetic resonance imaging during second attack: resolution of the initial left frontal lobe lesion now with T2-weighted and FLAIR recovery hyperintensity in the left parietotemporal region (C) with associated patchy enhancement (D)", "She presented again, 15 months later, with 2 weeks of dysarthria and progressive ataxia without change in mental status", "Subsequent brain MRI revealed resolution of the prior supratentorial involvement and new T2/fluid-attenuated inversion recovery changes with patchy enhancement in the brainstem and a large new lesion in the left parietotemporal area (Figure, C and D)", "Head magnetic resonance angiography findings were unremarkable", "The CSF studies revealed a mild lymphocytic pleocytosis (white blood cell count of 11/µL with 3 neutrophils, 6 lymphocytes, and 2 monocytes) with 1 red blood cell, glucose level of 57 mg/dL, and total protein level of 39 g/dL", "The results of the CSF viral polymerase chain reaction studies, including cytomegalovirus, varicella-zoster virus, herpes simplex virus, enterovirus, and Epstein-Barr virus, were negative", "The CSF bacterial culture returned without growth", "The results of tests for the CSF oligoclonal bands and IgG index were negative", "The patient received pulse dose corticosteroids (intravenous solumedrol, 30 mg/kg per dose every 24 hours) for 5 days with resolution of her truncal ataxia during hospitalization", "She was discharged with a month-long oral prednisone taper starting at 20 mg/d", "When tapered down to prednisone, 5 mg every other day, she developed a right facial droop with recurrent ataxia"], "s1": [0, 12, 9, 19, 20, 21], "s2": [1, 2, 3, 4, 5, 6, 7, 8, 10, 11, 13, 14, 15, 16, 17, 18]} {"key": 809, "questions": "What Is Your Diagnosis?", "options": [{"label": "A", "disease": "Congenital nevus"}, {"label": "B", "disease": "Melanoma"}, {"label": "C", "disease": "Angiokeratoma"}, {"label": "D", "disease": "Mastocytoma"}], "answer_idx": "C", "symptoms": ["A 16-year-old boy presented with a changing skin lesion on his right lower back", "The patient reported the presence of a stable red lesion since he was 7 years old", "however, over the preceding month, this lesion developed new black spots within it", "There was no associated pain, pruritus, or bleeding", "His family history was notable for dysplastic nevus syndrome in his father and melanoma in a paternal uncle and cousin", "On examination of the right lower back, there was a cluster of 1-mm flat purple-black papules coalescing into a 1.5 × 1.0-cm oval plaque with surrounding erythema (Figure, A)", "On reinspection of the lesion at a subsequent visit, the morphology had significantly changed", "there were 1-mm skin-colored to pink thin-walled vesicles clustered within a 1.5 × 1.0-cm oval telangiectatic plaque (Figure, B)", "A, Flat purple-black papules coalescing into an oval patch with surrounding erythema", "B, Skin-colored to pink thin-walled vesicles making up an oval telangiectatic patch"], "s1": [0, 1, 2, 4], "s2": [3, 5, 6, 7, 8, 9]} {"key": 810, "questions": "What Is Your Diagnosis?", "options": [{"label": "A", "disease": "Gynecomastia"}, {"label": "B", "disease": "Lipoma"}, {"label": "C", "disease": "Carcinoma"}, {"label": "D", "disease": "Epidermal inclusion cyst"}], "answer_idx": "C", "symptoms": ["A previously healthy 40-year-old man presented with a 3-month history of a right breast mass slowly enlarging with associated pain", "The patient denied history of local trauma, and his family history was negative regarding breast or ovarian cancers", "The patient had no history of liver disease, hormonal therapy, or radiation to the chest wall", "The physical examination disclosed an obese man with no signs of hypogonadism or liver failure", "There was a 3 × 4-cm hard, irregular, mobile nodule in the retroareolar area of the right breast, tethered to the overlying skin but not fixed to the underlying muscle", "There were no other masses on the chest wall", "The ipsilateral lymph nodes were palpable", "The left breast and axilla were normal", "Right mediolateral oblique mammography (Figure, A) and computed tomography scan of the chest (Figure, B) are shown", "A, Right mediolateral oblique view of male breast showing a breast mass and axillary lymph nodes", "B, Computed tomography of the chest showing the right breast mass involving the skin but not the pectoralis muscle"], "s1": [0, 1, 2, 3, 5, 7], "s2": [4, 6, 8, 9, 10]} {"key": 811, "questions": "What Is Your Diagnosis?", "options": [{"label": "A", "disease": "Inflammatory bowel disease"}, {"label": "B", "disease": "Small-bowel neoplasm"}, {"label": "C", "disease": "Mesenteric venous thrombosis"}, {"label": "D", "disease": "Superior mesenteric artery thromboembolism"}], "answer_idx": "C", "symptoms": ["A 68-year-old African American man presented to the emergency department with 72 hours of progressively worsening diffuse abdominal pain, distension, nausea, and nonbilious emesis", "He was previously healthy and had no prior abdominal operations", "The patient noted pencil-thin stools that occurred during the past several months but had not had a colonoscopy in more than a decade", "The rest of his history was noncontributory", "He was mildly tachycardic but normotensive and afebrile", "Physical examination revealed a lethargic patient with a distended abdomen, diffuse rebound tenderness, and involuntary guarding", "No surgical scars or inguinal hernias were apparent on examination", "The only abnormal laboratory values were as follows: sodium concentration, 126 mEq/L (reference range, 135-145 mEq/L) (to convert to millimoles per liter, multiply by 1)", "lactate concentration, 58.6 mg/dL (reference range, 0-18.0 mg/dL) (to convert to millimoles per liter, multiply by 0.111)", "and white blood cell count, 18 500/μL (reference range, 3400-10 000/μL) (to convert to ×109/L, multiply by 0.001)", "He was resuscitated with 2 L of lactated Ringer solution and underwent computed tomography of the abdomen and pelvis with intravenous contrast", "Computed tomography revealed diffuse free fluid but without free air and several loops of dilated small bowel with thickened, hypoenhancing bowel walls", "The mesentery was markedly edematous (Figure)", "The rest of the small bowel were dilated and air and fluid filled but had normal bowel wall enhancement", "The colon was normal", "Axial computed tomography image of the abdomen and pelvis", "Blue arrowhead indicates intravenous contrast demonstrating mesenteric edema", "yellow arrowhead, hypoenhancing bowel walls", "and white arrowhead, a contrast filling defect"], "s1": [0, 1, 2, 3, 4, 5, 6, 10], "s2": [7, 8, 9, 11, 12, 13, 14, 15, 16, 17, 18]} {"key": 812, "questions": "What Is Your Diagnosis?", "options": [{"label": "A", "disease": "Abdominal aortic aneurysm rupture"}, {"label": "B", "disease": "Spontaneous perinephric hematoma"}, {"label": "C", "disease": "Splenic rupture"}, {"label": "D", "disease": "Pyelonephritis"}], "answer_idx": "B", "symptoms": ["A woman in her 50s presented to the emergency department with acute, severe, left flank pain", "There was no medical history of trauma, fever, or hematuria", "She was receiving therapy for mantle cell lymphoma and had previously undergone 1 cycle of the Nordic protocol (comprising cyclophosphamide, doxorubicin, vincristine, prednisolone, cytarabine, and rituximab).1 However, this resulted in tumor lysis syndrome and neutropenic sepsis", "The patient was reluctant to undergo further infusional therapy and was switched to ibrutinib, 560 mg, once daily", "During the 2-month period that she was receiving ibrutinib therapy, there were no concurrent administrations of anticoagulants", "At presentation, she was hypotensive (blood pressure, 79/45 mm Hg) with conjunctival pallor and left renal angle tenderness", "The patient’s hemoglobin count was low (7.6 g/dL", "reference range, 12-16 g/dL", "to convert to g/L, multiply by 10.0), which represented a significant decrease from the baseline (10.6 g/dL) measured 1 month before presentation", "She was also found to have acute-onset thrombocytopenia (40×109/L", "reference range, 140-400×109/L) and leukocytosis (12.75×109/L, reference range, 4-10×109/L)", "Her coagulation profile (prothrombin time) was normal", "A computed tomographic (CT) scan of the abdomen and pelvis was performed (Figure 1A) and compared with a similar scan performed 4 months prior (Figure 1B)", "A, Findings from computed tomographic scan at presentation", "B, Previous findings from computed tomographic scan"], "s1": [0, 1, 5, 6, 7, 8, 9, 10, 11], "s2": [2, 3, 4, 12, 13, 14]} {"key": 813, "questions": "What Is Your Diagnosis?", "options": [{"label": "A", "disease": "Biphenotypic sinonasal sarcoma"}, {"label": "B", "disease": "Solitary fibrous tumor"}, {"label": "C", "disease": "Malignant peripheral nerve sheath tumor"}, {"label": "D", "disease": "Synovial sarcoma"}], "answer_idx": "A", "symptoms": ["A woman in her 30s presented with a 7-month history of right-sided nasal obstruction", "She had been treated with several courses of oral antibiotics and intranasal steroids without improvement of symptoms", "She had no history of sinus infections or allergic rhinitis", "Nasal obstruction was constant and associated with right paranasal pressure", "Results from a head and neck examination, including cranial nerve, were normal, except for the results from nasal endoscopy, which showed a deviated septum and a soft-tissue mass emanating from the right middle meatus inferior to the middle turbinate that seemed to have a polypoid appearance superiorly and a papillomatous appearance inferiorly", "A computed tomographic (CT) scan of the sinuses without contrast showed complete opacification of the right paranasal sinuses with mass effect on the medial maxillary wall with mild hyperostosis posterolaterally", "The patient underwent an endoscopic right sinonasal mass biopsy", "Intraoperatively, further excision of the mass was aborted owing to an unusual appearance of the mass as well as clear fluid exuding from the mass during microdebridement", "Pathologic analysis of the biopsied tissue revealed a uniform spindle cell tumor comprised of cellular fascicles and tumor cells with bland nuclei (Figure, A and B)", "A magnetic resonance image (MRI) of the face with contrast was obtained postoperatively to further delineate the mass and evaluate for intracranial extension", "Initial histologic stains showed the tumor to be positive for S-100 and actin (Figure, C and D) and negative for GFAP, CD34, and cytokeratin", "Additional immunostains showed the tumor to be negative for SOX-10 and B-catenin", "Histopathologic images", "A and B, Hematoxylin-eosin", "C, S-100 immunohistochemical stain", "D, Smooth muscle actin immunohistochemical stain"], "s1": [0, 1, 2, 3, 4, 5, 6, 7], "s2": [8, 9, 10, 11, 12, 13, 14, 15]} {"key": 814, "questions": "What Is the Diagnosis?", "options": [{"label": "A", "disease": "Mucoepidermoid carcinoma"}, {"label": "B", "disease": "Infectious pseudotumor"}, {"label": "C", "disease": "Histiocytic necrotizing lymphadenitis"}, {"label": "D", "disease": "Granulomatosis with polyangiitis"}], "answer_idx": "B", "symptoms": ["A young boy presented with a 4-week history of an enlarging right facial mass, which had not responded to a course of azithromycin prescribed by his pediatrician", "He had a history of asthma, chronic otitis media with effusion, and obstructive sleep apnea and had undergone 3 sets of tympanostomy tubes and an adenotonsillectomy", "He had no history of fevers or other constitutional symptoms", "Physical examination demonstrated a 3-cm firm and tender right parotid mass", "Clear saliva could be expressed from the Stensen duct", "A computed tomographic (CT) scan was performed, showing a discrete 2 × 2-cm necrotic mass in the right parotid gland (Figure, A)", "A fine-needle aspirate (FNA) yielded a cellular sample with spindloid features, suggesting a neoplasm of uncertain subtype", "A broad differential diagnosis was raised, including both benign and malignant salivary neoplasms", "Per recommendation of the head and neck multidisciplinary tumor board, the patient underwent total parotidectomy and neck dissection because the mass was grossly infiltrative of the parotid, and several enlarged level IB and II lymph nodes were observed at the time of surgery", "The mass was shaved off the marginal mandibular branch of the facial nerve because it was adherent to it", "Permanent histopathologic images showed normal parotid gland architecture effaced by confluent granulomata with Langhans-type giant cells and central necrosis (Figure, B)", "Additional studies, including flow cytometry and tissue cultures, were obtained", "A, Coronal computed tomographic (CT) image demonstrating right parotid mass and level II adenopathy", "B, The normal parotid gland architecture is effaced by confluent granulomata with necrosis (hematoxylin-eosin, original magnification ×200)"], "s1": [0, 1, 2, 3, 4, 7], "s2": [5, 6, 8, 9, 10, 11, 12, 13]} {"key": 815, "questions": "What Is Your Diagnosis?", "options": [{"label": "A", "disease": "Laryngotracheitis (croup)"}, {"label": "B", "disease": "Subglottic cyst"}, {"label": "C", "disease": "Subglottic stenosis"}, {"label": "D", "disease": "Subglottic hemangioma"}], "answer_idx": "B", "symptoms": ["A 7-month-old infant presented to the otolaryngology clinic for noisy breathing", "His noisy breathing had been present since birth, constant, exacerbated with feeding, sleeping, and supine positioning", "He fed poorly from the bottle with frequent choking, grunting, and irritability, and had minimal weight gain with 1 cyanotic event while feeding", "His medical history was significant for prematurity, born at 26 weeks, and a 60-day intubation during a 99-day neonatal intensive care unit stay", "The physical examination was remarkable only for biphasic stridor and a weak cry", "Nasopharyngolaryngoscopy showed no laryngomalacia, laryngeal lesions, or cord paralysis", "Shortly after this visit, the patient presented to the emergency department with increased work of breathing, tachypnea, retractions, weak cry, and stridor", "Examination was significant for subcostal retractions, and biphasic stridor", "Due to a lack of response to racemic epinephrine, albuterol, and steroids, he was admitted to the intensive care unit for heliox and intravenous steroids, resulting in mild improvements", "A viral panel was positive for rhinovirus", "A few days later he was taken to the operating room for direct laryngoscopy and bronchoscopy", "During sedation, a prolonged expiratory phase with quick chest wall rise on inspiration and slow fall with expiration was observed", "Capnography showed a restrictive “shark fin” waveform (Figure, A)", "The direct laryngoscopy and bronchoscopy were performed (Figure, C and D)", "Images show the (A) preintervention and (B) postintervention capnography wave form as seen on the anesthesia monitor", "Endoscopy images on (C) expiration and (D) inspiration"], "s1": [0, 1, 2, 3, 4, 6, 7, 8], "s2": [5, 9, 10, 11, 12, 13, 14, 15]} {"key": 816, "questions": "What Is Your Diagnosis?", "options": [{"label": "A", "disease": "Deep fungal infection"}, {"label": "B", "disease": "Cutaneous malakoplakia"}, {"label": "C", "disease": "Granular cell tumor"}, {"label": "D", "disease": "Metastatic prostate cancer"}], "answer_idx": "B", "symptoms": ["A man in his 60s presented with a 3-month history of intertrigo involving his groin that he had been treating with a combination of clotrimazole cream, 1%, and hydrocortisone cream, 1%", "His intertrigo improved, but he then developed bright red, burning and painful nodules around his scrotum (Figure 1)", "His medical history included prostate cancer, hypertension, anxiety disorder, hyperlipidemia, gastroesophageal reflux disease, and diverticulosis", "His medications included omeprazole, alprazolam, amlodipine besylate, atenolol, simvastatin, and valsartan", "On physical examination, there were 2 symmetrical solitary erythematous nodules in the right and left inguinal folds adjacent to the scrotum", "Clinical image of well-demarcated erythematous solitary papule in the right inguinal fold"], "s1": [0, 1, 4, 5], "s2": [2, 3]} {"key": 817, "questions": "What Is Your Diagnosis?", "options": [{"label": "A", "disease": "Psoriasis"}, {"label": "B", "disease": "Lupus erythematosus"}, {"label": "C", "disease": "Secondary syphilis"}, {"label": "D", "disease": "T-cell lymphoma"}], "answer_idx": "C", "symptoms": ["A previously healthy man in his 30s was referred to our department for evaluation of asymptomatic cutaneous lesions that had been present for 4 months", "He had been treated with a course of high-potency topical corticosteroids without any improvement", "Physical examination revealed annular red plaques with scale and superficial erosions over the trunk and extremities", "Strikingly, plaques on both arms showed a corymbiform pattern, with a central and multiple satellite lesions (Figure, A)", "On the lower limbs, infiltrated plaques with prominent crusts were seen (Figure, B)", "Also, there were papulosquamous lesions on the left sole, but the palms were spared", "The genital area was not involved", "There was neither regional lymph node enlargement nor systemic involvement", "Two biopsy specimens from both types of lesions were obtained for histopathologic evaluation, showing an identical histologic pattern (Figure, C and D)"], "s1": [0, 1, 8], "s2": [2, 3, 4, 5, 6, 7]} {"key": 818, "questions": "What Is Your Diagnosis?", "options": [{"label": "A", "disease": "Sporotrichosis"}, {"label": "B", "disease": "Subcutaneous sarcoidosis"}, {"label": "C", "disease": "Tumoral calcinosis"}, {"label": "D", "disease": "Erythema nodosum"}], "answer_idx": "B", "symptoms": ["A 40-year-old man of Scandinavian descent presented with enlarging, nonpainful upper extremity nodules developing over the preceding 9 months (Figure, A)", "He was renovating a commercial cannabis grow-house in California at time of onset", "He described the construction site as being filled with intoxicating mold", "He reported shortness of breath and a 20-pound weight loss", "He denied recent travel or aquarium exposure", "Most concerning was the inability to use his right upper extremity", "Also, he developed a pathological fracture of his right foot (Figure, B)", "Clinical examination of his forearms revealed several yam-size (4-6 cm), nontender, nonulcerated, boggy subcutaneous nodules in a sporotrichoid pattern and large nodules scattered on his trunk and abdomen", "There was no palpable lymphadenopathy or lower extremity lesions", "His medical history is significant for diabetes mellitus type 2 and hypothyroidism", "Medications included glipizide, metformin, and levothyroxine", "A, Nodules are 4 to 6 cm in diameter on the right upper extremity", "B, A pathological fracture on the fifth metatarsal on the right foot is indicated by the white arrowhead", "C, Histopathology shows pale, nodular aggregations of inflammatory cells in the deep dermis, extending into the fibrotic subcutis, as well as (D) aggregations composed of epitheliod histiocytes with few other inflammatory cells, so called “naked granulomas", "”The patient’s laboratory tests showed fasting glucose of 122.0 mg/dL (reference range, 65.0-115.0 mg/dL)", "total calcium, 10.2 mg/dL (reference range, 8.5-10.5 mg/dL)", "blood urea nitrogen, 16.0 mg/dL (reference range, 10.0-24.0 mg/dL)", "creatinine, 1.30 mg/dL (reference range, 0.6-1.3 mg/dL)", "hemoglobin A1c, 6.7 (reference range, 4.0-6.0)", "1,25-OH vitamin D 70, ng/mL (reference range, 15.9-55.6)", "Liver enzymes were within normal limits along with a negative purified protein derivative and quantiferon test results", "A culture from a cutaneous nodule with microbial stains for bacteria, fungal culture, and acid-fast bacilli was negative", "Computed tomography of the chest revealed hilar lymphadenopathy", "An incisional biopsy of a nodule on the right wrist was obtained, which showed nodular aggregations of epithelioid histiocytes with sparse peripheral lymphocytes, extending from the deep dermis into the subcutis (Figure C and D)", "Periodic acid–Schiff diastase and Fite stains were negative for fungal and mycobacterial organisms, respectively"], "s1": [0, 1, 2, 3, 5, 6, 7, 8, 11, 12, 22, 23], "s2": [4, 9, 10, 13, 14, 15, 16, 17, 18, 19, 20, 21, 24]} {"key": 819, "questions": "What Is Your Diagnosis?", "options": [{"label": "A", "disease": "Glaucoma"}, {"label": "B", "disease": "Vitamin A deficiency"}, {"label": "C", "disease": "Sjögren syndrome"}, {"label": "D", "disease": "Bilateral cataracts"}], "answer_idx": "B", "symptoms": ["An 11-year-old boy of East-Asian descent presented to the hospital with an 8-month history of progressive vision loss, night blindness, and light sensitivity", "He also had a long-standing complaint of dry eyes", "His medical history included diagnoses of eczema and multiple food allergies", "Owing to concerns about food triggers of his eczema, the child had a restrictive diet consisting solely of potato, pork, lamb, apples, cucumber, and Cheerios", "There was no history of redness, discharge, or trauma to the eyes", "The vision loss was not associated with pain, headache, fever, or photopsias (flashes of light)", "On examination, the child’s visual acuity was limited to detecting hand motion at 30 cm", "His conjunctiva had dry, keratinized patches with foamy appearance (Figure 1A)", "The cornea showed areas of marked epitheliopathy with opacity (Figure 1B)", "His pupils were equal and reactive to light with no relative afferent pupillary defect", "His funduscopic examination showed mild optic nerve pallor", "He had normal extraocular movements", "A, Photograph of the patient’s eye before treatment", "Arrowheads indicate dry keratinized patches (Bitot spots)", "B, Photograph of the patient’s eye before treatment demonstrating marked corneal epitheliopathy with opacity"], "s1": [0, 5, 10, 11, 12, 13, 14], "s2": [1, 2, 3, 4, 6, 7, 8, 9]} {"key": 820, "questions": "What Is Your Diagnosis?", "options": [{"label": "A", "disease": "Desmoid tumor"}, {"label": "B", "disease": "Acute appendicitis"}, {"label": "C", "disease": "Lymphoma"}, {"label": "D", "disease": "Carcinoid tumor"}], "answer_idx": "B", "symptoms": ["A 48-year-old man presented to the emergency department with a 1-week history of worsening right lower quadrant tenderness and fullness", "This was associated with diarrhea, vomiting, fevers, and chills", "He reported some relief of symptoms with loperamide", "He denied any history of weight loss, melena, or hematochezia", "Other than a previous open hemorrhoidectomy, the patient had no significant medical history", "Of note, he had no family history of intestinal disease", "On examination, he had a fever with a temperature of 39°C and tachycardia with a heart rate of 102 beats per minute", "On abdominal examination, there was localized tenderness with guarding and fullness in the right lower quadrant", "Results from his blood laboratory investigations demonstrated leukocytosis, with white blood cell count of 17 000/µL (to convert to ×109/L, multiple by 0.001) and a C-reactive protein level of 205 mg/L (to convert to nmol/L, multiple by 9.524)", "However, given the clinical history, the patient underwent a computed tomographic (CT) scan of the abdomen and pelvis with intravenous contrast (Figure 1)", "He then underwent an exploratory laparotomy", "Transverse computed tomographic scan indicated a large right-sided inflammatory phlegmon measuring 7.5 cm in transverse diameter in a 48-year-old man"], "s1": [0, 1, 2, 6, 7, 8, 11], "s2": [3, 4, 5, 9, 10]} {"key": 821, "questions": "What Is Your Diagnosis?", "options": [{"label": "A", "disease": "Mesenteric ischemia"}, {"label": "B", "disease": "Superior mesenteric venous thrombosis"}, {"label": "C", "disease": "Lupus enteritis"}, {"label": "D", "disease": "Superior mesenteric artery embolism"}], "answer_idx": "C", "symptoms": ["A woman in her 50s with a history of systemic lupus erythematosus, congestive heart failure, and type 2 diabetes presented to the emergency department with worsening abdominal pain 1 day after discharge from an admission for similar abdominal pain", "The patient described a 4-day history of progressively worsening abdominal pain and diarrhea and denied fevers, chills, diaphoresis, or any previous similar episodes", "Her abdomen was distended and mildly tender, and her pain was disproportionate to the findings of her physical examination", "There were no abnormal laboratory test findings", "The abdominal computed tomography (CT) scan from her initial admission (Figure, A) and the abdominal CT angiogram on readmission (Figure, B) showed dramatic progression of her disease", "A, Coronal view of computed tomography (CT) scan from 3 days prior to readmission demonstrating isolated duodenal thickening", "B, coronal view of CT angiography scan on readmission demonstrating thickened small bowel and free fluid"], "s1": [0, 1, 2, 3], "s2": [4, 5, 6]} {"key": 822, "questions": "What Is Your Diagnosis?", "options": [{"label": "A", "disease": "Fournier gangrene (necrotizing fasciitis of the perineum)"}, {"label": "B", "disease": "Scrotal ulceration secondary to induction chemotherapy with ATRA"}, {"label": "C", "disease": "Deep fungal infection"}, {"label": "D", "disease": "Mycobacterial infection"}], "answer_idx": "B", "symptoms": ["A previously healthy man in his 30s presented with spontaneous bruising, epistaxis, and haematuria", "Acute promyelocytic leukemia (APML) was diagnosed by bone marrow aspirate morphology, immunophenotyping, and detection of the bcr1 variant of the PML-RARα fusion transcript in peripheral blood", "He was commenced on treatment with the PETHEMA LPA 20051 protocol consisting of all-trans retinoic acid (ATRA) and idarubicin, an anthracycline based chemotherapy", "Dexamethasone was prescribed to reduce the risk of ATRA syndrome, a potentially life threatening complication seen in these patients and characterized by fever, pulmonary infiltrates, hypotension, and leucocytosis", "After 12 days of treatment, the patient developed pyrexia and was started on empiric antibiotics", "Two days later he developed tender black spots on the scrotum that progressed to painful ulcers over 8 days (Figure, A)", "Systemic antifungal and antiviral therapy were added with no improvement", "Peripheral blood cultures and skin swabs from the lesions did not identify bacterial or viral infection", "Scrotal ultrasonography showed a subcutaneous hypervascular lesion with no abscess", "The development and progression of scrotal ulceration coincided with recovery in neutrophil count from 0.3 × 109/L to 1.2 × 109/L and persistent pyrexia", "As scrotal ulceration progressed, they enlarged and became necrotic with a black eschar", "A specimen was obtained for histopathology", "A, Clinical photograph of scrotum and penis shows ulceration and necrosis of penile and scrotal skin with surrounding inflammation", "B, Scrotal biopsy histopathologic specimen (hematoxylin-eosin", "original magnification ×20)", "Scrotal ulceration secondary to induction chemotherapy with ATRA"], "s1": [0, 4, 5, 6, 7, 8, 9, 10, 12], "s2": [1, 2, 3, 11, 13, 14, 15]} {"key": 823, "questions": "What Is Your Diagnosis?", "options": [{"label": "A", "disease": "Pancreatitis panniculitis"}, {"label": "B", "disease": "Malignant atrophic papulosis"}, {"label": "C", "disease": "Systemic lupus erythematosus"}, {"label": "D", "disease": "Lymphomatoid papulosis"}], "answer_idx": "B", "symptoms": ["A woman in her 30s with a 1-year history of unintentional weight loss, mild diffuse abdominal pain, and occasional episodes of vomiting presented to the emergency department with a 24-hour history of intense abdominal pain, fever, and loose stools", "An abdominal computed tomographic scan showed findings consistent with bowel perforation, and she was admitted to the intensive care unit after emergent exploratory laparotomy", "Her medical history was significant for 1 other episode of idiopathic bowel perforation 3 years prior", "She denied taking any medications", "Dermatology was consulted to evaluate an asymptomatic skin eruption that had been present for at least 8 months before her hospitalization", "Physical examination revealed numerous 5- to 10-mm nontender papules with a porcelain-white atrophic center and an erythematous halo, alongside scattered atrophic white scars (Figure, A and B)", "The lesions were distributed across her trunk, neck, and upper and lower extremities", "Palms, soles, scalp, genitalia, and mucosal surfaces were spared", "Her neurological and mental state examination had unremarkable results", "Biopsy specimens from her thigh and back were obtained (Figure, C and D)", "Clinical photographs and biopsy specimens", "A, Papules on the back with white atrophic center and an erythematous rim", "B, Lesions on the thigh similar to those of the back, alongside atrophic white scars", "C, Thigh biopsy shows hyperkeratosis and wedge-shaped avascular necrosis in dermis and subcutaneous tissue (hematoxylin-eosin stain, original magnification ×10)", "D, Back biopsy shows occlusion of dermal blood vessels, perivascular lymphocytic infiltration, and mucin deposition (hematoxylin-eosin stain, original magnification ×40)"], "s1": [0, 1, 2, 3], "s2": [4, 5, 6, 7, 8, 9, 10, 11, 12, 13, 14]} {"key": 824, "questions": "What Is Your Diagnosis?", "options": [{"label": "A", "disease": "Bullous pemphigoid"}, {"label": "B", "disease": "Epidermolysis bullosa acquisita"}, {"label": "C", "disease": "Anti–laminin γ1 pemphigoid"}, {"label": "D", "disease": "Linear IgA dermatosis"}], "answer_idx": "C", "symptoms": ["A woman in her 80s presented with a 1-year history of pruritic erythema on the trunk and extremities", "The erythema did not respond to topical corticosteroid, oral roxithromycin (300 mg/d), or narrow-band UV-B phototherapy", "The patient had a history of type 2 diabetes and hypertension and was treated with glimepiride, sitagliptin phosphate hydrate, and amlodipine besylate", "The patient also had polymyalgia, which was treated with a low dose of prednisolone", "The prednisolone therapy had been discontinued 2 months before her visit to the dermatology clinic when she had been hospitalized with pneumonia", "Multiple tense blisters had subsequently appeared on the palms", "Physical examination at the first visit revealed scaly erythematous plaques on the whole body and tense blisters on the arms, hands, soles, and buttocks", "Neither scar formation or mucosal involvement was observed", "A skin biopsy specimen was obtained from a bulla on the forearm", "Histopathologic analysis revealed a subepidermal bulla with inflammatory infiltrates of neutrophils", "The results of an IgG chemiluminescence enzyme immunoassay for bullous pemphigoid (BP) 180 noncollagenous (NC) 16A were negative", "After treatment with oral prednisolone, 20 mg/d, the lesions healed without scar or milia formation (Figure 1)", "Clinical features before (A) and after (B) treatment with oral prednisolone, showing no scar formation"], "s1": [0, 1, 5, 6, 7, 8, 9, 10], "s2": [2, 3, 4, 11, 12]} {"key": 825, "questions": "What Is Your Diagnosis?", "options": [{"label": "A", "disease": "Chronic urticaria"}, {"label": "B", "disease": "Familial cold autoinflammatory syndrome "}, {"label": "C", "disease": "Muckle-Wells syndrome"}, {"label": "D", "disease": "Neonatal-onset multisystem inflammatory disorder"}], "answer_idx": "C", "symptoms": ["A 5-year-old white girl presented with a history of recurrent fevers and urticaria since 2 months of age", "The child had fevers (maximum temperature, 40°C) that lasted 3 to 5 days", "Her mother reported that the fevers resolved spontaneously but continued to recur almost every month", "The fever symptoms were associated with bilateral conjunctivitis, arthralgias, and irritability", "The hivelike, nonpruritic urticaria did not improve with antihistamine medication and there were no known precipitating or aggravating factors", "The child did not attend day care", "Family history was noncontributory and her parents denied consanguinity", "The child was hospitalized on multiple occasions during infancy for the fevers, and she had comprehensive sepsis evaluations, including lumbar punctures, with negative results", "Findings from chest and abdominal imaging and bone marrow aspiration with biopsy were negative for a malignant process", "Despite extensive evaluation, a definitive source for the fevers was not identified, and the patient was referred to our institution", "On physical examination, the patient was febrile (temperature, 39°C) and appeared fussy", "Growth measure findings were normal", "Skin examination confirmed nonpruritic, macular and urticarial lesions on the face, back, and lower extremities (Figure)", "Bilateral conjunctival hyperemia without drainage was noted", "There was neither swelling nor redness of the joints", "Her right ankle and knee examination showed limited active and passive range of motion secondary to pain symptoms", "Findings from the remainder of physical examination were normal", "Erythematous macular and urticarial lesions with areas of confluence on back and lower extremities"], "s1": [0, 1, 2, 3, 4, 7, 10, 12, 13, 17], "s2": [5, 6, 8, 9, 11, 14, 15, 16]} {"key": 826, "questions": "What Is Your Diagnosis?", "options": [{"label": "A", "disease": "Extensive deep vein thrombosis of the upper extremity"}, {"label": "B", "disease": "Compartment syndrome resulting from infiltration of intravenous infusion therapy into the soft tissues of the upper extremity"}, {"label": "C", "disease": "Secondary lymphedema of the upper extremity"}, {"label": "D", "disease": "Brachial artery pseudoaneurysm"}], "answer_idx": "D", "symptoms": ["An elderly woman was transferred from an extended care facility with lower abdominal pain", "She was afebrile with a heart rate of 120 beats/min on admission and leukocytosis (white blood cell count, 23 000/μL [to convert to ×109/L, multiply by 0.001])", "Computed tomography of the abdomen and pelvis and transvaginal ultrasonography revealed a large intrauterine abscess", "An attempt to insert a peripherally inserted central catheter (PICC) into the right basilic vein was unsuccessful", "An intravenous catheter in the left hand was established for the administration of antibiotics", "The following day, the patient developed right upper extremity pain and swelling", "Physical examination revealed tense and tender swelling of the right upper extremity that was more marked in the upper arm and caused significant impairment in motor function, but sensation to touch and pin prick was intact (Figure 1A)", "The right radial pulse was absent, but biphasic Doppler arterial signals were present in the right radial and ulnar artery", "Duplex ultrasonographic imaging of the right upper extremity demonstrated a 12.8 × 2.6-cm heterogeneous fluid collection with active color flow (Figure 1B)", "Patient presentation (A) and ultrasonographic imaging (B) of the upper arm", "Extensive deep vein thrombosis of the upper extremityCompartment syndrome resulting from infiltration of intravenous infusion therapy into the soft tissues of the upper extremity"], "s1": [0, 1, 2, 3, 4], "s2": [5, 6, 7, 8, 9, 10]} {"key": 827, "questions": "What Is Your Diagnosis?", "options": [{"label": "A", "disease": "Hemorrhagic pancreatitis"}, {"label": "B", "disease": "Acute perforation of a bleeding peptic ulcer"}, {"label": "C", "disease": "Rupture of a jejunal artery aneurysm"}, {"label": "D", "disease": "Rupture of a peripancreatic pseudoaneurysm"}], "answer_idx": "C", "symptoms": ["A man in his 50s with a history of smoking and chronic hypertension was admitted to our hospital with a 36-hour history of upper abdominal pain, nausea, and vomiting", "He had no history of previous surgery or trauma", "His systolic blood pressure was 109 mm Hg, his diastolic blood pressure was 70 mm Hg, and his pulse rate was 100 beats/min", "A physical examination revealed moderate epigastric tenderness with rebound and guarding", "Laboratory test results were remarkable for a white blood cell count of 13 970/μL, hemoglobin level of 8.4 g/dL, serum creatinine level of 1.67 mg/dL, serum amylase level of 57 U/L (reference range, 30-110 U/L), and serum triglyceride level of 282.42 mg/dL, with no evidence of coagulopathy", "(To convert the white blood cell count to ×109 per liter, multiply by 0.001", "hemoglobin level to grams per liter, by 10", "serum creatinine level to micromoles per liter, by 88.4", "serum amylase level to microkatals per liter, by 0.0167", "and serum triglyceride level to millimoles per liter, by 0.0113.) Contrast-enhanced computed tomography (CECT) and mesenteric angiography were performed (Figure 1)", "A, Contrast-enhanced computed tomographic (CECT) image showing an intra-abdominal hyperdense mass (arrowhead) and free fluid in the upper abdomen", "B, Visceral artery angiogram revealing an aneurysm (arrowhead)"], "s1": [0, 1, 2, 3, 4], "s2": [5, 6, 7, 8, 9, 10, 11]} {"key": 828, "questions": "What Is Your Diagnosis?", "options": [{"label": "A", "disease": "Carcinoma erysipeloides (cutaneous metastases of breast cancer)"}, {"label": "B", "disease": "Erysipelas"}, {"label": "C", "disease": "Radiation dermatitis"}, {"label": "D", "disease": "Radiation-induced morphea"}], "answer_idx": "A", "symptoms": ["A woman in her 60s presented with a 3-month history of a progressive, asymptomatic cutaneous eruption on her chest and back", "She had a complex history of breast cancer on the left side that was first diagnosed in 1991 and was treated with lumpectomy, radiotherapy, cyclophosphamide, methotrexate, and fluorouracil", "In 2001, she experienced local disease recurrence and underwent complete mastectomy with axillary lymph node dissection on the left side and received adjuvant treatment with tamoxifen citrate and anastrozole", "In 2014, she developed axillary lymphadenopathy on the right side and was found to have metastatic disease", "She subsequently underwent axillary lymph node dissection on the right side, followed by treatment with doxorubicin hydrochloride and cyclophosphamide", "Her most recent mammogram and positron-emission tomography–computed tomography (PET/CT) showed no evidence of active disease", "At the time of her evaluation, the patient’s physical examination was remarkable for discrete erysipeloid plaques on her chest, back, and right shoulder (Figure 1A)", "The lesions were warm and indurated but nontender to palpation", "Serum chemistry profile, complete blood cell counts, and hepatic and renal function testing had normal results", "Lesional skin biopsies were performed (Figure 1B)", "A, Clinical examination shows discrete erysipeloid plaques on the anterior chest", "B, Histopathologic analysis (hematoxylin-eosin stain, original magnification ×100)"], "s1": [0, 6, 7, 9, 10, 11], "s2": [1, 2, 3, 4, 5, 8]} {"key": 829, "questions": "What Is Your Diagnosis?", "options": [{"label": "A", "disease": "Pemphigus foliaceus"}, {"label": "B", "disease": "Generalized Hailey-Hailey disease"}, {"label": "C", "disease": "IgA pemphigus"}, {"label": "D", "disease": "Pemphigus vulgaris"}], "answer_idx": "B", "symptoms": ["A healthy woman in her 60s presented with multiple erythematous crusted plaques on her trunk that had appeared 4 weeks previously, sparing her limbs and facial region (Figure, A)", "She had a long-standing history of several outbreaks of skin lesions limited to both axillae and groin", "Previous episodes had been successfully treated with various topical drugs that were administered by her primary care physician", "No family history of similar lesions was recorded", "Physical examination revealed multiple annular erythematous plaques surrounded by a peripheral crusted and pustular collarette on her trunk (Figure, B)", "No active lesions were present in her axillae, inframammary folds, or groin", "After obtaining informed consent, 2 biopsy specimens were taken from the periphery of the lesion for hematoxylin-eosin stain and direct immunofluorescence (Figure, C and D)", "A skin swab specimen from a pustular lesion was taken for microbiological culture", "A, Multiple erythematous and crusted plaques diffusely distributed on the dorsal trunk at presentation", "B, An erythematous annular plaque, surrounded by a crusted and focally pustular peripheral collarette", "C and D, Histopathologic images, hematoxylin-eosin", "C, Biopsy specimen revealing suprabasilar acantholysis (original magnification ×4)", "D, Specimen from generalized lesions at clinical presentation (original magnification ×40)"], "s1": [0, 1, 2, 3, 4, 5, 8, 9], "s2": [6, 7, 10, 11, 12]} {"key": 830, "questions": "What Is Your Diagnosis?", "options": [{"label": "A", "disease": "Oral melanotic macule"}, {"label": "B", "disease": "Oral malignant melanoma"}, {"label": "C", "disease": "Oral melanocytic nevus"}, {"label": "D", "disease": "Oral melanoacanthoma"}], "answer_idx": "D", "symptoms": ["A white woman in her 50s presented to an oral medicine clinic for evaluation of asymptomatic pigmented lesions affecting the maxillary and mandibular gingiva of 6 years’ duration", "The patient reported previous orolabial melanotic macules", "Her medical history was significant for hypertension, hypercholesterolemia, asthma, and breast cancer", "Medications included levalbuterol, nadolol, and cetirizine", "Family medical history was significant for breast cancer and social history was unremarkable", "Review of systems was positive for cutaneous ephelides", "Physical examination revealed a 7 mm × 6 mm asymmetric, heterogeneously pigmented lesion without indurated borders on the gingiva associated with the left maxillary central incisor tooth (number 9) (Figure, A)", "A lesion similar in clinical appearance, although smaller, was observed on the gingiva associated with the right mandibular first premolar tooth (number 28) (Figure, B)", "Biopsy specimens were obtained with a 3 mm punch instrument from both lesions for routine histology which demonstrated similar microscopic findings (Figure, C and D)", "A, Heterogeneously pigmented lesion of the gingiva associated with the left maxillary central incisor", "B, Heterogeneously pigmented lesion of the gingiva associated with the right mandibular first premolar", "C, Mucosal epithelium exhibiting spongiosis and a diffuse proliferation of heavily pigmented dendritic melanocytes that extend the full thickness of the epithelium (arrowheads)", "Melanin incontinence is also noted (hematoxylin-eosin stain, original magnification ×100)", "D, Squamous mucosa exhibiting spongiosis with diffuse pigmentation in the basal cell layer and dendritic melanocytes (arrowheads) in the stratum spinosum (hematoxylin-eosin stain, original magnification ×200)"], "s1": [0, 1, 5, 6, 7, 8, 9, 10], "s2": [2, 3, 4, 11, 12, 13]} {"key": 831, "questions": "What Is Your Diagnosis?", "options": [{"label": "A", "disease": "Herpes simplex virus"}, {"label": "B", "disease": "Disseminated varicella zoster virus"}, {"label": "C", "disease": "Listeriosis"}, {"label": "D", "disease": "Cutaneous Bacillus cereus infection"}], "answer_idx": "D", "symptoms": ["A female infant with relapsed refractory infantile acute lymphoblastic leukemia, blast crisis (>60% blasts), and central nervous system involvement was admitted for reinduction chemotherapy with cytarabine, vincristine, daunorubicin, methotrexate, and dexamethasone", "Seventeen days after her last day of chemotherapy, while neutropenic, she became febrile and developed several vesicles with an erythematous base on the lower back (Figure, A) and left thigh", "Tissue and blood cultures and a punch biopsy specimen of a vesicle on the lower back were obtained", "Hematoxylin-eosin (Figures, B and C), gram (Figure, D), and periodic acid-Schiff stains were performed on the punch biopsy", "Testing for herpes simplex virus and varicella zoster virus were also ordered", "A, Scattered vesicles with an erythematous base on the lower back", "B, Hematoxylin and eosin stain (original magnification ×200)", "C, Hematoxylin and eosin stain (original magnification ×600)", "D, Gram stain (original magnification ×600)"], "s1": [0, 1, 4], "s2": [2, 3, 5, 6, 7, 8]} {"key": 832, "questions": "What Is Your Diagnosis?", "options": [{"label": "A", "disease": "Metastatic melanoma"}, {"label": "B", "disease": "Primary intracranial melanoma"}, {"label": "C", "disease": "Neurocutaneous melanosis"}, {"label": "D", "disease": "Lymphoma"}], "answer_idx": "C", "symptoms": ["A teenage Chinese girl with intermittent limp and increasing headache was admitted to the hospital", "She began to limp intermittently ten years earlier", "A brain magnetic resonance imaging (MRI) showed multiple parenchymal lesions involving the pons, bilateral amygdala, and the left temporal pole (Figure 1A)", "She refused further investigations until a year earlier, when she developed mild headache", "Magnetic resonance imaging showed minor focal enlargement of the lesions in the left temporal pole", "One month before hospital admission, her headache aggravated and became intolerable", "Enhanced MRI showed remarkable progression of the lesions in the left temporal lobe, involving the temporal pole and the parasylvian region, along with a newly developed mass in the third ventricle and obstructive hydrocephalus", "The lesions in the pons and the right amygdala remained stable (Figure 1B)", "Spinal cord MRI was normal", "There was no history of tumors elsewhere", "Family history was unremarkable", "Her consciousness was intact, but her memory and concentration were poor", "Pupil reflection was normal", "There were multiple congenital nevi distributed over the body, 2 of which on the legs were more than 5 cm in diameter, and another 3 on her back and face were more than 3 cm", "There were no focal motor or sensory deficits", "Muscle stretch reflexes were normal", "The rest of the neurologic and systemic examination was unremarkable", "Gene examinations showed a mutation in the N-ras-2 gene", "A and B, T1-weighted axial brain magnetic resonance imaging at the levels of pons, midbrain, and thalamus, performed ten years earlier, shows multiple parenchymal lesions including the pons, bilateral amygdala, and the left temporal pole (arrowheads)", "Sequence, spin echo", "repetition time, 650.0 milliseconds", "echo time, 14.0 milliseconds", "field of view, 144 × 230", "slice, 5.0 mm", "window, 1054", "contrast, 640. C and D, Enhanced, T1-weighted axial brain magnetic resonance imaging at the levels of pons, midbrain, and thalamus, performed at the most recent hospital admission, shows remarkable progression of the lesions in the left temporal lobe, involving the temporal pole and the parasylvian region, along with a newly developed mass in the third ventricle (arrowhead) and obstructive hydrocephalus", "The lesions in the pons and the right amygdala remained stable", "Sequence, fast spin echo", "repetition time, 600.0 milliseconds", "echo time, 9.0 milliseconds", "field of view, 230 × 230", "slice, 5.0 mm", "window, 1087", "contrast, 524."], "s1": [0, 1, 3, 5, 6, 8, 11, 13, 14, 15, 16, 17], "s2": [2, 4, 7, 9, 10, 12, 18, 19, 20, 21, 22, 23, 24, 25, 26, 27, 28, 29, 30, 31, 32, 33]} {"key": 833, "questions": "What Is Your Diagnosis?", "options": [{"label": "A", "disease": "Pyomyositis"}, {"label": "B", "disease": "Nicolau syndrome (livedoid dermatitis)"}, {"label": "C", "disease": "Retained foreign body"}, {"label": "D", "disease": "Necrotizing fasciitis"}], "answer_idx": "B", "symptoms": ["A previously healthy 15-year-old boy presented to our hospital with pain and purpura of his left thigh after an intramuscular injection in that leg", "Ten days prior, he had presented to a referring hospital’s emergency department for nausea and vomiting and was given 20 mg of intramuscular dicyclomine", "He reported severe pain at the time and site of injection", "Within a few hours, he noticed swelling and purpura overlying the site of injection", "Over the next 2 weeks, the swelling persisted and the purpura intensified, becoming black centrally (Figure A and B)", "Pain worsened over this period, initially restricting use of the leg, then preventing any weight bearing, prompting his admission to our hospital 10 days after the injection", "He also reported numbness and occasional tingling of the skin involved", "The patient denied fever or chills, and his nausea and vomiting had resolved", "A, Teenage boy with development of retiform purpura of the left thigh following intramuscular dicyclomine injection", "At 2 days after injection, red to purple retiform changes developed on the left thigh", "B, Worsening retiform purpura with central necrosis was noted 13 days after the injection", "Findings of a physical examination were significant for retiform purpura of the left thigh, approximately 8 × 12 cm in size", "Palpation of the area elicited severe pain", "Popliteal, posterior tibial, dorsalis pedis, and femoral pulses were strong", "There was no swelling of the remainder of the leg, and findings of joint examinations were normal", "Sensation was intact throughout the leg with the exception of the skin involved, which had diminished light touch", "Blood test results revealed a normal complete blood cell count, normal coagulation profile, and an elevated creatine phosphokinase level of 477 U/L (normal range, 22-198 U/L", "to convert to microkatals per liter, multiply by 0.0167)", "Magnetic resonance imaging of the thigh was completed, and a skin biopsy was performed for culture and histology"], "s1": [0, 1, 2, 4, 5, 6, 8, 9, 10, 11], "s2": [3, 7, 12, 13, 14, 15, 16, 17, 18]} {"key": 834, "questions": "What Is Your Diagnosis?", "options": [{"label": "A", "disease": "Perforated appendicitis"}, {"label": "B", "disease": "Meckel diverticulitis"}, {"label": "C", "disease": "Crohn disease"}, {"label": "D", "disease": "Ectopic ovarian torsion"}], "answer_idx": "D", "symptoms": ["A previously healthy 4-year-old girl with a history of prematurity (31 weeks’ gestational age) presented as a transfer from an outside facility with abdominal pain", "She had presented to an outside emergency department 2 days prior with lower abdominal pain that subsequently localized to the right lower quadrant", "Her pain was accompanied by anorexia and nonbloody, nonbilious emesis", "She denied fevers or chills", "She was noted to have voluntary guarding and focal right lower quadrant tenderness to palpation", "Laboratory analyses demonstrated a mild leukocytosis (white blood cell count = 14 × 103/μL [to convert to ×109/L, multiply by 0.001]) and normal urinalysis results", "Abdominal ultrasonography images were obtained that failed to show her appendix and demonstrated no secondary signs of inflammation", "Because of her notable tenderness, she was admitted for observation at the referring facility", "No antibiotics were initiated, and her activity improved with intravenous hydration", "However, her right lower abdominal pain persisted, prompting further evaluation", "Computed tomography images of the abdomen and pelvis were obtained, and the representative images are shown in Figure 1. She was transferred to our facility for surgical evaluation", "In our emergency department, pelvic ultrasonography images were obtained that failed to show both the appendix and right adnexal structures", "Computed tomographic images", "Axial (A) and (B) coronal sections demonstrate an inflamed retrocecal mass"], "s1": [0, 1, 2, 3, 4, 5, 8], "s2": [6, 7, 9, 10, 11, 12, 13]} {"key": 835, "questions": "What Is Your Diagnosis?", "options": [{"label": "A", "disease": "Acute jugular thrombosis"}, {"label": "B", "disease": "Spontaneous chylothorax"}, {"label": "C", "disease": "Contained esophageal perforation"}, {"label": "D", "disease": "Descending necrotizing mediastinitis"}], "answer_idx": "B", "symptoms": ["A 62-year-old woman presented to the emergency department with severe substernal pain radiating to the left shoulder and back and subsequent swelling of the left side of her neck", "She had been recently treated at an outpatient facility for a viral illness with multiple episodes of forceful coughing", "There was no recent history of trauma to the neck or chest", "Physical examination revealed normal vital signs and soft tissue swelling over the left supraclavicular area", "No overlying skin changes or crepitus were appreciated", "Laboratory data revealed leukocytosis (white blood cell count, 18 000/µL", "to convert to ×109 per liter, multiply by 0.001)", "A computed tomographic scan demonstrated extensive edema of the left neck, with fat stranding seen in the subcutaneous tissues extending into the left jugular, carotid, and esophageal spaces (Figure, A)", "Mediastinal edema and bilateral pleural effusions were also seen on computed tomographic scan (Figure, B)", "Owing to the concern for esophageal perforation, a barium swallow was performed, showing no extravasation of contrast", "A, Computed tomographic scan shows extensive soft tissue inflammation in the left neck (arrowhead)", "B, Computed tomographic scan shows bilateral pleural effusions (red arrowheads) and mediastinal edema (white arrowhead)"], "s1": [0, 1, 2, 3, 4, 5, 6], "s2": [7, 8, 9, 10, 11]} {"key": 836, "questions": "What Is Your Diagnosis?", "options": [{"label": "A", "disease": "Hemangioma"}, {"label": "B", "disease": "Neutrophilic dermatoses"}, {"label": "C", "disease": "Plasmacytoma"}, {"label": "D", "disease": "Leukemia cutis"}], "answer_idx": "C", "symptoms": ["A woman in her 60s presented for evaluation of multiple painful cutaneous nodules that had developed over the previous 2 months", "The patient had a history of IgGκ multiple myeloma, diagnosed approximately 6 months earlier", "At that time, flow cytometry showed a κ-restricted plasma cell population positive for CD38 and CD138 but negative for CD3, CD20, and CD45 expression", "Cytogenetic analysis with fluorescent in situ hybridization did not reveal high-risk genetic markers", "The patient demonstrated no evidence of end organ disease, and her myeloma was classified as stage II", "Since her initial diagnosis, she had completed 4 cycles of lenalidomide, bortezomib, and dexamethasone (RVD) therapy in addition to irradiation for lytic lesions of the thoracic and lumbar spine", "Physical examination revealed scattered red-violaceous nodules, with the largest measuring approximately 6.0 × 5.5 × 2.5 cm (Figure, A)", "They were found in the mouth, the trunk, and on all 4 extremities", "The lesions were nonulcerated, round with a smooth surface, fixed, and firm and tender to touch", "Serum protein levels were normal", "An ultrasound-guided needle biopsy and immunohistochemical analysis of the largest lesion on the patient’s right shoulder were performed (Figure, B and C)"], "s1": [0, 6, 7, 8, 10], "s2": [1, 2, 3, 4, 5, 9]} {"key": 837, "questions": "What Is Your Diagnosis?", "options": [{"label": "A", "disease": "Leprosy"}, {"label": "B", "disease": "Mastocytosis"}, {"label": "C", "disease": "Carcinoid syndrome"}, {"label": "D", "disease": "Leishmaniasis"}], "answer_idx": "C", "symptoms": ["A man in his 50s with a history of both hypertension and a pulmonary tumor that was resected 10 years earlier (Figure, A) presented with generalized phymatous changes affecting his entire face, as well as a few pustular lesions and a diffuse erythema that gave him an appearance of leonine facies (Figure, B)", "He complained of substantial, progressive seborrheic hyperplasia and flushing episodes over the past year, accompanied by ocular manifestations that included swelling of his eyelids", "Ophthalmological examination showed features of chronic blepharitis and lagophthalmos", "Detailed anamnesis revealed a history of intense diarrhea in prior months", "A biopsy specimen from the skin of the forehead was obtained and analyzed under hematoxylin-eosin staining (Figure, C)", "A computed tomography (CT) scan completed the etiological investigation (Figure, D)", "A, Immunohistochemical analysis of the earlier lung tumor using chromogranin A", "panoramic view shows that most of the cells diffusely express chromogranin (original magnification ×20)", "B, Leonine facies presentation shows phymatous changes, diffuse erythema, pustular lesions, and seborrheic hyperplasia", "C, Forehead skin shows scarce lymphocytic infiltrate close to follicular structure along with sebaceous hyperplasia (hematoxylin-eosin, original magnification ×40)", "D, Liver computed tomographic (CT) image showing multiple metastatic hepatic masses"], "s1": [0, 1, 2, 3, 8, 9], "s2": [4, 5, 6, 7, 10]} {"key": 838, "questions": "What Is The Diagnosis?", "options": [{"label": "A", "disease": "Tinea corporis"}, {"label": "B", "disease": "IgA pemphigus"}, {"label": "C", "disease": "Duhring disease"}, {"label": "D", "disease": "Psoriasis pustulosa"}], "answer_idx": "B", "symptoms": ["A woman in her 50s presented for evaluation of itching skin lesions", "The lesions had been present for months and increased gradually", "Mucosae were unaffected", "Treatment with topical steroids showed no lasting effect", "Skin examination revealed multiple flaccid pustules on scaly, erythematous plaques on her trunk, with an annular and circinate pattern (Figure, A and B)", "A and B, Clinical presentation showed flaccid pustules on scaly, erythematous plaques on the trunk", "C, Subcorneal acantholysis of keratinocytes (yellow arrowheads) resulting in intraepidermal separation/blistering and an inflammatory infiltrate consisting of neutrophilic granulocytes (black arrowheads) (hematoxylin-eosin staining, original magnification × 100", "inset original magnification × 400)", "D, Discrete subcorneal intercellular IgA depositions (white arrowheads) (direct immunofluorescence with FITC-labeled antihuman IgA antibodies, original magnification × 400)"], "s1": [0, 1, 2, 3], "s2": [4, 5, 6, 7, 8]} {"key": 839, "questions": "What Is Your Diagnosis?", "options": [{"label": "A", "disease": "Mucormycosis"}, {"label": "B", "disease": "Cutaneous diphtheria"}, {"label": "C", "disease": "Pseudomonal ecthyma gangrenosum"}, {"label": "D", "disease": "Aspergillosis"}], "answer_idx": "A", "symptoms": ["A 4-year-old girl with newly diagnosed acute lymphocytic leukemia, receiving her initial course of chemotherapy with vincristine, daunorubicin, and methotrexate, developed a painful lesion on her left triceps", "The tender area was beneath an adhesive bandage placed where she had received DTaP (diphtheria-tetanus-pertussis) and polio vaccinations 3 weeks earlier", "Otherwise, she appeared remarkably well, and her vital signs were normal", "Removing the bandage revealed a tender indurated gray-violet plaque (Figure, A)", "White blood cell and neutrophil counts were low at 0.4 K/μL (normal range, 4.86-13.18 K/μL) and 0.18 K/μL (normal range, 1.60-8.29 K/μL), respectively", "Blood culture findings were negative", "Punch biopsy specimens were obtained for histopathologic analysis (Figure, B) and fungal culture (Figure, C)", "A, Gray-violet plaque on posterior upper arm at site of adhesive portion of bandage", "B, Broad nonseptate hyphae surrounding and infiltrating dermal blood vessels (hematoxylin-eosin", "original magnification ×400)", "Of note, occasional septae are seen in this specimen, consistent with the characteristic finding that mucormycetes are pauciseptate, rather than aseptate, molds", "C, Nonseptate hyphae with round terminal sporangia and free spores (lactophenol cotton blue mount", "original magnification ×400)"], "s1": [0, 1, 2, 3, 4, 5], "s2": [6, 7, 8, 9, 10, 11, 12]} {"key": 840, "questions": "What Is Your Diagnosis?", "options": [{"label": "A", "disease": "Candidiasis"}, {"label": "B", "disease": "Transient neonatal pustular melanosis"}, {"label": "C", "disease": "Congenital varicella"}, {"label": "D", "disease": "Herpes simplex virus"}], "answer_idx": "D", "symptoms": ["A 3-day-old premature male infant, born at 26 weeks’ gestational age, presented to the dermatology department with a skin rash on the bilateral lower extremities", "History was notable for preterm premature rupture of membranes for 10 days", "The neonate was born via urgent cesarean delivery owing to breech positioning", "The mother was otherwise in good health with normal prenatal care and had no history of sexually transmitted diseases", "During the first few days of life, the neonate had recurrent hypoxic episodes", "He was receiving intravenous antibiotics for suspected sepsis and bacitracin for the topical lesions", "Per the primary team, the rash was first noted at 3 days of life", "Physical examination demonstrated a small, ill-appearing neonate with multiple, grouped vesicles and punched-out ulcerations on his scalp, axilla, extremities, and scrotum (Figure, A-D)", "He also had witnessed respiratory desaturations requiring urgent intubation", "Initial laboratory evaluation demonstrated a normal complete blood cell count and serum chemistry with the exception of a high lymphocyte count (34%, normal level <25% [to convert to proportion of 1.0, multiply by 0.01]) and high aspartate aminotransferase level (117 IU/L, normal level 8-39 IU/L [to convert to microkatals per liter, multiply by 0.0167])", "A chest radiograph demonstrated increased pulmonary markings bilaterally", "Neonate with grouped vesicles on an erythematous base on the scalp and bilateral lower extremities"], "s1": [1, 2, 3, 4, 8, 9, 10], "s2": [0, 5, 6, 7, 11]} {"key": 841, "questions": "What Is Your Diagnosis?", "options": [{"label": "A", "disease": "Inflammatory breast cancer"}, {"label": "B", "disease": "Tuberculous mastitis"}, {"label": "C", "disease": "Idiopathic granulomatous mastitis"}, {"label": "D", "disease": "Mastitis caused by immunosuppression"}], "answer_idx": "C", "symptoms": ["A 28-year-old woman visited the emergency department for a sudden, diffuse, and painful enlargement of the upper quadrants of the left breast with edema and palpable axillary lymph nodes without clinical signs of inflammation or infection (Figure 1A)", "She had neither fever nor skin redness but was experiencing fatigue", "Initial presentation of the patient", "A, Asymmetric mass in the upper quadrant of the left breast", "B, Breast ultrasonogram revealing hypoechogenicity of the breast tissue with periductal small cysts", "The patient had stopped breastfeeding 10 months before the emergency department admission, and she had a recent history of pneumonia self-medicated with antibiotics", "She had no family history of breast or ovarian cancer and no history of drug addiction or hyaluronic acid or free silicone injections in the breast", "She denied any trauma to the breast", "On physical examination, the patient had a 6-cm hard mass without nipple discharge or retraction and left axillary palpable lymph nodes", "Breast ultrasonography revealed diffuse nonspecific hypoechogenicity of the breast tissue with periductal small cysts (Figure 1B)", "Mammography revealed asymmetric density and distortion in the left upper quadrants", "Multiple 14-gauge ultrasound-guided biopsies in the upper quadrants of the left breast and fine-needle aspiration cytologic testing of an enlarged lymph node revealed granulomatous inflammation with epithelioid histiocytes, lymphocytes, plasma cells, and eosinophils", "Lymph nodes were normal", "Blood test results were as follows: hepatitis B surface antigen, hepatitis C antibody, human immunodeficiency virus enzyme-linked immunosorbent assay, and whole-blood interferon γ release assay (QuantiFERON), negative", "IgG cytomegalovirus, positive", "fibrinogen level, 234 mg/dL (to convert to micromoles per liter, multiply by 0.0294)", "erythrocyte sedimentation rate, 4 mm/h", "leukocyte count, 7660/µL (to convert to ×109/L, multiply by 0.001)", "no acquired immunosuppression", "and no hematologic disorders"], "s1": [0, 1, 3, 5, 6, 7, 8, 10], "s2": [2, 4, 9, 11, 12, 13, 14, 15, 16, 17, 18, 19]} {"key": 842, "questions": "What Is Your Diagnosis?", "options": [{"label": "A", "disease": "Angiosarcoma"}, {"label": "B", "disease": "Leiomyosarcoma"}, {"label": "C", "disease": "Pheochromocytoma"}, {"label": "D", "disease": "Unicentric Castleman disease"}], "answer_idx": "D", "symptoms": ["A 69-year-old white man presented to the emergency department after a tractor-on-tree collision", "Contrast-enhanced computed tomography demonstrated an incidental right infrarenal, retroperitoneal mass measuring 9.7 × 7.7 × 9.1 cm (Figure 1A), with dilated tortuous supply from a lumbar artery branch and retroperitoneal lymphadenopathy", "He denied any history of systemic symptoms, pain, urinary symptoms, or abnormal bowel function but described 6 years of decreased appetite", "Laboratory evaluation of plasma-free metanephrine levels showed a slight elevation in normetanephrine levels (1.21 nmol/L)", "Given the dramatic arterial enhancement and hypertrophied perivascular plexus, biopsy was not performed, and preoperative arterial embolization was deemed appropriate to minimize intraoperative blood loss", "A, Contrast-enhanced computed tomography scan demonstrating an incidental right infrarenal, retroperitoneal mass measuring 9.7 × 7.7 × 9.1 cm, with dilated tortuous supply from a lumbar artery branch", "B, Flush aortogram outlining supply to the mass from a hypertrophied lumbar artery (blue arrowhead) and from a branch of the internal iliac artery (red arrowhead)", "At preoperative embolization, initial aortography identified the predominant lumbar artery branch and also demonstrated supply from a branch of the internal iliac artery (Figure 1B)", "These branches were embolized and the mass was devascularized", "Twenty-four hours following embolization, surgical exploration was performed through a periumbilical midline incision", "With no evidence of distant metastasis (peritoneal or liver), the mass was resected with partial resection of the iliopsoas muscle to achieve negative margins in conjunction with aortocaval lymphadenectomy of the enlarged lymph nodes", "The mass was running parallel to the right ureter, which was meticulously dissected, and the right kidney was successfully saved", "There were no complications, and the postoperative course was uneventful"], "s1": [0, 2, 3, 12], "s2": [1, 4, 5, 6, 7, 8, 9, 10, 11]} {"key": 843, "questions": "What Is Your Diagnosis?", "options": [{"label": "A", "disease": "Rhabdomyoma"}, {"label": "B", "disease": "Granular cell tumor"}, {"label": "C", "disease": "Paraganglioma"}, {"label": "D", "disease": "Hibernoma"}], "answer_idx": "A", "symptoms": ["A man in his 60s presented with progressive onset of nocturnal dyspnea, dysphonia, dysphagia, and noisy breathing over the previous year", "In addition, he reported a 22.7-kg weight loss", "He also had a medical history significant for obstructive sleep apnea, gastroesophageal reflux disease with Barrett esophagitis, diabetes, and degenerative joint disease", "Findings from his head and neck physical examination were normal except for the laryngeal examination", "Laryngeal endoscopy revealed a large, well-mucosalized mass emanating from the left pyriform sinus, with mass effect on the larynx (Figure, A)", "The bilateral true vocal folds (TVFs) were visualized and were normal, but the full range of mobility of the left TVF could not be assessed owing to mass effect", "A computed tomographic scan with contrast of the neck was obtained and revealed a 5 × 6-cm, nonenhancing, well-circumscribed laryngeal mass (Figure, B)", "The tumor was originating from the left pyriform sinus and did not appear to be locally invasive", "Given these findings, the patient underwent direct laryngoscopy for transoral carbon dioxide laser–assisted excision without complications", "Microscopic examination of the mass demonstrated polygonal cells with peripherally placed nuclei and eosinophilic cytoplasm containing cross-striations (Figure, C)", "Postoperatively, the patient reported improvement in dysphagia, voice quality, sleep, and breathing patterns", "A, A large left pyriform sinus mass with mass effect on the larynx", "B, Preoperative computerized tomography scan showing a large laryngeal mass without local invasion", "C, Polygonal cells with peripherally placed nuclei and eosinophilic cytoplasm containing cross-striations (hematoxylin-eosin, original magnification ×40)"], "s1": [0, 1, 2, 10], "s2": [3, 4, 5, 6, 7, 8, 9, 11, 12, 13]} {"key": 844, "questions": "What Is Your Diagnosis?", "options": [{"label": "A", "disease": "Fusarium species infection"}, {"label": "B", "disease": "Bartonella henselae infection"}, {"label": "C", "disease": "Leukemia cutis"}, {"label": "D", "disease": "Sweet syndrome"}], "answer_idx": "D", "symptoms": ["A man in his 40s with newly diagnosed acute myelogenous leukemia (AML) arising from myelodysplasia presented with fever, cough, abdominal pain, difficulty swallowing, and 1 month of neutropenia", "He had abruptly developed a painful eruption on his scalp, face, and left leg 24 hours prior to presentation after being scratched in these areas by his pet cat", "The patient had not received any chemotherapy or antimicrobial prophylaxis", "Physical examination revealed an ill-appearing, diaphoretic, thin man with multiple erythematous to violaceous, tender, edematous, targetoid nodules and plaques with central dusky appearance on the left forearm and right upper thigh (Figure, A)", "Vital signs included temperature of 39.7° C, blood pressure of 141/69 mm Hg, and pulse of 122 bpm", "His white blood cell count was 1.8 × 103/µL (reference range, 4.5-11 × 103/µL", "to convert to ×109/L, multiply by .001), and absolute neutrophil count was 246 cells/µL (reference range, 1520-6370 cells/µL)", "Other laboratory values were remarkable for a red blood cell count of 2.13 × 106/µL (reference range, 3.9-5.5 × 106/µL", "to convert to ×1012/L, multiply by 1), hemoglobin of 6.1 g/dL (reference range, 14-17.5 g/dL", "to convert to g/L, multiply by 10), and platelet count of 14 × 103/µL (reference range 150-350 × 103/µL, to convert to ×109/L, multiply 1)", "Cellular cytogenetics of his AML were notable for a 5q deletion and p53 mutation", "Punch biopsy specimens from the right arm and left thigh were performed for histopathologic evaluation and tissue culture", "A brisk neutrophilic infiltrate was seen on histopathology (Figure, B)", "A, 1.5 cm-long, edematous plaque with dusky central erythema on upper thigh", "B, Punch biopsy specimen with hematoxylin-eosin staining (original magnification ×40)", "arrowhead indicates brisk dermal neutrophilic infiltrate"], "s1": [0, 1, 2, 4, 10], "s2": [3, 5, 6, 7, 8, 9, 11, 12, 13, 14, 15]} {"key": 845, "questions": "What Is Your Diagnosis?", "options": [{"label": "A", "disease": "Dermatopathia pigmentosa reticularis"}, {"label": "B", "disease": "Dowling-Degos disease"}, {"label": "C", "disease": "Dyskeratosis congenita"}, {"label": "D", "disease": "X-linked reticulate pigmentary disorder"}], "answer_idx": "D", "symptoms": ["A man in his 30s presented with reticulate hyperpigmentation, progressive since early childhood, overlying the trunk and extremities (Figure, A and B)", "Physical examination revealed normal fingernails and toenails, dermatoglyphics, and dentition", "Additionally, there was no evidence of alopecia or keratoderma", "Review of systems was remarkable for hypohidrosis, photophobia, and a history of recurrent sinopulmonary infections, including pneumonia, requiring hospitalization", "Family history was notable for similarly affected men on the patient’s mother’s side", "A punch biopsy specimen was obtained (Figure, C and D)", "A, Reticulate hyperpigmentation involving the lower and upper distal extremities, including the dorsal surface of the hand", "B, Diffuse reticulate hyperpigmentation of the trunk and flexures, including the posterior neck and axilla", "C, Hematoxlyn-eosin stain (original magnification ×200)", "D, Congo red stain (original magnification ×200)"], "s1": [0, 3, 4, 6, 7], "s2": [1, 2, 5, 8, 9]} {"key": 846, "questions": "What Is Your Diagnosis?", "options": [{"label": "A", "disease": "Lichen planus pigmentosus"}, {"label": "B", "disease": "Leukocytoclastic vasculitis"}, {"label": "C", "disease": "Macular lymphocytic arteritis"}, {"label": "D", "disease": "Erythema dyschromicum perstans"}], "answer_idx": "C", "symptoms": ["A woman in her early 20s presented with 1 year of discrete asymptomatic brown lesions distributed over her lower extremities", "She denied any history of trauma or prior lesions at these sites and could not pinpoint any exacerbating or alleviating factors", "She denied fever, arthralgias, myalgias, or constitutional symptoms", "She had no significant past medical history and took no medications or supplements", "A physical examination revealed several 2-mm to 4-mm round, hyperpigmented macules symmetrically distributed over the lower extremities and dorsal feet (Figure, A)", "No prior treatments had been attempted", "A skin biopsy was performed (Figure, B-D)", "What is your diagnosis?"], "s1": [0, 1, 2, 3, 5], "s2": [4, 6, 7]} {"key": 847, "questions": "What Is Your Diagnosis?", "options": [{"label": "A", "disease": "Xanthogranuloma"}, {"label": "B", "disease": "Erdheim-Chester disease"}, {"label": "C", "disease": "Xanthoma disseminatum"}, {"label": "D", "disease": "Generalized eruptive histiocytoma"}], "answer_idx": "C", "symptoms": ["A woman in her 20s presented with numerous grouped yellow-brown and red-brown papules on the upper eyelids, infraorbital rims, bilateral axillae, inframammary creases, lower back, buttocks, hips, inguinal folds, and proximal thighs (Figure, A and B)", "The lesions first appeared in her right axilla 6 months prior and were asymptomatic", "The patient also noted a 3-month history of polyuria, polydipsia, and nocturia", "Review of symptoms was otherwise negative", "She had no notable medical or family history", "On physical examination there were no other sites of involvement including mucosal surfaces", "Shave biopsies were obtained from the right (Figure, C) and left axillae (Figure, D) and sent for histopathological examination", "A, Clinical images show punctate yellow-brown papules on the upper eyelids and infraorbital rims", "B, numerous red-brown papules in the right axilla", "C, Histopathology of a shave biopsy sample from the right axilla (original magnification ×200)", "D, Histopathology of a shave biopsy sample taken from the left axilla (original magnification ×200)"], "s1": [0, 1, 6, 7, 8, 9, 10], "s2": [2, 3, 4, 5]} {"key": 848, "questions": "What Is Your Diagnosis?", "options": [{"label": "A", "disease": "Childhood bullous pemphigoid"}, {"label": "B", "disease": "Dyshidrotic eczema"}, {"label": "C", "disease": "Bullous tinea"}, {"label": "D", "disease": "Distal blistering dactylitis"}], "answer_idx": "B", "symptoms": ["A young girl presented to the pediatric emergency department for evaluation of blisters on both of her hands", "Three weeks prior, she had returned from school with 2 itchy papules on her right hand", "The papules continued to enlarge over the course of the day and progressed to form blisters", "The following day, she noticed similar lesions appearing on her left hand", "She was evaluated by her pediatrician, who prescribed oral cefdinir and hydrocortisone, 2.5%, cream, but there was no substantive improvement", "On further questioning, she recalled similar itchy papules appearing on her feet without progression to blisters", "Her medical history was significant for atopic dermatitis that was well controlled with topical triamcinolone, 0.1%, cream", "Physical examination demonstrated numerous skin-colored tense bullae on her bilateral hands (Figure 1, A)", "Xerosis was on the trunk and extremities", "On her dorsal and plantar feet, there were macerated, erosive plaques without vesicles or bullae (Figure 1, B)", "Results of herpes simplex virus polymerase chain reaction, bacterial culture, potassium hydroxide preparation, mineral oil preparation, and direct immunofluorescence were negative", "A, Bilateral palms with at least 20 tense, white-gold bullae, several with violaceous borders and overlying tapioca-like appearance, present diffusely but concentrated on the palmar surfaces", "B, Bilateral dorsal feet with macerated, erosive plaques", "No active vesicles or bullae present"], "s1": [6, 8, 10], "s2": [0, 1, 2, 3, 4, 5, 7, 9, 11, 12, 13]} {"key": 849, "questions": "What Is Your Diagnosis?", "options": [{"label": "A", "disease": "Appendix abscess"}, {"label": "B", "disease": "Cecal carcinoma"}, {"label": "C", "disease": "Mucocele of the appendix"}, {"label": "D", "disease": "Neuroendocrine tumor"}], "answer_idx": "C", "symptoms": ["A 60-year-old woman was referred to the emergency department with a history of abdominal discomfort and a palpable mass in the right iliac fossa", "She denied any change in her bowel habits or having experienced rectal bleeding", "She had a normal appetite and had no history of weight loss", "Before this admission, she was healthy, with no previous medical problems", "On examination, her vital signs were within normal limits", "She was mildly tender in the right iliac fossa and there was a large mobile mass, at least 10 cm in diameter, that was easily palpable in the right iliac fossa", "She was initially investigated with a computed tomography (CT) scan of the abdomen (Figure 1)", "She underwent a colonoscopy that showed no intraluminal mucosal abnormalities or masses, but evidence of external compression at the cecum was noted", "Coronal (A) and transverse (B) computed tomographic images show a large intra-abdominal cystic mass in the right iliac fossa"], "s1": [0, 1, 2, 3, 4, 5], "s2": [6, 7, 8]} {"key": 850, "questions": "What Is Your Diagnosis?", "options": [{"label": "A", "disease": "Tumor recurrence"}, {"label": "B", "disease": "Marginal ulcer"}, {"label": "C", "disease": "Afferent loop syndrome"}, {"label": "D", "disease": "Internal hernia"}], "answer_idx": "C", "symptoms": ["A man in his 70s with history of pancreatic adenocarcinoma who had undergone a Whipple procedure 2 years prior to presentation came to the emergency department with a 1-day history of waxing and waning sharp abdominal pain radiating to his back", "He admitted to nausea but no vomiting, fevers, or chills", "He reported having normal bowel movements and passing flatus without difficulty", "On examination, he was afebrile and not tachycardic", "He had a well-healed abdominal midline wound", "His abdomen was soft, not distended, and mildly tender diffusely, worse in the epigastrium, without rebound, guarding, or any palpable masses", "His bloodwork revealed an elevated lipase level of 2842 U/L (to convert to microkatals per liter, multiply by 0.0167) without leukocytosis or any electrolyte abnormalities", "Liver function test results were within normal limits", "Given a history of pancreaticoduodenectomy, the patient underwent computed tomography of the abdomen and pelvis with oral and intravenous contrast for further evaluation (Figure)", "A, Axial cross-section of patient's computed tomographic scan of his abdomen and pelvis after a Whipple procedure", "The stomach is distended with oral contrast and an air fluid level", "A tubular solid structure can be seen within the body of the stomach", "B, Coronal view demonstrating the tubular solid mass within the distended stomach"], "s1": [0, 1, 2, 3, 5, 6, 7], "s2": [4, 8, 9, 10, 11, 12]} {"key": 851, "questions": "What Is Your Diagnosis?", "options": [{"label": "A", "disease": "Nodular fasciitis"}, {"label": "B", "disease": "Inflammatory myofibroblastic tumor"}, {"label": "C", "disease": "Spindle cell carcinoma"}, {"label": "D", "disease": "Rhabdomyosarcoma"}], "answer_idx": "B", "symptoms": ["A 4-year-old white girl presented with left jaw pain and signs and symptoms of an upper respiratory tract infection", "Her medical history included recurrent acute otitis media requiring tympanostomy tube placement", "On physical examination, the left parotid gland was diffusely full and firm, which was consistent with viral parotitis", "Gland massage, warm compress, sialagogues, and increased hydration were recommended", "At the 2-week follow-up, the swelling had largely resolved with a now-firm, slightly mobile, subcentimeter mass palpable in the anterior parotid gland", "Four days later the mass enlarged and became painful", "A magnetic resonance imaging (MRI) study identified a 1.2 × 1.7 × 1.6-cm solid mass in the anterior parotid gland with well-defined borders and indention into the masseter muscle", "The mass was isointense to muscle on T1, hyperintense on T2, with strong contrast enhancement, heterogeneous internal enhancement and a central focal area of hypointensity (Figure, A)", "The patient underwent a left superficial parotidectomy", "Grossly the lesion was nonencapsulated, firm, dark colored, and was located within the anterior superficial lobe of the parotid gland", "Microscopically, the tumor was composed of spindle cells with elongated nuclei in a storiform pattern embedded in moderately dense admixed lymphohistiocytic infiltrate with no atypia (Figure, B)", "Immunohistochemical analysis (IHC) was positive for activin-like kinase-1 (Alk-1) (Figure, C), vimentin, CD68, and factor XIIIa, and negative for smooth muscle actin (SMA), desmin, cytokeratin, and CD34.A, Postcontrast axial magnetic resonance image (MRI) demonstrating mass (arrowhead) with strong contrast enhancement, heterogeneous internal enhancement with a central focal area of hypointensity", "B and C, Histopathologic images, original magnification ×200. B, Spindle cells with elongated nuclei in a storiform pattern embedded in moderately dense admixed lymphohistiocytic infiltrate with no atypia", "C, Positivity for Alk-1."], "s1": [0, 1, 2, 3, 4, 5], "s2": [6, 7, 8, 9, 10, 11, 12, 13]} {"key": 852, "questions": "What Is Your Diagnosis?", "options": [{"label": "A", "disease": "Burkitt lymphoma"}, {"label": "B", "disease": "Diffuse large B-cell lymphoma"}, {"label": "C", "disease": "Plasmablastic lymphoma"}, {"label": "D", "disease": "Kaposi sarcoma"}], "answer_idx": "C", "symptoms": ["A preadolescent girl presented with a 1-week history of left-sided facial swelling", "She had presented 3 months prior with neurologic and vision changes and was diagnosed as having varicella zoster encephalitis and cytomegalovirus retinitis", "Further testing led to a diagnosis of HIV/AIDS, and at that time she was started on highly active antiretroviral therapy (HAART) therapy as well as bactrim prophylaxis", "Physical examination revealed left-sided facial fullness and a left-sided violaceous, vascular-appearing lesion involving the left maxillary gingiva (Figure, A)", "The patient was afebrile, her white blood cell count was 3300/μL, and her CD4 count was 210 μg/mL", "Magnetic resonance imaging (MRI) of the face revealed a poorly enhancing mass arising from the left maxilla and extending into the left maxillary sinus (Figure, B)", "An incisional biopsy was undertaken, during which excessive bleeding was encountered", "Histologic analysis and immunophenotype of the specimen showed a dual population of lymphoma cells with plasmablasts having large vesicular nuclei and centrally located eosinophilic nucleoli and “plasmacytic” cells with comparatively smaller darker nuclei and smaller scanty nucleoli (Figure, C)", "Immunohistochemical expression was strongly positive for CD138 (Figure, D) and Epstein-Barr virus–encoded small nuclear RNA (EBV-EBER), and the Ki-67 index (marker of mitotic division) was greater than 90%, negative for CD45, CD20, HHV-8, TdT, CD34, PAX5, CD3, CD99, ALK-1, and CD30. Flow cytometry was noncontributory"], "s1": [0, 1, 2, 4], "s2": [3, 5, 6, 7, 8]} {"key": 853, "questions": "What Is Your Diagnosis?", "options": [{"label": "A", "disease": "Synovial sarcoma"}, {"label": "B", "disease": "Cervical neurilemmoma"}, {"label": "C", "disease": "Ectopic hamartomatous thymoma"}, {"label": "D", "disease": "Pleomorphic adenoma"}], "answer_idx": "C", "symptoms": ["A man in his 70s presented with a 6-year history of a painless lump in his left supraclavicular region", "Palpation detected a smooth, soft, and fluctuant mass in his left supraclavicular region", "Fiberscopic examination of the pharynx and larynx revealed no abnormalities", "Contrast-enhanced computed tomography (CT) demonstrated a poorly circumscribed, heterogeneous, and cystic mass near the sternocleidomastoid muscle (SCM)", "The SCM around the tumor was thickened, suggesting invasion of the SCM", "There was no continuity between the tumor and the thyroid gland (Figure, A)", "Magnetic resonance imaging revealed a poorly marginated, heterogeneous, and partially tubular mass", "Fine-needle aspiration cytology revealed cystic elements but no atypical cells", "Positron emission tomography showed hypermetabolism (maximum standardized uptake value, 1.6) in the mass, but no other hypermetabolic foci", "At surgery, the tumor was attached to the medial side of the origin of the left SCM", "No extensive adhesions were observed, and dissection of the tumor from the adjacent tissues was straightforward", "The lesion measured 2.5 × 3.0 × 4.0 cm, was a solitary well-circumscribed roundish nodule with a thin capsule, and its cut surface had a yellowish white homogeneous surface (Figure, B)", "Pathological examination revealed that the mass comprised an irregular combination of squamous epithelial and spindle cells, with a mature adipose tissue component (Figure, C and D)", "Immunohistochemical analysis revealed that the epithelial component was diffusely positive for cytokeratin AE1/AE3.A, Contrast-enhanced neck computed tomographic (CT) image", "B, Macroscopic findings", "C and D, Histopathologic images, hematoxylin-eosin", "C, Original magnification ×100. D, Original magnification ×200."], "s1": [0, 1, 4, 6, 9, 10, 11, 12], "s2": [2, 3, 5, 7, 8, 13, 14, 15, 16]} {"key": 854, "questions": "What Is Your Diagnosis?", "options": [{"label": "A", "disease": "Leiomyosarcoma"}, {"label": "B", "disease": "Olfactory neuroblastoma"}, {"label": "C", "disease": "Sinonasal mucosal melanoma"}, {"label": "D", "disease": "Sinonasal undifferentiated carcinoma"}], "answer_idx": "C", "symptoms": ["A man in his 70s presented to an outside institution with a 4-month history of right-sided nasal congestion, which failed to resolve with nasal sprays, antibiotics, and oral prednisone", "He also reported intermittent right-sided epistaxis occurring multiple times per day", "He had not experienced prior nasal trauma or surgery and had quit smoking 30 years ago", "At the outside institution, a nasal mass was found on examination and biopsied", "He was referred to our office for further management", "Endoscopic examination showed a dark, soft-tissue mass filling the right nasal cavity", "Computed tomography (CT) of the sinuses revealed opacification of the right nasal cavity with tissue protruding into the nasopharynx and complete opacification of the right maxillary, anterior ethmoid, and frontal sinuses (Figure, A and B)", "The patient underwent surgical resection", "Removal of the mass required a right medial maxillectomy, septectomy, and excision of the right nasolacrimal duct via endoscopic approach", "Resection also necessitated removal of the anterior maxillary sinus wall via Caldwell-Luc procedure", "Biopsy specimens were taken throughout", "Histopathologic analysis from both the outside hospital and our institution revealed sheets of atypical, pleomorphic epithelioid cells, some with prominent pigment (Figure, C)", "Immunohistochemical stains were positive for HMB-45 (Figure, D) and S-100.A and B, Noncontrast computed tomographic images of the paranasal sinuses", "C and D, Histopathologic images", "C, Hematoxylin-eosin", "D, HMB-45 stain"], "s1": [0, 1, 2, 4, 5, 6], "s2": [3, 7, 8, 9, 10, 11, 12, 13, 14, 15]} {"key": 855, "questions": "What Is Your Diagnosis?", "options": [{"label": "A", "disease": "Rat bite fever"}, {"label": "B", "disease": "Rocky mountain spotted fever"}, {"label": "C", "disease": "Septic emboli"}, {"label": "D", "disease": "Disseminated gonococcemia"}], "answer_idx": "A", "symptoms": ["A man in his 70s was admitted to the hospital with encephalopathy, recent falls, leukocytosis, nausea, and arthralgias", "On presentation to the emergency department, he received antibiotics owing to a concern for aspiration pneumonia", "He underwent a stroke workup, including transesophageal echocardiography (TEE), results of which were negative", "He was noted to have a progressing cutaneous eruption during his admission, and the dermatology department was consulted", "He was found to have scattered petechiae and hemorrhagic vesicles, with most concentrated on the hands and feet (Figure, A and B)", "Palpable purpura were noted across the extremities", "Several lesions were studded with pustules", "He had an enlarging bulla on his left wrist", "An abrasion on his left elbow was attributed to his recent falls", "The infectious disease department was also consulted, and the patient was given broad-spectrum antibiotics owing to concern about endocarditis", "As part of his dermatological workup, two 4-mm punch biopsy specimens were obtained (Figure, C and D)", "One was sent for histopathological examination, and the other was sent for fungal, bacterial, and mycobacterial cultures", "A bacterial culture swab was obtained from the tense bulla on his left wrist", "A, Pustule on dorsal surface of fingers", "B, Acral eruption on the dorsal surface of the foot", "C and D, Skin biopsy specimens, hematoxylin-eosin", "C, Specimen from the dorsal side of the right foot", "D, Focal ulceration and underlying vascular thrombosis with extramural fibrin deposition"], "s1": [0, 1, 2, 8, 9, 11], "s2": [3, 4, 5, 6, 7, 10, 12, 13, 14, 15, 16, 17]} {"key": 856, "questions": "What Is Your Diagnosis?", "options": [{"label": "A", "disease": "Papillary/exophytic squamous cell carcinoma"}, {"label": "B", "disease": "Verruca vulgaris"}, {"label": "C", "disease": "Rhinoscleroma"}, {"label": "D", "disease": "Exophytic Schneiderian papilloma"}], "answer_idx": "D", "symptoms": ["A man in his 30s presented with a mass in the right nostril that had been growing for about 1 year", "The lesion bled with minimal trauma but did not obstruct his breathing", "The patient did not smoke and denied any medical history of chronic rhinosinusitis", "Physical examination revealed a 1-cm, nontender, exophytic, verrucous, skin-colored plaque on the right nasal vestibule (Figure, A and B)", "A diagnostic shave biopsy specimen was taken after informed consent was obtained (Figure, C and D)", "A and B, An exophytic, verrucous, and skin-colored plaque on the right nasal vestibule", "C, Histologic sections display papillary fronds with slender fibrovascular cores that are lined by nonkeratinizing squamous epithelium (original magnification ×200)", "D, Higher magnification shows mucin-containing goblet cells", "Neither cytologic atypia nor atypical mitotic figures are present (original magnification ×400)"], "s1": [0, 1, 2, 3], "s2": [4, 5, 6, 7, 8]} {"key": 857, "questions": "What Is Your Diagnosis?", "options": [{"label": "A", "disease": "Bacterial folliculitis"}, {"label": "B", "disease": "Mycobacterium marinum"}, {"label": "C", "disease": "Sporotrichosis"}, {"label": "D", "disease": "Majocchi granuloma"}], "answer_idx": "D", "symptoms": ["A woman in her 20s developed a cutaneous eruption on her left leg 3 days after returning from a trout fishing trip in a freshwater lake", "She recalled having a small abrasion on her left leg before wading waist-deep in search of trout", "Prior to presentation, she was unsuccessfully treated by 3 different physicians for allergic contact dermatitis and cellulitis", "Attempted treatments included topical and oral corticosteroids, trimethoprim-sulfamethoxazole, ceftriaxone, doxycycline, and clindamycin", "Following 1 month of failed treatment, she presented to the dermatology clinic", "Physical examination revealed erythematous, crusted, edematous plaques and pustules with weeping of yellow exudate on the left leg (Figure, A)", "Plaques extended from the lower mid-shin to the thigh in a sporotrichoid spread, ending with a palpable enlarged lymph node in the left groin", "The left leg was tender to palpation", "The patient reported no history of cutaneous eruptions on her legs or feet, and there was no obvious thickening, yellowish discoloration, or subungual debris of the fingernails or toenails on examination", "A laboratory work-up, potassium hydroxide scraping (Figure, C), and punch biopsies for histopathologic analysis and fungal, bacterial, and atypical mycobacterial tissue cultures were performed", "A, Erythematous, crusted, edematous plaques and pustules on the left leg prior to definitive treatment", "B, Near complete resolution of pustules and resultant postinflammatory hyperpigmentation after definitive treatment", "C, Potassium hydroxide scraping taken from the left leg"], "s1": [0, 1, 4, 5, 6, 7, 10, 11], "s2": [2, 3, 8, 9, 12]} {"key": 858, "questions": "What Is Your Diagnosis?", "options": [{"label": "A", "disease": "Acute generalized exanthematous pustulosis"}, {"label": "B", "disease": "Pustular psoriasis"}, {"label": "C", "disease": "Deficiency of the interleukin-1 receptor antagonist"}, {"label": "D", "disease": "Staphylococcal scalded skin syndrome"}], "answer_idx": "A", "symptoms": ["An 18-month-old boy with a history of atopic dermatitis presented to the emergency department with a 2-day history of high fever and rash that began 2 days after completing an 8-day course of clindamycin for sinusitis", "On presentation, the child was lethargic and fussy, with a temperature of 38.1°C", "Physical examination revealed a strikingly symmetrical eruption of erythematous, edematous, annular plaques with superimposed small pustules at the periphery, and associated desquamation (Figure, A and B)", "The eruption was accentuated in the inguinal folds, antecubital fossae, and popliteal fossae", "Laboratory evaluation was significant for leukocytosis with neutrophilia (26000 μL", "to convert to × 109 per liter, multiply by 0.001)", "Neutrophil levels were 66.7% (to convert to proportion of 1.0, multiply by 0.01), and lymphocyte levels were 9.9% (to convert to proportion of 1.0, multiply by 0.01)", "C-reactive protein and erythrocyte sedimentation rate levels were within normal limits", "Test results for anti-DNase B antibodies, a respiratory viral panel, and blood and throat cultures were all negative", "A, Right arm with erythematous edematous plaque with superimposed pinpoint pustules and associated superficial desquamation", "B, Bilateral buttocks with well-demarcated, erythematous plaques with pustules at the periphery", "Superficial desquamation is present at the superior aspect of the buttocks", "C, Hematoxylin-eosin stain (original magnification × 20)", "There was no ocular or mucosal involvement and no lymphadenopathy", "There was no family history of psoriasis", "Biopsy of the left thigh revealed a subcorneal pustule with underlying neutrophilic spongiosis, with associated psoriasiform epidermal hyperplasia and rare individually necrotic keratinocytes", "There was edema of the papillary dermis and a mild superficial perivascular lymphocytic infiltrate with scattered neutrophils (Figure, C)"], "s1": [0, 1, 3, 8, 13, 14], "s2": [2, 4, 5, 6, 7, 9, 10, 11, 12, 15, 16]} {"key": 859, "questions": "What Is Your Diagnosis?", "options": [{"label": "A", "disease": "Giant cell tumor"}, {"label": "B", "disease": "Giant cell reparative granuloma"}, {"label": "C", "disease": "Brown tumor"}, {"label": "D", "disease": "Metastatic lytic bone lesions of unknown primary"}], "answer_idx": "C", "symptoms": ["A 43-year-old woman presented to the emergency department with substernal chest pain and dyspnea", "After ruling out acute coronary syndrome or pulmonary embolus, a computed tomography scan of her thorax revealed lytic lesions of the sternum and second rib and a large mass involving the right scapula (Figure 1)", "On further inquiry, the patient acknowledged right shoulder pain that had been progressing over 5 months and leading to a limited mobility of the right upper extremity", "Cross-sectional computed tomography demonstrating a mass of the right shoulder (arrowhead)", "Her comorbidities included hypertension, morbid obesity, fibromyalgia, and obstructive sleep apnea dependent on positive pressure ventilation", "She underwent a tonsillectomy at age 5 years", "She smoked for 1 year but quit 20 years before this presentation", "There was no family history of malignancy", "On examination she was morbidly obese with a body mass index (calculated as weight in kilograms divided by height in meters squared) of 63. There was fullness involving the right scapula", "She could not abduct her right arm", "Laboratory testing included comprehensive metabolic panel and complete blood cell count results within the normal range, except for a calcium level of 10.4 mg/dL (for millimoles per liter, multiply by 0.25) (reference range, 8.4-10.2) and alkaline phosphatase level of 400 U/L (for micokatals per liter, multiply by 0.0167) (reference range, 36-126)", "A computed tomography–guided biopsy of the mass gave a broad differential", "She was referred to our Cancer Center with a diagnosis of a metastatic giant cell tumor", "Magnetic resonance imaging of the shoulder was not possible because of weight limitations"], "s1": [0, 4, 5, 6, 7, 8, 10, 13], "s2": [1, 2, 3, 9, 11, 12]} {"key": 860, "questions": "What Is Your Diagnosis?", "options": [{"label": "A", "disease": "A foreign body reaction (piece of glove)"}, {"label": "B", "disease": "An osteitis"}, {"label": "C", "disease": "A recurrent herpetic whitlow"}, {"label": "D", "disease": "A bacterial cellulitis"}], "answer_idx": "C", "symptoms": ["A 35-year-old white head and neck surgeon with an unremarkable medical history was referred to the Surgical Emergency Department for a phlegmon of the flexor tendon sheath of the right hand", "The lesion progressively developed several days after an unintentional exposure to blood with a blade used to drain an abscess of the oral cavity", "The patient presented a deep wound induced by the bistoury piercing of the index finger pulp of the right hand, with a disrupted epidermal barrier", "Both the patient who was operated on and the surgeon were negative for human immunodeficiency virus and hepatitis B/C virus testing", "A soft tissue abscess of the digit appeared 7 days after the initial trauma", "Antibiotic therapy combining amoxicillin and clavulanate was started, without any significant improvement", "Surgery was performed urgently, with excision of the wound entrance and washing of the flexor tendon sheath", "Bacteriological swabs were performed in the same time", "An antibiotic regimen with amoxicillin/clavulanate was prescribed", "Forty-eight hours after surgery, pustules appeared", "Further drainage and washing of the sheath were repeated (Figure, A)", "The bacteriological samples were found to be sterile", "The surgeon was hospitalized for infusions combining piperacillin/tazobactam and teicoplanine, carried out over 72 hours", "An oral antibiotic regimen with amoxicillin/clavulanate (3 g every 24 hours) was prescribed for 10 additional days", "The lesions completely resolved within 2 weeks", "One year later, the surgeon presented with prodromal burning pain of the same index finger, and a similar soft tissue abscess recurred after 48 hours at the same site of the original lesion with a small painful vesicle progressively developed (Figure, B)", "A, Clinical aspect of the index finger 2 days after the second surgery", "B, Recurrence of the soft tissue abscess"], "s1": [0, 1, 2, 3, 4, 6, 7, 9, 15, 16, 17], "s2": [5, 8, 10, 11, 12, 13, 14]} {"key": 861, "questions": "What Is Your Diagnosis?", "options": [{"label": "A", "disease": "Cholesterol granuloma"}, {"label": "B", "disease": "Mucocele"}, {"label": "C", "disease": "Chondrosarcoma"}, {"label": "D", "disease": "Chordoma"}], "answer_idx": "A", "symptoms": ["A man in his 30s presented with a 1-week history of worsening blurry vision and gait instability", "He had been treated approximately 4 months prior at an outside facility for cerebral infarct with resultant right lower extremity weakness", "On presentation, he complained of headache and nasal congestion but denied diplopia or rhinorrhea", "He had no history of head trauma or surgery", "Findings from the ophthalmologic examination revealed bilateral visual acuity of 20/100 with intact visual fields", "Findings from a head and neck examination were otherwise within normal limits, with cranial nerves III to XII intact", "Computed tomography (CT) of the sinuses with image guidance protocol revealed a 5.6 × 2.7 × 2.0-cm, well-circumscribed expansile mass occupying the left petrous apex and sphenoid sinus (Figure, A)", "Magnetic resonance imaging (MRI) demonstrated that this mass was heterogeneous on T1-weighted images (Figure, B) and hyperintense on T2-weighted images", "The mass did not enhance with gadolinium", "The position of the internal carotid artery was not obviously displaced in either a posterior or anterior direction (Figure, A and B)", "Therefore, it was not entirely clear from imaging if this lesion originated in the petrous apex or sphenoid sinus", "The patient was taken to the operating room in conjunction with the neurosurgery team for an endoscopic, transnasal approach to this lesion", "A wide sphenoidotomy was performed, and a cystic mass was identified within the sphenoid sinus (Figure, C)", "On entering the capsule, a golden-brown fluid was encountered (Figure, D)"], "s1": [0, 1, 2, 3, 4, 5], "s2": [6, 7, 8, 9, 10, 11, 12, 13]} {"key": 862, "questions": "What Is Your Diagnosis?", "options": [{"label": "A", "disease": "Skull base osteomyelitis"}, {"label": "B", "disease": "Extramedullary chronic lymphocytic leukemia"}, {"label": "C", "disease": "Chronic nonsuppurative otitis media"}, {"label": "D", "disease": "Langerhans cell histiocytosis of the temporal bone"}], "answer_idx": "B", "symptoms": ["A man in his 70s presented with painless left-sided otorrhea, aural fullness, and hearing loss for nearly 1 year", "He had previous complaints of left-sided aural pressure and intermittent clear otorrhea that were unsuccessfully treated at another center with 2 sets of pressure equalization tubes (PETs) and 3 courses of ototopical antibiotics", "In the month before presenting to us, the otorrhea had turned bloody", "He had not experienced fever, malaise, weight loss, lymphadenopathy, skin lesions, or vertigo", "Pertinent medical history included obesity, type 2 diabetes mellitus, allergic rhinitis, and chronic lymphocytic leukemia (CLL)", "The CLL had been treated successfully with chemotherapy and had been in remission for the past 6 years", "The only notable finding from the physical examination was a hypomobile left tympanic membrane with an anterior-inferior PET plugged by granulation tissue", "Facial nerve function was normal bilaterally", "Audiometry results demonstrated a left moderate sloping to profound mixed hearing loss and speech discrimination score of 68%", "A fine-cut, temporal bone computed tomographic (CT) scan was obtained, which demonstrated near-complete opacification of the left mastoid air cells, and middle ear (Figure, A)", "The patient underwent a left tympanomastoidectomy for continuing otorrhea", "Bulky soft tissue was removed from the middle ear and mastoid", "Cytopathologic examination revealed numerous, small, basophilic cells (Figure, B)", "Immunostaining showed that the cells were CD20 positive (Figure, C) and CD3 negative (Figure, D)", "A, Coronal computed tomographic (CT) scan showing near-total opacification of the left mastoid air cells", "B-D, Histopathologic images, original magnification ×40. B-D, Histopathological images, original magnification ×40. B, Biopsy specimen demonstrating homogenous, small basophilic cells", "C, Positive immunostaining with anti-CD20 antibodies", "D, Negative Immunostaining with anti-CD3 antibodies"], "s1": [0, 1, 2, 3, 4, 5, 6, 7, 8, 10], "s2": [9, 11, 12, 13, 14, 15, 16, 17]} {"key": 863, "questions": "What Is Your Diagnosis?", "options": [{"label": "A", "disease": "Intraosseous meningioma"}, {"label": "B", "disease": "Mucocele"}, {"label": "C", "disease": "Fibrous dysplasia"}, {"label": "D", "disease": "Osteosarcoma"}], "answer_idx": "A", "symptoms": ["A woman in her 50s with a long history of allergic rhinitis was referred for progressive periorbital edema with proptosis of the left eye for the past 2 years, with associated nasal congestion, altered sense of smell, and intermittent left supraorbital pain", "There was no associated history of diplopia or worsening visual acuity", "She had not experienced facial trauma or sinus surgery", "Treatment of allergic rhinitis with cetirizine and intranasal fluticasone had provided only partial relief", "On examination, there was proptosis and periorbital edema of the left eye without any restriction of extraocular movements", "The left supraorbital area was tender to palpation, with no abnormal masses appreciated", "A computed tomographic (CT) scan demonstrated extensive thickening of the left frontal bone, extending over the entire roof of the orbit posteriorly (Figure, A and B)", "Changes within the frontal sinus were consistent with a soft-tissue component of the mass", "Subsequent magnetic resonance imaging (MRI) (Figure, C and D) showed markedly hypertrophic bone in the frontal region and a homogeneously enhancing lesion within the left frontal sinus that was hyperintense on T2-weighted images, with left extraconal superior orbital and left frontal intracranial extension with dural enhancement", "A combination above-and-below approach combining unilateral frontal sinusotomy with a trephination was performed", "Abnormal tissue appearing in the left frontal sinus was removed, along with diseased tissue that extended into the orbital roof and orbit", "Specimens were sent for pathologic evaluation, and findings demonstrated bony spaces containing lobules and whorls of spindle cells", "A and B, Preoperative computed tomography (CT)", "A, Extensive thickening of the left frontal bone", "B, Soft-tissue component in left frontal sinus", "C and D, Magnetic resonance imaging (MRI)", "C, Enhancement of the lesion", "D, Lesion within the left frontal sinus, with superior orbital and intracranial extension"], "s1": [0, 1, 2, 3, 4, 5, 9, 10, 11], "s2": [6, 7, 8, 12, 13, 14, 15, 16, 17]} {"key": 864, "questions": "What Is Your Diagnosis?", "options": [{"label": "A", "disease": "Pseudoaneurysm"}, {"label": "B", "disease": "Parapharyngeal abscess"}, {"label": "C", "disease": "Paraganglioma"}, {"label": "D", "disease": "Venolymphatic malformation"}], "answer_idx": "A", "symptoms": ["A man in his 50s presented to the emergency department with sudden onset of left-sided neck swelling, pain, and progressive respiratory distress that started while playing basketball", "He reported a 2-day history of left-sided jaw pain preceding the swelling but had no other clinically significant history", "Physical examination showed trismus, rightward tracheal deviation, and a hoarse voice", "The left side of his neck was diffusely swollen without palpable fluctuance, induration, or a discrete mass", "A complete blood cell count revealed no abnormalities", "A contrast-enhanced computed tomographic (CT) scan of the neck showed a large hyperdense mass interposed between the left common carotid artery and the hypopharynx (Figure, A-C)", "There was resultant lateral deviation of the left common carotid artery and medial deviation on the supraglottic larynx and hypopharynx, resulting in airway compromise", "Of note, the density of the mass was identical to that of the adjacent vasculature", "There was extensive edema surrounding the primary abnormality", "Additional findings include periapical dental abscesses in 2 teeth (Figure, D)"], "s1": [0, 1, 3, 4, 9], "s2": [2, 5, 6, 7, 8]} {"key": 865, "questions": "What Is Your Diagnosis?", "options": [{"label": "A", "disease": "Cecal volvulus"}, {"label": "B", "disease": "Aerophagia"}, {"label": "C", "disease": "Sigmoid volvulus"}, {"label": "D", "disease": "Colonic ileus/colonic pseudoobstruction (Ogilvie syndrome)"}], "answer_idx": "C", "symptoms": ["A 77-year-old institutionalized and bedridden man with severe Alzheimer dementia and long-standing schizophrenia was admitted to the emergency department for bowel obstruction with progressive abdominal distension and tenderness", "His family members and caregivers accompanied him", "At physical examination the patient was pale, sweaty, peripherally hypoperfused, tachypnoeic, and dehydrated, and the abdomen was extremely distended, diffusely tympanic, and tender at palpation", "His blood pressure was 90/70 mm Hg, his heart rate was 114 beats per minute, his oxygen saturation was at 90%, his arterial blood gas pH level was 7.43, his partial pressure of carbon dioxide was 32, his partial pressure of oxygen 68, his laceration lactate level was 5.0, and his white blood cell count was 24 000 mm3. A plain abdominal radiograph showed a huge colonic distension (Figure 1A)", "A computed tomography scan with multiplanar reconstruction was also performed (Figure 1B)", "A, A plain abdominal x-ray shows a huge colonic distension and a clear coffee-bean-like or “bent inner tube” sign is visible", "B, A computed tomography scan with multiplanar reconstruction"], "s1": [0, 1, 2], "s2": [3, 4, 5, 6]} {"key": 866, "questions": "What Is Your Diagnosis?", "options": [{"label": "A", "disease": "Metastatic lung cancer"}, {"label": "B", "disease": "Small-bowel intussusception secondary to pelvic adhesions"}, {"label": "C", "disease": "Phytobezoar"}, {"label": "D", "disease": "Endometriosis"}], "answer_idx": "C", "symptoms": ["A 45-year-old woman presented to the emergency department on the morning after Thanksgiving with sudden, severe, constant epigastric pain that migrated to the right upper quadrant as well as subjective fever and emesis since the previous evening", "She reported a failure to pass stool or flatus since the onset of symptoms", "She denied hematochezia, melena, or hematemesis", "Her medical history was significant for heavy smoking, reflux, and surgical procedures including a right inguinal hernia repair, total abdominal hysterectomy, and bilateral salpingo-oopherectomy, all more than 5 years prior", "On examination, she had a distended and tympanic abdomen with tenderness, guarding, and rebound in the epigastrium and right upper quadrant", "Vital signs and standard laboratory test results were unremarkable", "An upright abdominal plain radiograph showed a distended, thickened stomach and multiple dilated loops of small bowel with air-fluid levels", "Contrast-enhanced computed tomography demonstrated a focal area of mixed fat density involving the distal small bowel concerning for a lead point with adjacent mesenteric fat stranding (Figure 1)", "The scan also revealed multiple bilateral lung nodules", "Computed tomography of abdomen", "A, Coronal view demonstrating obstructing intraluminal lesion (arrowhead)", "B, Transverse view demonstrating obstructing intraluminal lesion (arrowhead)", "The patient was subsequently taken to the operating room for exploratory laparotomy and found to have an obstruction in the distal ileum"], "s1": [0, 1, 2, 3, 4, 5, 6, 12], "s2": [7, 8, 9, 10, 11]} {"key": 867, "questions": "What Is Your Diagnosis?", "options": [{"label": "A", "disease": "Low-grade malignant peripheral nerve sheath tumor"}, {"label": "B", "disease": "Monophasic synovial sarcoma"}, {"label": "C", "disease": "Biphenotypic sinonasal sarcoma"}, {"label": "D", "disease": "Glomangiopericytoma"}], "answer_idx": "C", "symptoms": ["A man in his 50s was referred for a 6-month history of swelling involving the nasal bridge and right eye, accompanied by intermittent ipsilateral epistaxis", "His medical history was notable for allergic rhinitis and well-controlled type 2 diabetes mellitus", "The prior biopsy diagnosis from the area of swelling was inflammatory nasal polyp", "Physical examination revealed tenderness to palpation over the right nasal bridge and mild right proptosis", "Flexible and rigid nasal endoscopy demonstrated a right-sided exophytic mass visible between the middle turbinate and nasal septum", "Computed tomography showed a large, right-sided mass centered in the ethmoid sinus with expansion into the anterior skull base, anterior cribriform plate, and right medial orbit", "Magnetic resonance imaging showed vascularity in the lesion and early intracranial involvement with transdural spread but no evidence of invasion into the meninges or brain", "The patient was taken for surgery, and findings from intraoperative frozen section analysis were consistent with a spindle cell neoplasm", "Definitive skull base resection of the mass was deferred until final pathological diagnosis was obtained, given the morbidity with skull base resection", "The final pathology report showed an infiltrative, low-grade, malignant spindle cell neoplasm with immunoreactivity for BCL2 protein and neural marker, S-100 protein, and focal immunoreactivity for smooth muscle markers SMA and HHF35 (Figure)", "Cytoplasmic immunoreactivity for β-catenin was noted, and nonreactive immunostains included CD31, CD34, epithelial membrane antigen (EMA), and CD99. In addition, there was minimal mitotic activity, and no tumoral necrosis was present", "There was no regional or distant metastasis, and TNM staging was cT4N0M0.Histopathologic photomicrographs of a biopsy specimen from a mass centered in the ethmoid sinus", "A and B, Hematoxylin-eosin", "A, Spindle cells in fascicular pattern", "B, Invagination of respiratory-type epithelium", "C, Tissue section showing strong nuclear and cytoplasmic immunologic staining for S-100 protein", "D, Immunologic staining for smooth muscle actin showing patchy immunoreactivity in tumor cells"], "s1": [0, 1, 2, 3, 4, 5, 6, 7, 8, 11], "s2": [9, 10, 12, 13, 14, 15, 16]} {"key": 868, "questions": "What Is Your Diagnosis?", "options": [{"label": "A", "disease": "Plasmacytoma"}, {"label": "B", "disease": "Ameloblastoma"}, {"label": "C", "disease": "Osteosarcoma"}, {"label": "D", "disease": "Venous malformation"}], "answer_idx": "A", "symptoms": ["A woman in her 60s presented with mild to moderate trismus and swelling on the left side of the face of a few months’ duration", "Her medical history was significant for herpes zoster and self-limited vertigo approximately 3 years prior", "Her physical examination was remarkable for left-sided facial swelling", "No deficits of the facial or trigeminal nerves were detected", "Magnetic resonance imaging (MRI) was performed (Figure)", "Magnetic resonance imaging, axial views", "A", "T1-weighted image demonstrating a mass lying on the anterior surface of the eroded left mandibular condyle with homogeneous isointensity relative to skeletal muscle", "B, Postcontrast T1-weighted image with fat suppression showing homogeneous, avid enhancement of the mass", "C, T2-weighted image showing homogeneous moderately hyperintense signal", "D, Diffusion-weighted image is significant for marked diffusion restriction"], "s1": [0, 1, 2, 3], "s2": [4, 5, 6, 7, 8, 9, 10]} {"key": 869, "questions": "What Is Your Diagnosis?", "options": [{"label": "A", "disease": "Pleomorphic adenoma"}, {"label": "B", "disease": "Mucoepidermoid carcinoma"}, {"label": "C", "disease": "Acinic cell carcinoma"}, {"label": "D", "disease": "HIV-associated benign lymphoepithelial cyst"}], "answer_idx": "C", "symptoms": ["A white man in his 40s presented with a slowly enlarging, nontender, left-sided neck mass at the angle of the mandible that had been present for 4 years", "Fine-needle aspiration was performed twice, each time with inconclusive results", "Computed tomography (CT) of the neck showed a heterogeneous mass located immediately adjacent to the tail of the left parotid", "No other masses were noted", "He underwent a left superficial parotidectomy with en bloc removal of the mass", "Pathologic findings revealed a 3.0 × 2.9 × 1.9-cm, encapsulated, tan, soft nodule within the specimen", "The nodule had peripheral cystic spaces as well as focal areas of hemorrhage and a 0.5 × 0.4 × 0.4-cm area of calcification within the periphery", "The entire tumor was confined within a periparotid lymph node, almost completely replacing it", "Histopathologic findings (Figure) showed a mixture of dark, granular, basophilic acinar cells and eosinophilic intercalated ductal cells", "The acinar cells contained periodic acid–Schiff–positive, diastase-resistant intracytoplasmic granules, whereas the ductal cells were both fusiform and cuboidal forming small ductlike structures", "The neoplasm also exhibited focal areas of hyaline sclerosis without necrosis"], "s1": [0, 1, 2, 3, 4], "s2": [5, 6, 7, 8, 9, 10]} {"key": 870, "questions": "What Is Your Diagnosis?", "options": [{"label": "A", "disease": "Esthesioneuroblastoma"}, {"label": "B", "disease": "Rhabdomyosarcoma"}, {"label": "C", "disease": "Ethmoid mucocele"}, {"label": "D", "disease": "Sinonasal undifferentiated carcinoma"}], "answer_idx": "B", "symptoms": ["A teenage boy was referred with a progressive 2-week history of blurred vision in his right eye and right ear pain, which had worsened in the past 7 days", "This change prompted a visit to an optometrist, who found a mild decrease in visual acuity and minor cataract in the right eye", "His primary care physician diagnosed his ear pain as acute otitis media and prescribed a course of amoxicillin-clavulanate", "however, the young man’s vision continued to deteriorate and became associated with the onset of right eye swelling and forehead tenderness", "A repeat eye examination by an ophthalmologist was performed and showed normal peripheral visual acuity with a decrease in central vision and confirmed proptosis", "The patient had not experienced nasal congestion, epistaxis, weight loss, fevers, or chills", "Physical examination included right eye proptosis and restricted eye movement", "A computed tomographic (CT) scan with contrast of the face showed a small relatively homogenous mass in the right anterior nasal cavity invading the right orbit, and a magnetic resonance image (MRI) with contrast showed a right superior nasal cavity mass, with invasion of the right orbit and right ethmoidal sinus with extension to the overlying soft-tissue scalp (Figure)", "A 30° endoscope was used to visualize the superior nasal cavity and revealed a fleshy red soft-tissue mass", "Several biopsy specimens were taken and sent for frozen pathologic evaluation", "A, Postintravenous contrast computed tomographic (CT) scan of the face showing a relatively homogenous mass of the right ethmoid complex with invasion of the orbit and extension to the nasal cavity (arrowhead)", "B, Fast spin echo (FSE) T2 magnetic resonance image (MRI) of the right ethmoidal/orbital soft-tissue mass (arrowhead)", "C, Positron emission tomographic (PET) scan of the face with fludeoxyglucose F 18 showing increased uptake within the right ethmoid air cells and nasal cavity, and in the medial right orbit"], "s1": [0, 1, 2, 3, 5, 6], "s2": [4, 7, 8, 9, 10, 11, 12]} {"key": 871, "questions": "What Is Your Diagnosis?", "options": [{"label": "A", "disease": "Thyroid lymphoma"}, {"label": "B", "disease": "Anaplastic thyroid carcinoma"}, {"label": "C", "disease": "Osteosarcoma metastasis"}, {"label": "D", "disease": "Squamous cell carcinoma metastasis"}], "answer_idx": "C", "symptoms": ["A woman in her 30s noticed a painless left neck mass", "She had no history of dysphagia, dyspnea, recent upper respiratory tract infection, injury, or surgery", "She also had no weight loss, pain, or fever and had no history of tobacco or alcohol use, radiation exposure, family history of thyroid cancer, and no previous thyroid dysfunction", "Thyroid function test results were normal", "Physical examination revealed a palpable nontender, firm mass and no cervical lymphadenopathy", "She had a medical history of high-grade osteosarcoma with areas of leimyosarcoma of the tibia which had been treated with right above-knee amputation and chemotherapy 4 years prior to presentation", "She was followed up in the oncology clinic without any recurrence or metastases", "Her last visit was 6 months earlier", "She was also diagnosed with squamous cell carcinoma of the cervix 2 months earlier and was awaiting surgery", "Computed tomography (CT) of the neck revealed a 6.0 × 3.4 × 4.4-cm calcified left thyroid lobe mass (Figure, A)", "Positron emission tomography–computed tomographic (PET-CT) imaging showed increased uptake in the left thyroid gland", "She underwent a left thyroid lobectomy", "The neoplastic cells were positive for p53, vimentin, and smooth muscle actin, and negative for S-100, thyroid transcription factor-1 (TTF-1), and CD34 (Figure, B)", "A, Computed tomographic image reveals a calcified mass (blue arrowhead)", "B", "Left, negative immunostain results for S-100", "right, positive immunostain results for vimentin (original magnification ×100)"], "s1": [0, 1, 2, 3, 4, 5, 6, 7, 8], "s2": [9, 10, 11, 12, 13, 14, 15, 16]} {"key": 872, "questions": "What Is Your Diagnosis?", "options": [{"label": "A", "disease": "Atypical mycobacterial infection"}, {"label": "B", "disease": "Cutaneous malakoplakia"}, {"label": "C", "disease": "Cryptococcosis"}, {"label": "D", "disease": "Calcinosis cutis"}], "answer_idx": "C", "symptoms": ["A white woman in her 60s presented for a routine visit to the outpatient dermatological graft-vs-host disease (GVHD) clinic at Westmead Hospital for review of lichenoid GVHD evolving into sclerodermatous GVHD", "The GVHD occurred secondary to a bone marrow transplant in 2014 for treatment of chronic lymphocytic leukemia", "At the time of review she was being treated with oral azathioprine (50 mg twice daily), tacrolimus (0.5 mg twice daily), and prednisone (25 mg daily) therapy", "She was also receiving monthly intravenous immunoglobulin, prophylactic oral fluconazole, acyclovir, sulfamethoxazole-trimethoprim, and penicillin", "One month prior to commencing oral azathioprine therapy, the patient noted the growth of nodules on the genital and perianal region, with 1 ulcerated nodule in the perianal area associated with painless bleeding on trauma", "The lesions were otherwise asymptomatic", "On examination she had several yellow-colored nodules, some of them ulcerated with extrusion of yellow-colored discharge on the left labia majora and on the perianal area with surrounding inflammatory reaction (Figure 1)", "Additionally, she had sclerodermatous plaques on the lower legs, scarring alopecia on the scalp with minimal hyperkeratosis, nail dystrophy and lichenoid changes, erythematous changes in both feet, and desquamation in the lower legs and on the back and arms suggestive of eczematous or postlichenoid changes as part of her lichenoid GVHD", "Initial investigations revealed a white blood cell count of 13.1 × 103/μL (to convert to billions per liter, multiply by 0.001)", "however, the results of the rest of her blood tests including the full blood count and electrolytes, urea and creatinine, and liver function tests were otherwise stable and unremarkable", "Clinical photographs of cutaneous nodules in a woman with graft-vs-host disease", "A, Multiple erythematous eroded nodules in the perianal area", "B, yellowish nodules located on the left labia majora"], "s1": [0, 1, 2, 3, 7, 8, 9], "s2": [4, 5, 6, 10, 11, 12]} {"key": 873, "questions": "What Is Your Diagnosis?", "options": [{"label": "A", "disease": "Hypertrophic discoid lupus"}, {"label": "B", "disease": "Eruptive keratoacanthomas"}, {"label": "C", "disease": "Pseudoepitheliomatous hyperplasia "}, {"label": "D", "disease": "Sarcoidosis arising in a tattoo"}], "answer_idx": "C", "symptoms": ["A black woman in her 50s presented to the clinic with a 3-month history of nontender mildly pruritic verrucous nodules in several areas of a polychromatic tattoo on her right lower leg and foot", "She reported that these appeared 1 to 2 months following placement of the tattoo and were restricted to areas where the artist created pink zones by layering white ink over red ink", "She denied pain, fevers, and chills and had no history of skin cancer", "She reported a distant history of discoid lupus, and was taking 2 antihypertensives but no other oral or topical medications", "Physical examination revealed well-demarcated erythematous to violaceous verrucous nodules on the right lateral leg and dorsal foot (Figure 1A)", "A punch biopsy specimen from a representative nodule was obtained, with an additional biopsy for tissue culture (Figure 1B)", "A, Verrucous nodules are restricted to zones of pink pigment within the tattoo, sparing other colors present", "B, Histologic reaction pattern from punch biopsy specimen of right lower leg nodule (hematoxylin-eosin, original magnification ×4)"], "s1": [0, 1, 4, 6], "s2": [2, 3, 5, 7]} {"key": 874, "questions": "What Is Your Diagnosis?", "options": [{"label": "A", "disease": "Pemphigus erythematosus"}, {"label": "B", "disease": "Generalized fixed drug eruption"}, {"label": "C", "disease": "Rowell syndrome"}, {"label": "D", "disease": "Toxic epidermal necrolysis"}], "answer_idx": "C", "symptoms": ["A 22-year-old woman with a history of systemic lupus erythematosus was readmitted with fever and a generalized eruption", "Six days prior to this admission, she had been hospitalized for a photo-induced flare of acute cutaneous lupus erythematosus (ACLE)", "At readmission, she was febrile with a temperature of 101.5°F, and on examination, she had a worsening, exquisitely tender rash with several blisters across her right arm", "Physical examination revealed erythematous plaques with superficial desquamation in the malar distribution of the face, nose, ears, and medial forehead (Figure, A), as well as bilateral extensor arms, chest, posterior neck, and upper back", "Several large bullae were located on the right arm (Figure, B)", "Numerous flat and raised 2-zoned atypical targets with dusky centers were appreciated on the forearms, abdomen, back (Figure, C), thighs, legs, palms and soles", "A, Dusky malar erythema with focal heme crusting", "B, Flaccid bullae and target lesions", "C, Diffuse typical and atypical target lesions of the back", "D, High-power view (original magnification 100×) of punch biopsy obtained from left abdomen demonstrating dyskeratosis with superficial perivascular inflammation and interface changes", "Hemogram test results revealed a normocytic anemia (mean corpuscular volume, 99 fL", "hemoglobin, 11.7 g/dL [to convert to g/L, multiply by 10.0]", "hematocrit, 33.4%), in addition to thrombocytopenia (platelet count, 81 K/μL)", "Other pertinent laboratories included hypocomplementemia (C3 of 42 mg/dL", "C4 of 3 mg/dL), a speckled antinuclear antibody titer of 1:1280, and an antidouble-stranded DNA titer of 1:80, along with positive anti-Smith, antiribonucleoprotein, and SSA/anti-Ro antibodies", "Bacterial blood, urine, stool and cerebrospinal fluid cultures, and a hepatitis panel revealed no infectious source", "Punch biopsy of an abdominal atypical target lesion was performed (Figure, D)"], "s1": [0, 1, 2, 3, 4, 5, 6, 7, 8, 9, 16], "s2": [10, 11, 12, 13, 14, 15]} {"key": 875, "questions": "What Is Your Diagnosis?", "options": [{"label": "A", "disease": "Transient ischemic attack"}, {"label": "B", "disease": "Demyelinating disease (multiple sclerosis or acute disseminated encephalomyelitis)"}, {"label": "C", "disease": "Periodic paralysis"}, {"label": "D", "disease": "X-linked Charcot-Marie-Tooth disease 1"}], "answer_idx": "D", "symptoms": ["A 21-year-old man was admitted to the hospital owing to recurrent, transient episodes of dysarthria, dysphagia, and limb weakness over the course of 3 days", "Although these symptoms had resolved after 3 hours’ rest, they recurred 2 hours prior to hospital admission", "In addition, the patient had begun to experience weakness and numbness of his limbs, with more severe symptoms on the right side", "The symptoms began to gradually resolve 2 hours following admission, and normal function had returned within 2 days", "No significant history of illness was noted in the patient or among family members", "Neurological examination on admission revealed normal mental status, and the results of cranial nerve examination were normal", "Although the patient had cavus feet, no atrophy was observed in the distal lower limbs (Figure, A)", "Muscle strength was classified as grade 4 for the left limbs and grade 3 for the right limbs, according to Medical Research Council criteria, and muscle tone was diminished", "The sensory examination was unremarkable", "Deep reflexes were diminished or absent", "However, bilateral positive Babinski signs were observed", "Results from laboratory examinations, including blood electrolytes, cerebrospinal fluid, and lactate, were normal", "Oligonucleotide bands and aquaporin 4 detection were negative", "Craniocervical computed tomographic angiography results were normal", "Cerebral magnetic resonance imaging (MRI) revealed abnormal bilateral increases in T2 signal and diffusion restriction in the periventricular areas and the splenium of corpus callosum (Figure, B)", "Interestingly, a significant reduction of the abnormal signal was noted on MRI 3 months later", "Electrophysiological examination of peripheral nerves revealed extensive bilateral nerve conduction abnormalities", "Significantly reduced motor nerve conduction velocity was also observed in the median nerves (38.3 m per second in the left and 36.5 m per second in the right)", "A, Image shows patient’s pes cavus", "B, Patient’s brain axial diffusion-weighted image at admission", "Demyelinating disease (multiple sclerosis or acute disseminated encephalomyelitis)"], "s1": [0, 1, 2, 3, 6, 7, 9, 10, 16, 17], "s2": [4, 5, 8, 11, 12, 13, 14, 15, 18, 19, 20]} {"key": 876, "questions": "What Is Your Diagnosis?", "options": [{"label": "A", "disease": "Ocular myasthenia"}, {"label": "B", "disease": "Ocular neuromyotonia of the third nerve"}, {"label": "C", "disease": "Giant cell arteritis"}, {"label": "D", "disease": "Cavernous sinus syndrome"}], "answer_idx": "B", "symptoms": ["A 66-year-old woman with a remote history of a pituitary macroadenoma was referred to the neuro-ophthalmology clinic for intermittent diplopia of several years’ duration", "Diplopia was frequently present after repeatedly changing direction of gaze", "She also noted mild right ptosis", "Her visual acuity was unaffected", "There was no associated headache or ocular or periorbital pain", "She had not noticed any injection of the sclera", "Her family and friends made no comments regarding exophthalmos", "She had not noticed any deficits in axial or appendicular strength, rapid fatigability, or dysphagia", "Her symptoms did not exhibit diurnal variation", "Her weight was stable", "She had not noted any changes in her hair, skin, or nails", "She had no subjective disturbance of thermoregulation", "Her relevant history included a pituitary macroadenoma, which had been resected 30 years prior, followed by radiation", "There was no evidence of subsequent recurrence", "There was no family history of ptosis, ophthalmoplegia, or myopathy", "The patient was a retired English teacher", "Examination demonstrated anisocoria (right pupil larger than left), which was accentuated in bright ambient light", "mild right ptosis", "and mildly restricted supraduction, adduction, and infraduction of the right eye (Video, prepared by N", "T", ")", "The second half of the Video (from 24 seconds on) demonstrates marked restriction of elevation, depression, and abduction in the right eye after eccentric gaze", "Ductions in the left eye were normal", "The patient’s neurological examination demonstrated a normal level of consciousness, fluent speech, normal comprehension, good attention, normal cranial nerve function aside from the aforementioned deficits, full strength, normal sensation to proprioception and temperature, symmetric reflexes, absence of ataxia, and a stable, narrow-based gait", "A recent contrast-enhanced brain magnetic resonance image showed a stable intracanalicular vestibular schwannoma on the left, which had been monitored for 8 years"], "s1": [0, 1, 2, 4, 5, 6, 8, 16, 17, 18, 21, 22], "s2": [3, 7, 9, 10, 11, 12, 13, 14, 15, 19, 20, 23, 24]} {"key": 877, "questions": "What Is Your Diagnosis?", "options": [{"label": "A", "disease": "Multiple system atrophy"}, {"label": "B", "disease": "Vascular dementia"}, {"label": "C", "disease": "Fragile X–associated tremor ataxia syndrome"}, {"label": "D", "disease": "Essential tremor"}], "answer_idx": "C", "symptoms": ["A 58-year-old man presented with a 1-year history of severe and progressive action tremor in his dominant hand when writing", "When questioned, he described subtle balance problems without falls developing over the past 3 years", "With the exception of profound difficulties with handwriting, motor symptoms did not affect livelihood or activities of daily living", "There was no tremor reported in the nondominant hand or other limbs or symptoms of autonomic dysfunction", "Neither he nor his wife reported any changes in cognitive function", "His medical history comprised surgeries of the hand (laceration repair), leg (fracture repair), knee (reconstruction), and ear (stapedectomy)", "these were not related to the presenting symptoms", "He had a family history of ischemic heart disease (both parents) but no family history of neurological disorder", "He exercised regularly (5 times per week) and consumed 2 to 4 standard drinks per week", "He was a former smoker with a 2-year pack history and denied any illicit drug use", "Cognitive testing revealed deficits in executive function and speed of information processing", "Examination of eye movements and speech revealed jerky eye pursuit, saccadic dysmetria, and moderate slurring of speech", "Additional motor signs included abnormal handwriting (Figure 1A), bilateral intention tremor, mild rigidity (upper extremities), and moderately impaired heel tapping", "Facial expression was normal", "There was no head or resting tremor", "He demonstrated moderate body sway while standing, was unable to stand on 1 foot for more than 10 seconds, and had impaired tandem walking", "Gait, posture, and walking capacities during a timed 6-m walk were normal", "A, Handwriting task from the motor examination showing abnormal handwriting", "B, T2-weighted fluid-attenuated inversion recovery image from brain magnetic resonance imaging showing white matter hyperintensities in the middle cerebellar peduncles", "Routine blood test results were normal", "Brain magnetic resonance imaging showed moderate volume loss in the cerebral hemispheres bilaterally as well as the cerebellum and brainstem", "Abnormal signal intensity was noted in the cerebral hemispheric white matter bilaterally, the middle cerebellar peduncles (Figure 1B), the cerebellar white matter, and the pons"], "s1": [0, 1, 2, 3, 4, 5, 6, 7, 8, 9], "s2": [10, 11, 12, 13, 14, 15, 16, 17, 18, 19, 20, 21]} {"key": 878, "questions": "What Is Your Diagnosis?", "options": [{"label": "A", "disease": "Teratoma"}, {"label": "B", "disease": "Spinal dysraphism"}, {"label": "C", "disease": "Dermoid cyst"}, {"label": "D", "disease": "Hemangioma"}], "answer_idx": "B", "symptoms": ["An 11-day-old, full-term girl presented for evaluation of multiple skin changes that were noted at birth (Figure)", "These skin changes included a red patch on the central lower back with a soft nodule located on top of the patch", "Additionally, she had a pedunculated red papule positioned on top of the red stain and soft nodule", "She was asymptomatic, moving all limbs, urinating, and defecating normally", "Her prenatal screens, including 20-week ultrasonography, were normal", "Pregnancy was uncomplicated and there was no history of teratogenic exposure or family history of congenital defects", "Ultrasonography of the spine obtained prior to the visit showed no abnormalities", "Photograph depicting a red patch with overlying soft nodule and red pedunculated papule in the lower midback of a neonate"], "s1": [0, 1, 2, 7], "s2": [3, 4, 5, 6]} {"key": 879, "questions": "What Is Your Diagnosis?", "options": [{"label": "A", "disease": "Middle ear paraganglioma"}, {"label": "B", "disease": "Facial nerve hemangioma"}, {"label": "C", "disease": "Facial nerve schwannoma"}, {"label": "D", "disease": "Middle ear adenoma"}], "answer_idx": "A", "symptoms": ["An elderly woman presented with 2 episodes of vertigo lasting a few minutes while stationary over 1 day and a several-year history of right-sided hearing loss", "She had not experienced otalgia, otorrhea, or tinnitus", "Physical examination revealed a mass obstructing the right external auditory canal and House-Brackmann grade 2 facial nerve weakness on the right", "Audiogram revealed right-sided moderate to profound mixed hearing loss", "Computed tomography revealed a soft-tissue mass emanating from the middle ear involving the facial canal with surrounding bony erosion, including dehiscence of the tegmen, cochlea, bony labyrinth, lateral semicircular canal, and jugular foramen", "The patient underwent transtemporal mastoidectomy and debulking", "However, owing to the mass’s vascularity and significant bleeding, the operation was aborted, and she was sent for embolization (Figure, A)", "Magnetic resonance imaging during embolization revealed intense enhancement in the right middle ear extending from the epitympanum to the hypotympanum with extension into the right external auditory canal (Figure, B)", "She returned to the operating room, and the vascular mass was found to extend inferiorly to the jugular bulb and carotid and anteriorly into the eustachian tube", "The mass was adherent to the underlying bone and had invaded the facial canal but not the nerve itself", "Both malleus and incus were eroded", "Immunohistochemical stains of the specimen indicated cells positive for synaptophysin with a sustentacular pattern of cells positive for S-100 (Figure, C and D)", "A, Mastoidectomy, visualization of vascular mass", "B, T1-weighted axial magnetic resonance image (MRI) showing enhancement of right middle ear extending to external auditory canal", "C, Hematoxylin-eosin, original magnification ×40. Clusters of cells surrounded by vascular tissue", "D, Synaptophysin stain, original magnification ×40. Nests of strongly staining neuroendocrine cells separated by vasculature"], "s1": [0, 1, 2, 3, 8, 9, 10], "s2": [4, 5, 6, 7, 11, 12, 13, 14, 15]} {"key": 880, "questions": "What Is Your Diagnosis?", "options": [{"label": "A", "disease": "Intraductal papilloma"}, {"label": "B", "disease": "Mucoepidermoid carcinoma"}, {"label": "C", "disease": "Mucocele"}, {"label": "D", "disease": "Oral inverted ductal papilloma"}], "answer_idx": "D", "symptoms": ["A man in his 50s was referred to the department of oral and maxillofacial surgery for evaluation and treatment of an intraoral cystic lesion of the left lower lip", "This lesion had developed spontaneously 3 months prior to consultation without an identifiable inciting event", "He could recall no trauma and described a slow but progressive increase in the volume of this otherwise asymptomatic lesion", "His medical history revealed a long-term history of substance abuse with active alcohol and tobacco consumption, chronic obstructive pulmonary disease, and epilepsy (tonic-clonic seizures, badly controlled owing to a lack of adherence to therapy)", "Clinical intraoral examination of this fully edentulous patient revealed a nontender, mobile, cystic nodule, approximately 1 × 1 cm on the vestibular side of the lower left lip with outflow of a clear serous fluid on palpation (Figure, A)", "The lesion was excised surgically and the specimen sent for additional histological examination", "After hematoxylin-eosin staining, light microscopy revealed an endophytic growing lesion, which consisted of an acanthotic epidermoid epithelium organized in broad papillary folds, with at its periphery an infiltrating border", "At the surface of these papillae, the epithelium was sometimes columnar", "Mucocytes were present, individually dispersed throughout the epithelium, but also organized in small tubuloacinar structures emptying onto the surface", "The epithelium was infiltrated by neutrophils", "Atypia or mitotic figures were absent, and the subepithelial stroma contained normal-appearing seromucous acini (Figure, B and C)", "A, Clinical presentation of the cystic lesion of the lower left lip", "B and C, Histopathologic images, hematoxylin-eosin", "The papillary fronds lining the interior of the lesion", "The lesion is well demarcated from the subepithelial stroma, and has an infiltrating border", "C, Detail at the base of one of the papillae", "The cells at the surface of the epithelial folds are columnar", "Mucocytes organized in acinar structures empty on the surface epithelium but are also individually dispersed between the epidermoid cells"], "s1": [0, 1, 2, 3, 4, 5, 11], "s2": [6, 7, 8, 9, 10, 12, 13, 14, 15, 16, 17]} {"key": 881, "questions": "What Is the Diagnosis?", "options": [{"label": "A", "disease": "Nasopharyngeal carcinoma"}, {"label": "B", "disease": "Juvenile nasopharyngeal angiofibroma"}, {"label": "C", "disease": "Rhabdomyosarcoma"}, {"label": "D", "disease": "Inverted papilloma"}], "answer_idx": "B", "symptoms": ["A previously healthy young man in his late teens presented to his dentist with a 3-week history of left-sided facial pain, fullness, and trismus", "He underwent a third maxillary molar extraction and attempted incision and drainage of the area with no improvement in symptoms", "No purulence or clinically significant bleeding was identified", "A contrast-enhanced computed tomographic (CT) scan was performed (Figure, A), and the patient was referred to the otolaryngology clinic", "On presentation, he also reported decreased acuity in his left lower visual field, continued swelling with increasing pain, and worsening trismus", "Physical examination revealed mild tenderness over a mass that extended inferiorly to the zygoma into the buccal cavity and posterolaterally to the left superior molars", "The patient also reported hypoesthesia of the left maxillary division of the trigeminal nerve", "Fiber-optic scope of the nasopharynx revealed a small lesion posterior to the left middle turbinate", "Contrast-enhanced magnetic resonance imaging of his head, neck, and brain was performed (Figure, B and C)", "A, Computed tomographic (CT) image shows a uniformly hyperdense mass extending into the left infratemporal fossa with slight extension of the mass through the medial aspect of the posterior wall of the maxillary sinus and widening of the sphenopalatine foramen", "B, Contrast-enhanced magnetic resonance image (MRI) with fat saturation shows a mass with mild punctuate areas of hypointense signal (suggestive of vascularity) terminating in the left buccinator space with extension into the nasal cavity, maxillary sinus and inferior orbital fissure", "C, A heterogenous mass in the left retromaxillary region and infratemporal fossa with hyperintense signal and mild salt-and-pepper appearance consistent with vascularity", "D, Gross specimen is in similar orientation as mass seen in panel B"], "s1": [0, 1, 2, 3, 4, 5, 6, 7], "s2": [8, 9, 10, 11, 12]} {"key": 882, "questions": "What Is Your Diagnosis?", "options": [{"label": "A", "disease": "Venous malformation"}, {"label": "B", "disease": "Lymphatic malformation"}, {"label": "C", "disease": "Arteriovenous malformation"}, {"label": "D", "disease": "Chronic sialadenitis"}], "answer_idx": "A", "symptoms": ["A 45-year-old woman initially presented for evaluation of a 6-month history of intermittent left facial swelling and pain", "She noted a discretely palpable mass in the left parotid region during these episodes", "However, at the time of clinic presentation, she had no palpable masses on physical examination", "Ultrasonography performed on the day of initial evaluation revealed no intraparotid masses, lesions, or stones", "The patient was encouraged to return if she noticed acute return of swelling so that she could be evaluated when symptomatic", "The patient returned days later with acutely worsening left facial swelling, which she noticed while straining", "On presentation, she was diagnosed as having malignant hypertension and had a palpable parotid lesion noted on physical examination", "When asked to perform a Valsalva maneuver, she was noted to have more pronounced left facial swelling (Figure, A)", "A short tau inversion recovery (STIR) image is shown in the Figure, B", "Magnetic resonance angiogram was obtained (Figure, C and D)", "A, The patient performed a Valsalva maneuver, with increased left parotid gland and neck swelling", "B, Coronal T2-weighted short tau inversion recovery (STIR) image demonstrating a T2 hyperintense lesion measuring 3.7 × 3.4 × 7.4 cm involving the left parotid gland and extending to the left side of the neck", "C and D, T1-weighted, fat-suppressed, gadolinium-enhanced angiography", "C, Arterial section demonstrating no evidence of significant high flow component", "D, Delayed phase demonstrating a homogenous contrast enhancing lesion"], "s1": [0, 1, 2, 4, 5, 6, 7], "s2": [3, 8, 9, 10, 11, 12, 13, 14]} {"key": 883, "questions": "What Is Your Diagnosis?", "options": [{"label": "A", "disease": "Diaphragmatic eventration with lung collapse"}, {"label": "B", "disease": "Pneumothorax with elevation of the left hemidiaphragm"}, {"label": "C", "disease": "Diaphragmatic rupture and a mediastinal hematoma"}, {"label": "D", "disease": "Diaphragmatic rupture and a traumatic rupture of the descending thoracic aorta"}], "answer_idx": "D", "symptoms": ["A 24-year-old man was brought to the emergency department after a high-speed motor vehicle collision", "He was a restrained driver noted to be awake at the scene of the collision and had to be extracted from the vehicle", "He was hemodynamically stable on route to the hospital", "In the emergency department, the patient was awake but restless and combative and was subsequently intubated", "He was hypotensive on arrival but responded transiently to fluid resuscitation", "Multiple closed long bone fractures of the upper and lower extremities were noted", "The chest radiography (Figure 1A) showed marked elevation of the left hemidiaphragm", "The pelvic radiography revealed an unstable fracture involving bilateral pubic rami and the left sacroiliac joint", "The pelvis was splinted in the emergency department", "He remained transiently responsive to fluid resuscitation and was taken for emergent computed tomography", "Computed tomography of the head and cervical spine showed no evidence of injury", "A computed tomographic scan of the chest (Figure 1B) and abdomen was then performed, and the patient was taken to the operating room", "A, single-view supine chest radiography", "B, Axial computed tomographic image of the chest", "Diaphragmatic rupture and a traumatic rupture of the descending thoracic aorta"], "s1": [0, 1, 2, 3, 4, 5, 7, 8, 9, 10], "s2": [6, 11, 12, 13, 14]} {"key": 884, "questions": "What Is Your Diagnosis?", "options": [{"label": "A", "disease": "Sigmoid adenocarcinoma"}, {"label": "B", "disease": "Rectosigmoid polyp"}, {"label": "C", "disease": "Sigmoid endometriosis"}, {"label": "D", "disease": "Rectosigmoid leiomyoma"}], "answer_idx": "C", "symptoms": ["A 45-year-old woman with factor 1 deficiency presented to the general surgery clinic with a history of multiple episodes of lower gastrointestinal bleeding", "These episodes started approximately 4 years prior, occur approximately every 3 to 4 months, and are associated with crampy abdominal pain", "Although she has been hospitalized, she never required a transfusion", "Colonoscopies were significant only for diverticulosis without a clear source of bleeding", "Her previous abdominal surgery included a hysterectomy for menorrhagia", "There is no family history of colon cancer", "Physical examination showed no abnormalities", "She had computed tomography of the abdomen and pelvis that showed eccentric and irregular masslike thickening of the sigmoid colon of approximately 7 cm with a wall thickness of 1.4 cm (Figure 1)", "Repeat colonoscopy showed an exquisitely painful nodule at the rectosigmoid junction", "There were no stigmata of recent bleed, and biopsy showed normal colonic mucosa", "Axial (A) and coronal (B) computed tomography of the pelvis with rectal contrast", "The images show thickening and narrowing of the sigmoid colon (arrowheads)"], "s1": [0, 1, 2, 3, 5, 6], "s2": [4, 7, 8, 9, 10, 11]} {"key": 885, "questions": "What Is Your Diagnosis?", "options": [{"label": "A", "disease": "Brain metastasis"}, {"label": "B", "disease": "Immune-related hypophysitis"}, {"label": "C", "disease": "Pituitary adenoma"}, {"label": "D", "disease": "Mucocele"}], "answer_idx": "B", "symptoms": ["A 56-year-old woman with metastatic melanoma receiving ipilimumab and nivolumab every 3 weeks developed headaches during the course of her treatment", "In the week following her first treatment, the patient reported a few mild morning headaches that improved with acetaminophen", "After receiving 3 cycles of ipilimumab, 3 mg/kg, and nivolumab, 1 mg/kg, and 8 weeks after her first treatment, the patient reported experiencing daily headaches for at least the prior week and a half", "The location of her headaches varied, and there was no associated eye or temple pain", "At best, with taking nonsteroidal anti-inflammatory drugs, the headache decreased to pain rated 3 on a scale of 1 to 10. Also at this time, the patient reported low energy levels and difficulty reading", "however, physical examination showed no gross visual field defects", "Laboratory workup conducted 8 weeks after initiating treatment included adrenocorticotropic hormone (ACTH) and cortisol levels that were measured at 25 pg/mL (reference range, 10-60 pg/mL) and 16.8 μg/dL (reference range, 6-24 μg/dL), respectively", "Thyroid-stimulating hormone (TSH) level was 0.82 mIU/L (reference range, 0.5-5.0 mIU/L), which was gradually decreasing compared with 1.41 mIU/L 5 weeks earlier and 1.21 mIU/L 2 weeks earlier", "Her total triiodothyronine level was measured at 73 ng/dL (reference range, 80-200 ng/dL) and free thyroxine level was 1.0 ng/dL (reference range, 0.9-1.7 ng/dL)", "Her serum sodium level was measured at 138 mEq/L", "1 week later it was measured at 135 mEq/L (reference range, 135-145 mEq/L)", "Contrast-enhanced magnetic resonance imaging (MRI) of the brain was performed for further assessment (Figure)", "Magnetic resonance imaging (MRI) of the brain obtained 8 weeks after initiating ipilimumab (A-C) and nivolumab treatment and at follow-up (D)"], "s1": [0, 1, 2, 3, 4, 5, 11, 12], "s2": [6, 7, 8, 9, 10]} {"key": 886, "questions": "What Is Your Diagnosis?", "options": [{"label": "A", "disease": "Elastolytic granuloma"}, {"label": "B", "disease": "Knee-located erythema elevatum diutinum (EED) with a calcaneal late-stage nodular component"}, {"label": "C", "disease": "Epithelioid sarcoma"}, {"label": "D", "disease": "Storiform collagenoma"}], "answer_idx": "B", "symptoms": ["A healthy man in his 20s presented with a several-month history of asymptomatic, slightly erythematous cutaneous nodules, measuring 1.5 to 3.0 cm, over the calcanea (Figure, A)", "Physical examination revealed violaceous plaques, measuring 2 to 3 cm, on the extensor surface of the knees, bilaterally (Figure, B)", "Excisional biopsy specimens were obtained (Figure, C and D)", "A, Multiple asymptomatic erythematous nodules measuring 1.5 to 3.0 cm at presentation", "B, Multiple violaceous plaques measuring 2 to 3 cm on the extensor surface of the knees", "C, Excisional biopsy specimen of a calcaneal nodule (hematoxylin-eosin, original magnification ×20)", "D, Excisional biopsy specimen of a plaque (hematoxylin-eosin, original magnification ×20)", "Knee-located erythema elevatum diutinum (EED) with a calcaneal late-stage nodular component"], "s1": [0, 3, 7], "s2": [1, 4, 2, 5, 6]} {"key": 887, "questions": "What Is Your Diagnosis?", "options": [{"label": "A", "disease": "Kaposi varicelliform eruption"}, {"label": "B", "disease": "Impetigo"}, {"label": "C", "disease": "Pemphigoid gestationis"}, {"label": "D", "disease": "Impetigo herpetiformis"}], "answer_idx": "A", "symptoms": ["A primigravida woman in her 30s at 35 weeks’ gestation and with a history of Darier disease was admitted with an acute, painful eruption on her face and neck", "She reported no history of pregnancy-related complications", "Physical examination revealed confluent erythema with keratotic papulovesicles distributed on the face and neck (Figure, A)", "Honey-colored crusts were present on the left ear and preauricular area", "The chest and inframammary area displayed occasional red, crusted papules", "Laboratory tests revealed an elevated white blood cell count of 17.18 × 103/μL (reference range, 4.50-11.00 × 103/μL)", "A shave biopsy specimen and tissue culture were obtained from the center of an umbilicated papule on the neck (Figure, B and C)", "A, Confluent erythema with papulovesicles on the neck and face", "B and C, Histopathological images, hematoxylin-eosin", "B, Full-thickness epidermal necrosis with focal areas of suprabasal acantholysis with corp ronds, corp grains, and parakeratosis", "C, Multinucleated cells with nuclear molding and margination"], "s1": [0, 1, 2, 3, 4], "s2": [5, 6, 7, 8, 9, 10]} {"key": 888, "questions": "What Is Your Diagnosis?", "options": [{"label": "A", "disease": "Rheumatoid arthritis"}, {"label": "B", "disease": "Systematic lupus erythematosus"}, {"label": "C", "disease": "Arthritis associated with other connective tissue diseases"}, {"label": "D", "disease": "Leprosy"}], "answer_idx": "D", "symptoms": ["A man in his 40s presented with gradual hair loss of the eyelashes, eyebrows, beard, and scalp for 4 years, swelling and painful hand joints for 13 months, and multiple asymptomatic nodules on the upper arms for 40 days", "The patient exhibited no associated fever, analgesia or hypesthesia", "He reported no previous infections or family or personal medical history of autoimmune diseases or psoriasis", "He was previously treated for rheumatoid arthritis, but various arthritis treatments showed no improvements over 1 year", "A physical examination showed diffuse infiltration of the face with loss of skin creases and diminished facial hair", "all joints of both hands were swollen (Figure, A)", "multiple asymptomatic nodules without ulceration were present on his upper arms (Figure, B)", "decreased sensation to pain, temperature and/or touch and neuropathic changes were not found", "Laboratory test results showed a slightly elevated erythrocyte sedimentation rate (27 mm/h) and significantly elevated rheumatoid factor (70.1 IU/mL)", "Additional immunology indicators, including C-reactive protein, anticyclic citrullinated peptide, antinuclear antibodies, and globulin levels were normal", "X-ray and joint ultrasound results suggested osteoarthritis, synovitis, flexor tendon tenosynovitis, and bone erosions", "Ultrasound results also indicated enlarged axillary, cervical, and inguinal lymph nodes and an enlarged spleen", "Fundus fluorescein angiography showed uveitis", "A, Joints of both hands were swollen", "B, Multiple asymptomatic nodules on the upper arms", "C, Skin biopsy from the patient’s upper arm nodule demonstrating diffuse infiltrate of foamy histiocytes in the dermis (hematoxylin-eosin stain, original magnification ×100)", "D, Numerous acid-fast bacilli (acid-fast stain, original magnification ×400)"], "s1": [0, 1, 2, 3, 4, 5, 7, 8, 9, 10, 11, 12], "s2": [6, 13, 14, 15, 16]} {"key": 889, "questions": "What Is Your Diagnosis?", "options": [{"label": "A", "disease": "Hypervitaminosis A"}, {"label": "B", "disease": "Postural orthostatic tachycardia syndrome"}, {"label": "C", "disease": "Primary adrenal insufficiency (Addison disease)"}, {"label": "D", "disease": "Anorexia nervosa"}], "answer_idx": "C", "symptoms": ["A white, nonobese, 15-year-old girl presented with 1 month of frontal and occipital throbbing headaches, tinnitus, and “whooshing” in the ears", "She had 1 week of nonbilious, nonbloody emesis, and “graying out” of her vision only when standing up or sitting down", "Examination noted normal visual acuities and swelling of both optic discs (Figure, A)", "Magnetic resonance imaging and magnetic resonance venography of the head showed no intracranial mass or venous sinus thrombosis", "Lumbar puncture showed an opening pressure of 550 mm of water and normal cerebrospinal fluid (CSF) composition", "A diagnosis of idiopathic intracranial hypertension was made, and she was treated with acetazolamide", "Headaches initially improved but worsened during the following weeks", "Three months after initial presentation, she reported a 9.07-kg weight loss (height 1.57 m", "body mass index dropped from 21.3 to 16.8 [calculated as weight in kilograms divided by height in meters squared]), postural lightheadedness, worsening fatigue, muscle weakness, salt cravings, loss of axillary hair, and missed periods for 2 months", "Physical examination was remarkable for ill appearance, blood pressure of 85/42 mm Hg, and hyperpigmentation of the knuckles, distal fingers, and tongue (Figure, B and C)", "Repeated ophthalmic examination showed normal visual acuities and worsening optic disc swelling", "The patient presented with papilledema bilaterally (A, right eye shown) patchy tongue darkening (B), and darkening of knuckles and distal fingers (C)"], "s1": [0, 1, 2, 3, 4, 5, 6, 10, 11], "s2": [7, 8, 9]} {"key": 890, "questions": "What Is Your Diagnosis?", "options": [{"label": "A", "disease": "Atypical carcinoid"}, {"label": "B", "disease": "Malignant carotid body tumor"}, {"label": "C", "disease": "Medullary thyroid carcinoma"}, {"label": "D", "disease": "Hemangiopericytoma"}], "answer_idx": "B", "symptoms": ["A man in his late 40s with a 30-pack-year history of smoking and a history of heavy drinking presented following 7 months of right-sided neck tenderness", "He also reported progressive dysphagia to solids but no dyspnea, otalgia, or weight loss", "Examination revealed very tender right-sided neck fullness of 6 cm", "Laryngoscopic findings were unremarkable", "Computed tomography (CT) with contrast demonstrated a 4.4 × 3.6 × 4.5-cm heterogeneously enhancing mass in the right side of the neck, splaying and completely surrounding both the internal and external carotid arteries (Figure, A)", "Scattered lymph nodes were also observed", "On magnetic resonance imaging (MRI), the mass was hyperintense on T2 (Figure, B)", "A second 0.9-cm enhancing mass was visualized at the inferior right jugular foramen", "Chest radiography demonstrated no suspicious pulmonary or mediastinal lesions", "He underwent excision of the mass with right-sided neck dissection", "Cranial nerves XI, X, and XII were identified and preserved", "During subadventitial dissection of tumor off the carotid bifurcation, a laceration of the internal carotid artery was noted", "Vascular surgery was performed to make an intraoperative repair, and an intraoperative angiogram demonstrated no dissections or thrombi", "The lesion was suspected to be benign, and the procedure was aborted", "The patient experienced no neurologic sequelae postoperatively", "A specimen was taken of the right-sided neck mass (Figure, C and D)", "A and B, Imaging of the carotid space", "A, Computed tomographic (CT) image", "B, Postcontrast T1 magnetic resonance image (MRI)", "C and D, Histopathologic images", "C, Hematoxylin-eosin", "D, Immunohistochemical staining"], "s1": [0, 1, 2, 3, 4, 5, 6, 7, 8, 9, 10, 11, 12, 13, 14, 15], "s2": [16, 17, 18, 19, 20, 21]} {"key": 891, "questions": "What Is Your Diagnosis?", "options": [{"label": "A", "disease": "Solid pseudopapillary neoplasia of the pancreas"}, {"label": "B", "disease": "Atypical mucinous cystic neoplasia of the pancreas"}, {"label": "C", "disease": "Gastrointestinal stromal tumor"}, {"label": "D", "disease": "Pancreatic pseudocyst"}], "answer_idx": "C", "symptoms": ["A primigravid woman in her 20s who had been pregnant for 16 weeks presented with significant abdominal pain", "A noncontrast computed tomographic (CT) scan revealed a 15 × 13 × 11-cm heterogeneous cystic mass that appeared to originate from the body of the pancreas and protrude into the gastric wall (Figure 1, A)", "Results of CT scan also revealed a 3-cm hypodense lesion on the left lobe of the liver, ascites, and splenomegaly", "Further evaluation with endoscopic retrograde cholangiopancreatography and endoscopic ultrasonography revealed extrinsic compression of the gastric wall from a large mixed solid/cystic lesion with septations and well-defined margins with no clear involvement of the gastric wall", "A thin, reddish-brown fluid (700 mL) was aspirated from the cystic component", "Analysis of the aspirate showed no tumor cells or mucinous material, and normal carcinoembryonic antigen, CA19-9, and amylase", "Fine-needle aspiration was performed on the solid component of the cystic tumor (Figure 1, B-E)", "A, Coronal view CT scan showing a 15 × 13 × 11-cm complex heterogeneous mass originating from the body of the pancreas (white arrowhead) and protruding into the gastric wall", "A hypodense lesion in segment 3 of the liver is observed (black arrowhead) and also moderate ascites (yellow arrowhead)", "B, Hematoxylin-eosin stain (original magnification ×20)", "C, α-smooth muscle actin immunostain (original magnification ×40)", "D, CD117 immunostain (original magnification ×40)", "and D, CD34 immunostain (original magnification ×40)"], "s1": [0], "s2": [1, 2, 3, 4, 5, 6, 7, 8, 9, 10, 11, 12]} {"key": 892, "questions": "What Is Your Diagnosis?", "options": [{"label": "A", "disease": "Acquired epidermolysis bullosa"}, {"label": "B", "disease": "Pseudoporphyria"}, {"label": "C", "disease": "Photoallergic drug reaction"}, {"label": "D", "disease": "Graft-vs-host disease"}], "answer_idx": "B", "symptoms": ["A man in his 60s presented with new-onset bullae and erosions over sun-exposed areas and bony prominences", "His medical history was relevant for acute myeloid leukemia, for which he had received an allogeneic stem cell transplant that was complicated by biopsy-proven grade 3 cutaneous and grade 2 gastrointestinal graft-vs-host disease", "As part of his posttransplant infectious prophylaxis, the patient had been prescribed valacyclovir hydrochloride, voriconazole, and trimethoprim/sulfamethoxazole", "He was also taking tacrolimus hydrate and prednisone for treatment of his graft-vs-host disease", "The patient reported fishing outdoors on a sunny day a few days prior to the onset of these skin lesions", "On physical examination, the patient’s skin was diffusely tan and there were flaccid bullae and erosions on the bilateral dorsal hands, elbows, and knees (Figure, A)", "A punch biopsy specimen from a bulla from the upper extremity was obtained (Figure, B)", "A, Flaccid bulla and erosions on the left dorsal hand at presentation", "B, Biopsy from the left elbow"], "s1": [0, 4, 5, 7], "s2": [1, 2, 3, 6, 8]} {"key": 893, "questions": "What Is Your Diagnosis?", "options": [{"label": "A", "disease": "Disseminated granuloma annulare"}, {"label": "B", "disease": "Erythrodermic sarcoidosis"}, {"label": "C", "disease": "Mycosis fungoides"}, {"label": "D", "disease": "Tuberculid"}], "answer_idx": "B", "symptoms": ["An African American male in his 50s with a longstanding history of atopic dermatitis presented with a 20- to 30-year medical history of occasionally pruritic erythematous papules involving the trunk and extremities", "He denied associated fevers, chills, night sweats, weight loss, joint pain, cough, difficulty breathing, or chest pain", "Findings from the physical examination revealed red-brown monomorphic papules coalescing into plaques on the face, trunk, and extremities (Figure, A and B)", "There were thin erythematous plaques with overlying scale on bilateral extensor upper and lower extremities", "A 4-mm punch biopsy specimen from his right upper extremity was obtained and stained with hematoxylin-eosin for histopathologic analysis", "Periodic acid–Schiff and acid-fast bacilli stains were also performed", "A, Distribution of papules over the chest, abdomen, and upper extremities", "B, Diffuse distribution of papules over the back with some papules coalescing into plaques", "C, Hematoxylin-eosin stain (original magnification, ×10)"], "s1": [1, 4, 5, 8], "s2": [0, 2, 3, 6, 7]} {"key": 894, "questions": "What Is Your Diagnosis?", "options": [{"label": "A", "disease": "Botryomycosis"}, {"label": "B", "disease": "Cutaneous tuberculosis"}, {"label": "C", "disease": "Digital mucous cyst"}, {"label": "D", "disease": "Foreign-body reaction"}], "answer_idx": "A", "symptoms": ["A man in his 50s with diagnoses of human immunodeficiency virus (HIV) infection, hepatitis B, and latent tuberculosis presented with a slow-growing nodule on the left foot", "The lesion started 1 year prior as a tender red nodule on the medial aspect of his first metatarsophalangeal joint, which would enlarge and become painful approximately once a month (Figure, A)", "The patient reported that with manipulation, a clear-to-yellow material drained from the lesion", "The patient denied any purulent drainage, additional skin lesions, fever, chills, or other systemic symptoms", "His medications included efavirenz–emtricitabine–tenofovir disoproxil fumarate for his HIV, with the most recent absolute CD4-positive cell count of 341 cells/mm3 (reference range, 263-2045 cells/mm3), and recent completion of an 8-month course of isoniazid treatment after being exposed to tuberculosis", "Physical examination demonstrated an erythematous papulonodule without warmth or drainage on the dorsal left great toe", "No lymphadenopathy was appreciated", "A punch biopsy was performed (Figure, B, C, and D) to help establish the diagnosis", "A, Clinical photograph of slow-growing red papulonodule on the left toe dorsal foot", "B, Biopsy with hematoxylin-eosin (H&E) staining demonstrating acral skin with acute and chronic inflammation within the deep dermis", "C, Higher magnification highlights eosinophilic material surrounding small cocci", "D, Gram staining of the biopsy supported the diagnosis"], "s1": [0, 1, 2, 3, 4, 5, 6], "s2": [7, 8, 9, 10, 11]} {"key": 895, "questions": "What Is Your Diagnosis?", "options": [{"label": "A", "disease": "Hyponatremia"}, {"label": "B", "disease": "Diabetes mellitus"}, {"label": "C", "disease": "Hypothyroidism"}, {"label": "D", "disease": "Fatty liver disease"}], "answer_idx": "C", "symptoms": ["A 13-year-old girl presented to an emergency department with bilateral hip pain for several months and acute worsening with left-sided limp following a school field trip to a large museum", "She stated that her pain worsened with walking and was minimally relieved with salicylic acid", "Her parents tried to schedule an outpatient appointment for evaluation of her hip pain but had difficulty owing to a lack of a primary care physician", "Review of systems was significant for heavy periods since menarche 6 months ago", "It was negative for easy bruising or bleeding", "The patient described normal stooling and voiding habits", "The patient was described as a shy eighth grader who earns A’s and B’s in school", "She was not physically active", "On physical examination, the patient was noted to be short for her age (1.3 m [4.4 ft]", "height in 25 pg/mL [to convert to picomoles per liter, multiply by 2.6]), and a normal phosphorous level of 4.5 mg/dL (to convert to millimoles per liter, multiply by 0.323)", "Oral and intravenous supplementations of calcium were started, but the patient’s calcium levels remained refractory, even after correction of other electrolytes, including potassium and magnesium", "He was placed on a continuous intravenous calcium infusion for 3 days with eventual normalization of his calcium level"], "s1": [0, 1, 2, 3, 4, 5], "s2": [6, 7, 8, 9, 10, 11, 12, 13, 14]} {"key": 1018, "questions": "What Is Your Diagnosis?", "options": [{"label": "A", "disease": "Acute viral pericarditis"}, {"label": "B", "disease": "Acute myocardial infarction"}, {"label": "C", "disease": "Acute aortic dissection"}, {"label": "D", "disease": "Pericardial tumor"}], "answer_idx": "C", "symptoms": ["A man in his early 70s walked into the emergency department with a 4-day history of fever and tingling sensation in the chest", "His medical history was significant with hypertension, intracranial hemorrhage, and chronic kidney disease (CKD)", "On arrival, his blood pressure was 130/64 mm Hg, heart rate was 104 beats/min, respiratory rate was 32 breaths/min, body temperature was 37.1°C, and arterial oxygen saturation was 95% on room air", "There was no murmur, rub, or gallop in chest auscultation", "Electrocardiographic (ECG) findings were significant for diffuse ST-segment elevation (Figure 1, A)", "Laboratory examination revealed leukocytosis with normal cardiac enzyme levels and mild renal dysfunction (serum creatinine, 1.4 mg/dL [to convert to micromoles per liter, multiply 88.4])", "The C-reactive protein level was also elevated", "He was admitted to the hospital with the suspected diagnosis of acute pericarditis", "After admission, his chest discomfort resolved in 2 days, but he was still febrile, and pericardial effusion did not decrease", "Computed tomography (CT) of the chest was performed (Figure 1, B and C)"], "s1": [0, 4, 5, 6, 7, 8], "s2": [1, 2, 3, 9]} {"key": 1019, "questions": "What Is Your Diagnosis?", "options": [{"label": "A", "disease": "Gorlin syndrome"}, {"label": "B", "disease": "Turcot syndrome"}, {"label": "C", "disease": "Gardner syndrome"}, {"label": "D", "disease": "Turner syndrome"}], "answer_idx": "C", "symptoms": ["A young man without antecedent episodes of rhinosinusitis or sinus surgery presented with progressive headache and left periorbital edema and pain recalcitrant to 4 days of outpatient oral antibiotics and 2 days of intravenous vancomycin and ceftriaxone", "His family history was positive for a similar presentation involving his mother", "Physical examination revealed left orbital swelling and erythema with proptosis", "Cranial nerve function and ocular range of motion were intact", "Contrast-enhanced computed tomographic (Figure, A) and magnetic resonance imaging studies of the paranasal sinuses (Figure, B) disclosed extensive preseptal and postseptal orbital inflammation, bilateral frontal sinus opacification from a large left frontal osteoma occupying the inferior aspect of the left frontal sinus, and left epidural abscess", "The patient underwent a bicoronal approach osteoplastic flap and endoscopic frontal sinusotomy for access and drainage of left frontal sinus disease and obstructive osteoma excision", "In addition to frontal sinus osteoma, multiple 3- to 5-mm osteomas covering the calvarium were encountered (Figure, C)", "The patient’s postoperative period progressed without incident, and he was discharged home with oral antibiotics following resolution of symptoms", "Histopathologic findings of operative specimens showed dense lamellar bone (Figure, D)", "Genetic analysis was positive for mutation in the APC gene at 5q21, resulting in a stop codon", "A, Contrast-enhanced computed tomographic (CT) in bone windows", "B, T1-weighted magnetic resonance image (MRI)", "C, Reflected osteoplastic flap, frontal sinusotomy", "D, Hematoxylin-eosin, original magnification ×400."], "s1": [0, 1, 2, 3, 4, 5, 6, 7], "s2": [8, 9, 10, 11, 12, 13]} {"key": 1020, "questions": "What Is Your Diagnosis?", "options": [{"label": "A", "disease": "Laryngeal squamous cell carcinoma"}, {"label": "B", "disease": "Laryngeal neurofibroma"}, {"label": "C", "disease": "Laryngeal schwannoma"}, {"label": "D", "disease": "Laryngeal paraganglioma"}], "answer_idx": "C", "symptoms": ["A man in his 50s was referred to the otolaryngology clinic because of a globus sensation in the throat and a 6-month history of throat pain", "His medical history was significant for hypertension, dyslipidemia, and cigarette smoking", "Flexible fiber-optic laryngoscopy revealed a 15-mm submucosal swelling in the right aryepiglottic fold", "The vocal folds were free of lesions and had intact motion", "Findings from the rest of the head and neck examination were unremarkable", "A computed tomographic (CT) scan of the neck revealed a submucosal hypodense supraglottic mass on the right side that was compressing the airway", "A magnetic resonance imaging study (MRI) of the neck demonstrated a soft-tissue mass measuring 27 × 17 mm and causing deviation of the right aryepiglottic fold medially and compressing the airway (Figure, A and B)", "The patient underwent direct laryngoscopy, and a submucosal mass was observed in the right aryepiglottic fold (Figure, C)", "The mass was dissected submucosally, excised completely, and sent for a histopathologic examination (Figure, D)", "Results from the S100 protein immunostain were positive", "The postoperative course was uneventful, and after 5 months, the patient had no detectable laryngeal lesions and the vocal fold movement was intact"], "s1": [0, 1, 3, 4, 10], "s2": [2, 5, 6, 7, 8, 9]} {"key": 1021, "questions": "What Is Your Diagnosis?", "options": [{"label": "A", "disease": "Endobronchial sarcoidosis"}, {"label": "B", "disease": "Granulomatosis with polyangiitis"}, {"label": "C", "disease": "Primary laryngotracheobronchial amyloidosis"}, {"label": "D", "disease": "Tracheobronchial osteochondropathia"}], "answer_idx": "C", "symptoms": ["A woman in her 40s had an 8-year history of progressive dyspnea associated with nonproductive cough", "Her medical history was notable for dyspepsia, hypertension, and diabetes", "She had no history of smoking, type B symptoms, angiotensin-converting enzyme-inhibitor use, or known cardiac pathologic abnormalities", "Findings from a complete head and neck examination were within normal limits aside from laryngeal findings", "Flexible laryngoscopy with videostroboscopy revealed right vocal fold polyp and left vocal fold desquamation, with bilateral linear bands along the vocal folds", "Initial treatment with proton pump inhibitors and evaluation by the gastroenterology unit did not improve her symptoms, so further workup was undertaken", "Noncontrast computed tomography (CT) revealed numerous calcified subglottic and tracheal nodules extending from the cricoid to the left mainstem bronchus (Figure, A)", "She underwent direct laryngoscopy and bronchoscopy with biopsies", "On induction, subglottic firm mass consistent with calcification prevented passage of an endotracheal tube, and the patient’s airway was secured with a rigid bronchoscope", "Further examination revealed edema of the vocal folds with multiple submucosal masses in the subglottis and upper trachea (Figure, B-D)", "The lesions were varying sizes, and not all were calcified", "They affected the anterior cartilaginous trachea and the posterior membranous portion", "Biopsy specimens of representative lesions revealed amorphous, eosinophilic material with calcification, metaplastic ossification, and multinucleated foreign-body giant cells", "The disease was treated with debulking of scattered lesions from the glottis through to both main stem bronchi and airway dilation, which provided symptomatic relief", "A, Calcified subglottic, upper tracheal, and left mainstem bronchus lesions", "B, Waxy, friable deposits in the supraglottic airway", "C, Nodular lesions scattered circumferentially in the trachea", "D, Nodular lesions extending into bronchi and lower airways"], "s1": [0, 1, 2, 3, 4, 5], "s2": [6, 7, 8, 9, 10, 11, 12, 13, 14, 15, 16, 17]} {"key": 1022, "questions": "What Is Your Diagnosis?", "options": [{"label": "A", "disease": "Bacterial folliculitis"}, {"label": "B", "disease": "Herpes infection"}, {"label": "C", "disease": "Pityrosporum folliculitis"}, {"label": "D", "disease": "Reaction to radiotherapy"}], "answer_idx": "D", "symptoms": ["A 72-year-old woman was treated with a total postoperative radiotherapy dose of 55.86 Gy (to convert to rad, divide by 0.01) in 21 fractions after lumpectomy of a cT1bN0M0 mammary carcinoma", "Two months after the last dose of radiation she developed pruritic pustules and papules on the skin of the ipsilateral right breast", "She mentioned that she had a cold sore once several years ago", "Levoceterizine gave some relief of the itch", "Physical examination findings showed pustules surrounding the areola on the right breast with an increasing number in the submammary fold (Figure 1)"], "s1": [0, 2], "s2": [1, 3, 4]} {"key": 1023, "questions": "What Is Your Diagnosis?", "options": [{"label": "A", "disease": "Squamous cell carcinoma"}, {"label": "B", "disease": "Myeloid sarcoma"}, {"label": "C", "disease": "Invasive aspergilloma"}, {"label": "D", "disease": "Condyloma acuminatum"}], "answer_idx": "B", "symptoms": ["A man in his 70s with hypertension, benign prostatic hypertrophy, and myelodysplastic syndrome (MDS) presented for a second opinion regarding treatment of MDS", "Three years prior he had developed neutropenia and mild thrombocytopenia", "A bone marrow biopsy sample demonstrated refractory cytopenia with multilineage dysplasia", "Cytogenetic testing results were normal, and there were 5% bone marrow blasts", "The patient was treated initially with cyanocobalamin 250 micrograms daily, then danazol 200 milligrams 3 times daily, then epoietin α 60,000 units weekly", "Filgrastim 300 micrograms weekly was added to epoietin α to attempt to augment response after 1 year of therapy", "After approximately 2 years, he developed more pronounced cytopenias and presented with a white blood cell count of 2100/μL(480 neutrophils), hemoglobin level of 8.4 g/dL, and platelet count of 92 000/μL", "He noted fatigue, but otherwise felt well", "Physical examination revealed an isolated, symptomatic polypoid mass involving his right upper labial mucosa (Figure)", "It had been recently biopsied without definitive diagnosis, but since the biopsy, the mass had increased in size", "An isolated polypoid mass affecting the right upper labial mucosa"], "s1": [0, 1, 2, 3, 4, 5, 6, 7], "s2": [8, 9, 10]} {"key": 1024, "questions": "What Is Your Diagnosis?", "options": [{"label": "A", "disease": "Argyria"}, {"label": "B", "disease": "Acute intermittent porphyria"}, {"label": "C", "disease": "Diffuse melanosis in metastatic melanoma"}, {"label": "D", "disease": "Alkaptonuria"}], "answer_idx": "D", "symptoms": ["A 65-year-old woman presented with a 5-year history of gradually increasing blue discoloration of her ears", "She had mild knee arthralgia that was treated with nonsteroidal anti-inflammatory drugs (NSAIDs)", "She was seeing an ophthalmologist annually regarding brown spots on both eyes diagnosed as conjunctival nevi", "No other family members were similarly affected", "As an infant, her mother had taken her to their physician owing to dark discoloration of her diapers, but this was not investigated further", "Examination revealed slate blue macular pigmentation within the antihelix and conchal bowls of both ears (Figure, A)", "There were prominent dark brown deposits on the conjunctivae medial and lateral to the iris of both eyes (Figure, B)", "No other pigmentary changes were noted on the skin or mucosal surfaces", "A punch biopsy specimen from the pigmented area on the left ear was obtained (Figure, C)", "A, Examination of the left ear revealing slate blue macular discoloration, most prominent over the antihelix", "B, Examination of the left eye showing dark brown pigment deposition of the conjunctiva lateral to the iris", "Similar deposits were seen medial to the iris, bilaterally in both eyes", "C, Hematoxylin-eosin stain on punch biopsy specimen of pigmented area on left ear demonstrating large pigment deposit in the dermis and adjacent to the cartilage (original magnification ×150)"], "s1": [0, 1, 2, 3, 4, 7], "s2": [5, 6, 8, 9, 10, 11, 12]} {"key": 1025, "questions": "What Is Your Diagnosis?", "options": [{"label": "A", "disease": "Neonatal candidiasis"}, {"label": "B", "disease": "Erythema toxicum neonatorum"}, {"label": "C", "disease": "Congenital dermatophytosis"}, {"label": "D", "disease": "Congenital cutaneous candidiasis"}], "answer_idx": "D", "symptoms": ["A male neonate born prematurely at 27 weeks’ gestational age was noted to have a erythematous eruption at birth", "The neonate was the product of a dichorionic, diamniotic twin pregnancy", "The other twin did not have any dermatalogic abnormalities", "They were born to a previously healthy woman who had been hospitalized 12 days prior for tocolytic therapy", "One day prior to delivery, the mother was noted to have yeast on urinalysis", "The presence of epithelial cells in the sample suggested vaginal contamination", "Despite tocolytic therapy, labor ultimately progressed the following day and cesarean delivery was performed owing to breech presentation of the unaffected twin", "Physical examination of the affected twin revealed an afebrile, 1200 g preterm infant", "Skin examination showed 100 to 200 pink papules, 0.05 to 0.2 cm in size, distributed on the trunk, extensor upper arms, and upper legs (Figure, A and B)", "A subset of the lesions appeared vesiculopustular, but no fluid was evident after attempted unroofing", "The diaper area, hands, feet, nails, and oral mucosa were unaffected", "The second twin had normal cutaneous examination findings", "Laboratory analysis revealed a white blood cell (WBC) count of 6000/μL, which was composed of 21% neutrophils, 38% lymphocytes, 24% monocytes, and 5% eosinophils", "Gram stain of lesional scrapings did not reveal bacteria nor WBCs, though potassium hydroxide staining showed numerous yeast forms", "Lesional polymerase chain reaction was negative for herpes simplex virus", "Bacterial cultures from the skin and the blood were ultimately negative", "The placentas from both neonates were sent for pathologic analysis (Figure, C and D)", "A, Pink papules and vesicopustules were noted on the trunk, arms, and extensor thighs, but spared the diaper area", "B, morphology of lesions", "C and D, histopathologic images of umbilical cord section"], "s1": [0, 1, 2, 3, 4, 5, 6, 16, 19], "s2": [7, 8, 9, 10, 11, 12, 13, 14, 15, 17, 18]} {"key": 1026, "questions": "What Is Your Diagnosis?", "options": [{"label": "A", "disease": "Galli-Galli disease"}, {"label": "B", "disease": "Grover disease"}, {"label": "C", "disease": "Classic Dowling-Degos disease"}, {"label": "D", "disease": "Kitamura disease"}], "answer_idx": "A", "symptoms": ["A 50-year-old woman presented with a 20-year history of slowly progressive red-to-brown pruritic hyperkeratotic papules and reticulated hyperpigmentation involving her face, neck, chest, back, and upper and lower extremities (Figure, A and B)", "She was previously diagnosed with Darier disease after biopsy revealed acantholytic dyskeratosis", "She was treated with fluorouracil and tazarotene, 0.1%, cream with mild improvement, but she continued to develop new lesions", "Social, family, and past medical history were noncontributory", "Physical examination revealed reddish-brown hyperkeratotic papules on the neck, trunk, and extremities and diffuse brown macules, many in a reticulated pattern, on the face, neck, trunk, and extremities", "There were no nail or mucosal changes", "Laboratory data was noncontributory", "A punch biopsy specimen was collected from both the arm and trunk and histologic examination was performed (Figure, C and D)", "Photographs of clinical findings of reticulate hyperpigmentation of the (A) neck and chest and (B) lower extremities", "Hematoxylin-eosin of (C) hyperorthokeratosis, epidermal acanthosis and elongated epidermal rete ridges with bud-like filiform projections and suprapapillary plate thinning (original magnification ×4) and (D) focal acantholysis without dyskeratosis (original magnification ×10)"], "s1": [0, 1, 2, 4, 8], "s2": [3, 5, 6, 7, 9]} {"key": 1027, "questions": "What Is Your Diagnosis?", "options": [{"label": "A", "disease": "Carotid artery dissection"}, {"label": "B", "disease": "Carotid atherosclerosis"}, {"label": "C", "disease": "Fibromuscular dysplasia of the carotid artery"}, {"label": "D", "disease": "Carotid aneurysm"}], "answer_idx": "B", "symptoms": ["A 45-year-old woman with no medical history presented with intermittent episodes of right hand and leg discoordination and paresthesia of increasing frequency 2 weeks prior to presentation", "She denied any antecedent trauma or neck manipulation", "She had no known history of renal dysfunction or poorly controlled hypertension and was a lifelong nonsmoker", "She was seen in the emergency department and underwent brain magnetic resonance imaging, which demonstrated patchy enhancement in the left basal ganglia and corona radiata concerning for demyelinating lesions", "She was admitted and underwent further workup", "Pertinent laboratory results included C-reactive protein level of 0.01 mg/L (to convert to nanomoles per liter, multiply by 9.524), erythrocyte sedimentation rate of 6 mm/h (to convert to millimeters per hour, multiply by 1), and normal cholesterol panel", "She underwent an extensive imaging workup consisting of bilateral carotid duplex, computed tomographic neck angiography (Figure 1A) and cerebral arteriogram (Figure 1B)", "A, Computed tomographic arteriogram of the neck demonstrating near occlusion of the proximal left internal carotid artery (arrowhead)", "B, Cerebral arteriogram demonstrating a string sign in the proximal left internal carotid artery (arrowhead)"], "s1": [0, 1, 2, 4, 5], "s2": [3, 6, 7, 8]} {"key": 1028, "questions": "What Is Your Diagnosis", "options": [{"label": "A", "disease": "Herpes simplex virus"}, {"label": "B", "disease": "Pseudomonas aeruginosa"}, {"label": "C", "disease": "Streptococcus pyogenes"}, {"label": "D", "disease": "Neisseria meningitidis"}], "answer_idx": "B", "symptoms": ["A 13-month-old girl was hospitalized with acute exudative tonsillitis", "The product of a full-term normal delivery, she had had occasional mouth ulcers and recurrent tonsillitis treated with oral antibiotics by her family physician", "Her vaccinations were all up to date including the measles, mumps, and rubella vaccine given 2 days before hospitalization, the day of fever onset", "Admission examination revealed severe exudative pharyngitis and oral candidiasis", "Laboratory parameters included a hemoglobin level of 11.4 g/dL (to convert to grams per liter, multiply by 10), white blood cell count of 5600/μL (to convert to × 109 perliter, multiply by 0.001), absolute lymphocyte count of 5600/μL (to convert to × 109 perliter, multiply by 0.001), absolute neutrophil count of 300/μL (to convert to × 109 perliter, multiply by 0.001), platelet count of 327 × 103/μ (to convert to × 109 per liter, multiply by 1), and C-reactive protein level of 183 mg/L (normal range, <10mg/L", "to convert to micromoles per liter, multiply by 9.524)", "Intravenous benzylpenicillin was initiated and was followed by defervescence and improvement in her condition", "Admission blood and throat culture results were negative", "Two days later, at approximately 8:00 am, her mother noticed 2 purple spots in the diaper area", "By 4:00 pm, the lesions were described as firm, nontender purpuric nodules", "She remained afebrile and clinically stable", "By 9:00 pm, the lesions had progressed, with the emergence of multiple discrete ulcers involving the labial, perineal, and gluteal areas", "She rapidly deteriorated with onset of septic shock, her white blood cell count fell to 0/μL, and her C-reactive protein level increased to 404 mg/L", "Following stabilization and initiation of broad-spectrum antibiotics, she was transferred to a pediatric intensive care unit where Figure 1 was obtained", "Multiple ulcers with an erythematous rim and notable absence of pus"], "s1": [0, 1, 2, 3, 4, 5, 6, 7], "s2": [8, 9, 10, 11, 12, 13, 14]} {"key": 1029, "questions": "What Is Your Diagnosis?", "options": [{"label": "A", "disease": "Paget disease"}, {"label": "B", "disease": "Fibrous dysplasia"}, {"label": "C", "disease": "Exostosis"}, {"label": "D", "disease": "Otosclerosis"}], "answer_idx": "B", "symptoms": ["A man in his 40s presented with a 2-year history of gradual-onset, left-sided hearing loss", "It was associated with a whistling tinnitus and occasional sharp otalgia lasting for a few minutes, approximately twice a week, without otorrhea", "He reported labyrinthitis 2 years previously with possible reduced hearing afterward", "He took no regular medications, had no allergies or clinically significant family history, and was an occasional smoker and alcohol drinker", "Examination showed a smooth mass behind the left tympanic membrane", "Findings from flexible nasoendoscopy were unremarkable", "There was no palpable lymphadenopathy", "Pure-tone audiometry demonstrated a mixed hearing loss on the left side", "Results from tympanometry of the left ear were normal", "A computed tomographic scan showed an expansile ground-glass density lesion arising within the lateral aspect of the otic capsule on the left (Figure, A and B)", "The lesion eroded into the lateral aspect of the basal and second turns of the cochlea as well as the tympanic portion of the facial nerve canal", "Magnetic resonance imaging demonstrated a 7 × 8-mm lesion arising from the otic capsule, with mild uniform enhancement following gadolinium", "A technetium bone scan showed a lesion in the temporal bone, and a focus of increased uptake in the left lateral eighth rib (no fracture was demonstrated) (Figure, C)", "To obtain a diagnosis, a bony sample was obtained, which consisted of small fragments of woven bony tissue with mild osteocyte nuclei enlargement and hyperchromasia, with no sign of neoplasia", "A and B, Computed tomographic (CT) images showing left otic capsule lesion", "C, Single photon emission tomography/CT showing uptake of tracer in the left temporal bone and left eighth rib"], "s1": [0, 1, 2, 4, 7, 8], "s2": [3, 5, 6, 9, 10, 11, 12, 13, 14, 15]} {"key": 1030, "questions": "What Is Your Diagnosis?", "options": [{"label": "A", "disease": "Angiosarcoma"}, {"label": "B", "disease": "Multinodular goiter with hemorrhagic cyst"}, {"label": "C", "disease": "Anaplastic (undifferentiated) thyroid carcinoma"}, {"label": "D", "disease": "Hemangioma"}], "answer_idx": "A", "symptoms": ["A woman in her 80s with a previous diagnosis of a multinodular goiter was referred for exertional dyspnea and chest discomfort", "Her medical history was significant for gastroesophageal reflux disease, hypertension, stable angina, and obstructive sleep apnea for which she was using continuous positive airway pressure therapy at night", "A repeated computed tomographic (CT) scan, when compared with a study 3 years prior, indicated clinically significant growth", "A positron emission tomographic/CT scan was significant for increased metabolic activity in the mass", "Surgical intervention was recommended for alleviation of symptoms and definitive diagnosis", "A left thyroidectomy with partial right thyroidectomy (Figure, A) and a left modified neck dissection (levels II-IV) were performed", "Gross examination revealed an encapsulated, 11.0 × 8.4 × 8.2-cm, tan-orange, predominantly hemorrhagic, necrotic nodule without grossly identifiable thyroid parenchyma (Figure, B)", "Extensive sampling revealed pleomorphic epithelioid cells with enlarged, hyperchromatic nuclei, irregular nuclear membranes, and increased mitotic activity (Figure, C)", "There was evidence of angiogenesis and formation of nests around areas with organizing hemorrhage", "Tumor cells showed strong membrane reactivity for CD31, confirming an endothelial origin of the lesion (Figure, D)", "Adjacent thyroid parenchyma showed multinodular goiter and chronic lymphocytic thyroiditis", "The neck dissection revealed normal lymphatic contents", "A, Intraoperative view of the mass", "B, Central hemorrhage and necrosis", "C, Enlarged, hyperchromatic nuclei and mitoses", "D, Neoplastic cells showing immunoreactivity to CD31."], "s1": [0, 1, 2, 3, 4, 5, 10, 11], "s2": [6, 7, 8, 9, 12, 13, 14, 15]} {"key": 1031, "questions": "What Is Your Diagnosis?", "options": [{"label": "A", "disease": "Laryngeal squamous cell carcinoma"}, {"label": "B", "disease": "Laryngeal actinomycosis"}, {"label": "C", "disease": "Laryngeal candidiasis"}, {"label": "D", "disease": "Laryngeal amyloidosis"}], "answer_idx": "B", "symptoms": ["A woman in her 90s with no significant medical history presented to the otolaryngology clinic with a chief concern of worsening hoarseness over several years", "She had not experienced shortness of breath, gastric reflux, smoking, cough, dysphagia, odynophagia, fevers, or chills", "Findings from a physical examination were remarkable for a rough voice", "Flexible nasolaryngoscopy revealed normal abduction and adduction of the vocal folds with a white sessile lesion starting at the right midfold on the superior and medial surfaces that crossed the midline to involve the entire left vocal fold as shown in the Figure, A", "Histopathologic images of the biopsied lesion are shown in the Figure, B-D", "Laryngeal lesion", "A, Gross appearance of vocal folds by flexible nasolaryngoscopy", "B, Hematoxylin-eosin, original magnification ×400. C, Gram stain, original magnification ×400. D, Gomori methenamine silver stain, original magnification  × 400."], "s1": [0, 1, 2, 3, 5], "s2": [4, 6, 7]} {"key": 1032, "questions": "What Is Your Diagnosis?", "options": [{"label": "A", "disease": "Thymoma"}, {"label": "B", "disease": "Teratoma"}, {"label": "C", "disease": "Lymphoma"}, {"label": "D", "disease": "Substernal goiter"}], "answer_idx": "D", "symptoms": ["An asymptomatic 47-year-old woman underwent a radiographic screening to rule out tuberculosis prior to volunteering at her local hospital", "The incidental finding of a mediastinal mass prompted a referral to her primary care physician, who elicited from the patient some complaints of mild but worsening orthopnea that was exacerbated when she would lie down", "She denied any recent weight loss, muscle weakness, or diplopia", "Her surgical history was significant for having undergone a cesarean section", "She denied any radiation exposure", "A computed tomographic scan of the chest was obtained (Figure, A), and the patient was referred to surgery for evaluation", "A, Computed tomographic scan of the chest revealing a mediastinal mass", "B, Intraoperative dissection following median sternotomy", "A physical examination identified a midline trachea, no significant thyromegaly in the neck, no significant lymphadenopathy, and no additional significant findings", "The patient was taken to the operating room for surgery to remove the mass (with gross findings as seen in Figure, B)"], "s1": [0, 2, 3, 4, 8], "s2": [1, 5, 6, 7, 9]} {"key": 1033, "questions": "What Is Your Diagnosis?", "options": [{"label": "A", "disease": "Metastatic non–small-cell lung cancer "}, {"label": "B", "disease": "Rib fracture"}, {"label": "C", "disease": "Radiation-induced myositis"}, {"label": "D", "disease": "Polymyalgia rheumatica"}], "answer_idx": "C", "symptoms": ["A man in his 70s with a history of rheumatoid arthritis and a 50-pack-year smoking history presented with a worsening unproductive cough", "Chest radiography identified a right lung nodule, with complete workup revealing a T2aN0M0 right upper lobe non–small-cell lung cancer (NSCLC) measuring 4.3 × 2.9 cm", "A clinically significant cardiac history precluded the option of surgery", "He was treated with a definitive accelerated hypofractionated course of radiotherapy to a total of dose 60 Gy in 8 fractions", "Acute toxic effects were not observed, with the exception of minor fatigue that resolved 1 month after treatment", "A surveillance computed tomographic (CT) scan obtained at 3 months demonstrated that the right upper lobe tumor had decreased by 1 cm in all dimensions, and at this time the patient denied any new symptoms, including pain", "Approximately 5 months after treatment, he awakened with abrupt onset of severe right shoulder pain", "The pain was sharp, constant, and localized to his right scapula and ultimately progressed to the point of radiating down the extensor surface of his arm ending at his elbow", "He denied paresthesias, weakness, or any inciting events that could have led to its onset", "His primary care physician obtained a CT scan, followed 2 days later by a magnetic resonance image (MRI) of his right shoulder (Figure 1)", "Right shoulder diagnostic imaging 5 months after treatment", "CT indicates computed tomography", "MRI, magnetic resonance imaging"], "s1": [0, 1, 2, 3, 4, 5], "s2": [6, 7, 8, 9, 10, 11, 12]} {"key": 1034, "questions": "What Is Your Diagnosis?", "options": [{"label": "A", "disease": "Chondrodermatitis nodularis helicis chronicus"}, {"label": "B", "disease": "Relapsing polychondritis"}, {"label": "C", "disease": "Petrified ear"}, {"label": "D", "disease": "Scleroderma"}], "answer_idx": "C", "symptoms": ["A man in his 70s presented for evaluation of a nonhealing lesion of his left cheek", "Examination of the head and neck revealed the incidental finding of bilateral auricular rigidity and immobility", "Both ears appeared normal and were nontender and asymptomatic (Figure, A)", "The patient recalled that his barber had noted an increase in difficulty trimming hair in the areas around his auricles owing to their inflexibility", "The only other notable finding was increased rigidity of his nasal cartilage", "The patient denied any decrease in auditory acuity, frostbite, or any previous trauma to his ears", "His medical history included type 2 diabetes mellitus, hypertension and secondary chronic renal insufficiency, cardiovascular disease, and gout", "His long-term medications included lisinopril, glipizide, terazosin, insulin glargine, labetalol, allopurinol, hydrochlorothiazide, and erythropoietin", "An incisional biopsy of his left auricle was performed", "Hematoxylin-eosin staining of the biopsy specimen showed regions of ossification and calcification (Figure, B)", "A computed tomographic (CT) scan of the head was performed within 2 weeks of his original biopsy and showed extensive calcification in the area of the auricles (Figure, C)", "The patient’s laboratory workup showed an elevated creatinine level of 1.8 mg/dL (reference range, 0.6-1.2 mg/dL), which had been his baseline level for the past 2 years", "He had a hemoglobin level of 11.1 g/dL (reference range, 14-18 g/dL), which was felt to be secondary to his chronic renal failure", "His serum calcium level had been normal with the exception of an elevated value noted of 10.4 mg/dL noted 1 year ago (reference range, 8.7-10.2 mg/dL)", "His phosphorus, alkaline phosphorus, electrolyte, and AM cortisol levels", "parathyroid function", "and thyroid function were within normal limits", "A, Photograph of ear", "B, Photomicrograph of biopsy sample", "C, Head computed tomographic image"], "s1": [0, 1, 3, 4, 5, 8, 9, 10, 17, 18, 19], "s2": [2, 6, 7, 11, 12, 13, 14, 15, 16]} {"key": 1035, "questions": "What Is Your Diagnosis?", "options": [{"label": "A", "disease": "Erythema nodosum leprosum"}, {"label": "B", "disease": "Lucio phenomenon"}, {"label": "C", "disease": "Cutaneous small vessel vasculitis"}, {"label": "D", "disease": "Cutaneous polyarteritis nodosa"}], "answer_idx": "B", "symptoms": ["A man in his 30s had ulceration of the skin of the soles, legs, hands, elbow joints, and face of 15 days’ duration", "It had started as swelling of the feet, followed by erythematous acral lesions", "Numbness of the hands and feet had begun 1½ years previously, spreading gradually to the forearm and legs", "He had no history of fever, joint pains, or epistaxis", "Black eschar and ulceration with purulent discharge was seen over the soles and elbows", "There were triangular and bizarre purpuric and necrotic lesions over the legs, thighs, and dorsum of the hands (Figure, A) with necrosis of pinnae and nasolabial folds", "The scrotum was also affected", "There was diffuse infiltration of the face and madarosis of the lateral half of the eyebrows", "He also had hypoesthesia over the palms, proximal forearms, feet, and legs", "Peripheral nerve examination revealed thickening of the right radial cutaneous and both ulnar nerves", "Bilateral inguinal lymphadenopathy was present", "Findings from the rest of the general physical and systemic examination were normal", "Punch biopsy specimens were obtained (Figure, B-D)", "A, Multiple bizarrely shaped purpuric and necrotic plaques on the hands and lower limbs", "Erosion is seen on the scrotum", "B-D, Histopathologic images"], "s1": [0, 1, 2, 3, 7, 8, 9, 10], "s2": [4, 5, 6, 11, 12, 13, 14, 15]} {"key": 1036, "questions": "What Is Your Diagnosis?", "options": [{"label": "A", "disease": "Erythema elevatum diutinum"}, {"label": "B", "disease": "Pyoderma gangrenosum"}, {"label": "C", "disease": "Multicentric reticulohistiocytosis"}, {"label": "D", "disease": "Sweet syndrome"}], "answer_idx": "A", "symptoms": ["A woman in her 60s presented with a 6-month history of itchy eruptions that initially started on the arms and then spread to involve the lower legs", "The patient admitted to occasional burning sensation of new lesions", "On review of systems, the patient denied fevers, fatigue, myalgias, arthritis, shortness of breath, oral ulcers, and recent infections", "Clinical examination revealed several erythematous patches and plaques on the arms and lower legs, and nontender violaceous firm nodules on the elbows, wrist, ankle, interphalangeal, and metatarsophalangeal joints (Figure 1A and B)", "There was no lymphadenopathy present", "A punch biopsy specimen from a plaque on the right forearm was obtained (Figure 1C and D)"], "s1": [0, 1, 2], "s2": [3, 4, 5]} {"key": 1037, "questions": "What Is Your Diagnosis?", "options": [{"label": "A", "disease": "Primary central nervous system lymphoma–leptomeningeal variant"}, {"label": "B", "disease": "Carcinomatous meningitis–primary adenocarcinoma of the lung"}, {"label": "C", "disease": "Tubercular meningitis"}, {"label": "D", "disease": "Neurosarcoidosis"}], "answer_idx": "A", "symptoms": ["A 69-year-old white man presented to the emergency department reporting 2 months of progressive bilateral lower extremity weakness, left facial weakness, and diplopia in the setting of a 20.4-kg weight loss over the previous 6 months and new-onset urinary incontinence", "Two months prior he had undergone lumbar vertebra 3–sacra vertebra 1 lumbar laminectomies performed for symptomatic spinal stenosis", "Despite this, his leg weakness worsened", "He denied headaches, fevers, cough, risk factors for immunosuppression, or travel outside of the northwestern United States", "His medical history was notable for hypertension and a 50-packs-per-year smoking history", "Physical examination was notable for a thin, frail-appearing man with normal vital signs and intact mentation", "Cranial nerve examination revealed visual acuity of 20/400 OS, with an afferent pupillary defect and left-sided facial weakness including eye closure", "He demonstrated profoundly decreased bulk and hypotonia throughout all extremities, with fasciculations apparent across his chest and lower extremities", "Lower extremity muscle strength was graded 2/5 throughout with absent reflexes and intact sensation", "Initial laboratories revealed a normal complete blood cell count and chest radiography", "Magnetic resonance imaging of the neuroaxis revealed leptomeningeal enhancement around the cerebral convexities, cerebellum, and spinal cord including the cauda equina (Figure 1)", "Cerebrospinal fluid (CSF) studies revealed a white blood cell count of 29/μL (to convert to × 109 per liter, multiply by .001) with a near-complete lymphocytic predominance, a red blood cell count of 363 × 106/μL (to convert to × 1012 per liter, multiply by 1), a protein level of 0.142 g/dL (to convert to grams per liter, multiply by 10), and glucose level of 19 mg/dL (to convert to millimoles per liter, multiply by 0.0555)", "Cerebrospinal fluid cytology analysis did not identify malignant cells, with negative assay results for tuberculosis and fungal infections", "Repeated flow cytometry did not identify abnormal B-cell or T-cell population", "Whole-body positron emission tomography–computed tomography showed extensive leptomeningeal and nerve root hypermetabolism without evidence of a primary malignancy", "Sagittal T1-weighted MRI of the lumbar spine without (A) and with (B) contrast showing leptomeningeal enhancement of the lumbar spinal cord, cauda equina, and multilevel postsurgical changes"], "s1": [0, 1, 2, 3, 4, 5, 8], "s2": [6, 7, 9, 10, 11, 12, 13, 14, 15]} {"key": 1038, "questions": "What Is Your Diagnosis?", "options": [{"label": "A", "disease": "Opiate-induced anamnestic recall of prior deficits"}, {"label": "B", "disease": "Focal seizures with postictal hemiparesis"}, {"label": "C", "disease": "Ischemic stroke"}, {"label": "D", "disease": "Acute demyelinating disorder"}], "answer_idx": "C", "symptoms": ["Four days following resection of a scrotal lipoma, a 15-year-old boy with trisomy 21 presented to the emergency department with somnolence, pain, and paucity of speech with right hand and face weakness", "The scrotal lipoma resection had been performed under general anesthesia without complications", "He was somnolent immediately following the procedure, and had not spoken since the day after the procedure", "He was taking acetaminophen and 10 mg of oxycodone every 4 hours", "Right-sided hand and face weakness developed 2 days after the procedure", "His medical history was significant for trisomy 21, with mild cognitive delay and baseline dysarthria, but no aphasia", "He had been self-sufficient with daily activities", "In the emergency department, his examination was significant for a temperature of 38.1°C and significant scrotal swelling and pain", "The neurologic examination was remarkable for somnolence, right hemineglect, right facial droop with right hand weakness, right patellar hyperreflexia, and mutism, although he attempted to use sign language", "His laboratory values showed a white blood cell count of 6.8 × 103/μL (to convert to  × 109/L, multiply by 0.001), platelet count of 248×103/µL (to convert to × 109/L, multiply by 1), hematocrit level of 30.9% (to convert to proportion of 1.0, multiply by 0.01), hemoglobin level of 10.2 g/dL (to convert to grams per liter, multiply by 1) with a normal differential, activated partial thromboplastin time of 40 seconds, fibrinogen level of 678 mg/dL (to convert to grams per liter, divide by 100), and normal urinalysis results", "He received naloxone, after which his alertness improved, he could say his name, and he could follow commands well", "The right face and hand weakness persisted, and he was unable to walk independently", "Magnetic resonance imaging of the brain was performed (Figure 1)"], "s1": [0, 2, 4, 5, 8, 10, 11], "s2": [1, 3, 6, 7, 9, 12]} {"key": 1039, "questions": "What Is Your Diagnosis?", "options": [{"label": "A", "disease": "Low-grade mucoepidermoid carcinoma"}, {"label": "B", "disease": "Intraductal papilloma"}, {"label": "C", "disease": "Warthin tumor"}, {"label": "D", "disease": "Acinic cell carcinoma"}], "answer_idx": "B", "symptoms": ["A woman in her 50s presented with a 4-week history of a slowly enlarging mass on the left side of the neck", "Physical examination revealed a mobile, nontender, 4-cm mass in the left side of the floor of mouth", "A contrast-enhanced computed tomographic scan of the neck showed a 3.8-cm cystic neck mass in the left floor of mouth (Figure, A)", "An ultrasonography (US)-guided fine-needle aspiration (FNA) biopsy showed bland-appearing cuboidal to columnar epithelial cells arranged in clusters", "no malignant cells were identified", "Repeated US-guided FNA biopsy showed a hypercellular specimen composed of numerous fragments of bland-appearing cuboidal and columnar epithelium and granular debris, which was eosinophilic in cell block sections and papillary structures with fibrovascular cores", "The hypercellularity was atypical, and a low-grade neoplasm could not be ruled out", "At this point, the patient elected observation of the mass and a repeated biopsy", "A core biopsy showed a well-differentiated papillary neoplasm with hyalinized fibrovascular cores and some degenerative changes (Figure, B-D)", "Immunophenotypically, the neoplastic cells were negative for the markers TTF-1, PAX-8, estrogen receptor, and thyroglobulin", "These results, with the morphologic characteristics, were most consistent with a low-grade salivary neoplasm", "She underwent excision of the cystic floor of mouth mass and a level I neck dissection", "Pathologic examination revealed a cystic, 2.5-cm encapsulated mass arising from a minor salivary gland", "A, Computed tomographic image of the neck showing a mass in the left floor of mouth", "B-D, Histopathologic images"], "s1": [0, 1, 11, 12, 13], "s2": [2, 3, 4, 5, 6, 7, 8, 9, 10, 14]} {"key": 1040, "questions": "What Is Your Diagnosis?", "options": [{"label": "A", "disease": "Complications of otitis media"}, {"label": "B", "disease": "Langerhans cell histiocytosis"}, {"label": "C", "disease": "Rhabdomyosarcoma"}, {"label": "D", "disease": "Squamous cell carcinoma"}], "answer_idx": "A", "symptoms": ["A 5-year-old girl with a history of autism presented to an outside emergency department with a 10-day history of otitis media and recent fevers", "She was eventually transferred to our hospital after a computerized tomographic (CT) scan was performed showing potential clinically significant ear abnormalities", "On evaluation, the patient had a bulging, hyperemic left tympanic membrane, along with fullness of the postauricular, upper cervical region", "Facial nerve function was intact", "No focal neurologic deficits or meningeal signs were observed", "A CT scan of the temporal bones with contrast was performed at our facility (Figure)", "A-C, Axial computed tomographic (CT) scans with contrast"], "s1": [0, 1, 2], "s2": [3, 4, 5, 6]} {"key": 1041, "questions": "What Is Your Diagnosis?", "options": [{"label": "A", "disease": "Endolymphatic sac tumor"}, {"label": "B", "disease": "Choroid plexus papilloma"}, {"label": "C", "disease": "Papillary meningioma"}, {"label": "D", "disease": "Middle ear adenoma"}], "answer_idx": "A", "symptoms": ["An elderly man was evaluated for long-standing discharge and hearing loss in the left ear and mild dizziness", "Biopsy of the soft tissue at the external ear canal (EEC) showed granulation tissue", "He did not have facial weakness", "He was obese and had congestive heart failure and hypertension", "Findings from the otolaryngologic examination were otherwise unremarkable", "Advanced-stage mixed conductive and sensorineural hearing loss in the left ear were noted", "Computed tomography (CT) showed a soft-tissue mass in the mastoid and middle ear cavity extending to the EEC with no significant erosion of ossicles (Figure, A)", "During modified radical mastoidectomy, the mastoid cavity was found to be full of edematous, fragile soft tissue with mucoid secretions", "The incus was intact but dislocated", "Histopathological examination revealed a polypoid lesion showing typical papillary architecture with central fibrovascular cores and dilated glandular structures, lined by a bland-looking single layer of high columnar/cuboidal epithelial cells (Figure, B)", "No mitotic activity or necrosis was detected", "The richly vascularized fibrotic stroma consisted of cholesterol clefts, foamy macrophages, lymphocytes, and eosinophils, reminiscent of inflammatory granulation tissue (Figure, C)", "The epithelial cells were positive for cytokeratin-7 (CK7) (Figure, D), CK19, epithelial membrane antigen, and glial fibrillary acidic protein", "Ki-67 immunostain showed a proliferation index of less than 1%", "A, Temporal bone at the level of external acoustic canal", "M indicates mastoid cavity", "J, jugular bulb", "C, carotid canal", "E, external ear canal", "B-D, Histopathologic images", "B, Papillary structures", "original magnification ×20. C, Papillary and dilated glandular structures", "original magnification ×10. D, Strong CK7 positivity", "original magnification ×20."], "s1": [0, 1, 2, 3, 4, 5, 6, 7, 8], "s2": [9, 10, 11, 12, 13, 14, 15, 16, 17, 18, 19, 20, 21, 22, 23]} {"key": 1042, "questions": "What Is Your Diagnosis?", "options": [{"label": "A", "disease": "Cerebrospinal fluid leak"}, {"label": "B", "disease": "Isolated anosmia"}, {"label": "C", "disease": "Sarcoidosis"}, {"label": "D", "disease": "Upper respiratory infection"}], "answer_idx": "B", "symptoms": ["A teenaged girl was evaluated for loss of smell with a probable duration of years, but she was uncertain about exact duration", "Her parents stated that at the age of 3 years she sustained minor trauma to her nose while at a playground but that she had not experienced any major head trauma", "The patient had a history of frequent cough and congestion, seasonal allergies, monthly migraine headaches, and considerable nose congestion at night", "She did not recall a history of recurrent sinusitis, snoring, or mouth breathing", "Her family history was otherwise normal and did not include any pattern of familial anosmia or delayed puberty", "She reported the ability to detect very strong odors but not subtle ones", "She was even unable to smell smoke at times", "Physical examination revealed a clinically significant hump on the dorsum of the nose consistent with minimal prior trauma, but no other abnormal findings", "The patient failed the Penn smell tests, correctly identifying only 3 of the 11 smells", "Flexible fiber-optic examination of the nose revealed no masses, polyps, or cysts", "A magnetic resonance image of the anterior skull base demonstrated absent olfactory sulci (OS) and bilateral olfactory grooves containing minimal tissue (Figure, A, yellow arrowheads)", "The Figure, B, depicts normal OS (pink arrowheads) and bulbs (yellow arrowheads) from a different patient for comparison", "A, The anterior skull base of the study patient", "B, A normal patient"], "s1": [0, 1, 2, 3, 4, 5, 6], "s2": [7, 8, 9, 10, 11, 12, 13]} {"key": 1043, "questions": "What Is Your Diagnosis?", "options": [{"label": "A", "disease": "Thyroglossal duct fistula"}, {"label": "B", "disease": "Branchial cleft fistula"}, {"label": "C", "disease": "Nonhealing surgical tract"}, {"label": "D", "disease": "Nocardia infection"}], "answer_idx": "A", "symptoms": ["A man in his early 20s presented to the otolaryngology clinic with a history of a small opening in the left lateral side of the neck that produced chronic purulent discharge", "Despite several previous attempts at surgical resection, the drainage had persisted for 6 years", "Initial computed tomography (CT) identified infiltration of the subcutaneous tissues in the left side of the neck anterior to the strap muscle at the level of the larynx, without discrete abscess", "Computed tomography also showed a small tract that extended from the skin of the lateral side of the neck to the left strap muscles", "The patient was referred to radiology, where a 22-gauge intravenous access catheter was inserted into the draining puncta", "Isovue-300 was infused under conventional fluoroscopic guidance (Figure, A)", "When the patient stated that he tasted contrast material, the infusion was terminated, and the patient was transferred to the CT service, where axial images of the neck were obtained and reformatted in multiple planes (Figure, B and C)", "A, Conventional fluoroscopic fistulogram of the neck", "B and C, Maximum-intensity projection reformatted computed tomographic (CT) images of the neck"], "s1": [0, 1, 2, 3], "s2": [4, 5, 6, 7, 8]} {"key": 1044, "questions": "What is Your Diagnosis?", "options": [{"label": "A", "disease": "Middle aortic syndrome"}, {"label": "B", "disease": "Inflammatory aneurysm"}, {"label": "C", "disease": "Giant cell arteritis"}, {"label": "D", "disease": "Takayasu arteritis"}], "answer_idx": "D", "symptoms": ["A 39-year-old active woman presented with a progressive 6-month history of bilateral lower extremity pain after walking 1 block or ascending a flight of stairs", "The pain was aching, relieved with rest, and primarily in her buttocks and thighs bilaterally", "The patient denied history of prior abdominal or lower extremity surgery, trauma, or cardiovascular problems", "Review of systems and family history revealed nothing remarkable", "Physical examination revealed normal sinus rhythm and normal blood pressure", "She had no abdominal tenderness and no palpable masses", "Femoral pulses were weakly palpable and symmetric, and pedal pulses were not palpable", "Laboratory findings were unremarkable except for an elevated erythrocyte sedimentation rate (ESR) (34 mm/h)", "Blood and urine cultures were negative", "Noninvasive arterial flow studies demonstrated an ankle brachial index of 0.60 on the right and 0.62 on the left with biphasic waveforms in the femoral, popliteal, and tibial vessels", "Subsequent magnetic resonance angiography (MRA) of the chest, abdomen, and pelvis showed mild stenosis of bilateral subclavian arteries and an abnormal-appearing infrarenal aorta (Figure 1)", "Magnetic resonance angiography of the abdomen and pelvis"], "s1": [0, 1, 6, 9, 10, 11], "s2": [2, 3, 4, 5, 7, 8]} {"key": 1045, "questions": "What Is Your Diagnosis?", "options": [{"label": "A", "disease": "Ruptured giant cavernous hemangioma"}, {"label": "B", "disease": "Ruptured complex cystic liver mass"}, {"label": "C", "disease": "Ruptured hepatocellular carcinoma"}, {"label": "D", "disease": "Spontaneous liver bleeding"}], "answer_idx": "C", "symptoms": ["A previously healthy, African American man in his mid-60s with a history of hepatitis C cirrhosis (compensated) presented to the emergency department after collapsing at work with sudden-onset back and abdominal pain", "He was hypotensive (blood pressure, 87/55 mm Hg) on arrival to the emergency department", "The patient’s condition was rapidly stabilized and computed tomography with contrast demonstrated a large (12.4 × 10.2-cm) right hepatic mass with heterogeneous enhancement on arterial phase and intrahepatic and intraperitoneal hemorrhage (Figure 1A)", "Emergency angiography was performed by interventional radiology, demonstrating active contrast extravasation originating from the right hepatic artery", "Embolization was performed with Gelfoam and microparticles, resulting in hemodynamic stability and a quick recovery", "Delayed liver resection was subsequently performed (Figure 1B)", "A, Large hepatic mass seen on computed tomography, with evidence of intrahepatic and intraperitoneal hemorrhage", "B, Pathologic specimen after delayed surgical treatment of the underlying mass"], "s1": [0, 1], "s2": [2, 3, 4, 5, 6, 7]} {"key": 1046, "questions": "What Is Your Diagnosis?", "options": [{"label": "A", "disease": "Bullous pemphigoid"}, {"label": "B", "disease": "Bullous amyloidosis"}, {"label": "C", "disease": "Herpes simplex infection"}, {"label": "D", "disease": "Cutaneous candidiasis"}], "answer_idx": "B", "symptoms": ["A woman in her 80s presented with a 1-week history of progressive, painful blisters and ulcers of the groin", "This was associated with enlargement of the tongue over the past 3 months, leading to difficulty with mastication", "She also reported numbness with no associated weakness of her distal lower limbs for 3 months", "This restricted her mobility to the extent that she required diapers", "However, she declined any further investigations for her peripheral neuropathy", "She had IgG λ-type multiple myeloma complicated by anemia and lytic lesions in the calvarium, which was managed conservatively with transfusions", "She was taking folic acid and intramuscular cyanocobalamin supplements owing to deficiencies attributed to a dietary cause", "On examination, there were circinate purpuric patches with erosions on the natal cleft, buttocks, inguinal creases, and vulva (Figure, A)", "Hemorrhagic flaccid bullae were observed on the posterior perineum", "This was accompanied by macroglossia, glossitis, and a hemorrhagic erosion on the hard palate (Figure, B)", "There were purpuric macules periorbitally and on the lips, inframammary folds, forearms, and thighs", "Laboratory findings revealed a platelet count of 171 × 103/μL (reference range, 140-440 × 103/μL), prothrombin time of 11.7 seconds (reference range, 9.9-11.4 seconds), activated partial thromboplastin time of 36.0 seconds (reference range, 25.7- 32.9 seconds), and serum zinc levels of 34.4 μg/dL (reference range, 72.4-124.4 μg/dL)", "(To convert zinc to micromoles per liter, multiply by 0.153.) Results from initial punch biopsies from a purpuric macule on the right inframammary fold and a blister at the perineum were inconclusive", "A repeated biopsy from a hemorrhagic blister at the left groin was performed (Figure, C and D)", "A, Purpura and erosions at the buttocks", "B, Clinical findings of macroglossia and glossitis with purpura of the lips", "C, Punch biopsy specimen of a flaccid bulla showing the floor of a subepidermal blister lined by fibrin and a chronic inflammatory infiltrate", "D, Scattered short eosinophilic amorphous bands in the dermis"], "s1": [1, 2, 5, 6, 9, 10, 11, 12], "s2": [0, 3, 4, 7, 8, 13, 14, 15, 16, 17]} {"key": 1047, "questions": "What Is Your Diagnosis?", "options": [{"label": "A", "disease": "Oral sarcoidosis"}, {"label": "B", "disease": "Postherpetic granulomatous reaction"}, {"label": "C", "disease": "Granulomatous cheilitis"}, {"label": "D", "disease": "Crohn disease"}], "answer_idx": "C", "symptoms": ["A man in his early 40s with a medical history significant for verruca vulgaris, dyshidrotic eczema, and human immunodeficiency virus (HIV) diagnosed 1 year prior to presentation reported a chief complaint of dry, swollen lips", "The patient reported a 1-month history of “puffy and bumpy” lips", "He also described his lower lip as sometimes becoming “dry and scaly” during this time but denied any bleeding from the area", "On physical examination, there were several erythematous papules at the vermillion border of the lower lip within a background of edema (Figure, A and B)", "In addition, the patient had slight fissuring of the tongue, but the remainder of the oral mucosa was normal and intact", "Findings of a complete review of systems were unremarkable, other than occasional headaches", "The patient denied fevers, night sweats, abdominal pain, neurologic symptoms, chest pain, cough, and shortness of breath", "He disclosed full compliance with his HIV highly active antiretroviral therapy (HAART), which included ritonavir, darunavir, etravirine, and raltegravir", "He also reported taking sumatriptan as needed for headaches", "Numerous laboratory tests were performed, including a complete blood cell count, complete metabolic panel, CD4 count, HIV viral load, and assays for C-reactive protein, erythrocyte sedimentation rate, angiotensin-converting enzyme, and rapid plasma reagin", "all results were normal except for a slightly elevated angiotensin converting enzyme level (84 µg/L", "normal, 9-67 µg/L)", "His CD4 count was within normal limits (520 cells/µL) and HIV viral load was undetectable", "A biopsy was performed on the right lower lip (Figure, C and D)", "Clinical images (A and B) show persistent, nontender swelling of the lips in a patient with human immunodeficiency virus, showing erythematous papules at the vermillion border and edema of the lower lip", "C and D, Histologic images of a biopsy specimen from a papule on the lower lip show, at low magnification (C), a nodular, nonnecrotizing granulomatous dermatitis and, at higher magnification (D), the presence of epithelioid histiocytes with admixed lymphocytes"], "s1": [0, 1, 2, 3, 4, 14, 15], "s2": [5, 6, 7, 8, 9, 10, 11, 12, 13]} {"key": 1048, "questions": "What Is Your Diagnosis?", "options": [{"label": "A", "disease": "Cowden syndrome"}, {"label": "B", "disease": "Muir-Torre syndrome"}, {"label": "C", "disease": "Birt-Hogg-Dube syndrome"}, {"label": "D", "disease": "Basal cell nevus syndrome"}], "answer_idx": "B", "symptoms": ["A 41-year-old woman presented with a rapidly growing, painful nodule on her upper cutaneous lip (Figure, A) as well as several smaller papules on her face and neck, all of which had developed within the preceding 6 months", "She was otherwise asymptomatic", "A, Centrally depressed, firm, pink 1.1-cm nodule on the upper cutaneous lip and skin-colored papule on the left medial cheek (above marker dots)", "B, Cup-shaped tumor demonstrated keratinocyte atypia with a central core of keratinaceous debris", "C, Sebaceous tumor with a modestly increased ratio of basaloid cells to sebocytes", "The patient’s medical history included diabetes mellitus type 2, hypercholesterolemia, hypertension, and depression", "Review of her family medical history revealed that her paternal grandfather died from melanoma and that her father died from colon cancer", "She stated that most of the men in her family had died from some form of cancer by the fourth decade of life", "On physical examination, a tender, centrally depressed, firm, pink 1.1-cm nodule was present on her upper cutaneous lip (Figure, A)", "Three other skin-colored to pink papules were present on her face and neck", "Shave biopsies of all 4 skin lesions were performed (Figure, B and C)"], "s1": [0, 2, 3, 4, 8, 9, 10], "s2": [1, 5, 6, 7]} {"key": 1049, "questions": "What Is Your Diagnosis?", "options": [{"label": "A", "disease": "Pressure ulcer"}, {"label": "B", "disease": "Cutaneous fungal infection"}, {"label": "C", "disease": "Neonatal lupus erythematosus"}, {"label": "D", "disease": "Ecthyma gangrenosum"}], "answer_idx": "B", "symptoms": ["A male neonate was born at 25 weeks’ gestation by cesarean delivery because of hemolysis, elevated liver enzymes, and low platelet count (HELLP) syndrome in his primigravid mother, who also had systemic lupus erythematosus managed with hydroxychloroquine, aspirin, and prednisone during pregnancy", "The patient’s premature birth was complicated by acute respiratory distress that necessitated intubation and necrotizing enterocolitis that required parenteral nutrition", "At birth, the patient was found to be mildly anemic (hemoglobin level, 11.0 g/dL [to convert to grams per liter, multiply by 10]) and thrombocytopenic (platelet count, 120 × 103/µL [to convert to 109/L, multiply by 1]) with concomitant intraventricular hemorrhage", "Although afebrile, he was treated empirically for sepsis with vancomycin hydrochloride and cefepime hydrochloride", "A cerebrospinal fluid culture result was positive for Staphylococcus epidermidis, but the results of additional blood cultures were negative for organisms", "On day 8 after birth, a 0.5-cm, circular, crusted papule with a surrounding rim of erythema on the left hip was noted (Figure, A)", "A punch biopsy of the lesion was performed (Figure, B and C)", "A, Crusted circular papule with a surrounding rim of erythema and induration on the left hip", "B, Biopsy specimen revealing epidermal necrosis with fungal elements within the superficial dermis (hematoxylin-eosin, original magnification ×20)", "C, Biopsy specimen highlighting branching and septate hyphae within the superficial dermis (specimen was stained with Gomori methenamine silver, original magnification ×40)"], "s1": [0, 1, 2, 3, 4], "s2": [5, 6, 7, 8, 9]} {"key": 1050, "questions": "What Is Your Diagnosis?", "options": [{"label": "A", "disease": "Acute hepatitis associated with Epstein-Barr virus infection"}, {"label": "B", "disease": "Vanishing bile duct syndrome associated with Hodgkin lymphoma"}, {"label": "C", "disease": "Sarcoidosis"}, {"label": "D", "disease": "Metastatic cholangiocarcinoma secondary to primary sclerosing cholangitis"}], "answer_idx": "B", "symptoms": ["A Hispanic man in his 20s with no known medical history presented with worsening abdominal pain over 5 days, persistent pruritus, and scleral icterus", "The patient noted swelling on the right side of the neck and a 5-kg weight loss over the past 2 months", "He reported a history of moderate alcohol consumption, drinking 1 beer per day during the week and approximately 5 to 8 beers over the weekend, but denied any use of nutritional supplements or illicit drugs", "There was no recent travel outside the state", "Human immunodeficiency virus test results on admission were negative", "Physical examination revealed scleral icterus, jaundice, and a fixed right supraclavicular lymph node conglomerate measuring 6 × 3 cm", "There was no cervical, left supraclavicular, axillary, or inguinal lymphadenopathy", "Computed tomography of the neck, chest, abdomen, and pelvis demonstrated a large right supraclavicular lymph node, extensive mediastinal lymphadenopathy, a large extracardiac mass abutting the left atrium, hepatomegaly, and multiple splenic lesions", "Laboratory evaluation demonstrated a total serum bilirubin level of 10.5 mg/dL with a direct component of 8.4 mg/dL (to convert to micromoles per liter, multiply by 17.104)", "Renal function was normal", "Mild leukopenia and slight thrombocytopenia in the absence of associated anemia was noted on initial complete blood cell count", "A liver biopsy was performed, with results shown in Figure 1.Hepatic biopsy at time of presentation shows a portal tract containing both hepatic artery and hepatic vein branches (hematoxylin-eosin, original magnification ×40)", "Vanishing bile duct syndrome associated with Hodgkin lymphoma"], "s1": [0, 2, 3, 4, 9], "s2": [1, 5, 6, 7, 8, 10, 11, 12]} {"key": 1051, "questions": "What Is Your Diagnosis?", "options": [{"label": "A", "disease": "Gastrointestinal stromal tumor"}, {"label": "B", "disease": "Type III neuroendocrine tumor of the stomach"}, {"label": "C", "disease": "Gastric metastasis of Merkel cell carcinoma"}, {"label": "D", "disease": "Gastric adenocarcinoma"}], "answer_idx": "C", "symptoms": ["A woman in her 70s was referred to our tertiary care center for an asymptomatic mass of the gastric fundus", "Her medical history was significant for chronic gastritis, which was being treated with esomeprazole 20 mg", "a laparoscopic cholecystectomy 18 years prior", "and excision of a 26-mm Merkel cell carcinoma of the left knee with negative results of the sentinel lymph node biopsy (stage II) 19 months prior", "Results of the patient’s follow-up examinations were negative until a high-resolution computed tomographic scan showed a 45 × 40-mm intraluminal protruding mass with slightly enlarged lymph nodes along the lesser curvature of the gastric fundus (Figure, A)", "Results of fluorine 18–labeled fluorodeoxyglucose and gallium citrate Ga 68–labeled 1,4,7,10-tetraazacyclododecane-N,N',N'',N'''-tetraacetic (DOTA)-D-Phe1-Try3-octreotide (DOTATOC) positron emission tomography confirmed the presence of a metabolically active mass in the gastric fundus (maximum standardized uptake value, 6) with intense 68Ga–DOTATOC uptake (maximum standardized uptake value, 24) as well as mild 68Ga–DOTATOC uptake in 2 lymph nodes of the lesser curvature", "A, Multislice computed tomographic scan showing the 45 × 40-mm mass in the gastric fundus", "B, Endoscopic image showing the endophytic lesion in the posterior wall of the gastric fundus at the esophagogastric junction", "Preoperative workup was completed via esophagogastroduodenoscopy, which showed the presence of the polypoid lesion in the posterior wall of the gastric fundus at the level of the esophagogastric junction (Figure, B)", "Multiple biopsies were performed at the time of the endoscopy"], "s1": [0, 1, 2, 3], "s2": [4, 5, 6, 7, 8, 9]} {"key": 1052, "questions": "What Is Your Diagnosis?", "options": [{"label": "A", "disease": "Cystadenoma"}, {"label": "B", "disease": "Cystadenocarcinoma"}, {"label": "C", "disease": "Hydatid cyst of the pancreas"}, {"label": "D", "disease": "Main duct intraductal papillary mucinous neoplasia"}], "answer_idx": "C", "symptoms": ["We present the case of a 40-year-old man who was admitted to our hospital with acute abdominal pain on the right flank", "A similar episode had occurred 1 month before admission that improved spontaneously", "A physical examination did not reveal any clinical abnormities, but the patient reported dysuria and abnormally frequent urination", "A computed tomographic scan of the abdomen revealed nephrolithiasis with a 4-mm kidney stone on the right side and a slightly congested right kidney", "During computed tomography, we incidentally found a 10 × 10 × 8-cm mass in the tail of the pancreas (Figure 1), as well as thrombosis of the splenic vein", "His medical history was uneventful, and there was no weight loss", "However, the patient’s father had died of a pancreatic carcinoma the previous year at the age of 65 years", "The patient’s laboratory values, including cancer antigen 19-9 and carcinoembryonic antigen levels, were normal", "Computed tomographic (CT) scans of the abdomen reveal the mass in the pancreas (arrowheads)"], "s1": [0, 1, 2, 3], "s2": [4, 5, 6, 7, 8]} {"key": 1053, "questions": "What Is Your Diagnosis?", "options": [{"label": "A", "disease": "Lymphoma"}, {"label": "B", "disease": "Melanoma"}, {"label": "C", "disease": "Rhabdomyoma"}, {"label": "D", "disease": "Metastatic carcinoma"}], "answer_idx": "D", "symptoms": ["A woman in her 50s presented for evaluation of intermittent epigastric pain, palpitations, and hot flashes for 2 months", "The patient had a history of an ovarian cyst treated conservatively, and cosmetic breast augmentation", "She had a 10 pack-year smoking history and denied alcohol or illicit drug use", "Examination revealed distant heart sounds but otherwise had unremarkable results", "An echocardiogram demonstrated a mass in the left ventricular myocardium", "Cardiac magnetic resonance imaging was obtained to further assess the mass lesion", "This revealed multiple intramyocardial masses involving the left ventricle, the largest of which measured 2.7 × 2.2 cm", "aortopulmonary window lymphadenopathy", "and moderate pericardial effusion (Figure)", "Computed tomographic imaging of the chest, abdomen, and pelvis with contrast failed to reveal discrete masses or lymphadenopathy elsewhere", "A transcutaneous biopsy of the left ventricular mass was nondiagnostic"], "s1": [0, 1, 2], "s2": [3, 4, 5, 6, 7, 8, 9, 10]} {"key": 1054, "questions": "What Is The Diagnosis?", "options": [{"label": "A", "disease": "Trichodysplasia spinulosa–associated polyomavirus"}, {"label": "B", "disease": "Demodex folliculorum–associated spinulosus"}, {"label": "C", "disease": "Ulerythema ophryogenes"}, {"label": "D", "disease": "Hyperkeratotic spicules of multiple myeloma"}], "answer_idx": "B", "symptoms": ["A woman in her 70s with a medical history that was significant for polycythemia vera and acute myelogenous leukemia was seen for evaluation of textural skin changes on her face that had been present for 2½ years", "She denied having pruritus or pain, but her skin had a rough sandpapery sensation", "She reported that the involvement extended to her neck, the upper part of her chest, and her bilateral forearms", "She associated the onset of the textural changes with her diagnosis of polycythemia vera", "She stated that it began before her diagnosis of acute myelogenous leukemia and the initiation of treatment with hydroxyurea 4 years later", "She had previously received treatment with long courses of triamcinolone acetonide cream, 0.1%, bacitracin ointment, and a combination antibiotic ointment of neomycin sulfate, polymyxin B, and bacitracin, with no improvement of her skin’s condition", "Her additional daily medication regimen included donepezil hydrochloride, melatonin, and aspirin, 81 mg", "Findings from the patient’s physical examination revealed background erythema with numerous rough horny spicules at the follicular ostia on her face, ears, and neck", "the upper part of her chest", "and her bilateral forearms", "Findings from the rest of her examination revealed a pigmented macule on her right cheek", "Two punch biopsy specimens were obtained from her right cheek and right postauricular area and submitted for histopathologic examination (Figure)", "Histopathologic images (hematoxylin-eosin) of the punch biopsy specimen from the right side of the lateral neck"], "s1": [6, 10, 11, 12], "s2": [0, 1, 2, 3, 4, 5, 7, 8, 9]} {"key": 1055, "questions": "What Is Your Diagnosis?", "options": [{"label": "A", "disease": "Cutaneous tuberculosis"}, {"label": "B", "disease": "Ecthyma gangrenosum"}, {"label": "C", "disease": "Leishmaniasis"}, {"label": "D", "disease": "Cutaneous diphtheria"}], "answer_idx": "D", "symptoms": ["A 12-year-old boy presented for evaluation of a 3-month history of ulcerated lesions on the dorsal aspect of his feet since returning from Pakistan, where his family was living", "The patient had no history of medical problems", "He was born in Spain and had received his childhood immunizations", "He remembered insect bites on his feet, but the lesions later became ulcerated and painful", "The patient denied experiencing fever, arthralgia, or a sore throat", "Findings from the physical examination revealed 6 painful ulcerated, exudative nodules with a grayish membrane over the ulcers on the dorsal aspect of his feet (Figure 1)", "No signs of lymphangitis or lymphadenopathy were detected", "A 4-mm punch biopsy specimen from one lesion was obtained and stained with hematoxylin-eosin and Gram stain for histopathologic analysis", "Another skin specimen was cultured for microbiologic analysis", "Ulcerated and exudative nodules with grayish membrane on the dorsal aspect of the feet"], "s1": [0, 3, 5, 9], "s2": [1, 2, 4, 6, 7, 8]} {"key": 1056, "questions": "What Is Your Diagnosis?", "options": [{"label": "A", "disease": "Child abuse"}, {"label": "B", "disease": "Morquio syndrome"}, {"label": "C", "disease": "GM1 gangliosidosis"}, {"label": "D", "disease": "Hurler-Scheie syndrome"}], "answer_idx": "C", "symptoms": ["A 6-month-old daughter of nonconsanguineous parents was seen with multiple polycyclic dark to less dark brown, partly livid shimmering macules, which covered large areas of the back, neck, buttocks, and lower extremities (Figure, A)", "According to her mother, new lesions had appeared during the past weeks on the shoulders", "Physical examination revealed coarse facial features, hepatomegaly (7 cm below the costal arch), distinctive lumbar kyphosis, and muscular hypotonia compared with her twin brother", "Dermoscopy showed faint clouds of nonhomogeneously distributed light brown pigment", "A, Polycyclic, brown, partly livid shimmering macules covered large areas of the back, neck, buttocks, and lower extremities", "B, Fusiform and dendritic cells were found scattered in the dermis (hematoxylin-eosin, original magnification ×100)", "C, Immunohistological staining of the same specimen (original magnification ×200)", "S-100 protein positivity identified these cells as melanocytes", "D, A triangle-shaped first lumbar vertebral body is shown"], "s1": [0, 1, 3, 4, 5, 6, 7], "s2": [2, 8]} {"key": 1057, "questions": "What Is Your Diagnosis?", "options": [{"label": "A", "disease": "Central nervous system toxoplasmosis"}, {"label": "B", "disease": "Metastatic carcinoma"}, {"label": "C", "disease": "Central nervous system lymphoma"}, {"label": "D", "disease": "Posttransplant lymphoproliferative disorder"}], "answer_idx": "D", "symptoms": ["A woman in her 50s was seen by her physician with a 1-month history of gait unsteadiness and imbalance and was suspected to have neuropathy, but she then went to the emergency department reporting 2 successive transient episodes, each lasting less than 1 hour, of acute dizziness, disorientation, gait dysfunction, and confusion", "Urinalysis revealed a urinary tract infection, but computed tomography (CT) of the head showed extensive mass lesions with bilateral edema", "Magnetic resonance imaging (MRI) of the brain delineated expansile mass lesions of the splenium of the corpus callosum, with T2–fluid-attenuated inversion recovery hyperintensity in the bilateral parietal lobes, left thalamus, and midbrain (Figure, A)", "administration of contrast revealed multiple ring-enhancing lesions (Figure, B)", "The woman’s medical history was significant for type 2 diabetes mellitus (DM), hypertension, recurrent urinary tract infections, onychomycosis, end-stage renal failure, and a history of living, unrelated donor renal transplant 12 years earlier, kidney transplant rejection, and prior episodes of syncope", "Medications included tacrolimus, mycophenolate mofetil hydrochloride, prednisone (1 mg/d), losartan potassium, and insulin", "Family history was notable for her father having died of lung cancer and her mother had heart disease and DM", "Social history did not reveal any unusual exposures", "Physical examination results showed a transplant surgical scar and onychomycosis", "Neurologic findings revealed reasonably preserved mental status, with bedside examination revealing normal attention, language, and orientation but mild memory deficits for short-term recall (2 of 3 words recalled at a 5-minute delay) and long-term fund of knowledge", "Cranial nerve, motor, sensory, and reflex examination results were significant only for gait ataxia, with an inability to perform tandem gait", "Blood test results were consistent with DM", "Findings from cerebrospinal fluid (CSF) examination included a white blood cell count of 9/µL, with differential examination showing 84% lymphocytes, 13% monocytes, and 3% neutrophils", "red blood cell count of 1080/µL", "protein level, 180 mg/dL", "and glucose level, 87 mg/dL", "Flow cytometry revealed B and T lymphocytes but no monoclonal populations of cells", "Polymerase chain reaction amplification tests on CSF revealed no evidence of cytomegalovirus or herpes simplex virus 1 or 2 genetic material", "Serum and CSF serologic test results were negative for antibodies to toxoplasma", "testing using polymerase chain reaction on blood for Epstein-Barr virus (EBV) revealed minimal (very low positive) viral DNA at 52 IU/mL", "A CT scan of the chest, abdomen, and pelvis showed a small, subpleural nodule", "fatty liver", "diverticulosis", "failed native kidneys", "and normal transplanted kidney", "Whole-body positron emission tomography revealed hypermetabolic regions in the brain", "Dexamethasone sodium phosphate and levetiracetam were administered", "The patient underwent a diagnostic procedure", "Brain magnetic resonance imaging showing unsuspected mass lesions", "A, A T2 fluid-attenuated inversion recovery (FLAIR) sequence illustrating prominent T2 hyperintensity in the right more than the left parietal white matter suggestive of edema, an expansive mass lesion on the right more than the left splenium of the corpus callosum, and an infiltrative mass lesion in the left thalamus extending into the mesencephalon", "B, A T1 sequence after administration of gadolinium contrast agent", "Ring-enhancing regions are visible in the right parietal lobe and splenium of the corpus callosum, and a more solidly enhancing lesion is seen in the left thalamus as well as some apparent leptomeningeal enhancement"], "s1": [0, 4, 5, 6, 7, 8, 9, 10, 11, 20, 21, 22, 23, 24, 26], "s2": [1, 2, 3, 12, 13, 14, 15, 16, 17, 18, 19, 25, 27, 28, 29, 30, 31]} {"key": 1058, "questions": "What Is Your Diagnosis?", "options": [{"label": "A", "disease": "Herpetic whitlow"}, {"label": "B", "disease": "Cutaneous anthrax"}, {"label": "C", "disease": "Orf"}, {"label": "D", "disease": "Sporotrichosis"}], "answer_idx": "C", "symptoms": ["A healthy young girl presented in June 2014 with a 4-week history of a painless, enlarging lesion on her right thumb that started as a small erythematous blister", "She reported recently injuring her hand on a metal fence at home and biting her nails frequently", "She lives on a farm in Ohio and has extensive direct contact with sheep and cows", "family pets include a cat, a dog, and a hamster", "There was no known recent travel or contact with anyone with a similar lesion", "She reported mild tender lymphadenopathy of the right axilla but denied fever", "Examination revealed an erythematous, 1 × 1.5-cm, well-defined eroded pustule with central crusting and an erythematous border (Figure)", "There were several slightly enlarged, mildly tender lymph nodes noted in the right axilla", "Eroded and crusted pustule with surrounding inflammation on the thumb", "Complete blood cell count, erythrocyte sedimentation rate, and basic metabolic panel results were unremarkable", "Findings on radiography revealed focal soft-tissue swelling with no evidence of foreign body, subcutaneous gas, or osseous involvement"], "s1": [0, 5, 6, 7, 8], "s2": [1, 2, 3, 4, 9, 10]} {"key": 1059, "questions": "What Is Your Diagnosis?", "options": [{"label": "A", "disease": "True vocal fold polyp"}, {"label": "B", "disease": "Vocal fold squamous cell carcinoma"}, {"label": "C", "disease": "True vocal fold myxoma"}, {"label": "D", "disease": "Thyroarytenoid muscle rhabdomyoma"}], "answer_idx": "C", "symptoms": ["A man in his 60s presented to the clinic with a 5-month history of sudden-onset, progressively worsening dysphonia, dry cough, and the sensation of a lump in his throat", "He described a progressively worsening voice with use to the point where he was nearly aphonic by the end of his workday", "He denied any heartburn, hemoptysis, pain, neck masses, or smoking history", "Prior treatment with 2 rounds of oral antibiotics and 1 round of oral steroids from his primary care physician did not improve his symptoms", "Objectively, the patient’s voice was rough, diplophonic, and very strained", "Videostroboscopic examination in the clinic revealed a broad-based submucosal mass in the anterior two-thirds of the right true vocal fold (Figure, A) that disrupted vibration of both vocal folds owing to its size and anterior location (Figure, B and Video)", "Findings from the remainder of his head and neck examination were unremarkable", "The patient was taken to the operating room for microdirect laryngoscopy under general anesthesia (Figure, C)", "A microflap incision along the lateral aspect of the right true vocal fold mass was performed, and a firm, submucosal, tan-colored, seemingly encapsulated, multilobulated mass was removed en bloc with narrow margins and sent for permanent section analysis (Figure, D)", "The mass was deep to the epithelium of the true vocal fold and superficial to the vocal ligament", "A and B, Clinical views of endoscopic appearance of the anterior right true vocal fold mass", "A, View showing vocal folds abducted", "B, View showing vocal folds adducted", "C and D, Intraoperative endoscopic views", "C, Submucosal mass", "D, A microflap incision was performed along the lateral aspect of the lesion and a multilobulated, firm submucosal mass deep to the epithelium but superficial to the vocal ligament was resected"], "s1": [0, 1, 2, 3, 4, 6], "s2": [5, 7, 8, 9, 10, 11, 12, 13, 14, 15]} {"key": 1060, "questions": "What Is Your Diagnosis?", "options": [{"label": "A", "disease": "Angiomyolipoma"}, {"label": "B", "disease": "Angiofibroma"}, {"label": "C", "disease": "Nasal angioleiomyoma"}, {"label": "D", "disease": "Hemangioma"}], "answer_idx": "C", "symptoms": ["A woman in her 70s presented with a 4-day history of new-onset, left-sided acute epistaxis described as episodes of bright red bleeding and clotting from the left anterior nare, occurring multiple times per day, lasting minutes each time, and stopped with pressure", "She denied other sinonasal symptoms, history of epistaxis, nasal trauma, or surgery", "Clinical evaluation revealed a soft-tissue mass filling the left nasal cavity and based on the mucosal surface of the anterolateral nasal vestibule", "Noncontrast paranasal sinus computed tomography showed a 1.2 × 0.8-cm ovoid soft-tissue mass opacifying the posterior aspect of the left nasal vestibule (Figure, A, arrowhead)", "Endoscopic excision under local anesthesia was performed to remove a broad-based soft-tissue mass from the left nasal vestibule mucosa with a cuff of grossly healthy submucosa", "A tissue specimen was biopsied (Figure, B-D)", "The patient had an uneventful postoperative course, and her epistaxis symptoms were resolved at the 5-month follow-up", "A, Noncontrast paranasal sinus computed tomographic image", "B-D, Histopathologic images from the nasal cavity mass, original magnifications ×100."], "s1": [0, 1, 6], "s2": [2, 3, 4, 5, 7, 8]} {"key": 1061, "questions": "What Is Your Diagnosis?", "options": [{"label": "A", "disease": "Inverted papilloma"}, {"label": "B", "disease": "Fungal ball"}, {"label": "C", "disease": "Esthesioneuroblastoma"}, {"label": "D", "disease": "Sinonasal undifferentiated carcinoma"}], "answer_idx": "C", "symptoms": ["A teen-aged boy with a recent history of resolved streptococcal pharyngitis presented with dizziness, vomiting, unstable gait, and headaches of 2 weeks’ duration", "Laboratory evaluation revealed a sodium level of 109 mEq/L", "Rapid correction to 136 mEq/L occurred, resulting in lethargy, aphasia, and ataxia", "He was diagnosed as having idiopathic syndrome of inappropriate antidiuretic hormone secretion (SIADH)", "Magnetic resonance imaging (MRI) demonstrated an abnormal signal within the basal ganglia and insular cortex", "A 2.5 × 1.5 × 1.8-cm mass was found in the superior right maxillary sinus with extension into the infundibulum (Figure, A)", "Computed tomographic (CT) scans of the sinuses without intravenous contrast demonstrated that the mass had calcifications", "Owing to persistent hyponatremia (sodium level, 125-130 mEq/L) and an identifiable lesion, the patient was taken to the operating room for biopsy", "He underwent a maxillary antrostomy with near-complete removal of the mass", "Over the subsequent 2 weeks, the hyponatremia resolved and the patient was left with slight dysarthria and difficulties with fine-motor movements", "The submucosa contained a small, round, blue-cell tumor growing in circumscribed lobules and nests separated by vascularized fibrous stroma", "The neoplastic cells had uniform, small, round nuclei with scant cytoplasm", "Cells were arranged in a pseudorosette pattern (Figure, B)", "The nuclei demonstrated dispersed, salt and pepper nuclear chromatin and inconspicuous nucleoli", "The cells were surrounded by a neurofibrillary matrix", "There were prominent microcalcifications", "No mitotic figures were seen", "The tumor cells were diffusely positive for immunohistochemical marker synaptophysin (Figure, C)", "Results from cytokeratin staining were negative (Figure, D)", "A, T1-weighted postgadolinium magnetic resonance image, coronal view", "B-D, Histopathological images", "B, Hematoxylin-eosin, original magnification ×100. C, Tissue positivity for synaptophysin, original magnification ×200. D, Tissue negativity for cytokeratin, original magnification ×200."], "s1": [0, 1, 2, 3, 4, 7, 9], "s2": [5, 6, 8, 10, 11, 12, 13, 14, 15, 16, 17, 18, 19, 20, 21]} {"key": 1062, "questions": "What Is Your Diagnosis?", "options": [{"label": "A", "disease": "Squamous cell carcinoma"}, {"label": "B", "disease": "Lymphoma"}, {"label": "C", "disease": "Tuberculous lymphadenitis"}, {"label": "D", "disease": "Papillary thyroid carcinoma"}], "answer_idx": "C", "symptoms": ["A woman in her 60s presented to the emergency department with a 3-week history of enlarging and painful cervical masses", "She reported a distant history of breast cancer and a recent history of an oral cavity lesion, which was not biopsied or followed up", "She did not have any known tuberculosis exposure, recent foreign travel, exposure to unpasteurized food, history of immunodeficiency, or alcohol or tobacco use", "She did not have fever, chills, cough, dysphagia, or odynophagia", "Physical examination revealed a healthy-appearing woman with normal vital signs", "She was found to have bilateral cervical lymphadenopathy, which included fixed and bulky nodes in level IB and IIA and nontender level IV and V nodes", "An oral cavity examination revealed no evidence of masses or lesions, floor of mouth and base of tongue were soft, and the palatine tonsils had no lesions or ulcers", "Flexible laryngoscopy revealed no evidence of pharyngeal or laryngeal lesions", "Contrast-enhanced computed tomographic (CT) scans of the neck (Figure) revealed bilateral lymphadenopathy with central necrosis", "Computed tomographic scan with contrast", "A, At the level of mandible", "B, At the level of thyroid cartilage", "C, Through the sphenoid", "D, Through the temporomandibular joint"], "s1": [0, 1, 2, 3, 5, 8], "s2": [4, 6, 7, 9, 10, 11, 12, 13]} {"key": 1063, "questions": "What Is Your Diagnosis?", "options": [{"label": "A", "disease": "Arteriovenous malformation of the oropharynx"}, {"label": "B", "disease": "Aberrant internal carotid artery"}, {"label": "C", "disease": "Cavernous hemangioma of the oropharynx"}, {"label": "D", "disease": "Internal carotid artery pseudoaneurysm"}], "answer_idx": "B", "symptoms": ["A woman in her 70s was referred to our hospital for evaluation of a dry cough, foreign body sensation in the throat, and mild dysphagia of 2 months’ duration", "She had no dyspnea or hoarseness", "Her medical history included hypertension, diabetes mellitus, and right-sided chronic otitis media of 4 years’ duration successfully treated surgically 1 year earlier (tympanoplasty type 1)", "On oral endoscopic examination, mild bilateral hyperemia of the tonsillar area and posterior pharyngeal wall were noted", "A submucosal pulsatile mass was clearly visible in the right retropharyngeal area (Figure, A and B", "Video)", "The base of tongue was normal", "No other clinically significant abnormalities were noted", "Part of the clinical findings were suggestive of laryngopharyngeal reflux disease, and oral esomeprazole, 40 mg/d, was prescribed", "Head and neck magnetic resonance angiography was recommended (Figure, C and D)", "She was reevaluated after 15 days", "A and B, Endoscopic examination of the oropharynx revealing a submucosal pulsatile mass in the right retropharyngeal area", "C, Three-dimensional reconstruction", "Abnormal medial course of the right internal carotid artery (arrow)", "D, T1-weighted gradient echo", "Aberrant internal carotid artery in the retropharyngeal space"], "s1": [0, 1, 3, 6, 7, 8, 10], "s2": [2, 4, 5, 9, 11, 12, 13, 14, 15]} {"key": 1064, "questions": "What Is Your Diagnosis?", "options": [{"label": "A", "disease": "Neurosarcoid"}, {"label": "B", "disease": "Melanoma metastasis"}, {"label": "C", "disease": "Hypophysitis"}, {"label": "D", "disease": "Pituitary macroadenoma"}], "answer_idx": "C", "symptoms": ["A 60-year-old man with advanced melanoma presented with headaches and increasing fatigue during treatment with ipilimumab", "Several months earlier, he presented to the emergency department with shortness of breath", "Imaging revealed pulmonary emboli and incidental groin lymphadenopathy", "Biopsy of a lymph node showed melanoma", "He then underwent inguinal lymph node dissection and was found to have 9 lymph nodes involved with melanoma (stage IIIC melanoma)", "He was then treated with ipilimumab (3 mg/kg), a monoclonal antibody to cytotoxic T lymphocyte antigen-4 (CTLA-4), every 3 weeks, as part of an investigational adjuvant therapy clinical trial", "When presenting for his fourth and final dose of ipilimumab, he reported central headaches, severe fatigue, poor appetite, intermittent nausea, and a mild rash", "On examination, vital signs were within normal limits", "Physical examination had largely unremarkable results except for a slightly red, raised rash over his chest and proximal extremities", "Laboratory testing showed the following: thyrotropin, 0.02 mIU/L (reference range, 0.3-5.0 mIU/L), and free thyroxine, less than 0.25 ng/dL (reference range, 0.5-1.2 ng/dL", "to convert to picomoles per liter, multiply by 12.871)", "Cosyntropin stimulation test showed a cortisol peak level of 0.9 µg/dL (reference range, >20 µg/dL", "to convert to nanomoles per liter, multiply by 27.588)", "He had low corticotropin (<5 pg/mL", "reference range, 7-51 pg/mL", "to convert to picomoles per liter, multiply by 0.22) and testosterone levels (<0.1 ng/mL", "reference range, 2-11 ng/mL", "to convert to nanomoles per liter, multiply by 0.0347)", "Magnetic resonance imaging (MRI) of the brain is shown in the Figure", "Magnetic resonance imaging of the brain with contrast"], "s1": [0, 1, 2, 3, 4, 5, 6, 7, 8, 18, 19], "s2": [9, 10, 11, 12, 13, 14, 15, 16, 17]} {"key": 1065, "questions": "What Is Your Diagnosis?", "options": [{"label": "A", "disease": "Giant perianal abscess"}, {"label": "B", "disease": "Giant condyloma"}, {"label": "C", "disease": "Anal adenocarcinoma"}, {"label": "D", "disease": "Anal squamous cell carcinoma"}], "answer_idx": "C", "symptoms": ["A 65-year-old man was admitted to our hospital with a painful anal mass and fecal incontinence", "The anal lesion appeared 1 year earlier and grew slowly to the current size", "The patient reported bleeding from the rectum after defecation and denied weight loss or fever", "The patient noted no other symptoms", "His medical history included atopic dermatitis and pulmonary tuberculosis", "There was no family history of gastrointestinal pathology", "He had not undergone any previous abdominal surgery, except inguinal hernia repair", "He claimed to be a heavy smoker", "No history of sexually transmitted diseases was found", "Physical examination revealed a 12 × 10-cm hard mass on the left side of the anus (Figure 1A)", "It was ulcerated, covered by purulent exudate, and very painful", "The scrotum was mildly tender to palpation", "Digital rectal examination and rectoscopy were not performed because of the pain and fecal incontinence", "The abdomen was soft and nontender", "Routine blood, liver, and renal function test results were normal, as were results for carcinoembryonic antigen and cancer antigen 19-9. Chest radiography showed no abnormalities", "Computed tomographic scan of the pelvis showed a hypodense 12 × 7 × 3-cm mass extending to the anus, anal canal, and lower perirectal space (Figure 1B)", "Abdominal computed tomography showed no pathology", "Clinical photograph of the giant perianal mass in the left perianal region (A) and abdominal computed tomographic scan showing the mass in the anus (B)"], "s1": [0, 1, 2, 3, 7, 8, 10, 12, 15, 17], "s2": [4, 5, 6, 9, 11, 13, 14, 16]} {"key": 1066, "questions": "What Is Your Diagnosis?", "options": [{"label": "A", "disease": "Hepatocellular carcinoma"}, {"label": "B", "disease": "Fibrolamellar carcinoma"}, {"label": "C", "disease": "Hepatic metastasis"}, {"label": "D", "disease": "Gossypiboma"}], "answer_idx": "D", "symptoms": ["A woman in her 40s was referred to our service owing to unusual findings detected on routine abdominal ultrasonography ordered by her gynecologist", "The ultrasonographic scan showed a hepatic mass in the right lobe composing approximately 50% of the liver parenchyma", "The patient was asymptomatic, and there were no significant findings on abdominal physical examination— the liver was not palpable", "Results of whole blood analyses, including α-fetoprotein, were within the reference ranges", "She had not received any medication, including contraceptive pills or hormones, for more than 10 years", "Nine years before this examination, the patient underwent elective splenectomy and hepatic biopsy at another institution", "However, she did not know the reason for removal of the spleen, and results of the histopathologic examination were not available", "She had undergone a cesarean section at age 25 years", "The patient had no history of trauma or other previous medical events", "An abdominal computed tomographic scan revealed a well-defined, solid, heterogeneous mass, completely intrahepatic, with no enhancement and no liquid component, causing slight displacement of the right hepatic vein (Figure 1)", "Cross-sectional computed tomographic image of the abdomen shows a large, intrahepatic, 20-cm solid mass"], "s1": [0, 4, 5, 6, 7, 8], "s2": [1, 2, 3, 9, 10]} {"key": 1067, "questions": "What Is Your Diagnosis?", "options": [{"label": "A", "disease": "Calciphylaxis"}, {"label": "B", "disease": "Septic emboli"}, {"label": "C", "disease": "Vasculitis"}, {"label": "D", "disease": "Necrotizing fasciitis"}], "answer_idx": "B", "symptoms": ["A woman in her mid-70s with a history of type 2 diabetes mellitus, end-stage renal disease, and coronary artery disease was admitted to the hospital for fever, lethargy, and altered mental status", "On day 7 of her hospital stay, she developed severe, unremitting left ankle pain with purpuric lesions on the skin", "She had previously undergone coronary artery bypass grafting with simultaneous aortic valve replacement and ileal conduit urinary diversion as a result of to recurrent upper urinary tract infections", "Laboratory findings were significant for neutrophil-predominant leukocytosis (leukocyte count, 19 400/μL [to convert to ×109/L, multiply by 0.001])", "Physical examination revealed multiple stellate, purpuric plaques with central flaccid bullae on the medial and lateral aspects of the left ankle (Figure, A)", "The remainder of her skin examination findings were unremarkable", "Punch biopsy specimens were obtained along the border of normal and lesional skin (Figure, B and C), and she underwent transesophageal echocardiography", "A, Stellate purpura with overlying flaccid bulla on the medial aspect of the ankle", "B, Partially necrotic epidermis with intraepidermal bulla formation with marked perivascular infiltrates in the deep dermis and subcutaneous tissue (hematoxylin-eosin, original magnification ×25)", "C, A small blood vessel occluded with fibrinoid material with a perivascular lymphoneutrophilic infiltrate (hematoxylin-eosin, original magnification ×400)"], "s1": [0, 1, 2, 3], "s2": [4, 5, 6, 7, 8, 9]} {"key": 1068, "questions": "What Is Your Diagnosis?", "options": [{"label": "A", "disease": "Discoid lupus erythematosus"}, {"label": "B", "disease": "Erythema dyschromicum perstans"}, {"label": "C", "disease": "Lichen planus actinicus"}, {"label": "D", "disease": "Polymorphous light eruption"}], "answer_idx": "C", "symptoms": ["A man in his 50s of Indian descent presented with a pruritic eruption on his scalp, arms, neck, and feet", "The eruption began during the summer 3 months before presentation", "His medical history was significant for type 2 diabetes mellitus and hypertension, which were managed with metformin, glimepiride, and atenolol", "Physical examination revealed hyperpigmented annular plaques with violaceous borders on his scalp and the superior part of his forehead (Figure, A)", "There were violaceous papules on the dorsal aspect of the hands (Figure, B), extensor surface of the arms, and the nape of the neck", "Physical examination findings", "A, Hyperpigmented annular plaques with violaceous borders on the scalp", "B, Violaceous papules on the dorsal aspect of the hands", "C, Histopathologic analysis of violaceous eruption on the hand", "Lymphocytic infiltrate at the dermoepidermal junction (hematoxylin-eosin, original magnification ×10)", "A punch biopsy specimen was obtained and sent for histopathologic evaluation (Figure, C)"], "s1": [0, 1, 2, 3, 4, 5, 6, 7], "s2": [8, 9, 10]} {"key": 1069, "questions": "What Is Your Diagnosis?", "options": [{"label": "A", "disease": "Pemphigus erythematosus"}, {"label": "B", "disease": "Lupus erythematosus"}, {"label": "C", "disease": "Eczematous dermatitis"}, {"label": "D", "disease": "Pemphigus foliaceus"}], "answer_idx": "A", "symptoms": ["A woman in her 40s with a medical history of pulmonary embolism and deep venous thrombosis presented with a 6-month history of pruritic, pink scaling plaques on her face and neck (Figure, A)", "The lesions subsequently involved her chest, abdomen, and back and flared with sun exposure", "A diagnosis of lupus erythematosus (LE) was favored, but the lesions failed to improve on a regimen that included hydroxychloroquine sulfate, fluocinonide ointment, 0.05%, and tacrolimus ointment, 0.1%", "Biopsy samples were obtained for histopathologic evaluation", "A, This case shows well-demarcated plaques with an overlying scale crust occurring in a photodistributed pattern on the central face", "B, Hematoxylin-eosin, original magnification ×4. C, Hematoxylin-eosin, original magnification ×10.Laboratory findings were significant for positive antinuclear antibody titers at 1:640, lymphopenia, and normocytic anemia", "Test results were negative or within normal limits for levels of antiribonucleoprotein, antihistone, antiphospholipid, anti-Smith, anti-Ro/Sjögren syndrome antigen A, and anti-La/Sjögren syndrome antigen B antibodies, C3, C4, CH50, rapid plasma reagin, and creatinine", "erythrocyte sedimentation rate", "and a comprehensive metabolic profile", "Urinalysis results were within normal limits", "A biopsy specimen was obtained (Figure, B and C)"], "s1": [0, 1, 2, 4, 10], "s2": [3, 5, 6, 7, 8, 9]} {"key": 1070, "questions": "What Is Your Diagnosis?", "options": [{"label": "A", "disease": "Adenovirus"}, {"label": "B", "disease": "Ehrlichiosis"}, {"label": "C", "disease": "Epstein-Barr virus "}, {"label": "D", "disease": "Parvovirus B19"}], "answer_idx": "B", "symptoms": ["A previously healthy 6-year-old boy presented with 6 days of fever (maximum temperature of 40°C), intermittent nonbilious emesis, and 4 days of crampy, diffuse abdominal pain", "Physical examination findings were significant for ill appearance, bilateral nonexudative conjunctivitis, shotty bilateral anterior cervical lymphadenopathy, diffuse abdominal tenderness (greatest in the right upper quadrant), and associated rebound and guarding", "There was also a blanching, erythematous, macular rash on his cheeks (Figure, A) and an intermittent, erythematous, reticular rash on his chest, abdomen, arms, and legs (Figure, B) that spared his palms and soles and flared with fever", "The remainder of his examination findings were unremarkable", "Laboratory studies were significant for an elevated aspartate transaminase level of 264 U/L (reference range, 12-47 U/L", "to convert to microkatals per liter, multiply by 0.0167) and an alanine transaminase level of 251 U/L (reference range, 17-63 U/L", "to convert to microkatals per liter, multiply by 0.0167)", "A complete blood cell count revealed leukopenia with a white blood cell count of 28 300/µL (to convert to ×109/L, multiply by 0.001) (neutrophils, 64%", "lymphocytes, 18%", "monocytes, 4%", "eosinophils, 2%", "basophils, 1%", "and bands, 11%), a low-normal hemoglobin level of 11.4 g/dL (to convert to grams per liter, multiply by 10), and thrombocytopenia with a platelet count of 610 × 103/µL (to convert to ×109/L, multiply by 1)", "Computed tomography of the abdomen revealed a dilated appendix (7 mm) with mucosal hyperemia and an internal appendicolith", "acute appendicitis could not be excluded radiographically, so the patient was admitted for further workup because of a concern for acute abdomen", "The patient lives in the southeastern United States and had no reported history of unusual exposures or insect or tick bites", "A, A blanching, erythematous, macular rash present on the patient’s cheeks", "B, An example of an intermittent, erythematous, reticular rash present on the patient’s left arm"], "s1": [0, 1, 13, 14, 15], "s2": [2, 3, 4, 5, 6, 7, 8, 9, 10, 11, 12, 16, 17]} {"key": 1071, "questions": "What Is Your Diagnosis?", "options": [{"label": "A", "disease": "Parosteal osteosarcoma"}, {"label": "B", "disease": "Peripheral ossifying fibroma"}, {"label": "C", "disease": "Periosteal osteosarcoma"}, {"label": "D", "disease": "Sarcomatoid carcinoma"}], "answer_idx": "B", "symptoms": ["A woman in her 60s was referred for a large, left-sided, progressively enlarging mandibular mass, present for approximately 7 months", "She had no pain or neurosensory disturbances related to the lesion", "however, she did have difficulty tolerating a regular diet owing to the bulk of the mass", "She had no significant medical history, had quit smoking cigarettes 4 year prior to presentation, but had a 20 pack-year smoking history", "Findings from the physical examination were notable for a 4.9-cm exophytic, nodular, slightly mobile mass of the left lower posterior alveolar ridge, involving the buccal and lingual gingiva", "Teeth 20 to 24 were mobile", "She had no cervical lymphadenopathy", "Previous biopsy demonstrated squamous mucosa with ulceration and chronic inflammation", "Contrasted computed tomography revealed an exophytic complex mass with areas of calcification and ossification inferiorly and a necrotic, cystic irregularly enhancing portion superiorly (Figure, A-C)", "An additional biopsy was performed (Figure, D)", "The mass was excised, and teeth 20 to 26 were removed", "The alveolar ridge was recontoured, and the incision was closed primarily", "A-C, Computed tomographic (CT) scans", "A, Bone window, coronal view", "B, Bone window, sagittal view", "C, Soft-tissue window", "D, Histopathologic image of biopsy specimen"], "s1": [0, 1, 2, 3, 4, 5, 6, 7], "s2": [8, 9, 10, 11, 12, 13, 14, 15, 16]} {"key": 1072, "questions": "What Is Your Diagnosis?", "options": [{"label": "A", "disease": "Squamous cell carcinoma "}, {"label": "B", "disease": "Mucoepidermoid carcinoma"}, {"label": "C", "disease": "Acinic cell carcinoma "}, {"label": "D", "disease": "Adenoid cystic carcinoma"}], "answer_idx": "B", "symptoms": ["A woman in her 40s with no significant medical history presented with a 4-year history of intermittent left-sided throat pain, which eventually progressed to left ear pain approximately 1 year prior to her presentation", "She had not experienced any dysphagia, dysphonia, dyspnea, fevers, or weight loss", "She had no significant use of tobacco or alcohol in the past", "Findings from an otoscopic examination was unremarkable", "Nasopharyngoscopy was performed and failed to reveal any masses, ulcerations, or mucosal lesions", "A magnetic resonance image (MRI) revealed an enhancing 1 × 6-mm lesion in the left side of the posterolateral tongue base (Figure, A)", "A biopsy of the mass revealed an intermediate-grade infiltrating neoplasm forming solid sheets and glandular structures (Figure, B)", "The tumor cells stained positive for CK7 and p63, and negative for CK20, p16, and S-100 by immunohistochemical analysis", "Stain for mucicarmine was positive in glandular areas (Figure, C)", "After establishing the diagnosis, the patient was taken to the operating room for a left base of tongue resection via suprahyoid approach along with a left-sided neck dissection of levels I to IV", "The tumor was removed with negative margins, and no positive cervical lymph nodes were found on final pathologic analysis", "A, Magnetic resonance (MRI) image of a lesion in the left side of the posterolateral tongue base", "B and C, Histopathologic images", "B, Intermediate-grade infiltrating neoplasm forming solid sheets and glandular structures", "Hematoxylin-eosin, original magnification ×20. C, Mucicarmine stain in the glandular area, original magnification ×40."], "s1": [0, 1, 2, 3, 4], "s2": [5, 6, 7, 8, 9, 10, 11, 12, 13, 14]} {"key": 1073, "questions": "What Is Your Diagnosis?", "options": [{"label": "A", "disease": "Viral infection"}, {"label": "B", "disease": "Malignant neoplasm"}, {"label": "C", "disease": "Pemphigus vulgaris"}, {"label": "D", "disease": "Invasive necrotizing fungal gingivitis"}], "answer_idx": "D", "symptoms": ["A woman in her 70s with a 7-year history of essential thrombocythemia was diagnosed as having acute myelogenous leukemia 4 days prior to hospital admission for induction chemotherapy", "She was febrile at admission and treated empirically with antifungal, antibiotic, and antiviral medications in the setting of cultures that were negative for organisms", "The initial chemotherapy regimen failed, requiring a more aggressive regimen, which left her pancytopenic", "During the course of the second chemotherapy treatment, she developed left-sided mandibular pain", "A computed tomographic scan revealed cellulitic changes along the buccal cortex of the mandible and a small 2 × 12-mm abscess formation", "At initial evaluation by the otolaryngology service on hospital day 32, she had decreased sensation at the left mandibular branch of the trigeminal nerve and ecchymoses along the mandibular buccal-gingival borders bilaterally with induration of the left floor of mouth and buccal mucosa (Figure, A)", "She continued to receive broad spectrum antimicrobial and antifungal coverage", "By hospital day 38, the patient had developed evidence of ischemia and necrosis along the gingival and lingual aspects of the left mandibular alveolar ridge, extending past midline to involve mucosa on the right side", "Magnetic resonance imaging revealed a 7 × 10 × 35-mm abscess involving both gingival and lingual aspects of the left anterior mandible (arrowheads) (Figure, B)", "Biopsy specimens of the right mandibular gingiva are shown in the Figure, C and D", "A, Patient photograph", "B, Magnetic resonance image of abscess", "C and D, Histopathologic images from biopsy specimens", "C, Original magnification ×250. D, Original magnification ×500."], "s1": [0, 1, 2, 3, 6, 7], "s2": [4, 5, 8, 9, 10, 11, 12, 13]} {"key": 1074, "questions": "What Is Your Diagnosis?", "options": [{"label": "A", "disease": "Extraskeletal chondrosarcoma"}, {"label": "B", "disease": "Malignant fibrous histiocytoma"}, {"label": "C", "disease": "Ancient schwannoma"}, {"label": "D", "disease": "Synovial sarcoma"}], "answer_idx": "C", "symptoms": ["A man in his 20s presented with a painless, gradually increasing swelling in the right side of the neck of 6 years’ duration", "There was no history of dysphagia, hoarseness of voice, dyspnea, or constitutional symptoms, such as fever, weight loss, or night sweats", "Clinical examination revealed a 7 × 6-cm, well-defined, firm, nontender swelling extending from the angle of the mandible to the clavicle", "Oropharyngeal examination revealed a mucosal bulge on the lateral pharyngeal wall with normal overlying mucosa", "A computed tomographic (CT) scan of the neck was performed, which revealed a solid 7 × 8 × 10-cm right carotid space mass, with a few cystic areas and coarse specks of calcification within (Figure, A, yellow arrowhead)", "It showed heterogeneous postcontrast enhancement, predominantly around the calcific foci", "It extended from the C2 to D1 vertebral level and displaced the right common carotid artery and internal jugular vein anterolaterally (Figure, A, red arrowhead)", "Fat planes around the mass were well maintained (Figure, B, arrowheads), and there was no bone destruction", "No cervical lymphadenopathy was noted", "A biopsy was performed from the mass (Figure, C and D)", "A, Axial postcontrast computed tomographic (CT) image of the neck", "B, Coronal postcontrast CT image of the neck", "C and D, Histopathologic images", "C, Hematoxylin-eosin, original magnification ×200. D, Hematoxylin-eosin, original magnification ×400."], "s1": [0, 1, 2, 3, 4, 5, 6, 7, 8], "s2": [9, 10, 11, 12, 13]} {"key": 1075, "questions": "What Is Your Diagnosis?", "options": [{"label": "A", "disease": "Endogenous endophthalmitis"}, {"label": "B", "disease": "HLA-B27 uveitis"}, {"label": "C", "disease": "Herpes simplex uveitis"}, {"label": "D", "disease": "Leukemia relapse"}], "answer_idx": "D", "symptoms": ["A man in his 70s presented with blurred vision in his left eye", "The patient had a history of myelodysplastic syndrome (refractory anemia with excess blasts–2), prostate cancer treated with prostatectomy and radiation therapy 5 years prior, and colonic adenocarcinoma treated with right hemicolectomy 6 years prior", "Four months before presentation, his myelodysplastic syndrome transformed into acute myeloid leukemia (AML)", "He was randomized to experimental drug CPX-351 for induction, but treatment was discontinued as a result of renal failure", "However, remission was successfully induced with mitoxantrone hydrochloride and etoposide therapy, as confirmed by bone marrow biopsy, and he was prescribed continuing melphalan therapy on an outpatient basis", "Three days prior to presentation, the patient on awakening noticed an acute onset of blurred vision in his left eye without any pain, photophobia, diplopia, vision loss, or floaters", "He had no history of ocular surgery or trauma", "Visual acuity was 20/20 in the right eye and 20/30 in the left", "External examination had unremarkable results", "Pupillary and motility assessments had normal results", "Intraocular pressure was elevated bilaterally, measuring 29 mm Hg in the right eye and 41 mm Hg in the left", "Slitlamp examination of the right eye revealed a 0.5-mm hypopyon with small hemorrhages on the iris surface (Figure 1A)", "In the left eye, examination demonstrated a 1.5-mm hypopyon mixed with hemorrhage, 1+ cells in the anterior chamber, and small hemorrhages on the iris surface (Figure 1B)", "Bilaterally, gonioscopy revealed open angles with few hemorrhages in the angle but no neovascularization", "Dilated fundoscopic examination had unremarkable results bilaterally, with no retinal hemorrhages or optic disc swelling", "Color slitlamp photographs illustrating bilateral hypopyon (black arrowheads) with iris surface hemorrhages (white arrowheads)", "A, Right eye with 0.5-mm hypopyon with small hemorrhages on the iris surface and presumed infiltration", "B, Left eye with 1.5-mm hypopyon mixed with hemorrhage and small iris surface hemorrhages and presumed infiltration"], "s1": [1, 2, 3, 4], "s2": [0, 5, 6, 7, 8, 9, 10, 11, 12, 13, 14, 15, 16, 17]} {"key": 1076, "questions": "What Is Your Diagnosis?", "options": [{"label": "A", "disease": "Choledochoduodenal fistula"}, {"label": "B", "disease": "Biloma"}, {"label": "C", "disease": "Gallbladder duplication with double cystic duct"}, {"label": "D", "disease": "Choledochal cyst and long common biliopancreatic channel"}], "answer_idx": "D", "symptoms": ["A 57-year-old woman with a medical history of recurrent biliary colic was admitted with symptoms consistent with mild gallstone pancreatitis", "Routine laboratory test results showed total bilirubin of 4.74 mg/dL (to convert to millimoles per liter, multiply by 17.104) and amylase of 1.041 U/L (to convert to microkatals per liter, multiply by 0.0167)", "An abdominal ultrasonography scan demonstrated gallstones and a mildly dilated common bile duct (CBD)", "Magnetic resonance imaging of the abdomen confirmed these results and revealed cholelithiasis", "The patient consequently underwent urgent laparoscopic cholecystectomy with intraoperative cholangiography (Figure, A)", "Intraoperative cholangiography revealed an unusual anatomical variant and subsequent transcystic choledochoscopy was performed, which demonstrated a double lumen at the distal CBD (Figure, B)", "Multiple gallstones were found within a dilated region of cystic duct"], "s1": [0, 1, 2, 3], "s2": [4, 5, 6]} {"key": 1077, "questions": "What Is Your Diagnosis?", "options": [{"label": "A", "disease": "Mycotic aneurysm"}, {"label": "B", "disease": "Inflammatory abdominal aortic aneurysm"}, {"label": "C", "disease": "Pseudoaneurysm"}, {"label": "D", "disease": "Infrarenal aortitis due to diverticulitis"}], "answer_idx": "B", "symptoms": ["An elderly man presented to the emergency department with a 1-month history of new-onset, persistent lower abdominal and flank pain", "The pain was sharp and constant with intermittent radiation to the back and associated anorexia with a 10-kg weight loss", "He did not report any previous abdominal surgery", "The patient stated that he had not had any procedures with intravascular instrumentation in the past year, dental cleanings, intravenous drug use, cutaneous abscesses, or recent trauma", "Physical examination revealed he was afebrile and in normal sinus rhythm", "there was minimal epigastric abdominal tenderness without signs of peritonitis and no palpable masses", "Femoral pulses were palpable and symmetric", "Laboratory examination revealed mild anemia (hematocrit, 35% [to convert to a proportion of 1.0, multiply by 0.01]), no leukocytosis, and an elevated erythrocyte sedimentation rate (71 mm/h)", "Blood and urine cultures were negative for organisms", "A computed tomographic (CT) angiogram showed asymmetric attenuation surrounding the infrarenal aorta (Figure 1)", "On comparison, a CT scan that was performed 6 months before presentation had identified no aneurysmal disease", "Computed tomographic scan with arterial phase intravenous contrast with asymmetric attenuation surrounding the infrarenal aorta"], "s1": [0, 1, 7, 8, 9, 10, 11], "s2": [2, 3, 4, 5, 6]} {"key": 1078, "questions": "What Is Your Diagnosis?", "options": [{"label": "A", "disease": "Artifactual peripilar hair casts"}, {"label": "B", "disease": "Trichomycosis axillaris"}, {"label": "C", "disease": "Pediculosis axillaris"}, {"label": "D", "disease": "White piedra"}], "answer_idx": "B", "symptoms": ["A man in his 20s was referred to our department because of the presence of a sticky material around the underarm hair for approximately 1 year", "The patient explained that he usually plays soccer and sweats abundantly", "He did not apply deodorant or any other products in the underarm, and he ensured proper hygiene of the area", "The lesions did not produce itching", "Physical examination revealed creamy yellow concretions along several hair shafts, in both armpits, with strong axillary odor (Figure, A)", "These concretions did not disappear when cleansed with alcohol", "Hair samples of the affected areas were sent for microscopy examination (Figure, B-D) and microbiologic culture", "Treatment with antiseptic soap and a miconazole nitrate cream was initiated", "After 2 weeks of treatment, the lesions had resolved and the patient had normal axillary hairs", "Confirmation of the suspected diagnosis was made with the results of complementary tests", "A, Clinical photograph of axillary hairs shows creamy yellow concretions along several axillary hairs", "B, Direct microscopic examination of the hair shaft (original magnification ×40)", "C, Periodic acid–Schiff stain (original magnification ×400)", "D, Grocott silver stain (original magnification ×400)"], "s1": [0, 1, 2, 3, 4, 5, 7, 8], "s2": [6, 9, 10, 11, 12, 13]} {"key": 1079, "questions": "What Is Your Diagnosis?", "options": [{"label": "A", "disease": "Eruptive vellus hair cysts"}, {"label": "B", "disease": "Urticaria pigmentosa"}, {"label": "C", "disease": "Eruptive syringoma"}, {"label": "D", "disease": "Lichen nitidus"}], "answer_idx": "C", "symptoms": ["A healthy teenaged boy was seen with a papular eruption on the trunk and legs of 3 months’ duration", "He had initially developed lesions on his legs, which then spread to his neck, chest, and buttocks", "This eruption was asymptomatic, and the patient reported no history of vesiculation", "His mother noted minimal evolution of the lesions, except for recent flattening of those on the legs", "Physical examination showed numerous discrete (1 to 3 mm) smooth brown papules concentrated on the neck, chest, anterior and lateral thighs, and buttocks (Figure 1)", "Several lesions had a positive Darier sign (Figure 1A)", "A, Multiple small brown papules of the chest", "Localized erythema developed around several lesions after firm stroking", "B, Scattered small brown papules of the lateral thigh", "There was no family history of similar lesions", "A punch biopsy of a papule on the left thigh was performed, and the specimen was submitted for pathological review"], "s1": [0, 1, 2, 3, 9], "s2": [4, 5, 6, 7, 8, 10]} {"key": 1080, "questions": "What Is Your Diagnosis?", "options": [{"label": "A", "disease": "Postinflammatory hyperpigmentation"}, {"label": "B", "disease": "Argyria"}, {"label": "C", "disease": "Chrysiasis"}, {"label": "D", "disease": "Tattoo pigmentation"}], "answer_idx": "C", "symptoms": ["A woman in her 50s presented with diffuse blue to slate-gray patches on her entire face immediately after administration of Q-switched Nd:YAG laser toning (Figure 1A)", "These pigmented lesions corresponded exactly to areas that had previously received laser treatment", "Her history was remarkable for rheumatoid arthritis treated with oral gold salts 15 years ago, but the total dosage of gold salts could not be determined", "Rechallenge with the original Q-switched Nd:YAG laser was administered on the right forearm, and a similar slate-gray macule developed immediately (Figure 1B)", "A skin biopsy specimen of the lesion on the right side of the woman’s face was obtained, and histopathologic evaluation was conducted", "A, Diffuse blue to slate-gray patches developed on the face, corresponding exactly to areas that had previously received laser treatment", "B, A rechallenge test administered on the right forearm with the original Q-switched Nd:YAG laser yielded a similar slate-gray macule (arrowhead)"], "s1": [0, 1, 3, 5, 6], "s2": [2, 4]} {"key": 1081, "questions": "What Is Your Diagnosis?", "options": [{"label": "A", "disease": "Acrodermatitis enteropathica"}, {"label": "B", "disease": "Ulcerated segmental infantile hemangioma"}, {"label": "C", "disease": "Candidal dermatitis"}, {"label": "D", "disease": "Jacquet erosive diaper dermatitis"}], "answer_idx": "B", "symptoms": ["A 7-week-old full-term girl was referred to our clinic for persistent and expanding diaper dermatitis present since 1 week of age", "She was initially treated at an outside hospital with intravenous clindamycin hydrochloride for presumed perianal cellulitis and was discharged with a regimen of oral clindamycin hydrochloride", "When the patient was aged 4 weeks, the lesion became ulcerated and she was admitted to our hospital", "She was given intravenous antibiotics and mupirocin, 2%, ointment, triamcinolone, 0.1%, ointment, clotrimazole, 1%, cream, nystatin cream, and zinc oxide paste", "Results of cultures for bacteria and herpes simplex virus were negative", "The patient had no systemic symptoms and was discharged and scheduled for an outpatient dermatology appointment", "Physical examination showed a well-nourished, well-developed infant whose gluteal region had well-demarcated ulcers with scalloped borders on either side of the midline, superior to the anal verge", "There was no exudate or drainage", "The base of the ulcers was covered with fibrinous material and the residue of topical medications", "Bright red matted telangiectasias extended from the anus to the ulcers (Figure)", "Two well-demarcated ulcers covered with white fibrinous material", "The ulcerations are superimposed on prominent red matted telangiectasias"], "s1": [0, 1, 2, 3, 4, 5], "s2": [6, 7, 8, 9, 10, 11]} {"key": 1082, "questions": "What Is Your Diagnosis?", "options": [{"label": "A", "disease": "Adenoid cystic carcinoma"}, {"label": "B", "disease": "Salivary duct carcinoma"}, {"label": "C", "disease": "Carcinoma ex pleomorphic adenoma"}, {"label": "D", "disease": "Metastatic breast carcinoma"}], "answer_idx": "D", "symptoms": ["A woman in her 60s presented to her primary care physician with a right-sided lower motor neuron facial nerve palsy", "This resolved with oral steroids and antiviral drugs consistent with a presumed diagnosis of idiopathic facial nerve (Bell) palsy", "Six months later the patient developed discomfort around her right ear, prompting a referral to the local otolaryngology service", "Her medical history was significant for localized invasive ductal carcinoma of the right breast treated with wide local excision 13 years previously", "At presentation she was considered disease-free", "Findings from the clinical examination, including otoscopy, were unremarkable", "A magnetic resonance imaging (MRI) scan revealed an 8-mm lesion within the right parotid gland with a spiculated border and ill-defined surrounding enhancement extending close to the stylomastoid foramen (Figure, A)", "This was of low T2 signal with no perineural spread evident", "A fine-needle aspirate and core biopsy were obtained under ultrasonographic guidance", "The cytologic findings showed loosely cohesive groups of atypical epithelial cells", "Histologic examination of the core biopsy revealed an infiltrative lesion composed of cords and nests of medium-sized cells surrounded by a densely hyalinized stroma (Figure, B)", "The cells contained hyperchromatic vesicular nuclei with moderate amounts of eosinophilic cytoplasm (Figure, C)", "Immunohistochemical analysis showed the cells were diffusely positive for cytokeratin 7 (CK7), estrogen receptor (ER), progesterone receptor (PR), and androgen receptor (Figure, D)", "A, Axial fast spoiled gradient echo gadolinium-enhanced magnetic resonance imaging (MRI) scan with fat suppression demonstrating lesion of the right parotid gland (arrowhead)", "B, Right parotid core biopsy specimen", "C, Cells with hyperchromatic vesicular nuclei and moderate amounts of eosinophilic cytoplasm", "D, Immunohistochemical (IHC) analysis for estrogen receptor"], "s1": [0, 1, 2, 4, 5, 7], "s2": [3, 6, 8, 9, 10, 11, 12, 13, 14, 15, 16]} {"key": 1083, "questions": "What Is Your Diagnosis?", "options": [{"label": "A", "disease": "Spitz nevus"}, {"label": "B", "disease": "Pyogenic granuloma"}, {"label": "C", "disease": "Juvenile xanthogranuloma"}, {"label": "D", "disease": "Dermatofibroma"}], "answer_idx": "C", "symptoms": ["A male infant presented to the pediatric otolaryngology office for evaluation of a left postauricular mass", "Initially the lesion appeared to be acutely inflamed with erythema, overlying crusting, and tenderness to palpation", "The child was treated with a course of antibiotics", "Four weeks after the initial evaluation, the acute inflammation had resolved, but the lesion had increased in size", "The child was otherwise healthy, and the parents stated that he had no systemic symptoms such as fever or weight loss", "On physical examination, a 1.5-cm superficial, ovoid firm lesion was noted in the left postauricular area (Figure, A)", "While the lesion was attached to the skin, it was mobile over the mastoid bone", "There were no middle ear abnormalities on physical examination", "Ultrasonography of the lesion revealed a nonvascular, well-circumscribed 1.7 × 1.4 × 0.6-cm lesion in the soft tissue posterior to the left ear", "It demonstrated a heterogenous echotexture but was predominantly hypoechoic without internal vascularity or distinct calcifications", "Excisional biopsy under general anesthesia was performed owing to rapid enlargement and unclear etiology of the lesion", "The lesion had distinct borders and separated easily from the surrounding soft-tissue attachments", "It was completely excised, including the overlying skin", "Pathologic examination revealed a neoplasm extending from the reticular dermis into the subcutaneous adipose tissue (Figure, B)", "There were lobules of lipidized histiocytic cells with eosinophils in a background of collagen (Figure, C)", "The lesion stained negatively for S-100 and CD1a (Figure, D)", "A, Gross photograph of left postauricular mass", "B-D, Histopathologic images (hematoxylin-eosin)", "B, High-powered (original magnification ×100) view depicting some pale areas representing lipidized cells", "C, High-powered (original magnification ×400) view demonstrating pale, foamy cells (xanthoma cells)", "D, Neoplastic cells staining negatively for S100 (original magnification ×400)"], "s1": [0, 1, 2, 3, 4, 5, 6, 7, 8, 9, 10, 11, 12], "s2": [13, 14, 15, 16, 17, 18, 19, 20]} {"key": 1084, "questions": "What Is Your Diagnosis?", "options": [{"label": "A", "disease": "Ameloblastoma"}, {"label": "B", "disease": "Central mucoepidermoid carcinoma"}, {"label": "C", "disease": "Ameloblastic carcinoma"}, {"label": "D", "disease": "Odontogenic myxoma"}], "answer_idx": "C", "symptoms": ["A man in his 60s presented to the oral and maxillofacial surgery department with a 7-month history of a painless progressive swelling of the anterior mandible", "The patient was completely edentulous", "He reported numbness of the lower lip and difficulty on swallowing", "There was no remarkable weight loss", "His medical history was significant for hypertension and coronary artery bypass surgery", "On intraoral examination, the mass was hard in consistency with relatively normal overlying mucosa", "Computed tomographic (CT) images with bone window showed a grossly expansile, multilocular, low-attenuation lesion of the anterior segment of the mandible (Figure, A and B)", "The images demonstrated severe erosion of the facial and lingual cortices with a remarkable scalloping pattern, and multiple wispy, ill-defined intralesional septa", "Soft-tissue CT images indicated a cystic and solid nature of the mass (Figure, C and D)"], "s1": [4, 3, 1], "s2": [0, 2, 5, 6, 7, 8]} {"key": 1085, "questions": "What Is Your Diagnosis?", "options": [{"label": "A", "disease": "Lipoma"}, {"label": "B", "disease": "Sarcoidosis"}, {"label": "C", "disease": "Localized laryngeal amyloidosis"}, {"label": "D", "disease": "Gouty tophi"}], "answer_idx": "C", "symptoms": ["A woman in her 60s presented with a history of persistent globus pharyngis spanning 2 decades", "She stated that she had no other symptoms, and her family medical history was not remarkable", "On examination, the rigid indirect laryngoscopy revealed yellowish floppy granular proliferation affecting the bilateral arytenoids and posterior aspects of the aryepiglottic folds, with symmetrical vocal cord motion (Figure, A)", "The main lesions were clearly demarcated from the surrounding normal mucosa", "The patient then underwent microlaryngeal surgery with biopsy under general anesthesia", "The nidus was fragile and bled copiously when traumatized", "The specimens mounted on the hematoxylin-eosin slides showed eosinophilic amorphous material underlying the epithelium (Figure, B)", "Staining for thioflavin T revealed yellow-green fluorescence (Figure, C)", "The fibrillar component emitted a characteristic apple-green birefringence under polarized light after binding to Congo red dye", "No clinical evidence of systemic involvement and/or plasma cell dyscrasia was found", "A, Endoscopic examination demonstrated symmetric yellowish lesions affecting the arytenoids and posterior aspects of the aryepiglottic folds, with a patent airway and mobile vocal folds", "B, Eosinophilic amorphous material underlying the epithelium (hematoxylin-eosin, original magnification ×200)", "C, Thioflavin T exhibited a typical yellow-green fluorescence after binding to amyloid fibrils (original magnification ×200)"], "s1": [0, 1, 3, 4, 5, 9], "s2": [2, 6, 7, 8, 10, 11, 12]} {"key": 1086, "questions": "What Is Your Diagnosis?", "options": [{"label": "A", "disease": "Ranula"}, {"label": "B", "disease": "Lymphangioma"}, {"label": "C", "disease": "Dermoid cyst"}, {"label": "D", "disease": "Suprahyoid thyroglossal duct cyst"}], "answer_idx": "C", "symptoms": ["An adolescent female with a medical history significant for absent corpus callosum, polycystic ovarian syndrome, and papillary thyroid cancer status post–total thyroidectomy and I-131 ablation, presented with a 1-year history of submental swelling", "She had no associated symptoms", "On examination, she had a 6-cm nontender mass in the submentum", "Her floor of mouth (FOM) was soft, and there was no cervical lymphadenopathy", "Attempted open biopsy at the time of thyroidectomy revealed only adipose tissue", "Ultrasonography (US) (Figure, A) revealed a well-circumscribed, avascular, homogeneous, isoechoic lesion, measuring 6.2 × 6.0 × 6.0 cm along the FOM and left submandibular regions", "Magnetic resonance imaging (MRI) of the neck revealed the lesion within the FOM, above the mylohyoid muscle", "The lesion involved the root of the tongue in the midline and extended to the left, causing significant thinning of the left mylohyoid muscle without extension through it", "The lesion demonstrated a fluid signal with T2 hyperintensity (Figure B), T1 hypointensity (Figure, C), and peripheral rim enhancement", "Mild restricted diffusion was also noted", "The patient underwent transcervical excision of the mass (Figure, D)", "The mass was encapsulated and positioned above the mylohyoid muscle and extended into the FOM between bilateral genioglossus muscles", "The mass was removed entirely via transcervical excision", "A, Sagittal midline ultrasonography shows a floor of mouth lesion with uniform low-level echoes", "B, Axial T2-weighted magnetic resonance image (MRI) revealing a hyperintense lesion involving the floor of mouth", "C, Coronal postcontrast MRI revealing a T1 hypointense unilocular lesion involving the floor of the mouth and root of the tongue", "D, Intraoperative photograph during mass excision"], "s1": [0, 1, 2, 3, 4], "s2": [5, 6, 7, 8, 9, 10, 11, 12, 13, 14, 15, 16]} {"key": 1087, "questions": "What Is Your Diagnosis?", "options": [{"label": "A", "disease": "Plantar warts"}, {"label": "B", "disease": "Shingles"}, {"label": "C", "disease": "Cutaneous lymphoma"}, {"label": "D", "disease": "Toxic erythema of chemotherapy"}], "answer_idx": "B", "symptoms": ["A 50-year-old man with transformed marginal zone lymphoma presented with intensely painful lesions on his left foot for 2 days", "He noted a childhood history of plantar warts and believed these lesions to be identical in appearance", "He did not recall any injury or unusual exposure", "He had received his third cycle of R-CHOP (rituximab, cyclophosphamide, doxorubicin hydrochloride, vincristine sulfate, prednisone) 2 weeks prior to presentation as well as an injection of granulocyte-colony stimulating factor", "He was anticipating his fourth cycle of treatment the day of his presentation", "He also complained of a pruritic rash on his left thigh that had started several days before the appearance of his plantar foot lesions", "The patient was afebrile, and his vital signs were within normal limits", "Physical examination revealed grouped, brown, flat-topped papules on his distal plantar left foot (Figure)", "It was exquisitely tender to palpation, and the patient was visibly uncomfortable when walking", "He also demonstrated erythematous papules and plaques on his posterolateral thigh", "A few areas had shallow erosions", "Laboratory studies revealed pancytopenia, with a white blood cell count of 2800/μL, hemoglobin level of 11.6 g/dL, and a platelet count of 81 × 103/μL", "His lactate dehydrogenase level was elevated at 914 U/L, and results of his comprehensive metabolic panel were unremarkable", "(To convert white blood cell count to ×109/L, multiply by 0.001", "to convert hemoglobin to grams per liter, multiply by 10", "to convert lactate dehydrogenase to microkatals per liter, multiply by 0.0167.)Photographs of a patient with painful lesions on the foot"], "s1": [0, 1, 2, 5, 7, 8, 9, 10, 15], "s2": [3, 4, 6, 11, 12, 13, 14]} {"key": 1088, "questions": "What Is Your Diagnosis?", "options": [{"label": "A", "disease": "Gastrointestinal stromal tumor"}, {"label": "B", "disease": "Gastric cancer"}, {"label": "C", "disease": "Peptic ulcer disease"}, {"label": "D", "disease": "Ectopic pancreas"}], "answer_idx": "D", "symptoms": ["A 52-year-old woman with a history of diabetes mellitus and cigarette smoking who had a prior laparoscopic cholecystectomy was referred to our institution for severe postprandial epigastric abdominal pain that had been ongoing for several months and unintentional weight loss", "She had been prescribed total parenteral nutrition prior to referral", "On presentation, her physical examination was remarkable for mild epigastric tenderness", "The results of laboratory testing, including a complete blood cell count, chemistry values, liver function tests, and amylase and lipase levels, were unremarkable", "Computed tomography demonstrated focal gastric thickening", "Esophagogastroduodenoscopy with endoscopic ultrasonography was performed that confirmed a submucosal gastric mass (Figure, A)", "A fine-needle aspiration biopsy revealed benign and reactive glandular cells and debris", "Magnetic resonance cholangiopancreatography demonstrated a 2.4-cm hypervascular and exophytic mass in the gastric antrum and locoregional lymphadenopathy (Figure, B)", "A, Endoscopic ultrasonography of the gastric antrum mass reveals an area of gastric thickening (arrowheads) corresponding to computed tomographic findings", "During endoscopy, this area was visualized grossly as a bulge in the atrium along the incisura", "A fine-needle aspiration biopsy was nondiagnostic, showing gastric antral-type mucosa without pathologic changes (inset)", "B, The coronal image in T2 phase reveals a 1.8 × 2.4 × 1.5-cm hypotense lesion (arrowhead) in the gastric antrum", "The lesion was also enhanced on T1 arterial phase sequencing"], "s1": [0, 1, 2, 3], "s2": [4, 5, 6, 7, 8, 9, 10, 11, 12]} {"key": 1089, "questions": "What Is Your Diagnosis?", "options": [{"label": "A", "disease": "Schwannoma"}, {"label": "B", "disease": "Pseudoaneurysm"}, {"label": "C", "disease": "Cavernous hemangioma"}, {"label": "D", "disease": "Paraganglioma"}], "answer_idx": "C", "symptoms": ["A woman in her mid-50s presented to the clinic with a 6-month history of progressive swelling and pain on the right side of her neck", "The patient described a history of sleep apnea, a remote transient ischemic attack with left-sided weakness, and a prior thyroidectomy", "She had increasing fatigue during the last 6 months, as well as worsening anxiety", "On physical examination, the neck was supple, with no palpable masses or lymphadenopathy, and had a well-healed transverse neck incision", "Magnetic resonance imaging revealed a 1.1-cm, round, partially enhancing, cystic mass near the tail of the parotid gland that appeared hypointense on T2-weighted imaging (Figure 1)", "An enlarged and distended venous structure that measured up to 7 mm was also visualized in the area", "Laboratory work revealed no abnormalities", "Magnetic resonance images of the parotid gland", "T1-weighted image (A) and T2-weighted image (B) revealing a round, partially enhancing, cystic mass near the tail of the parotid gland (white and red arrowheads)"], "s1": [0, 1, 2, 3], "s2": [4, 5, 6, 7, 8]} {"key": 1090, "questions": "What Is Your Diagnosis?", "options": [{"label": "A", "disease": "Sweet syndrome"}, {"label": "B", "disease": "Iododerma"}, {"label": "C", "disease": "Sporotrichosis"}, {"label": "D", "disease": "Pemphigus vegetans"}], "answer_idx": "B", "symptoms": ["A 72-year-old man with a medical history of atrial fibrillation, type 2 diabetes mellitus, chronic kidney disease, hypertension, and congestive heart failure presented with a 2-month history of scaly erythematous papules on both forearms (Figure, A)", "The lesions were pruritic at times and the patient reported no improvement with chlorhexidine wash or hydrocortisone, 1%, cream", "He was otherwise asymptomatic, and review of his systems revealed no abnormalities", "His medications included amiodarone, furosemide, nifedipine, insulin glargine, and allopurinol", "He denied preceding trauma, recent travel, or new medications", "A, Red, scaly papules on the patient's forearm", "B, Punch biopsy specimen from the center of a papule on the forearm shows pseudoepitheliomatous hyperplasia, spongiosis with vesiculation, intraepidermal neutrophilic microabscesses, focal subepidermal bulla formation, and dermal infiltrate of neutrophils, lymphocytes, and histiocytes (hematoxylin-eosin, original magnification ×4)", "Physical examination revealed numerous red to violaceous scaly papules symmetrically distributed on both forearms", "Laboratory tests revealed a serum creatinine level of 3.79 mg/dL (normal range, 0.70-1.40 mg/dL) and serum iodine level of 42 305 ng/mL (normal range, 40-92 ng/mL)", "A punch biopsy was obtained from the center of a papule on the right forearm (Figure, B)"], "s1": [0, 1, 4, 5, 6, 7, 9], "s2": [2, 3, 8]} {"key": 1091, "questions": "What Is Your Diagnosis?", "options": [{"label": "A", "disease": "Nocardia brasiliensis skin infection"}, {"label": "B", "disease": "Staphylococcus aureus skin infection"}, {"label": "C", "disease": "Mycobacterium marinum skin infection"}, {"label": "D", "disease": "Sporotrichosis schenckii skin infection"}], "answer_idx": "A", "symptoms": ["A woman in her 50s admitted for bacterial pneumonia was noted to have an ulcerated plaque on the forearm", "The patient had been discharged approximately 2 weeks prior, after an admission for congestive heart failure", "A few days after that discharge, the patient noticed a painful ulceration on the right distal forearm, at the site where a peripheral intravenous catheter had been inserted during her initial hospitalization", "On physical examination, the patient had an ulcerated plaque on the right forearm with 2 tender, 1- to 2-cm red nodules in the right antecubital fossa (Figure, A)", "On further questioning, the patient noted that between her hospitalizations she had gone home and cleaned her fish tank, played with her pet cat, and gardened in her indoor cactus garden", "She said she had not recently traveled, and was not taking any immunosuppressive medications", "A skin biopsy and culture were performed from the forearm lesion (Figure, B and C)", "A, An ulcerated plaque on the right forearm", "Note 2 tender, 1- to 2-cm red nodules in the right antecubital fossa", "B, Dense neutrophilic infiltrate", "Note associated tissue necrosis", "Hematoxylin-eosin, original magnification ×20. C, Organisms seen on Gram stain", "Original magnification ×60."], "s1": [0, 1, 2, 3, 4, 5], "s2": [6, 7, 8, 9, 10, 11, 12]} {"key": 1092, "questions": "What Is Your Diagnosis?", "options": [{"label": "A", "disease": "Sarcoidosis"}, {"label": "B", "disease": "Axillary acne agminata"}, {"label": "C", "disease": "Miliary tuberculosis"}, {"label": "D", "disease": "Talc granuloma"}], "answer_idx": "B", "symptoms": ["A woman in her early 40s presented with a 6-month history of rash in the bilateral axillae", "The areas involved were not itchy or tender but bothered her cosmetically", "The patient reported using an aluminum-containing antiperspirant spray for years before the onset of the eruption but had since discontinued use of deodorants or antiperspirants without improvement in her lesions", "Her medical history was significant only for epilepsy, and her daily medications included lamotrigine and cholecalciferol", "Physical examination revealed multiple red-brown to purplish follicular papules and nodules in bilateral axillae with focal areas of scarring (Figure 1)", "No facial lesions were noted", "Clinical examination findings", "Multiple red-brown papules within the axilla bilaterally", "She did not improve with multiple medications, including topical antifungals, topical corticosteroids, oral ketoconazole, topical dapsone, doxycycline (40 mg/d), minocycline (100 mg twice daily), a combination of amoxicillin and clavulanate, and intralesional triamcinolone acetonide", "A 4-mm punch biopsy was performed on the right axilla for evaluation"], "s1": [0, 1, 2, 4, 7], "s2": [3, 5, 6, 8, 9]} {"key": 1093, "questions": "What Is Your Diagnosis?", "options": [{"label": "A", "disease": "Acute parotitis or parotid abscess"}, {"label": "B", "disease": "Venous malformation"}, {"label": "C", "disease": "Plexiform neurofibroma"}, {"label": "D", "disease": "Infantile hemangioma"}], "answer_idx": "C", "symptoms": ["A 9-month-old boy presented with a left-sided facial mass", "His perinatal history was notable for macrosomia, macrocephaly, left-sided failure on newborn hearing screen, and a congenital hypertrichotic patch of the left scalp", "During the first few months, he was observed to frequently tug at his left ear while displaying apparent discomfort and sleeping difficulties", "He was treated empirically with multiple antibiotics for presumed otitis media and subsequently had bilateral myringotomy tubes placed", "When the left side of his face became more swollen, magnetic resonance imaging was performed at 8 months of age and revealed a left-sided parotid mass", "He was referred to our institution for further evaluation", "Additional magnetic resonance imaging and magnetic resonance angiography of the brain and neck revealed an infiltrative vascular mass of the left parotid, skull base, and deeper facial structures with occlusion of the left internal jugular vein and extension into the external auditory canal", "The differential diagnosis included atypical infantile hemangioma, congenital hemangioma, or venous malformation", "The dermatology department was consulted to consider treatment with propranolol", "On examination, the patient was afebrile, intermittently irritable, but otherwise well appearing with stable vital signs", "Of note, his length was in the 76th percentile for age, whereas his weight and head circumference were both greater than the 99th percentile for age", "Swelling of the left lower part of the face was noted", "A nontender, faint blue patch with subtle swelling and thick, coarse overlying hair was present on the left preauricular-temporal part of the scalp (Figure1A)", "Subtle scattered tan patches were present on the trunk and extremities (Figure 1B)", "A, Blue patch of the left periauricular scalp", "Note the difference in the textural quality of the hair overlying the affected area, which the parents trimmed regularly to maintain a more uniform appearance", "B, Representative light tan patches with irregular borders on the back", "Arrowheads indicate the patches", "A total of 15 similar lesions were noted on examination"], "s1": [0, 1, 2, 3, 4, 5, 6, 7, 8], "s2": [9, 10, 11, 12, 13, 14, 15, 16, 17, 18]} {"key": 1094, "questions": "What Is Your Diagnosis?", "options": [{"label": "A", "disease": "Lymphoepithelial carcinoma of parotid gland"}, {"label": "B", "disease": "Undifferentiated carcinoma of parotid gland"}, {"label": "C", "disease": "Lymphoma"}, {"label": "D", "disease": "Metastatic squamous carcinoma to parotid gland"}], "answer_idx": "A", "symptoms": ["A previously healthy man in his 20s who was of Asian heritage presented with a nontender mass in the left parotid region, which had been slowly enlarging for 18 months", "He had experienced occasional blepharospasm and paresthesia in the ipsilateral eye and cheek", "He had also noted an unintended 3-kg weight loss in the same duration, which he subjectively attributed to the stress of school", "Physical examination demonstrated a firm, immobile, irregular mass n the left parotid and approximately 3 × 3 cm in size", "Mild left-sided buccal weakness was noted", "Otherwise, his cranial nerve function was intact", "Fine needle aspiration demonstrated medium-size ovoid cells with minimal cytoplasm in crowded groups", "T2-weighted magnet resonance imaging following the initial computed tomographic (CT) scan demonstrated a poorly marginated mass that was 4 cm in maximum diameter abutting the left superficial parotid with involvement of the masseter (Figure, A)", "A parotidectomy with excision of the tumor was planned", "During the initial exposure of the tumor, an intraoperative frozen section demonstrated a cellular infiltrate, and a diagnosis of lymphoma could not be excluded", "Given the size and fixed nature of the tumor, the decision was made to stop resection on establishing a definitive diagnosis", "Flow cytometry demonstrated phenotypically normal T-cell and B-cell populations", "Permanent histologic sections demonstrated nests of nonkeratinizing undifferentiated atypical epithelial cells intimately intermingled with a dense lymphoplasmacytic infiltrate (Figure, B)", "Immunohistochemical analysis for cytokeratin AE1/3 was positive in the carcinoma cells (Figure, C)", "In situ hybridization for Epstein-Barr virus–encoded small RNAs (EBERs) was positive (Figure, D)", "A, T2-weighted coronal magnetic resonance image of the face", "B-D, Histopathologic images", "B, Hematoxylin-eosin, original magnification ×20. C, Cytokeratin AE1/3 immunostain, original magnification ×20. D, Epstein-Barr virus in situ hybridization, original magnification ×20."], "s1": [0, 1, 2, 3, 4, 5], "s2": [6, 7, 8, 9, 10, 11, 12, 13, 14, 15, 16, 17]} {"key": 1095, "questions": "What Is Your Diagnosis?", "options": [{"label": "A", "disease": "Schwannoma"}, {"label": "B", "disease": "Hamartoma"}, {"label": "C", "disease": "Epidermoid cyst"}, {"label": "D", "disease": "Calcinosis cutis"}], "answer_idx": "D", "symptoms": ["A 5-year-old boy presented with a 1-year history of a mass gradually growing over the helix of his left auricle", "It was associated with minimal pain and intermittently fluctuated in size", "Physical examination revealed a single 1.0 × 0.7-cm mass on the left mid helix that was firm, nontender, nonerythematous, and nonerosive", "Surgical resection was performed", "The resected specimen appeared cystic (Figure, A) with granulomatous reaction at the periphery (Figure, B)", "The lesion was found to be superficially located in the dermis and filled with a soft white material that appeared as purple to blue crystals", "Histopathologic images", "A, Specimen shows purple to blue crystals with cystic components", "B, Cystic structure with amorphous material and granulomatous reaction at the periphery with occasional foreign-body giant cells"], "s1": [0, 1, 2, 3], "s2": [4, 5, 6, 7, 8]} {"key": 1096, "questions": "What Is Your Diagnosis?", "options": [{"label": "A", "disease": "Parotitis"}, {"label": "B", "disease": "Lipoma"}, {"label": "C", "disease": "Sialolipoma"}, {"label": "D", "disease": "Hemangioma"}], "answer_idx": "C", "symptoms": ["An infant girl presented to the pediatric otolaryngology clinic with a 2-month history of an enlarging left preauricular mass", "The parents first noticed a pea-sized nodule in front of her left ear that was now 3 cm, soft, mobile, and nontender, without overlying skin changes or facial nerve impairment", "The infant was otherwise well without fevers, feeding well, and growing appropriately", "Her mother had a full-term vaginal delivery without complications and a medical history that was notable only for gastroesophageal reflux disease treated with ranitidine", "Ultrasonography showed a 2.2 × 3.0-cm solid nodule", "The patient was then referred for magnetic resonance imaging (MRI) with contrast, which showed a 2.9 × 1.6 × 2.3-cm mass with a well-defined capsule arising from within the superficial left parotid gland", "The mass was largely of fat signal intensity on both T1-weighted (Figure, A) and T2-weighted sequences with heterogeneous soft-tissue enhancement on T1 post-contrast (Figure, B)", "The mass also suppressed relatively uniformly on short T1 inversion recovery (STIR) sequence (Figure, C)", "The Figure, D, shows the mass in situ during surgical excision with facial nerve monitoring", "Magnetic resonance images of a patient with a left preauricular mass of the superficial parotid lobe"], "s1": [0, 1, 2, 3], "s2": [4, 5, 6, 7, 8, 9]} {"key": 1097, "questions": "What Is Your Diagnosis?", "options": [{"label": "A", "disease": "Hyoid bursitis"}, {"label": "B", "disease": "Temporomandibular joint dysfunction"}, {"label": "C", "disease": "Eagle syndrome"}, {"label": "D", "disease": "Metastatic thyroid cancer"}], "answer_idx": "C", "symptoms": ["A man in his 60s with a 20 pack-year smoking history presented with a several-month history of intermittent left otalgia, facial pain, swelling, and dysphagia", "The pain was exacerbated by eating and prolonged talking", "He was initially seen at an emergency department, where his symptoms were unsuccessfully treated with antibiotics", "He stated that he had not experienced hoarseness, weight loss, fever, night sweats, parotid mass, or involuntary facial nerve spasms", "His medical history was significant for blunt trauma to the left maxilla region approximately 10 years prior to presentation", "His surgical history was noncontributory", "Physical examination demonstrated a well-developed man in no distress who spoke in a normal tone", "His vital signs were normal", "The external auditory canals and the tympanic membranes were intact and clear bilaterally", "The nasal cavity and the nasopharynx were normal", "The oral cavity and oropharynx revealed moist mucosa with no lesions or erythema", "Fiber-optic flexible nasopharyngoscopy and laryngoscopy revealed normal pharyngeal walls and pyriform sinuses", "The vocal folds were symmetric and mobile bilaterally", "Computed tomography (CT) of the neck demonstrated a ossified mass (arrowheads) in the left parapharyngeal space (Figure)"], "s1": [0, 1, 2, 3, 4, 13], "s2": [5, 6, 7, 8, 9, 10, 11, 12]} {"key": 1098, "questions": "What Is Your Diagnosis?", "options": [{"label": "A", "disease": "Mucinous cystic neoplasm"}, {"label": "B", "disease": "Primary duodenal carcinoid tumor"}, {"label": "C", "disease": "Intraductal papillary mucinous neoplasm"}, {"label": "D", "disease": "Serous cystadenoma"}], "answer_idx": "C", "symptoms": ["A man in his 50s with a history of hypercholesterolemia and type 2 diabetes mellitus was referred for diarrhea (5-6 times per day) and a 27-kg weight loss in the preceding 5 months", "The patient did not have abdominal pain, nausea, vomiting, or a history of abdominal surgical procedures and did not use alcohol or tobacco", "On physical examination, the patient’s abdomen was soft, nontender, and nondistended, with no palpable masses, and he did not appear jaundiced", "Results of laboratory testing demonstrated a carbohydrate antigen 19-9 level of less than 1 U/mL and a carcinoembryonic antigen level of 6.0 ng/mL (to convert to micrograms per liter, multiply by 1.0)", "A duodenal mass was found using esophagogastroduodenoscopy and biopsies were taken", "To further evaluate the lesion, the patient underwent a computed tomographic scan of the abdomen and pelvis with contrast dye (Figure 1)", "A, Cross-sectional computed tomographic scan demonstrating a 4.7 × 2.9-cm, heterogeneous, solid mass involving the ampulla of Vater and the head of the pancreas", "A heterogeneous soft-tissue nodule can be seen (arrowhead)", "The common bile duct (CBD), duct of Santorini (S), and duct of Wirsung (W) can all be identified traversing the mass compressing the duodenum (D)", "B, Coronal computed tomographic scan demonstrating a polypoid pedunculated ampullary tumor extending into the duodenal lumen"], "s1": [0, 1, 2, 3], "s2": [4, 5, 6, 7, 8, 9]} {"key": 1099, "questions": "What Is Your Diagnosis?", "options": [{"label": "A", "disease": "Carotid stump syndrome"}, {"label": "B", "disease": "Subclavian pseudoaneurysm"}, {"label": "C", "disease": "Innominate artery pseudoaneurysm"}, {"label": "D", "disease": "Kommerell diverticulum"}], "answer_idx": "C", "symptoms": ["A man in his late 60s with a history of laryngeal cancer was transferred to our institution", "The patient had undergone laryngectomy with radiation, modified radical neck dissection, and a pectoralis major flap into the right-sided neck wound in 1998. This operation was subsequently complicated by right carotid blowout requiring right-sided carotid artery ligation on November 5, 2005. The patient initially presented to our hospital with a Glasgow Coma Scale score of 3. Findings from computed tomography of the head at the referring hospital showed acute right-hemispheric stroke with left-sided hemiparesis", "On examination, the patient was afebrile, with a heart rate of 108 beats/min and blood pressure of 169/91 mm Hg with 100% oxygen saturation", "Pertinent findings included an improved Glasgow Coma Scale score of 12 and left arm and leg hemiparesis", "The right upper arm was cool to the touch below the elbow, with cyanotic right fingers, and the right-sided radial and brachial pulses were absent", "The patient also presented with a pulsatile mass in the upper chest wall on the right side", "A confirmatory magnetic resonance image of the head is shown in Figure 1A, and a selective angiogram of the innominate artery is shown in Figure 1B", "A, Magnetic resonance imaging of the head", "B, Selective angiogram of the innominate artery"], "s1": [0, 1, 6, 7, 8], "s2": [2, 3, 4, 5]} {"key": 1100, "questions": "What Is Your Diagnosis?", "options": [{"label": "A", "disease": "Rocky Mountain spotted fever"}, {"label": "B", "disease": "Secondary syphilis"}, {"label": "C", "disease": "Hand, foot, and mouth disease"}, {"label": "D", "disease": "Primary human immunodeficiency virus infection"}], "answer_idx": "B", "symptoms": ["A 23-year-old white man who has sex with men (MSM) presented with a 1-month history of skin lesions on the palms and soles as well as a 2-week history of lesions in his oropharynx", "He reported that a fraction of the lesions had blistered earlier in the course of his illness", "The patient also described myalgias and fatigue over the preceding month", "He moved to Texas from the northeast a few months prior to his illness", "He denied any exposure to sexually transmitted infections or a history of genital lesions", "On examination, the patient had erythematous papules on bilateral palms and soles as well as eroded vesicles on the palate (Figure, A and B)", "Lymphadenopathy was not present", "A biopsy specimen of a papule on the left sole was obtained for immunohistochemical staining (Figure, C and D)", "A serologic antibody test revealed an elevated coxsackievirus titer of 1:16 (reference range, <1:8", ">1:32 indicative of infection)", "A, Patient’s soles at presentation", "B, Patient’s palate at presentation", "C, Hematoxylin-eosin–stained section of the biopsy specimen (original magnification, ×20)", "D, Immunohistochemistry stain of biopsy specimen (anti–Treponema pallidum polyclonal antibody immunostain", "original magnification, ×20)"], "s1": [0, 1, 2, 3, 4, 5, 6], "s2": [7, 8, 9, 10, 11, 12, 13, 14]} {"key": 1101, "questions": "What Is Your Diagnosis?", "options": [{"label": "A", "disease": "Pyoderma gangrenosum"}, {"label": "B", "disease": "Squamous cell carcinoma"}, {"label": "C", "disease": "Cutaneous aspergillosis"}, {"label": "D", "disease": "Cutaneous mucormycosis"}], "answer_idx": "D", "symptoms": ["A man in his 50s was referred to the dermatology clinic for a chronic, painful nonhealing ulcer on the right forearm following a ground-level fall 3 months previously", "He reported subjective fevers, as well as increasing erythema and tenderness immediately around the right forearm lesion", "He stated that he did not have a history of poor wound healing or painful skin ulcers", "He had a history of type 2 diabetes and an allogenic bone marrow transplant 1 year prior for myelodysplastic syndrome complicated by pancytopenia and chronic graft-vs-host disease", "Physical examination revealed an isolated slightly erythematous, edematous plaque with a prominent central eschar on the right forearm (Figure, A)", "An incisional biopsy at the edge of the eschar was performed, and a specimen was sent for pathological evaluation (Figure, B-D)", "A, Clinical photograph of a nonhealing ulcer with central eschar on the right forearm", "B-D, Histopathologic images", "B, Necrotic debris and fat necrosis with saponification (hematoxylin-eosin, original magnification ×40)", "C, Saponification and needle-like clefts within necrotic adipocytes (hematoxylin-eosin, original magnification ×200)", "D, Numerous nonseptate, thick-walled fungal hyphae in the dermis (hematoxylin-eosin, original magnification ×400)"], "s1": [0, 1, 2, 3, 4], "s2": [5, 6, 7, 8, 9, 10]} {"key": 1102, "questions": "What Is Your Diagnosis?", "options": [{"label": "A", "disease": "Relapsing polychondritis"}, {"label": "B", "disease": "Granulomatosis with polyangiitis (Wegener granulomatosis)"}, {"label": "C", "disease": "Cold agglutinin disease"}, {"label": "D", "disease": "Chilblain"}], "answer_idx": "C", "symptoms": ["A man in his 70s presented with erythematous ears associated with a burning sensation", "He reported that the lesions had begun 6 months previously and that they were most bothersome at the end of the day or when he was exposed to the cold and damp environment while out on his boat", "He applied ice to alleviate the burning, but this worsened the condition", "A heating pad provided some measure of relief, as did a brief course of systemic corticosteroids", "Physical examination revealed that some papular lesions were associated with slight purpura and hemorrhagic crust (Figure, A)", "Histologic findings from biopsy specimens were also obtained by routine histologic analysis (Figure, B) and direct immunofluorescence examination (Figure, C)", "A, Clinically, the left ear demonstrates erythema, edema, and focal crusting", "B, Biopsy specimen from the left ear reveals thrombi within engorged blood vessels but fails to reveal significant inflammation of blood vessel walls (hematoxylin-eosin, original magnification ×200)", "C, Biopsy specimen from the left ear under direct immunofluorescence and stained for anti-IgM highlights thrombi within vessels (original magnification ×200)"], "s1": [0, 1, 2, 3, 6], "s2": [4, 5, 7, 8]} {"key": 1103, "questions": "What Is Your Diagnosis?", "options": [{"label": "A", "disease": "Recurrent meningitis"}, {"label": "B", "disease": "Posterior communicating artery aneurysm"}, {"label": "C", "disease": "Recurrent painful ophthalmoplegic neuropathy"}, {"label": "D", "disease": "Myasthenia"}], "answer_idx": "C", "symptoms": ["A young boy presented for evaluation of “droopiness” of the eyelid", "He had 3 episodes of right-sided ptosis, each of which occurred at 18 months, 3 years, and 5 years of age", "None of these episodes was preceded by fever", "During the last 2 episodes, he experienced severe right-sided headache lasting for 6 to 7 days before development of ptosis", "he also experienced double vision", "There was no history of loss of consciousness, seizures, loss of vision or hearing, facial deviation, trouble swallowing, or weakness of the extremities in any of these episodes", "There was history of migraine on the maternal side of the family", "There was no family history of autoimmune disorders or cerebral aneurysms", "At the time of the clinic visit, the patient was asymptomatic and results of the neurologic examination were normal", "The family provided a photograph of the patient’s face at the time of the first episode (Figure, A)", "After the first episode, with a presumed diagnosis of acute disseminated encephalomyelitis, he received intravenous pulse methylprednisolone sodium succinate, 30 mg/kg, for 3 days and recovered completely in 3 weeks", "The patient received intravenous immunoglobulin, 2 g/kg, for 2 days after the subsequent 2 episodes, with a presumed diagnosis of chronic inflammatory demyelinating neuropathy, and recovered fully", "Results of brain magnetic resonance imaging performed at 18 months showed thickening of the cisternal part of the right third cranial nerve in the perimesencephalic cistern (Figure, B) with no parenchymal lesions", "Results of tests on cerebrospinal fluid collected via lumbar puncture were normal", "A, The patient’s right-sided ptosis during the first episode at age 18 months", "B, Results of brain magnetic resonance imaging showing thickening of the cisternal part of the right third cranial nerve in the perimesencephalic cistern (white arrowhead)"], "s1": [0, 1, 3, 4, 9, 14], "s2": [2, 5, 6, 7, 8, 10, 11, 12, 13, 15]} {"key": 1104, "questions": "What Is Your Diagnosis?", "options": [{"label": "A", "disease": "Malignant melanoma of nasopharynx"}, {"label": "B", "disease": "Metastatic melanoma"}, {"label": "C", "disease": "Melanin-pigmented oncocytic metaplasia of the nasopharynx"}, {"label": "D", "disease": "Hemangioma"}], "answer_idx": "C", "symptoms": ["A man in his 60s with a history of hyperlipidemia, gout, and sleep apnea had a consultation for an incidental finding of a left torus tubarius, blackish, mucosal lesion noted during a health examination at a local clinic in July 2014. He had no nasal obstruction, nasal bleeding, or feeling of aural fullness", "Fibroscopic evaluation revealed a black nevus–like lesion over the left torus tubarius, without surface necrosis or signs of bleeding when touched", "Examination of the neck, nasal cavity, and larynx showed no remarkable findings", "It was recommended that he receive laser excision of the lesion under endoscopy", "During the surgery, the superficial lesion was found to be confined to the left torus tubarius, without invasion of the surrounding structures (Figure, A, arrowhead)", "The margins were clearly identified, and laser excision was performed smoothly with only minimal bleeding", "The surgical specimen was a single black, soft-tissue fragment measuring 0.4 × 0.3 × 0.2 cm", "Microscopy revealed small mixed serous and mucinous glands, normal respiratory epithelia with marked oncocytic cell changes, and abundant eosinophilic granular cytoplasm arranged in a tubular and microcystic pattern (Figure, B)", "Scattered brown pigments were noted in the cytoplasm of most oncocytic cells (Figure, C)", "There was no dyskeratosis", "The brown pigment was negative for iron stain (Figure, D) and HMB45 stain, but positive for Fontana Masson stain", "Ki-67 staining revealed low proliferative index in the lesion", "Some lymphocytic infiltration of the stroma was also noted", "A, Endoscopic view of the nasopharynx", "B-D, Histopathologic images", "B, Hematoxylin-eosin, original magnification ×100. C, Hematoxylin-eosin, original magnification ×200. D, Iron stain, original magnification ×100."], "s1": [0, 1, 2, 3, 4, 5], "s2": [6, 7, 8, 9, 10, 11, 12, 13, 14, 15]} {"key": 1105, "questions": "What Is Your Diagnosis?", "options": [{"label": "A", "disease": "Lymphoma"}, {"label": "B", "disease": "Osteosarcoma"}, {"label": "C", "disease": "Myositis ossificans traumatica"}, {"label": "D", "disease": "Tumoral calcinosis"}], "answer_idx": "C", "symptoms": ["A woman in her 50s developed severe trismus several months after treatment of a dental abscess", "Approximately 10 years after onset of symptoms, she was seen by an oral surgeon who attempted mandibular release via a cervical incision", "According to the patient, the surgeon was “unable to reach” the affected area, and the operation was unsuccessful", "There was no other significant medical or surgical history", "On physical examination, severe trismus was present with maximal interdental distance of 1 mm", "The patient was brought to the operating room for excision", "A maxillofacial computed tomographic (CT) examination was performed (Figure)", "A-C, Noncontrast computed tomographic images", "A, Axial slice from an image of the facial bones", "B, Coronal slice from an enhanced scan of the facial bones", "C, Coronal image of the facial bones using a soft-tissue algorithm", "Asterisk indicates left medial pterygoid artery", "D, Three-dimensional reconstructed image of the facial bones"], "s1": [0, 1, 2, 4, 5], "s2": [3, 6, 7, 8, 9, 10, 11, 12]} {"key": 1106, "questions": "What Is Your Diagnosis?", "options": [{"label": "A", "disease": "Allergic fungal rhinosinusitis"}, {"label": "B", "disease": "Sinonasal undifferentiated carcinoma"}, {"label": "C", "disease": "Chronic invasive fungal sinusitis"}, {"label": "D", "disease": "Chronic rhinosinusitis with polyposis"}], "answer_idx": "C", "symptoms": ["A man in his 30s presented to the emergency department with a worsening sinus infection and vision loss", "His medical history was significant for previous nasal endoscopy with biopsy of nasal polyps 6 years previously, with a finding of benign nasal polyps by surgical pathology", "The patient had then been lost to follow-up", "He now complained of nasal drainage, severe headaches, and long-standing visual loss", "There had been no acute worsening of headaches or visual loss in the previous months", "Physical examination showed severe left proptosis, telecanthus, purulent nasal drainage, and nasal polypsis filling the bilateral nasal cavities", "Ophthalmology consultation revealed no evidence of restriction of gaze or papilledema, and a recommendation was made for endoscopic sinus surgery to remove the source of infection", "Computed tomographic imaging revealed extensive nasal polyposis, bony remodeling consistent with long-standing inflammation, and massive erosion of the bilateral skull base and orbits (Figure 1A)", "The patient underwent endoscopic sinus surgery", "Nasal polyps were present and had a typical appearance superficially (Figure 1B)", "However, on deeper dissection, they became increasingly difficult to remove even with powered instrumentation, and an atypical, fibrotic endoscopic appearance was noted (Figure 1C)", "Tissue was sent for histologic evaluation", "Following this, the dissection was terminated", "A, Coronal computed tomographic scan demonstrating extensive nasal polyposis and bony remodeling of bilateral skull base and orbits", "B, Rigid nasal endoscopy demonstrating superficial appearance consistent with nasal polyps", "C, Rigid nasal endoscopy following deeper dissection of area in question, demonstrating atypical fibrotic appearance"], "s1": [0, 1, 2, 3, 4, 5, 6], "s2": [7, 8, 9, 10, 11, 12, 13, 14, 15]} {"key": 1107, "questions": "What Is Your Diagnosis?", "options": [{"label": "A", "disease": "Recurrent adenoid cystic carcinoma"}, {"label": "B", "disease": "Radiation-induced liposarcoma"}, {"label": "C", "disease": "Radiation-induced neurofibroma"}, {"label": "D", "disease": "Radiation-induced rhabdomyosarcoma"}], "answer_idx": "C", "symptoms": ["A woman in her 50s presented with a several-month history of symptoms that included foreign body sensation in her throat, mild odynophagia, and dyspnea", "She had a history of adenoid cystic carcinoma of the right submandibular gland and had undergone wide tumor excision and postoperative radiation therapy 20 years previously (60 Gy/30 fractions in 1993)", "She also had a history of hyperthyroidism, which was under medical control (with propylthiouracil, 50 mg/d)", "She stated that she did not have blurred vision, headache, seizures, or trismus", "She had no family history of neurofibromatosis", "A physical examination revealed a bulge in the floor of mouth, limitation of tongue movement, and no palpable lymphadenopathy", "She had no suspicious spots or nodules on the skin", "No neurological deficits were revealed", "Postgadolinium T1-weighted magnetic resonance imaging disclosed a 7.0 × 4.0 × 3.0-cm, well-defined enhancing mass in the right side of the floor of the mouth, extending posteriorly to the tongue base and medially across the midline (Figure, A)", "The results of laboratory investigations were normal", "Transcervical removal of the tumor was performed", "The surgical specimen was a 7.2 × 5.8 × 4.3-cm encapsulated tumor composed of a homogeneous, whitish, rubbery, and glistening content (Figure, B)", "Microscopic examination demonstrated oval to eel-like spindle nuclei distributed in a background of generally wavy collagenous fibers and mucinous matrix (Figure, C)", "The specimen showed sparse cellularity and no cellular atypia", "Neither mitotic figure nor tumor necrosis was found", "The specimen was immunoreactive for S-100 (Figure, D), focally positive for CD34, but nonreactive for smooth muscle actin", "A, Axial T1-weighed gadolinium-enhanced magnetic resonance image of the neck", "B, Gross image of the surgical specimen", "C, Histopathologic image", "Hematoxylin-eosin, original magnification ×200. D, Histopathologic image", "S-100 protein stain, original magnification ×200."], "s1": [0, 1, 2, 3, 5, 7, 9], "s2": [4, 6, 8, 10, 11, 12, 13, 14, 15, 16, 17, 18, 19, 20]} {"key": 1108, "questions": "What Is Your Diagnosis?", "options": [{"label": "A", "disease": "Laryngeal squamous cell carcinoma"}, {"label": "B", "disease": "Angioedema"}, {"label": "C", "disease": "Laryngeal tuberculosis"}, {"label": "D", "disease": "Laryngeal sarcoidosis"}], "answer_idx": "D", "symptoms": ["The patient was a woman in her 30s with a history of goiter, hypertension, asthma, obstructive sleep apnea, and skin lesions in the lower extremities for which biopsy findings more a decade before suggested sarcoidosis", "She presented with a 1-day history of sore throat, fever, chills, productive cough, globus sensation, dysphagia, and a change in her voice to the point it had become muffled and hoarse", "Throughout the day she noticed progressive worsening of her symptoms in addition to becoming dyspneic", "The patient tried using her albuterol inhaler, but this provided no relief of her symptoms", "The patient was afebrile at presentation, and there were no signs of infection", "On examination, there was mildly tender right cervical lymphadenopathy but no evidence of peritonsillar exudates", "Laryngoscopy demonstrated diffuse swelling of the supraglottic structures, particularly the epiglottis and left aryepiglottic fold, with a pale appearance of the mucosa (Figure, A)", "In addition, a neck computed tomographic (CT) scan was obtained, which showed marked swelling of the supraglottic structures with areas of patchy enhancement and narrowing of the airway, as well as enlarged right cervical lymph nodes (Figure, B-D)", "Owing to airway compromise, the patient was intubated and received intravenous steroids and ceftriaxone", "Subsequently, she underwent debulking surgery and carbon dioxide laser ablation of the left arytenoid", "A, Laryngoscopic photograph", "B, Axial contrast-enhanced computed tomographic (CT) image through level of the aryepiglottic folds", "C, Axial contrast-enhanced CT through level of the epiglottis", "D, Sagittal contrast-enhanced CT image"], "s1": [0, 1, 2, 3, 4, 5], "s2": [6, 7, 8, 9, 10, 11, 12, 13]} {"key": 1109, "questions": "What Is Your Diagnosis?", "options": [{"label": "A", "disease": "Non-Hodgkin lymphoma"}, {"label": "B", "disease": "Gastrointestinal stromal tumor"}, {"label": "C", "disease": "Fibrolamellar hepatocellular carcinoma"}, {"label": "D", "disease": "Neuroendocrine tumor of the pancreas"}], "answer_idx": "C", "symptoms": ["A 20-year-old otherwise healthy woman presented to the emergency department after a motor vehicle crash in which computed tomographic images of the abdomen were obtained as part of the trauma evaluation", "Small free fluid within the pelvis and 2 large mesenteric masses were discovered adjacent to the liver, pancreas, and stomach (Figure 1)", "An endoscopic ultrasound-guided fine-needle aspiration of the abdominal mass was obtained and the diagnosis was made", "The patient underwent exploratory laparotomy and en-bloc resection of the intraperitoneal tumors", "Computed tomographic images", "During the trauma workup after a motor vehicle crash, 2 large peritoneal tumors were identified in her upper abdomen", "A, An exophilic mass in segment 3 of the liver invading the distal stomach", "B, A mass in the lessor sac attaching to the body of pancreas", "The arrowheads indicate 2 isolated tumors close to the liver, pancreas, and stomach"], "s1": [0, 1, 4, 5], "s2": [2, 3, 6, 7, 8]} {"key": 1110, "questions": "What Is Your Diagnosis?", "options": [{"label": "A", "disease": "Pelvic teratoma"}, {"label": "B", "disease": "Meckel diverticulum"}, {"label": "C", "disease": "Enteric duplication cyst"}, {"label": "D", "disease": "Carcinoid tumor"}], "answer_idx": "B", "symptoms": ["A 5-year-old boy with a 1-year history of intermittent abdominal pain presented to the emergency department with 2 days of nausea, nonbilious emesis, and diffuse abdominal pain", "He was afebrile and had normal vital signs", "Physical examination findings were notable for mild abdominal distension and tenderness without guarding or rebound sign", "Abdominal ultrasonography revealed a normal appendix", "Abdominal radiography (Figure 1A) revealed multiple polygonal calcified lesions in the pelvis and a nonobstructive bowel gas pattern", "Computed tomography (Figure 1B) revealed a 5.6 × 5.3 × 4.6-cm midline cystic pelvic mass adjacent to the anterior rectal wall that contained several irregularly shaped ossifications", "Laboratory study results were significant for normal levels of α-fetoprotein and chorionic gonadotropin and a white blood cell count of 19 700/μL (reference range, 5500-15 500/μL", "to convert to ×109/L, multiply by 0.001)", "The patient was taken to the operating room for an exploratory laparotomy", "A, Supine abdominal radiograph reveals multiple polygonal calcified lesions in the pelvis (black arrowhead)", "B, Computed tomographic sagittal image of the abdomen reveals a midline cystic pelvic mass that contains several irregularly shaped ossifications (white arrowhead)", "L indicates left"], "s1": [0, 1, 2, 6, 8, 11], "s2": [3, 4, 5, 7, 9, 10]} {"key": 1111, "questions": "What Is Your Diagnosis?", "options": [{"label": "A", "disease": "Giant cell fibroblastoma"}, {"label": "B", "disease": "Infantile myofibromatosis"}, {"label": "C", "disease": "Dermoid cyst"}, {"label": "D", "disease": "Keratoacanthoma"}], "answer_idx": "B", "symptoms": ["A newborn girl with a nodular lesion on the scalp present since birth was seen at the dermatology clinic", "The parents reported occasional erosions and crusting that resolved with topical mupirocin, 2%, cream, but with no continued growth", "The mother had had a normal pregnancy", "Physical examination revealed a firm, exophytic, and flat-topped nodule, with superficial erosion and crust on the right parietal side of the scalp measuring 3 cm in diameter and 1 cm thick (Figure, A)", "No other skin lesions were found", "A 4-mm punch skin biopsy was performed, and tissue was sent for pathologic analysis (Figure, B and C)", "Cerebral and abdominal ultrasonography revealed no evidence of visceral involvement", "The lesion regressed without treatment over a period of 8 months", "Close follow-up every 3 months was scheduled, and complete involution of the tumor was noted at 10 months of age (Figure, D)", "A, Nodular lesion on the right parietal scalp at initial presentation", "B, Histopathologic image of a biopsy specimen (hematoxylin-eosin, original magnification ×200)", "C, Immunohistochemical image of a biopsy specimen (actin stain, original magnification ×100)", "D, Complete regression of tumor at 10 months of age"], "s1": [0, 1, 2, 3, 4, 6, 7, 8], "s2": [5, 9, 10, 11, 12]} {"key": 1112, "questions": "What Is Your Diagnosis?", "options": [{"label": "A", "disease": "Ecthyma gangrenosum"}, {"label": "B", "disease": "Cytomegalovirus ulcer"}, {"label": "C", "disease": "Mucormycosis ulcer"}, {"label": "D", "disease": "Pyoderma gangrenosum"}], "answer_idx": "B", "symptoms": ["A man in his 60s presented with a painless rapidly enlarging ulcer on his left leg 1 week after the onset of diverticulitis, a neutropenic fever, and severe sepsis due to multidrug-resistant Pseudomonas aeruginosa infection that necessitated admission to the intensive care unit", "His medical history included a heart transplant for ischemic cardiomyopathy in his early 50s, and he was receiving long-term immunosuppressive therapy with cyclosporine, mycophenolate mofetil, and deflazacort", "He had received a diagnosis of plasmablastic lymphoma 6 months earlier and was being treated with polychemotherapy (cyclophosphamide, doxorubicin hydrochloride, vincristine sulfate, and prednisone [CHOP] regimen)", "Physical examination revealed a well-demarcated 5 × 5-cm round ulcer with raised border, surrounding erythema, and a base with purulent debris (Figure 1A)", "A superficial swab culture sample grew P aeruginosa", "A punch biopsy specimen from the edge of the ulcer was obtained and sent for histopathologic evaluation (Figure 1B)", "A, Cutaneous ulcer on the patient’s left leg with a raised erythematous border and a base with purulent debris", "B, Diagnostic biopsy shows enlarged endothelial cells with intranuclear inclusions surrounded by a clear halo, and some cytoplasmic inclusions", "Similar findings are detected in some macrophages (hematoxylin-eosin, original magnification ×400)"], "s1": [0, 1, 2], "s2": [3, 4, 5, 6, 7, 8]} {"key": 1113, "questions": "What is your diagnosis?", "options": [{"label": "A", "disease": "Leukocytoclastic vasculitis"}, {"label": "B", "disease": "Antiphospholipid syndrome"}, {"label": "C", "disease": "Type I cryoglobulinemia associated with multiple myeloma"}, {"label": "D", "disease": "Purpura fulminans"}], "answer_idx": "C", "symptoms": ["A woman in her 80s with a history of nonischemic cardiomyopathy and chronic kidney disease presented with purpuric lesions on her face, elbows, knees, and feet", "She was initially admitted for altered mental status in the setting of an unwitnessed fall and was noted to have “bruises” on her lower extremities", "Dermatology was consulted after the development of new lesions on her nose, ear, and the extensor surfaces of her arms bilaterally", "Admission laboratory test results showed hypercalcemia (calcium level, 11.5 mg/dL) and acute kidney injury (creatinine level, 5.6 mg/dL)", "On physical examination, the patient’s nose, bilateral cheeks, forehead, and left ear (helical rim and antihelix [Figure, A]) were found to have dusky purpuric macules and patches", "Her knees, feet, and the extensor surfaces of her arms had reticulated purpuric macules and patches", "There was skin sloughing off the lesions on her elbows (Figure, B) and knees", "Four-millimeter punch biopsy specimens were taken from the left ear, right arm, and forehead (Figure, C and D)", "Clinical and histopathologic images from a case of acral retiform purpura", "A, Dusky purpuric macules on the helical rim and antihelix of the ear", "B, Reticulated purpuric patches with focal areas of necrosis on the extensor surfaces of the arms", "C and D, In specimens from the forehead, multifocal intravascular fibrinoid material consisting of a mix of cryoglobulins and fibrin thrombi with relatively little vessel wall inflammation was seen (hematoxylin-eosin, original magnification ×4 [C] and ×10 [D])"], "s1": [0, 1, 3], "s2": [2, 4, 5, 6, 7, 8, 9, 10, 11]} {"key": 1114, "questions": "What Is Your Diagnosis?", "options": [{"label": "A", "disease": "Retroperitoneal abscess"}, {"label": "B", "disease": "Lymphatic malformation"}, {"label": "C", "disease": "Urinoma"}, {"label": "D", "disease": "Pancreatic pseudocyst"}], "answer_idx": "B", "symptoms": ["An adolescent girl presented to her pediatrician reporting abdominal distention", "She first became aware of her increased abdominal girth when friends at school asked whether she was pregnant", "She had noticed her clothing becoming tighter over recent weeks", "She had not experienced abdominal pain or other symptoms, and there was no history of trauma", "Physical examination revealed a firm, distended abdomen without bowel sounds", "Pregnancy test results were negative", "Results of laboratory studies, including a complete blood cell count, basic metabolic panel, and liver function tests, were within normal limits", "An abdominal radiograph showed a paucity of bowel gas markings (Figure 1A)", "A subsequent computed tomographic scan of the abdomen and pelvis revealed a 30-cm retroperitoneal cystic structure causing significant mass effect on the abdominal viscera, displacing the right kidney superiorly under the liver causing moderate hydronephrosis (Figure 1B)", "A, Abdominal radiograph showing a paucity of bowel gas markings", "B, Abdominal computed tomographic scan showing a massive retroperitoneal cystic lesion with displacement of abdominal organs and right moderate hydronephrosis"], "s1": [0, 1, 2, 3, 5, 6], "s2": [4, 7, 8, 9, 10]} {"key": 1115, "questions": "What Is Your Diagnosis?", "options": [{"label": "A", "disease": "Fungal ball"}, {"label": "B", "disease": "Chronic invasive fungal sinusitis"}, {"label": "C", "disease": "Invasive squamous cell carcinoma"}, {"label": "D", "disease": "Fungal ball and invasive squamous cell carcinoma"}], "answer_idx": "D", "symptoms": ["A man in his 60s had an 8-month history of right-sided nasal congestion and rhinorrhea", "He had no clinically significant improvement in symptoms despite multiple courses of oral antibiotics", "Over the previous 2 months, he had developed progressive right-sided facial swelling, pain, blurred vision, and a 6.8-kg weight loss", "There was no history of immunocompromise or foreign travel", "A computed tomographic (CT) scan at an outside hospital showed complete opacification and an invasive process centered in the right maxillary sinus", "An endoscopic maxillary antrostomy and biopsy showed inflammatory changes and numerous fungal organisms consistent with Aspergillus species", "The patient was started on treatment with intravenous antifungal and broad-spectrum antibiotics, and he was transferred for further evaluation and management", "The patient’s examination was notable for right-sided facial and periorbital swelling, severe pain, and limitation of lateral gaze of the right eye", "A repeated CT scan was completed (Figure)", "A-C, Computed tomographic images", "A, Coronal view of the paranasal sinuses in bone windows showing near-complete opacification of the right maxillary sinus with soft-tissue thickening, a calcific focus, and associated erosion of the bony walls", "B, Coronal view of the paranasal sinuses in soft-tissue windows", "C, Axial view of the paranasal sinuses in bone windows"], "s1": [0, 1, 2, 3, 7], "s2": [4, 5, 6, 8, 9, 10, 11, 12]} {"key": 1116, "questions": "What Is Your Diagnosis?", "options": [{"label": "A", "disease": "Hemangioma of the larynx"}, {"label": "B", "disease": "Well-differentiated liposarcoma of the larynx"}, {"label": "C", "disease": "Schwannoma of the larynx"}, {"label": "D", "disease": "Laryngocele of the larynx"}], "answer_idx": "B", "symptoms": ["A man in his 40s with a 10-year history of smoking 180 packs of cigarettes per year choked easily and had experienced a foreign body sensation in his throat for 6 months", "He was a tour guide, a profession that requires speech", "Because he experienced voice fatigue frequently, he visited a hospital for professional assistance", "Fibroscopy showed a well-circumscribed mass of approximately 3 × 3 × 2 cm, with a smooth surface, located at the laryngeal side of the left aryepiglottic fold and covering nearby structures, such as the bilateral vocal cords and left pyriform sinus (Figure 1, A)", "A biopsy was performed, and the patient visited our institution (Taipei Veterans General Hospital) for further intervention", "The laryngeal mass was removed through transoral laser microsurgery", "The tumor was at the submucosa with focally ulcerated overlying epithelium", "It was composed of adipocytes of various sizes in a fibromyxoid background", "In the fibromyxoid areas, atypical spindle cells with hyperchromatic and enlarged nuclei were present, as well as scattered lipoblasts without areas of dedifferentiation (Figure 1, B)", "Immunohistochemically, the tumor exhibited substantial nuclear staining for both CDK4 and MDM2 and stains (Figure 1, C)"], "s1": [0, 1, 2, 3, 4, 5], "s2": [6, 7, 8, 9]} {"key": 1117, "questions": "What Is the Diagnosis?", "options": [{"label": "A", "disease": "Rhabdomyosarcoma"}, {"label": "B", "disease": "Venous malformation"}, {"label": "C", "disease": "Masticator space abscess"}, {"label": "D", "disease": "Chronic hematoma"}], "answer_idx": "B", "symptoms": ["A teenage girl presented with new onset of bilateral upper extremity numbness and paresthesias", "The rest of the clinical history and neurological examination was unremarkable", "A magnetic resonance imaging (MRI) study of the brain and cervical spine obtained to evaluate for demyelinating disease demonstrated a normal brain and cervical cord, but an incidental T2 hyperintense lesion within the right pterygoid muscles was noted", "On further consultation with the otolaryngology department, the patient stated that she had not experienced any sensation of a mass, and results from a focused otolaryngologic examination were also normal", "A dedicated fifth cranial nerve MRI examination performed to better characterize the lesion revealed a well-defined, lobulated T2 hyperintense lesion within the right pterygoid muscles without mass effect, adjacent edema, or other invasive features (Figure, A)", "There was a small focus of signal void on the T2-weighted images, suspicious for tiny calcification", "On gadolinium administration, there was sequential increased enhancement on the postcontrast T1-weighted axial images (Figure, B and C)", "Further imaging workup with single-proton emission computed tomography-CT (SPECT-CT) using technetium Tc 99m–labeled red blood cells (RBCs) demonstrated increased tracer pooling within the lesion (Figure, D)", "A-C, Magnetic resonance imaging studies (axial views) of a teenager with an incidental masticator space lesion", "D, Single-proton emission computed tomographic image (axial view) using technetium Tc 99m–labeled red blood cells demonstrating increased tracer pooling within the lesion"], "s1": [0, 1, 3], "s2": [2, 4, 5, 6, 7, 8, 9]} {"key": 1118, "questions": "What Is Your Diagnosis?", "options": [{"label": "A", "disease": "Paraganglioma"}, {"label": "B", "disease": "Adult rhabdomyoma"}, {"label": "C", "disease": "Squamous cell carcinoma"}, {"label": "D", "disease": "Sarcoidosis "}], "answer_idx": "B", "symptoms": ["A woman in her 50s presented with dysphagia and frequent throat clearing", "A right pyriform sinus mass was seen on fiber-optic laryngoscopy", "Computed tomography (CT) and magnetic resonance imaging (MRI) confirmed the finding of a hypopharyngeal mass", "The patient underwent an endoscopic laser excision", "Immunohistochemical stains revealed tumor cells that were focally positive for myogenin and diffusely positive for desmin and myoglobin", "An MRI scan obtained 1 month postoperatively demonstrated a persistent mass or swelling", "The patient continued to experience dysphagia, globus, and a choking sensation when coughing", "On fiber-optic examination, right pyriform sinus fullness was visualized", "During operative laryngoscopy, no mass was identified", "An interval MRI demonstrated a 2-cm soft-tissue mass in the cervical esophagus (Figure, A)", "Using rigid suspension laryngoscopy, we identified a large, smooth-walled ovoid mass herniating from the cervical esophageal opening", "A mucosal stalk anchored the mass to the lateral pyriform sinus wall", "Endoscopic snare cautery was used to excise the mass", "The tumor was well-defined, solitary, tan-red, and composed of sheets of large, round-to-polygonal cells with marked eosinophilic, granular cytoplasm (Figure, B)", "Most tumor cells exhibited peripherally or centrally located vesicular nuclei with prominent nucleoli", "Peripheral intracellular vacuoles indenting the cytoplasm were seen, which represent a processing artifact due to intracellular glycogen loss (Figure, C)", "A, Axial contrast-enhanced spoiled gradient magnetic resonance image", "B and C, Histopathological images", "B, Hematoxylin-eosin, original magnification ×20. C, Hematoxylin-eosin, original magnification ×40."], "s1": [0, 1, 2, 5, 6, 7, 8, 9, 10, 11, 12], "s2": [3, 4, 13, 14, 15, 16, 17, 18]} {"key": 1119, "questions": "What Is Your Diagnosis?", "options": [{"label": "A", "disease": "Pleomorphic adenoma"}, {"label": "B", "disease": "Desmoid tumor"}, {"label": "C", "disease": "Fibrosarcoma"}, {"label": "D", "disease": "Rhabdomyosarcoma"}], "answer_idx": "B", "symptoms": ["A man in his 50s presented with a facial mass growing over 1 year, with 28.5-kg weight loss due to poor oral intake", "He reported a 44–pack-year smoking history and remote alcohol abuse", "On physical examination he had a 10-cm right-sided facial mass extending under the auricle from the mastoid to the zygoma and inferiorly to the angle of the mandible", "The mass was firm, mildly tender, but without overlying skin changes or cervical lymphadenopathy", "Computed tomographic (CT) and magnetic resonance imaging (MRI) scans showed maxillary sinus compression with partial destruction of the zygomatic arch, mandible, and orbital floor (Figure, A and B)", "The mass extended into the right pterygopalatine fossa, disrupting the floor of the middle cranial fossa and widening the foramen ovale", "A core biopsy specimen revealed spindle cell proliferation with abundant collagen that stained focally with CD34, smooth muscle actin, and Bcl-2. The Figure, C, shows a gross pathologic image after excision of the mass", "Histopathologic findings demonstrated spindle cells arranged in long fascicles with abundant interstitial collagen (Figure, D)", "Tumor cells exhibited central, oval nuclei, and moderate amounts of pale-pink cytoplasm with indistinct borders", "There was no increased or atypical mitosis, necrosis, or vascular invasion", "Immunostaining demonstrated strong nuclear staining for β-catenin antibody and was positive for smooth muscle actin antibody, while stains for desmin, pan-keratin, S-100, CD34, and Bcl-2 antibodies were negative", "The Ki67 index was less than 1%", "A, Coronal computed tomographic scan of the mass", "B, Postcontrast axial T1-weighted magnetic resonance image", "C, Gross pathologic image of the mass", "D, Histopathologic image of excised tissue (hematoxylin-eosin, original magnification ×40)"], "s1": [0, 1, 2, 3], "s2": [4, 5, 6, 7, 8, 9, 10, 11, 12, 13, 14, 15]} {"key": 1120, "questions": "What Is Your Diagnosis?", "options": [{"label": "A", "disease": "Adrenal liposarcoma"}, {"label": "B", "disease": "Renal hamartoma"}, {"label": "C", "disease": "Adrenal myelolipoma"}, {"label": "D", "disease": "Nonfunctional paraganglioma"}], "answer_idx": "C", "symptoms": ["A man in his 40s presented with sudden left-sided abdominal pain that was sharp, severe, and associated with emesis", "He denied fevers, dizziness, or recent trauma", "He had no notable medical or surgical history", "He was afebrile with normal vital signs", "He was well nourished, alert, and in no acute distress", "Results of an abdominal examination revealed normoactive bowel sounds, no distention, but tenderness on the left side", "Computed tomography (CT) of the abdomen and pelvis with intravenous contrast revealed a 12 × 13 × 11-cm left-sided adrenal mass (Figure 1)", "Results of laboratory examination revealed plasma levels of metanephrines, normetanephrines, cortisol, renin, and aldosterone within reference limits"], "s1": [0, 1, 2, 3, 4, 5], "s2": [6, 7]} {"key": 1121, "questions": "What Is Your Diagnosis?", "options": [{"label": "A", "disease": "Leiomyosarcoma"}, {"label": "B", "disease": "Gastrointestinal stromal tumor"}, {"label": "C", "disease": "Gastric duplication"}, {"label": "D", "disease": "Pancreatic pseudocyst"}], "answer_idx": "C", "symptoms": ["A man in his mid-20s with a history of alcohol abuse presented with abdominal pain after 3 days of drinking", "He described the pain as a sharp, left upper-quadrant pain migrating to the epigastric region for 24 hours", "The patient had a history of multiple episodes of hematemesis attributed to alcoholic gastritis", "however, at this admission, he indicated he had not experienced vomiting", "He admitted to early satiety over the past few months", "Physical examination revealed a soft nondistended abdomen, but it was very tender in the epigastric region without rebound or guarding", "There was no palpable mass", "Elevated results of liver function tests, as well as amylase and lipase levels consistent with alcoholic pancreatitis, were noted", "Magnetic resonance imaging, computed tomographic imaging, and upper endoscopy findings are shown in Figure 1. Biopsy results of the mass were reported as normal gastric mucosa", "Diagnostic imaging", "A, Computed tomographic scan and magnetic resonance imaging (inset) demonstrating a heterogeneous mass lesion in the fundus and body of the stomach", "B, Upper endoscopic image directly visualizing the mass intruding into the lumen of the stomach"], "s1": [0, 1, 2, 3, 4, 5, 7], "s2": [6, 8, 9, 10, 11]} {"key": 1122, "questions": "What Is Your Diagnosis?", "options": [{"label": "A", "disease": "Levamisole-induced vasculopathy"}, {"label": "B", "disease": "Mixed cryoglobulinemia"}, {"label": "C", "disease": "Propylthiouracil-induced hypersensitivity vasculitis"}, {"label": "D", "disease": "Septic vasculitis"}], "answer_idx": "C", "symptoms": ["A woman in her 70s with a medical history of hyperthyroidism, hypertension, and neuropathy was admitted to the hospital with a 6-day history of painful lesions involving both ears", "Initially, the left ear developed very painful lesions that resembled bruises", "Subsequently, her right ear became involved and developed similar lesions, but she denied any other areas being affected", "She had recently been treated for a urinary tract infection with trimethoprim-sulfamethoxazole and levofloxacin", "The lesions appeared while she was taking levofloxacin", "however, on admission the antibiotic therapy was continued", "Her other medications included propylthiouracil, amlodipine besylate, pravastatin sodium, solifenacin succinate, and gabapentin", "On physical examination, the patient had localized retiform purpura on the right earlobe and bilateral helices that were tender to palpation (Figure, A)", "Findings from laboratory tests, including a hypercoagulability workup, complete blood cell count, comprehensive metabolic panel, urinalysis, cryoglobulins, cytoplasmic antineutrophil cytoplasmic antibodies, and complement levels were all unremarkable", "However, the test result for perinuclear antineutrophil cytoplasmic antibodies (p-ANCAs) was positive", "A punch biopsy for histopathologic examination was performed (Figure, B and C)", "A, Retiform purpura on the right helix", "B, Abundant red blood cell extravasation and fibrin thrombi (hematoxylin-eosin, original magnification ×10)", "C, Leukocytoclastic vasculitis and fibrin thrombi (hematoxylin-eosin, original magnification ×40)"], "s1": [0, 1, 2, 7, 9, 11], "s2": [3, 4, 5, 6, 8, 10, 12, 13]} {"key": 1123, "questions": "What is Your Diagnosis?", "options": [{"label": "A", "disease": "Granulomatosis with polyangiitis"}, {"label": "B", "disease": "Microscopic polyangiitis"}, {"label": "C", "disease": "Leukocytoclastic vasculitis"}, {"label": "D", "disease": "Polyarteritis nodosa"}], "answer_idx": "D", "symptoms": ["A woman in her 40s with a history of atopy presented with a 1-year history of asymptomatic red-brown patches on the distal extensor extremities", "The lesions began on the bilateral lower legs and spread to the thighs, buttocks, hands, forearms, and proximal arms", "Physical examination revealed numerous stellate red-brown patches without induration, scale, or ulceration (Figure, A)", "Review of systems was notable for episodic muscle cramping and numbness involving the lower extremities", "The patient had no notable recent medical illnesses, illicit drug use, or new medications", "An incisional wedge biopsy was performed on a lesion from the lower extremity, and the specimen was stained with hematoxylin-eosin (Figure, B and C)", "A, Stellate red-brown patches on the patient's right lower extremity", "B, Specimen from a lower-extremity lesion (original magnification ×10)", "C, Same specimen as shown in panel B but at higher magnification (×20)"], "s1": [0, 1, 2, 3, 4], "s2": [5, 6, 7, 8]} {"key": 1124, "questions": "What Is Your Diagnosis?", "options": [{"label": "A", "disease": "Basal cell carcinoma"}, {"label": "B", "disease": "Apocrine poroma"}, {"label": "C", "disease": "Congenital nevus"}, {"label": "D", "disease": "Seborrheic keratosis"}], "answer_idx": "B", "symptoms": ["A man in his 30s with no medical history of illness presented with a lesion of concern on his left scalp", "The patient noted that the lesion had slowly enlarged over the preceding 5 years from when he first noticed it as a small bump", "He denied a history of trauma prior to its development", "The patient denied any spontaneous bleeding, crusting, or itching, and he was primarily concerned about its growth", "There was no personal or family history of malignancy", "On physical examination, the patient’s left frontal scalp revealed a mobile 1.2-cm mammillated, soft, brown plaque with protruding terminal hairs (Figure, A)", "The remainder of the patient’s skin examination revealed no other concerning lesions", "There was no lymphadenopathy present", "Dermoscopically, the lesion was characterized by irregular pigment globules, milialike cysts, and focal arborizing vessels (Figure, B)", "A shave biopsy was performed for histopathologic examination (Figure, C and D)"], "s1": [0, 1, 2, 3, 4, 6, 7], "s2": [5, 8, 9]} {"key": 1125, "questions": "What is your diagnosis?", "options": [{"label": "A", "disease": "Alzheimer disease, frontal variant"}, {"label": "B", "disease": "Frontotemporal dementia"}, {"label": "C", "disease": "Progressive supranuclear palsy"}, {"label": "D", "disease": "Corticobasal degeneration"}], "answer_idx": "C", "symptoms": ["A woman in her 50s was referred to a neurologist for a history of gait, balance, personality, and judgment changes over 4 years", "Her family noted that her first symptoms appeared at age 55, with subtle gait dysfunction and then evident imbalance", "By age 57, there was concern over progressive withdrawal from social interchange, personality changes, and impairment of judgment", "By age 59, she was experiencing bouts of inappropriate out-of-context laughter and tears, inattention, daytime fatigue, and frank memory impairment", "She developed visuoperceptive changes and difficulties using the television remote control device", "She developed obsessive checking behaviors", "She started having swallowing difficulties but nonetheless had unexpected weight gain", "She developed progressively worse motoric problems, with stiffness, abnormal posturing, a left toe pointing upward, and falls", "Her medical history was significant only for restless legs syndrome, hypertension, hyperlipidemia, myopia, and a deviated nasal septum", "Her family history revealed that her father was alive and well at approximately age 90 years, and her mother had died at approximately age 80 years with a history of neuropsychiatric disease and gait change", "There was no history of tobacco, excess alcohol, or recreational drug use", "General examination findings were noncontributory", "Neurological examination revealed abnormal mental status, with full orientation and good registration, concentration, digit span, and language function but with poor calculations and constructions (Figure 1A) and decreased short-term memory", "Nonetheless, her Mini-Mental State Examination score was 28 of 30. Cranial nerve examination revealed some decreased upgaze and mild facial masking", "Motor examination revealed rigidity and dystonic posturing in the left arm and leg, imbalance on gait, decreased arm swing, and retropulsion but no significant tremor", "Sensory examination findings were normal", "Reflex examination showed mild left hyperreflexia with an upgoing left toe", "Blood and urine laboratory study results were within normal limits", "Cerebrospinal fluid was unremarkable except for biomarkers that showed a mid-range β-amyloid 42 level of 530 pg/mL and normal total tau and phosphotau levels of 182 and 25 pg/mL, respectively", "Magnetic resonance imaging of the brain was reported as unremarkable (Figure 1B)", "Positron emission tomography (PET) imaging of the brain revealed right frontotemporoparietal hypometabolism with some sensorimotor sparing (Figure 1C), which was neither typical nor atypical of Alzheimer disease", "Electroencephalography showed left frontotemporal sharp waves and slowing", "She had continued progressive decline in cognition and motor function and died almost mute and bedridden in her early 60s after a 9-year course of illness", "A, Patient testing", "Constructional apraxia is seen in her copying of the top row template figures, with preserved writing in script", "B, Brain magnetic resonance imaging at the time of presentation", "The top panel shows selected sagittal T1-weighted images, and the bottom panel shows selected axial T1-weighted images", "C, Brain positron emission tomography using fluorodeoxyglucose F 18 tracer in the axial plane", "Frontal and temporal hypometabolism is seen, more marked on the right, with some apparent sensorimotor sparing bilaterally"], "s1": [0, 1, 2, 3, 4, 5, 6, 7, 12, 13, 14, 16, 20, 21, 22, 24, 28], "s2": [8, 9, 10, 11, 15, 17, 18, 19, 23, 25, 26, 27]} {"key": 1126, "questions": "What Is Your Diagnosis?", "options": [{"label": "A", "disease": "Psoriasis"}, {"label": "B", "disease": "Ichthyosis"}, {"label": "C", "disease": "Circumscribed juvenile pityriasis rubra pilaris"}, {"label": "D", "disease": "Atopic dermatitis"}], "answer_idx": "C", "symptoms": ["A healthy 7-year-old girl presented with a 2-month history of an itchy rash on her elbows, knees, hands, and feet", "The patient was otherwise healthy with normal development and growth", "She had no other medical problems and was not taking any medications", "The rash started as small pink bumps on her knees and elbows, spreading to her hands and feet and progressively thickening", "The patient’s history was otherwise unremarkable", "she had no pets, had no recent travel outside the United States, and her vaccinations were up to date", "On physical examination, the patient had well-defined, follicular, pink, scaly papules coalescing into thin plaques on her knees (Figure 1A), elbows, and dorsal hands and feet, as well as diffuse, marked orange-red hyperkeratosis and hyperlinearity of her palms with clear demarcation at the wrists (Figure 1B)", "A, Well-defined follicular, pink, scaly papules coalescing into plaques on the patient’s knees", "B, Marked orange-red hyperkeratosis of the patient’s palms", "Initial treatment with hydrocortisone valerate cream, 0.2%, provided minimal improvement", "A 4-mm skin punch biopsy was performed"], "s1": [0, 3, 6, 7, 8], "s2": [1, 2, 4, 5, 9, 10]} {"key": 1127, "questions": "What Is Your Diagnosis?", "options": [{"label": "A", "disease": "Alveolar rhabdomyosarcoma"}, {"label": "B", "disease": "Nasal alveolar soft part sarcoma"}, {"label": "C", "disease": "Malignant melanoma"}, {"label": "D", "disease": "Juvenile nasopharyngeal angiofibroma"}], "answer_idx": "B", "symptoms": ["A preteen boy presented with a 1-year history of right-sided nasal obstruction and a 4-day history of intermittent right-sided epistaxis following blunt trauma to the nose", "The epistaxis occurred 3 to 4 times a day and resolved with pressure", "He did not have facial pain, facial paresthesia, or visual changes", "There was no family or personal history of bleeding disorders", "Nasal endoscopy revealed a large, well-vascularized, polypoid mass filling the right anterior nasal cavity", "A computed tomographic scan showed a right nasal cavity mass (4.5 × 1.7 cm) extending to the posterior choana with opacification and bony remodeling of the right maxillary sinus", "The mass had heterogeneous intermediate signal intensity on T2-weighted magnetic resonance imaging (Figure, A)", "The patient was taken to the operating room for biopsy and possible excision of the nasal mass", "As the lesion was biopsied, there was brisk bleeding", "However, the lesion was found to have a narrow pedicle of attachment and was resected in its entirety", "The mass was based on the superior aspect of the nasal septum and cribriform plate", "Hematoxylin-eosin stain showed a sheet-like proliferation of epithelioid and polygonal cells with pale eosinophilic granular cytoplasm and relatively uniform vesicular nuclei (Figure, B)", "The cells were arranged in nests", "Branching, staghorn-like blood vessels were scattered throughout the tumor", "The tumor cells stained positive for smooth muscle actin (Figure, C), muscle specific actin, Bcl-2, INI-1, and transcription factor E3 (Figure, D)", "A, T2-weighted magnetic resonance imaging (MRI) scan of a right nasal cavity mass extending to the posterior choana with heterogeneous intermediate signal intensity", "B and C, Histopathologic images", "B, A sheetlike proliferation of epithelioid and polygonal cells with pale eosinophilic granular cytoplasm and relatively uniform vesicular nuclei (hematoxylin-eosin, original magnification ×40)", "C, The tumor cells stained positive for smooth muscle actin (original magnification ×20)", "D, The tumor cells stained positive for transcription factor E3 (original magnification ×20)"], "s1": [0, 1, 2, 3, 4, 5, 6, 7, 8, 9, 10], "s2": [11, 12, 13, 14, 15, 16, 17, 18, 19]} {"key": 1128, "questions": "What Is Your Diagnosis?", "options": [{"label": "A", "disease": "Metastatic neuroblastoma"}, {"label": "B", "disease": "Rhabdomyosarcoma"}, {"label": "C", "disease": "Lymphoma"}, {"label": "D", "disease": "Lipoma"}], "answer_idx": "B", "symptoms": ["A young boy with mild developmental delay and autism presented with a 1-week history of progressive left periorbital swelling", "His parents reported that he complained of left orbital pain just before they noticed the swelling, and he was treated with cold compresses with only slight improvement", "He had never experienced these symptoms before", "The patient had not experienced diplopia, change in visual acuity, nasal obstruction, change in oral intake, weight loss, recent upper respiratory infection, sick contacts, fevers, or chills", "At the time of presentation, the patient was breathing comfortably and was afebrile", "Findings from his head and neck examination were significant for a firm mass over the left zygoma and lateral infraorbital rim, 2 × 3 cm in diameter", "The mass was nontender, nonerythematous, and fixed to the underlying zygoma and lateral infraorbital rim", "There was no cervical lymphadenopathy", "Ophthalmology examination revealed intact extraocular movements, bilateral visual acuity of 20/50, and no evidence of afferent papillary defect", "Computed tomographic (CT) images showed a round, soft-tissue mass with osseous destruction and erosion into the zygoma, infratemporal fossa, inferolateral orbit, and maxillary sinus (Figure, A and B)", "Concern for orbital involvement prompted magnetic resonance imaging (MRI) of the maxillofacial region (Figure, C and D)", "He was taken to the operating room the following day for open biopsy", "A and B, Computed tomographic images of the maxillofacial region showing a round, soft-tissue mass", "A, Coronal view", "B, Axial view", "C and D, Magnetic resonance imaging (MRI) scans of the maxillofacial region", "C, T2-weighted MRI scan, axial view", "D, Postcontrast T1-weighted MRI scan, coronal view"], "s1": [0, 1, 2, 3, 4, 5, 6, 7, 8], "s2": [9, 10, 11, 12, 13, 14, 15, 16, 17]} {"key": 1129, "questions": "What Is Your Diagnosis?", "options": [{"label": "A", "disease": "Keratoacanthoma"}, {"label": "B", "disease": "Chondrodermatitis nodularis chronica helicis"}, {"label": "C", "disease": "Glomus tumor"}, {"label": "D", "disease": "Rheumatoid nodule"}], "answer_idx": "B", "symptoms": ["A man in his 40s presented with a 2-year history of a few small painful nodules on both ears", "The nodules were painful on touch, especially while the patient was sleeping in a lateral position", "There was no history of trauma", "However, he used to wear a cap, which created pressure and friction on the upper part of both ears", "On examination, the free border of the helix of both the ears showed a few dull red, dome-shaped, firm nodules 2 to 3 mm in size with central crusting (Figure, A)", "They were tender on palpation", "No cervical lymphadenopathy was observed", "One of the nodules on the right ear was excised", "histopathologic features of the nodule are shown in the Figure, B", "It showed a sharply defined, centrally depressed ulcer covered by a hyperkeratotic parakeratotic scale, occasional bacterial colonies, and plasma", "The adjacent epidermis showed hyperplasia", "The base of ulcer showed eosinophilic degeneration of collagen and solar elastosis", "In addition, there were a few proliferating blood vessels and mild lymphomononuclear infiltrate", "Underlying cartilage was not seen in the section", "On serial sectioning, cartilage was seen and found to be normal", "A, An ear with dull red, dome-shaped, firm nodules 2 to 3 mm in size with central crusting (arrowheads)", "B, Histopathologic image (hematoxylin-eosin, original magnification ×100)"], "s1": [0, 1, 2, 3, 4, 5, 6, 7, 15], "s2": [8, 9, 10, 11, 12, 13, 14, 16]} {"key": 1130, "questions": "What Is Your Diagnosis?", "options": [{"label": "A", "disease": "Osteolipoma"}, {"label": "B", "disease": "Myositis ossificans"}, {"label": "C", "disease": "Liposarcoma"}, {"label": "D", "disease": "Calcified teratoma/dermoid"}], "answer_idx": "A", "symptoms": ["A healthy nonsmoking woman in her 30s presented with a 6-year history of a nontender mass on the right side of the inferior face, overlying the mandible", "She first noticed the mass following an uncomplicated dental procedure", "It grew slowly for a few years but then stabilized", "She stated that it was sensitive to cold liquids but was otherwise asymptomatic", "Results from routine laboratory tests and hematologic markers were normal", "Examination revealed a roughly 4 × 2.5-cm, nontender, firm mass anterior to the right mandibular parasymphysis", "The mass was fully mobile relative to the underlying mandible and could be seen bulging into the gingivolabial sulcus", "There were no overlying skin or mucosal changes, and the skin and mucosa moved freely over the mass", "Computed tomographic (CT) imaging demonstrated a well-defined, hyperdense, mildly heterogeneous mass with few focal areas of fat attenuation along the inferior aspect of the lesion (Figure, A-C)", "The mass was separate from the right mandibular parasymphysis without evidence of periosteal reaction or erosion into the underlying bone", "The surrounding soft tissues appeared unremarkable", "No additional lesions were noted", "The patient was taken to the operating theater, where a firm, multilobulated, yellow mass was excised through a gingivolabial incision (Figure, D)", "The mass was not fixed to any adjacent structures", "A-C, Computed tomographic images", "Axial views of soft tissue (A) and bony windows (B) demonstrating an ossified hyperdense mass within the right gingivobuccal space with well-defined margins", "D, The intraoperative photograph shows the well-defined mass, just prior to removal, arising in the right gingivobuccal space, separate from the underlying mandible"], "s1": [0, 1, 2, 3, 4, 5, 6, 7], "s2": [8, 9, 10, 11, 12, 13, 14, 15, 16]} {"key": 1131, "questions": "What Is Your Diagnosis?", "options": [{"label": "A", "disease": "Inflammatory demyelinating processes such as multiple sclerosis"}, {"label": "B", "disease": "Neoplasms such as multicentric glioma, glioblastoma multiforme, and metastases"}, {"label": "C", "disease": "Infectious causes including tuberculosis, neurosyphilis, and toxoplasmosis"}, {"label": "D", "disease": "Primary central nervous system lymphoma"}], "answer_idx": "D", "symptoms": ["A previously healthy woman in her 50s presented with complaints of increasing forgetfulness, fatigue, and depressed mood after dealing with the deaths of several close relatives", "She was diagnosed with depression and began treatment with antidepressant medication", "After 8 months, she had worsening complaints", "At that time, a family member reported that her memory problems had progressed", "She was no longer able to find misplaced objects, remember appointments, or handle financial affairs", "She was often subdued, her mood depressed, and her sleeping time increased", "Clinical examination at that time revealed a flattened affect and a 22 of 30 Mini-Mental State Examination score", "Findings were otherwise normal, and there were no focal neurologic deficits", "Basic laboratory workup results were unremarkable", "She was referred to a neurologist, who ordered contrast-enhanced magnetic resonance imaging (MRI) of the brain", "The MRI (Figure) revealed extensive ill-defined areas of FLAIR (fluid-attenuated inversion recovery) and T2 signal abnormalities with additional enhancing nodular foci located within deep white matter structures, the basal ganglia, the caudate, the pons, and the midbrain", "Also identified was enhancement of the interpeduncular cistern and cerebral peduncles, suggestive of leptomeningeal involvement", "Magnetic resonance images (MRIs) of the head obtained 8 months after patient’s first clinical visit", "A, Axial T2-weighted MRI of the head at the level of the lateral ventricles demonstrates extensive ill-defined areas of increasing signal with only minimal associated mass effect involving the frontal lobe white matter, greater on the left than on the right", "B, Contrast-enhanced T1-weighted MRI at the same level demonstrates abnormal patchy enhancement of the head of the left caudate nucleus and the deep anterior left frontal lobe white matter", "Neoplasms such as multicentric glioma, glioblastoma multiforme, and metastases"], "s1": [0, 1, 2, 3, 4, 5, 6, 7, 8], "s2": [9, 10, 11, 12, 13, 14, 15]} {"key": 1132, "questions": "What Is Your Diagnosis?", "options": [{"label": "A", "disease": "Anterior thoracolumbar spinal cord herniation"}, {"label": "B", "disease": "Malignant neoplasm of the spinal cord"}, {"label": "C", "disease": "Spinal dural arteriovenous fistula"}, {"label": "D", "disease": "Spinal extradural arachnoid cyst"}], "answer_idx": "D", "symptoms": ["A 36-year-old man presented with 2 weeks of episodic numbness and paresthesias in both feet, with symptom-free intervals between episodes", "Episodes progressed in frequency and duration, occurring several times daily and lasting for hours", "His symptoms were prominent after activity and reliably triggered by straining with urination and terminated by lying supine", "The patient also reported a 1-week history of urinary retention requiring intermittent self-catheterization", "Neurological examinations both during and between episodes revealed symmetric patellar and Achilles hyperreflexia with bilateral crossed adductor response, but otherwise the results were normal", "A thoracic spine magnetic resonance imaging scan with fast imaging employing steady-state acquisition (FIESTA) sequences was obtained (Figure)", "A dynamic computed tomographic myelogram demonstrated a lesion compressing the dorsal spinal cord at the T7 level (Video)", "Magnetic resonance imaging (MRI) of the thoracic spine using fast imaging employing steady-state acquisition (FIESTA) sequences", "The MRI scan with FIESTA sequences revealed a blurry spinal cord signal alteration at the T7-T8 level with deformation of the cord posteriorly (arrowhead)"], "s1": [0, 1, 2, 3, 4], "s2": [5, 6, 7, 8]} {"key": 1133, "questions": "What Is Your Diagnosis?", "options": [{"label": "A", "disease": "Acute cholecystitis"}, {"label": "B", "disease": "Xanthogranulomatous cholecystitis"}, {"label": "C", "disease": "Gallbladder cancer"}, {"label": "D", "disease": "Adenomyomatosis of the gallbladder"}], "answer_idx": "B", "symptoms": ["A woman in her 60s presented to the emergency department with a 1-day history of abdominal pain", "She reported acute onset of pain that woke her up from sleep, was crampy in nature, and was localized to the right upper quadrant", "She denied any nausea or vomiting and there was no association with eating", "She reported similar pain over the previous month but not to the current severity", "There was no history of fevers/chills, diarrhea, melena, or hematochezia", "She had never had a colonoscopy", "Her medical history included hypertension and gastroesophageal reflux", "She had no smoking or alcohol history", "Her only operations in the past were nononcologic, nongastrointestinal procedures", "On examination, her heart and lungs were normal", "Her abdomen was soft but tender in the right upper quadrant", "Laboratory evaluation revealed a normal compete blood cell count, including a white blood cell count of 8900/µL (to convert to ×109 per liter, multiply by 0.001)", "Results from a complete metabolic panel, including lipase, and from carcinoembryonic antigen and carbohydrate antigen 19-9 testing were normal", "Ultrasonography revealed asymmetric thickening of the gallbladder wall without pericholecystic fluid", "Computed tomographic scans of the abdomen with oral and intravenous contrast were obtained (Figure 1)", "Computed tomographic scan with coronal (A) and sagittal (B) views showing a soft-tissue mass associated with the fundus of the gallbladder"], "s1": [0, 1, 3, 4, 5, 6, 7, 8, 9, 10], "s2": [2, 11, 12, 13, 14, 15]} {"key": 1134, "questions": "What Is the Diagnosis?", "options": [{"label": "A", "disease": "Sarcoidosis"}, {"label": "B", "disease": "Syphilis"}, {"label": "C", "disease": "Indeterminate cell histiocytosis"}, {"label": "D", "disease": "Adult T-cell leukemia/lymphoma"}], "answer_idx": "D", "symptoms": ["An elderly woman of Bahamian heritage with Fitzpatrick VI skin type presented with a 1-week history of indolent papules all over her body", "She was otherwise asymptomatic", "Findings from physical examination were notable for discrete skin-colored to pink papules coalescing into plaques on her face, trunk (Figure, A and B), and all 4 extremities", "There was no scale or other epidermal change", "A punch biopsy for hematoxylin-eosin staining was performed (Figure, C)", "The patient returned 1 week later with worsening cutaneous eruptions and new complaints of fever, weakness, and malaise", "Repeat physical examination revealed splenomegaly and axillary lymphadenopathy", "A peripheral blood smear (Figure, D) and other diagnostic tests were performed", "A, Discrete skin-colored to pink papules on the abdomen", "B, Discrete papules coalescing into plaques on superior gluteal cleft", "C, Punch biopsy specimen (hematoxylin-eosin)", "D, Peripheral blood smear (hematoxylin-eosin, original magnification ×100)"], "s1": [0, 1, 2, 3, 5, 6, 8, 9], "s2": [4, 7, 10, 11]} {"key": 1135, "questions": "What Is the Diagnosis?", "options": [{"label": "A", "disease": "Bullous pemphigoid"}, {"label": "B", "disease": "Stevens-Johnson syndrome"}, {"label": "C", "disease": "Paraneoplastic pemphigus"}, {"label": "D", "disease": "Pemphigus vulgaris"}], "answer_idx": "C", "symptoms": ["A man in his 70s with a history of T2N2 melanoma, locally metastatic squamous cell carcinoma, and non-Hodgkin follicular lymphoma presented with weakness, a blistering erythematous eruption, and painful oral lesions of 1 month’s duration", "On physical examination, the patient was in severe discomfort with limited movement owing to cutaneous pain", "He was cachectic and had widespread erosions and tense and flaccid bullae with overlying crust affecting his trunk and extremities (including the palms and soles) but sparing his scalp (Figure, A)", "He had severe stomatitis with localized hemorrhagic lesions, shaggy erosions on the buccal and gingival mucosa, and erythematous macules on his hard and soft palates (Figure, B)", "Findings from laboratory tests, including a complete blood cell count and a comprehensive metabolic panel, were unremarkable except for mild anemia", "Results from a computed tomographic scan of the chest and neck showed adenopathy suspicious for lymphoproliferative disorder or potentially metastatic disease", "A punch biopsy for histopathologic examination (Figure, C) and direct immunofluorescence (DIF) was performed, and serum was tested for indirect immunofluorescence (IIF) on rodent epithelium (Figure, D)", "A, Erosions, tense and flaccid bullae, and overlying crust over the dorsal side of the foot", "B, Severe stomatitis and hemorrhagic crust", "C, Histopathologic image of a biopsy specimen showing suprabasal acantholysis and a dermal infiltrate of lymphocytes, histiocytes, and scattered eosinophils (hematoxylin-eosin, original magnification ×20)", "D, Positive indirect immunofluorescence test of rat urinary bladder epithelium"], "s1": [0, 1, 2, 3, 7, 8], "s2": [4, 5, 6, 9, 10]} {"key": 1136, "questions": "What Is the Diagnosis?", "options": [{"label": "A", "disease": "Cellulitis"}, {"label": "B", "disease": "Tumid lupus erythematosus"}, {"label": "C", "disease": "Gyrate erythema"}, {"label": "D", "disease": "Erythema nodosum"}], "answer_idx": "B", "symptoms": ["A woman in her 20s with a medical history significant for systemic lupus erythematosus (SLE) presented with tender erythematous nodules after undergoing right knee arthroscopy (Figure, A)", "SLE had been diagnosed ten years previously after she presented with a positive antinuclear antibody titer (most recent titer, 1:1280), arthralgias, leukopenia, oral ulcers, and a malar erythematous eruption", "There was no history of discoid or tumid lupus lesions", "Initially, the lesions were thought secondary to soft-tissue infection and were treated with courses of clindamycin, vancomycin, and doxycycline", "Physical examination revealed poorly demarcated erythematous plaques and nodules, tender to palpation, ranging from the distal thigh to the lower leg, adjacent to arthroscopy sites", "Lesions around the suture sites appeared more violaceous", "A punch biopsy of the lesion was performed (Figure, B and C)", "A, Clinical photograph of right knee demonstrating ill-defined erythematous plaques and nodules adjacent to 2 small incisional scars from previous arthroscopy", "B and C, Photomicrographs of punch biopsy specimen", "B, Hematoxylin-eosin, original magnification ×4. C, Hematoxylin-eosin, original magnification ×10."], "s1": [1, 2, 3, 4, 5], "s2": [0, 6, 7, 8, 9]} {"key": 1137, "questions": "What Is Your Diagnosis?", "options": [{"label": "A", "disease": "Drug intake"}, {"label": "B", "disease": "Kawasaki syndrome"}, {"label": "C", "disease": "Vaccination reaction"}, {"label": "D", "disease": "Hand-foot-and-mouth disease"}], "answer_idx": "D", "symptoms": ["A 7-year-old girl presented to the dermatology clinic with a 4-week history of progressive nail changes", "Her mother reported a sudden onset of brittle nails with cracks followed by painless sloughing of nails", "Initially only the fingernails had been affected, but the toenails soon showed the same pathology", "The family was concerned and suspected an internal disease or vitamin deficiency", "The child was otherwise well and no previous skin or nail conditions were known", "On clinical examination, most fingernails showed semilunar whitish grooves in the middle of the nail plate (Figure, A)", "The toenails had transverse partial cracks of the proximal nail plate with distal onycholysis but normal proximal nail growth (Figure, B)", "Findings on clinical examination", "A, Fingernails showed semilunar whitish grooves in the middle of the nail plate", "B, Toenails had transverse partial cracks of the proximal nail plate with distal onycholysis"], "s1": [0, 1, 2, 3, 4], "s2": [5, 6, 7, 8, 9]} {"key": 1138, "questions": "What Is Your Diagnosis?", "options": [{"label": "A", "disease": "Olfactory neuroblastoma"}, {"label": "B", "disease": "Sinonasal teratocarcinosarcoma"}, {"label": "C", "disease": "Small cell carcinoma"}, {"label": "D", "disease": "Alveolar rhabdomyosarcoma"}], "answer_idx": "B", "symptoms": ["A man in his 50s presented with a 7-month history of worsening, primarily left-sided, sinus obstruction and congestion with associated hyposmia, facial pressure, and headaches", "He had no history of sinus disease or surgery and had no epistaxis, weight loss, or visual symptoms", "Computed tomography showed a 3.1-cm mass in the left ethmoid sinus and nasal cavity (Figure, A)", "Positron emission tomographic scanning showed no evidence of metastasis", "Nasal endoscopy revealed a friable, polypoid mass in the ethmoid sinus with extension into the nasal cavity and superiorly to the skull base", "A biopsy specimen showed a heterogeneous tumor with small, angulated, blue cells with hyperchromatic nuclei, apoptosis, and mitosis, malignant glands lined by cuboidal cells with round nuclei, prominent nucleoli, and eosinophilic cytoplasm", "The glands had luminal mucin production (Figure, B)", "Focally, there were also scattered round cells with abundant eosinophilic, refractile cytoplasm, features of rhabdomyoblasts (Figure, C)", "These latter cells were strongly positive for desmin (Figure, D) and myogenin", "Ethmoid sinus mass", "A, Computed tomographic scan", "The arrowhead indicates the mass", "B, Blastema as indicated by the asterisk and malignant glands as indicated by the arrowhead", "C, Rhabdomyoblasts as indicated by the arrowheads (hematoxylin-eosin, original magnification ×40)", "D, Desmin immunohistochemistry with strong positive staining in the rhabdomyoblasts (original magnification ×20)"], "s1": [0, 1, 3, 9], "s2": [2, 4, 5, 6, 7, 8, 10, 11, 12, 13, 14]} {"key": 1139, "questions": "What Is Your Diagnosis?", "options": [{"label": "A", "disease": "Juvenile nasopharyngeal angiofibroma"}, {"label": "B", "disease": "Lobular capillary hemangioma"}, {"label": "C", "disease": "Inverting papilloma"}, {"label": "D", "disease": "Respiratory epithelial adenomatoid hamartoma"}], "answer_idx": "B", "symptoms": ["A woman in her 50s presented with an 8-week history of increasing headache, hyposmia, hypogeusia, and nasal congestion", "Endoscopic nasopharyngoscopy revealed a large polypoid nasal mass occupying the left nasal roof and completely obstructing the nasal passage", "Contrast-enhanced T1-weighted magnetic resonance imaging (MRI) revealed a 1.7 × 3.5 × 2.6-cm soft-tissue mass within the left ethmoid cavity, eroding through the cribriform plate with adjacent dural enhancement (Figure, A)", "Intraoperative pathologic examination showed polypoid spindle cell proliferation", "Histopathologic findings showed respiratory epithelium with submucosal vascular proliferation with no atypia among the lining endothelial cells", "The Figure, B, shows a central vessel surrounded by lobules of endothelial lined capillaries", "The Figure, C, highlights the endothelial cells of the numerous capillaries and the central vessel", "A, Contrast-enhanced T1-weighted magnetic resonance imaging of a soft-tissue mass within the left ethmoid cavity", "B, Central vessel surrounded by lobules of endothelial-lined capillaries (hematoxylin-eosin, original magnification ×20)", "C, Endothelial cells of the numerous capillaries and the central vessel on CD34 staining (original magnification ×20)"], "s1": [0, 1, 2, 3], "s2": [4, 5, 6, 7, 8, 9]} {"key": 1140, "questions": "What Is Your Diagnosis?", "options": [{"label": "A", "disease": "Exuberant osteophytes"}, {"label": "B", "disease": "Diffuse idiopathic skeletal hyperostosis"}, {"label": "C", "disease": "Osteochondroma"}, {"label": "D", "disease": "Osteoid osteoma"}], "answer_idx": "C", "symptoms": ["A woman in her 60s presented with a 10-year history of oropharyngeal discomfort and recent episodes of snoring and obstructive sleep disturbance", "Although she had been experiencing a gradually increasing lumpy feeling on swallowing for at least 6 months, she did not experience of dysphagia, dyspnea, or pain", "Physical examination revealed a mass measuring approximately 4 × 3 cm and covered with normal mucosa on the posterior oropharyngeal wall", "This tumor was firm, nontender, and firmly adherent (Figure, A)", "Computed tomography (CT) showed a segmented bony lesion located at the odontoid process and the body of the C2 vertebra (Figure, B)", "This mass showed high signal intensity with a hypointense rim in both T1- and T2-weighted magnetic resonance images (MRI) (Figure, C and D, respectively)", "Polysomnography was performed and showed an apnea-hypopnea index of 58.7/h and a minimal oxygen saturation of 76%, which led to a diagnosis of obstructive sleep apnea", "The mass was considered causative of obstructive sleep apnea", "Hence, we performed total excision of the mass using a per-oral approach", "A, Photograph of an oropharyngeal mass", "B, Computed tomographic image showing a segmented bony lesion located at the odontoid process and the body of the C2 vertebra", "C, T1-weighted magnetic resonance image (MRI) of an oropharyngeal mass", "D, T2-weighted MRI of an oropharyngeal mass"], "s1": [0, 1, 6, 7, 8], "s2": [2, 3, 4, 5, 9, 10, 11, 12]} {"key": 1141, "questions": "What Is Your Diagnosis?", "options": [{"label": "A", "disease": "Mucoepidermoid carcinoma"}, {"label": "B", "disease": "Pleomorphic adenoma"}, {"label": "C", "disease": "Hemangioma"}, {"label": "D", "disease": "Granular cell tumor"}], "answer_idx": "B", "symptoms": ["A teenage girl presented with a 3-month history of a gradually enlarging soft palate mass associated with a change in her voice", "She had no associated pain, difficulty swallowing, or difficulty breathing", "She was otherwise healthy, with no medical or surgical history", "Findings from her physical examination were unremarkable with the exception of a 2 × 2-cm firm, nontender, submucosal posterior soft palate mass just left of the uvula", "There was no overlying ulceration or discoloration", "Flexible nasopharyngoscopy was normal", "Prior to presentation, a computed tomographic scan and biopsy had been performed at an outside hospital", "Magnetic resonance imaging (MRI) of the face with contrast was also performed (Figure, A-C)", "Imaging showed a well-circumscribed solid lesion with no evidence of infiltration of soft-tissue structures, the pterygopalatine fossa, or palatine nerve foramina", "There was no cervical lymphadenopathy", "The lesion was resected via a transoral approach and was easily dissected from the surrounding tissue (Figure, D)", "The overlying mucosa was spared"], "s1": [0, 3, 4, 8, 9, 10, 11], "s2": [1, 2, 5, 6, 7]} {"key": 1142, "questions": "What Is Your Diagnosis?", "options": [{"label": "A", "disease": "Fungal infection"}, {"label": "B", "disease": "Extramammary Paget disease"}, {"label": "C", "disease": "Bowen disease"}, {"label": "D", "disease": "Lichen sclerosis"}], "answer_idx": "B", "symptoms": ["A man in his 60s presented for further evaluation of a right inguinal and scrotal lesion (Figure 1)", "The rash initially started 4 years prior as a dime-sized, pink, scaly area in the right inguinal crease and was pruritic", "Results of a biopsy at the time were consistent with lichen simplex chronicus, and treatment with topical steroids was initiated", "Since then, the rash had failed to resolve and the affected area had increased in size to its present dimensions", "The patient had a history of prostate cancer treated with prostatectomy 4 years previously without evidence of recurrence and melanoma of the posterior neck treated with wide local excision, in addition to non–insulin-dependent diabetes mellitus and hyperlipidemia controlled with oral medications", "He does not smoke and has been in a monogamous marriage for more than 30 years", "Examination revealed a multifocal scaly, pink-red beefy plaque in the right inguinal crease extending from the superior aspect of the scrotum to the gluteal crease measuring approximately 6 × 7 cm in total area with some areas of excoriation and superficial open wounds", "An additional area of macular erythema extended superiorly along the inguinal crease toward the medial thigh", "No lymphadenopathy of the inguinal region was noted"], "s1": [0, 1, 2, 3, 6, 7, 8], "s2": [4, 5]} {"key": 1143, "questions": "What Is Your Diagnosis?", "options": [{"label": "A", "disease": "De novo kidney graft tumor"}, {"label": "B", "disease": "Recurrence of polycystic kidney disease"}, {"label": "C", "disease": "Polycystic liver disease"}, {"label": "D", "disease": "Caroli disease"}], "answer_idx": "C", "symptoms": ["A man in his 50s who previously underwent renal transplantation for polycystic kidney disease presented with worsening chronic malaise, fatigue, dyspnea, early satiety, and abdominal distention with extreme discomfort", "An abdominal magnetic resonance image is shown in Figure 1A", "Given the symptoms and magnetic resonance imaging findings, the patient was eventually taken to the operating room", "Laparotomy exposed a giant multinodular mass (Figure 1B)", "A, Abdominal magnetic resonance image", "B,Intraoperative photograph of the giant multinodular mass occupying most of the upper abdomen"], "s1": [0], "s2": [1, 2, 3, 4, 5]} {"key": 1144, "questions": "What Is Your Diagnosis?", "options": [{"label": "A", "disease": "Congenital cyst"}, {"label": "B", "disease": "Splenic abscess"}, {"label": "C", "disease": "Cystic metastasis to the spleen"}, {"label": "D", "disease": "Echinococcal disease"}], "answer_idx": "D", "symptoms": ["A woman in her 30s was referred for evaluation of a large splenic cyst identified on a computed tomographic (CT) scan", "She had previously lived in Beijing, China", "In 2011, she was evaluated in a Chinese clinic for abdominal bloating and moderate left upper quadrant pain", "she was offered a splenectomy, but she declined and pursued a second opinion at a hospital in the United States in 2012. During that evaluation, the patient reported that she had run in areas with dog feces on the ground and had eaten sheep meat (notably, not organ meat)", "she denied consuming canine meat", "Results of serologic tests for Echinococcus antibody IgG were negative", "Results of a CT scan were remarkable for a 9.7 × 9.0 × 10.5-cm anterior cyst and a smaller posterior cyst measuring 4.6 × 4.7 × 3.8 cm, both with rim calcifications (Figure 1)", "A large splenic cyst with calcified capsule is present in the anterior aspect of the spleen, with a smaller cyst in the posterior aspect of the spleen, also with calcified capsule", "Arrowheads indicate the location of the cysts", "Under higher resolution, debris is noted within the larger cyst"], "s1": [0, 1, 2, 3, 4], "s2": [5, 6, 7, 8, 9]} {"key": 1145, "questions": "What Is the Diagnosis?", "options": [{"label": "A", "disease": "Pyoderma gangrenosum"}, {"label": "B", "disease": "Fibrosis and ulceration caused by meperidine"}, {"label": "C", "disease": "Localized scleroderma (morphea)"}, {"label": "D", "disease": "Mycobacterium haemophilum infection"}], "answer_idx": "B", "symptoms": ["A white man in his 30s with a 7-year history of severe, mechanical, low back pain and long-standing mental depression was referred to the dermatology department for evaluation of an asymptomatic cutaneous ulcer that had developed over the past year", "Physical examination disclosed a large and deep, irregularly shaped, cutaneous ulcer on the dorsum of his right forearm", "Necrotic tissue and muscle exposure was seen at the base of the ulcer (Figure, A)", "Woody induration of skin on both forearms and on the abdominal region was also observed", "Bilateral contracture of deltoid, triceps, and biceps muscles was noted", "Active and passive range of motion was restricted at the shoulders and elbows", "No signs of joint inflammation were seen", "At the time of consultation, the patient was taking oral treatment with duloxetine hydrochloride, clonazepam, oxcarbazepine, fentanyl, sulpiride, zopiclone, omeprazole magnesium, and baclofen", "He also admitted to self-administering subcutaneous injections of meperidine, 100 mg 4 times per day, for the past 3 years, at different sites, including the deltoid areas and abdomen", "Growth from culture specimens taken from the ulcer was negative for bacteria, mycobacteria, and fungal organisms", "His serum creatinine kinase level was raised (192 U/L", "reference range, 0-174 U/L), but test results for complete blood cell count", "erythrocyte sedimentation rate", "antinuclear antibody, rheumatoid factor, aspartate aminotransferase, alanine aminotransaminase, and aldolase levels", "and serum electrophoresis were all within normal limits", "A wedge biopsy from the indurated skin of the abdominal region was performed (Figure, B and C)", "(To convert creatinine kinase to microkatals per liter, multiply by 0.0167.)A, Clinical photograph of the large, deep, irregularly shaped, cutaneous ulcer on the dorsum of the right forearm", "Three small ulcers are seen adjacent to the larger ulcer", "B and C, Histologic images of a wedge biopsy specimen from the indurated skin of the abdominal region (hematoxylin-eosin)", "B, Original magnification ×10. C, Original magnification ×40."], "s1": [0, 1, 2, 3, 4, 5, 6, 7, 8, 9, 16, 17], "s2": [10, 11, 12, 13, 14, 15, 18, 19]} {"key": 1146, "questions": "What Is The Diagnosis?", "options": [{"label": "A", "disease": "Condyloma acuminata"}, {"label": "B", "disease": "Condyloma lata"}, {"label": "C", "disease": "Verrucous herpes simplex virus"}, {"label": "D", "disease": "Granuloma inguinale"}], "answer_idx": "B", "symptoms": ["A man in his 60s presented with a slowly enlarging, painful mass on his buttock", "He recalled that a small lesion had appeared approximately 8 months prior, and over the past several months he had noted significant growth and pain", "A nonpruritic rash had also become apparent on his forearms and thighs", "Physical examination showed a 2-cm macerated pedunculated growth on the left buttock within the intergluteal cleft (Figure, A)", "An erythematous rash was noted on the forearms and thighs with fine scale present at the wrists", "A, Photograph of a 2-cm macerated pedunculated growth on the left buttock", "B, Histopathologic image of an exophytic growth on the buttock with marked epidermal hyperplasia (hematoxylin-eosin, original magnification ×40)", "C, Histopathologic image of immunohistochemical staining for spirochetes (original magnification ×400)", "A shave biopsy specimen of the exophytic growth on the buttock revealed marked epidermal hyperplasia (Figure, B) with a dense dermal infiltrate containing lymphocytes and plasma cells"], "s1": [0, 1, 3, 5, 6, 8], "s2": [2, 4, 7]} {"key": 1147, "questions": "What Is the Diagnosis?", "options": [{"label": "A", "disease": "Squamous cell carcinoma"}, {"label": "B", "disease": "Primary oral histoplasmosis"}, {"label": "C", "disease": "Mucosal leishmaniasis"}, {"label": "D", "disease": "Tubercular chancre"}], "answer_idx": "B", "symptoms": ["A 40-year-old man, who resided in the eastern part of India, was referred to us from the department of otorhinolaryngology for evaluation of a nonhealing ulcer on the tongue present for the last 4 months", "The lesion had started as a small painless mass on the right lateral aspect of the tongue, which gradually grew and subsequently ulcerated", "It was asymptomatic except for causing mild difficulty in eating", "The patient could not recall any history of trauma or tongue bite preceding the lesion", "He denied consuming tobacco or alcohol regularly", "He had type 2 diabetes mellitus for 2 years, which was well controlled with oral hypoglycemic agents", "The patient had no systemic complaints", "There was no history of high-risk behavior for human immunodeficiency virus (HIV) acquisition", "On physical examination, there was a single well-defined ulcer measuring 2 × 2 cm on the right lateral aspect of tongue with a clean base and irregular and elevated heaped-up margins (Figure 1A)", "There was no underlying induration, friability, or bleeding on manipulation", "The rest of the oral cavity was normal", "A 3-mm punch biopsy from the edge of the ulcer was taken and sent for histopathological examination (Figure 1B and C)", "A, Tongue ulcer", "B, Biopsy from the ulcer showing a mixed cell infiltrate (hematoxylin-eosin, original magnification ×40)", "C, Intracellular yeast forms within the histiocytes (original magnification ×100)"], "s1": [0, 1, 2, 3, 4, 5, 6, 7, 8, 9, 10], "s2": [11, 12, 13, 14]} {"key": 1148, "questions": "What Is Your Diagnosis?", "options": [{"label": "A", "disease": "Gastroesophageal reflux"}, {"label": "B", "disease": "Tracheoesophageal fistula"}, {"label": "C", "disease": "Esophageal perforation"}, {"label": "D", "disease": "Aspiration"}], "answer_idx": "C", "symptoms": ["Following 2 days of cough and congestion, an otherwise healthy full-term 5-week-old neonate developed difficulty breathing and apnea requiring cardiopulmonary resuscitation", "He was transported to a local emergency department", "Following a difficult orotracheal intubation with multiple attempts, the infant was transferred to a pediatric intensive care unit with a presumptive diagnosis of bronchiolitis", "During the next 2 days, he was weaned off mechanical ventilation and received oxygen via the nasal cannula", "He began feeding orally and was transferred to the general inpatient floor", "At the time of transfer, the patient was noted to have choking with feeds as well as a large volume of clear oral secretions that required frequent suctioning", "Based on these clinical findings, oral food and fluids were withheld from the patient and a fluoroscopic video swallowing study (FVSS) was performed (Figure 1 and Video)"], "s1": [0, 1, 2, 3], "s2": [4, 5, 6]} {"key": 1149, "questions": "What Is Your Diagnosis?", "options": [{"label": "A", "disease": "Carcinoma of the gallbladder"}, {"label": "B", "disease": "Acute cholecystitis"}, {"label": "C", "disease": "Xanthogranulomatous cholecystitis"}, {"label": "D", "disease": "Lymphoma"}], "answer_idx": "C", "symptoms": ["A woman in her 60s presented to the hospital because she felt abdominal pain and increasing asthenia during the previous 4 months", "She reported a weight loss of 40 kg during the previous 12 months concomitant with depression", "The patient had a history of cardiac arrhythmia, hypertension, and sigmoidectomy for diverticulitis", "Medications included lasilix, amlodipine, amiodarone, and atenolol", "On examination, the patient appeared well", "Her vital signs were normal", "The abdomen was soft without distension", "A positive Murphy sign was observed", "Her white blood cell count was 11400/μL (to convert to ×109/L, multiply by 0.001), her hemoglobin level was 11.5 g/dL (to convert to g/L, multiply by 10.0), and her platelet count was 233 ×103/μL (to convert to ×109/L, multiply by 1.0)", "The C-reaction protein level was elevated to 41 mg/L (to convert to nmol/L, multiply by 9.524)", "The results of renal and liver function, coagulation, blood level of electrolytes, total protein, albumin, antigen carcino embryonnaire, carbohydrate antigen 19-9, carbohydrate antigen 125, and β2 microglobulin tests were normal", "The patient had a normal esophagogastroduodenoscopy and colonoscopy", "Ultrasonography revealed a 10 × 8-m mass in the right hypochondrium", "Abdominal computed tomography scan revealed a thickened gallbladder wall infiltrating the liver parenchyma", "There were 3 perihepatic lymph nodes", "Magnetic resonance imaging showed a greatly enlarged gallbladder with a thickened wall without invading adjacent structures, a continuous mucosal line, and a hypoattenuated intramural nodule (Figure 1)"], "s1": [0, 1, 2, 3, 4, 5, 6], "s2": [7, 8, 9, 10, 11, 12, 13, 14, 15]} {"key": 1150, "questions": "What Is Your Diagnosis?", "options": [{"label": "A", "disease": "Cecal duplication"}, {"label": "B", "disease": "Small-bowel volvulus"}, {"label": "C", "disease": "Infantile leiomyosarcoma"}, {"label": "D", "disease": "Ovarian cyst with torsion"}], "answer_idx": "D", "symptoms": ["A 2-day-old girl (weight, 3.7 kg) developed abdominal distention and vomiting after feeding", "Her Apgar scores at 1 and 5 minutes were 7 and 9, respectively", "The infant’s postnatal course was remarkable for a febrile episode that was presumed to be due to chorioamnionitis in the mother and for which antibiotic treatment was initiated", "Findings from the prenatal ultrasound, most recently conducted at 33 weeks’ gestation, were normal", "The infant was active and well perfused, with a soft but distended abdomen and right-sided labial swelling", "Findings from the remainder of her examination were unremarkable", "An abdominal radiograph revealed dilated small-bowel loops (Figure 1A)", "Radiography of the upper gastrointestinal tract with water-soluble contrast showed a normal pattern of rotation", "Results of a suction rectal biopsy revealed ganglion cells to be present", "Serial abdominal radiographic images showed retained contrast in the terminal ileum 6 days following the gastrointestinal series (Figure 1B)", "A, Abdominal radiograph showing distended small-bowel loops with absent gas in the colon and rectum", "B, Delayed radiography of the upper gastrointestinal tract with water-soluble contrast demonstrating retained contrast in the terminal ileum 6 days after initial radiography"], "s1": [0, 1, 2, 3, 4, 5], "s2": [6, 7, 8, 9, 10, 11]} {"key": 1151, "questions": "What Is the Diagnosis?", "options": [{"label": "A", "disease": "Tubular carcinoma"}, {"label": "B", "disease": "Syringomatous adenoma of the nipple"}, {"label": "C", "disease": "Low-grade adenosquamous carcinoma"}, {"label": "D", "disease": "Florid papillomatosis of the nipple"}], "answer_idx": "B", "symptoms": ["A healthy 60-year-old woman presented with a painless subareolar nodule of her left nipple measuring 2 cm in diameter (Figure, A) that had been present for several months", "Screening mammography (Figure, B) performed 4 weeks prior revealed a large grouping of left breast subareolar calcifications extending into the skin and nipple", "Follow-up ultrasonography demonstrated no discrete abnormal mass or shadowing but did show multiple echogenic foci consistent with findings on mammography", "An incisional biopsy specimen of the mass was obtained and sent for histopathologic evaluation (Figure, C)", "A, Firm, irregular, subareolar nodule of left nipple measuring 2 cm in diameter", "B, Mammography of the left breast", "C, Biopsy specimen of the subareolar nodule (hematoxylin-eosin, original magnification ×20)"], "s1": [0, 4], "s2": [1, 2, 3, 5, 6]} {"key": 1152, "questions": "What Is the Diagnosis?", "options": [{"label": "A", "disease": "Kindler syndrome"}, {"label": "B", "disease": "Dermatitis artefacta"}, {"label": "C", "disease": "Epidermolysis bullosa simplex superficialis"}, {"label": "D", "disease": "Peeling skin disease"}], "answer_idx": "D", "symptoms": ["A 2-year-old girl was referred to our department for skin fragility since early infancy", "She had 2 older brothers, and the family had no medical history of note", "Physical examination revealed mild xerosis with superficial skin erosions and erythematous residual macules from previous erosions", "Some of the lesions had an unusual linear geographic contour and were predominantly located at areas of friction (Figure, A and B)", "Hair, nails, and mucous membranes were normal", "There was no history of blistering", "The mother explained that she often found the child peeling off skin with marked facility", "Symptoms improved in winter and worsened in summer", "Physical and mental developments were normal", "Serological markers for celiac disease were negative", "A skin biopsy was performed (Figure, C)", "A, Clinical photograph of a disease outbreak", "Skin fragility can be seen as superficial erosions distributed predominantly around the trunk, some with geometric shape", "B, Photograph showing localized affectation of the disease", "Erythemato-desquamative well-demarcated plaque with peeling borders on right foot", "C, Hematoxylin-eosin staining (original magnification ×40) of healthy skin", "Remarkable detachment of the entire stratum corneum and mild psoriasiform epidermis hyperplasia with no other significant findings"], "s1": [0, 2, 3, 6, 7, 12, 14], "s2": [1, 4, 5, 8, 9, 10, 11, 13, 15, 16]} {"key": 1153, "questions": "What is the diagnosis?", "options": [{"label": "A", "disease": "Acute lymphoblastic leukemia of the breast"}, {"label": "B", "disease": "Breast implant foreign body reaction"}, {"label": "C", "disease": "Severe postpartum mastitis"}, {"label": "D", "disease": "Burkitt lymphoma of the breast"}], "answer_idx": "D", "symptoms": ["A healthy woman in her 40s, lactating, 5 months post partum, presented with a 1-month history of enlargement and induration of both breasts (Figure, A)", "A skin biopsy specimen from the breast was obtained (Figure, B)", "She was initially diagnosed with mastitis and received several courses of antibiotics for 2 months without improvement", "Subsequently she presented with back pain, paresthesias, shooting pain in the mandibular region, and diplopia", "All of these conditions improved under treatment with oral corticosteroids, but the breast enlargement continued", "Laboratory test results highlighted a hemoglobin level of 7.6 g/dL and a platelet count of 80 000/μL", "(To convert hemoglobin to grams per liter, multiply by 10", "to convert platelets to number of cells × 109/L, multiply by 1.0.)A, Enlargement and induration of both breasts", "B, Histopathologic specimen from the breast (hematoxylin-eosin, original magnification ×40)"], "s1": [0, 1, 2, 4, 7, 8], "s2": [3, 5, 6]} {"key": 1154, "questions": "What Is Your Diagnosis?", "options": [{"label": "A", "disease": "Mycobacterial infection"}, {"label": "B", "disease": "Kerion celsi"}, {"label": "C", "disease": "Dissecting cellulitis"}, {"label": "D", "disease": "Abscess"}], "answer_idx": "B", "symptoms": ["A previously healthy 8-year-old boy presented to the emergency department for evaluation of a large, tender scalp nodule", "One month prior to presentation, the patient’s mother noticed multiple annular, scaly patches on the patient’s scalp and upper back", "Both of the patient’s brothers developed similar patches", "Two weeks before presentation, the area on the scalp grew in size, becoming a tender nodule with yellow suppurative discharge", "Results of the physical examination were notable for a somewhat tired-appearing boy with a 6-cm tender, red nodule on the right frontal scalp (Figure 1)", "The nodule had a boggy consistency and thick yellow crust and was draining yellow material", "There was no hair within this area on the scalp", "On the patient’s right upper back was an annular scaly patch", "significant posterior auricular and cervical lymphadenopathy was noted bilaterally", "View on presentation of a boggy, erythematous nodule with suppurative discharge and loss of hair"], "s1": [1, 2, 7], "s2": [0, 3, 4, 5, 6, 8, 9]} {"key": 1155, "questions": "What Is Your Diagnosis?", "options": [{"label": "A", "disease": "Lymphoma"}, {"label": "B", "disease": "Malignant transformation of an enterogenous cyst"}, {"label": "C", "disease": "Retroperitoneal sarcoma"}, {"label": "D", "disease": "Primary neoplasm arising from the kidney"}], "answer_idx": "B", "symptoms": ["A 57-year-old woman with non–insulin-dependent diabetes mellitus and no other medical history had worsening left-sided back pain over 4 months and no other symptoms", "Physical examination findings were significant for mild left flank pain, but there was no costovertebral angle tenderness", "Laboratory results were within normal limits", "Axial imaging demonstrated an exophytic, retroperitoneal mass with solid and cystic components involving the left kidney, posterior abdominal wall, and mesocolon (Figure 1)", "The mass measured 20 cm in its greatest dimension", "Findings on colonoscopy and mammography were negative", "Positron emission tomography–computed tomography did not reveal any other abnormality", "Coronal computed tomographic image showing a retroperitoneal mass with a cystic component and involvement of adjacent kidney and mesocolon", "Red arrowhead indicates descending colon", "blue arrowhead, left kidney invaded by mass", "and yellow arrowhead, retroperitoneal mass with cystic component"], "s1": [0, 1, 2, 5, 6], "s2": [3, 4, 7, 8, 9, 10]} {"key": 1156, "questions": "What Is Your Diagnosis?", "options": [{"label": "A", "disease": "Colon cancer perforation"}, {"label": "B", "disease": "Stent perforation"}, {"label": "C", "disease": "Acute mesenteric ischemia"}, {"label": "D", "disease": "Perforated diverticulitis"}], "answer_idx": "B", "symptoms": ["A 73-year-old woman presented to the emergency department with 2 days of crampy abdominal pain and obstipation and 2 months of fecaluria and pneumaturia", "She was hemodynamically stable", "Physical examination revealed hepatomegaly and left lower abdominal tenderness", "She had normal liver function test results and a carcinoembryonic antigen level of 23.8 ng/mL (to convert to micrograms per liter, multiply by 1)", "Computed tomography of the abdomen and pelvis revealed a 9-cm, obstructed sigmoid colon mass with fistulous extensions to the bladder and small bowel and numerous metastases to the liver, peritoneum, and omentum", "Pathologic analysis of an endoscopic biopsy specimen revealed a moderately differentiated colon adenocarcinoma", "The patient was referred to a gastroenterologist for stent placement (Figure 1A)", "The night after the procedure, the patient developed generalized peritonitis, and computed tomography revealed abundant free air (Figure 1B)", "A, Colonoscopic stent placement", "B, Computed tomogram depicting perforation", "The patient went to the operating room for an emergency laparotomy"], "s1": [0, 2, 4, 5, 1], "s2": [3, 6, 7, 8, 9, 10]} {"key": 1157, "questions": "What is the Diagnosis?", "options": [{"label": "A", "disease": "Erythema nodosum"}, {"label": "B", "disease": "Pancreatic panniculitis"}, {"label": "C", "disease": "Infectious panniculitis"}, {"label": "D", "disease": "α1-Antitrypsin deficiency"}], "answer_idx": "B", "symptoms": ["A man in his 60s was admitted to the hospital for an 8-day history of abdominal pain, bloating, vomiting, diarrhea, subjective fever, weight loss, and fatigue", "He also complained of multiple tender red lesions on his bilateral ankles and legs that appeared 2 days prior to admission", "Findings from a review of systems were negative for cough, sore throat, history of hepatitis, tuberculosis, sarcoidosis, inflammatory bowel disease, or sulfonamide use", "Physical examination revealed several ill-defined, exquisitely tender erythematous subcutaneous nodules on his bilateral shins, medial feet, ankles, and right lower thigh without overlying ulceration, necrosis, or drainage (Figure, A and B)", "A punch biopsy was performed, demonstrating pathognomonic histopathologic findings (Figure, C and D)"], "s1": [0, 2], "s2": [1, 3, 4]} {"key": 1158, "questions": "What Is the Diagnosis?", "options": [{"label": "A", "disease": "Warfarin necrosis"}, {"label": "B", "disease": "Calciphylaxis"}, {"label": "C", "disease": "Cutaneous cholesterol embolism"}, {"label": "D", "disease": "Intravascular B-cell lymphoma"}], "answer_idx": "C", "symptoms": ["A man in his 60s presented to the emergency department for left flank pain of 1 week’s duration", "His medical history included severe peripheral arterial disease and a remote history of ruptured abdominal aortic aneurysm", "Three months before presentation, he underwent percutaneous endovascular stenting of a left popliteal artery aneurysm", "Physical examination of the left flank showed a well-defined 5 × 12-cm purpuric patch with a darker stellate patch within it (Figure, A)", "Two punch biopsy specimens were obtained (Figure, B and C)", "A, Photograph of a purpuric patch on the left flank", "B, Biopsy specimen 1 (hematoxylin-eosin, original magnification ×40) showing needle-shaped clefts within the lumen of a blood vessel with minimal perivascular inflammation", "C, Biopsy specimen 2 (hematoxylin-eosin, original magnification ×40)", "Eccrine ducts show cellular dyshesion and necrosis"], "s1": [0, 1, 2], "s2": [3, 4, 5, 6, 7, 8]} {"key": 1159, "questions": "What Is the Diagnosis?", "options": [{"label": "A", "disease": "Bullous tinea pedis"}, {"label": "B", "disease": "Allergic contact dermatitis"}, {"label": "C", "disease": "Dyshidrosiform pemphigoid"}, {"label": "D", "disease": "Epidermolysis bullosa acquisita"}], "answer_idx": "C", "symptoms": ["A man in his 70s presented with a 2-week history of a bullous eruption localized to the hands and feet", "He denied having pruritus or spontaneous bleeding but reported discomfort from the pressure of intact bullae", "His medical history was notable for diabetes mellitus and hypertension, which was well controlled with a stable regimen of lisinopril and atenolol for several years", "He denied using any new medications or supplements or making recent dose adjustments in existing medications", "There was no history of a prior bullous eruption", "Physical examination revealed numerous small intact bullae and large erosions on the lateral feet, soles, and toes (Figure, A)", "The extent of the bullae led to difficulty with ambulation", "There were also intact vesicles on the lateral palms along with erosions demonstrating evidence of previous bullae formation", "There were no vesicular or bullous lesions on the trunk, arms, legs, or mucosal surfaces", "Hematoxylin-eosin staining of a biopsy specimen obtained from a bulla on the foot was performed (Figure, B)", "A, Numerous small bullae in various stages of healing and large erosions on the feet with areas of denuded pink dermis on clinical presentation", "B, Biopsy specimen obtained from a bulla on the foot (hematoxylin-eosin, original magnification ×600)"], "s1": [0, 1, 5, 6, 7, 8, 10], "s2": [2, 3, 4, 9, 11]} {"key": 1160, "questions": "What Is Your Diagnosis?", "options": [{"label": "A", "disease": "Granuloma annulare"}, {"label": "B", "disease": "Neonatal lupus erythematosus"}, {"label": "C", "disease": "Tinea capitis"}, {"label": "D", "disease": "Annular urticaria"}], "answer_idx": "C", "symptoms": ["A 24-day-old African American boy born at full term via an uncomplicated spontaneous vaginal delivery presented with a 7-day history of annular plaques on the upper forehead and scalp (Figure)", "The plaques emerged on day 17 of life and did not change in appearance or increase in number", "He had normal growth and development and no illnesses or fevers prior to presentation", "He had no contact with sick persons or with household members with similar skin lesions, and there was no history of recent sun exposure", "There was no family history of autoimmune disease, and the mother denied symptoms of dry eyes, dry mouth, cavities, or photosensitivity", "A physical examination revealed a well-appearing afebrile male neonate with 5 annular plaques located on the frontal and posterior scalp", "The thin red border had fine scaling and several pinpoint pustules", "In addition, there were faintly pink patches with scaling on the medial eyebrows and mesolabial folds", "There was no cervical or postauricular lymphadenopathy"], "s1": [0, 1, 5, 6, 7], "s2": [2, 3, 4, 8]} {"key": 1161, "questions": "What Is Your Diagnosis?", "options": [{"label": "A", "disease": "Trapped air embolus"}, {"label": "B", "disease": "Free-floating arterial thrombus"}, {"label": "C", "disease": "Ruptured arterial plaque"}, {"label": "D", "disease": "Radiographic artifact"}], "answer_idx": "B", "symptoms": ["A woman in her early 60s with a history of hypertension and hyperlipidemia was transferred to our hospital after multiple transient ischemic attacks and left internal carotid artery (ICA) stenosis during the past several months", "Her most recent presentation was 1 week prior for a minor stroke with amaurosis fugax, slurred speech, right-sided weakness, and facial droop lasting longer than 24 hours", "Magnetic resonance imaging of the brain demonstrated subacute infarctions involving the left frontal and parietal hemispheres", "She was a current smoker and was taking aspirin and statin medication at the time of presentation", "Workup at another facility prior to transfer included computed tomographic angiography of the neck, which showed chronic occlusion of the right ICA and 95% stenosis of the left ICA", "On examination, the patient was afebrile with a heart rate of 69 beats/min and blood pressure of 134/69 mm Hg", "Pertinent findings included no carotid bruit, a regular cardiac rhythm, and equally palpable upper extremity pulses bilaterally", "The patient was neurologically intact, save for 4/5 motor strength in her right upper extremity", "Computed tomographic angiography of the head and neck was performed (Figure 1)", "Sagittal view demonstrates the left internal carotid stenosis (A indicates anterior", "P, posterior) (A) and the distal extent of the findings (arrowhead) (B)"], "s1": [0, 1, 2, 3, 4, 7], "s2": [5, 6, 8, 9, 10]} {"key": 1162, "questions": "What Is the Diagnosis?", "options": [{"label": "A", "disease": "Janeway lesions"}, {"label": "B", "disease": "Idiopathic thrombocytopenic purpura"}, {"label": "C", "disease": "Dermatitis herpetiformis"}, {"label": "D", "disease": "Petechiae due to pinprick testing"}], "answer_idx": "C", "symptoms": ["A man in his 50s with type 1 diabetes mellitus presented with a 3-month eruption of small reddish brown spots affecting most of his fingers", "The lesions were persistent, asymptomatic, and without aggravating or alleviating factors", "The patient used standard finger sticks to test his blood glucose levels daily", "However, he had pricked his fingers for decades without incident", "A review of symptoms was positive only for intermittent bloating and diarrhea", "Physical examination revealed multiple petechiae in a linear arrangement extending from the volar and lateral aspects of the fingers to the finger pads (Figure, A)", "Results from initial laboratory tests, including a complete blood cell count, comprehensive metabolic panel, antinuclear antibody, erythrocyte sedimentation rate, rheumatoid factor, and cardiolipin antibody, were unremarkable", "Results from a urinalysis and transthoracic echocardiogram were also unremarkable", "Punch biopsies for histopathologic examination (Figure, B) and direct immunofluorescence (Figure, C) were performed", "A, Petechial eruption on the fingers", "B, Histopathologic image showing subtle collections of neutrophils within the dermal papillae (hematoxylin-eosin, original magnification ×200)", "C, Direct Immunofluorescence revealing granular IgA deposition within the dermal papillae and at the dermoepidermal junction (original magnification ×400)"], "s1": [0, 1, 2, 3, 4, 5, 9], "s2": [6, 7, 8, 10, 11]} {"key": 1163, "questions": "What Is the Diagnosis?", "options": [{"label": "A", "disease": "Lichen planus of the nails"}, {"label": "B", "disease": "Cutaneous sarcoidosis"}, {"label": "C", "disease": "Amyloidosis"}, {"label": "D", "disease": "Alopecia areata"}], "answer_idx": "C", "symptoms": ["A man in his 60s presented with a 1-year history of dystrophic nails, doughy palms, and redundant skin on each side of the gluteal cleft", "He also reported easy bruising with minor trauma", "Physical examination showed trachyonychia (rough accentuated linear ridges) on 8 fingernails (Figure, A), thin and brittle toenails, diffuse nonscarring alopecia, and multiple purpuric patches on the forehead, scalp, chest, and arms", "He had substantial laxity of the skin on finger pads (Figure, A), and both sides of the gluteal cleft revealed soft, nontender, rugated, skin-colored linear kissing plaques with no vesicles, bullae, ulcers, or erosions present", "A, Nail and cutaneous findings including trachyonychia of the fingernails and cutaneous laxity of the fingerpads", "B, Histopathologic specimen of the nail bed showing orange, amorphous and extracellular material deposited in the subungual dermis of the nail bed (Congo red special stain, original magnification ×10)", "C, Same specimen under polarized light microscopy revealed intense apple-green birefringence of the amorphous material in the subungual dermis (lower half of the photomicrograph) (Congo red special stain, polarized light microscopy, original magnification ×10)"], "s1": [0, 1, 2, 3, 4], "s2": [5, 6]} {"key": 1164, "questions": "What Is the Diagnosis?", "options": [{"label": "A", "disease": "Staphylococcus and Pseudomonas"}, {"label": "B", "disease": "Candida and Aspergillus"}, {"label": "C", "disease": "Fusarium and Mucormycosis"}, {"label": "D", "disease": "Cryptococcus and Penicillium"}], "answer_idx": "C", "symptoms": ["A man in his 20s sustained a 92% total body surface area (TBSA), full-thickness burn after being doused in gasoline and set on fire", "On arrival to the emergency department, the patient was intubated and resuscitated", "He underwent emergent chest, bilateral upper and lower extremity escharotomies, and a decompressive laparotomy for abdominal compartment syndrome", "Thirteen days after admission, he required a left upper extremity amputation at the level of the proximal humerus owing to development of nonviable muscle tissue that demonstrated black plaques and white nodules (Figure, A and B)", "Once the patient was hemodynamically stable he was taken to the operating room and underwent 4 operations for debridement and grafting, with the initial operation taking place 10 days after admission", "His hospital course was complicated by recurrent polymicrobial aspiration pneumonia, caused by methicillin-sensitive Staphylococcus aureus, Pseudomonas aeruginosa, and Enterococcus", "All were successfully treated with antibiotics", "A, Newly excised burn wound with stitches at the amputation site and a large white nodule", "B, Close-up view of a burn wound newly excised to the subcutaneous tissue with black, plaque-shaped growth", "C and D, Histopathologic images, original magnification ×50. C, Hematoxylin-eosin", "D, Gomori methenamine silver stain"], "s1": [0, 1, 2, 3, 4, 5, 6], "s2": [7, 8, 9, 10]} {"key": 1165, "questions": "What Is Your Diagnosis?", "options": [{"label": "A", "disease": "Disseminated gonococcal infection"}, {"label": "B", "disease": "Streptobacillus moniliformis infection"}, {"label": "C", "disease": "Rocky Mountain spotted fever"}, {"label": "D", "disease": "Disseminated Staphylococcus aureus"}], "answer_idx": "B", "symptoms": ["A 16-year-old girl presented with fever, rash, and diffuse pain for 5 days", "She reported a pruritic, painful rash on her hands and feet and multiple painful joints", "Animal exposures included 3 new puppies and rodent infestation in the home and neighborhood cats", "She did not report recent travel or arthropod bites", "On examination, her right elbow and left knee were swollen and warm", "She reported pain with passive movement of the right shoulder and left wrist", "Petechial and pustular lesions were present on both hands and palms (Figure, A)", "Her feet, including the soles, had many petechial lesions and a few purpuric lesions (Figure, B)", "A, Numerous scattered erythematous papules and petechiae with a few pustules on the palm and fingers of the left hand", "B, Many petechiae on the dorsum of the feet", "Results of laboratory testing were significant for a white blood cell count of 5050/μL (to convert to ×109/L, multiply by 0.001) and a platelet count of 55×103 (to convert to ×109/L, multiply by 1)", "Results of urinalysis and comprehensive metabolic panel were unremarkable"], "s1": [0, 1, 4, 5, 6, 7], "s2": [2, 3, 8, 9, 10, 11]} {"key": 1166, "questions": "What Is Your Diagnosis?", "options": [{"label": "A", "disease": "Gastrointestinal tuberculosis"}, {"label": "B", "disease": "Gastrointestinal stromal tumor with central necrosis"}, {"label": "C", "disease": "Gastrocolic fistula"}, {"label": "D", "disease": "Gastric leiomyosarcoma"}], "answer_idx": "C", "symptoms": ["A 54-year-old woman presented with a 6-month history of watery diarrhea and severe halitosis", "She also lost more than 30 kg of weight during this period", "Physical examination revealed acute cachexia with conjunctival pallor and an immobile nontender mass in the left upper abdomen", "Her laboratory test results revealed a microcytic anemia and hypoalbuminemia", "Abdominal computed tomography was performed (Figure 1)", "The patient was scheduled for exploratory laparotomy and resection", "A large central necrotic mass is shown in this computed tomography image of the abdomen"], "s1": [0, 1, 3], "s2": [2, 4, 5, 6]} {"key": 1167, "questions": "What Is the Diagnosis?", "options": [{"label": "A", "disease": "Intestinal duplication"}, {"label": "B", "disease": "Meckel diverticulum"}, {"label": "C", "disease": "Gastrointestinal stromal tumor"}, {"label": "D", "disease": "Small-bowel diverticular disease"}], "answer_idx": "B", "symptoms": ["A 28-year-old white man presented to the gastrointestinal (GI) medicine clinic for persistent nausea and vomiting", "He had a long-standing history of gastroesophageal reflux disease that was only partially controlled with proton pump inhibitor therapy", "The symptoms were exacerbated by cessation of proton pump inhibitor medication, and he reported an associated 9-kg weight loss", "Results of laboratory testing were significant for iron deficiency anemia (hematocrit, 31% [to convert to proportion of 1.0, multiply by 0.01]) and hemeoccult-positive stool", "Upper and lower endoscopy, computed tomography enterography, PillCam capsule endoscopy, and a Meckel scan failed to identify a source of GI bleeding", "The patient received a retrograde double-balloon enteroscopy that showed an outpouching in the distal ileum (Figure, A) with an accompanying ulceration of the adjacent mucosa (Figure, B)", "Exploratory laparotomy with small-bowel resection was performed, and the gross specimen is shown in Figure, C", "A, Outpouching of the distal ileum", "B, Ulceration adjacent to the outpouching of the distal ileum", "C, Resected distal ileum revealing an ulceration of the mucosa", "Arrowhead indicates site of ulceration"], "s1": [0, 1, 2, 3, 4], "s2": [5, 6, 7, 8, 9, 10]} {"key": 1168, "questions": "What Is Your Diagnosis?", "options": [{"label": "A", "disease": "Abscess of the lesser sac (omental bursa)"}, {"label": "B", "disease": "Perforated ulcer of the posterior stomach wall"}, {"label": "C", "disease": "Lesser sac (omental bursa) internal hernia through the epiploic foramen"}, {"label": "D", "disease": "Large diverticle of the transverse colon"}], "answer_idx": "C", "symptoms": ["A woman in her 50s presented to our emergency department with a 3-hour history of sudden epigastric and right upper quadrant pain", "She did not report any previous abdominal surgery", "The colicky pain did not migrate during that period and occurred in paroxysms at 9- to 10-minute intervals", "The patient was afebrile and had no nausea or vomiting", "Physical examination revealed mild tachycardia (96/min) and moderate abdominal distension without signs of peritonism", "the bowel sounds were hyperactive", "Careful examination ruled out incarcerated hernias in the groin, femoral triangle, and obturator foramen", "Blood analysis results showed mild leukocytosis (total white blood cell count, 10.5 × 103/μL [to convert to ×109 per liter, multiply by 0.001]) without elevation of the C-reactive protein level (<3 mg/L [to convert to nanomoles per liter, multiply by 9.524])", "Serum electrolyte levels were within the normal range", "Computed tomography of the abdomen was conducted (Figure)", "Computed tomographic images of the abdomen", "A, Axial imaging", "B, Coronal imaging", "Arrowheads indicate the border of the epiploic foramen", "asterisk, the air-filled cecum in the lesser sac", "Lesser sac (omental bursa) internal hernia through the epiploic foramen"], "s1": [0, 1, 2, 3, 4, 5, 6, 7, 8], "s2": [9, 10, 11, 12, 13, 14, 15]} {"key": 1169, "questions": "What is the Diagnosis?", "options": [{"label": "A", "disease": "Amyloidosis"}, {"label": "B", "disease": "Systemic sclerosis"}, {"label": "C", "disease": "Hypoplasminogenemia"}, {"label": "D", "disease": "Lipoid proteinosis"}], "answer_idx": "C", "symptoms": ["A white woman in her 50s presented for evaluation of recurrent gingival ulcers of 8 years’ duration", "She reported that at the time of initial onset, multiple painful ulcers had appeared on the anterior maxillary gingiva", "She stated that the lesions had resolved spontaneously without treatment but recurred 8 years later as swelling and ulceration of the maxillary and mandibular gingiva", "She denied involvement of cutaneous surfaces and reported a history of ligneous conjunctivitis since childhood (Figure, A)", "Her medical history was significant for hypothyroidism, seizures, and functional heart murmur", "Medications included levothyroxine sodium, mometasone furoate, levocetirizine, and benzonatate", "Review of systems was significant for easy bruising and prolonged bleeding", "Intraoral examination revealed generalized mild edema with rolled margins of the maxillary and mandibular gingiva accompanied by nodular ulceration of the left maxillary gingiva (Figure, B)", "Biopsy specimens were obtained from the gingival ulcer (Figure, C and D) and buccal mucosa for routine histological and direct immunofluorescence analysis, respectively", "Also obtained were a complete blood cell count with differential and comprehensive metabolic panel", "no abnormal findings were identified", "A, Pseudomembranous masses affecting conjunctival mucosa of the left eye consistent with ligneous conjunctivitis", "B, Edema with rolled margins and nodular ulceration of the left maxillary gingiva", "C and D, Punch biopsy specimen from the maxillary gingiva showing aggregates of relatively amorphous, eosinophilic material beneath the epithelium and around blood vessels (C, hematoxylin-eosin, original magnification ×100", "D, hematoxylin-eosin, original magnification ×200)"], "s1": [0, 1, 2, 3, 7, 8, 11, 12, 13, 14], "s2": [4, 5, 6, 9, 10]} {"key": 1170, "questions": "What Is the Diagnosis?", "options": [{"label": "A", "disease": "Epidermolysis bullosa, Dowling-Meara type"}, {"label": "B", "disease": "Neonatal herpesvirus infection"}, {"label": "C", "disease": "Neonatal pemphigus vulgaris"}, {"label": "D", "disease": "Widespread bullous impetigo"}], "answer_idx": "C", "symptoms": ["A newborn male infant was transferred to the neonatal intensive care unit for widespread vesicles and erosions evident at birth, following a spontaneous vaginal delivery complicated by fetal meconium exposure", "The mother, initially unavailable for examination because she remained at the facility where she had given birth, reported via telephone that she was healthy except for a urinary tract infection for which she had taken a 7-day course of nitrofurantoin monohydrate approximately 3 to 4 weeks before delivery", "She denied a history of herpesvirus infection, blistering disorder, or autoimmune disease", "On physical examination, the neonate exhibited extensive erosions rimmed by purulent vesicles involving the groin, neck, scalp, fingers, and left side of the upper chest (Figure, A and B)", "Mucous membranes were spared", "Biopsy specimens were obtained, with results as shown in the Figure, C and D", "A, Erosions rimmed with flaccid turbid vesicles involving the groin, thighs, and abdomen", "B, Large erosion with a rim of flaccid purulent vesicles and crusting on the left side of the chest", "C, Histopathologic analysis of truncal skin sample demonstrating suprabasilar acantholysis with an eosinophil-rich infiltrate (hematoxylin-eosin, original magnification ×40)", "D, Direct immunofluorescence of perilesional skin from the right side of the chest demonstrates IgG in an intracellular “chicken wire” pattern involving the lower spinous layer (original magnification ×40)"], "s1": [1, 2, 4, 9], "s2": [0, 3, 5, 6, 7, 8]} {"key": 1171, "questions": "What is the Diagnosis?", "options": [{"label": "A", "disease": "Granuloma annulare"}, {"label": "B", "disease": "Cutaneous lupus"}, {"label": "C", "disease": "Neutrophilic sebaceous adenitis"}, {"label": "D", "disease": "Erythema annulare centrifugum, deep variant"}], "answer_idx": "C", "symptoms": ["A young man in his 20s with no significant medical history presented with an annular eruption on the face and upper torso of 2 weeks’ duration", "The eruption began on his upper back, at which time he felt well", "He then received significant sun exposure while on a 7-day trip to Miami, Florida", "After returning from his trip, he noted that the eruption had become more prominent on his back and had spread to his face", "In addition, he began to have a febrile illness with joint pain and swollen nodes, which subsequently resolved without treatment", "The lesions continued to rapidly enlarge without scaling, and his plaques were not associated with itching or pain", "The patient was not taking any medications at the time and denied any recent international travel", "Physical examination revealed multiple annular plaques with raised erythematous-violaceous borders on the face, distributed mainly along the forehead and temporal region (Figure, A)", "There were similar coalescing lesions found on back (Figure, B)", "No other mucocutaneous findings were noted, and the rest of examination was unremarkable", "A punch biopsy specimen was obtained from a plaque on the right upper back and submitted for pathologic review (Figure, C and D)", "A, Multiple annular plaques with raised erythematous-violaceous borders are distributed mainly along the frontal and temporal region of the head", "B, Similar findings are seen on the patient’s face with more central clearing and attenuated coloring", "C, This pathologic specimen from a lesion on the upper back depicts distended sebaceous lobules with associated follicular dilatation and a mild chronic perivascular and perifollicular lymphomononuclear infiltrate", "D, This pathologic specimen from the upper back exhibits sebaceous lobules with sebocyte necrosis and an associated neutrophilic infiltrate that also includes scattered lymphomononuclear cells"], "s1": [0, 1, 2, 3, 4, 5, 6, 7, 8, 9], "s2": [10, 11, 12, 13, 14]} {"key": 1172, "questions": "What Is the Diagnosis?", "options": [{"label": "A", "disease": "Psoriasis vulgaris"}, {"label": "B", "disease": "Erythema ab igne with superimposed chemotherapy effect"}, {"label": "C", "disease": "Pityriasis rubra pilaris"}, {"label": "D", "disease": "Cutaneous lupus erythematosus"}], "answer_idx": "B", "symptoms": ["A man in his 60s presented in consultation from his oncologist for a 3-week history of a pruritic eruption on his back in the setting of stage IV, progressive, neuroendocrine pancreatic cancer", "He had undergone numerous chemotherapeutic regimens over the past 3 years, all of which had failed", "For the past 4 months he had been receiving oral etoposide (daily for the first 3 weeks of each 4-week cycle), daily pancrelipase, and monthly octreotide acetate intramuscular injections", "He denied the use of heating pads but admitted to sitting on the hearth, directly in front of his fireplace, for long periods because of the cold weather and chills presumably from his underlying malignant neoplasm", "He acknowledged a mild, untreated seasonal rhinitis but otherwise had enjoyed good health prior to his cancer diagnosis", "Physical examination revealed a large, reticulated, erythematous plaque with micaceous and silvery scale localized to the mid-back (Figure, A)", "A punch biopsy was performed (Figure, B-D)", "A, Large, reticulated, erythematous plaque with silvery scale localized to the mid-back", "B-D, Punch biopsy specimens", "B, Mild hyperkeratosis overlying an epidermis with dysmaturation and apoptotic cells, increased small blood vessels, pigment incontinence, and perivascular inflammation with lymphocytes and eosinophils (hematoxylin-eosin, original magnification ×20)", "C, Keratinocyte dysmaturation demonstrated by large keratinocytes with hyperchromatic nuclei and prominent nucleoli, and increased mitoses with starburst cells (arrowheads) (hematoxylin-eosin, original magnification ×60)", "D, Squamous metaplasia and apoptosis of the eccrine ducts (hematoxylin-eosin, original magnification ×60)"], "s1": [0, 1, 2, 3, 4], "s2": [5, 6, 7, 8, 9, 10, 11]} {"key": 1173, "questions": "What Is Your Diagnosis?", "options": [{"label": "A", "disease": "Infantile hemangioma"}, {"label": "B", "disease": "Neuroblastoma"}, {"label": "C", "disease": "Dermoid cyst"}, {"label": "D", "disease": "Infantile myofibroma"}], "answer_idx": "D", "symptoms": ["A newborn presented on the first day of life with a chest mass found on initial examination in the nursery (Figure 1)", "She was born at 38.3 weeks via spontaneous vaginal delivery after an uncomplicated pregnancy to a 35-year-old, G3, now P2 mother with no significant medical history", "Results of maternal prenatal laboratory tests were all unremarkable", "She received regular prenatal care, and routine ultrasonography revealed no anomalies", "The Apgar scores were 9 at 1 minute and 9 at 5 minutes", "No resuscitation was required in the delivery room", "The newborn was already breastfeeding well, voiding, and stooling by the time of the initial examination", "Physical examination revealed a 3.5 × 4-cm mobile mass on the right lateral trunk at the midaxillary line at levels approximately T4 to T6. The mass was firm and round, although when palpated it was not homogeneous or smooth", "There was no discoloration of the overlying skin", "Results of the remainder of the physical examination were normal"], "s1": [0, 7, 8], "s2": [1, 2, 3, 4, 5, 6, 9]} {"key": 1174, "questions": "What Is Your Diagnosis?", "options": [{"label": "A", "disease": "Peritoneal mesothelioma"}, {"label": "B", "disease": "Sclerosing encapsulated peritonitis"}, {"label": "C", "disease": "Malrotation with Ladd bands"}, {"label": "D", "disease": "Peritoneal encapsulation"}], "answer_idx": "B", "symptoms": ["An 85-year-old man with a history of occupational exposure to asbestos presented with 1 week of nausea and vomiting without any bowel function", "He reported only mild abdominal pain", "The patient denied any history of abdominal surgery", "On examination, he was mildly distended and tender focally on the right side", "In the emergency department, a nasogastric tube was placed, yielding a moderate amount of dark, foul-smelling output", "Laboratory results were significant for a white blood cell count of 13 300/μL (to convert to ×109 per liter, multiply by 0.001) (bands 20%)", "A contrast-enhanced computed tomographic scan of the abdomen and pelvis demonstrated dilated small intestine (measuring up to 4.7 cm), with air-fluid levels extending to the terminal ileum (Figure 1A and B)", "The entire colon was collapsed", "There were several loops of normal-caliber bowel proximal to the area of concern", "The decision was made to take him to the operating room for exploration given the lack of any previous abdominal surgical procedures and no other explanation for his symptoms", "Intraoperatively, a large portion of the small intestine was involved in dense adhesions", "After performing careful lysis on the distal involved bowel, a tightly coiled and twisted segment was encountered (Figure 1C)", "At this point, further attempts to separate loops of small intestine would have been difficult, and the decision was made to resect this segment (approximately 110 cm) and perform a primary anastomosis", "Cross-sectional computed tomographic scan of the abdomen", "Dilated small intestine with air-fluid levels extending to the terminal ileum (A and B)", "C, Intraoperative image of involved small intestine", "Tightly coiled and twisted small bowel with adhesive bands"], "s1": [0, 1, 2, 3, 4, 5], "s2": [6, 7, 8, 9, 10, 11, 12, 13, 14, 15, 16]} {"key": 1175, "questions": "What Is The Diagnosis?", "options": [{"label": "A", "disease": "Carcinoid tumor"}, {"label": "B", "disease": "Gastrointestinal stromal tumor"}, {"label": "C", "disease": "Neurofibroma"}, {"label": "D", "disease": "Intra-abdominal abscess"}], "answer_idx": "B", "symptoms": ["A 72-year-old man presented to the hospital after sustaining a fall, with prior generalized weakness and fatigue", "He had been having recurrent gastrointestinal bleeding and melena during the past year", "He reported a 6.75-kg weight loss during the past few months but did not have abdominal pain, nausea, vomiting, fevers, or chills", "His surgical history was notable for a coronary artery bypass graft 9 months earlier and γ-knife resection of an acoustic neuroma 10 years ago", "Physical examination revealed a soft, nontender abdomen without obvious palpable masses", "Inspection of the patient’s skin demonstrated multiple fleshy, soft cutaneous nodules (Figure 1A)", "The patient’s laboratory testing was only notable for anemia with a hemoglobin level of 10.4 g/dL (reference range, 12-16 g/dL)", "A prior extensive examination, including upper and lower endoscopy, upper gastrointestinal series with small-bowel follow-through, and capsule endoscopy did not reveal a diagnosis", "A computed tomography image of the abdomen and pelvis is shown in Figure 1B", "A, Multiple cutaneous nodules and pigmented spots are evident on physical examination", "B, Cross-sectional imaging reveals a large, complex, septated, right-upper-quadrant abdominal mass"], "s1": [0, 1, 2, 6, 7, 10], "s2": [3, 4, 5, 8, 9]} {"key": 1176, "questions": "What Is the Diagnosis?", "options": [{"label": "A", "disease": "Cutaneous anaplastic large cell lymphoma"}, {"label": "B", "disease": "Extranodal NK/T-cell lymphoma, nasal type"}, {"label": "C", "disease": "Blastomycosis"}, {"label": "D", "disease": "Subcutaneous panniculitis-like T-cell lymphoma"}], "answer_idx": "B", "symptoms": ["A white man in his 60s was referred to our dermatology clinic for rapidly enlarging lesions on his calf, thigh, and cheek", "Three months prior, he had noticed a small nodule on his calf, with no overlying skin changes or tenderness", "This progressed to a tender, nodular plaque while other lesions appeared", "The patient denied other signs or symptoms", "His medical history included Parkinson disease, for which he took no medication", "Laboratory findings showed an elevated lactate dehydrogenase (LDH) level at 439 U/L (reference range, 118-242 U/L) (to convert LDH to microkatals per liter, multiply by 0.0167)", "Results from serum chemical analyses, complete blood cell count, and tests of liver functions were normal", "Physical examination revealed a healthy-appearing man with a 4.0 × 4.0-cm pink, firm, tender nodular plaque on his left calf (Figure, A), a 5.0-cm firm, subcutaneous mass on his right thigh, and a 0.5 × 0.5-cm subcutaneous nodule on his left cheek", "There was no cervical, axillary, or inguinal lymphadenopathy", "A punch biopsy specimen was obtained from each lesion (Figure, B and C)", "A, Clinical photograph of the nodular plaque on the left lateral calf", "B and C, Histopathologic images from the punch biopsy specimen demonstrating angiocentrism", "B, The large atypical lymphoid cells can be seen infiltrating a vessel (outlined by arrowheads) (hematoxylin-eosin, original magnification ×400)", "C, Histopathologic image from the punch biopsy specimen showing positive EBER in situ hybridization", "The atypical lymphocytic infiltrate is strongly positive for EBV, as shown by EBV-encoded RNA in situ hybridization (original magnification ×600)"], "s1": [0, 1, 2, 3, 7, 8], "s2": [4, 5, 6, 9, 10, 11, 12, 13, 14]} {"key": 1177, "questions": "What Is Your Diagnosis?", "options": [{"label": "A", "disease": "Hereditary hemorrhagic telangiectasia"}, {"label": "B", "disease": "Cutaneous collagenous vasculopathy"}, {"label": "C", "disease": "Generalized essential telangiectasia"}, {"label": "D", "disease": "Systemic lupus erythematosus"}], "answer_idx": "C", "symptoms": ["A woman in her 50s presented with “speckled” red lesions on her bilateral arms, lower legs, and feet", "They had first appeared on her lower legs 15 years ago and were slowly increasing in number and spreading to involve her feet and bilateral upper extremities", "The patient denied tenderness, pruritus, and edema", "She reported no family history of this problem and denied any personal history of anemia, epistaxis, hematemesis, melena, hematochezia, or hemoptysis", "She had a medical history of hypertension, hyperlipidemia, and tobacco abuse, requiring treatment with losartan and atorvastatin", "Physical examination revealed punctate, blanchable telangiectasias coalescing to form large, retiform patches on her bilateral lower extremities, feet, and upper extremities (Figure, A)", "There was no conjunctival or mucosal involvement", "The patient reported no dermatographism or purpura", "Histologic findings from a biopsy specimen are shown in the Figure, B", "A, Multiple telangiectasias coalescing to form large, retiform patches on the bilateral lower extremities", "B, Dilated, ectatic vessels in the superficial dermis with no associated inflammation or fibrinoid necrosis (hematoxylin-eosin, original magnification ×200)"], "s1": [0, 1, 2, 3, 4, 6, 7], "s2": [5, 8, 9, 10]} {"key": 1178, "questions": "What Is the Diagnosis?", "options": [{"label": "A", "disease": "Intertrigo"}, {"label": "B", "disease": "Hailey-Hailey disease"}, {"label": "C", "disease": "Langerhans cell histiocytosis"}, {"label": "D", "disease": "Inverse psoriasis"}], "answer_idx": "C", "symptoms": ["A woman in her late 30s presented for evaluation and management of tender, reddish brown, slightly pruritic papules and plaques on the scalp, in the inframammary region, and in the perineum that had been present for approximately 1 year", "Previous treatment with multiple courses of topical and systemic antifungals, topical and systemic corticosteroids, and oral antibiotics had failed", "She reported pain, pruritus, and serous discharge associated with the plaques but denied any systemic symptoms", "Her medical history was unremarkable, and she was not taking any medications at the time of presentation", "Results of a physical examination revealed well-demarcated, moist, erythematous plaques and erosions in the bilateral inframammary regions (Figure 1A) and the intertriginous regions of the groin and scattered excoriated papules on the scalp", "A punch biopsy specimen of the plaque along the left inframammary region was sent for pathologic evaluation (Figure 1B-D)", "A, Well-demarcated erosive plaques on the bilateral inframammary region", "B, Abundant atypical cells within the epidermis and papillary dermis (hematoxylin-eosin, original magnification ×10)", "C, Abundant atypical cells with reniform nuclei within the epidermis and papillary dermis (hematoxylin-eosin, original magnification ×40)", "D, Immunohistochemical analysis (original magnification ×10)"], "s1": [0, 2, 3, 4, 6], "s2": [1, 5, 7, 8, 9]} {"key": 1179, "questions": "What Is Your Diagnosis?", "options": [{"label": "A", "disease": "Microcystic lymphatic malformation"}, {"label": "B", "disease": "Congenital melanocytic nevus"}, {"label": "C", "disease": "Venous malformation"}, {"label": "D", "disease": "Nevus sebaceous"}], "answer_idx": "A", "symptoms": ["An 8-year-old girl presented to the dermatology clinic for further evaluation of a lesion over the right posterior thigh (Figure, A)", "Her mother reported the lesion appeared at age 3 months, was previously diagnosed as a hemangioma, and was treated with observation", "She also reported the lesion originated as a “cystlike papule” that had continuously expanded", "In addition to causing cosmetic concerns, the lesion seemed to be irritated by clothing and would occasionally bleed and swell", "The patient denied pain or itching associated with the lesion", "Since its appearance at 3 months of age, several abscesses formed underneath the region of concern requiring incision and drainage", "Medical history revealed no other medical problems", "On physical examination, at the proximal posterior thigh, there was an 11 × 10-cm cobblestoned, gray plaque composed of numerous purplish, vesicular papules with surrounding satellite papules", "No swelling was present", "A 4-mm punch biopsy was performed (Figure, B and C)", "Translucent papules and histologic examination in a child", "A, Clinical examination shows numerous grouped translucent papules", "B and C, Dilated thin-walled vascular spaces containing valvelike papillary projections in the superficial dermis", "The overlying epidermis shows acanthosis and hyperkeratosis (hematoxylin-eosin, original magnification ×20 [B] and ×400 [C])"], "s1": [0, 1, 2, 3, 4, 5, 6, 7, 8], "s2": [9, 10, 11, 12, 13]} {"key": 1180, "questions": "What Is Your Diagnosis?", "options": [{"label": "A", "disease": "Dropped gallstones"}, {"label": "B", "disease": "Lymphoma"}, {"label": "C", "disease": "Peritoneal carcinomatosis"}, {"label": "D", "disease": "Peritoneal mesothelioma"}], "answer_idx": "A", "symptoms": ["A previously healthy 75-year-old woman was referred to our tertiary care center when a workup for abdominal pain and jaundice revealed a duodenal mass", "She underwent a metastatic workup", "High-resolution computed tomography revealed a 3 × 4-cm circumferential lesion in the second and third portions of the duodenum involving the pancreatic head (Figure 1)", "Multiple nodules were noted along the peritoneal reflection of the right paracolic gutter", "The results of a biochemical analysis were remarkable with a total bilirubin level of 6.8 mg/dL (to convert to micromoles per liter, multiply by 17.104), an aspartate aminotransferase level of 394 U/L, an alanine transaminase level of 746 U/L, and an alkaline phosphatase level of 1146 U/L (to convert the last 3 analytes to microkatals per liter, multiply by 0.0167)", "Her medical history was significant for hypertension and a previous laparoscopic cholecystectomy approximately 6 years ago", "Coronal computed tomographic (CT) image of abdominal mass (arrowhead) (A) and axial CT image of peritoneal implant (arrowhead) (B)"], "s1": [0, 1, 2, 3, 6], "s2": [4, 5]} {"key": 1181, "questions": "What Is Your Diagnosis?", "options": [{"label": "A", "disease": "Desmoid fibromatosis"}, {"label": "B", "disease": "Inflammatory myofibroblastic tumor"}, {"label": "C", "disease": "Nodular fasciitis"}, {"label": "D", "disease": "Leiomyosarcoma"}], "answer_idx": "C", "symptoms": ["A woman in her mid-30s pregnant with twins at 13 weeks’ gestation presented with a tender right-flank mass, which she noticed after falling and sustaining trauma to the area 1 week previously", "Medical history was significant for a total thyroidectomy for papillary thyroid cancer 7 years before as well as a strong family history of malignant breast cancer on the maternal side and colon and liver cancer on the paternal side", "Physical examination revealed an 8 × 8-cm, tender, firm posterior right-flank mass that was mobile but fixed to the abdominal wall musculature", "Laboratory examination revealed no abnormalities", "Ultrasonography demonstrated a hypoechoic lesion measuring 3.3 × 3.6 cm with internal echoes and no significant flow (Figure 1A)", "A noncontrast magnetic resonance image (MRI) revealed this to be a well-circumscribed T2 hyperintense lesion with diffuse surrounding soft tissue edema within the adjacent muscular and subcutaneous tissues abutting the posterior margin of the right chest wall and external oblique muscle at the level of the T11 to T12 intercostal space (Figure 1B and C)", "A, Ultrasonography demonstrating a hypoechoic mass in the musculature of the right flank", "B and C, T1- and T2-weighted images demonstrating an enhancing right-flank mass within the right-flank musculature", "Because of the concern of malignancy, an ultrasonography-guided core biopsy was performed and demonstrated a myofibroblastic proliferation with immunostains positive for actin and negative for S100 and desmin", "Her case was presented at the soft tissue tumor board, and given that the patient was asymptomatic and pregnant, the decision was made to closely watch the progression of this lesion with serial imaging with a planned resection after delivery", "Unfortunately, she developed premature rupture of membranes and lost the pregnancy", "Follow-up computed tomography was not able to identify the lesion but MRI showed that it was still present but decreased in size (2.4 × 1 cm) with interval resolution of surrounding inflammatory changes", "She subsequently underwent wide local resection of the lesion"], "s1": [0, 1, 10], "s2": [2, 3, 4, 5, 6, 7, 8, 9, 11, 12]} {"key": 1182, "questions": "What Is the Diagnosis?", "options": [{"label": "A", "disease": "Hereditary papulotranslucent acrokeratoderma"}, {"label": "B", "disease": "Dyshidrotic pemphigoid"}, {"label": "C", "disease": "Epidermolysis bullosa simplex"}, {"label": "D", "disease": "Dyshidrotic eczema"}], "answer_idx": "A", "symptoms": ["A woman in her 50s with a medical history of asthma and seasonal allergies presented with numerous asymptomatic flesh-colored to translucent papules on her palms and palmar and lateral aspects of her fingers (Figure, A and B)", "Many of the papules were depressed centrally", "There was no extension onto the dorsal aspect of the hands", "There were similar translucent papules over the margins of her feet, particularly her great toes, and hyperkeratotic yellow plaques with scattered pits over her bilateral soles, concentrated in areas of pressure", "The lesions on her hands first developed in her 40s and were exacerbated by alcohol-based hand sanitizer at work", "They became more prominent with exposure to water but remained asymptomatic", "Occasional application of urea cream to her feet did not lead to significant improvement", "She also had fine-textured hair, present since childhood with thinning noted in the past 7 years", "She had no history of eczema", "At the time, she denied a family history of similar skin findings but recently reported similar hand and foot findings in her mother", "Two punch biopsies were performed from the left thenar eminence (Figure, C)", "A, Clinical photograph of the palm showing dozens of flesh-colored to translucent palmar papules", "B, Closer clinical view of the palmar papules over the thenar eminence, some with central depression", "C, Histologic image from a skin biopsy specimen from the thenar eminence (hematoxylin-eosin, original magnification ×40)"], "s1": [0, 1, 2, 3, 4, 5, 6, 8, 9], "s2": [7, 10, 11, 12, 13]} {"key": 1183, "questions": "What Is the Diagnosis?", "options": [{"label": "A", "disease": "Sweet syndrome"}, {"label": "B", "disease": "Herpes genitalis"}, {"label": "C", "disease": "Deep fungal or atypical mycobacterial infection"}, {"label": "D", "disease": "Perivulvar neutrophilic eccrine hidradenitis"}], "answer_idx": "D", "symptoms": ["A woman in her 60s developed fever, rigors, periorbital swelling, and a painful perivulvar eruption, 7 days after her first cycle of cytarabine and daunorubicin hydrochloride for newly diagnosed acute myeloid leukemia (AML)", "She had a history of type 2 diabetes mellitus, hypothyroidism, and hypercholesterolemia", "Her regular medications included metformin, insulin, rosuvastatin calcium, levothyroxine sodium, and aspirin", "Examination revealed edematous labia and multiple perivulvar tender nodules on violaceous and indurated plaques sparing the vaginal mucosa and inguinal creases (Figure, A)", "She also had bilateral injected conjunctivae with periorbital swelling", "A punch biopsy of the perivulvar lesions was performed for histopathologic analysis (Figure, B and C) and tissue culture", "A, Perivulvar plaques and nodules that developed after chemotherapy for acute myeloid leukemia", "Edematous labia with perivulvar violaceous plaques and nodules", "B and C, Skin histologic images from perivulvar plaques and nodules", "B, Spongiosis and hyperplasia of the epidermis with neutrophilic infiltrate within the dermis (hematoxylin-eosin, original magnification ×10)", "C, Necrotic collagen and eccrine gland destruction (hematoxylin-eosin, original magnification ×10)"], "s1": [0, 1, 2, 4], "s2": [3, 5, 6, 7, 8, 9, 10]} {"key": 1184, "questions": "What Is the Diagnosis?", "options": [{"label": "A", "disease": "Psoriasis"}, {"label": "B", "disease": "Lichen planus"}, {"label": "C", "disease": "Porokeratosis ptychotropica"}, {"label": "D", "disease": "Cutaneous graft-vs-host disease"}], "answer_idx": "C", "symptoms": ["A man in his 50s was referred for evaluation of pruritic lesions in the perianal area and buttocks of 2 years’ duration", "He reported no history of cutaneous diseases", "He had undergone heterologous bone marrow transplantation for chronic myeloid leukemia 18 years before and had hypertension controlled with lisinopril and hydrochlorothiazide and type 2 diabetes mellitus controlled by diet", "Physical examination revealed multiple hyperkeratotic purplish brown papules and plaques, more confluent in the perianal area, with well-demarcated keratotic prominent margins (Figure, A and B)", "Dermoscopy showed structureless brownish pigmentation and erythema in the center, demarcated by a white-yellow hyperkeratotic scale at the periphery", "Because of suspicion of lichen planus annularis, the patient was treated intermittently with intramuscular triamcinolone acetonide depot and topical treatments such as corticosteroids, antifungal agents, pimecrolimus, and retinoic acid, resulting in subjective pruritus relief", "however, the lesions did not show satisfactory improvement in 2 years’ follow-up", "Therefore, 2 skin biopsies were performed (Figure, C)", "A, Multiple hyperkeratotic purplish brown papules and plaques on both buttocks, more confluent in the perianal area, with well-demarcated slightly elevated keratotic margins", "B, Detail of the clinical lesions", "C, Biopsy specimen (hematoxylin-eosin, original magnification ×400)"], "s1": [0, 1, 2, 5, 6], "s2": [3, 4, 7, 8, 9, 10]} {"key": 1185, "questions": "What is the diagnosis?", "options": [{"label": "A", "disease": "Confluent and reticulated papillomatosis"}, {"label": "B", "disease": "Bullous prurigo pigmentosa"}, {"label": "C", "disease": "Linear IgA bullous dermatosis"}, {"label": "D", "disease": "Lichen planus pigmentosus"}], "answer_idx": "B", "symptoms": ["A previously healthy man in his late teens and of Japanese ancestry presented with a 4-month history of a pruritic eruption that started on his neck, upper back, and buttocks (Figure 1A)", "He felt well otherwise, denying fever, gastrointestinal upset, or any antecedent illness or exposure", "He denied any family history of similar eruptions and was not taking any medications aside from whey protein supplementation", "He was eating a normal diet", "The cutaneous eruption had continued to spread despite topical and oral corticosteroid therapy, and diphenhydramine provided no relief from the pruritus", "A, Initial clinical presentation of bullae on an erythematous base in a reticular pattern on the back", "B, Biopsy specimen from the mid-abdomen showing dense infiltration of the epidermis by numerous eosinophils (hematoxylin-eosin)", "C, Biopsy specimen from the back showing abrupt necrosis of the epidermis and an intraepidermal blister filled with neutrophils and eosinophils", "The blister correlates with the bulla seen clinically", "The upper dermis showed a mixed infiltrate of lymphocytes, eosinophils, and neutrophils (hematoxylin-eosin)", "A complete blood cell count and comprehensive metabolic panel revealed only a minor lymphopenia (lymphocyte percentage, 16.8%", "lymphocyte count, 1000/μL [normal, 1300-3600/μL]), a high monocyte percentage (13.6%", "normal, 4.0%), but a normal absolute monocyte count", "(To convert lymphocyte count to ×109/L, multiply by 0.001.) No evidence of ketosis was noted", "Results from assays for herpes simplex virus IgG, antinuclear antibodies, glucagon, and zinc were all within normal limits", "On clinical examination, vesicles and bullae were found coalescing into large reticulated plaques with background erythema on the upper neck, inframammary areas, groin, mid back, and extending down onto the upper buttocks", "The bullae were eroded in some areas with hemorrhagic crust (Figure 1A)", "Punch biopsy specimens from characteristic lesions were obtained and sent for hematoxylin-eosin (H&E) examination (Figure 1B and C)", "Punch biopsy specimens of lesional and perilesional skin were also sent for direct immunofluorescence (DIF) study"], "s1": [0, 1, 2, 3, 4, 15, 16], "s2": [5, 6, 7, 8, 9, 10, 11, 12, 13, 14, 17, 18]} {"key": 1186, "questions": "What Is Your Diagnosis?", "options": [{"label": "A", "disease": "Varicella-zoster virus"}, {"label": "B", "disease": "Incontinentia pigmenti"}, {"label": "C", "disease": "Herpes simplex virus"}, {"label": "D", "disease": "Langerhans cell histiocytosis"}], "answer_idx": "B", "symptoms": ["A 6-day-old male infant presented to the emergency department with a linear band of skin-colored papules on his left upper extremity", "These papules had been present at birth and were persistent at the time of the patient’s initial hospital discharge", "Because no resolution had been noted over the next few days, the patient was taken to his primary care physician for evaluation and then sent to the emergency department for further workup", "The patient was afebrile", "The patient’s mother noted that both the pregnancy and the delivery were uncomplicated", "She denied ever having a miscarriage and had another healthy male child at home without any similar lesions", "The mother had a history of varicella as a child", "she did not have a history of herpes simplex virus and had no oral or genital lesions at the time of delivery", "On examination, the patient was noted to have 2- to 3-mm vesicles on the left upper extremity in a linear, bandlike distribution (Figure, A), and similar 2- to 3-mm deep-seated vesicles on his left medial thigh in a whorled configuration, approximately 4 × 3 cm in total size (Figure, B)", "In addition, the patient had a 1-mm vesicle on his abdomen", "A, Vesicles (2-3 mm in diameter) on the left forearm are in a linear, bandlike distribution", "B, Deep-seated vesicles on left medial thigh are in a whorled configuration"], "s1": [0, 1, 2, 8, 9, 10, 11], "s2": [3, 4, 5, 6, 7]} {"key": 1187, "questions": "What Is the Diagnosis?", "options": [{"label": "A", "disease": "Intussusception"}, {"label": "B", "disease": "Colon cancer"}, {"label": "C", "disease": "Small-bowel volvulus"}, {"label": "D", "disease": "Mesenteric ischemia"}], "answer_idx": "C", "symptoms": ["A 90-year-old woman presented with vomiting and acute abdominal pain that started 24 hours earlier", "Her medical history included a partial oophorectomy of a right ovarian cyst, hypertension, and vomiting and abdominal pain once a year that started 10 years earlier", "She had always been diagnosed as having adhesive ileus, and her symptoms always improved within a day after hospital admission", "A physical examination at admission revealed that her abdomen was slightly distended and tender to palpation on the right side of the paraumbilical point without muscle guarding", "The hematologic findings revealed slight anemia and amylasemia", "her hemoglobin level was 9.3 g/dL (to convert to grams per liter, multiply by 10.0), and her amylase level was 168 U/L (to convert to microkatals per liter, multiply by 0.0167)", "Abdominal computed tomography (CT) with intravenous contrast agent revealed a mass on the right side of the paraumbilical point (Figure 1)", "Contrast-enhanced computed tomographic scan through the abdomen showing a mass on the right side of the paraumbilical point"], "s1": [0, 1, 2], "s2": [3, 4, 5, 6, 7]} {"key": 1188, "questions": "What Is Your Diagnosis?", "options": [{"label": "A", "disease": "Serous cystadenoma of the pancreas"}, {"label": "B", "disease": "Endothelial adrenal cyst"}, {"label": "C", "disease": "Retroperitoneal abscess"}, {"label": "D", "disease": "Pheochromocytoma"}], "answer_idx": "B", "symptoms": ["A woman in her 30s with a history of chronic hepatitis B was found on computed tomography to have a 13-cm retroperitoneal mass on her left side with a significant mass effect", "The patient was symptomatic with intermittent left upper quadrant abdominal pain that was associated with nausea and constipation", "She reported no other gastrointestinal, genitourinary, or overall symptoms", "In addition to chronic hepatitis B, her medical history was notable for hypertension and her surgical history was significant for a recent liver biopsy", "Physical examination revealed a soft, nontender, minimally distended abdomen with no palpable masses and no costovertebral angle tenderness", "Laboratory test results at presentation revealed complete blood cell count, comprehensive metabolic panel, and amylase values within the reference ranges and a hepatitis B virus quantification level of 1590 IU/mL", "A computed tomographic scan of the abdomen and pelvis using intravenous contrast was obtained (Figure 1)", "Coronal computed tomographic imaging on initial presentation"], "s1": [0, 1, 6, 7], "s2": [2, 3, 4, 5]} {"key": 1189, "questions": "What Is the Diagnosis?", "options": [{"label": "A", "disease": "Tuberculosis"}, {"label": "B", "disease": "Nontuberculous mycobacterial infection"}, {"label": "C", "disease": "Coccidioidomycosis"}, {"label": "D", "disease": "Lymphoma"}], "answer_idx": "C", "symptoms": ["An 18-year-old Hispanic man residing in Los Angeles, California, presented with bilateral masses at the base of his neck, which he had for 2 months", "The left-sided mass had been spontaneously draining straw-colored fluid for 5 days", "Three months prior to presentation, he had received a diagnosis of pneumonia", "his condition improved after a course of azithromycin", "He denied ever having a fever, weight loss, contact with any sick persons, or sexual activity", "using illicit drugs", "or traveling outside of southern California", "Examination revealed mildly tender, fluctuant masses at the base of his neck, each measuring 4 cm in diameter (Figure, A)", "Laboratory values included a white blood cell count of 13/μL (to convert to ×109 per liter, multiply by 0.001) (69% neutrophils, 21% lymphocytes, and 2.3% eosinophils [to convert to proportion of 1.0, multiply by 0.01]), a hemoglobin level of 12.3 g/dL (to convert to grams per liter, multiply by 10.0), and a platelet count of 337 × 103/μL (to convert to ×109 per liter, multiply by 1.0)", "The results of a screening antibody test for human immunodeficiency virus and of a Mantoux tuberculin skin test were negative", "A chest radiograph revealed bilateral hilar adenopathy (Figure, B)", "An excisional lymph node biopsy of the left-sided mass was performed (Figure, C)", "A, Bilateral level IV (low jugular) cervical adenopathy", "B, Chest radiograph showing bilateral hilar adenopathy", "C, Histological examination of the excised lymph node showing multiple spherules containing endospores (arrowheads) (hematoxylin-eosin, original magnification ×200)"], "s1": [0, 1, 4, 5, 6, 7], "s2": [2, 3, 8, 9, 10, 11, 12, 13, 14]} {"key": 1190, "questions": "What Is the Diagnosis?", "options": [{"label": "A", "disease": "Splenic pelosis"}, {"label": "B", "disease": "Echinococcal splenic cystic disease"}, {"label": "C", "disease": "Splenic lymphangiomatosis"}, {"label": "D", "disease": "Non-Hodgkins lymphoma"}], "answer_idx": "C", "symptoms": ["An otherwise healthy woman in her early 20s was referred to our general surgery clinic for massive splenomegaly after developing a migratory, urticaric maculopapular rash over her body earlier in the year", "On physical examination, the inferior tip of the spleen extended well below the costal margin into her left lower quadrant", "In addition to the rash and splenomegaly, the patient also described several months of fatigue, early satiety, and back pain", "However, she denied other constitutional symptoms such as fevers, night sweats, chills, arthralgias, or weight loss", "She did not have any personal history of mononucleosis, Lyme disease, arthritis, or bleeding disorders", "In addition, there was no family history of hematologic diseases or lymphoma", "Findings from a hematologic workup were unremarkable", "Cross-sectional imaging was performed (Figure 1)"], "s1": [0, 1, 2, 3], "s2": [4, 5, 6, 7]} {"key": 1191, "questions": "What Is the Diagnosis?", "options": [{"label": "A", "disease": "Catheterization of left pericardiacophrenic vein"}, {"label": "B", "disease": "Catheterization of left internal thoracic vein"}, {"label": "C", "disease": "Catheterization of left anomalous pulmonary vein"}, {"label": "D", "disease": "Catheterization of left superior intercostal vein"}], "answer_idx": "C", "symptoms": ["A man in his mid-50s presented to the hospital with malaise, fevers, hypotension, and altered mental status", "Three months prior, he had an episode of native aortic valve endocarditis, necessitating aortic valve replacement with a pericardial bioprosthesis", "He developed postoperative acute renal failure, which progressed to chronic renal failure requiring dialysis via tunneled catheter", "On this admission, he was found to have methicillin-resistant Staphylococcus aureus (MRSA) bacteremia", "Transesophageal echocardiography demonstrated a vegetation on his prosthetic aortic valve", "He was diagnosed as having MRSA prosthetic valve endocarditis and admitted to the cardiovascular surgical intensive care unit prior to surgical intervention", "In the setting of MRSA bacteremia, his tunneled dialysis catheter had been removed", "Owing to his dialysis needs, a temporary dialysis catheter was placed in the right internal jugular vein", "As intravenous access was required for vasoactive medications and intravenous antibiotics, a triple-lumen central catheter was placed in the left internal jugular vein under ultrasonographic guidance", "The catheter went in easily without resistance, was drop tested to confirm venous placement, and flushed and drew easily, and venous waveforms and pressures were transduced", "However, on postprocedure radiography, the tip of the triple-lumen catheter was not in the expected location (Figure 1)", "The catheter was removed, and a new-stick insertion of a new catheter was attempted under ultrasonographic guidance", "Chest radiograph showing left internal jugular catheter placement (arrowhead)"], "s1": [0, 1, 2, 3, 4, 5, 6], "s2": [7, 8, 9, 10, 11, 12]} {"key": 1192, "questions": "What Is Your Diagnosis?", "options": [{"label": "A", "disease": "Orbital cellulitis"}, {"label": "B", "disease": "Rhabdomyosarcoma of the extraocular muscles"}, {"label": "C", "disease": "Idiopathic orbital inflammatory syndrome"}, {"label": "D", "disease": "Thyroid ophthalmopathy"}], "answer_idx": "C", "symptoms": ["An adolescent boy presented to our hospital with a 5-day history of right eyelid swelling and redness accompanied by blurred vision and pain with ocular movement", "This was his third hospitalization for these symptoms", "the previous admissions were 2 and 4 months earlier, with no symptoms in the intervals", "During the prior admissions, treatment included antibiotics for orbital cellulitis and corticosteroids for suspected antibiotic allergies", "Physical examination was notable for right upper and lower eyelid edema and erythema, conjunctival hyperemia, chemosis, and proptosis (Figure, A)", "The left eye appeared normal at admission", "However, within 24 hours after admission, our patient developed left chemosis with eyelid edema and erythema", "His vision and pupillary responses were normal", "Fundus examination revealed mild blurring of the disc margins only on the right side", "Slitlamp examination identified numerous cells in the right anterior chamber", "Extraocular movements were limited in all directions in both eyes, with a larger deficit in the right eye", "Adolescent boy with eyelid swelling, redness, and pain", "A, Swelling of the right orbital (white arrowhead) and periorbital (black arrowhead) tissues and proptosis, with mild swelling of the left upper eyelid (blue arrowhead)", "B, Maxillofacial computed tomography with intravenous contrast showed severe right proptosis with stretching of the right optic nerve (white arrowhead) and thickening of the rectus muscles and soft tissues (black arrowhead)", "Mild enlargement of the lacrimal gland on the left side can also be seen (blue arrowhead)", "C, Maxillofacial magnetic resonance imaging with intravenous contrast", "Coronal (left) and axial (right) views showed inflammation involving the intraconal fat and perineural soft tissues (white arrowhead), as well as the inferior and lateral extraocular muscles, on the right side", "The black arrowheads indicate the muscles in the coronal and axial views", "Computed tomography (Figure, B) and magnetic resonance imaging (MRI) (Figure, C) of the orbit were performed", "The results of these examinations, in conjunction with the patient’s history, prompted biopsy"], "s1": [0, 1, 2, 3, 6, 11], "s2": [4, 5, 7, 8, 9, 10, 12, 13, 14, 15, 16, 17, 18, 19]} {"key": 1193, "questions": "What Is the Diagnosis?", "options": [{"label": "A", "disease": "Appendiceal abscess"}, {"label": "B", "disease": "Ileal intussusception"}, {"label": "C", "disease": "Cecal volvulus"}, {"label": "D", "disease": "Appendiceal intussusception"}], "answer_idx": "D", "symptoms": ["A woman in her 40s presented to our department with lower abdominal pain, lasting for a few weeks, with nausea and vomiting but no diarrhea or fever", "Her medical history was unremarkable", "in particular, she had no history of endometriosis", "She had visited the emergency department of another hospital 2 weeks before with the same symptoms and had received conservative treatment", "On physical examination in the present visit, she had lower abdominal tenderness but no sign of peritoneal irritation", "A mass was palpable in the lower right abdominal quadrant", "The results of routine blood investigations, including white blood cell count, hemoglobin concentration, and C-reactive protein level, were within normal limits, as they had been in the previous hospital", "Abdominal ultrasonographic examination was performed, followed by abdominal computed tomography", "the results were suggestive of an ileal intussusception (Figure, A)", "A colonoscopy revealed the presence of a bulge in the cecal lumen (Figure, B)", "A, Abdominal computed tomography suggesting the presence of an ileal intussusception", "B, Colonoscopy revealing the presence of a bulge in the cecal lumen"], "s1": [0, 1, 2, 3, 4, 5, 6], "s2": [7, 8, 9, 10, 11]} {"key": 1194, "questions": "What Is the Diagnosis?", "options": [{"label": "A", "disease": "Recurrent prostatic carcinoma"}, {"label": "B", "disease": "Neurofibroma"}, {"label": "C", "disease": "Malignant peripheral nerve sheath tumor"}, {"label": "D", "disease": "Pseudosarcomatous fibromyxoid tumor"}], "answer_idx": "D", "symptoms": ["A 66-year-old man was doing well 6 months after robotic radical prostatectomy for prostatic carcinoma and synchronous left radical nephrectomy for renal cell carcinoma", "However, on surveillance computed tomography, he was noted to have a new pelvic mass that was concerning for a recurrent malignant neoplasm", "He denied having pelvic pain, urinary symptoms, or bowel symptoms", "His prostate-specific antigen level was undetectable", "Subsequent magnetic resonance imaging revealed a heterogeneously enhancing mass within the prostatic bed, which had central necrosis (Figure)", "The mass measured 4.8 × 5.5 × 5.0 cm and was contiguous with the right obturator internus", "No appreciable lymphadenopathy was noted", "Computed tomography–guided biopsy revealed spindle cells with a low mitotic index", "Of note, the patient’s son has a history of neurofibromatosis 1. The result of immunohistochemical staining was diffusely positive for desmin and smooth muscle actin, as well as focally positive for pan-keratin and S100. The staining result was negative for CD34 and CDK4. Fluorescence in situ hybridization failed to demonstrate MDM2 (OMIM 164785) gene amplification", "Magnetic resonance imaging of the pelvis reveals a large heterogeneously enhancing mass within the prostatic bed", "Axial (A and B) and sagittal (C and D) views reveal that the mass has T1 isointensity to hypointensity and T2 hyperintensity, suggesting internal areas of necrosis", "A marked mass effect is seen on the anterior-inferior bladder, which displaces the bladder superiorly", "The mass is contiguous with the right anterior aspect of the obturator internus and with the inner cortex of the midline superior pubic rami, which demonstrate erosion of the midline but no appreciable enhancing marrow signal"], "s1": [0, 1, 2, 3, 6, 7, 8], "s2": [4, 5, 9, 10, 11, 12]} {"key": 1195, "questions": "What Is the Diagnosis?", "options": [{"label": "A", "disease": "Malignant sarcoma"}, {"label": "B", "disease": "Fungal mycetoma"}, {"label": "C", "disease": "Giant infiltrating lipoma"}, {"label": "D", "disease": "Bacterial abscess"}], "answer_idx": "B", "symptoms": ["A man in his 60s presented to the emergency department with a right flank mass that had been present for 4 weeks", "The mass was associated with increasing pain as it grew rapidly, starting from the size of a dime and increasing to extend 15 cm", "The patient reported spontaneous drainage of pus from the mass and an unexplained weight loss", "His medical history included hypertension and diabetes mellitus", "he had no recent travel", "Physical examination revealed a large palpable mass with central fluctuance and surrounding erythema", "The mass extended from the subcostal region to the anterior superior iliac spine on palpation and was tender to touch", "No active drainage was noted", "however, multiple pustules were present (Figure 1A)", "Computed tomography of the chest, abdomen, and pelvis showed a large (10 × 15 cm) soft-tissue mass in the right lateral abdominal wall, extending from the level of the diaphragm to the iliac crest (Figure 1B and C)", "The mass appeared to directly invade the liver and extend through intercostal spaces with multiple small areas of calcification", "Core needle biopsy showed inconclusive results, and a recommendation was made for a fresh tissue biopsy", "The patient was then taken to the operating room where an excisional biopsy was performed", "A, Right flank mass on initial presentation", "B, Coronal view of a computed tomography (CT) scan showing a right flank mass invading the liver", "C, Axial view of the CT scan showing the flank mass extending from the diaphragm to the iliac crest"], "s1": [0, 1, 2, 3, 4, 6, 5, 7, 8], "s2": [9, 10, 11, 12, 13, 14, 15]} {"key": 1196, "questions": "What Is Your Diagnosis?", "options": [{"label": "A", "disease": "Fixed drug eruption"}, {"label": "B", "disease": "Tinea corporis"}, {"label": "C", "disease": "Lyme disease"}, {"label": "D", "disease": "Granuloma annulare"}], "answer_idx": "C", "symptoms": ["A 13-year-old girl with a history of Crohn disease presented to the pediatric dermatology clinic for evaluation of a rash on her buttocks and back", "The patient noticed the rash 1 day prior to presentation", "The rash was asymptomatic with no pain or pruritus", "Fourteen days prior to presentation, the patient was seen in the emergency department for a temperature greater than 102°F", "Since that time, the patient reported low-grade fever and 1 week of headache for which she had been taking ibuprofen", "She also reported fatigue", "She denied any preceding upper respiratory tract symptoms", "Physical examination revealed a well-nourished girl in no acute distress with approximately 10 pink-violaceous nummular and annular patches ranging from 1 to 4 cm in diameter (Figure)", "These patches were located on the trunk, buttocks, and thighs bilaterally", "The patient’s medical history is significant for a diagnosis of Crohn disease for which she receives several long-term immunomodulatory medications with no recent changes", "She has a known allergy to infliximab and had not been exposed to this drug recently", "She lives in a heavily wooded area with 2 dogs and 3 cats, although she did not recall a tick bite", "Photograph of the left buttock depicting one of the annular pink-violaceous patches"], "s1": [0, 1, 2, 7, 8, 12], "s2": [3, 4, 5, 6, 9, 10, 11]} {"key": 1197, "questions": "What Is the Diagnosis?", "options": [{"label": "A", "disease": "Carcinoid tumor"}, {"label": "B", "disease": "Desmoid tumor"}, {"label": "C", "disease": "Angiofollicular lymph node hyperplasia"}, {"label": "D", "disease": "Gastrointestinal stromal tumor"}], "answer_idx": "C", "symptoms": ["A 28-year-old man presented with several months of constant, burning upper abdominal pain centered on the epigastrium", "The pain was associated with morning nausea and was worsened by food and alcohol", "He was given trials of proton pump inhibitors and dicyclomine with no relief", "He had no prior chronic medical conditions", "His family history was positive for a brother with kidney stones", "He was a former smoker but denied current drug or alcohol use", "Review of systems was positive for dry throat, gastroesophageal reflux, and frequent headaches", "Physical examination was notable for a faintly palpable midabdominal mass slightly to the left and above the umbilicus with localized discomfort to palpation", "Ultrasonography of the abdomen showed a hypoechoic, hypervascular, solid mass left of the midline (Figure, A)", "A subsequent computed tomography scan of the abdomen and pelvis demonstrated a left adrenal myelolipoma and a 4.4 × 3.7-cm hypervascular soft tissue mass just left of the midline within the mesentery in close approximation with the small bowel, centered at the level of the L3-L4 intervertebral disc (Figure, B)", "A laparotomy was performed and a round, well-circumscribed soft tissue mass located approximately 90 cm distal to the ligament of Treitz at the base of the mesentery was removed intact without the need for bowel resection (Figure, C)", "Pathologic photomicrograph of the specimen is shown (Figure, D)", "Preoperative imaging of the mesenteric mass and gross surgical specimen and histopathologic analysis", "A, Preoperative ultrasonography showing a hypoechoic, solid mass", "B, Preoperative computed tomography scan showing a hypervascular soft tissue mass", "C, Gross surgical specimen", "D, Histopathologic analysis of the mesenteric mass (hematoxylin-eosin)"], "s1": [0, 1, 6, 7, 3, 4, 5], "s2": [2, 8, 9, 10, 11, 12, 13, 14, 15, 16]} {"key": 1198, "questions": "What Is the Diagnosis?", "options": [{"label": "A", "disease": "Duodenal injury"}, {"label": "B", "disease": "Mesenteric hematoma"}, {"label": "C", "disease": "Avulsed uterine fibroid"}, {"label": "D", "disease": "Right gonadal vein injury"}], "answer_idx": "C", "symptoms": ["A 33-year-old woman presented to the emergency department as a restrained driver in a motor vehicle collision with air bag deployment", "The patient had diffuse lower abdominal pain with nausea and one episode of emesis", "Her vital signs were notable for a blood pressure of 108/69 mm Hg and a heart rate of 124 beats/min", "Physical examination findings revealed tachycardia and a distended, diffusely tender abdomen without evidence of a seatbelt sign or peritonitis", "Her laboratory findings were unremarkable except for a hemoglobin level of 10.5 g/dL (to convert to grams per liter, multiply by 10) and a white blood cell count of 21 300/μL (to convert to ×109/L, multiply by 0.001)", "Her medical history was significant only for asthma", "Computed tomography revealed a large amount of free fluid throughout the abdomen and pelvis with an area of focal density consistent with a hematoma in the left midabdomen that measured 7.3 × 6.1 × 7.5 cm (Figure 1)", "There was also an area of free fluid in the right lower quadrant with irregularity of the right gonadal vein and a heterogeneous and bulky uterus", "Computed tomographic image (sagittal view)", "Image shows a large amount of free fluid throughout the abdomen and pelvis with an area of focal density consistent with hematoma in the left midabdomen that measured 7.3 × 6.1 × 7.5 cm"], "s1": [0, 1, 2, 3, 5], "s2": [4, 6, 7, 8, 9]} {"key": 1199, "questions": "What Is the Diagnosis?", "options": [{"label": "A", "disease": "Colorectal carcinoid tumor"}, {"label": "B", "disease": "Ileocolic intussusception"}, {"label": "C", "disease": "Cecal volvulus"}, {"label": "D", "disease": "Metastatic carcinomatosis"}], "answer_idx": "B", "symptoms": ["A 34-year-old female Jehovah’s Witness presented to the emergency department with a 3-day history of colicky lower abdominal pain, nausea without vomiting, and constipation after passing a single small bloody bowel movement", "Physical examination demonstrated a soft, nontender, and nondistended abdomen without peritoneal signs or palpable masses", "Digital rectal examination revealed no stool in the vault and gross blood and pelvic examination were negative", "Surgical history was notable only for a previous cesarean delivery", "Medical history was significant for 2 years of waxing and waning generalized abdominal pain and a hospital admission for small-bowel obstruction 11 months prior that was managed conservatively", "Endoscopic evaluation revealed multiple polyps in the colon, stomach, and duodenum consistent with a diagnosis of juvenile polyposis syndrome", "Admission laboratories demonstrated a lactate level of 18 mg/dL (to convert to millimoles per liter, multiply by 0.111), white blood cell count of 6000/μL (to convert to ×109/L, multiply by 0.001), and hemoglobin level of 13.3 g/dL (to convert to grams per liter, multiply by 10)", "Amylase and lipase levels and chemistry findings were normal", "Computed tomography with oral and intravenous contrast was obtained and is shown with imaging from her initial presentation 11 months prior (Figure 1)", "Axial and coronal views of computed tomography imaging on initial presentation (A) and 1 year later (B)"], "s1": [0, 3, 4, 5], "s2": [1, 2, 6, 7, 8, 9]} {"key": 1200, "questions": "What Is the Diagnosis?", "options": [{"label": "A", "disease": "Ruptured acute appendicitis"}, {"label": "B", "disease": "Cecal inflammation with abscess"}, {"label": "C", "disease": "Iatrogenic pancreas transplant thrombosis"}, {"label": "D", "disease": "Hematoma"}], "answer_idx": "C", "symptoms": ["A 52-year-old man presented with a 2-day history of nausea and low-grade fever to a local hospital", "He had a history of simultaneous pancreas-kidney transplantation 15 years ago and both grafts had been functioning well", "Two days prior to presentation, he had a peripheral angiogram done to evaluate a nonhealing left foot ulcer by an interventional cardiologist, at which time a stent graft was deployed to the right common iliac artery for a pseudoaneurysm that was incidentally discovered", "On examination, he was in no distress, had a temperature of 37° C, and blood pressure of 91/44 mm Hg", "Two days after presentation, a computed tomographic scan of the abdomen and pelvis was obtained (Figure)", "It showed an air-containing abscess and a stent in the common iliac artery", "A stent can be seen in the right common iliac artery (A and C, arrowheads)", "A gas-containing phlegmon was also evident (B and C, asterisks)"], "s1": [0, 1, 2, 3], "s2": [4, 5, 6, 7]} {"key": 1201, "questions": "What Is The Diagnosis?", "options": [{"label": "A", "disease": "Acute appendicitis"}, {"label": "B", "disease": "Cecal diverticulum"}, {"label": "C", "disease": "Carcinoid tumor of the appendix"}, {"label": "D", "disease": "Appendiceal intussusception"}], "answer_idx": "D", "symptoms": ["A 26-year-old woman in the 11th week of gestation with a history of nephrolithiasis was admitted to our medical center reporting of right lower quadrant pain, loss of appetite, nausea, and vomiting occurring over 5 days", "The pain was initially transient and colicky, but had increased in intensity and became constant", "She denied having fevers, chills, or change in bowel habits", "During examination, the patient had significant focal tenderness to palpation, but displayed no peritoneal signs", "Her laboratory results were unremarkable, without any findings for leukocytosis or bandemia", "Computed tomograpy (CT) scan and ultrasonography were inconclusive and revealed only mild haziness in the right lower quadrant and an abnormal-appearing cecum", "The appendix was not visualized", "Magnetic resonance imaging of the abdomen was then performed to further investigate the area surrounding the cecum (Figure, A)", "Given these symptoms and findings, the decision was made to explore this patient laparoscopically", "After careful intraoperative evaluation, it became evident that she would require an ileocecectomy", "The removed specimen was sent to pathology for analysis", "The histologic analysis is presented in Figure, B", "A, Abdominal magnetic resonance imaging", "Arrowhead points at the suspicious structure within the cecum", "B, Histologic slide of the resected appendix shows positive progesterone receptor immunostaining (original magnification x400)", "The patient recovered well and was discharged home", "On postoperative day 11 she returned with symptoms of fever and increasing right lower quadrant pain", "Imaging revealed an abscess near the ileocolic anastomosis, which was successfully treated with percutaneous drain placement", "The patient recovered and was discharged", "Weeks later, she presented to an outside facility with spotting and cramping", "Intrauterine fetal demise was noted and the patient underwent an emergency dilation and curettage"], "s1": [0, 19, 20], "s2": [1, 2, 3, 4, 5, 6, 7, 8, 9, 10, 11, 12, 13, 14, 15, 16, 17, 18]} {"key": 1202, "questions": "What Is Your Diagnosis?", "options": [{"label": "A", "disease": "Langerhans cell histiocytosis"}, {"label": "B", "disease": "Contact dermatitis"}, {"label": "C", "disease": "Scabies"}, {"label": "D", "disease": "Postscabetic acropustulosis"}], "answer_idx": "C", "symptoms": ["A 9-week-old girl had a rash on her back that had been present for 7 weeks", "The rash was asymptomatic and started as red bumps that then evolved into pustules and blisters", "On further questioning, it was discovered that the patient’s father worked in a nursing home that had experienced a recent outbreak of scabies", "Shortly after the onset of the outbreak, the patient and all family members were treated with 2 once-weekly doses of permethrin, 5%, cream", "All other family members had significant improvement with the treatment", "however, our patient’s lesions continued to worsen during the subsequent 2 weeks", "On physical examination, the patient had numerous erythematous macules, papules, and pustules on her back", "Many of the lesions had superimposed hemorrhagic crust and surrounding scale", "In addition, several grouped papules were noted, including some in a linear array (Figure 1A and B)", "The rest of her cutaneous examination results were unremarkable except for a few fading pink papules noted on her left abdomen (Figure 1C)", "Numerous erythematous papules and pustules on the back, many with overlying hemorrhagic crust and surrounding scale (A) and (B), and few erythematous papules on the left lower abdomen (C)"], "s1": [0, 1, 6, 7, 8, 9, 10], "s2": [2, 3, 4, 5]} {"key": 1203, "questions": "What Is The Diagnosis?", "options": [{"label": "A", "disease": "Hepatic artery pseudoaneurysm"}, {"label": "B", "disease": "Angioleiomyoma"}, {"label": "C", "disease": "Neuroendocrine tumor"}, {"label": "D", "disease": "Angiosarcoma"}], "answer_idx": "B", "symptoms": ["A previously healthy, 37-year-old man presented for a routine physical examination during which a pulsatile mass in his midepigastrium was palpated", "Abdominal ultrasonography showed a well-defined hypoechoic lesion abutting the pancreas", "Further imaging by contrast-enhanced computed tomography (CT) and 3-dimensional reconstruction revealed a well-circumscribed, 3-cm lesion surrounding the common hepatic artery (Figure, A and B)", "The lesion demonstrated patchy arterial enhancement, and a small irregularity of the common hepatic artery was present within the lesion", "Endoscopic ultrasonography demonstrated a hypoechoic, well-defined mass", "fine-needle aspiration showed no abnormal cells within the paucicellular specimen", "Surgical exploration revealed a firm, ovoid lesion surrounding the hepatic artery at the origin of the gastroduodenal artery (Figure, C)", "Following resection, the lesion was bisected to reveal a smooth white arterial lumen surrounded by grayish fibrotic tissue with punctate hemorrhagic foci", "Histologic sections of the specimen demonstrated small, singly dispersed vessels embedded in a background of fascicles of spindled cells with monotonous, fusiform nuclei and abundant eosinophilic cytoplasm", "Focally, areas of myxoid degeneration were present", "Despite extensive sampling, no mitotic activity, cytologic atypia, or areas of necrosis were identified", "There was strong, diffuse cytoplasmic staining for desmin (Figure, D), and staining results for S100 were negative", "Furthermore, CD34 stained strongly positive in vessels but failed to highlight the intervening stroma", "A, Computed tomography of an enhancing mass (dotted circle) surrounding the common hepatic artery with emanating gastroduodenal artery, right hepatic artery, and left hepatic artery (arrowheads)", "B, 3-Dimensional reconstruction of the location of the lesion (dotted circle) with relation to the involved vessels and surrounding structures", "C, Intraoperative view of a 3-cm mass surrounding the common hepatic artery just prior to the trifurcation of the gastroduodenal artery, right hepatic artery, and left hepatic artery", "D, The tumor cells showed strong cytoplasmic expression of the smooth-muscle marker desmin", "Original magnification ×20."], "s1": [0, 1, 2, 3, 4, 6, 7, 13, 14, 15], "s2": [5, 8, 9, 10, 11, 12, 16, 17]} {"key": 1204, "questions": "What Is the Diagnosis?", "options": [{"label": "A", "disease": "Hedrocele"}, {"label": "B", "disease": "Rectal prolapse"}, {"label": "C", "disease": "Anal neoplasm"}, {"label": "D", "disease": "External hemorrhoids"}], "answer_idx": "A", "symptoms": ["A woman in her 50s was hospitalized for a painful anal mass", "She had a history of hypertension and depression and was receiving omeprazole and paroxetine", "The mass had appeared 2 days before presentation and was associated with vomiting", "Additionally, the patient had not passed stools or had intestinal gas for 2 days", "On physical examination, the mass was exteriorized through the anal canal", "It was covered by hypoxemic rectal mucosa, but no digestive lumen could be identified inside the mass (Figure 1)", "Furthermore, the patient had a distended abdomen, absent bowel sounds, and no tenderness to palpation", "No abdominal scar was visible, and no groin hernia was palpable", "Her blood pressure was 160/80 mm Hg and her temperature was 37.8°C", "Results of complete blood cell count, coagulation tests, and basic chemistry panel were all within normal limits", "An abdominal radiograph showed several air-fluid levels without pneumoperitoneum, confirming an intestinal occlusion", "View of the mass exteriorized through the anus"], "s1": [0, 2, 3, 4, 5, 6, 10, 11], "s2": [1, 7, 8, 9]} {"key": 1205, "questions": "What Is Your Diagnosis?", "options": [{"label": "A", "disease": "Erythema nodosum"}, {"label": "B", "disease": "Sarcoidosis"}, {"label": "C", "disease": "Systemic lupus erythematosus"}, {"label": "D", "disease": "Minocycline-induced cutaneous polyarteritis nodosa"}], "answer_idx": "D", "symptoms": ["An 18-year-old woman presented with a 4-week history of a solitary painful bullous skin lesion over the lower right anterior tibia accompanied by generalized myalgia, polyarthalgia, joint stiffness, paraesthesia, and episodic ankle swelling", "The lesion first appeared as a skin-colored papule and, within a week, developed an erythematous halo", "Her medical history includes asthma and acne", "To our knowledge, there is no family history of this condition", "Her medications included minocycline hydrochloride (for 25 months for acne), ranitidine hydrochloride, contraceptive pills, and iron supplements", "An initial physical examination revealed mild synovitis in both ankles and an erythematous, raised papule overlying the distal aspect of the right anterior tibia", "Treatment with prednisone acetate (10 mg daily) was initiated", "Seven weeks later, the original skin lesion evolved into an ulcer with erythematous, raised borders (Figure 1)", "The case patient had new eruptions of nonindurated, violaceous, nontender, reticulated patches overlying the skin on the bilateral anterior tibias, consistent with livedo reticularis", "She denies any previous episodes, and the results of a review of mitigating factors were negative", "Ulcer with raised border and erythematous halo in distal right shin", "A laboratory workup revealed a positive antinuclear antibody 1:640 nucleolar pattern, an elevated erythrocyte sedimentation rate, and elevated C-reactive protein and serum IgG levels", "The results of the following laboratory studies were negative or normal: complete blood cell count, urinalysis, liver profile, Lyme disease, parvovirus B19, Monospot test, uric acid, rheumatoid factor, antistrepolysin antibody, anti-Scl70, antihepatitis C virus, hepatitis B surface antigen, hepatitis B core antibody, cytomegalovirus antibody, and human T-lymphotropic viruses 1 and 2 antibody", "Culture results of the skin lesion were negative", "The results from chest radiography were normal"], "s1": [0, 1, 5, 7, 8, 10, 13], "s2": [2, 3, 4, 6, 9, 11, 12, 14]} {"key": 1206, "questions": "What Is the Diagnosis?", "options": [{"label": "A", "disease": "Biliary cystadenomas"}, {"label": "B", "disease": "Amebic liver abscess secondary to Entamoeba histolytica infection"}, {"label": "C", "disease": "Metastatic cystic neuroendocrine tumors"}, {"label": "D", "disease": "Cystic hepatocellular carcinomas"}], "answer_idx": "C", "symptoms": ["A 74-year-old woman was seen by her primary care physician for dull epigastric and right upper quadrant abdominal pain of 1-year duration", "Routine laboratory values were within normal limits", "The patient underwent ultrasonography of the abdomen, and 2 cystic lesions were discovered in the liver", "Magnetic resonance imaging of the abdomen revealed 2 multiloculated cystic lesions in segments IV and V of the liver (Figure, A)", "Additional laboratory values included normal carcinoembryonic antigen and cancer antigen 19-9 levels", "Percutaneous aspiration with a biopsy of one of the cystic lesions was performed that revealed atypical cells suspicious for malignancy", "The patient was then referred for management to our institution, where she underwent an exploratory laparotomy and enucleation of these 2 complex multiloculated cystic lesions (Figure, B)", "A, Axial image from abdominal magnetic resonance imaging", "B, Intraoperative view of enucleated lesions in hepatic segments IV and V", "Amebic liver abscess secondary to Entamoeba histolytica infection"], "s1": [0, 1, 4, 9], "s2": [2, 3, 5, 6, 7, 8]} {"key": 1207, "questions": "What Is the Diagnosis?", "options": [{"label": "A", "disease": "A. Metastatic rectal adenocarcinoma"}, {"label": "B", "disease": "B. Spindle cell sarcoma of the chest wall"}, {"label": "C", "disease": "C. Metastatic breast carcinoma"}, {"label": "D", "disease": "D. Desmoid tumor of chest wall"}], "answer_idx": "B", "symptoms": ["A 55-year-old woman presented to the clinic with a mass in her left breast that had been enlarging over the previous 3 months", "The patient denied any antecedent trauma or infection", "Her medical history was significant for hypertension and left breast invasive ductal carcinoma in 2007 treated with lumpectomy and adjuvant radiation as well as synchronous rectal cancer after neoadjuvant chemoradiation and abdominoperineal resection", "As part of her treatment course, the patient received a total dose of 50.5 Gy pelvic irradiation and 60 Gy irradiation to the left breast and axillary basin", "The patient was followed up with yearly computed tomography of the abdomen and pelvis in addition to yearly digital mammography", "Her last mammogram, 5 months prior to presentation, was unremarkable", "On physical examination, she had a hard, well-circumscribed, fixed mass in the upper, outer quadrant of her left breast", "The lesion was not tender to palpation nor was there any associated axillary or supraclavicular lymphadenopathy", "Examination of the nipple as well as the contralateral breast and axilla was unremarkable", "Computed tomography of the thorax revealed a 6.7 × 4 cm mass arising from the left chest wall distinct from the left breast with extent to the pleural surface of the left thorax (Figure, A)", "Positron emission tomography revealed metabolic activity within the mass but did not reveal any distant disease", "Core needle biopsy confirmed the diagnosis and the patient was offered resection with reconstruction", "A, Computed tomographic axial section of the thorax illustrating a left chest wall mass measuring 6.7 × 4 cm apparently distinct from the left breast parenchyma with possible pleural invasion", "B, Postresection wound including chest wall", "Lung parenchyma adjacent to the chest wall reveals stippling and fibrosis consistent with previous irradiation", "The patient underwent a left radical mastectomy including resection of the pectoralis major and minor and resection of the adjacent chest wall resulting in a chest wall defect spanning ribs 2 through 4 (Figure, B)", "The patient underwent immediate reconstruction of the chest wall with 2-mm-thick expanded polytetrafluoroethylene and latissimus dorsi pedicle flap reconstruction of the soft tissue defect and breast mound", "B", "Spindle cell sarcoma of the chest wall"], "s1": [0, 6, 7, 8, 10, 11, 13, 16], "s2": [1, 2, 3, 4, 5, 9, 12, 14, 15, 17, 18]} {"key": 1208, "questions": "What Is Your Diagnosis?", "options": [{"label": "A", "disease": "Numular eczema"}, {"label": "B", "disease": "Erythema annulare centrifugum"}, {"label": "C", "disease": "Erythema chronicum migrans"}, {"label": "D", "disease": "Tinea incognito"}], "answer_idx": "D", "symptoms": ["A 4-year-old, 15-kg girl presented with a 3-month history of a pruritic rash bilaterally on the cheeks, initially thought to be related to mosquito bites", "Treatment with desonide, 0.05%, ointment twice daily proved to be ineffective", "The patient’s rash also failed to clear with a therapeutic trial with griseofulvin liquid (125 mg/5 mL), 5 mL daily, and econazole, 1%, cream twice daily for 2 weeks and later ciclopirox, 0.77%, cream twice daily for 2 weeks, although the patient reported some improvement", "The presence of persistent acneiform papules on the cheeks with postinflammatory hyperpigmentation led to a therapeutic trial of erythromycin, 400 mg/5 mL, 1.3 mL orally twice a day, with food", "Test results for antinuclear antibody and rheumatoid factor were negative", "Three weeks later, the patient presented with 2 slowly expanding, hyperpigmented, ringlike lesions with a tendency toward central clearing and peripheral scaling and erythema on the right cheek (Figure 1)", "A 4-year-old girl presenting with two 5-cm-diameter annular patches (arrowheads) slowly increasing in size over 3 weeks on the right cheek"], "s1": [0, 4], "s2": [1, 2, 3, 5, 6]} {"key": 1209, "questions": "What Is the Diagnosis?", "options": [{"label": "A", "disease": "Glucagonoma"}, {"label": "B", "disease": "Insulinoma"}, {"label": "C", "disease": "VIPoma"}, {"label": "D", "disease": "Skin amyloidosis"}], "answer_idx": "A", "symptoms": ["A 66-year-old man presented with a skin rash on his lower limb", "He has been unemployed and bedridden for the last 3 years owing to his skin disease", "He has a history of ischemic heart disease, familial Mediterranean fever, and arthropathy", "He was treated with nonsteroidal anti-inflammatory drugs for pain relief and steroids for the skin disease", "Except for the skin rash, he was newly diagnosed as having diabetes mellitus", "On presentation, he noted anorexia and reported a 10-kg weight loss during the last 8 months", "On physical examination, he appeared cachectic", "His abdomen was soft with no palpable mass", "A macular rash with multiple infected ulcerated areas and diffuse lower limb edema was noticed (Figure 1)", "The remainder of his physical examination results were unremarkable"], "s1": [0, 1, 3, 4, 8], "s2": [2, 5, 6, 7, 9]} {"key": 1210, "questions": "What Is the Diagnosis?", "options": [{"label": "A", "disease": "Subacute bacterial endocarditis"}, {"label": "B", "disease": "Warfarin-induced skin necrosis"}, {"label": "C", "disease": "Calcific uremic arteriolopathy"}, {"label": "D", "disease": "Buerger disease"}], "answer_idx": "C", "symptoms": ["A 52-year-old man with type 2 diabetes mellitus and end-stage renal disease presented with painful and progressive cutaneous necrosis of his fingers, toes, and calves and generalized muscle weakness with difficulty getting out of bed and walking", "He had undergone hemodialysis for the past year and had a brachiocephalic fistula in his left arm", "His medical history was also significant for peripheral vascular disease, hypertension, and antiphospholipid antibody syndrome for which he was receiving warfarin sodium since 2010. His other medications were epoetin alpha, ferrous gluconate, cinacalcet hydrochloride, paricalcitol, sevelamer carbonate, amlodipine besylate, labetalol hydrochloride, lisinopril, simvastatin, pentoxifylline, and Humalog and Lantus insulin", "He did not smoke", "The patient had a temperature of 37.6°C and a grade II/VI systolic ejection murmur", "He had necrosis and eschar formation involving his fingers (Figure 1A), toes (Figure 1B and C), and left posterior medial calf (Figure 1D)", "He had violaceous skin with central eschar on the posterior medial aspect of his right leg (Figure 1E) that was exquisitely tender with gangrenous change", "He had normal femoral pulses with no distal pulses palpable", "Both feet were warm with good Doppler signals in the dorsalis pedis and posterior tibial arteries", "He had a palpable pulse and thrill in the brachiocephalic fistula in his left upper arm", "There was also a weak distal radial pulse", "He had good Doppler signals over the radial, ulnar, and palmar arch arteries bilaterally", "Photographs of necrosis and eschar formation involving fingers (A), right (B) and left (C) toes, left posterior medial calf (D), and violaceous skin with central eschar on the posterior medial aspect of the right leg (E)", "Laboratory studies revealed a white blood cell count of 13 900/μL (to convert to ×109 per liter, multiply by 0.001), a hemoglobin level of 7.7 g/dL (to convert to grams per liter, multiply by 10.0), a platelet count of 309 × 103/μL (to convert to ×109 per liter, multiply by 1.0), an international normalized ratio of 2.4, a blood urea nitrogen level of 88 mg/dL (to convert to millimoles per liter, multiply by 0.357), a creatinine level of 9.9 mg/dL (to convert to micromoles per liter, multiply by 88.4), a calcium level of 7.5 mg/dL (to convert to millimoles per liter, multiply by 0.25), an albumin level of 2.4 g/dL (to convert to grams per liter, multiply by 10), and an intact parathyroid hormone level of 349.7 pg/mL (to convert to nanograms per liter, multiply by 1.0)", "A skin biopsy specimen of the calf showed epidermal necrosis, dermal hemorrhage, focal dermal intravascular thrombi, and subcutaneous intravascular and extravascular calcifications"], "s1": [0, 1, 2, 4, 5, 6, 7, 8, 9, 10, 11, 14], "s2": [3, 12, 13]} {"key": 1211, "questions": "What Is Your Diagnosis?", "options": [{"label": "A", "disease": "Pyoderma gangrenosum"}, {"label": "B", "disease": "Ecthyma gangrenosum"}, {"label": "C", "disease": "Wegener granulomatosis"}, {"label": "D", "disease": "Meningococcemia"}], "answer_idx": "B", "symptoms": ["A 25-day-old newborn was admitted to our hospital with a diagnosis of bronchiolitis", "During admission, the infant developed violaceous skin lesions with edema in inguinal and axillary regions (Figure)", "Lesions became ulcerated within 3 to 4 days, with infarcted necrotic areas", "Blood tests showed leukopenia (white blood cell count, 2460/μL) with neutropenia (neutrophil count, 220/μL) (to convert both to ×109 per liter, multiply by 0.001)", "The lesions were biopsied and samples were sent for cultures and histopathological analysis", "Intravenous floxacillin was started, followed by surgical debridement due to progression of the major lesions", "Axillary inflammatory nodule (A) and inguinal inflammatory plaques with violaceous hue (B)", "His mother was addicted to drugs, she had a known hepatitis C virus infection, and the pregnancy was unsupervised", "Thirteen days before delivery, she visited the emergency department with flulike symptoms", "Routine infectious screening revealed negative serology for syphilis, hepatitis B virus, and human immunodeficiency virus (HIV) types 1 and 2 by enzyme-linked immunosorbent assay (ELISA)", "The infant was born at 40 weeks’ gestation by forceps delivery"], "s1": [0, 1, 2, 3, 4, 5, 6], "s2": [7, 8, 9, 10]} {"key": 1212, "questions": "What Is the Diagnosis?", "options": [{"label": "A", "disease": "A. Prostate cancer"}, {"label": "B", "disease": "B. Recurrent colon cancer"}, {"label": "C", "disease": "C. Internal iliac artery aneurysm"}, {"label": "D", "disease": "D. Primary lymphoma of the urinary bladder"}], "answer_idx": "C", "symptoms": ["An 88-year-old man with a history of hypertension, atrial fibrillation while receiving warfarin sodium, congestive heart failure, extended right hemicolectomy for colon cancer, and abdominal aortic aneurysm repair visited the hospital with epigastric pain of 3 days’ duration", "In addition, the patient had worsening lower back and lower extremity pain, treated as sciatica on an outpatient basis, for 3 months", "The patient had constipation, urinary incontinence, and generalized weakness but no fever, nausea or vomiting, or change in bowel movements", "Physical examination revealed a firm, nonpulsatile, slightly tender mass in the left lower quadrant and significant perineal ecchymosis", "Vital signs were normal", "Laboratory test results were remarkable for white blood cell count (13 800/μL [to convert to ×109 per liter, multiply by 0.001]), blood urea nitrogen level (72 mg/dL [to convert to millimoles per liter, multiply by 0.357]), creatinine level (4.2 mg/dL [to convert to micromoles per liter, multiply by 88.4]), hemoglobin level (9.8 g/dL [to convert to grams per liter, multiply by 10.0]), and international normalized ratio (2.7)", "Computed tomography of the abdomen and pelvis was performed (Figure)", "Computed tomographic images of the abdomen and pelvis", "A, Axial imaging shows that the bladder (black arrow) is displaced to the right, and a Foley catheter is located anteriorly", "The rectum (white arrow) is compressed posteriorly", "B, Coronal imaging shows that the bladder (arrow) is severely displaced, and a Foley catheter is located medially"], "s1": [0, 1, 2, 3, 5], "s2": [4, 6, 7, 8, 9, 10]} {"key": 1213, "questions": "What Is the Diagnosis?", "options": [{"label": "A", "disease": "A. Pulmonary aspergilloma"}, {"label": "B", "disease": "B. Pulmonary tuberculosis"}, {"label": "C", "disease": "C. Squamous cell carcinoma"}, {"label": "D", "disease": "D. Pulmonary hydatid cyst"}], "answer_idx": "C", "symptoms": ["A 62-year-old man was admitted to our hospital with a concern of productive cough, profuse hemoptysis, and intermittent breathlessness for about 5 months", "He had a 50-year history of cigarette smoking and had stopped smoking 5 months prior to being seen", "When the symptom of hemoptysis occurred 5 months earlier, chest radiography and computed tomography (CT) revealed a ground-glass opacity in the left upper lobe", "The patient was treated with an antifibrinolytic agent (tranexamic acid), and the hemoptysis gradually improved and follow-up chest radiography showed resolution of the lesion (Figure 1)", "However, about 1 month later, hemoptysis reoccurred and increased volume and frequency over the following 4 months", "After treatment with an antifibrinolytic agent, a ball-like mass with an air crescent sign in the left upper lobe was noted", "On physical examination, sonorous rhonchi were audible in the bilateral lungs", "Chest CT revealed a cavitary lesion with a central contrast-enhanced ball-like mass at the site of the prior ground-glass lesion", "Sputum examinations for acid-fast bacilli smear and culture, fungal culture, and cytology showed negative results", "Fiberoptic bronchoscopy revealed blood clots over the orifice of the left upper lobe bronchus, but no endobronchial lesion was detected", "The chest CT finding, characteristic of a fungus ball-like shadow with an air crescent sign, highly suggested a pulmonary aspergilloma", "However, the contrast enhancement was present within the mass, and lung malignancy could not be excluded", "Because of persistent massive hemoptysis, the patient underwent left exploratory thoracotomy", "A whitish, firm nodule about 2 cm was noted in a cavity in the left upper lung"], "s1": [0, 1, 4, 6, 8, 9], "s2": [2, 3, 5, 7, 10, 11, 12, 13]} {"key": 1214, "questions": "What Is Your Diagnosis?", "options": [{"label": "A", "disease": "Congenital melanocytic nevus"}, {"label": "B", "disease": "Epidermal nevus"}, {"label": "C", "disease": "Nevus sebaceous of Jadassohn"}, {"label": "D", "disease": "Inflammatory linear verrucous epidermal nevus"}], "answer_idx": "C", "symptoms": ["A 15-year-old male presented to the dermatology clinic with a lesion that had been present on his right nasal sidewall since birth", "At puberty, the lesion expanded in size, became irritated, and developed a papillated, rough texture", "The patient otherwise had no significant medical history and was in good health", "On physical examination, there was a 3.6-cm × 1.5-cm verrucous skin-colored to light-brown plaque on the right nasal sidewall (Figure)", "Clinical photograph of a 3.6-cm × 1.5-cm verrucous skin-colored to light-brown plaque on the right nasal sidewall"], "s1": [2], "s2": [0, 1, 3, 4]} {"key": 1215, "questions": "What Is the Diagnosis?", "options": [{"label": "A", "disease": "Carcinoma of the cecum"}, {"label": "B", "disease": "Ileocecal intussusception"}, {"label": "C", "disease": "Intussusception of the appendix"}, {"label": "D", "disease": "Diverticulum of the cecum"}], "answer_idx": "C", "symptoms": ["A 32-year-old woman presented to the hospital complaining about having colicky abdominal pain for the last 2 weeks", "The pain was usually generalized but sometimes radiated to the right iliac fossa", "The pain got worse with meals and was associated with nausea but no vomiting", "In the last 2 days, the pain was associated with loose stools, but there was no mucus or blood", "She had no comorbidities, such as diabetes mellitus or hypertension, and no previous admission to the hospital", "She is married and has 4 children", "her last child was born 40 days prior to her presentation to the hospital", "On examination, her vital signs were normal (temperature, 37.1°C", "pulse, 86 beats/min", "and blood pressure, 121/86 mm Hg)", "Her abdomen was soft and lax", "There was mild tenderness in the right iliac fossa, but no lump or lumps were felt", "Bowel sounds were not exaggerated", "Her white blood cell count was 8500/μL (to convert to ×109 per liter, multiply by 0.001), her hemoglobin level was 1.3 g/dL (to convert to grams per liter, multiply by 10.0), and her platelet count was 361 × 103/μL (to convert to ×109 per liter, multiply by 1.0)", "The results of renal and liver function tests were normal", "Abdominal ultrasonography revealed a 4.2 × 4.2-cm mass in the midabdominal region lateral to the gall bladder fossa", "The mass has the “donut sign” with central echogenicity (Figure 1A)", "An abdominal computed tomographic scan (Figure 1B) showed a 8.2 × 4 × 2.6-cm tubular thin-walled filling defect in the cecum and lower part of ascending colon", "The oral contrast agent can be seen around, proximal, and distal to the defect, with no obstruction noted", "Abdominal ultrasonographic image revealing a 4.2 × 4.2-cm mass with central echogenicity, the “donut sign” (A)", "Abdominal computed tomographic scan with oral contrast agent revealing a 8.2 × 4 × 2.6-cm tubular thin-walled filling defect in the cecum and lower part of ascending colon (B)", "The contrast is seen around, proximal, and distal to the defect, with no obstruction noted"], "s1": [0, 1, 2, 3, 4, 5, 6, 7, 8, 9, 10, 11, 12], "s2": [13, 14, 15, 16, 17, 18, 19, 20, 21]} {"key": 1216, "questions": "What Is the Diagnosis?", "options": [{"label": "A", "disease": "Duplicate gallbladder"}, {"label": "B", "disease": "Biliary cystadenoma"}, {"label": "C", "disease": "Type 1 choledochal cyst"}, {"label": "D", "disease": "Hydatid cyst"}], "answer_idx": "C", "symptoms": ["A 30-year-old female immigrant from Laos was referred to our institution after admission to an outside hospital for right upper quadrant abdominal pain with associated nausea and vomiting", "Her imaging results from the outside hospital included abdominal magnetic resonance cholangiopancreatography revealing a cystic abnormality in the region of the porta hepatis (Figure 1A)", "The patient denied any previous medical or surgical history", "On examination, the patient had mild right upper quadrant tenderness to palpation without signs of peritonitis", "Routine laboratory values were within normal limits, except for a mildly elevated amylase level", "A computed tomographic scan was obtained to further delineate the abnormality (Figure 1B)", "Magnetic resonance cholangiopancreatography (A) and abdominal computed tomography (B) revealing a cystic lesion in the region of the porta hepatis (arrowheads)", "F indicates feet", "H, head", "L, left", "and R, right"], "s1": [0, 1, 3, 4, 5, 6], "s2": [2, 7, 8, 9, 10]} {"key": 1217, "questions": "What Is Your Diagnosis?", "options": [{"label": "A", "disease": "Supernumerary nostril"}, {"label": "B", "disease": "Nasal encephalocele"}, {"label": "C", "disease": "Proboscis lateralis"}, {"label": "D", "disease": "Cerebro-oculo-nasal syndrome"}], "answer_idx": "C", "symptoms": ["A 10-day-old, full-term girl presented with a trunklike projection from the medial canthus of the left eye (Figure)", "There was no other craniofacial anomaly and bilateral nasal cavities were normal", "The trunk was 2.5 cm in length connected with a thin pedicle", "There was mucus discharge from the opening", "There was no history of teratogenic exposure, no family history of congenital defects, and no history of consanguinity", "Computed tomography (CT) scan and magnetic resonance imaging (MRI) showed no other associated craniofacial abnormality", "Full-term, 10-day-old girl with a trunklike projection from the medial canthus of the left eye"], "s1": [0, 1, 2, 6], "s2": [3, 4, 5]} {"key": 1218, "questions": "What Is the Diagnosis?", "options": [{"label": "A", "disease": "Liposarcoma"}, {"label": "B", "disease": "Gossypiboma"}, {"label": "C", "disease": "Mesenteric cyst"}, {"label": "D", "disease": "Uterine leiomyoma"}], "answer_idx": "B", "symptoms": ["A 28-year-old woman presented with a complaint of a painless abdominal mass that had been increasing in size during the past few years", "Several different physicians had evaluated the patient previously without a definitive diagnosis or treatment", "Her only significant medical history was a routine cesarean section 4 years earlier", "Physical examination findings revealed a firm, mobile, nontender, midline abdominal mass", "A computed tomographic (CT) scan was performed, which showed a well-circumscribed, 10-cm mass (Figure 1)", "The patient was scheduled for elective exploratory laparotomy and resection", "Scout (A) and cross-sectional (B) images from a computed tomographic scan of the abdomen", "A large, well-circumscribed mass (white arrow) is shown on the cross-sectional image"], "s1": [0, 1, 2, 3, 5], "s2": [4, 6, 7]} {"key": 1219, "questions": "What Is the Diagnosis?", "options": [{"label": "A", "disease": "Lymphoma"}, {"label": "B", "disease": "Pheochromocytoma"}, {"label": "C", "disease": "Cystic neoplasm of the pancreas"}, {"label": "D", "disease": "Rhabdomyosarcoma"}], "answer_idx": "B", "symptoms": ["A white woman in her late 40s presented to her primary care physician for her routine annual checkup and was noted to have abdominal distention", "The patient had been experiencing early satiety and heat intolerance", "Physical examination was significant for orthostatic hypotension and an immobile, nontender abdominal mass", "An abdominal computed tomographic scan was obtained (Figure, A)", "Based on these findings, the patient underwent surgical resection (Figure, B)", "Coronal imaging illustrating a large heterogeneous mass in the left abdomen (A)", "Specimen after resection (B)"], "s1": [0, 1, 2], "s2": [3, 4, 5, 6]} {"key": 1220, "questions": "What Is Your Diagnosis?", "options": [{"label": "A", "disease": "Herpetiform dermatitis"}, {"label": "B", "disease": "Pemphigus"}, {"label": "C", "disease": "Linear IgA dermatosis"}, {"label": "D", "disease": "Erythema multiforme"}], "answer_idx": "C", "symptoms": ["A 5-year-old girl presented with a 2-week history of pruritic, bullous skin lesions that had started on her lower limbs", "She had been treated with topical mupirocin and oral amoxicillin–clavulanate potassium for a suspected superficial skin infection", "She presented for evaluation because new lesions of similar appearance appeared on the upper limbs, trunk, vulva, and perineum despite antibiotic therapy", "Physical examination revealed many large, tense bullae filled with clear fluid, rounded with an erythematous base and “rosette-like” blisters (Figure)", "Some of them appeared superinfected", "Also the vulva and perineum had an intense erythematous rash", "She was apyretic and vital parameters were normal", "Laboratory tests showed a normal white blood cell count and lymphocyte subpopulations, erythrocyte sedimentation rate, and C-reactive protein level", "Test results for anti-transglutaminase and antiendomysial antibodies were negative"], "s1": [0, 1, 2, 3, 4, 5], "s2": [6, 7, 8]} {"key": 1221, "questions": "What Is the Diagnosis?", "options": [{"label": "A", "disease": "Splenic artery aneurysm rupture"}, {"label": "B", "disease": "Non-Hodgkin lymphoma with associated hepatosplenomegaly"}, {"label": "C", "disease": "Spontaneous rupture of the spleen"}, {"label": "D", "disease": "Pancreatitis with fluid collection"}], "answer_idx": "C", "symptoms": ["A 23-year-old otherwise healthy man presented to the emergency department after a 3-hour flight with complaints of headache and severe abdominal pain", "His symptoms followed a 1-day discharge from a recent hospital admission for a viral illness", "He denied any history of trauma, and the remaining medical history was unremarkable", "At the time of evaluation, he reported associated left shoulder pain and was in noticeable distress", "Physical examination revealed a distended abdomen with associated diffused peritoneal signs", "He was afebrile, with a heart rate of 77 beats/min, blood pressure of 96/46 mm Hg, oxygen saturation of 100% in room air, and a respiratory rate of 16 breaths/min", "Abdominal ultrasonography was performed (Figure 1)", "Ultrasonography image of the right (A) and left (B) upper quadrants of the abdomen"], "s1": [0, 1, 3, 4], "s2": [2, 5, 6, 7]} {"key": 1222, "questions": "What Is the Diagnosis?", "options": [{"label": "A", "disease": "Acute appendicitis with colon diverticula"}, {"label": "B", "disease": "Acute appendiceal diverticulitis"}, {"label": "C", "disease": "Appendiceal diverticulum"}, {"label": "D", "disease": "Appendiceal mucocele"}], "answer_idx": "B", "symptoms": ["A 50-year-old man presented to the emergency department with a 4-day history of right lower quadrant pain that did not migrate during that period of time", "On examination, he tested positive for the Blumberg sign", "Blood analysis showed leucocytosis, neutrophilia, and an increased level of C-reactive protein", "Ultrasonographic (Figure, A) and computed tomographic images (Figure, B) showed a thickened wall outpouching arising from a discreetly thickened appendix and inflamed surrounding fat", "Right-sided colonic diverticulosis was also seen", "The patient underwent an appendectomy with gross findings as shown in Figure, C", "Ultrasonographic image of the right lower quadrant (A) and sagittal reformatted computed tomographic image (B), both showing a thickened wall outpouching (arrow) arising from the vermiform appendix (arrowheads) with inflammatory changes in the surrounding mesenteric fat (asterisk)", "C, Gross findings of excised appendix (arrowheads) reveal an inflamed wall outpouching (arrow) and mesenteric fat (asterisk)"], "s1": [0, 1, 2, 4], "s2": [3, 5, 6, 7]} {"key": 1223, "questions": "What Is Your Diagnosis?", "options": [{"label": "A", "disease": "Keratosis pilaris"}, {"label": "B", "disease": "Filiform verrucae"}, {"label": "C", "disease": "Virus-associated trichodysplasia spinulosa"}, {"label": "D", "disease": "Phrynoderma"}], "answer_idx": "C", "symptoms": ["A 5-year-old boy, known to have acute lymphoblastic leukemia since the age of 3 years, presented with a 3-week history of asymptomatic skin lesions primarily affecting the face and to a lesser extent the trunk and extremities (Figure, A and B)", "He had been undergoing a maintenance chemotherapy regimen consisting of vincristine sulfate, 6-mercaptopurine, methotrexate, and high-dose prednisone", "He did not undergo bone marrow or stem cell transplant as part of his treatment", "Multiple topical medications including keratolytics were applied to the affected areas without effect", "Skin examination and microscopy analysis of punch biopsy specimen", "A and B, Multiple tiny, spiny, follicular hyperkeratotic papules on the face", "C and D, Microscopy showed abnormal maturation of anagen hair follicles with excessive inner root sheath differentiation (original magnification ×4 [C] and ×40 [D])", "Skin examination revealed multiple 1- to 3-mm, spiny, follicular, flesh-colored to erythematous hyperkeratotic papules predominantly on the face and to a lesser extent on the trunk, and extremities, with fine hairlike spicules emanating from most of them (Figure, A and B)", "Alopecia was also noted involving the eyebrows and the eyelashes, but the scalp was spared", "A punch biopsy specimen was taken (Figure, C and D)"], "s1": [0, 1, 2, 3], "s2": [4, 5, 6, 7, 8, 9]} {"key": 1224, "questions": "What Is the Diagnosis?", "options": [{"label": "A", "disease": "Acute cholecystitis"}, {"label": "B", "disease": "Gallbladder polyp"}, {"label": "C", "disease": "Gallbladder cancer"}, {"label": "D", "disease": "Gallbladder volvulus"}], "answer_idx": "D", "symptoms": ["An 89-year-old woman presented to her geriatrician with abdominal pain for 2 days", "The pain was localized to her right abdomen, both upper and lower quadrants, with acute tenderness to palpation", "She had some nausea and vomiting", "Ultrasonographic findings gave concern for acute cholecystitis", "A computed tomographic scan was ordered by her primary care physician after admission to the hospital (Figure)"], "s1": [0, 1, 2], "s2": [3, 4]} {"key": 1225, "questions": "What Is The Most Likely Diagnosis?", "options": [{"label": "A", "disease": "Disseminated herpes simplex"}, {"label": "B", "disease": "Primary varicella"}, {"label": "C", "disease": "Staphylococcal pustulosis"}, {"label": "D", "disease": "Rickettsialpox"}], "answer_idx": "D", "symptoms": ["A previously well 7-month-old male infant presented to our emergency department with 1 week of loose stools, 4 days of fever up to 39.2°C, and 2 days of skin rash", "His parents reported several pink papules that first appeared on his trunk and spread to his extremities within 24 hours", "His increasing irritability and poor oral intake led to his admission for further evaluation", "The patient’s vaccinations were up to date", "He had no history of perinatal infections, sick contacts, recent travel, or pets", "The family lives in an apartment in New York City", "On physical examination, the patient had fewer than 20 pink papulovesicles, some with hemorrhagic crust, scattered on the upper chest, abdomen, lower back, and extremities (Figure, A and B)", "His palms, soles, and oropharynx were otherwise unremarkable", "Clinical and histologic images of papulovesicles", "The clinical images show scattered pink papulovesicles, some with overlying hemorrhagic crust (A and B)", "C, A histologic section (hematoxylin and eosin, original magnification ×10) of a papule shows a focally eroded epidermis (asterisk) with a relatively dense and diffuse perivascular and interstitial dermal infiltrate (arrow) of predominantly mononuclear cells and a few neutrophils, with changes indicative of a mild vasculitis", "Laboratory test results were notable for a mild leukopenia with a white blood cell count of 4.6 × 109/L (normal, 6.0-18.0 × 109/L) with 18% monocytes", "C-reactive protein was slightly elevated at 3.38 mg/L (normal, <2.9 mg/L)", "He was started treatment with intravenous ceftriaxone and intravenous acyclovir", "Blood, urine, and wound cultures were taken, and a skin punch biopsy was performed (Figure, C)"], "s1": [0, 1, 2, 6, 7, 9, 10], "s2": [3, 4, 5, 8, 11, 12, 13, 14]} {"key": 1226, "questions": "What Is Your Diagnosis?", "options": [{"label": "A", "disease": "Perforated bowel"}, {"label": "B", "disease": "Crohn disease"}, {"label": "C", "disease": "Cholecystocolonic fistula"}, {"label": "D", "disease": "Gallstone ileus"}], "answer_idx": "C", "symptoms": ["A 64-year-old woman presented to the emergency department with a 1-day temperature of 38.9°C (102°F)", "The patient had some accompanying cough, nausea, and arthralgias but denied having dyspnea, emesis, and chest or abdominal pain", "She had a complicated medical history that was significant for non-Hodgkin lymphoma, chronic diarrhea, myelodysplastic syndrome treated with chemotherapy, and Behçet syndrome treated with prednisone acetate (6 mg daily)", "Her surgical history was significant for a previous right-sided hemicolectomy due to recurrent right-sided diverticulitis", "On examination, she was alert and awake with appropriate mental status", "Her vital signs were as follows: blood pressure of 150/59 mm Hg, heart rate of 127 beats per minute, respiratory rate of 20 breaths per minute, temperature of 38.5°C (101.3°F) (oral), and an oxygen saturation as measured by pulse oximetry of 99% in room air", "The physical examination was unremarkable", "Significant laboratory data included a white blood cell count of 2700/μL (reference range, 4800-10 800/μL", "to convert to ×109 per liter, multiply by 0.001), a lactic acid level of 19.8 mg/dL (to convert to millimoles per liter, multiply by 0.111), and a platelet count of 72×103/μL (to convert to ×109 per liter, multiply by 1.0)", "Urinalysis showed trace blood, positive nitrite, +1 leukocyte esterase, and a white blood cell count of 2000/μL to 5000/μL", "A urine culture came back positive for Klebsiella pneumonia, and a blood culture came back positive for Enterobacter sakazakii (now know as Cronobacter sakazakii)", "There was concern about the source of the bacteremia, so an infectious disease consult was ordered, and a computed tomographic scan of the abdomen/pelvis with contrast was performed", "Figure 1 shows a coronal, reformatted image of the scan", "The patient was scheduled for surgery, and the findings are shown in Figure 2."], "s1": [0, 1, 4, 5, 6], "s2": [2, 3, 7, 8, 9, 10, 11, 12, 13]} {"key": 1227, "questions": "What Is the Diagnosis?", "options": [{"label": "A", "disease": "Exophytic solitary giant cavernous hemangioma of stomach"}, {"label": "B", "disease": "Complicated cyst"}, {"label": "C", "disease": "Gastrointestinal stromal tumor"}, {"label": "D", "disease": "Neurilemmoma"}], "answer_idx": "A", "symptoms": ["A 23-year-old woman had a progressively enlarging mass associated with abdominal distention and a dragging type of discomfort for 1 week", "She denied any history of hematemesis or hematochezia", "On abdominal examination, an irregular mass measuring 20 × 10 cm without tenderness was found, and no bruit was heard on auscultation", "No other vascular anomalies were detected on systemic examination", "All laboratory test results were within normal limits", "Upper gastrointestinal endoscopy showed normal gastric mucosa", "Nonenhanced computed tomography of the abdomen showed a partially well-circumscribed, mixed cystic and solitary mass that partially enveloped the stomach and compressed adjacent organs, likely arising from the stomach (Figure 1)", "Axial nonenhanced computed tomographic image showing a mixed cystic and solitary mass that partially enveloped the stomach"], "s1": [0, 1, 2, 3], "s2": [4, 5, 6, 7]} {"key": 1228, "questions": "What Is the Diagnosis?", "options": [{"label": "A", "disease": "Hydatid cyst"}, {"label": "B", "disease": "Infected biliary cystadenoma"}, {"label": "C", "disease": "Hepatic abscess"}, {"label": "D", "disease": "Inflammatory pseudotumor of the liver"}], "answer_idx": "C", "symptoms": ["A 26-year-old man was admitted to our institution for a fever (temperature, 39°C) and abdominal pain on the right side of his hypochondrium", "He complained of nausea, vomiting, and asthenia as well", "His medical history was significant for pharyngitis associated with scarlet fever that he had a month before", "the pharyngitis was treated by his general practitioner with clarithromycin (500 mg twice daily for a week)", "No throat culture or rapid antigen test for group A streptococci was performed at that time", "On physical examination, he presented with mild right upper abdominal quadrant tenderness", "He met all the criteria for systemic inflammatory response syndrome, and his blood tests showed a marked increase in inflammatory markers", "Blood and urine cultures were obtained, and the results were negative", "Empirical antibiotic therapy with ampicillin sodium/sulbactam sodium was started", "His chest radiograph was normal", "An abdominal ultrasonographic examination revealed a 6-cm, solid, inhomogeneous mass in liver segment 6. A contrast-enhanced computed tomographic scan of the abdomen (Figure 1) showed that the lesion was hypodense with numerous septa without contrast enhancement", "Magnetic resonance imaging (Figure 2) evidenced a mixed solid-liquid lesion, with some septa delimiting large areas of necrosis", "The results of a serological detection test for echinococcosis were negative", "His carcinoembryonic antigen, carbohydrate antigen 19-9, and α1-fetoprotein blood levels were normal", "Serological markers for hepatitis B virus, hepatitis C virus, and human immunodeficiency virus were negative", "A study of leukocyte populations and immunoglobulin electrophoresis did not reveal any disorder of the immune system", "Computed tomographic scan showing a large, hypodense, multiloculated lesion in liver segment 6.Magnetic resonance imaging scan showing an heterogeneous, capsulated lesion, with numerous septa delimiting large areas of necrosis"], "s1": [0, 1, 2, 3, 4, 5, 6, 7, 8], "s2": [9, 10, 11, 12, 13, 14, 15, 16]} {"key": 1229, "questions": "What Is the Diagnosis?", "options": [{"label": "A", "disease": "Malignant bone tumor"}, {"label": "B", "disease": "Aneurysmatic bone cyst"}, {"label": "C", "disease": "Primary hyperparathyroidism with multiple brown tumors"}, {"label": "D", "disease": "Giant cell tumor"}], "answer_idx": "C", "symptoms": ["In April 2010, a 19-year-old soccer player presented with a 4-week history of pain after a tap against his right knee", "Medical and family histories were unremarkable and no abnormalities were found on physical examination", "A radiograph of the right knee showed osteolytic lesions in the distal femur and the head of the tibia", "On magnetic resonance imaging, the epiphyseal lesions were visualized as cystic and nonsclerotic and extended to the articular surface of the distal femur and proximal tibia (Figure 1)", "Soft tissue extension was suspected from the tibia", "Laboratory test results showed normal blood cell count, renal function, and concentration of C-reactive protein", "Aneurysmatic bone cysts were suspected and a biopsy was performed", "Histopathological examination detected spindle cell infiltrates with hemorrhages, fragmented trabecular bone, and numerous osteoclastic giant cells proliferating at a rate of 10% suggesting an aggressive giant cell lesion", "Since it was still not determined whether the lesions were malignant, flourodeoxyglucose F 18 positron emission tomography/computed tomography was performed and found additional lesions in the left distal femur and the left mandible", "Biopsy of the mandible showed the same type of tissue as seen in the right femur and tibia"], "s1": [0, 1, 4, 5, 9], "s2": [2, 3, 6, 7, 8]}